Крок 2 - Медицина 2024 Центр тестування (буклет)

1 / 1500
Дівчинка 13 років надійшла до гінекологічного відділення зі скаргами на значні кров'янисті виділення зі статевих шляхів протягом 10 днів. З анамнезу відомо: менархе з 11 років, менструальний цикл нерегулярний. Під час ректо-абдомінального обстеження патології не виявлено. Який попередній діагноз? A 13-year-old girl came to the gynecological department with complaints of significant bleeding from the genital tract for 10 days. It is known from the anamnesis: menarche since the age of 11, menstrual cycle irregular. During the recto-abdominal examination, no pathology was detected. What was the previous diagnosis?

Поліп ендометрія Endometrial polyp

Аденоміоз Adenomyosis

Травма зовнішніх статевих шляхів Injury of the external genital tract

Хвороба Верльгофа Werlhof's disease

Ювенільна маткова кровотеча Juvenile uterine bleeding

2 / 1500
35-річний чоловік звернувся до лікаря зі скаргами на сильний біль у правому оці, світлобоязнь, сльозотечу, погіршення зору. Скарги з'явились після роботи зі зварювальним апаратом. Контактні лінзи не використовує. Під час огляду: очна щілина звужена, помірна перикорнеальна ін'єкція, гострота зору збережена. Після інстиляції розчину анестетика, стан полегшився, що дало можливість провести огляд у щілинній лампі та виявити дефект рогівки розміром 3 мм, що зафарбувався флюоресцеїном. Яке лікування буде найбільш доречним? A 35-year-old man consulted a doctor with complaints of severe pain in the right eye, photophobia, lacrimation, impaired vision. The complaints appeared after working with a welding machine. He does not use contact lenses. During the examination: the eye slit is narrowed, the pericorneal injection is moderate, the visual acuity is preserved. After instillation of the anesthetic solution, the condition was relieved, which made it possible to conduct a slit-lamp examination and reveal a 3 mm corneal defect stained with fluorescein. . What treatment would be most appropriate?

Інстиляція офлоксацину Ofloxacin instillation

Інстиляція таурину Taurine instillation

Сухе тепло Dry heat

Інстиляція сульфацилу натрію Sulfacyl sodium instillation

Інстиляція дексаметазону Dexamethasone instillation

3 / 1500
У хворого 60-ти років скарги на задуху, парестезії, запаморочення, втомлюваність. В анамнезі: атрофічний гастрит. Об'єктивно: шкіра та слизові бліді, іктеричні. Сосочки язика згладжені. Печінка, селезінка не збільшені. ЗАК: Нb- 77 г/л, ер.- 1,65 Г/л, КП- 1,4, ретикулоцити - 0,2%, Л- 2,8 Г/л; ШОЕ- 22 мм/год, макроцити. Яке додаткове обстеження дозволить вия-вити причину анемії? A 60-year-old patient complains of suffocation, paresthesias, dizziness, fatigue. History: atrophic gastritis. Objectively: skin and mucous membranes are pale, icteric. The papillae of the tongue are smoothed. ; ESR - 22 mm/h, macrocytes. What additional examination will reveal the cause of anemia?

Визначення рівня заліза Iron level determination

Визначення рівня міді Determining copper level

Визначення рівня кальцію Calcium level determination

Визначення рівня вітаміну D Determination of vitamin D level

Визначення рівня вітаміну B_12 Determining the level of vitamin B_12

4 / 1500
Пацієнтка віком 25 років скаржиться на значні пінисті виділення із піхви, з неприємним запахом, печіння та свербіж в ділянці геніталій. З анамнезу відомо: хворіє протягом тижня, не має постійного статевого партнера. Під час піхвового дослідження виявлено: слизова оболонка піхви гіперемійована, під час дотику кровоточить, виділення піністого характеру. Який найімовірніший діагноз? A 25-year-old patient complains of significant foamy discharge from the vagina, with an unpleasant odor, burning and itching in the genital area. It is known from the anamnesis: she has been ill for a week, she has no a permanent sexual partner. During the vaginal examination, the vaginal mucosa is hyperemic, bleeding when touched. What is the most likely diagnosis?

Кандидомікоз піхви Vaginal candidiasis

Бактеріальний вагіноз Bacterial vaginosis

Гонорейний цервіцит Gonorrheal cervicitis

Хламідійний цервіцит Chlamydial cervicitis

Трихомонадний кольпіт Trichomonad colpitis

5 / 1500
Пацієнт 15 років відстає у фізичному розвитку, періодично відзначається пожовтіння шкіри. Об'єктивно спостерігається: селезінка 16х12х10 см, холецистолітіаз, виразка шкіри нижньої третини лівої гомілки. У крові виявлено: еритроцити - 3,0·10^12/л, Hb - 90 г/л, КП - 1,0, мікросфероцитоз, ретикулоцитоз. Білірубін сироватки загальний - 56 мкмоль/л, непрямий - 38 мкмоль/л. Який метод лікування найдоцільніший? The patient is 15 years behind in physical development, yellowing of the skin is periodically noted. Objectively observed: spleen 16x12x10 cm, cholecystolithiasis, skin ulcer of the lower third of the left leg. In the blood revealed: erythrocytes - 3.0·10^12/l, Hb - 90 g/l, microspherocytosis, reticulocytosis. Serum bilirubin - 56 μmol/l, indirect - 38 μmol/l. What is the treatment method the most appropriate?

Пересадження селезінки Spleen transplant

Портокавальний анастомоз Portocaval anastomosis

Спленектомія Splenectomy

Оментоспленопексія Omentosplenopexy

Оментогепатопексія Omentohepatopexy

6 / 1500
Хвора 47-ми років, яка 2 дні тому повернулась з Перу, скаржиться на біль та збільшення лімфатичних вузлів у правій пахвинній ділянці. Діагностована бубонна форма чуми. Який з препаратів необхідно призначити контактним особам для екстреної профілактики захворювання? A 47-year-old patient who returned from Peru 2 days ago complains of pain and enlarged lymph nodes in the right inguinal region. The bubonic form of the plague is diagnosed. Which of the drugs should be prescribed to contact persons for emergency prevention of the disease?

Імуноглобулін людини Human immunoglobulin

Хлорохін Chloroquine

Доксициклін Doxycycline

Гетерологічна сироватка Heterologous serum

Флуконазол Fluconazole

7 / 1500
Сім'я звернулася за прогнозом потомства у зв'язку з народженням дитини з тяжким дефектом закриття невральної трубки. Дитина померла у неонатальному періоді. Ваша порада родині: The family applied for the prognosis of the offspring due to the birth of a child with a severe neural tube closure defect. The child died in the neonatal period. Your advice to the family:

Планувати вагітність, оскільки це більше не повториться Plan pregnancy because it won't happen again

У разі вагітності провести біопсію хоріону, кордоцентез In case of pregnancy, conduct a chorionic biopsy, cordocentesis

Відмовитися від народження дітей Refuse to have children

Провести преконцепційну профілактику, запланувати вагітність, провести пренатальну діагностику Conduct preconception prevention, plan pregnancy, conduct prenatal diagnosis

Провести цитогенетичне обстеження батьків Conduct cytogenetic examination of parents

8 / 1500
Пацієнт віком 27 років скаржиться на болі в симетричних суглобах рук, лихоманку, ранкову скутість. Захворів гостро. Лабораторно встановлено: ревматоїдний фактор, високий титр С-реактивного білка, високий титр антитіл до цитрулінованого віментину. Рентгенологічно виявлено одиничні ерозії міжфалангових суглобів. Яке початкове базисне лікування необхідно призначити у цьому разі? A 27-year-old patient complains of pain in the symmetrical joints of the hands, fever, morning stiffness. He became acutely ill. Laboratory tests revealed: rheumatoid factor, high titer of C-reactive protein, high titer of citrullinated vimentin. Single erosions of the interphalangeal joints were detected. What initial treatment should be prescribed in this case?

Метотрексат Methotrexate

НПЗП NSAIDs

Інгібітори ксантиноксидази Xanthine oxidase inhibitors

Целекоксиб Celecoxib

Диклофенак Diclofenac

9 / 1500
У хворого 38-ми років скарги на переймоподібний біль в животі, часті рідкі випорожнення з домішками слизу та свіжої крові. Хворіє 2 роки. Схуд на 12 кг. Об'єктивно: Ps - 92/хв, АТ - 100/70 мм рт.ст., температура тіла - 37,4^oC. Живіт м'який, болючий за ходом товстого кишечника. Сигмоподібна кишка спазмована. В крові: еритроцити - 3,2·10^12/л, Нb - 92 г/л, лейкоцити - 10,6·10^9/л, ШОЕ - 32 мм/год. Іригоскопія: товста кишка звужена, гаустри відсутні, контури нечіткі, симптом ''водопровідної труби''. Який найбільш імовірний діагноз? A 38-year-old patient complains of spasm-like abdominal pain, frequent loose stools with impurities of mucus and fresh blood. He has been ill for 2 years. He has lost 12 kg. About Objectively: BP - 100/70 mmHg, body temperature - 37.4 °C. Abdomen is painful, colon is spasmodic. Blood: erythrocytes ,2·10^12/l, Hb - 92 g/l, leukocytes - 10.6·10^9/l, ESR - 32 mm/h. Irigoscopy: colon is narrowed, haustra are absent, contours are unclear, symptom water pipe''. What is the most likely diagnosis?

Виразковий коліт Ulcerative colitis

Хвороба Крона Crohn's disease

Рак кишечника Intestinal cancer

Дивертикульоз кишечника Intestinal diverticulosis

Амебна дизентерія Amoebic dysentery

10 / 1500
Вагітна жінка віком 19 років перебуває у відділенні патології вагітності з приводу вагітності в терміні 36 тижнів, тазове передлежання, великий плід, важка форма цукрового діабету. На КТГ у плода зареєстрована брадикардія до 90/хв. Пологової діяльності немає. Яка тактика ведення вагітності в цій ситуації? A 19-year-old pregnant woman is in the department of pregnancy pathology due to a 36-week pregnancy, breech presentation, large fetus, severe form of diabetes mellitus. CTG of the fetus registered bradycardia up to 90/min. There is no labor activity. What are the tactics of pregnancy management in this situation?

Родостимуляція Rodostimulation

Виведення плода за тазовий кінець Expulsion of the fetus by the pelvic end

Терміновий кесарів розтин Urgent cesarean section

Розродження через природні пологові шляхи Birth through natural birth canals

Продовження вагітності Continuation of pregnancy

11 / 1500
Доношена дівчинка народилася від ІІ вагітності, яка перебігала з загрозою переривання на 25-27 тижнях, з хронічною фетоплацентарною недостатністю. Під час пологів було одноразове туге обвиття пуповини навколо шиї плода, утруднення виведення плечиків. Після народження дитина була оцінена за шкалою Апгар на першій хвилині: дихання відсутнє, ЧСС - 50/хв., тотальний ціаноз, атонія, арефлексія. Яка це оцінка за шкалою Апгар? A full-term girl was born from the II pregnancy, which ran with a threat of termination at 25-27 weeks, with chronic fetoplacental insufficiency. During childbirth, there was a single tight wrapping of the umbilical cord around the neck of the fetus, difficulty in withdrawing the shoulders. After birth, the child was assessed according to the Apgar scale: no breathing, heart rate - 50/min., total cyanosis, atony, areflexia. What is this assessment according to the Apgar scale?

0 балів 0 points

1 бал 1 point

2 бали 2 points

4 бали 4 points

3 бали 3 points

12 / 1500
Хлопчик віком 10 років скаржиться на задишку під час ходіння, підвищення температури тіла до 38^oС , біль і припухлість обох колінних суглобів. Два тижні тому переніс ангіну. Об'єктивно спостерігається: припухлість, гіперемія, обмеження рухів в колінних суглобах, розширення меж серця вліво, тахікардія, приглушення тонів серця, систолічний шум на верхівці. Який найімовірніший діагноз? A 10-year-old boy complains of shortness of breath while walking, an increase in body temperature up to 38°C, pain and swelling of both knee joints. Two weeks ago he suffered from angina. About objectively observed: swelling, hyperemia, limitation of movements in the knee joints, expansion of the heart borders to the left, tachycardia, muffled heart sounds, systolic murmur at the apex. What is the most likely diagnosis?

Неревматичний кардит Non-rheumatic carditis

Синдром Стілла Still's Syndrome

Гостра ревматична лихоманка Acute rheumatic fever

Інфекційно-алергічний артрит Infectious-allergic arthritis

Системний червоний вовчак Systemic lupus erythematosus

13 / 1500
Для боротьби з тютюнопалінням Україна ратифікувала Рамкову конвенцію Всесвітньої організації охорони здоров'я з боротьби з тютюном, у зв'язку із чим було заборонено палити в громадських місцях. До якого виду профілактики належать зазначені заходи? In order to combat tobacco smoking, Ukraine ratified the World Health Organization's Framework Convention on Tobacco Control, in connection with which smoking in public places was prohibited. what type of prevention do these measures belong to?

Вторинна профілактика Secondary prevention

Індивідуальна профілактика Individual prevention

Третинна профілактика Tertiary prevention

Громадська профілактика Public prevention

Первинна профілактика Primary prevention

14 / 1500
Здорова жінка 47-ми років прийшла до свого гінеколога на планове взяття мазку Папаніколау. Скарг не має. Викурює 20 сигарет на день протягом 30-ти років (індекс паління - 30 пачко/років). Рік тому самостійно зробила аналіз крові на визначення концентрації загального холестерину, ЛПНЩ, ЛПВЩ та тригліцеридів. Усі показники аналізу в межах норми. При фізикальному обстеженні температура - 37,1^oC, пульс - 76/хв., частота дихання - 13/хв., артеріальний тиск - 120/70 мм рт.ст. Аускультативних змін у легенях не виявлено. Лабораторні показники без відхилень. Який наступний крок є найбільш доречним? A healthy 47-year-old woman came to her gynecologist for a routine pap smear. She has no complaints. She has smoked 20 cigarettes a day for 30 years (smoking index - 30 pack/years) A year ago, she did a blood test to determine the concentration of total cholesterol, HDL and triglycerides. During the physical examination, the temperature was 37.1°C, the heart rate was 76/min. respiratory rate - 13/min, blood pressure - 120/70 mm Hg. No abnormalities in laboratory parameters. What is the most appropriate step?

Кольпоскопія після ПАП-тесту Colposcopy after Pap test

Мамографія через 3 роки Mammography in 3 years

Колоноскопія через 1 рік Colonoscopy in 1 year

Визначення загального холестерину та ЛПВЩ через 2 роки Determination of total cholesterol and HDL after 2 years

Низькодозова КТ легень через 3 роки Low-dose lung CT after 3 years

15 / 1500
За 2 місяці після операційного втручання з приводу парапроктиту жінка віком 53 роки звернулась до лікаря зі скаргами на перианальний дискомфорт, тенезми, слизисто-гнійні виділення. Під час ректального обстеження відмічається зовнішній норицевий отвір, при надавлюванні на який виходить гній, по ходу отвору відмічається шнуроподібне ущільнення в напрямку прямої кишки. Під час анаскопії встановлено: слизова облонка кишківника по типу бруківки. Який імовірний діагноз у пацієнтки? 2 months after surgery for paraproctitis, a 53-year-old woman consulted a doctor with complaints of perianal discomfort, tenesmus, mucopurulent discharge. During a rectal examination an external fistula opening is noted, on which pus comes out, a cord-like compaction is noted in the direction of the rectum. During the anascopy, the mucous membrane of the intestine of the cobblestone type is found. What is the probable diagnosis of the patient?

Неспецифічний виразковий коліт Nonspecific ulcerative colitis

Рак прямої кишки Rectal cancer

Гострий парапроктит Acute paraproctitis

Неповна внутрішня параректальна нориця Incomplete internal pararectal fistula

Хвороба Крона, параректальна нориця Crohn's disease, pararectal fistula

16 / 1500
У жінки віком 25 років після пологів посилилися хиткість під час ходи та слабкість в ногах. Хворіє протягом 6 років, зазначає погіршення стану щороку восени. Об'єктивно спостерігається: жінка ейфорична, знижена критичність до свого стану, горизонтальний ністагм, високі сухожилкові рефлекси, клонус стоп, патологічні стопні рефлекси, черевні рефлекси відсутні, атаксія при пробі Ромберга, інтенційний тремор та промахування під час виконання координаторних проб. На очному дні спостерігається темпоральне збліднення дисків зорових нервів. Який діагноз найімовірніший? A 25-year-old woman after giving birth has increased unsteadiness while walking and weakness in her legs. She has been sick for 6 years, notes that her condition worsens every fall. Objectively observed: the woman is euphoric, reduced criticality of her condition, horizontal nystagmus, high tendon reflexes, foot clonus, pathological foot reflexes, absent abdominal reflexes, ataxia during Romberg's test, intention tremor and lapses during coordination tests nerves. What is the most likely diagnosis?

Міастенія гравіс Myasthenia gravis

Гострий розсіяний енцефаломієліт Acute diffuse encephalomyelitis

Дисциркуляторна енцефалопатія Dyscirculatory encephalopathy

Бічний аміотрофічний склероз Amyotrophic lateral sclerosis

Розсіяний склероз Multiple sclerosis

17 / 1500
Пацієнт віком 50 років скаржиться на задишку, що посилюється під час фізичного навантаження. З анамнезу відомо що палить протягом 30 років. Об'єктивно спостерігається: температура тіла 36,5^oС, ЧД - 22/хв, ЧСС - 88/хв, АТ - 130/85 мм рт. ст. Грудна клітка бочкоподібної форми, під час аускультації вислуховується послаблене везикулярне дихання з коробковим відтінком над усією поверхнею легень. Який найімовірніший діагноз? A 50-year-old patient complains of shortness of breath, which worsens during physical exertion. It is known from the anamnesis that he has been smoking for 30 years. Objectively observed: body temperature 36, 5^oС, heart rate - 88/min, barrel-shaped chest, during auscultation weak vesicular breathing is heard over the entire surface of the lungs. What is the most likely diagnosis?

Хронічне обструктивне захворювання легень Chronic obstructive pulmonary disease

Бронхогенний рак Bronchogenic cancer

Позагоспітальна пневмонія Community-acquired pneumonia

Бронхоектатична хвороба Bronchoectatic disease

Туберкульоз легень Pulmonary tuberculosis

18 / 1500
Чоловік 58-ми років, скаржиться на постійний біль в епігастрії, що іррадіює в спину і посилюється після прийому жирної їжі. Пацієнт відмічає слабкість, за останні 2 місяці втратив у вазі до 9 кг. Місяць тому з'явилась жовтяниця, що поступово зростає. Живіт м'який, печінка не збільшена. При лабораторному дослідженні крові гемоглобін - 68 г/л, лейкоцити - 9,0·10^9/л, швидкість зсідання еритроцитів - 36 мм/год. На УЗД: жовчний міхур великих розмірів, конкрементів не містить, холедох розширений, в головці підшлункової залози - округле утворення до 4 см в діаметрі з нечіткими контурами. Який діагноз є найбільш імовірним? A 58-year-old man complains of constant pain in the epigastrium, which radiates to the back and worsens after eating fatty food. The patient notes weakness, over the past 2 months he has lost in weight up to 9 kg. Jaundice appeared a month ago, the abdomen is soft, the liver is not enlarged. In the laboratory examination of blood - 68 g/l, leukocytes - 9.0·10^9/l sedimentation of erythrocytes - 36 mm/h. On ultrasound: the gallbladder is large, does not contain calculi, the choledochus is enlarged, in the head of the pancreas - a rounded formation with a diameter of 4 cm. What is the most likely diagnosis?

Рак шлунка з метастазами в печінку Gastric cancer with liver metastases

Виразкова хвороба дванадцятипалої кишки з пенетрацією в головку підшлункової залози Duodenal ulcer with penetration into the head of the pancreas

Хронічний холецистопанкреатит Chronic cholecystopancreatitis

Рак головки підшлункової залози Cancer of the head of the pancreas

Хронічний індуративний панкреатит Chronic indurative pancreatitis

19 / 1500
У пацієнта віком 16 років уперше раптово виникли біль у потиличній ділянці, миготіння <<мушок>>, одноразове блювання, що не принесло полегшення. Об'єктивно виявлено: пацієнт у свідомості, гіперемія шкіри обличчя, АТ - 160/110 мм рт. ст., пульс - 122/хв, напружений. Сухожилкові рефлекси рівномірні. Стенокардія напруги. Акцент ІІ тону над аортою. Який препарат потрібно насапмеред призначити для лікування цього пацієнта? A 16-year-old patient first developed sudden pain in the back of the head, flickering <>, single vomiting, which did not bring relief. Objectively revealed: the patient is conscious, the blood pressure is 160/110 mm Hg, the tendon reflexes are tense. What drug should be prescribed for the treatment of this patient?

Ніфедипін 20 мг сублінгвально Nifedipine 20 mg sublingual

25%-й розчин магнію сульфату 0,2 мл/кг в/м 25% solution of magnesium sulfate 0.2 ml/kg in/m

1%-й розчин дибазолу 0,1 мл/рік життя в ізотонічному розчині натрію хлориду в/в 1% solution of dibazol 0.1 ml/year of life in isotonic solution of sodium chloride IV

2%-й розчин еуфіліну 3 мг/кг у 100 мл ізотонічного розчину натрію хлориду в/в 2% solution of euphilin 3 mg/kg in 100 ml of isotonic sodium chloride solution IV

Пропранолол 2 мг в/в крапельно в ізотонічному розчині натрію хлориду Propranolol 2 mg IV drip in isotonic sodium chloride solution

20 / 1500
Робітниця 40 років працює на фабриці з виробництва льону, стаж 12 років. Пред'являє скарги на задишку, кашель, слабкість. Загальний стан поліпшується у вихідні дні, але знову стає гіршим у разі виходу на роботу. У легенях вислуховуються розсіяні сухі хрипи з обох сторін. У крові спостерігається незначний лейкоцитоз. Який найвірогіднішиий діагноз? A 40-year-old female worker works at a flax factory, 12 years of experience. She complains of shortness of breath, cough, weakness. Her general condition improves on weekends, but becomes worse when going to work. Scattered wheezes are heard on both sides. What is the most likely diagnosis?

Силікоз Silicosis

Пильовий бронхіт Dust bronchitis

Алергійний альвеоліт Allergic alveolitis

Бісиноз Bisinosis

Бронхіальна астма Bronchial asthma

21 / 1500
Чоловік 69 років протягом декількох останніх років спостерігав прогресуюче порушення сечовипускання. Протягом останньої доби зазначив відсутність сечовипускання, постійний розпираючий біль над лоном. Які діагностичні заходи слід запропонувати у цьому разі? A 69-year-old man has had a progressive urinary incontinence for the past several years. During the last day, he noted the absence of urination, constant throbbing pain above the pubic area. What diagnostic measures should be offered in this case ?

Внутрішньовенна урографія Intravenous urography

Оглядова урографія Review urography

Уретротомія Urethrotomy

Катетеризація сечового міхура Bladder catheterization

Радіоізотопна ренографія Radioisotope renography

22 / 1500
Чоловік 62 років скаржиться на помірний біль у лівій ступні в ділянці плюснефалангового суглоба, який посилюється під час рухів. Хворіє протягом 12 років, захворювання почалося з <<гострого нападу>> болю. 2 роки тому під шкірою суглоба з'явився жовтувато-білий вузлик. Під час огляду виявлено суглоб деформований, синюшний. На рентгенограмі суглоба спостерігаються звуження суглобової щілини та чітко обмежені дефекти кісткової тканини в епіфізі (<<симптом пробійника>>). Який діагноз найімовірніший? A 62-year-old man complains of moderate pain in the left foot in the area of ​​the metatarsophalangeal joint, which worsens during movements. He has been ill for 12 years, the disease began with an <> pain. A yellowish-white nodule appeared under the skin of the joint. During the examination, the joint was found to be bluish. Narrowing of the joint space and clearly limited bone tissue defects in the epiphysis were observed (<>). . What is the most likely diagnosis?

Реактивний артрит Reactive arthritis

Подагричний артрит Gouty arthritis

Синдром Рейтера Reiter syndrome

Остеоартрит Osteoarthritis

Ревматоїдний артрит Rheumatoid arthritis

23 / 1500
До лікаря звернувся чоловік зі скаргами на біль у нижній щелепі. Об'єктивно спостерігається: маса тіла знижена, шкірні покриви бліді, неприємний запах з рота, ясна кровоточать. На рентгенограмі щелепи виявлено явища остеопорозу. Пацієнт протягом 30 років працював на хімічному підприємстві. Для якого патологічного стану характерні ці симптоми? A man went to the doctor with complaints of pain in the lower jaw. Objectively observed: body weight is reduced, skin is pale, bad breath, gums are bleeding. An x-ray of the jaw revealed osteoporosis. The patient worked at a chemical plant for 30 years. What pathological condition are these symptoms characteristic of?

Хронічного остеомієліту нижньої щелепи Chronic osteomyelitis of the lower jaw

Отруєння перхлоратом калію Poisoning with potassium perchlorate

Флюорозу Fluorosis

Хронічного отруєння фосфором Chronic phosphorus poisoning

Отруєння метилртуттю Methylmercury poisoning

24 / 1500
Хлопець 18 років перебуває на лікуванні в терапевтичному відділенні з приводу загострення хронічного бронхіту. З 6 років страждає епілепсією. На фоні підвищення температури до фебрильних цифр розвинувся епілептичний статус у вигляді тоніко-клонічних судом. Яка Ваша найперша тактика у цьому разі? An 18-year-old boy is being treated in the therapeutic department for an exacerbation of chronic bronchitis. He has been suffering from epilepsy since he was 6 years old. Against the background of an increase in temperature to febrile figures, he developed a status epilepticus in the form of tonic-clonic seizures. What is your first tactic in this case?

Організація консультації психіатра Organization of psychiatrist consultation

Переведення до психіатричного стаціонару Transfer to a psychiatric hospital

Надання невідкладної допомоги на місці Providing emergency care on site

Переведення до неврологічного відділення Transfer to the neurology department

Переведення до реанімаційного відділення Transfer to intensive care unit

25 / 1500
У вагітної жінки, що страждає на гіпертонічну хворобу I ступеня, в терміні 35 тижнів з'явилися набряки на нижніх кінцівках і передній черевній стінці, в добовій сечі кількість білка - до 5 г/л, АТ підвищився до 170/120 мм рт.ст., став турбувати головний біль і погіршився зір. Проведене впродовж чотирьох годин інтенсивне лікування не дало ефекту. Яка тактика необхідна в цій ситуації? A pregnant woman suffering from hypertension of the first degree, in the period of 35 weeks, edema appeared on the lower limbs and the anterior abdominal wall, the amount of protein in the daily urine - up to 5 g/l, blood pressure rose to 170/120 mm Hg, headache and vision worsened. Intensive treatment for four hours did not give any effect. What tactics are necessary in this situation?

Негайне розродження шляхом кесаревого розтину Immediate cesarean delivery

Консервативне розродження Conservative birth

Підготовка шийки матки для дострокового розродження Preparation of the cervix for premature birth

Продовження інтенсивної терапії Continuation of intensive care

Проведення родозбудження Conducting genealogy

26 / 1500
Під час обстеження пацієнта виявлено такі симптоми: прогресуюча кволість, зниження працездатності, швидка фізична та психічна втомлюваність, відчуття мерзлякуватості та голоду, втрата маси тіла. Установіть вид аліментарного захворювання: During the examination of the patient, the following symptoms were revealed: progressive frailty, reduced work capacity, rapid physical and mental fatigue, feeling of coldness and hunger, loss of body weight. Establish the type of alimentary disease :

Недостатність поліненасичених жирних кислот Deficiency of polyunsaturated fatty acids

Мінеральна недостатність Mineral deficiency

Вітамінна недостатність Vitamin deficiency

Білково-енергетична недостатність Protein-energy deficiency

Недостатність харчових волокон Insufficient dietary fiber

27 / 1500
В гінекологічне відділення онкодиспансеру доставлено хвору 35-ти років зі скаргами на кров'янисті виділення після статевого акту. При дослідженні в дзеркалах: на шийці матки навколо зовнішнього вічка дефект епітелію. Матка, додатки і параметрії без патологічних змін. При гістологічному дослідженні біоптату шийки матки виявлено рак in situ. Яке лікування буде найбільш доречним? A 35-year-old patient was brought to the gynecological department of the oncology clinic with complaints of bleeding after sexual intercourse. When examined in mirrors: a defect is found on the cervix around the outer eye epithelium. Uterus, appendages without pathological changes. Cancer in situ was detected during histological examination of the cervix. What treatment would be most appropriate?

Кріодеструкція шийки матки Cryodestruction of the cervix

Хіміотерапевтичне лікування Chemotherapy treatment

Променева терапія Radiotherapy

Конусоподібна електроексцизія шийки матки Cone-shaped electroexcision of the cervix

Розширена екстирпація матки з додатками Advanced uterine extirpation with attachments

28 / 1500
Чоловік 35 років збитий автомобілем, ударився головою об асфальт. Об'єктивно встановлено: стан важкий, без свідомості, у комі, дихання уривчасте, шумне, на больові подразники не реагує. У ділянці лоба велика рана м'яких тканин голови, видно фрагменти кістки, витікає мозковий детрит, є кровотеча. Визначте лікувальну тактику на догоспітальному етапі: A 35-year-old man was hit by a car, hit his head on the asphalt. Objectively established: the condition is serious, unconscious, in a coma, breathing is intermittent, noisy, on painful stimuli does not react. There is a large wound of the soft tissues of the head, brain debris is leaking out. Determine the treatment tactics at the pre-hospital stage:

Зашивання рани Wound suturing

Видалення мозкового детриту Removing brain detritus

Видалення фрагментів кісток Removing bone fragments

Зупинка кровотечі, асептична пов'язка Stop bleeding, aseptic dressing

Протинабрякова терапія Anti-edema therapy

29 / 1500
Чоловік 33-х років прийшов до лікаря на профілактичний прийом. Скарг не має. При фізикальному обстеженні лікарем випадково виявлено грубий систолічний шум в тoчці Боткіна, який проводиться в точку аускультації верхівки та на судини шиї. Від подальшого обстеження пацієнт відмовився. Через 2 місяці на фоні грипу у пацієнта розвинулися симптоми серцевої недостатності. Яку ваду найбільш імовірно було виявлено лікарем? A 33-year-old man came to the doctor for a preventive appointment. He has no complaints. During the physical examination, the doctor accidentally detected a rough systolic murmur at Botkin's point, which is performed at the point auscultation of the apex and the vessels of the neck. The patient refused further examination. After 2 months, the patient developed symptoms of heart failure.

Мітральна недостатність Mitral regurgitation

Вроджене ураження тристулкового клапана Congenital tricuspid valve lesion

Аортальна недостатність Aortic insufficiency

Аортальний стеноз Aortic stenosis

Мітральний стеноз Mitral stenosis

30 / 1500
Хвора 17-ти років має скарги на біль в колінних та гомілковостопних суглобах, підвищення температури тіла до 39^oC. 2 тижні тому перенесла гострий тонзиліт. Об'єктивно: припухлість та різка болючість, обмеження рухів в суглобах. На шкірі тулуба та кінцівок є ледве помітні кільцеподібні блідо-рожеві плями. ЧСС- 95/хв., АТ- 90/60 мм рт.ст., тони ослаблені, м'який систолічний шум над верхівкою. Поставте попередній діагноз: A 17-year-old patient complains of pain in the knee and ankle joints, an increase in body temperature to 39°C. She suffered acute tonsillitis 2 weeks ago. Objectively : swelling and sharp pain in the joints. On the skin of the trunk and limbs there are barely visible ring-shaped spots. HR- 95/min., BP- 90/60 mm Hg, tones are weakened, soft systolic make a preliminary diagnosis:

Гостра ревматична лихоманка Acute rheumatic fever

Ревматоїдний артрит Rheumatoid arthritis

Вузлувата еритема Erythema nodosum

Системна склеродермія Systemic scleroderma

Хвороба Рейтера Reiter's disease

31 / 1500
Дівчинка 14-ти років звернулася до лікаря зі скаргами на відчуття стиснення у ділянці шиї, покашлювання, потовщення шиї. При фізикальному обстеженні щитоподібна залоза дифузно збільшена, щільна при пальпації, неболюча, поверхня гладка. При ультразвуковому дослідженні тканина щитоподібної залози неоднорідна. Яке лабораторне дослідження буде найбільш доречним? A 14-year-old girl went to the doctor with complaints of a feeling of compression in the neck area, coughing, thickening of the neck. On physical examination, the thyroid gland is diffusely enlarged, dense on palpation , painless, smooth surface. Thyroid tissue is heterogeneous during ultrasound examination. Which laboratory test would be most appropriate?

Вміст гормону росту Content of growth hormone

Вміст кальцитоніну Calcitonin content

Вміст йоду в добовій сечі Iodine content in daily urine

Вміст паратгормону Parathormone content

Титр антитіл до тиреоглобуліну Titer of antibodies to thyroglobulin

32 / 1500
Пацієнтка віком 26 років протягом тижня почала виконувати багато нових і незвичних для неї справ зокрема: самотужки взялась фарбувати стіни в під'їзді, ночами писала вірші, водночас роблячи до них ілюстрації. Об'єктивно спостерігається: балакуча, швидко змінює теми розмови, активно жестикулює, жартує, кокетує з чоловіками. Вважає, що в ній загинула велика актриса, письменниця і художниця, запрошує всіх у гості на 'мистецькі вечори'. Який психопатологічний стан спостерігається у пацієнтки? A 26-year-old patient began doing many new and unusual things for her during the week, in particular: she started painting the walls in the entrance hall on her own, wrote poems at night, while doing They are objectively observed: she is chatty, she is actively gesticulating, she is flirting with men. She thinks that she is a great actress, writer and artist observed in the patient?

Істерично-невротичний синдром Hysteric-neurotic syndrome

Маніакальний синдром Manic syndrome

Гебефренічний синдром Hebephrenic syndrome

Псевдопаралітична деменція Pseudoparalytic dementia

Кататонічне збудження Catatonic excitement

33 / 1500
У ході проведення медичного огляду студентів, лікар звернув увагу на хлопця 24-х років з ростом 176 см та масою тіла 68 кг, у якого спостерігалась сухість шкіри та гіперкератоз. При офтальмологічному обстеженні виявлене суттєве зниження гостроти сутінкового зору. У харчуванні студента практично ніколи не були включені в раціон молочні продукти, овочі та фрукти. Яке із зазначених захворювань можна припустити? During the medical examination of students, the doctor noticed a 24-year-old boy with a height of 176 cm and a body weight of 68 kg, who had dry skin and hyperkeratosis During the ophthalmological examination, a significant decrease in twilight vision was detected. Dairy products, vegetables and fruits were practically never included in the diet of the student. Which of the mentioned diseases can be assumed?

Міопія Myopia

Астигматизм Astigmatism

C-гіповітаміноз C-hypovitaminosis

Діенцефальний синдром Diencephalic syndrome

A-гіповітаміноз A-hypovitaminosis

34 / 1500
Пацієнт віком 56 років звернувся до сімейного лікаря зі скаргами на загальну слабкість, підвищення температури тіла до 38,5^oС, кашель із виділенням гнійного мокротиння, незначну задишку. Хворіє протягом трьох днів. З першого дня клінічних проявів приймав амоксицилін-клавуланат, однак стан не покращився. З анамнезу відомо, що пацієнт не палить, протягом останніх шести місяців не приймав антибіотики, п’ять років хворіє на цукровий діабет 2 типу, приймає метформін. Об'єктивно спостерігається: АТ - 130/80 мм рт. ст., пульс - 85/хв. ЧД - 20/хв. Під час перкусії легень виявлено притуплення перкуторного звуку в нижніх відділах правої легені, аускультативно над цією ділянкою вислуховуються ослаблене везикулярне дихання та дрібнопухирчасті вологі хрипи. З боку інших органів та систем патології не виявлено. За результатами загального аналізу крові: нейтрофільний лейкоцитоз із зсувом формули вліво. Яка подальша тактика сімейного лікаря? A 56-year-old patient turned to a family doctor with complaints of general weakness, an increase in body temperature up to 38.5°C, a cough with purulent sputum, slight shortness of breath. He has been sick for three days. He has been taking amoxicillin-clavulanate since the first day of clinical symptoms, but his condition has not improved. It is known that the patient does not smoke, has not taken antibiotics for the past six months, and has been suffering from type 2 diabetes for five years. Objectively, blood pressure is 130/80 mmHg, heart rate is 20/min. breathing and small vesicular wheezes. No pathology was detected on the part of other organs and systems. According to the results of the general blood analysis: neutrophilic leukocytosis with a shift to the left. What is the next tactic of the family doctor?

Призначити доксициклін Prescribe doxycycline

Призначити цефалоспорини І покоління Prescribe 1st generation cephalosporins

Направити на консультацію до ендокринолога Send for a consultation with an endocrinologist

Призначити фторхінолон ІІІ-IV покоління Prescribe fluoroquinolone III-IV generation

Збільшити дозу амоксицилін-клавуланата та додати муколітичні засоби Increase amoxicillin-clavulanate dose and add mucolytics

35 / 1500
У пацієнта віком 29 років протягом трьох тижнів на шкірі червоної кайми нижньої губи спостерігається безболісна виразка, яка останній тиждень супроводжується появою значного набряку підлеглих та навколишніх тканин. Під час огляду виявлено: виразка діаметром до 2,5-3 см, чіткі краї, дно кольору «старого сала», хрящеподібний інфільтрат під нею. Яке лабораторне обстеження потрібно призначити в першу чергу? A 29-year-old patient has had a painless ulcer on the skin of the red border of the lower lip for three weeks, which in the last week has been accompanied by the appearance of significant swelling of the underlying and surrounding tissues. During the examination revealed: an ulcer with a diameter of up to 2.5-3 cm, clear edges, the bottom is the color of 'old lard', a cartilaginous infiltrate under it. What laboratory examination should be prescribed first?

Бактеріальний посів Bacterial culture

Дослідження на еозинофіли Research on eosinophils

Обстеження на бліду трепонему Examination for pale treponema

Обстеження на акантолітичні клітини Examination for acantholytic cells

Зіскоб на мікози Scraping for mycosis

36 / 1500
При проведенні поточного профілактичного медичного огляду робітника цеху з виробництва автомобільних акумуляторів встановлено: шкіра бліда з сірувато-землистим відтінком, видимі слизові бліді. На яснах, переважно біля передніх зубів, є зміна кольору слизової у бузковий колір у вигляді смуги. Є тремор пальців рук. При пальпації рук є біль по ходу нервів. Назвіть токсичну сполуку та основний шлях її надходження до організму людини в умовах виробництва: During the current preventive medical examination of a worker in the workshop for the production of automobile batteries, it was established: the skin is pale with a grayish-earthy hue, the mucous membranes are pale. On the gums, mainly near the front teeth , there is a change in the color of the mucous membrane in the form of a stripe. There is pain along the nerves when palpating the hands. Name the toxic compound and its main route of entry into the human body:

Арсен, інгаляційний Arsenic, inhalation

Кадмій, перкутанний Cadmium, percutaneous

Свинець, інгаляційний Lead, inhalation

Нікель, перкутанний Nickel, percutaneous

Ртуть, пероральний Mercury, oral

37 / 1500
Чоловік госпіталізований після тупої травми грудної клітки. АТ - 100/60 мм рт. ст., пульс - 98/хв. На рентгенограмі виявлено: ОГК без особливостей. Ехокардіографія встановила: вільна рідина в порожнині перикарду в кількості 100 мл. Через годину спостерігається зниження АТ до 85/50 мм рт. ст., набухання шийних вен, пульс - 106/хв., тони серця різко ослаблені. Які найперші заходи слід вжити? A man was hospitalized after a blunt chest injury. Blood pressure - 100/60 mm Hg, pulse - 98/min. X-ray showed: OGK without features. Echocardiography showed: free fluid in the pericardial cavity in the amount of 100 ml. A decrease in blood pressure to 85/50 mmHg is observed, the pulse is 106/min. What are the first measures to be taken?

Інфузія симпатоміметиків Infusion of sympathomimetics

Пункція перикарду Pericard puncture

В/в введення серцевих глікозидів IV administration of cardiac glycosides

Антибіотикотерапія Antibiotic therapy

Ввести діуретики в/в Enter intravenous diuretics

38 / 1500
Жінка 31 року протягом 3 років скаржиться на біль та підпухлість променево-зап'ясних та п'ясно-фалангових суглобів, ранкову скутість до 1,5 години. Два тижні тому з'явився біль, підпухлість та почервоніння колінних суглобів, підвищення температури тіла до 37,5^oС. Під час обстеження внутрішніх органів патологічних змін не виявлено. Поставлено діагноз: ревматоїдний артрит. Які зміни на рентгенограмах суглобів найвірогідніші? A 31-year-old woman has been complaining of pain and swelling of the radiocarpal and metatarsophalangeal joints for 3 years, morning stiffness up to 1.5 hours. Two pain, swelling and redness of the knee joints appeared weeks ago, an increase in body temperature up to 37.5 °C. No pathological changes were detected. A diagnosis was made: rheumatoid arthritis. What changes are most likely on the X-rays of the joints?

Остеоліз епіфізів Osteolysis of epiphyses

Звуження суглобової щілини, субхондральний остеосклероз Narrowing of the joint space, subchondral osteosclerosis

Кісти в субхондральній кістці Cysts in the subchondral bone

Звуження суглобової щілини, узури Narrowing of the joint space, patterns

Множинні краєві остеофіти Multiple marginal osteophytes

39 / 1500
Жінка 30-ти років скаржиться на загальну слабкість, утруднене ковтання їжі, сухість шкіри та ламкість волосся. Об'єктивно: температура тіла - 36,6^oC, ЧД- 16/хв., Ps- 92/хв., АТ- 110/70 мм рт.ст. Шкірні покриви та видимі слизові бліді. У крові: Hb- 65 г/л, еритроцити - 3,2·10^12/л, КП- 0,6, рет - 3%, лейкоцити - 6,7·10^9/л, е - 2%, п - 3%, с - 64%, л - 26%, м - 5%, ШОЕ- 17 мм/год. Сироваткове залізо - 7,4 мкмоль/л, загальний білок - 78 г/л. Дефіцит якого фактору зумовив виникнення захворювання? A 30-year-old woman complains of general weakness, difficulty swallowing food, dry skin and brittle hair. Objectively: body temperature - 36.6^oC, BH- 16/min., Ps- 92/min., BP- 110/70 mm Hg. Skin and mucous membranes are pale. Blood: Hb- 65 g/l, erythrocytes - 3.2·10^12 /l, CP - 0.6, ret - 3%, leukocytes - 6.7·10^9/l, e - 2%, p - 3%, c - 64%, l - 26%, m - 5% , ESR - 17 mm/h. Serum iron - 7.4 μmol/l, total protein - 78 g/l. What factor caused the disease?

Вітаміну B_6 Vitamin B_6

Заліза Iron

Білку Squirrel

Фолієвої кислоти Folic acid

Глюкозо-6-фосфатдегідрогенази Glucose-6-phosphate dehydrogenase

40 / 1500
У пацієнтки віком 63 роки протягом 5 тижнів прогресує безбольова жовтяниця, шкірний свербіж, схуднення на 10 кг, ахолія. Пальпується позитивний симптом Курвуазьє. Який найімовірніший діагноз? A 63-year-old female patient developed painless jaundice, skin itching, weight loss of 10 kg, acholia for 5 weeks. A positive Courvoisier sign is palpated. What is the most likely diagnosis?

Рак підшлункової залози Pancreatic cancer

Рак жовчного міхура Gall Bladder Cancer

Вірусний гепатит Viral hepatitis

Малярія Malaria

Рак печінки Liver cancer

41 / 1500
Раніше здорова жінка 47-ми років звернулася до сімейного лікаря через підвищену температуру та кашель із жовтим мокротинням, що тривають вже 5 днів. Температура тіла - 37,4^oC , пульс - 82/хв., частота дихання - 15/хв., артеріальний тиск - 130/80 мм рт.ст. При аускультації вислуховуються двосторонні розсіяні сухі хрипи. На оглядовій рентгенограмі грудної клітки патологічних змін не виявлено. Яка тактика лікування буде найбільш доречною для цієї пацієнтки? A previously healthy 47-year-old woman consulted a family doctor because of a high temperature and a cough with yellow sputum that has been going on for 5 days. Body temperature is 37.4^ oC, pulse - 15/min, blood pressure - 130/80 mmHg. On auscultation, bilateral diffuse rales are heard. No pathological changes are detected on the chest X-ray most appropriate for this patient?

Інтерферону альфа-2b у сиропі протягом тижня Interferon alfa-2b in syrup for a week

Ципрофлоксацин перорально на 7 днів Ciprofloxacin orally for 7 days

Аскорбінова кислота (вітамін C) перорально на 5 днів Ascorbic acid (vitamin C) orally for 5 days

Вживання достатньої кількості рідини Drinking enough fluids

Комбінований препарат із противірусним та антигістамінним компонентом перорально на 5 днів Combined drug with antiviral and antihistamine component orally for 5 days

42 / 1500
У дитини 9-ти років з'явився біль у верхній третині правої гомілки, підвищилась температура тіла до 39^oC. Дитина не може стати на праву ногу. З анамнезу відомо, що дитина мала травму гомілки та перенесла ангіну. Яке захворювання можна припустити? A 9-year-old child developed pain in the upper third of the right shin, the body temperature rose to 39^oC. The child cannot stand on his right leg. With it is known in the anamnesis that the child had a leg injury and suffered angina. What disease can be assumed?

Туберкульозний артрит Tuberculous arthritis

Гострий ревматизм Acute rheumatism

Гострий гематогенний остеомієліт Acute hematogenous osteomyelitis

Злоякісна пухлина кістки Malignant bone tumor

Перелом кісток правої гомілки Fracture of the bones of the right tibia

43 / 1500
Чоловік скаржиться на болючість, почервоніння шкіри на правій литці. Об'єктивно встановлено: температура тіла - 38,5^oС, збільшені і болючі лімфатичні пахвинні вузли з правого боку, шкіра червона, межі між почервонінням і здоровою шкірою чіткі, є набряк, пухирці з темною рідиною всередині, пальпація болюча. Який попередній діагноз? The man complains of soreness, redness of the skin on the right calf. Objectively established: body temperature - 38.5°C, enlarged and painful inguinal lymph nodes on the right side, the skin is red, the boundaries between redness and healthy skin are clear, there is swelling, blisters with dark liquid inside, palpation is painful. What is the previous diagnosis?

Вітряна віспа Chicken Pox

Герпетична інфекція Herpes infection

Флегмона ноги Phlegmon of the leg

Бешиха Beshikha

Сибірка, шкіряна форма Anthrax, skin form

44 / 1500
Чоловік 46-ти років звернувся до лікаря зі скаргами на появу висипань в ділянці кистей рук, що супроводжуються сильним свербежем і турбують протягом тижня. У анамнезі бронхіальна астма, сезонні алергії. Пацієнт відзначає, що має дуже суху шкіру. При фізикальному обстеженні на тильній стороні правої та лівої кисті виявлено множинні еритематозні папули та везикули з серозним вмістом, ділянки ліхеніфікації та екскоріації. Який діагноз є найбільш імовірним? A 46-year-old man turned to the doctor with complaints about the appearance of rashes in the area of ​​the hands, accompanied by severe itching and bothering him for a week. He has a history of bronchial asthma, seasonal allergies. The patient notes that he has very dry skin. On physical examination, multiple erythematous papules and vesicles with serous contents, areas of lichenification and excoriation are found.

Псоріаз Psoriasis

Себорейний дерматит Seborrheic dermatitis

Короста Scabies

Контактний дерматит Contact dermatitis

Екзема Eczema

45 / 1500
Пацієнтка 37 років. Переведена до очного відділення із ЛОР відділення, у якому лікувалася з приводу гнійного пансинуситу. Об'єктивно встановлено: загальна слабість, температура тіла - 38,5^oC. Гострота зору правого ока - 0,6 (з анамнезу VIS OD=1,0). Повіки синюшні, набряклі, щільні. Хемоз кон'юнктиви. Екзофтальм. Нерухомість очного яблука. Репозиція ока затруднена. Поставте діагноз: The patient is 37 years old. She was transferred to the eye department from the ENT department, where she was treated for purulent pansinusitis. Objectively established: general weakness, body temperature - 38, 5^oC. Visual acuity - 0.6. Eyelids are swollen. Conjunctiva. Immobility of the eyeball. Make a diagnosis:

Псевдотумор орбіти Pseudotumor of the orbit

Гострий блефарокон'юнктивіт Acute blepharoconjunctivitis

Флегмона орбіти Phlegmon of the orbit

Ретробульбарна пухлина Retrobulbar tumor

Ретробульбарна гематома Retrobulbar hematoma

46 / 1500
До сімейного лікаря звернувся па-цієнт з астено-вегетативним синдромом. Пацієнт пред'являв скарги на порушення cну, частий головний біль, запаморочення, дратівливість, підвищену втомлюваність, порушення пам'яті. Пацієнт протягом 2-х років не працював, проживає в житловому будинку, який розташований поруч з аеро-портом. З впливом якого чинника найімовірніше можливе виникнення даних симптомів? A patient with astheno-vegetative syndrome consulted a family doctor. The patient complained of cna disorders, frequent headache, dizziness, irritability, increased fatigue, memory impairment. The patient has not worked for 2 years, lives in a residential building located near the airport. Which factor is most likely to cause these symptoms?

Електромагнітні поля Electromagnetic fields

Ультразвук Ultrasound

Хімічне забруднення атмосферного повітря Chemical air pollution

Шум Noise

Вібрація Vibration

47 / 1500
Батьки восьмирічного хлопчика звернулися до лікаря зі скаргами на відставання дитини у зрості. З анамнезу відомо: народжений доношеним, із вагою 3100 г, довжиною тіла 50 см. Батьки нормального зросту. На момент огляду хлопчика співвідношення зросту до віку = -3, вага - теж знаходиться у діапазоні < -2. <<Кістковий вік>> дорівнює 5 років. Патології з боку інших органів і систем не виявлено. Яке дослідження необхідно призначити дитині для уточнення діагнозу? The parents of an eight-year-old boy turned to the doctor with complaints about the child's growth retardation. It is known from the anamnesis: he was born full-term, with a weight of 3100 g, a body length of 50 cm. The parents of a normal At the time of the examination, the boy's height-to-age ratio is in the range of <-2. No pathology from other organs and systems has been found for the child clarifying the diagnosis?

Визначення рівня лютеїнізуючого гормону Determining the level of luteinizing hormone

Визначення рівня гонадотропного гормону Determining the level of gonadotropic hormone

Визначення рівня соматотропного гормону Determining the level of somatotropic hormone

Визначення рівня тиреотропного гормону Determining the level of thyroid-stimulating hormone

Визначення рівня адренокортикотропного гормону Determining the level of adrenocorticotropic hormone

48 / 1500
Дитина народилась із масою 3800 г від перших фізіологічних пологів. На другий день життя у неї з’явилася везикульозна висипка на шкірі та волосяній частині голови, підвищилася температура тіла до 37,5^oС. Встановлено діагноз: вітряна віспа. До якого віку дитини вітряна віспа вважається вродженою? The child was born with a weight of 3800 g from the first physiological delivery. On the second day of life, she developed a vesicular rash on the skin and scalp, the body temperature rose to 37.5°C. Diagnosed: chicken pox. Until what age is chicken pox considered congenital?

До 6 місяців Up to 6 months

До 10 дня Up to 10 days

До 14 дня Up to 14 days

До 21 дня Up to 21 days

До 30 дня Up to 30 days

49 / 1500
Пацієнта віком 62 роки турбують постійний біль в епігастрії та схуднення на 12 кг. За даними фізикального та інструментального обстежень (фіброгастроскопія з біопсією, УЗД органів черевної порожнини, рентгенографія органів грудної клітки) у пацієнта виявлено рак тіла шлунка без ознак віддаленого метастазування. Висновок гістологічного дослідження: помірно диференційована аденокарцинома. Який адекватний обсяг оперативного втручання? A 62-year-old patient is concerned about constant pain in the epigastrium and weight loss of 12 kg. According to physical and instrumental examinations (fibrogastroscopy with biopsy, ultrasound of the abdominal organs, radiography of the organs of the chest) the patient was found to have cancer of the body of the stomach without signs of distant metastasis. The conclusion of the histological examination: moderately differentiated adenocarcinoma. What is the adequate amount of surgical intervention?

Гастректомія Gastrectomy

Обхідний гастроентероанастомоз Bypass gastroenteroanastomosis

Проксимальна субтотальна резекція шлунка Proximal subtotal gastric resection

Дистальна субтотальна резекція шлунка Distal subtotal gastric resection

Операція Льюїса Lewis operation

50 / 1500
Пацієнт віком 37 років неодноразово перебував на лікуванні в психіатричній лікарні. Поточна госпіталізація зумовлена порушеннями поведінки у вигляді замкненості, відмови від їжі, періодичного хаотичного збудження зі стереотипними рухами. Під час огляду виявлено: пацієнт перебуває в ліжку, загальмований, контакт відсутній, на запитання не відповідає, поза одноманітна, гіпомімічний, симптом «хоботка», воскова гнучкість м'язів, симптом «повітряної подушки». Такий стан триває близько тижня. Встановіть попередній діагноз. A 37-year-old patient was repeatedly treated in a psychiatric hospital. The current hospitalization is due to behavioral disorders in the form of withdrawal, refusal to eat, periodic chaotic excitement with stereotyped movements. Under the time of the examination revealed: the patient is in bed, inhibited, does not respond to questions, the posture is monotonous, the symptom is 'proboscis', the symptom is 'air cushion'. This condition lasts about a week. Establish a preliminary diagnosis .

Шизофренія, гебефренічна форма Schizophrenia, hebephrenic form

Реактивний ступор Reactive stupor

Шизофренія, кататонічна форма Schizophrenia, catatonic form

Шизофренія, проста форма Schizophrenia, simple form

Шизофренія, параноїдна форма Schizophrenia, paranoid form

51 / 1500
Пацієнтка 74 років скаржиться на біль та здуття живота, нудоту. Страждає на ішемічну хворобу серця, постінфарктний та дифузний кардіосклероз. Об'єктивно спостерігається: стан важкий, живіт здутий, черевна стінка слабко бере участь в акті дихання. Під час лапароскопії виявлено: у черевній порожнині невелика кількість каламутного випоту, одна з петель тонкої кишки темно-синюшнього кольору. Поставте діагноз. A 74-year-old patient complains of pain and abdominal distension, nausea. She suffers from coronary heart disease, post-infarction and diffuse cardiosclerosis. Objectively observed: the condition is severe, the abdomen is distended , the abdominal wall is weakly involved in the act of breathing. During laparoscopy, a small amount of cloudy effusion was found in the abdominal cavity, one of the loops of the small intestine is dark blue. Make a diagnosis.

Гостра кишкова непрохідність Acute intestinal obstruction

Заворот кишечнику Intestinal twist

Бешихове запалення Beshikov inflammation

Ішемічний абдомінальний синдром Ischemic abdominal syndrome

Тромбоз мезентеріальних судин Thrombosis of mesenteric vessels

52 / 1500
Лабораторією СЕС визначений вміст афлотоксинів у м'ясі корів, яким згодовували запліснявілий корм. Рівень афлотоксину перевищує гранично-допустимий у 5 разів. Назвіть можливі зрушення в здоров'ї внаслідок афлатоксикозу у населення, що тривалий час вживає таке м'ясо: The SES laboratory determined the content of aflatoxins in the meat of cows that were fed with moldy feed. The level of aflatoxin exceeds the permissible limit by 5 times. Name the possible changes in health as a result of aflatoxicosis in the population that consumes such meat for a long time:

Гломерулонефрит Glomerulonephritis

- -

Цироз і первинний рак печінки Cirrhosis and primary liver cancer

Анемія Anemia

Атеросклероз Atherosclerosis

53 / 1500
У чоловіка 22 років з'явились сильні болі в нижній частині спини, які ірра-діюють у п'яткові кістки, внутрішню поверхню стегон, спостерігається підвищення температури до 38,2^oС. За три тижні відчув важкорухомість у хребті, болі в ілеосакральних суглобах. Рентгенологічно виявлено: розширення щілини ілеосакрального з'єднання, склерозування спінальних зв'язків. Про яке захворювання слід думати? A 22-year-old man developed severe pain in the lower back, radiating to the heel bones, the inner surface of the thighs, an increase in temperature to 38 ,2^oC. In three weeks, I felt stiffness in the spine, pain in the ileosacral joints. X-ray revealed: enlargement of the ileosacral joint. What disease should I think about?

Грижа спинального каналу Hernia of the spinal canal

Анкілозуючий спондилоартрит Ankylosing spondylitis

Оперізуючий лишай Zingles

Міжреберна невралгія Intercostal neuralgia

Киста сакрального відділу хребта Sacral spine cyst

54 / 1500
У чоловіка 33-х років з'явилися множинні висипання на тулубі, розгинальних поверхнях верхніх та нижніх кінцівок, які місцями зливаються з утворенням бляшок, супроводжуються свербінням. Поверхня елементів висипань вкрита сріблясто-білими дрібнопластинчастими лусочками, які при зішкрібанні легко знімаються. При проведенні граттажу спостерігаються три послідовні феномени: стеаринової плями, термінальної плівки та крапкової кровотечі. Який діагноз можна припустити? A 33-year-old man developed multiple rashes on the trunk, extensor surfaces of the upper and lower limbs, which in places merge with the formation of plaques, accompanied by itching. The surface of the elements the rash is covered with silvery-white scales, which are easily removed when scraping. Three consecutive phenomena are observed: stearin spot, terminal film, and dot bleeding?

Парапсоріаз Parapsoriasis

Вторинний папульозний сифілід Secondary papular syphilis

Червоний плескатий лишай Red lichen planus

Псоріаз Psoriasis

Піодермія Pyoderma

55 / 1500
Жінка 22 років із підозрою на системний червоний вовчак скаржиться на <<летючий>> біль у суглобах рук і ніг, підвищення температури до 38,5-39^oС упродовж 3 тижнів, задишку, серцебиття, слабкість. Об'єктивно встановлено: еритема на щоках та носі. У крові виявлено: Нb - 90 г/л, тромбоцити - 135·10^9/л, ШОЕ - 43 мм/год. У сечі виявлено: білок - 2,66 г/л, еритроцитів - 8-10 в п/з. Виявлення яких антитіл у пацієнтки найбільш значиме для постановки діагнозу? A 22-year-old woman with suspicion of systemic lupus erythematosus complains of <> pain in the joints of her hands and feet, an increase in temperature to 38.5-39^oС for 3 weeks, shortness of breath, weakness. Objectively established: erythema on the cheeks and nose. Blood - 90 g/l, platelets - 43 mm/h urine was found: protein - 2.66 g/l, erythrocytes - 8-10 in p/z. Which antibodies are most significant for the diagnosis in the patient?

До тромбоцитів To platelets

До 2-х спіральної нативної ДНК Up to 2-helix native DNA

Ревматоїдний фактор Rheumatoid factor

Кріоглобуліни Cryoglobulins

До фосфоліпідів To phospholipids

56 / 1500
Хлопчик 11-ти років хворіє на бронхіа-льну астму. На тлі ГРВІ з'явилася ядуха, сухий кашель, свистяче дихання. Па-цієнт впродовж години тричі з інтервалом у 20 хвилин отримував інгаляції бета-2-агоніста короткої дії та антихолінергічного засобу, проте стан не покращився. Наступним кроком невідкладної допомоги є призначення: An 11-year-old boy suffers from bronchial asthma. On the background of SARS, dyspnea, dry cough, and wheezing appeared. The patient three times within an hour with received inhalations of a short-acting beta-2-agonist and an anticholinergic agent at 20-minute intervals, but the condition did not improve. The next step in emergency care is to prescribe:

Системних глюкокортикостероїдів Systemic glucocorticosteroids

Метилксантинів короткої дії Short-acting methylxanthines

Муколітичних препаратів Mucolytic drugs

Седативних препаратів Sedative drugs

Магнезії сульфату Magnesium sulfate

57 / 1500
Хворий 52-х років звернувся до лікаря зі скаргами на задишку, кашель із мокротинням з домішками крові, підвищену пітливість вночі. В сімейному анамнезі хворих на рак легень немає. За даними картки профілактичних щеплень вакцину БЦЖ отримав вчасно. При фізикальному обстеженні температура - 37,2^oC, артеріальний тиск - 110/70 мм рт.ст., пульс - 100/хв., частота дихання - 20/хв., SpO_2 - 94% при кімнатному повітрі. При аускультації грудної клітки справа дихальні шуми не прослуховуються. На рентгенограмі ОГК правосторонній гідроторакс. Який наступний крок лікаря буде найбільш доречним? A 52-year-old patient consulted a doctor with complaints of shortness of breath, cough with blood-tinged sputum, increased sweating at night. There is no family history of lung cancer patients. According to the preventive vaccination card, he received the BCG vaccine on time. During the physical examination, the temperature was 37.2°C, the blood pressure was 110/70 mm Hg, the pulse was 100/min., the respiratory rate was 20/min., SpO_2 - 94% with room air. When auscultating the chest, no breathing sounds are heard. Right-sided hydrothorax will be the most appropriate step for the doctor?

Діагностичний торакоцентез Diagnostic thoracentesis

УЗД легень Lung ultrasound

Проба Манту Mantoux Test

Тест вивільнення гамма-інтерферону Gamma interferon release test

Бронхоскопія Bronchoscopy

58 / 1500
Чоловік віком 49 років скаржиться на напади стенокардії, що виникають під час ходьби на відстань до 500 м. З анамнезу відомо, що він багато років хворіє на хронічний бронхіт. Під час обстеження виявлено: в легенях невелика кількість сухих розсіяних хрипів, ЧД - 18/хв. Межі серця розширені вліво, тони приглушені, ЧСС=ПС=86/хв., АТ - 160/100 мм рт. ст. У клінічному аналізі крові виявлено: Hb - 160 г/л, лейкоцити - 6,4 тис., ШОЕ - 7 мм/год. На ЕКГ спостерігається гіпертрофія лівого шлуночка. Застосування якої групи ліків протипоказане з урахуванням супутньої патології у пацієнта? A 49-year-old man complains of angina attacks that occur while walking up to 500 m. From the anamnesis, it is known that he has been suffering from chronic bronchitis for many years. During the examination, a small number of scattered wheezes were detected, heart rate was 18/min, heart rate was muffled, HR=86/min revealed: Hb - 160 g/l, leukocytes - 6.4 thousand, ESR - 7 mm/h. On the ECG, the use of which group of drugs is contraindicated in the patient's accompanying pathology?

Пролонговані нітрати Prolonged Nitrates

Ангіопротектори Angioprotectors

Бета-блокатори Beta blockers

Антиагреганти Antiplatelets

Антагоністи кальцію Calcium antagonists

59 / 1500
Хворий 48-ми років скаржиться на випорожнення 2-3 рази на день, великий обсяг калових мас з неприємним запахом, супроводжується болем навколо пупка, випадінням волосся, парестезіями. При огляді: блідість шкіри, знижена маса тіла, набряки на нижніх кінцівках. Болючість при пальпації навколо пупка та кишечника. В аналізах: в крові - анемія, в аналізі калу - стеаторея, креаторея, амілорея. Який синдром спостерігається у хворого? A 48-year-old patient complains of defecation 2-3 times a day, a large volume of feces with an unpleasant smell, accompanied by pain around the navel, hair loss, paresthesias. On examination: paleness of the body, swelling on the lower limbs. In the tests: in the blood - anemia, in the analysis of the stool - what syndrome is observed in the patient?

Синдром привідної кишки Syndrome of the duodenum

Синдром Золлінгера-Еллісона Zollinger-Ellison Syndrome

Синдром мальабсорбції Malabsorption syndrome

Синдром гіперкатаболічної ексудативної ентеропатії Syndrome of hypercatabolic exudative enteropathy

Демпінг-синдром Dumping syndrome

60 / 1500
В операційній під час проведення контролю за дотриманням санітарно-гігієнічного режиму відібрано проби повітря для лабораторного аналізу. Отримано такі результати: загальна кількість мікроорганізмів в 1 м^3 повітря - 490, зокрема гемолітичний стрептокок - не виявлено. Встановіть ступінь чистоти повітря в операційній. Air samples were taken for laboratory analysis in the operating room during monitoring of compliance with the sanitary and hygienic regime. The following results were obtained: the total number of microorganisms in 1 m^3 of air - 490, in particular, hemolytic streptococcus - not detected. Set the degree of air cleanliness in the operating room.

Чисте Net

Дуже чисте Very clean

Забруднене Contaminated

Помірно забруднене Moderately polluted

Сильно забруднене Heavily polluted

61 / 1500
Хвора 35-ти років захворіла гостро з лихоманки до 39,0^oC та кашлю. Через 3 дні задишка спокою зросла до 35/хв. Вниз від кута правої лопатки перкуторно визначається тупий звук, відсутнє голосове тремтіння, дихальні шуми не вислуховую-ться. Яка тактика лікування? A 35-year-old patient became acutely ill with fever up to 39.0^oC and cough. After 3 days, rest dyspnea increased to 35/min. Down from the corner of the right a dull sound is determined by percussion of the scapula, there is no voice tremor, no breathing sounds are heard. What is the treatment strategy?

Штучна вентиляція легень Artificial lung ventilation

Плевральна пункція Pleural puncture

Фізіотерапія Physiotherapy

Антибіотикотерапія Antibiotic therapy

Оксигенотерапія Oxygenotherapy

62 / 1500
Жінка 31 року доставлена до лікарні в непритомному стані. Зі слів супроводжувачів в анамнезі у хворої зазначена двобічна адреналектомія з приводу хвороби Іценко-Кушинга. Об'єктивно встановлено: ціаноз слизових, гіперпігментація, клонічні судоми, загальмованість, АТ - 80/60 мм рт. ст., температура тіла - 34,5^oС, тони серця глухі. Під час пальпації живота спостерігається напруження передньої черевної стінки. Які невідкладні засоби лікарняної допомоги слід використати насамперед? A 31-year-old woman was brought to the hospital in an unconscious state. According to the attendants, the patient had bilateral adrenalectomy due to Itsenko-Cushing's disease. Objectively established: cyanosis mucous membranes, clonic convulsions, blood pressure - 80/60 mm Hg, heart sounds dull. During palpation of the abdomen, what emergency measures should be used first of all?

Призначення панангіну Panangin appointment

Уведення інсуліну Insulin administration

Уведення 40%-го розчину глюкози Introduction of 40% glucose solution

Внутрішньовенне введення електролітів Intravenous administration of electrolytes

Уведення гідрокортизону сукцинату Hydrocortisone succinate administration

63 / 1500
Жінка 35 років скаржиться на задишку, відчуття стискання в правій половині грудної клітки, кашель із виділенням невеликої кількості слизово-гнійного харкотиння. Хворіє тиждень. Скарги пов'язує з переохолодженням. Об'єктивно виявлено: температура тіла - 38,7^oС, легкий акроціаноз губ, Рs - 90/хв., АТ - 140/85 мм рт. ст. Права половина грудної клітки відстає в акті дихання. Перкуторно встановлено: справа нижче кута лопатки тупість з межею до верху, дихання не вислуховується. Який найімовірніший діагноз? A 35-year-old woman complains of shortness of breath, a feeling of tightness in the right half of the chest, cough with the release of a small amount of mucous-purulent sputum. She has been ill for a week. She associates the complaints with Objectively revealed: body temperature - 38.7°C, Рs - 140/85 mmHg below the angle of the scapula, dullness with a border to the top, breathing is not heard. What is the most likely diagnosis?

Негоспітальна пневмонія Community-acquired pneumonia

Ателектаз легені Atelectasis of the lung

Абсцес легені Lung abscess

Госпітальна пневмонія Nosocomial pneumonia

Ексудативний плеврит Exudative pleurisy

64 / 1500
Під час огляду студента лікарем було виявлено сухість та зроговіння шкіри на ліктях та колінах. На шкірі сідниць, стегон та литок зроговіння епітелію, якщо провести рукою, відчувається шорсткуватість шкіри цих ділянок, яке нагадувало <<гусячу шкіру>>. Студент також скаржився на погіршення зору у сутінках. Яке із наведених захворювань можна припустити? During the examination of the student, the doctor found dryness and keratinization of the skin on the elbows and knees. On the skin of the buttocks, thighs, and calves, keratinization of the epithelium, if you run your hand over it, you can feel the roughness of the skin of these areas, which resembled 'goosebumps'. The student also complained about the deterioration of vision at dusk. Which of the following diseases can be assumed?

Гіповітаміноз В_6 Hypovitaminosis B_6

Гіповітаміноз D Hypovitaminosis D

Гіповітаміноз С Hypovitaminosis C

Гіповітаміноз А Hypovitaminosis A

Гіповітаміноз К Hypovitaminosis K

65 / 1500
Добовий раціон харчування дівчинки 10 років містить: вітамін В_1 - 1,2 мг, вітамін В_2 - 0,6 мг, вітамін В_6 - 1,4 мг, вітамін РР - 15 мг, вітамін С - 65 мг. під час оцінювання харчового статусу у дівчинки виявлено хейлоз, глосит, ангулярний стоматит, кон'юктивіт. У дівчинки, найімовірніше: The daily diet of a 10-year-old girl contains: vitamin B_1 - 1.2 mg, vitamin B_2 - 0.6 mg, vitamin B_6 - 1.4 mg, vitamin RR - 15 mg, vitamin C - 65 mg. Cheilosis, glossitis, angular stomatitis, conjunctivitis were detected in the girl, most likely:

С - гіповітаміноз C - hypovitaminosis

РР - гіповітаміноз RR - hypovitaminosis

В_2 - гіповітаміноз B_2 - hypovitaminosis

В_6 - гіповітаміноз B_6 - hypovitaminosis

В_1 - гіповітаміноз B_1 - hypovitaminosis

66 / 1500
Хвора 17-ти років при зрості 172 см і масі тіла 40 кг вважає себе дуже повною. Останні 2 роки дотримується суворої низькокалорійної дієти, одночасно до знемагання займається фізичними вправами, стягує талію шнурами. Часто викликає штучну блювоту. Скаржиться на неприєм-ні відчуття в епігастрії, аменорею. Визначте найбільш імовірний діагноз: A 17-year-old patient with a height of 172 cm and a body weight of 40 kg considers herself very fat. For the past 2 years, she has been following a strict low-calorie diet, and at the same time she is doing physical exercises until exhaustion , tightens the waist with cords. Often causes artificial vomiting. Complains of unpleasant sensations in the epigastrium. Determine the most likely diagnosis:

Реакція на стрес Reaction to stress

Розлад особистості Personality Disorder

Булімія Bulimia

Депресія Depression

Анорексія Anorexia

67 / 1500
Жінка 48 років звернулась із приводу того, що протягом останніх 8-9 місяців менструації дуже рясні, призводять до анемізації, порушена працездатність. Протягом 2 років спостерігається гінекологом з приводу фіброміоми матки. Об'єктивно встановлено: шийка матки циліндрична, чиста, вічко закрите. Тіло матки збільшене до 9-10 тижнів вагітності, цупке, рухоме, безболісне. Придатки з обох сторін не визначаються. Параметрії вільні. Склепіння глибокі. Виділення слизові. Який найбільш вірогідний діагноз? A 48-year-old woman complained about the fact that during the last 8-9 months, menstruation is very abundant, leads to anemia, impaired working capacity. For 2 years, she has been observed by a gynecologist for Objectively, the uterus is cylindrical, the eye is closed, the uterus is tender, the appendages are not defined. The discharge is deep the most likely diagnosis?

Дисфункциональна маткова кровотеча Dysfunctional uterine bleeding

Ендометріоз із переважним ураженням тіла матки Endometriosis with predominant damage to the body of the uterus

Рак ендометрію Endometrial cancer

Фіброміома матки та вагітність Uterine fibroids and pregnancy

Фіброміома матки Uterine fibromyoma

68 / 1500
У чоловіка віком 25 років після переохолодження з'явилась гематурія. Об'єктивно спостерігається: АТ - 160/110 мм рт. ст. У аналізі сечі визначається: протеїнурія 3,5 г/л, еритроцити покривають усе поле зору, циліндри гіалінові 5-6 у полі зору. Який метод діагностики є найінформативнішним у цьому разі? A 25-year-old man developed hematuria after hypothermia. Objectively observed: blood pressure - 160/110 mm Hg. Urinalysis revealed: proteinuria 3.5 g/l, erythrocytes cover the entire field of vision, hyaline cylinders 5-6 in the field of vision. What diagnostic method is the most informative in this case?

Біопсія нирки Kidney biopsy

Екскреторна урографія Excretory urography

УЗД нирок Kidney ultrasound

Томографія Tomography

Цистоскопія Cystoscopy

69 / 1500
У потерпілого опікова травма 15 % поверхні тіла II-III ступеня. На двадцяту добу після травми у пацієнта спостерігаються різке підвищення температури тіла, загальна слабкість, часте поверхневе дихання, артеріальний тиск - 90/50 мм рт. ст., пульс - 112/хв. Яке ускладнення можна припустити? The victim has a burn injury of 15% of the body surface II-III degree. On the twentieth day after the injury, the patient has a sharp rise in body temperature, general weakness, frequent shallow breathing, blood pressure - 90/50 mm Hg, pulse - 112/min. What complications can be assumed?

Пневмонія Pneumonia

Анаеробна інфекція Anaerobic infection

Гнійний бронхіт Purulent bronchitis

Сепсис Sepsis

Гостра інтоксикація Acute intoxication

70 / 1500
На прийом до лікаря-венеролога звернувся пацієнт, якому на підставі клінічних проявів і даних лабораторного обстеження був встановлений діагноз «урогенітальний хламідіоз». Під час призначення лікування цьому пацієнту основним препаратом потрібно вважати: A patient who, on the basis of clinical manifestations and laboratory examination data, was diagnosed with 'urogenital chlamydia'. During the appointment of treatment for this patient, the main the drug should be considered:

Сульфаніламідні препарати Sulfonamide drugs

Препарати групи тинідазолу Drugs of the tinidazole group

Антибіотики пеніцилінового ряду Antibiotics of the penicillin series

Циклоферон або інші імуномодулятори Cycloferon or other immunomodulators

Антибіотики макроліди Macrolide antibiotics

71 / 1500
Який недостатньо оброблений продукт харчування може викликати захворювання - дифілоботріоз? Which insufficiently processed food product can cause the disease - diphyllobotriosis?

Баранина Lamb

М'ясо птиці Poultry

Свинина Pork

Риба Fish

Яловичина Beef

72 / 1500
Хлопчик 6 років поступив із скаргами на набряки обличчя і нижніх кінцівок. Об'єктивно встановлено: у нижніх відділах легень притуплення, ослаблене дихання. Набряклість передньої черевної стінки. Асцит. АТ - 90/50 мм рт. ст. Загальний аналіз сечі виявив: білок - 4,2 г/л, лейкоцити - 5-6 у п/з, еритроцити - 2-3 у п/з. Загальний білок крові - 48,6 г/л, холестерин - 8,2 ммоль/л. Попередній діагноз: гострий гломерулонефрит. Укажіть клінічний варіант захворювання: A 6-year-old boy was admitted with complaints of swelling of the face and lower limbs. Objectively established: dullness in the lower parts of the lungs, weakened breathing. Swelling of the anterior abdominal wall. Ascites Blood pressure - 90/50 mmHg. Urinalysis - 4.2 g/l, erythrocytes - 2-3 g/l .6 g/l, cholesterol - 8.2 mmol/l. Preliminary diagnosis: acute glomerulonephritis. Specify the clinical variant of the disease:

Нефротичний синдром Nephrotic syndrome

Нефротичний синдром з гематурією і гіпертензією Nephrotic syndrome with hematuria and hypertension

Нефротичний синдром з гематурією Nephrotic syndrome with hematuria

Сечовий синдром Urinary syndrome

Нефритичний синдром Nephritic syndrome

73 / 1500
Проведено дослідження з метою визначення зв'язку між споживанням мікронут-рієнтів та захворюванням на катаракту серед людей похилого віку. Участь у дослідженні взяли 2900 людей у віці від 49 до 97 років. Кожний учасник заповнював опитувальник частоти споживання певних харчових продуктів та проходив офтальмоскопію з фотографуванням кришталика. У результаті дослідження, було виявлено, що достатня кількість у раціоні вітаміну A, рибофлавіну, ніацину та тіаміну поперед-жують виникнення ядерної катаракти. Який з перерахованих варіантів найбільш точно описує вид проведеного клінічного дослідження? A study was conducted to determine the relationship between micronutrient intake and cataracts in the elderly. The study involved 2,900 people aged 49 to 97 years old. Each participant filled out a questionnaire on the frequency of consumption of certain foods and underwent ophthalmoscopy. As a result of the study, it was found that sufficient amounts of vitamin A, niacin and thiamine in the diet prevent the occurrence of nuclear cataracts most accurately describes the type of clinical research conducted?

Крос-секціональне поперечне дослідження Cross-sectional cross-sectional study

Опис серії клінічних випадків Description of a series of clinical cases

Дослідження ''випадок-контроль'' Case-control study

Рандомізоване клінічне дослідження Randomized clinical trial

Проспективне когортне дослідження Prospective cohort study

74 / 1500
Хлопчика 5-ти років привели на прийом до лікаря зі скаргами на стрибкоподібне підвищення температури до 39,3^oC протягом 12-ти днів. При фізикальному обстеженні виявлені: ін'єкція бульбарних кон'юн-ктив з обох сторін, потріскані червоні губи та ''малиновий'' язик, шийна лімфаденопатія та еритематозний папульозний висип промежини, набряклість рук та ніг. Що з перерахованого лікар має призначити першочергово? A 5-year-old boy was brought to a doctor's appointment with complaints of a sudden rise in temperature up to 39.3^oC for 12 days. Physical examination revealed: injection of the bulbar conjunctiva on both sides, cracked red lips and 'raspberry' tongue, cervical lymphadenopathy and erythematous papular rash of the perineum, swelling of the hands and feet. Which of the following should the doctor prescribe first?

Пеніцилін внутрішньовенно Penicillin IV

Фуросемід перорально Furosemide Oral

Імуноглобулін людини нормальний внутрішньовенно Human immunoglobulin normal IV

Валацикловір перорально Valacyclovir Oral

Преднізолон внутрішньовенно Prednisone IV

75 / 1500
Жінка 32-х років звернулась до лікаря зі скаргами на відсутність вагітності впродовж 4-х років. У анамнезі: 5 років тому перша вагітність закінчилась штучним абортом. За даними вагінального дослідження та ультразвукового дослідження (УЗД) встановлено діагноз: ендометріоїдна кіста правого яєчника. Який оптимальний метод лікування? A 32-year-old woman turned to the doctor with complaints about the absence of pregnancy for 4 years. History: 5 years ago, the first pregnancy ended in an artificial abortion. According to a diagnosis of endometrioid cyst of the right ovary was established by vaginal examination and ultrasound examination. What is the optimal method of treatment?

Оперативна лапароскопія Operative laparoscopy

Протизапальна терапія Anti-inflammatory therapy

Санаторно-курортне лікування Sanatorium-resort treatment

Терапія андрогенами Androgen therapy

Консервативна терапія естроген-геста-генними препаратами Conservative therapy with estrogen-progestogen drugs

76 / 1500
Пацієнтка 40 років доставлена зі скаргами на переймоподібні болі внизу живота та надмірні кров'яні виділення зі статевих шляхів. Останні два роки менструації тривають до 16 днів, надмірні, із згустками, болісні. Під час бімануального дослідження встановлена наявність фіброматозного вузла, що народжується. Оберіть правильну тактику: A 40-year-old female patient was brought in with complaints of spasm-like pains in the lower abdomen and excessive bleeding from the genital tract. For the last two years, menstruation has lasted up to 16 days, excessive, with clots, painful. During the bimanual examination, the presence of a nascent fibromatous node is established. Choose the correct tactics:

Видалення фіброматозного вузла вагінально Removal of a fibromatous node vaginally

Надпіхвова ампутація матки без додатків Supravaginal amputation of the uterus without attachments

Вітамінотерапія Vitamin therapy

Екстирпація матки без додатків Extirpation of the uterus without attachments

Гормональний гемостаз Hormonal hemostasis

77 / 1500
Пацієнт віком 35 років звернувся до лікарні зі скаргами на напади болю в правій поперековій ділянці, часте сечовипускання. В загальному дослідженні сечі білок 0,066 г/л; свіжі еритроцити 6-8 в полі зору. За даними УЗД помірний уростаз справа. Яке дослідження необхідно виконати пацієнту для встановлення діагнозу? A 35-year-old patient came to the hospital with complaints of pain attacks in the right lumbar region, frequent urination. In the general examination of urine, protein 0.066 g/l; fresh red blood cells 6 -8 in the field of vision. According to the ultrasound, moderate urostasis is needed for the patient to establish the diagnosis?

Екскреторну урографію Excretory urography

Хромоцистоскопію Chromocystoscopy

Пнеморетроперітонеографію Pneumoretroperitoneography

Ретроградну уретеропієлографію Retrograde ureteropyelography

Комп’ютерну томографію Computed tomography

78 / 1500
Пацієнт віком 20 років звернувся до лікаря зі скаргами на появу висипу на шкірі. Об'єктивно спостерігається: на шкірі тулуба, верхніх кінцівок, задньої поверхні шиї плямистий висип розміром 1-3 см, світло-коричневого кольору, що місцями зливається. Позитивна проба Бальцера. Який найімовірніший діагноз? A 20-year-old patient turned to the doctor with complaints about the appearance of a rash on the skin. Objectively observed: on the skin of the trunk, upper limbs, back of the neck, a spotted rash 1-3 cm, light brown, confluent in places. Positive Balzer's test. What is the most likely diagnosis?

Різнокольоровий лишай Multicolored lichen

Парапсоріаз Parapsoriasis

Вторинний сифіліс Secondary syphilis

Мікробна екзема Microbial Eczema

Рожевий лишай Pink lichen

79 / 1500
Робітник 40 років звернувся в поліклініку зі скаргами на швидку втомлюваність, епізодичні болі в животі, порушення сну. Півтора роки працює на приватному підприємстві у цеху зарядки акумуляторів та вулканізації шин. Об'єктивно встановлено: тони серця приглушені, АТ - 140/70 мм рт. ст., пульс - 84/хв., дихання везикулярне. У крові виявлено: Нb - 120 г/л, еритроцити - 4,0·10^12/л, еритроцити з базофільною зернистістю, загальна кількість ретикулоцитів - 0,3%. Аналіз сечі: питома вага - 1,018 г/см^3, лейкоцити - 5 в полі зору, креатинін сечі - 300 мкг/г. Визначте професійну патологію та вкажіть, які лабораторні дослідження підтвердять її наявність: A 40-year-old worker came to the clinic with complaints of rapid fatigue, episodic abdominal pain, sleep disturbances. He has been working for a year and a half at a private enterprise in a battery charging and tire vulcanizing shop Objectively, heart sounds are muffled, blood pressure - 140/70 mm Hg, vesicular respiration is found: Нb - 120 g/l, erythrocytes - 4.0·10 12/l, erythrocytes with basophilic granularity, total number of reticulocytes - 0.3%. Urine analysis: specific gravity - 1.018 g/cm^3, leukocytes - 5 in the field of vision, urine creatinine - 300 μg/g indicate which laboratory tests will confirm its presence:

Хронічна свинцева інтоксикація (сатурнізм); базофільна зернистість еритроцитів та креатинін сечі Chronic lead intoxication (saturnism); basophilic granularity of erythrocytes and urine creatinine

Хронічна свинцева інтоксикація (сатурнізм), скарги хворого Chronic lead intoxication (saturnism), complaints of the patient

Хронічна ртутна інтоксикація (меркуріалізм); креатинін сечі Chronic mercury intoxication (mercurialism); urinary creatinine

Цинкова інтоксикація; загальна кількість ретикулоцитів Zinc intoxication; total reticulocyte count

Карбокситоксикоз; рівень гемоглобіну в крові Carboxytoxicosis; hemoglobin level in blood

80 / 1500
Пацієнтка віком 17 років скаржиться на біль та набряк ІІ пальця правої кисті. З анамнезу відомо що 3 доби по тому зробила манікюр. Біль з'явився на другий день. Об'єктивно спостерігається: навколонігтьовий валик набрякший, гіперемований, нависає над нігтьовою пластинкою, болючий під час пальпації. Який найімовірніший діагноз? A 17-year-old patient complains of pain and swelling of the II finger of the right hand. From the anamnesis, it is known that she had a manicure 3 days later. The pain appeared on the second day. Objectively observed: the nail plate is swollen, hyperemic, hanging over the nail plate, painful during palpation. What is the most likely diagnosis?

Шкірний панарицій Skin panaritium

Піднігтьовий панарицій Subungual panaritium

Підшкірний панарицій Subcutaneous panaritium

Пароніхія Paronychia

Еризипелоїд Erysipeloid

81 / 1500
У шестимісячної дитини гостро підвищилася температура тіла до 39^oС, з'явилося блювання, відмова від їжі, короткочасні тоніко-клонічні судоми. Об'єктивно спостерігається: дитина в'яла, сонлива, ригідність м'язів потилиці, симптоми Керніга, вибухання та напруження тім'ячка. Яке лабораторно-інструментальне обстеження необхідно провести для підтвердження діагнозу? A six-month-old child's body temperature rose sharply to 39^oС, vomiting, refusal to eat, short-term tonic-clonic convulsions appeared. Objectively observed: the child flabby, sleepy, stiffness of the occipital muscles, Kernig's symptoms, bulging and tension of the crown of the head. What laboratory-instrumental examination should be performed to confirm the diagnosis?

МРТ головного мозку MRI brain

Рентгенографія черепа X-ray skull

Біохімічне дослідження сироватки крові Biochemical examination of blood serum

Нейросонографія Neurosonography

Люмбальна пункція Lumbar puncture

82 / 1500
У хворого 2 років із рецидивуючим бронхо-легеневим захворюванням, гіпотрофією, установлений діагноз: муковісцидоз. Яке дослідження повинно підтвердити клінічний діагноз? A 2-year-old patient with recurrent broncho-pulmonary disease, hypotrophy, diagnosed with cystic fibrosis. What research should confirm the clinical diagnosis?

Рівень кальцію в крові Calcium level in blood

Рівень лужної фосфатози в сироватці Serum alkaline phosphatase level

Наявність білку в сечі Presence of protein in urine

Кількість хлоридів у поті Number of chlorides in sweat

Вміст 2-фракції гамма-глобулінів Content 2-fraction of gamma-globulins

83 / 1500
Пацієнтка віком 32 роки скаржиться на загальну слабкість, головний біль, надлишкову масу тіла, біль у кістках, порушення менструального циклу. Об'єктивно спостерігається: шкіра суха, ціанотична. У ділянці живота, плечей, стегон багряно-ціанотичні смуги розтягу. Відкладення жиру переважно на обличчі, шиї, тулубі. АТ - 165/100 мм рт. ст. У аналізі крові виявлено: глюкоза - 7,2 ммоль/л, рівень АКТГ - підвищений. Під час рентгенологічного дослідження кісток виявлено: ознаки остеопорозу. Який найімовірніший діагноз? A 32-year-old patient complains of general weakness, headache, excess body weight, bone pain, irregular menstrual cycle. Objectively observed: dry, cyanotic skin Cyanotic stretch marks on the abdomen, neck, and trunk. Blood pressure is 7.2 mmol/l - increased. During the X-ray examination of the bones, signs of osteoporosis were found. What is the most likely diagnosis?

Ожиріння аліментарне Alimentary obesity

Хвороба Іценко-Кушинга Itsenko-Cushing disease

Синдром Іценко-Кушинга Itsenko-Cushing syndrome

Патологічний клімакс Pathological climax

Гіпертонічна хвороба Hypertensive disease

84 / 1500
У чоловіка о 5-й годині ранку стався напад сильного болю за грудиною, що супроводжувався кволістю та холодним потом. На ЕКГ виявлено: куполоподібний підйом сегмента ST у відведеннях ІІ, ІІІ, avF, V5-V6. Після вщухання болю показники ЕКГ повернулися до норми. Який імовірний діагноз? At 5 o'clock in the morning, a man had an attack of severe pain behind the sternum, accompanied by weakness and cold sweat. The ECG revealed: dome-shaped elevation of the ST segment in leads II , ІІІ, avF, V5-V6. After the pain subsided, the ECG parameters returned to normal. What is the probable diagnosis?

Спонтанна стенокардія типу Prinzmetal Spontaneous angina of the Prinzmetal type

Дилятаційна кардіоміопатія Dilated cardiomyopathy

Міокардит Myocarditis

Гострий інфаркт міокарда в задньобоковій ділянці лівого шлуночка Acute myocardial infarction in the posterolateral area of ​​the left ventricle

Перикардит Pericarditis

85 / 1500
Вагітна жінка віком 25 років скаржиться на підвищення температури до 38,5^oC впродовж двох днів, кашель, задишку. Ці скарги з'явилися після переохолодження. Під час аускультації виявлено: крепітація та локалізовані вологі хрипи в нижній частині правої легені, там же спостерігається притуплення перкуторного звуку. У загальному аналізі крові спостерігається: лейкоцити - 11,0·10^9/л, ШОЕ - 22 мм/год. Який антибактеріальний засіб потрібно призначити? A 25-year-old pregnant woman complains of an increase in temperature to 38.5^oC for two days, cough, shortness of breath. These complaints appeared after hypothermia. During auscultation revealed: crepitation and localized moist rales in the lower part of the right lung, there is a dullness of the percussion sound. In the general blood analysis, leukocytes are observed - 11.0·10^9/l, ESR - 22 mm/h appoint?

Доксициклін Doxycycline

Амікацин Amikacin

Амоксицилін Amoxicillin

Левофлоксацин Levofloxacin

Карбеніцилін Carbenicillin

86 / 1500
Раніше здоровий хлопець 22-х років прийшов на прийом до лікаря зі скаргами на підвищену температуру та кашель з жовтим мокротинням, що продовжуються протягом 7-ми днів. При фізикальному обстеженні температура - 38,3^oC, артеріальний тиск - 130/70 мм рт.ст., частота серцевих скорочень - 79/хв., частота дихання - 17/хв., SpO_2 - 95% при кімнатному повітрі. При аускультації звучні вологі хрипи зліва. На оглядовій рентгенограмі консолідація у нижній частці лівої легені. Який з перерахованих препаратів буде найбільш доцільно призначити цьому пацієнту? A previously healthy 22-year-old boy came to the doctor with complaints of high temperature and cough with yellow sputum, which continued for 7 days. With physical examination temperature - 38.3°C, blood pressure - 130/70 mmHg, heart rate - 79/min, respiratory rate - 17/min, SpO_2 - 95% in room air wheezing on the left. Consolidation in the lower lobe of the left lung. Which of the following drugs would be most appropriate for this patient?

Азитроміцин Azithromycin

Левофлоксацин Levofloxacin

Ципрофлоксацин Ciprofloxacin

Гентаміцин Gentamicin

Триметоприм-сульфаметоксазол Trimethoprim-sulfamethoxazole

87 / 1500
У шестимісячної дитини різко підвищилася температура тіла до 39^oС, з'явилися блювання, відмова від їжі, короткочасні тоніко-клонічні судоми. Дитина в'яла, сонлива. Виявлені ригідність м'язів потилиці, симптоми Керніга, вибухання та напруження тім'ячка. Яке лабораторно-інструментальне обстеження треба провести для верифікації діагнозу? A six-month-old child's body temperature rose sharply to 39^oС, vomiting, refusal to eat, and short-term tonic-clonic convulsions appeared. The child was lethargic, sleepy . The stiffness of the occiput muscles, Kernig's symptoms, bulging and tension of the crown of the head were detected. What laboratory-instrumental examination should be carried out to verify the diagnosis?

МРТ головного мозку MRI brain

Люмбальну пункцію Lumbar puncture

Біохімічне дослідження сироватки крові Biochemical examination of blood serum

Нейросонографію Neurosonography

Рентгенографію черепа X-ray skull

88 / 1500
У трирічної дитини спостерігаються напади, що супроводжуються ціанозом, різким занепокоєнням, присіданням навпочіпки. Об'єктивно виявлено: деформація фаланг пальців у вигляді 'барабанних паличок', нігті у формі 'скелець годинника'. Межі серцевої тупості зсунуті вліво та вправо, у ІІ міжребер'ї біля лівого краю грудини визначається систолічне дрижання, вислухується грубий систолічний шум з р.max. у ІІ міжребер'ї, ІІ тон над основою серця ослаблений. Під час рентгенологічного дослідження виявлено: серце у вигляді 'дерев'яного черевика', легеневий малюнок виражений слабо. Який найімовірніший діагноз? A three-year-old child has seizures accompanied by cyanosis, sharp anxiety, squatting. Objectively revealed: deformation of the phalanges of the fingers in the form of 'drumsticks', nails in The borders of the heart are shifted to the left and to the right, a systolic murmur is heard near the left edge of the sternum, and the second sound over the base of the heart is weakened X-ray examination revealed: the heart in the form of a 'wooden shoe', the pulmonary pattern is weak. What is the most likely diagnosis?

Дефект міжпередсердної перетинки Atrial membrane defect

Тетрада Фалло Tetrad of Fallot

Первинний бактеріальний ендокардит Primary bacterial endocarditis

Дилатаційна кардіоміопатія Dilated cardiomyopathy

Дефект міжшлуночкової перетинки Defect of interventricular membrane

89 / 1500
Роділля, І вагітність, 38 тижнів, скаржиться на нерегулярний переймоподібний біль внизу живота та в попереку протягом 2 діб, вночі спала погано. При зовнішньому акушерському дослідженні - тонус матки підвищений, поздовжнє положення плода І позиція, передній вид, голівка плода рухома над входом в малий таз. Піхвове дослідження - шийка матки до 2,5 см, відхилена дозаду, розм’якшена нерівномірно, зовнішнє вічко відкрите до 0,5 см, внутрішнє вічко закрите. Ваш діагноз: Mother in labor, 1st pregnancy, 38 weeks, complains of irregular spasm-like pain in the lower abdomen and lower back for 2 days, slept poorly at night. During external obstetric examination - uterine tone elevated, longitudinal position of the fetus I position, front view, the fetal head is mobile above the entrance to the pelvis. Vaginal examination - the cervix is ​​up to 2.5 cm, deviated backward, unevenly softened, the external eye is open to 0.5 cm, the internal eye close. Your diagnosis:

Патологічний прелімінарний період Pathological preliminary period

Дискоординована пологова діяльність Discoordinated birth activity

Вторинна слабкість пологової діяльності Secondary weakness of labor activity

Передвісники пологів Forerunners of childbirth

Первинна слабкість пологової діяльності Primary weakness of labor activity

90 / 1500
У дитини 11 років з цукровим діабетом 1-го типу на фоні гострої респіраторної вірусної інфекції посилилася спрага, з'явилася нудота та блювота. У видихуваному повітрі різкий запах ацетону. Очі запалі. Дихання шумне, аритмічне, типу Кусмауля. ЧСС - 128/хв., глухість серцевих тонів. Діагностована кетоацидотична кома. Укажіть провідний патогенетичний механізм розвитку невідкладного стану у дитини: In an 11-year-old child with type 1 diabetes, against the background of an acute respiratory viral infection, thirst increased, nausea and vomiting appeared. The exhaled air has a sharp smell of acetone Eyes inflamed. Breathing is noisy, heart rate is 128/min. Diagnosed ketoacidotic coma:

Абсолютна інсулінова недостатність Absolute insulin deficiency

Гіперактивність центральної нервової системи Hyperactivity of the central nervous system

Активація симпато-адреналової системи Activation of sympatho-adrenal system

Надлишкова продукція інсуліну Excess insulin production

Недостатність функції кори наднирників Adrenal cortex insufficiency

91 / 1500
Жінка 32 років звернулась з приводу збільшення щитоподібної залози. Під час аварії на ЧАЕС перебувала у зоні підвищених радіоактивних опадів. Об'єктивно встановлено: щитоподібна залоза збільшена до ІІ ст., у ній пальпується щільний вузол, малорухомий, неболючий. Підщелепні лімфатичні вузли збільшені, неболючі. Під час УЗД у щитоподібній залозі виявлено гіпоехогенний утвір без чітких меж з кальцинатами. АТ - 120/70 мм рт. ст. Пульс - 78/хв. На основі приведених даних у хворої можна запідозрити: A 32-year-old woman applied for an enlarged thyroid gland. During the accident at the Chernobyl nuclear power plant, she was in the zone of increased radioactive fallout. It was objectively established: the thyroid gland was enlarged to the II century ., a dense, non-moving node is palpable. During ultrasound, a hypoechoic mass with calcifications was detected. Pulse - 78 mmHg. Based on the given data, the patient may be suspected of:

Аденому щитоподібної залози Adenoma of the thyroid gland

Рак щитоподібної залози Thyroid cancer

Підгострий тиреоїдит Subacute thyroiditis

Дифузний нетоксичний зоб Diffuse non-toxic goiter

Дифузний токсичний зоб Diffuse toxic goiter

92 / 1500
Пацієнтка віком 22 роки скаржиться на ниючий біль у правій здухвинній ділянці, що турбує протягом тижня, нудоту зранку, зміну відчуття смаку. З анамнезу відомо: затримка менструації - 3 тижні. Об'єктивно спостерігається: АТ - 110/70 мм рт. ст., Рs - 78/хв, температура тіла - 37,0^oС. Під час бімануального дослідження виявлено: матка дещо збільшена, пом'якшена, рухома, неболюча. Додатки: справа пальпується болюче утворення 3·4 см, щільно-еластичної консистенції, помірно рухоме. Який попередній діагноз? A 22-year-old patient complains of aching pain in the right pubic region, which has been bothering her for a week, morning sickness, a change in the sense of taste. From the anamnesis, it is known: delayed menstruation - 3 Objectively observed: blood pressure - 110/70 mm Hg, body temperature - 37.0 °C. During bimanual examination, the uterus was slightly enlarged, mobile, painless Appendices: a painful mass of 3·4 cm is palpated on the right, of a dense and elastic consistency. What is the preliminary diagnosis?

Трубна вагітність, що перервалась Aborted tubal pregnancy

Гострий апендицит Acute appendicitis

Прогресуюча трубна вагітність Progressing tubal pregnancy

Маткова вагітність Uterine pregnancy

Кіста правого яєчника Right ovarian cyst

93 / 1500
Унаслідок неправильного зберігання проросла або позеленіла картопля має гіркуватий смак. Яка отруйна речовина, що міститься в такій картоплі, може спричинити харчове отруєння? Due to improper storage, sprouted or green potatoes have a bitter taste. What poisonous substance contained in such potatoes can cause food poisoning?

Мускаридин Muscaridine

Соланін Solanine

Мускарин Muscarine

Гельвелова кислота Helvelic acid

Фазин Fazin

94 / 1500
Чоловік віком 34 роки захворів 3 дні тому після переохолодження. Скаржиться на підвищення температури до 39,2^oС, виражену загальну слабкість, пітливість, кашель. Кашель був спочатку сухий, протягом останньої доби з'явилася невелика кількість «іржавого» мокротиння. Під час об'єктивного обстеження спостерігається: герпес на губах, перкуторно над легенями справа в нижніх відділах спостерігається притуплений звук. Аускультативно виявлені бронхіальне дихання, тахікардія. В органах черевної порожнини змін не виявлено. Який діагноз є найімовірнішим? A 34-year-old man fell ill 3 days ago after hypothermia. He complains of an increase in temperature to 39.2^oС, severe general weakness, sweating, cough. The cough was initially dry, during the last day a small amount of 'rusty' sputum appeared. During the objective examination, a dull sound was observed over the lungs on the right. Bronchial breathing, tachycardia were detected in the abdominal organs not detected. What is the most likely diagnosis?

Абсцес легені Lung abscess

Крупозна пневмонія Croup pneumonia

Ексудативний плеврит Exudative pleurisy

Негоспітальна вогнищева пневмонія Community-acquired focal pneumonia

Нозокоміальна пневмонія Nosocomial pneumonia

95 / 1500
Під час операції у жінки 67 років із приводу гострого деструктивного холециститу визначена наявність гнійного холангіта. Під час ревізії холедоха не знайдені конкременти та ознаки стенозу дуоденального соска. Виконано холецистектомію. Як слід закінчити операцію? During the operation of a 67-year-old woman for acute destructive cholecystitis, the presence of purulent cholangitis was determined. No calculi or signs of stenosis of the duodenal nipple were found during the choledochal revision. A cholecystectomy was performed. How should the operation be completed?

Виконати трансдуоденальну холедоходуоденостомію Perform transduodenal choledochoduodenostomy

Виконати зовнішнє дренування холедоха Perform external choledochal drainage

Накласти гепатікоєюностомію Apply hepaticojejunostomy

Накласти супрадуоденальний холедоходуоденоанастомоз Apply supraduodenal choledochoduodenal anastomosis

Виконати холедохоеюностомію Perform choledochojejunostomy

96 / 1500
Пацієнт 27 років близько року спостерігає втому, пітливість, важкість у лівому підребер'ї, особливо після прийому їжі. Об'єктивно спостерігається: збільшена селезінка, печінка. В крові виявлено: ермтроцити - 3,2·10^12/л, Hb - 100 г/л, КП - 0,87, лейкоцити - 100·10^9/л, базофіли - 7%, еозинофіли - 5%, моноцити - 15%, юні - 16%, паличкоядерні - 10%, сегментоядерні - 45%, лімфоцити - 2%, моноцити - 0%, ретикулоцити - 0,3%, тромбоцити - 400·10^9/л, ШOЕ - 25 мм/год. Поставте діагноз. A 27-year-old patient has been experiencing fatigue, sweating, and heaviness in the left hypochondrium for about a year, especially after eating. Objectively observed: enlarged spleen, liver. In blood revealed: erythrocytes - 3.2·10^12/l, Hb - 100 g/l, CP - 0.87, leukocytes - 100·10^9/l, basophils - 7%, eosinophils - 5%, monocytes - 15%, young - 16%, rod-nuclear - 10%, segmentonuclear - 45%, lymphocytes - 2%, monocytes - 0%, reticulocytes - 0.3%, platelets - 400·10^9/l, SOE - 25 mm/ Make a diagnosis.

Еритремія Erythremia

Цироз печінки Liver cirrhosis

Гострий лейкоз Acute leukemia

Хронічний лімфолейкоз Chronic lymphocytic leukemia

Хронічний мієлолейкоз Chronic myelogenous leukemia

97 / 1500
Чоловік 50-ти років звернувся до лікаря зі скаргами на біль в стегні після падіння на сходах. Піднятись та ходити не може. Права нога коротша за ліву на 5 см, ступня ротована назовні. Пульс на правій тильній артерії стопи задовільний. Стегно збільшене в об'ємі, деформоване, в середній третині болюча пальпація. Виявляється патологічна рухомість, крепітація. Який діагноз є найбільш імовірним? A 50-year-old man went to the doctor complaining of hip pain after falling down the stairs. He can't get up and walk. The right leg is 5 cm shorter than the left , the foot is rotated outwards. The pulse on the right back artery is satisfactory. The thigh is enlarged, painful palpation. What is the most likely diagnosis?

Закритий перелом кісток миски Closed pelvic fracture

Перелом діафіза стегнової кістки Fracture of the diaphysis of the femur

Вивих правого стегна Dislocation of right hip

Забій сідничного нерва Contusion of sciatic nerve

Хибний суглоб False joint

98 / 1500
Дайте оцінку фізичного розвитку десятирічної дівчинки за шкалою регресії, якщо показники ваги тіла та окружності грудної клітки знаходяться у межах pm 1 сигми. Give an estimate of the physical development of a ten-year-old girl on a regression scale, if the indicators of body weight and chest circumference are within pm 1 sigma.

Високий High

Дисгармонійний Disharmonious

Низький Low

Гармонійний Harmonic

Середній Average

99 / 1500
У комп'ютерному залі науково-дослідного сектору політехнічного інституту робочі місця лаборантів-операторів розташовані біля передніх панелей моніторів. Упродовж усього робочого дня оператори перебувають під впливом електромагнітних хвиль надвисокої частоти. Вплив електромагнітних хвиль великої інтенсивності у вказаному діапазоні частот є особливо небезпечним для: In the computer hall of the research sector of the polytechnic institute, the workplaces of laboratory assistants-operators are located near the front panels of the monitors. Throughout the working day, the operators are under the influence of ultra-high electromagnetic waves frequencies. The influence of electromagnetic waves of high intensity in the indicated frequency range is especially dangerous for:

Слухового аналізатора Hearing Analyzer

Зорового аналізатора Visual Analyzer

Тактильної чутливості Tactile sensitivity

Сенсорної чутливості Touch sensitivity

Больової чутливості Pain sensitivity

100 / 1500
Встановлено, що на 100 пологів у жінок, які мають фактори ризику, було 30 передчасних пологів, а у жінок, які не мають факторів ризику, було 5 передчасних пологів. Який метод статистичної обробки даних буде оптимальним для використання лікарем, щоб оцінити вірогідність розбіжностей у порівнюваних групах? For every 100 births, women with risk factors had 30 premature births, while women without risk factors had 5 premature births . What statistical data processing method would be optimal for a physician to assess the likelihood of differences in the compared groups?

Розрахунок середніх величин Calculation of average values

Кореляційний аналіз Correlation Analysis

Розрахунок відносних величин Calculation of relative values

Розрахунок критерію Стьюдента Student's criterion calculation

Метод стандартизації Standardization method

101 / 1500
Жінка 28 років звернулась до гінеколога зі скаргами на неплідність протягом 3 років. Менструальна функція не порушена. В анамнезі вказано: один штучний аборт, хронічний сальпінгоофорит. Контрацептивні засоби не використовує. Спермограмма чоловіка без патології. З якого методу слід почати обстеження для виявлення причин безпліддя? A 28-year-old woman turned to a gynecologist with complaints of infertility for 3 years. Menstrual function is not disturbed. The anamnesis indicates: one artificial abortion, chronic salpingo-oophoritis. Contraceptives are not uses. A man's spermogram without pathology. What method should be used to identify the causes of infertility?

Гормональне дослідження Hormonal study

Гістероскопія Hysteroscopy

Діагностичне вишкрібання порожнини матки Diagnostic scraping of the uterine cavity

Гістеросальпінгографія Hysterosalpingography

Лапароскопія Laparoscopy

102 / 1500
Пацієнтка віком 44 роки скаржиться на загальну слабкість, сонливість, серцебиття, сухість шкіри, зниження працездатності. Об'єктивно спостерігається: шкіра звичайного кольору, пульс - 72/хв, АТ - 125/80 мм рт. ст., щитоподібна залоза дифузно збільшена до ІІ ступеня, щільна, рухлива, болюча. Під час УЗД виявлено: зниження ехогенності, неоднорідність ехоструктури, потовщення капсули залози. У аналізі крові спостерігається: рівень Т4 - знижений, ТТГ - підвищений, антитіла до тиреопероксидази та антимікросомальні антитіла значно підвищені. Який найімовірніший діагноз? A 44-year-old female patient complains of general weakness, drowsiness, palpitations, dry skin, reduced work capacity. Objectively observed: normal-colored skin, pulse - 72/min , blood pressure - 125/80 mmHg, the thyroid gland is diffusely enlarged to the II degree, during ultrasound, it is found: a decrease in echogenicity, a thickening of the gland capsule. In the blood analysis, the level of T4 is reduced , TSH is increased, antibodies to thyroperoxidase and antimicrosomal antibodies are significantly increased. What is the most likely diagnosis?

Аутоімунний тиреоїдит Autoimmune thyroiditis

Рак щитоподібної залози Thyroid cancer

Підгострий тиреоїдит Subacute thyroiditis

Дифузний токсичний зоб ІІ ступеня Diffuse toxic goiter II degree

Дифузний еутиреоїдний зоб ІІ ступеня Diffuse euthyroid goiter II degree

103 / 1500
Дівчина 22-х років прийшла до лікаря на планове обстеження. Вона викурює 1 пач-ку сигарет у день протягом 5-ти років. Має одного постійного статевого партнера, користуються презервативами. Дідусь по татовій лінії помер від серцевого нападу у 60 років. При фізикальному обстеженні пульс - 78/хв., частота дихання - 14/хв., артеріальний тиск - 110/70 мм рт.ст. При аус-культації серця вислуховується голосистолічний шум у ІІ міжребер'ї зліва від грудини. Яка рекомендація лікаря цій пацієнтці буде найбільш доречною? A 22-year-old girl came to the doctor for a routine examination. She has smoked 1 pack of cigarettes a day for 5 years. She has one permanent sexual partner, paternal grandfather died of a heart attack at the age of 60. Heart rate is 78/min., blood pressure is 110/70 mm Hg Holosystolic murmur in the 2nd intercostal space to the left of the sternum. What would be the most appropriate recommendation for this patient?

Скринінг гіперліпідемії Screening for hyperlipidemia

Кольпоскопія з біопсією Colposcopy with biopsy

Навчитися самообстеженню молочних залоз Learn breast self-examination

Пройти ПАП-тест Pass the Pap test

Пройти ПАП-тест та здати аналіз на ВПЛ Pass a Pap test and pass an analysis for HPV

104 / 1500
Під час профілактичного огляду населення міста у 25% людей виявлено ознаки флюорозу: плямиста емаль зубів, у частини обстежених - генералізований остеосклероз із кальцифікацією міжхребетних зв'язок. Яка найімовірніша причина виникнення флюорозу? During a preventive examination of the city's population, 25% of people were found to have signs of fluorosis: mottled tooth enamel, some of the examined had generalized osteosclerosis with calcification of the intervertebral ligaments. What is the most likely the cause of fluorosis?

Надмірний вміст фтору в рослинних продуктах Excessive content of fluorine in vegetable products

Недостатній вміст фтору у грунті та воді Insufficient fluoride content in soil and water

Надмірний вміст фтору у грунті та воді Excessive fluoride content in soil and water

Недостатній вміст фтору в тваринних продуктах Insufficient fluoride content in animal products

Недостатнє надходження фтору в організм із чаєм Insufficient intake of fluoride in the body with tea

105 / 1500
Восьмирічний пацієнт спостерігається через затримку зросту. Народився в асфіксії з масою тіла 2800 г. В школі навчається добре. Батьки середнього зросту. Об'єктивно спостерігається: зріст - 107 см, маса тіла - 23 кг, недорозвинення лицьового скелета на кшталт лялькового обличчя. Волосся тонке, шкіра суха з іктеричним відтінком. Підшкірно-жирова клітковина розвинута добре на шиї, грудях, животі. М'язи розвинуті недостатньо. Який найімовірніший діагноз? An eight-year-old patient is observed due to growth retardation. He was born asphyxiated with a body weight of 2800 g. He studies well at school. Parents are of average height. Objectively observed: height - 107 cm, body weight - 23 kg, underdevelopment of the facial skeleton like a doll's face. The skin is dry with a icteric shade. The subcutaneous fat is well developed on the neck, chest, abdomen. What is the most likely diagnosis?

Хондродистрофія Chondrodystrophy

Краніофарінгіома Craniopharyngioma

Синдром Дауна Down Syndrome

Синдром Фанконі Fanconi Syndrome

Гіпофізарний нанізм Pituitary dwarfism

106 / 1500
Чоловік віком 38 років скаржиться на переймоподібні болі в животі, нудоту, блювоту, вздуття живота, невідходження газів, відсутність стільця. Із анамнезу відомо, що нещодавно пацієнта проперовано з приводу апендициту. Пальпаторно живіт болісний у всіх відділах, напружений. Визначається шум плескоту, нечіткі симптоми подразнення очеревини, гіперперистальтика. Який попередній діагноз? A 38-year-old man complains of spasm-like abdominal pain, nausea, vomiting, abdominal distension, non-emission of gases, absence of stool. It is known from the anamnesis that the patient was recently operated on of appendicitis. The abdomen is painful on palpation, tense. There is a rumbling sound, vague symptoms of peritoneum irritation. What is the preliminary diagnosis?

Дивертикулярний коліт Diverticular Colitis

Виразкова хвороба шлунка Gastric ulcer disease

Розлитий перитоніт Diffuse peritonitis

Злукова кишкова непрохідність Intestinal obstruction

Рак товстого кишечника Colon cancer

107 / 1500
Роділля віком 29 років, пологи треті, у терміні 40 тижнів народила хлопчика масою 1900 г, зростом 48 см, із оцінкою за шкалою Апгар 7-8 балів. У ІІІ періоді пологів народилася плацента розмірами 17х16х1,5см, масою 340 г, із множинними петрифікатами, оболонки зеленуватого кольору. Яка патологія найімовірніше спричинила затримку внутрішньоутробного росту плода? A 29-year-old woman in labor, her third delivery, gave birth to a boy weighing 1900 g, height 48 cm, with an Apgar score of 7-8 points at 40 weeks. In the third period of labor, a placenta was born measuring 17x16x1.5 cm, weighing 340 g, with multiple petrifications, a greenish membrane. What pathology most likely caused intrauterine growth retardation?

Передчасне відшарування нормально розташованої плаценти Premature detachment of a normally located placenta

Гіпотрофія плода І стуненя Fetal hypotrophy and moaning

Плацентарна недостатність Placental insufficiency

Внутрішньоутробне інфікування плода Intrauterine fetal infection

Передчасні пологи Premature birth

108 / 1500
Чоловік 56-ти років, звернувся до лікаря з носовою кровотечею, що розпочалася з лівої ніздрі 30 хвилин тому. Будь-які травми носа пацієнт заперечує. В анамнезі: частих носових кровотеч немає, подібний епізод вперше. Хворіє на фібриляцію передсердь (приймає варфарин) та артеріальну гіпертензію (лікується гідрохлортіазидом, атенололом). Температура тіла - 37,2^oC, пульс - 86/хв., артеріальний тиск - 120/70 мм рт.ст. Перед тим, як визначити необхідність тампонади носа, який першочерговий крок лікаря буде найбільш доречним? A 56-year-old man consulted a doctor with a nosebleed that started from the left nostril 30 minutes ago. The patient denies any injuries to the nose. In the anamnesis: there are no frequent nosebleeds, this is the first time he has atrial fibrillation (he takes warfarin) and hypertension (he is treated with hydrochlorothiazide, atenolol). Body temperature - 37.2°C, pulse - 86/min. Before determining the need for nasal tamponade, what is the most appropriate first step of the doctor?

Оксиметазоліну інтраназально, попросити хворого затиснути крило носа та нахилитися вперед Oxymetazoline intranasally, ask the patient to pinch the wing of the nose and lean forward

Прикласти холодний компрес на перенісся Apply a cold compress to the bridge of the nose

- -

Призначити каптоприл перорально Prescribe oral captopril

Ввести вітамін K внутрішньом'язово Inject vitamin K intramuscularly

109 / 1500
Жінка 27-ми років на 8-му тижні вагітності скаржиться лікарю, що за останні 8 днів вона відмічає тривалу нудоту та блювання майже після усіх прийомів їжі. За останній тиждень пацієнтка втратила 3 кг ваги. Зараз при рості 160 см жінка важить 46 кг. Пульс - 100/хв., артеріальний тиск - 90/50 мм рт.ст. При огляді відмічаються сухість слизових оболонок, зниження тургору шкіри та астенічна статура. Гінекологічний огляд виявив розмір матки, що відповідає 8-му тижню вагітності, без патологічних змін. На УЗД виявлена вагітність одним плодом. Концентрація гемоглобіну - 150 г/л. У загальному аналізі сечі виявлені кетонові тіла (+++). Що із перерахованого є найбільш доречним наступним кроком у веденні пацієнтки? A 27-year-old woman in the 8th week of pregnancy complains to the doctor that for the past 8 days she has been experiencing prolonged nausea and vomiting after almost every meal. In the last The patient has lost 3 kg of weight. Now she is 160 cm tall. The pulse is 100/min. During the examination, the mucous membranes are dry and the body is asthenic the examination revealed the size of the uterus, which corresponds to the 8th week of pregnancy, and the pregnancy was detected by one fetus. In the general analysis of the urine, ketone bodies were found (+++). appropriate next step in the management of the patient?

Внутрішньовенне введення beta-адрено-блокаторів та парентеральне харчування Intravenous administration of beta-blockers and parenteral nutrition

Внутрішньовенна інфузійна терапія та призначення антиеметиків Intravenous infusion therapy and prescription of antiemetics

Пероральний прийом антиеметиків та антихолінергічних препаратів Oral intake of antiemetics and anticholinergic drugs

Ендоскопічне дослідження та промивання шлунка Endoscopy and gastric lavage

Постільний режим та часте годування малими порціями Bed rest and frequent feeding in small portions

110 / 1500
Жінку віком 24 років шпиталізовано зі скаргами на блювання за типом <<кавової гущі>>, запаморочення, загальну слабкість. Під час об'єктивного обстеження встановлено: пацієнтка у свідомості, шкіра бліда, суха, над легенями аускультативно - жорстке дихання. ЧД - 20/хв, АТ - 100/80 мм рт. ст, пульс - 100/хв. Живіт м'який, безболісний. Перитонеальні симптоми негативні. Стілець рідкий, чорний. Діурез збережений. У клінічному аналізі крові виявлено: Нb - 95 г/л, еритроцити - 3,1 г/л, лейкоцити - 9,8 г/л. Який показник шокового індекса Альговера у цієї пацієнтки? A 24-year-old woman was hospitalized with complaints of vomiting of the type <>, dizziness, general weakness. During the objective examination, it was found: the patient in consciousness, skin is dry, respiratory rate is 20/min, pulse is 100/min. Stool is liquid. Diuresis is preserved. Clinical blood analysis revealed: Hb - 3.1 g/l, leukocytes - 9.8 g/l. What is this patient's shock index?

2 2

1,25 1.25

0,85 0.85

1 1

1,5 1.5

111 / 1500
Жінка 46 років, яка рік тому мала резекцію щитоподібної залози з приводу багатовузлового зобу, скаржиться на загальну слабкість, сонливість, постійне відчуття втоми, зниження працездатності, закрепи, набряк обличчя та кінцівок. Об'єктивно спостерігається: температура тіла - 36^oC. Шкірні покриви сухі, зморшкуваті, почало випадати волосся. Аменорея. Яке захворювання виникло у пацієнтки? A 46-year-old woman, who a year ago had a resection of the thyroid gland due to a multinodular goiter, complains of general weakness, drowsiness, a constant feeling of fatigue, reduced work capacity, constipation, edema face and limbs. Body temperature is 36°C. The skin is wrinkled. What disease did the patient have?

Гіпопаратиреоз Hypoparathyroidism

Тиреотоксикоз Thyrotoxicosis

Субклінічний гіпотиреоз Subclinical hypothyroidism

Хронічний тиреоїдит Chronic thyroiditis

Первинний гіпотиреоз Primary hypothyroidism

112 / 1500
60-річна жінка доставлена у відділення невідкладної допомоги зі скаргами на раптовий інтенсивний головний біль та нудоту. Останні півроку спостерігалася легка диплопія. Протягом багатьох років хворіє на артеріальну гіпертензію та цукровий діа-бет II типу. Артеріальний тиск - 160/90 мм рт.ст., частота серцевих скорочень - 82/хв. Фізикальне обстеження виявило правосторонній птоз, легку анізокорію та ригідність потиличних м'язів. Атаксія не спостерігається. Який діагноз є найбільш імовірним? A 60-year-old woman was brought to the emergency department with complaints of sudden intense headache and nausea. Mild diplopia has been observed for the past six months. She has been suffering from arterial hypertension for many years and Type II diabetes. Blood pressure - 160/90 mm Hg. Physical examination revealed right-sided ptosis and stiffness of the occipital muscles. What is the most common diagnosis probable?

Метаболічна енцефалопатія Metabolic encephalopathy

Лакунарний інсульт Lacunar stroke

Субарахноїдальний крововилив Subarachnoid hemorrhage

Гліома стовбура головного мозку Brain stem glioma

Невралгія трійчастого нерва Trigeminal neuralgia

113 / 1500
Чоловік 28 років після піднімання ваги відчув сильний біль у попереку, який іррадіював у праву ногу. Звернувся до лікаря. Після огляду лікар поставив діагноз: гострий дискогенний попереково-крижовий радикуліт. Яке обстеження треба пройти хворому для підтвердження діагнозу? A 28-year-old man, after lifting weights, felt severe pain in the lower back, which radiated to the right leg. He consulted a doctor. After an examination, the doctor made a diagnosis: acute discogenic lumbosacral sciatica. What examination should the patient undergo to confirm the diagnosis?

Люмбальна пункція Lumbar puncture

Електроміографія м'язів ніг Electromyography of leg muscles

Рентгенографія нирок Kidney X-ray

Загальний аналіз сечі General urinalysis

МРТ поперекового відділу хребта MRI of the lumbar spine

114 / 1500
Працівник заводу з виробницва фарб скаржиться на неприємний смак у роті, переймоподібні болі в животі, закрепи, які не знімаються послаблюючими засобами. Об'єктивно встановлено: шкіра обличчя блідо-сіруватого кольору, по краю ясен біля передніх зубів - смужка сіровато-бузкового кольору. Під час лабораторного дослідження виявлено: у крові анемію, ретикулоцитоз, базофільну зернистість еритроцитів, педвищений вміст білірубіну; у сечі - підвищений вміст порфірину. Яке захворювання у робітника? An employee of a paint production plant complains of an unpleasant taste in the mouth, spasm-like pains in the stomach, constipation that is not removed by laxatives. Objectively established: the skin of the face is pale - grayish color, along the edge of the gums - a grayish-lilac color. During the laboratory examination, anemia, reticulocytosis, increased bilirubin content in the urine - increased porphyrin content were found. ;

Хронічне отруєння свинцем Chronic lead poisoning

Хронічне отруєння марганцем Chronic manganese poisoning

Хронічне отруєння бензолом Chronic benzene poisoning

Хронічне отруєння чотирихлористим вуглецем Chronic carbon tetrachloride poisoning

Хронічне отруєння ртуттю Chronic mercury poisoning

115 / 1500
Пацієнтка 20-ти років проходить лікування з приводу анемії (гемоглобін - 72 г/л). Півтора роки тому після мимовільного викидня у терміні 16 тижнів та крововтрати, відмічає зниження пам'яті, втомлюваність, втрату апетиту, сухість шкіри, ламкість нігтів, набряклість, порушення менструальної функції. Об'єктивно: артеріальний тиск - 80/55 мм рт.ст., пульс - 54/хв., зріст - 168 см, вага - 48 кг, гіпоплазія статевих органів. Призначення якого з перерахованих препаратів буде найбільш доречним цій пацієнтці? A 20-year-old female patient is being treated for anemia (hemoglobin - 72 g/l). One and a half years ago, after an involuntary miscarriage at 16 weeks and blood loss, she notes memory loss, fatigue, loss of appetite, dry skin, brittle nails, swelling, menstrual dysfunction Objectively: blood pressure - 80/55 mm Hg, pulse - 54/min, height - 168 cm, weight - 48 kg, genital hypoplasia. Which of the listed drugs would be most appropriate for this patient?

Метотрексат Methotrexate

Гідроксихлорохін Hydroxychloroquine

Гідрокортизон Hydrocortisone

Імуноглобулін людини нормальний Human immunoglobulin is normal

Інфліксимаб Infliximab

116 / 1500
Чоловіка 63 років госпіталізовано зі скаргами на біль у поперековій ділянці ліворуч, погіршення апетиту, слабкість, періодичну появу крові в сечі протягом місяця. Шкіра бліда. Анемія: еритроцити - 3,1·10^12/л, Нb - 101 г/л, ШОЕ - 37 мм/год., протеїнурія - 0,37 г/л, гематурія на все поле зору, креатінемія - 0,270 ммоль/л. Яке діагностичне припущення? A 63-year-old man was hospitalized with complaints of pain in the lumbar region on the left, loss of appetite, weakness, periodic appearance of blood in the urine for a month. The skin is pale. Anemia: erythrocytes - 3.1·10^12/l, Hb - 101 g/l, ESR - 37 mm/h, proteinuria - 0.37 g/l, hematuria in the entire field of vision, creatininemia - 0.270 mmol/l. What is the diagnostic assumption ?

Амілоїдоз нирок Kidney amyloidosis

Інфекційна токсична нефропатія Infectious toxic nephropathy

Гострий пієлонефрит Acute pyelonephritis

Гострий гломерулонефрит Acute glomerulonephritis

Новоутворення нирки Kidney Neoplasm

117 / 1500
Чоловік 40 років протягом 10 років страждає на хронічний гломерулонефрит. Скаржиться на блювоту, судоми м'язів нижніх кінцівок. АТ - 180/120 мм рт. ст., креатинін у сироватці крові - 770 мкмоль/л, швидкість клубочкової фільтрації - 5 мл/хв. Протягом останніх двох днів діурез знизився до 400 мл на добу. Яка лікувальна тактика показана для цього хворого? A 40-year-old man has been suffering from chronic glomerulonephritis for 10 years. He complains of vomiting, muscle spasms of the lower limbs. BP - 180/120 mm Hg, creatinine in blood serum - 770 μmol/l, glomerular filtration rate - 5 ml/min. During the last two days, diuresis has decreased to 400 ml per day. What therapeutic tactics are indicated for this patient?

Гемодіаліз Hemodialysis

Плазмаферез Plasmapheresis

Сорбенти Sorbents

Гемофільтрація Hemofiltration

Гемосорбція Hemosorption

118 / 1500
Пацієнта віком 55 років шпиталізовано у непритомному стані до відділення реанімації. Об'єктивно спостерігається: стан важкий, зіниці розширені, реагують на світло, дихання шумне, ЧД - 10/ хв, пульс на променевих і сонних артеріях не визначається, АТ - 40/20 мм рт. ст. На ЕКГ виявлено: тріпотіння шлуночків з ЧСС - 210/хв. Які реанімаційні заходи потрібно провести насамперед? A 55-year-old patient was hospitalized in an unconscious state to the intensive care unit. Objectively observed: the condition is severe, the pupils are dilated, react to light, breathing is noisy, BH - 10 / min, the pulse on the radial and carotid arteries is not determined, BP - 40/20 mmHg. Ventricular flutter with heart rate - 210/min. What resuscitation measures should be carried out first?

Внутрішньовенне введення норадреналіну Intravenous administration of norepinephrine

Зовнішній масаж серця External heart massage

Внутрішньовенне введення лідокаїну Intravenous administration of lidocaine

Електроімпульсну терапію Electropulse therapy

Внутрішньовенне введення адреналіну з атропіном Intravenous administration of epinephrine with atropine

119 / 1500
Хворий 10-ти років надійшов до клініки зі скаргами на експіраторну задишку, частота дихання - 30/хв. Погіршення стану пов'язує з зміною метеорологічних умов. Хлопчик стоїть на диспансерному обліку впродовж 4-х років з діагнозом бронхіальна астма, 3 ступінь, персистуюча. З чого слід розпочати надання невідкладної допомоги? A 10-year-old patient came to the clinic with complaints of expiratory shortness of breath, respiratory rate - 30/min. The deterioration of the condition is associated with a change in meteorological conditions. The boy is standing at the dispensary for 4 years with a diagnosis of bronchial asthma, grade 3, persistent. Where should emergency care be started?

beta_2-агоністи короткої дії short-acting beta_2 agonists

Адреналін Adrenaline

Дексаметазон Dexamethasone

Кларитин Claritin

Еуфілін Euphilin

120 / 1500
Другі пологи у повторнороділлі в терміні 36-37 тижнів. Навколоплідні води вилились 8 годин тому, пологова діяльність триває 4 години, регулярна, перейми через 3-4 хвилини по 35 секунд. Передлегла голівка притиснута до входу в малий таз. Роділля поскаржилась на раптовий різкий біль в животі. Пульс - 100/хв., АТ- 110/70-100/70 мм рт.ст. Матка напружена, не розслабляється поза переймами. Серцебиття плода приглушене - 100/хв. Підтікають навколоплідні води, забарвлені кров'ю. Вкажіть найбільш імовірний діагноз: Second birth in repeated labor at 36-37 weeks. Amniotic fluid spilled 8 hours ago, labor lasts 4 hours, regular, takes after 3-4 minutes 35 seconds. The head is pressed against the entrance to the pelvis. The woman in labor complained of sudden pain in the abdomen. Pulse - 110/70/70 mm Hg. The uterus does not relax. The fetal heartbeat is muffled - 100/min. Amniotic fluid is flowing, stained with blood. Specify the most likely diagnosis:

Розрив судин пуповини Rupture of umbilical cord vessels

Розрив матки Rupture of uterus

Передчасне відшарування нормально розташованої плаценти Premature detachment of a normally located placenta

Неповне передлежання плаценти Incomplete placenta previa

Розрив шийки матки Rupture of the cervix

121 / 1500
Вагітна віком 25 років направлена до стаціонарного відділення для лікування. В анамнезі - 2 самовільних викидня. Під час обстеження виявлено вагітність 14-15 тижнів. Вагінально спостерігається: шийка матки вкорочена, зовнішнє вічко пропускає кінчик пальця. Встановлено діагноз: істміко-цервікальна недостатність. Яка тактика лікаря у цьому разі? A 25-year-old pregnant woman was sent to the inpatient department for treatment. She had 2 spontaneous miscarriages in her history. During the examination, a pregnancy of 14-15 weeks was detected. Vaginal observation: cervix shortened, the outer eye passes the tip of the finger. The diagnosis is established: isthmic-cervical insufficiency. What is the doctor's tactic in this case?

Провести гормональне лікування Conduct hormonal treatment

Не чекати виникнення самовільного аборту, увести утеротонічні препарати Do not wait for spontaneous abortion to occur, introduce uterotonic drugs

Накласти циркулярний шов на шийку матки Put a circular suture on the cervix

Ліжковий режим із призначенням седативних препаратів Bed bed with the appointment of sedatives

Провести амніоцентез та перервати вагітність Perform amniocentesis and terminate pregnancy

122 / 1500
У хворої 65-ти рокiв при пальпацiї живота в ділянцi пупка i вище пухлина розмiром 13х8 см, помiрно болюча, не змiщується, пульсує. Аускультативно: систолічний шум. Який найбільш імовірний дiагноз? A 65-year-old patient has a 13x8 cm tumor on palpation of the abdomen in the area of ​​the navel and above, moderately painful, does not move, pulsates. Auscultation: systolic murmur. What the most likely diagnosis?

Недостатність двостулкового клапана Bicuspid valve failure

Артеріовенозна аневризма Arteriovenous aneurysm

Недостатність тристулкового клапана Tricuspid valve failure

Аневризма черевної аорти Abdominal aortic aneurysm

Пухлина шлунка Stomach tumor

123 / 1500
Водопостачання районного центру здійснюється з міжпластового водоносного горизонту з високим вмістом солей кальцію та магнію. Вкажіть найоптимальніший метод обробки води перед подачею в розподільну мережу. The water supply of the district center is carried out from an interlayer aquifer with a high content of calcium and magnesium salts. Specify the most optimal method of water treatment before feeding into the distribution network.

Знезараження Disinfection

Опріснення Desalination

Озонування Ozonation

Пом'якшення Mitigating

Відстоювання Advocacy

124 / 1500
Жінка 29-ти рокiв звернулася до лікаря зі скаргами на остуду, бiль у горлi, утруднене ковтання та підвищення температури до 38^oC. При фізикальному обстеженні лiкарем на обох мигдаликах виявлено нальоти у виглядi сiрих щiльних плiвок, збiльшення i болючiсть пiдщелепних лiмфатичних вузлiв. Використання якого діагностичного методу буде найбільш доречним наступним кроком? A 29-year-old woman consulted a doctor with complaints of a cold, sore throat, difficulty swallowing, and an increase in temperature up to 38°C. During a physical examination by a doctor on both plaques in the form of gray dense films, enlargement and tenderness of the submandibular lymph nodes were found. The use of which diagnostic method would be the most appropriate next step?

Аналіз на гетерофільні антитіла Heterophilic antibody analysis

Рентгенографія м'яких тканин шиї X-ray of soft tissues of the neck

Експрес-тест для виявлення антигенів стрептококу групи А Express test for detection of group A streptococcus antigens

Пункцiя підщелепних лiмфовузлiв Puncture of submandibular lymph nodes

Бактерiологiчне дослідження мазку із порожнини носа та горла Bacteriological examination of a smear from the cavity of the nose and throat

125 / 1500
У хворого 26-ти років має місце зовнішня кровотеча з рваної рани гомілки. З рани постійним потоком витікає кров темно-червоного кольору, загальний обся-г крововтрати приблизно 400 мл. Який метод зупинки кровотечі потрібно використати на догоспітальному етапі? A 26-year-old patient has external bleeding from a torn leg wound. Dark red blood is flowing out of the wound in a constant stream, the total amount of blood loss is approximately 400 Jr. What method of stopping bleeding should be used at the pre-hospital stage?

Накладання джгута дистальніше місця кровотечі Applying a tourniquet distal to the bleeding site

Стискальна пов'язка на рану Wound compression bandage

Накладання артеріального джгута на стегно Applying an arterial tourniquet to the thigh

Накладання затискача на сосуд, що кровить Clamping a bleeding vessel

Пальцеве притиснення стегнової артерії Finger pressure of the femoral artery

126 / 1500
Після перенесеного ішемічного інсу-льту, зумовленого кардіоемболією, хворому з фібриляцією передсердь у якості засобу вторинної профілактики призначають: After an ischemic stroke caused by cardioembolism, a patient with atrial fibrillation is prescribed as a means of secondary prevention:

Аспірин чи клопідогрель Aspirin or clopidogrel

Ноотропні препарати Nootropic drugs

beta-адреноблокатори beta blockers

Оральні антикоагулянти Oral anticoagulants

Антагоністи кальцію Calcium antagonists

127 / 1500
Жінка 45 років звернулась до лікарні за 12 діб після початку захворювання зі скаргами на біль та припухлість І пальця лівої кисті. Під час обстеження виявлено підвищення температури тіла до 38,9^oС. Нігтьова фаланга І пальця колбоподібно потовщена, синьо-багряного кольору. Під нігтьовою пластинкою та місцями під епідермісом видно гній. Під час пальпації відчуває різкий біль. На Ro-грамі пальця видні деструктивні зміни в кістці нігтьової фаланги. Який вид панарицію у хворої? A 45-year-old woman went to the hospital 12 days after the onset of the disease with complaints of pain and swelling of the I finger of the left hand. During the examination, an increase in body temperature up to 38, 9^C. The nail phalanx is thickened, purple in color. During palpation, there is a sharp pain in the bone of the nail phalanx sick?

Шкіряний Leather

Суглобовий Articulated

Підшкіряний Subcutaneous

Сухожильний Tendino

Кістковий Bone

128 / 1500
Дитина 5-ти років хворіє 2 тижні. Спочатку з'явилися напади кашлю, потім - репризи. Під час кашлю обличчя хворого червоніє, вени шиї набухають. Напади кашлю закінчуються блюванням. На рентгенограмі: підсилення бронхіального малюнка. Аналіз крові: лейкоцити - 16·10^9/л, лімф.- 72%, швидкість осідання еритроцитів - 4 мм/год. Який найбільш імовірний діагноз? A 5-year-old child has been ill for 2 weeks. At first, coughing attacks appeared, then repeated attacks. During coughing, the patient's face turns red, neck veins swell. Coughing attacks end with vomiting. On X-ray: increase in bronchial pattern. Blood count - 16·10^9/l, erythrocyte sedimentation rate - 4 mm/h. What is the most likely diagnosis?

Стороннє тіло Foreign body

Пневмонія Pneumonia

Обструктивний бронхіт Obstructive bronchitis

Кашлюк Whooping cough

Аденовірусна інфекція Adenovirus infection

129 / 1500
Хворий 52-х років скаржився на раптову слабкість і оніміння в лівих кінцівках, утруднення при ходьбі. Об'єктивно відзначалися лівобічні гемігіпестезія і легкий геміпарез. Через 4 години стан хворого нормалізувався, вогнищева симптоматика регресувала, хворий зміг нормально ходити. АТ- 120/80 мм рт.ст. Який найбільш імовірний діагноз? A 52-year-old patient complained of sudden weakness and numbness in the left limbs, difficulty walking. Objectively, left-sided hemipaesthesia and mild hemiparesis were noted. After 4 hours, the condition the patient normalized, the focal symptoms regressed, the patient was able to walk normally. Blood pressure - 120/80 mm Hg. What is the most likely diagnosis?

Транзиторна ішемічна атака Transient ischemic attack

Ішемічний інсульт Ischemic stroke

Гіпертензивний криз Hypertensive crisis

Геморагічний інсульт Hemorrhagic stroke

Асоційована мігрень Associated Migraine

130 / 1500
Хвора 46-ти років надійшла до клініки з приводу відкритого перелому лівого стегна в середній третині, при надходженні оперована - накістковий остеосинтез пластиною. На 4-й день після операції скаржиться на біль в рані, підвищення температури вище 39^oC. Яких заходів нео-бхідно вжити в цьому випадку? A 46-year-old patient came to the clinic with an open fracture of the left hip in the middle third, upon admission she underwent surgery - osseous osteosynthesis with a plate. On the 4th day after the operation complains of pain in the wound, an increase in temperature above 39°C. What measures should be taken in this case?

Внутрішньокістково ввести антибіотики, місцево гіпотермію Intraosseous inject antibiotics, local hypothermia

Видалити металевий фіксатор, призначити сульфаніламіди Remove metal retainer, prescribe sulfonamides

Розпустити шви, дренувати рану та призначити антибіотики Dissolve sutures, drain the wound and prescribe antibiotics

Обколоти рану антибіотиками, призначити спазмолітики й анальгетики Surround the wound with antibiotics, prescribe antispasmodics and analgesics

Призначити антибіотики широкого спектра дії та гормональні препарати Prescribe broad-spectrum antibiotics and hormonal drugs

131 / 1500
У пацієнта віком 35 років хвороба розпочалась бурхливо: з ознобу, підвищення температури тіла до 39^oС, блювання, болю в епігастрії, проносу з водянистими випорожненнями з домішками слизу і зеленкуватим відтінком. За 6 годин до захворювання з'їв сире яйце, смажену картоплю з тушкованим м`ясом, випив сік. Який збудник найімовірніше викликав подібний стан? In a 35-year-old patient, the disease began violently: with chills, an increase in body temperature to 39°C, vomiting, pain in the epigastrium, diarrhea with watery stools mixed with mucus and a greenish hue. 6 hours before the illness, he ate a raw egg, fried potatoes with stewed meat, and drank juice. What pathogen most likely caused this condition?

Кишкова паличка Escherichia coli

Шигела Shigela

Холерний вібріон Vibrio cholerae

Кампілобактер Campylobacter

Сальмонела Salmonella

132 / 1500
Першовагітна з'явилась в жіночу консультацію в 37 тижнів вагітності. Скарг немає. Протягом останніх 2 тижнів набрала 2 кг ваги. Визначаються набряки ніг. АТ - 120/70 мм рт. ст. Білок у сечі - 0,8 г/л. Поставлено діагноз: пре-екламсія легкого ступеня. Яка лікувальна тактика? A first-time pregnant woman came to the antenatal clinic at 37 weeks of pregnancy. There are no complaints. She has gained 2 kg of weight over the past 2 weeks. Swelling of the legs is detected. BP - 120/70 mm Hg. Protein in urine - 0.8 g/l. Diagnosed: mild pre-eclampsia?

Кесарський розтин Caesarean section

Пролонгування вагітності Prolongation of pregnancy

Стаціонарне лікування Inpatient treatment

Амбулаторне лікування Outpatient treatment

Термінове розродження Urgent delivery

133 / 1500
Шахтар із 15-річним стажем роботи звернувся до лікаря зі скаргами на погіршення стану здоров'я, задишку, кашель, біль у ділянці легень. Про яке захворювання слід думати? A miner with 15 years of work experience went to the doctor with complaints of deteriorating health, shortness of breath, cough, pain in the lung area. What disease should you think about ?

Бронхіальна астма Bronchial asthma

Пневмоконіоз Pneumoconiosis

Грип Flu

Бронхіт Bronchitis

Туберкульоз Tuberculosis

134 / 1500
На 8-му добу життя у новонародженого хлопчика з масою тіла 3500 г підвищується температура тіла та з'являється висип. Напередодні дитина була дуже дратівлива та мала субфебрильну температуру, що поступово підвищувалася. Мати повідомила, що до зазначеного епізоду дитина була здорова та ніяких ліків не приймала. Температура тіла - 38,9^oC, артеріальний тиск - 90/50 мм рт.ст., пульс - 160/хв., частота дихання - 17/хв., SpO_2 - 98% при кімнатному повітрі. При огляді виявлено висип у вигляді пухирів, що легко лопаються. Висип займає 60% поверхні тіла, навкруги рота, але не виявлено на слизових оболонках. Позитивний симптом Нікольського. Який діагноз є найбільш імовірним? On the 8th day of life, a newborn boy with a body weight of 3500 g has a fever and a rash appears. The day before, the child was very irritable and had a low-grade fever, which gradually increased. The mother reported that the child was healthy and did not take any medications. Body temperature - 38.9°C, blood pressure - 90/50 mmHg, pulse - 160/min. - 17/min., 98% in room air. Rash in the form of blisters, around 60% of the body surface, but not found in the mucous membranes. What is the diagnosis most likely?

Кропив'янка Hives

Стафілококовий синдром ''обпеченої шкіри'' (SSSS, синдром Ріттера) Scalded skin staphylococcal syndrome (SSSS, Ritter's syndrome)

Вроджена вітряна віспа Congenital chicken pox

Токсичний епідермальний некроліз Toxic epidermal necrolysis

Синдром Стівенса-Джонсона Stevens-Johnson syndrome

135 / 1500
Проведено дослідження щодо встановлення зв'язку між ожирінням і інфарктом міокарду у медичних сестер. Протягом декількох років реєстрували і порівнювали частоту появи нових випадків інфаркту міокарда в групах та зміни ІМТ. Вкажіть тип епідеміологічного дослідження: A study was conducted to establish the relationship between obesity and myocardial infarction in nurses. For several years, the frequency of new cases of myocardial infarction in groups and changes in BMI were recorded and compared Specify the type of epidemiological study:

Дослідження опис серії випадків Study description of case series

Когортне дослідження Cohort Study

Дослідження випадок-контроль Case-control study

Поперечне дослідження Cross-sectional study

Експериментальне дослідження Experimental study

136 / 1500
Дівчинка, 7 років, надійшла у відділення зі скаргами на біль у горлі, підвищення температури до 39^oC. Під час огляду стан дитини важкий, шийні лімфатичні вузли до 1,5 см. При пальпації печінка на 3 см, селезінка на 2 см виступає з-під краю реберної дуги. При лабораторному дослідженні крові: еритроцити - 4,0·10^12/л, Hb- 121 г/л, кольоровий показник - 0,9, тромбоцити - 190·10^9/л, лейкоцити - 19·10^9/л, е- 0, п/я- 1, с/я- 0, л- 87, м- 2, швидкість зсідання еритроцитів - 36 мм/год. Що з перерахованого буде найбільш доречним наступним кроком у веденні пацієнта? A 7-year-old girl came to the department with complaints of a sore throat, temperature rise to 39^oC. During the examination, the child's condition is serious, cervical lymph nodes to 1.5 cm. During palpation, the liver is 3 cm, the spleen protrudes 2 cm from the edge of the costal arch. During the laboratory blood test: erythrocytes - 4.0·10^12/l, Hb - 121 g/l, color index - 0.9, platelets - 190·10^9/l, leukocytes - 19·10^9/l, e- 0, p/ya- 1, s/ya- 0, l- 87, m- 2, speed erythrocyte sedimentation rate - 36 mm/h. Which of the following would be the most appropriate next step in the patient's management?

Спостереження протягом 2 тижнів Observation for 2 weeks

Дослідження кісткового мозку Bone marrow research

- -

Повторний загальний аналіз крові через 1 тиждень Repeat general blood test in 1 week

Аналіз крові на виявлення гетерофільних антитіл Blood analysis for detection of heterophilic antibodies

137 / 1500
Жінка 45 років скаржиться на загальну кволість, задишку, запаморочення. Упродовж року посивіло волосся, почали розшаровуватися нігті, змінився смак. 5 років перебуває на <<Д>> обліку у гінеколога з приводу фіброміоми матки. У крові виявлено: еритроцити - 3,0·10^12/л, Нb - 76 г/л, КП - 0,7, ретикулоцити - 0,7%, тромбоцити - 160·10^9/л , лейкоцити - 5,0 ·10^9/л, еозинофіли - 2%, паличкоядерні - 3%, сегментоядерні - 63%, лімфоцити - 28%, моноцити - 4%, анізо-, мікроцитоз, ШОЕ - 30 мм/год. Яку форму анемії можна припустити? A 45-year-old woman complains of general weakness, shortness of breath, dizziness. Over the course of a year, her hair turned gray, her nails began to peel, her taste changed. She has been on <> for 5 years examination by a gynecologist for uterine fibroids. In the blood, erythrocytes - 3.0·10^12/l, Hb - 76 g/l, CP - 0.7, reticulocytes - 0.7%, platelets - 160·10^ 9/l, leukocytes - 5.0 ·10^9/l, eosinophils - 2%, rod-nuclear - 3%, segmentonuclear - 63%, lymphocytes - 28%, monocytes - 4%, aniso-, microcytosis, ESR - 30 mm /h. What form of anemia can be assumed?

Мінковського-Шоффара Minkowski-Shofar

Аутоімунну гемолітичну Autoimmune hemolytic

Гіпопластичну Hypoplastic

В_12-дефіцитну B_12-deficient

Залізодефіцитну Iron deficiency

138 / 1500
Чоловік 64-х років надійшов у відділення невідкладної допомоги зі скаргами на набряки нижніх кінцівок, попереку та передньої черевної стінки. В анамнезі хронічний бронхіт з бронхоектазами. При фізикальному обстеженні температура тіла - 37,2^oC, артеріальний тиск - 110/75 мм рт.ст., пульс - 82/хв., частота дихання - 19/хв. При лабораторному дослідженні у загальному аналізі сечі (ЗАС): питома вага - 1025, білок - 9,9 г/л, лейкоцити - 2-3 у полі зору, еритроцити - 1-2 у полі зору, циліндри - немає. Добова протеїнурія - 11,4 г/добу. У біохімічному аналізі крові загальний білок - 52 г/л, альбуміни - 30 г/л, холестерин - 9,6 ммоль/л. Який тип ураження нирок у цього хворого є найбільш імовірним? A 64-year-old man was admitted to the emergency department with complaints of swelling of the lower extremities, lower back, and anterior abdominal wall. He has a history of chronic bronchitis with bronchiectasis. During physical examination body temperature - 37.2°C, blood pressure - 110/75 mmHg, heart rate - 82/min, respiratory rate - 19/min. In the laboratory examination in the general analysis of urine (ZAS): specific gravity - 1025 , protein - 9.9 g/l, leukocytes - 2-3 in the field of vision, erythrocytes - 1-2 in the field of vision. Daily proteinuria - 11.4 g/day. In the biochemical analysis of blood - 52 g/l, albumin - 30 g/l, cholesterol - 9.6 mmol/l. What type of kidney damage is most likely in this patient?

Діабетична нефропатія, нефритичний синдром Diabetic nephropathy, nephritic syndrome

Гломерулонефрит, нефритичний синдром Glomerulonephritis, nephritic syndrome

Тубуло-інтерстиціальний нефрит, нефротичний синдром Tubulo-interstitial nephritis, nephrotic syndrome

Пієлонефрит, сечовий синдром Pyelonephritis, urinary syndrome

АА амілоїдоз, нефротичний синдром AA amyloidosis, nephrotic syndrome

139 / 1500
У пацієнтки віком 32 роки, що народжує вперше, почалися інтенсивні потуги з інтервалом 1-2 хв, тривалістю 55-60 с. Об'єктивно спостерігається: настає прорізування голівки плода. Промежина, висота якої 4 см, надмірно випинається. Шкіра промежини бліда, напружена, після припинення потуги із статевої щілини з'явився тоненький струмок крові. Вкажіть подальшу тактику ведення пологів. In a 32-year-old female patient, who is giving birth for the first time, intense efforts began with an interval of 1-2 minutes, lasting 55-60 seconds. Objectively observed: teething occurs the head of the fetus, the height of which is 4 cm. The skin of the perineum is pale, tense, a thin stream of blood appeared from the genital opening.

Кесарський розтин Caesarean section

Вакуум-екстракція плода Vacuum fetal extraction

Епізіотомія Episiotomy

Очікувальна тактика Waiting tactics

Накладання акушерських щипців Applying obstetric forceps

140 / 1500
Чоловік віком 45 років скаржиться на інтенсивний біль за грудиною, що іррадіює у нижню щелепу, виникає у спокої, вночі, кілька разів по 10-15 хвилин. Під час больового нападу на ЕКГ реєструється елевація сегменту SТ у відведеннях V 3-4. Який попередній діагноз? A 45-year-old man complains of intense pain behind the sternum, radiating to the lower jaw, occurring at rest, at night, several times for 10-15 minutes. During of a pain attack, ST segment elevation is recorded in leads V 3-4. What is the preliminary diagnosis?

Стабільна стенокардія II функціонального класу Stable angina of functional class II

Стенокардія, що прогресує Progressive Angina

Стенокардія Принцметала Prinzmetal Angina

Інфаркт міокарда Myocardial infarction

Стабільна стенокардія IV функціонального класу Stable angina of functional class IV

141 / 1500
32-річна вагітна у терміні 5-6 тижнів була вакцинована проти грипу неживою вакциною разом з усією родиною. На той момент про вагітність вона не знала. Вагітність бажана. Пацієнтка звернулась до сімейного лікаря для отримання консультації щодо можливого впливу вакцини на розвиток і перебіг вагітності, виникнення вад розвитку у плода. Яку пораду слід надати вагітній? A 32-year-old pregnant woman was vaccinated against influenza with a non-live vaccine in 5-6 weeks along with the whole family. She did not know about the pregnancy at that time. The pregnancy is desirable. The patient turned to the family doctor for advice on the possible impact of the vaccine on the development and course of pregnancy, the occurrence of developmental defects in the fetus. What advice should be given to the pregnant woman?

Запропонувати медикаментозний аборт Offer medical abortion

Обов'язкова консультація інфекціоніста і генетика Mandatory consultation of an infectious disease specialist and geneticist

УЗД для виявлення вад розвитку плода Ultrasound to detect fetal malformations

Тест на антитіла до вірусу грипу Test for antibodies to influenza virus

Вакцинація проти грипу є безпечною протягом вагітності Flu vaccination is safe during pregnancy

142 / 1500
У хворого 10 років діагностовано геморагічний васкуліт, шкіряна форма. Одним з основних лікувальних заходів є тривала преднізолонотерапія. З якою метою застосовується ця терапія? A 10-year-old patient was diagnosed with hemorrhagic vasculitis, cutaneous form. One of the main treatment measures is long-term prednisolone therapy. What is the purpose of this therapy?

Підвищення синтезу простагландинів Increased synthesis of prostaglandins

Зменшення синтезу патологічних імунних комплексів Decreased synthesis of pathological immune complexes

Зменшення синтезу простагландинів Decreased synthesis of prostaglandins

Підвищення синтезу антитіл Increasing synthesis of antibodies

Як замістна терапія As replacement therapy

143 / 1500
У заводській їдальні виникло харчове отруєння, клініка якого вказувала на стафілококову етіологію. Захворіло 15 чоловік. Які матеріали від хворого необхідно надіслати на дослідження у лабораторію аби підтвердити харчове отруєння? Food poisoning occurred in the factory canteen, the clinic of which indicated a staphylococcal etiology. 15 people fell ill. What materials from the patient should be sent to the laboratory for research to confirm food poisoning?

Блювотні маси хворого Vomiting masses of the patient

Слину Saliva

Кров (клінічний аналіз) Blood (clinical analysis)

Сечу Urine

Кров на гемокультури Blood for blood cultures

144 / 1500
Працівник оформлюється на роботу, пройшов медичний профілактичний огляд. Визнаний придатним для роботи в умовах даного виробництва. Який вид медичного профілактичного огляду прой-шов працівник? The employee is registered for work, passed a preventive medical examination. Recognized as suitable for work in the conditions of this production. What type of preventive medical examination did the employee undergo?

Попередній Previous

Плановий Planned

Комплексний Comprehensive

Цільовий Target

Періодичний Periodic

145 / 1500
Жінка віком 59 років скаржиться на біль та набряклість дрібних суглобів кистей, задуху, слабкість. Хворіє 8 років. Об'єктивно спостерігається: t^o тіла - 37,8^oC, дрібноточкові крововиливи на тулубі та кінцівках, ульнарна девіація кистей. Межі серця зміщені вліво, систолічний шум над верхівкою. Пульс - 96/хв. АТ - 170/100 мм рт. ст. У загальному аналізі крові виявлено: еритроцити - 3,2·10^12/л, Hb - 108 г/л, лейкоцити - 6,8·10^9/л, тромбоцити - 220·10^9, ШОЕ - 48 мм/год., С-реактивний білок +++. У загальному аналізі сечі виявлено: щільність - 1016, білок - 2,8 г/л, лейкоцити - 10-12 в п/з, еритроцити - 2-4 в п/з. Який діагноз найімовірніший? A 59-year-old woman complains of pain and swelling of the small joints of the hands, shortness of breath, weakness. She has been ill for 8 years. Objectively observed: body t^o - 37, 8^oC, small point hemorrhages on the trunk and extremities, ulnar deviation of the hands, systolic murmur above the apex - 170/100 mm Hg ,2·10^12/l, Hb - 108 g/l, leukocytes - 6.8·10^9/l, platelets - 220·10^9, ESR - 48 mm/h, C-reactive protein ++ +. The general analysis of urine revealed: density - 1016, protein - 2.8 g/l, leukocytes - 10-12 in p/z, erythrocytes - 2-4 in p/z. What is the most likely diagnosis?

Тромбоцитопенічна пурпура Thrombocytopenic purpura

Хронічний гломерулонефрит Chronic glomerulonephritis

Ревматизм Rheumatism

Ревматоїдний артрит Rheumatoid arthritis

Системний червоний вовчак Systemic lupus erythematosus

146 / 1500
Після прогулянки на вулиці в ясний сонячний день у дитини 8 місяців з проя-вами рахіту ІІ ступеню, підгострого перебігу, періоду розпалу, з'явилися судоми у вигляді карпопедального спазму. Вони свідчать про наявність у дитини: After a walk on the street on a clear sunny day, an 8-month-old child with manifestations of rickets of the II degree, subacute course, period of exacerbation, developed convulsions in the form of carpopedal spasm. They indicate that the child has:

Епілепсії Epileptics

Черепно-мозкової травми Traumatic brain injury

ГРВІ ARV

Менінгіту Meningitis

Спазмофілії Spasmophilia

147 / 1500
Пацієнт скаржиться на різкий біль у горлі праворуч, що іррадіює в ліве вухо, неможливість відкрити рот, підвищення температури тіла до 40^oС. Об'єктивно спостерігається: виражений тризм жувальних м'язів, асиметрія праворуч, лівий піднебінний мигдалик гіперемований, зміщений до середини ротоглотки, язичок зміщений ліворуч. Неприємний запах із рота. Гіперсалівація. Защелепні лімфатичні вузли праворуч збільшені, болісні під час пальпації. Риноскопічна і отоскопічна картини в нормі. Встановіть діагноз. The patient complains of a sharp pain in the right throat radiating to the left ear, inability to open the mouth, an increase in body temperature up to 40°C. Objectively observed: pronounced masticatory muscles, asymmetry of the left palate, shifted to the middle of the oropharynx, unpleasant smell from the mouth, enlarged lymph nodes on the right, rhinoscopic and otoscopic findings are normal .

Правобічний паратонзилярний абсцес Right-sided paratonsillar abscess

Підщелепний лімфаденіт праворуч Submandibular lymphadenitis on the right

Правобічний сіалоаденіт Right-sided sialoadenitis

Пухлина правого піднебінного мигдалика Tumor of the right palatine tonsil

Правобічний парафарингеальний абсцес Right-sided parapharyngeal abscess

148 / 1500
Пацієнтка віком 35 років звернулась до лікаря зі скаргами на задишку, погіршення зору ('туман в очах'), подвоєння зображень, косоокість, ністагм, виражену сухість в роті, гугнявість голосу, утруднене ковтання, м'язеву слабкість та здуття живота. З харчового анамнезу відомо, що 20 годин тому споживала м’ясні консерви, рибу домашнього приготування. Який найімовірніший діагноз? A 35-year-old patient turned to the doctor with complaints of shortness of breath, impaired vision ('fog in the eyes'), doubling of images, strabismus, nystagmus, severe dry mouth , hoarseness of voice, difficulty swallowing, muscle weakness and bloating. It is known from the food history that she consumed canned meat and home-cooked fish 20 hours ago. What is the most likely diagnosis?

Ботулізм Botulism

Харчовий мікотоксикоз Food mycotoxicosis

Харчова токсикоінфекція Food poisoning

Харчове отруєння хімічного походження Food poisoning of chemical origin

Стафілококова інтоксикація Staphylococcal intoxication

149 / 1500
Чоловік 37 років скаржиться на біль у поперековому і грудному відділах хребта, обмеження рухливості в ньому протягом 5 років. Під час обстеження встановлено діагноз: анкілозуючий спондилоартрит, центральна форма. Носієм якого антигену HLA, най-імовірніше, є пацієнт? A 37-year-old man complains of pain in the lumbar and thoracic regions of the spine, limited mobility in it for 5 years. During the examination, the diagnosis was established: ankylosing spondylitis, central form. Which HLA antigen carrier is the patient most likely?

HLA-DR8 HLA-DR8

HLA-B27 HLA-B27

HLA-DR4 HLA-DR4

HLA-DR20 HLA-DR20

HLA-B5 HLA-B5

150 / 1500
Жінка 42-х років прийшла до лікаря на профілактичний прийом. Скарг не має. Загальний стан задовільний. Зріст 162 см, маса тіла 87 кг, ІМТ = 33 кг/м^2. Артеріальний тиск справа - 140/90 мм рт.ст., зліва - 145/85 мм рт.ст., пульс - 72/хв. Аускультативно тони серця приглушені, шуми не прослуховуються. В легенях дихання везикулярне. Нижній край печінки виступає на 1,5-2 см з-під нижнього краю. Набряків не виявлено. Яку оцінку конституціональній будові тіла цієї пацієнтки має надати лікар (за рекомендаціями ВООЗ)? A 42-year-old woman came to the doctor for a preventive appointment. She has no complaints. The general condition is satisfactory. Height 162 cm, body weight 87 kg, BMI = 33 kg /m^2. Arterial pressure - 140/90 mmHg, left - 72/min. Heart sounds are muffled. In the lungs, breathing is vesicular the edge of the liver protrudes 1.5-2 cm from the lower edge. No swelling was detected. What assessment should the doctor give to the constitutional structure of this patient (according to WHO recommendations)?

Ожиріння II класу Obesity class II

Надлишкова вага Overweight

Ожиріння III класу Obesity class III

Нормальна вага Normal weight

Ожиріння I класу Obesity class I

151 / 1500
До дерматолога звернулася жінка 38 років зі скаргами на сухість та лущення шкіри. Під час огляду на розгинальних поверхнях ліктьових і колінних суглобів виявлено папульозний висип і дрібне лущення, у ділянці волосяних фолікулів є вузлики воскоподібного кольору, що виступають над поверхнею шкіри. Ці клінічні прояви, найімовірніше, пов'язані з недостатнім надходженням з їжею в організм: A 38-year-old woman consulted a dermatologist with complaints of dryness and peeling of the skin. During the examination, a papular rash and small peeling were found on the extensor surfaces of the elbow and knee joints, in the area of hair follicles are nodules of a waxy color protruding above the surface of the skin. These clinical manifestations are most likely associated with insufficient intake of food into the body:

Піридоксину Pyridoxine

Рибофлавіну Riboflavin

Ретинолу Retinol

Аскорбінової кислоти Ascorbic acid

Тіаміну Thiamine

152 / 1500
Хворий 54-х років звернувся до сімейного лікаря зі скаргами на утруднення дихання, слабкість. Останні два тижні турбував біль та набряк правої нижньої кінцівки. Дані симптоми вперше в житті, раніше на обліку у лікаря не перебував. АТ- 110/70 мм рт.ст., ЧСС- 96/хв. Який діаг-ностичний метод має вирішальне значення? A 54-year-old patient turned to his family doctor with complaints of difficulty breathing, weakness. For the past two weeks, he has been troubled by pain and swelling of the right lower limb. These symptoms are the first time in his life , was not previously registered with a doctor. Blood pressure - 110/70 mmHg, heart rate - 96/min. What diagnostic method is crucial?

Функція зовнішнього дихання Function of external breathing

Мультиспіральна КТ ангіографія Multispiral CT angiography

Електрокардіографія Electrocardiography

Рентгенографія органів грудної клітки X-ray of chest organs

Ехокардіографія Echocardiography

153 / 1500
Пацієнт 66 років скаржиться на значну слабкість. З'явився озноб, коли відчув підвищення температури тіла, біль у суглобах та за ходом м'язів ніг. Об'єктивно спостерігається: фіолетово-синюшна еритема навколо очей та над колінними суглобами. ЧСС - 120/хв., тони серця ослаблені. В крові виявлено: лейкоцити - 12·10^9/л, ШОЕ - 40 мм/год. Поставте діагноз. A 66-year-old patient complains of significant weakness. Chills appeared when he felt an increase in body temperature, pain in the joints and along the course of the muscles of the legs. Objectively observed: purple-blue erythema around the eyes and over the knee joints. Heart rate - 120/min. In the blood: leukocytes - 12·10^9/l. Make a diagnosis.

Атопічний дерматит Atopic dermatitis

Системний червоний вовчак Systemic lupus erythematosus

Реактивний поліартрит Reactive polyarthritis

Ревматоїдний артрит Rheumatoid arthritis

Дерматоміозит Dermatomyositis

154 / 1500
32-річна жінка скаржиться на нерегулярні менструації з затримками до 2-3 місяців, значну прибавку маси тіла, гірсу-тизм. Заміжня 5 років. Вагітностей не було. При піхвовому дослідженні матка незначно зменшена, з обох сторін визначаю-ться щільні, рухомі яєчники до 4-5 см в діаметрі. Яку патологію можна припустити в даному випадку? A 32-year-old woman complains of irregular menstruation with delays of up to 2-3 months, a significant increase in body weight, hirsutism. She has been married for 5 years. There have been no pregnancies. During the vaginal examination, the uterus is slightly reduced, dense, mobile ovaries up to 4-5 cm in diameter are detected on both sides. What pathology can be assumed in this case?

Синдром полікістозних яєчників Polycystic ovary syndrome

Гіпоменструальний синдром Hypomenstrual syndrome

Туберкульоз придатків матки Tuberculosis of uterine appendages

Двобічні кісти яєчників Bilateral ovarian cysts

Хронічний двобічний сальпінгіт Chronic bilateral salpingitis

155 / 1500
Амортизаційні видатки на відновлення томографу закладено у вартість томографії у розмірі 10% річних від його первісної вартості. Через який термін стане можливим оновлення томографу? Amortization expenses for the restoration of the tomograph are included in the cost of the tomography in the amount of 10% per annum of its initial cost. After what period will it be possible to update the tomograph?

Через 10 років After 10 years

Через 15 років After 15 years

Через 20 років After 20 years

Через 7 років After 7 years

Через 5 років After 5 years

156 / 1500
На сільській лікарській дільниці зростає захворюваність на рак шийки матки. Прийнято рішення провести обстеження жінок. Який це вид медичних оглядів? The incidence of cervical cancer is increasing at the rural medical district. It was decided to examine women. What kind of medical examinations are these?

Поточний Current

Цільовий Target

Скринінговий Screening

Комплексний Comprehensive

Попередній Previous

157 / 1500
Лікар-неонатолог оглядає доношену дитину від ІІ вагітності, ІІ термінових пологів, з масою тіла - 3980 г. З анамнезу відомо, що в пологах виникла первинна слабкість пологової діяльності, використовувалась акушерська допомога. Об'єктивно спостерігається: права рука приведена до тулуба, ротована, відсутні рухи в плечовому та ліктьовому суглобах, спостерігається симптом 'лялькової ручки', кисть в положенні долонного згинання. Годується грудьми, смокче активно. Який найімовірніший діагноз? A neonatologist examines a full-term child from the 2nd pregnancy, the 2nd emergency delivery, with a body weight of 3980 g. From the anamnesis, it is known that during childbirth there was a primary weakness of labor activity , obstetric care was used. Objectively observed: the right arm is brought to the body, there is no movement in the shoulder and elbow joints, the hand is in a palmar flexion position. What is the most likely diagnosis?

Вивих плеча справа Right shoulder dislocation

Парез Дежерін-Клюмпке Parez Dezherin-Klumpke

Тотальний парез плечового сплетення Total paresis of the brachial plexus

Парез Дюшена-Ерба Duchen-Erb paresis

Перелом плечової кістки справа Fracture of humerus on the right

158 / 1500
В організм людини з атмосферного повітря надходять декілька хімічних речовин. Як називається тип комбінованої дії, при якому сумісний ефект менший від суми ефектів кожної з речовин, що входить у комбінацію, при їх ізольованій дії на організм? Several chemical substances enter the human body from atmospheric air. What is the type of combined action called, in which the combined effect is less than the sum of the effects of each of the substances included in the combination , with their isolated action on the body?

Поєднана дія Combined Action

Антагонізм Antagonism

Ізольована дія Isolated Action

Комплексна дія Complex Action

Потенціювання Potentiation

159 / 1500
Огляд у педіатра проходить дівчинка віком 14 років. Об'єктивно виявлено: дівчинка високого зросту, астенічної статури, на шкірі черевної порожнини стрії, голубі склери. Діагностовані сколіотична постава, деформація з боку грудної клітки. Відзначаються гіперрухливість суглобів, довгі пальці та руки. На УЗД серця візуалізується пролапс мітрального клапана. Що з перерахованого є найімовірнішою причиною її високого зросту? A 14-year-old girl is being examined by a pediatrician. Objectively revealed: a girl of tall stature, asthenic physique, stretch marks on the skin of the abdominal cavity, blue sclerae. Scoliotic posture was diagnosed , deformation on the side of the chest. Hypermobility of the joints, long fingers and hands are noted. Mitral valve prolapse is visualized on the ultrasound. Which of the following is the most likely cause of its high growth?

Синдром Елерса-Данлоса Ehlers-Danlos Syndrome

Синдром Нунан Noonan Syndrome

Синдром Марфана Marfan syndrome

Синдром Клайнфельтера Klinefelter syndrome

Синдром Вільямса Williams Syndrome

160 / 1500
Восьмирічна дитина скаржиться на тривалий вологий кашель з виділенням великої кількості харкотиння гнійного характеру з неприємним запахом, інколи з домішкою крові. Загальний стан тяжкий, температура тіла - 38,7^oC, шкіра бліда, периоральний ціаноз, пальці у вигляді ''барабанних паличок''. Над легенями: ослаблене дихання, різнокаліберні вологі хрипи. Рентгенологічно: ателектатичні ділянки. Яке з обстежень для уточнення діагнозу буде найбільш доцільним? An eight-year-old child complains of a long wet cough with the release of a large amount of sputum of a purulent nature with an unpleasant odor, sometimes with an admixture of blood. The general condition is serious, the body temperature is 38.7 ^oC, perioral cyanosis, fingers in the form of 'drumsticks.

Бронхоскопія Bronchoscopy

- -

Рентгенографія ОГК Roentgenography of OGK

Комп'ютерна томографія з високою розподільною здатністю Computed tomography with high resolution

Ехокардіографія Echocardiography

161 / 1500
Вагітній у терміні 32 тижні із загрозою передчасних пологів проводиться профілактика респіраторного дистрес-синдрому плода. Який препарат призначено? A 32-week pregnant woman with a risk of premature birth is being treated for fetal respiratory distress syndrome. What drug is prescribed?

Мізопростол Misoprostol

Дексаметазон Dexamethasone

Прогестерон Progesterone

Гініпрал Ginipral

Окситоцин Oxytocin

162 / 1500
У 38-річної жінки після сварки виникло головокружіння, слабкість. В анамнезі вказані: анорексія, втрата ваги, нудота, діарея. Гіпотензія - 60/30 мм рт. ст. у вертикальному положенні. Пульс - 110/хв., малий, ритмічний. Глюкоза крові - 3,3 мМ/л. Гіпонатріємія. Гіперкаліємія. Гіперпігментація шкіри. Екскреція 17-КС та 17-ОКС із сечею знижені. Який попередній діагноз? A 38-year-old woman developed dizziness and weakness after a quarrel. The anamnesis indicated: anorexia, weight loss, nausea, diarrhea. Hypotension - 60/30 mm Hg. Art. Pulse - 110/min. Hyponatremia. 17-KS excretion in urine?

Вагітність, гіпотонічний стан Pregnancy, hypotonic state

Цукровий діабет, гіпоглікемічний стан Diabetes, hypoglycemic state

- -

Прихована внутрішня кровотеча Hidden internal bleeding

Наднирникова недостатнісь Adrenal insufficiency

163 / 1500
У чоловіка 20-ти років невдовзі після екстракції зуба розпочалася сильна кровотеча. З анамнезу відомо, що пацієнт хворіє на гемофілію А. При лабораторному дослідженні у загальному аналізі крові: еритроцити - 3,2·10^12/л, Hb- 98 г/л, кольоровий показник - 0,92, лейкоцити - 7,4·10^9/л, тромбоцити - 240·10^9/л, швидкість осідання еритроцитів - 11 мм/год. Час кровотечі за Дюке - 3 хв., час зсідання крові за Лі-Уайтом - 20 хв. Яка фармакотерапія є найбільш ефективною для цього пацієнта? A 20-year-old man started bleeding heavily shortly after tooth extraction. From the anamnesis, it is known that the patient suffers from hemophilia A. During the laboratory examination in the general blood test: erythrocytes - 3.2·10^12/l, Hb - 98 g/l, color indicator - 0.92, leukocytes - 7.4·10^9/l, platelets - 240·10^9/l, sedimentation rate erythrocytes - 11 mm/h. Bleeding time according to Duque - 20 minutes. Which pharmacotherapy is the most effective for this patient?

Рекомбінантний VIII фактор Recombinant factor VIII

Рекомбінантний X фактор Recombinant X Factor

Рекомбінантний XІ фактор Recombinant XI factor

Еритроцитарна маса Erythrocyte mass

Тромбоцитарна маса Platelet mass

164 / 1500
25-річна жінка пів року тому перенесла аборт, скаржиться на зникнення апетиту, слабкість, артралгію. Через два тижні з'явилися темний колір сечі та жовтяниця, на тлі якої загальний стан продовжує погіршуватися. Припущено вірусний гепатит. Який з маркерів вірусного гепатиту ймовірно буде позитивним у пацієнтки? A 25-year-old woman had an abortion half a year ago, complains of loss of appetite, weakness, arthralgia. Two weeks later, dark urine and jaundice appeared, against the background whose general condition continues to deteriorate. Viral hepatitis is suspected. Which of the markers for viral hepatitis is likely to be positive in the patient?

Anti-HAV IgM Anti-HAV IgM

Anti-CMV IgM Anti-CMV IgM

Anti-HBc IgM Anti-HBc IgM

Anti-HEV IgM Anti-HEV IgM

Anti-HBs Anti-HBs

165 / 1500
Жінка 68 років була госпіталізована зі скаргами на головний біль у потиличній ділянці, виражену задишку в спокої, яка посилюється у положенні лежачи, та сухий кашель. Об'єктивно встановлено: положення ортопное, акроціаноз. Над легенями вислуховується ослаблене везикулярне дихання, середньо- і дрібнопухірчасті вологі хрипи в нижніх відділах. Тони серця ослаблені, ритм галопу. Пульс - 102/хв., АТ - 210/110 мм рт. ст. Печінка виступає на 2 см з-під краю реберної дуги, чутлива під час пальпації. Набряки на гомілках. Які препарати потрібно призначити насамперед? A 68-year-old woman was hospitalized with complaints of a headache in the occipital region, pronounced shortness of breath at rest, which worsens when lying down, and a dry cough. Objectively established : orthopneic position, weak vesicular breathing is heard in the lower parts, heart rate is 102/min 2 cm from the edge of the costal arch, sensitive during palpation. Swelling on the lower legs. What drugs should be prescribed first?

Лабеталол в/в, фуросемід в/м Labetalol IV, Furosemide IV

Цефтріаксон, лазолван в/в Ceftriaxone, Lazolvan IV

Сульфат магнію в/в, фуросемід в/м Magnesium sulfate IV, furosemide IV

Нітрогліцерин, фуросемід в/в Nitroglycerin, IV furosemide

Левофлоксацин в/в, амброксол перорально Levofloxacin IV, ambroxol oral

166 / 1500
Пацієнт віком 47 років, захворів 3 дні тому. Скаржиться на підвищення температури до 39^oС, продуктивний кашель з мокротинням жовто-зеленого кольору, задишку, біль у лівій половині грудної клітини. Під час обстеження виявлено: ЧД - 26/хв, укорочення перкуторного звуку та крепітація нижче кута лопатки зліва. SpO2 в межах норми (96%). Який з наведених методів досліджень найбільш інформативний для встановлення остаточного діагнозу? A 47-year-old patient became ill 3 days ago. He complains of an increase in temperature up to 39°C, a productive cough with yellow-green sputum, shortness of breath, pain in the left half of the chest. During the examination, HR - 26/min, and crepitation below the angle of the scapula were found. Which of the following research methods is the most informative for establishing the final diagnosis?

Клінічний аналіз крові Clinical blood test

Спірографія Spirography

Мікробіологічне дослідження мокротиння Microbiological examination of sputum

Рентгенографія органів грудної клітки X-ray of chest organs

Бронхоскопія Bronchoscopy

167 / 1500
Пацієнту віком 65 років з приводу гіпертонічної хвороби з супутньою ІХС була призначена фармакотерапія (раміприл, аторвастатин, амлодипін, ацетилсаліцилова кислота, бісопролол). Через 2 тижні він звернувся до лікаря з приводу сухого кашлю. Під час обстеження не було виявлено ознак ГРВІ, ураження бронхо-легеневого апарата. Припущено побічну дію раміприлу. Яким з наведених препаратів можна замінити раміприл у схемі лікування? A 65-year-old patient was prescribed pharmacotherapy (ramipril, atorvastatin, amlodipine, acetylsalicylic acid, bisoprolol) for hypertension with concomitant coronary artery disease. After 2 weeks, he turned to doctor about a dry cough. During the examination, no signs of ARVI were detected. A side effect of ramipril was assumed. Which of the following drugs can be replaced with ramipril in the treatment regimen?

Ніфедипін Nifedipine

Торасемід Torasemide

Еналаприл Enalapril

Валсартан Valsartan

Небіволол Nebivolol

168 / 1500
Жінка 60-ти років надійшла до відділення невідкладної допомоги зі скаргами на сильний головний біль у потиличній ділянці, нудоту, блювання, запаморочення. При огляді хвора трохи загальмована, обличчя гіперемоване, артеріальний тиск - 220/130 мм рт.ст., пульс - 78/хв. При неврологічному обстеженні парезів не виявлено, рефлекси рівномірно жваві, м'язовий тонус не змінений, чутливість збережена. Менінгеальні симптоми відсутні. Який стан найбільш імовірно розвинувся у хворої? A 60-year-old woman came to the emergency department with complaints of a severe headache in the back of the head, nausea, vomiting, dizziness. On examination, the patient is slightly depressed, her face hyperemic, blood pressure - 220/130 mmHg, pulse - 78/min. No paresis was detected, muscle tone was not changed. Meningeal symptoms are most likely to have developed in the patient?

Транзиторне порушення мозкового кровообігу Transient cerebral circulation disorder

Пухлина головного мозку Brain tumor

Ішемічний інсульт Ischemic stroke

Субарахноїдальний крововилив Subarachnoid hemorrhage

Гіпертонічний криз Hypertensive crisis

169 / 1500
Під час огляду трупа людини, яка померла внаслідок повішення, виявлено: трупні плями під час натискування зникають, відновлюються за 50 сек, трупне заклякання помірно виражене тільки в жувальних м'язах, у м'язах шиї та пальців кисті. Температура тіла - 31,0^oC. Визначте час настання смерті? During examination of the corpse of a person who died as a result of hanging, it was found: corpse spots disappear when pressed, recover in 50 seconds, corpse incantation is moderately expressed only in chewing m in the muscles of the neck and fingers. Body temperature - 31.0°C. Determine the time of death?

8-10 годин тому 8-10 hours ago

6-7 годин тому 6-7 hours ago

10-18 годин тому 10-18 hours ago

16-24 години тому 16-24 hours ago

1-2 години тому 1-2 hours ago

170 / 1500
На території із підвищеною ендемічною захворюваністю на зоб сімейний лікар для вторинної профілактики запропонував вживати багаті на йод харчові продукти. Які із наведених продуктів варто вживати за цих умов? In an area with a high endemic incidence of goiter, the family doctor suggested consuming iodine-rich food products for secondary prevention. Which of the following products should be consumed under these conditions?

М'ясні продукти Meat products

Морепродукти Seafood

Овочі та фрукти Vegetables and fruits

Молочні продукти Dairy products

Вироби з борошна Flour products

171 / 1500
Жінка 28 років скаржиться на загальну слабкість, головний біль, підвищення температури тіла до 37-38^oС, незначний біль у горлі. Хворіє 3-й день. Об'єктивно встановлено: шкіра бліда, губи ціанотичні. Гіперемія ротоглотки з ціанотичним відтінком, набряклі язичок, піднебінні дужки, мигдалики. На поверхні мигдаликів - суцільні щільні білуваті з перламутровим відтінком нальоти, які знімаються шпателем з великим зусиллям, після їх видалення підлегла слизова оболонка кровоточить. Збільшені підщелепні лімфатичні вузли. Набряк шиї. Тахікардія. АТ - 105/65 мм рт. ст. Який найбільш імовірний діагноз? A 28-year-old woman complains of general weakness, a headache, an increase in body temperature to 37-38°C, a slight sore throat. She has been ill for the 3rd day. About objectively established: pale skin, cyanotic lips. Hyperemia of the oropharynx with a cyanotic tinge, swollen tongue, palatal brackets, tonsils - continuous whitish plaques with a pearly tint, which are removed with a spatula with great effort, the underlying mucous membrane bleeds Enlarged lymph nodes. Tachycardia - 105/65 mmHg. What is the most likely diagnosis?

Аденовірусна інфекція Adenovirus infection

Дифтерія ротоглотки Oropharyngeal diphtheria

Ангіна Angina

Інфекційний мононуклеоз Infectious mononucleosis

Гострий лейкоз Acute leukemia

172 / 1500
Жінку на четвертий день після приїзду з Індії шпиталізовано до інфекційного відділення зі скаргами на біль у животі, випорожнення рідкої консистенції в невеликій кількості, 7-8 разів за добу, що містять велику кількість склоподібного слизу і кров. Загальний стан задовільний, шкіра бліда, чиста. Живіт м’який, чутливий у ділянці сліпої та висхідної кишок. Який лікарський засіб потрібно призначити жінці? On the fourth day after arriving from India, a woman was hospitalized in the infectious disease department with complaints of abdominal pain, small amounts of liquid stools, 7-8 times a day, containing a large amount of vitreous mucus and blood. The general condition is satisfactory, the skin is soft, sensitive in the area of ​​the cecum and the ascending colon. What medicine should be prescribed to the woman?

Еритроміцин Erythromycin

Лоперамід Loperamide

Цефтріаксон Ceftriaxone

Метронідазол Metronidazole

Ніфуроксазид Nifuroxazide

173 / 1500
Пацієнт віком 47 років отримав опік полум’ям верхніх кінцівок та тулуба. Під час шпиталізації спостерігається: стан важкий, свідомість затьмарена, лихоманка, АТ - 80/50 мм рт. ст., пульс - 118/хв. Місцево констатовано опіки III Б ступеня площею 20%. Які лікувальні заходи необхідно провести насамперед? A 47-year-old patient received flame burns on his upper limbs and trunk. During hospitalization, the following conditions were observed: his condition was severe, his consciousness was clouded, fever, blood pressure - 80/50 mm rt., pulse - 118/min. Locally, 20% degree of burns were diagnosed. What treatment measures should be taken first?

Провести первинну хірургічну обробку опікової рани Perform primary surgical treatment of a burn wound

Призначити дезінтоксикаційну терапію Prescribe detoxification therapy

Призначити антибактеріальну та дезінтоксикаційну терапію Prescribe antibacterial and detoxification therapy

Провести некротомію опікової поверхні, гемотрансфузію Perform necrotomy of the burn surface, hemotransfusion

Увести наркотичні анальгетики, почати інфузійну терапію Introduce narcotic analgesics, start infusion therapy

174 / 1500
Чоловікові 38 років. З анамнезу відомо, що він багато років страждає на епілепсію. 3 доби тому переніс стан, який розпочався раптово і супроводжувався злістю. Перебуваючи в цьому стані, пацієнт говорив щось дивне і незрозуміле, поламав меблі, ударив дружину. Тривав цей стан близько години, після чого пацієнт заснув. Потім казав, що не пам’ятає подій цього періоду. Найімовірнішим визначенням цього стану є: The man is 38 years old. It is known from the anamnesis that he has suffered from epilepsy for many years. 3 days ago he suffered a condition that began suddenly and was accompanied by anger. Being in this condition , the patient said something strange and incomprehensible, hit his wife. This state lasted for about an hour, after which the patient said that he did not remember the events of this period. The most likely definition of this state is:

Амбулаторний автоматизм Ambulatory automatism

Аментивний розлад свідомості Amentative disorder of consciousness

Фуга Fugue

Сутінковий розлад свідомості Twilight disorder of consciousness

Транс Trans

175 / 1500
До приймального відділення звернулася хвора 28-ми років зі скаргами на біль в ділянці лівого плечового суглобу, який посилюється при рухах. При огляді: в ділянці ключично-акроміального з'єднання виявляється набряк і сходинкоподібна деформація. При пальпації - болючість, позитивний симптом ''клавіші''. Який попередній діагноз? A 28-year-old female patient came to the emergency department with complaints of pain in the area of ​​the left shoulder joint, which worsens with movement. On examination: in the area of ​​the clavicular-acromial with 'the joint is swollen and stair-like deformation. On palpation - soreness, a positive symptom of 'keys'. What is the preliminary diagnosis?

Перелом акроміального відростка лопатки Fracture of the acromial process of the scapula

Передньо-верхній вивих плечової кістки Anterior-superior dislocation of the humerus

Перелом акроміального кінця ключиці Fracture of the acromial end of the clavicle

Вивих акроміального кінця ключиці Dislocation of the acromial end of the clavicle

Перелом головки плечової кістки Fracture of humeral head

176 / 1500
Пацієнтка віком 39 років скаржиться на періодичний головний біль, пульсуючий за характером, завжди зліва, частіше перед менструаціями. Які групи препаратів доцільно призначити насамперед? A 39-year-old patient complains of periodic headache, throbbing in nature, always on the left side, more often before menstruation. What groups of drugs should be prescribed first?

Антидепресанти Antidepressants

Валеріану Valeriana

Триптани Triptans

Дексаметазон Dexamethasone

Парацетамол Paracetamol

177 / 1500
До лікарні шпиталізовано вагітну у терміні гестації 38 тижнів, вагітність друга. Перша вагітність завершилася природніми пологами, народився хлопчик вагою 3600 гр. У жінки було виявлено ВІЛ-позитивний статус під час другої вагітності на 12 тижні гестації. Жінка отримує ААРТ із 22 тижня вагітності. Положення плода поздовжнє, головне передлежання. Передбачувана вага 3200pm200 гр. Серцебиття плода звучне, ритмічне, 122-138/хв. Вірусне навантаження становить 40 вірусних копій/мл. Яка тактика ведення пологів у цьому разі? A pregnant woman was hospitalized at 38 weeks' gestation, second pregnancy. The first pregnancy ended in natural childbirth, a boy weighing 3600 g was born. The woman was found to be HIV-positive During the 12th week of pregnancy, the woman receives AART from the 22nd week of pregnancy. What are the delivery tactics in this case?

Вести пологи консервативно Give birth conservatively

Виключити ІІ період пологів Exclude II period of childbirth

Кесаревів розтин на 38 тижнів вагітності Caesarean section at 38 weeks of pregnancy

Провести індукцію пологів на 38 тижні вагітності Induce labor at 38 weeks of pregnancy

Кесарів розтин на 40 тижнів вагітності Caesarean section at 40 weeks of pregnancy

178 / 1500
Для вивчення фізичного розвитку дітей та підлітків широко використовують антропометричні дослідження. Виберіть із наведеного фізіометричний метод досліджень: Anthropometric studies are widely used to study the physical development of children and adolescents. Choose a physiometric research method from the following:

Визначення форми грудної клітки Determining the shape of the chest

Визначення маси тіла Determination of body weight

Визначення життєвої ємності легень Determining the vital capacity of the lungs

Вимірювання зросту Height measurement

Визначення форми хребта Definition of spine shape

179 / 1500
Під час медичного огляду робітників металургійного заводу після вимірювання індексу маси тіла (ІМТ) у 20% осіб було вия-влено надлишкову вагу. Вміст яких продуктів необхідно зменшити в раціоні в першу чергу для нормалізації маси тіла у цієї групи осіб? During the medical examination of workers at a metallurgical plant, after measuring the body mass index (BMI), 20% of them were found to be overweight. Which foods should be reduced in the diet primarily to normalize body weight in this group of people?

Овочів Vegetables

Хлібобулочних виробів Bakery

Молока та молочних продуктів Milk and milk products

М'ясних та рибних продуктів Meat and fish products

Фруктів Fruits

180 / 1500
У хворого 37-ми років, ВІЛ-позитивного, впродовж 2-х тижнів наростала задишка, температура підвищилась до 37,7^oC. Діагностована пневмоцистна пневмонія. Який з препаратів етіотропної терапії показаний у цьому випадку? A 37-year-old HIV-positive patient developed shortness of breath over the course of 2 weeks, and the temperature rose to 37.7°C. Pneumocystis pneumonia was diagnosed. What from drugs of etiotropic therapy indicated in this case?

Цефтріаксон Ceftriaxone

Флуконазол Fluconazole

Метронідазол Metronidazole

Ацикловір Acyclovir

Ко-тримоксазол Co-trimoxazole

181 / 1500
Під час профілактичного огляду населення одного з міст України у 25 % мешканців виявлені наступні симптоми: плямиста емаль зубів, генералізований остеосклероз із кальцифікацією міжхребцевих зв'язок. Яка найімовірніша причина виникнення цих симптомів? During a preventive examination of the population of one of the cities of Ukraine, 25% of the residents were found to have the following symptoms: spotted tooth enamel, generalized osteosclerosis with calcification of the intervertebral ligaments. What is the most likely cause occurrence of these symptoms?

Надмірний вміст фтору в рослинних продуктах Excessive fluoride content in vegetable products

Надмірний вміст фтору у воді Excessive fluoride content in water

Недостатнє надходження фтору в організм із чаєм Insufficient intake of fluoride in the body with tea

Недостатній вміст фтору у ґрунті та воді Insufficient fluoride content in soil and water

Недостатній вміст фтору в тваринних продуктах Insufficient fluoride content in animal products

182 / 1500
Хлопчик 10-ти років звернувся в поліклініку зі скаргами на закладеність носу. Відомо, що подібні симптоми виникають періодично (весна, осінь). З анамнезу життя: атопічний дерматит. Батько дитини хворіє на бронхіальну астму. Об'єктивно: обличчя бліде, трохи набрякле. ЧД- 22/хв. Аускультативно: над легенями везикулярне дихання. Риноскопічно: слизова носової порожнини набрякла, бліда. Яке захворювання можна припустити? A 10-year-old boy came to the polyclinic with complaints of nasal congestion. It is known that similar symptoms occur periodically (spring, autumn). Life history: atopic dermatitis The child's father is suffering from bronchial asthma. Objectively, the face is slightly swollen. Auscultation: vesicular breathing. Nasal mucosa is pale.

Алергічний риніт Allergic rhinitis

Гострий аденоїдит Acute adenoiditis

Гострий риніт Acute rhinitis

Рекурентне респіраторне захворювання Recurrent respiratory disease

Гострий гайморит Acute sinusitis

183 / 1500
У хворого 35-ти років, який у зв'язку із хронічним гломерулонефритом 3 останні роки перебуває на гемодіалізі, з'явилися перебої в діяльності серця, гіпотонія, наростаюча слабкість, задишка. На ЕКГ: брадикардія, атріовентрикулярна блокада І ступеня, високі загострені зубці Т. Напередодні було грубе порушення питного та дієтичного режимів. Які біо-хімічні зміни є найбільш імовірною причиною вищевказаної клінічної картини? A 35-year-old patient, who has been on hemodialysis for the last 3 years due to chronic glomerulonephritis, had heart failure, hypotension, increasing weakness, shortness of breath. On the ECG: bradycardia, atrioventricular blockade of the first degree, high sharp T waves. The day before, there was a gross violation of the drinking and dietary regimes. What bio-chemical changes are the most likely cause of the above-mentioned clinical picture?

Гіпокаліємія Hypokalemia

Гіперкаліємія Hyperkalemia

Гіпокальціємія Hypocalcemia

Гіпернатріємія Hypernatremia

Гіпергідратація Hyperhydration

184 / 1500
У 10-річної дівчинки на тлі підвищеної до 37,5^oC температури тіла спостерігаються помірне збільшення лімфовузлів шиї, сірувато-білі нашарування на мигдаликах у вигляді щільної плівки. Плівка важко знімається, поверхня під нею кровоточить. Набряку шиї немає. Попередній діагноз: A 10-year-old girl, against the background of a body temperature raised to 37.5^oC, has a moderate increase in neck lymph nodes, grayish-white layering on the tonsils in the form of a dense film. The film is difficult to remove, the surface is bleeding. There is no swelling of the neck. Preliminary diagnosis:

Стерптококовий тонзилофарингіт Streptococcal tonsillopharyngitis

Ангіна Симановського-Венсана Symanovsky-Vincent angina

Дифтерія мигдаликів Diphtheria of tonsils

Скарлатина Scarlatina

Інфекційний мононуклеоз Infectious mononucleosis

185 / 1500
Пацієнтка віком 26 років скаржиться на переймоподібний біль в животі, проноси зі значною кількістю слизу і крові, підвищення температури тіла до 37,5-38,0^oС. Об'єктивно спостерігається: блідість шкіри та слизових оболонок, астенічна будова тіла. Під час пальпації виявлена болючість за ходом товстої кишки. В ході колонофіброскопії виявлено: стінка прямої та сигмоподібної кишки набрякла, ерозії, дрібні виразки, в просвіті слиз із кров'ю. Який імовірний діагноз? A 26-year-old patient complains of spasm-like abdominal pain, diarrhea with a significant amount of mucus and blood, an increase in body temperature to 37.5-38.0°C. Objectively observed: pallor of the skin and mucous membranes, tenderness of the colon during palpation, swelling of the rectal and sigmoid wall, small ulcers, mucus in the lumen. What is the probable diagnosis?

Неспецифічний виразковий коліт Nonspecific ulcerative colitis

Хвороба Крона Crohn's disease

Хронічний ентерит Chronic enteritis

Рак товстої кишки Colon cancer

Дизентерія Dysentery

186 / 1500
У воді річки, нижче спуску стічних вод гірничо-металургійного комбінату в місті водозабору, вміст кадмію перевищує гранично-допустиму концентрацію в 8-10 раз. Які захворювання серед населення, пов'язані з даною речовиною, будуть спостерігатися? In the water of the river, below the discharge of wastewater of the mining and metallurgical plant in the city of water intake, the cadmium content exceeds the maximum permissible concentration by 8-10 times. What are the diseases among the population , related to this substance, will be observed?

Хвороба Прасада Prasad Disease

Уровська хвороба Urov's disease

Хвороба Минамата Minamata disease

Хвороба Ітай-Ітай Itai-Itai disease

Хвороба Юшо Yusho's disease

187 / 1500
Пацієнтка віком 12 років скаржиться на високу температуру тіла, болі в м'язах, утруднене ковтання їжі. Об'єктивно спостерігається: параорбітальний набряк з рожево-фіолетовим відтінком, під час пальпації біль та зниження тонусу м'язів, капілярити в ділянках подушечок пальців та долонь, розширення меж та приглушеність тонів серця, гепато-спленомегалію. Лабораторно виявлено: збільшення рівня креатиніну в крові та сечі. Який найімовірніший діагноз? A 12-year-old patient complains of high body temperature, muscle pain, difficulty swallowing food. Objectively observed: paraorbital swelling with a pink-purple hue, during palpation, pain and decreased muscle tone, capillaritis in the areas of the pads of the fingers and palms, expansion of the borders and muffled tones of the heart. Laboratory findings: an increase in the level of creatinine in the blood and urine. What is the most likely diagnosis?

Ювенільний ревматоїдний артрит Juvenile rheumatoid arthritis

Склеродермія Scleroderma

Системний червоний вовчак Systemic lupus erythematosus

Дерматоміозит Dermatomyositis

Вузликовий періартеріїт Nodular periarteritis

188 / 1500
Жінка 29 років звернулась до центру планування сім'ї та репродуктивного здоров'я зі скаргами на безпліддя протягом 6 років подружнього життя. Місячні з 14 років по 3-4 дні, кількість виділень незначна, нерегулярні. Об'єктивно встановлено: гірсутизм, галакторея. Базальна температура - монофазна. Яка найбільш імовірна причина жіночого безпліддя? A 29-year-old woman applied to the center of family planning and reproductive health with complaints of infertility during 6 years of married life. Monthly from 14 years to 3-4 days, the amount of discharge is insignificant, objectively established: hirsutism, basal temperature - monophasic. What is the most likely cause of female infertility?

Трубні, перітонеальні фактори Tubular, peritoneal factors

Нез'ясованого генезу Of unknown origin

Ендокринні фактори Endocrine factors

Матковий фактор Maternal Factor

Імунологічні фактори Immunological factors

189 / 1500
26-річна вагітна жінка доставлена до відділення невідкладної терапії на 36-му тижні вагітності зі скаргами на інтенсивний головний біль у лобній ділянці. При фізикальному обстеженні: артеріальний тиск - 170/90 мм рт.ст., пульс - 85/хв., частота дихання - 15/хв., температура - 36,9^oC, набряки кінцівок. Серцебиття плоду - 159/хв. Під час огляду у жінки розвивається напад генералізованих тоніко-клонічних судом. Який препарат лікар має ввести першочергово? A 26-year-old pregnant woman was brought to the emergency department at the 36th week of pregnancy with complaints of an intense headache in the frontal area. On physical examination: blood pressure - 170/90 mm Hg, pulse - 85/min, temperature - 36.9°C, fetal heart rate - 159/min During the examination, the woman developed a generalized seizure tonic-clonic convulsions. What drug should the doctor administer first?

Натрію вальпроат Sodium valproate

Ламотриджин Lamotrigine

Фенітоїн Phenytoin

Магнію сульфат Magnesium sulfate

Діазепам Diazepam

190 / 1500
У пацієнтки віком 30 років під час обстеження виявлено добову протеїнурію - 2,2 г. Під час біопсії виявлено: зміни стінок капілярів клубочків у вигляді дротяних петель. Про яку морфологічну форму ураження нирок свідчать результати біопсії? During the examination, a 30-year-old patient was found to have daily proteinuria - 2.2 g. During the biopsy, it was found: changes in the walls of the glomerular capillaries in the form of wire loops. About which the morphological form of kidney damage is evidenced by biopsy results?

Мезангіальний гломерулонефрит Mesangial glomerulonephritis

Дифузний вовчаковий гломерулонефрит Diffuse lupus glomerulonephritis

Нефросклероз Nephrosclerosis

IgA нефропатія IgA nephropathy

Мембранозний гломерулонефрит Membranous glomerulonephritis

191 / 1500
Пацієнт 18 років хворіє з раннього дитинства. Під час обстеження встановлено: Нb - 110 г/л, еритроцити - 3,9·10^12/л, КП - 0,8, лейкоцити - 6,0·10^9/л; ШOЕ - 30 мм/год. Коагулограма показала: протромбіновий індекс - 95%, ретракція кров'яного згустка - 50%, час згортання крові - за 40 хвилин не відбулось, тривалість кровотечі - 3 хв. Які механізми складають основу патогенезу цього захворювання? An 18-year-old patient has been sick since early childhood. During the examination, the following was found: Hb - 110 g/l, erythrocytes - 3.9·10^12/l, CP - 0.8, leukocytes - 6.0·10^9/l; ESR - 30 mm/h. The coagulogram showed: prothrombin index - 50%, blood clotting time - 40 minutes. occurred, duration of bleeding - 3 minutes. What mechanisms form the basis of the pathogenesis of this disease?

Дефіцит вітаміну С Vitamin C deficiency

Недостатність у крові антигемофільного глобуліну А Antihemophilic globulin A deficiency in the blood

Наявність специфічних антитіл до ендотеліальних стінок судин Presence of specific antibodies to the endothelial walls of vessels

Імунне пригнічення кісткового мозку Bone marrow immune suppression

Екзогенний дефіцит заліза Exogenous iron deficiency

192 / 1500
Серед пацієнтів загальної лікарської практики за останні роки відбувається неухильне збільшення кількості осіб похилого та старечого віку. Переважання якого типу патології необхідно чекати в структурі захворюваності населення, яке обслуговується? Among the patients of general medical practice, there has been a steady increase in the number of elderly and senile people in recent years. What type of pathology should be expected to predominate in the morbidity structure of the population being served?

Неепідемічної патології Non-epidemic pathology

Гострої патології Acute pathologies

Хронічної патології Chronic pathology

Виробничої патології Production pathology

Інфекційної патології Infectious pathology

193 / 1500
Жінка віком 36 років скаржиться на біль у суглобах та м'язах, втрату апетиту, закрепи, швидку втомлюваність та субфебрильну температуру. Об'єктивно виявлено: дисфагія, симетричний артрит та потовщення шкіри на кистях та стопах, атрофія та дрібні виразки на пучках пальців, синдром Рейно, телеангіектазія. Яке захворювання зумовлює таку картину? A 36-year-old woman complains of joint and muscle pain, loss of appetite, constipation, rapid fatigue, and low-grade fever. Objectively found: dysphagia, symmetrical arthritis and thickening of the skin on the hands and feet, atrophy and small ulcers on the bundles of the fingers, Raynaud's syndrome, telangiectasia. What disease causes such a picture?

Дерматоміозит Dermatomyositis

Ревматоїдний артрит Rheumatoid arthritis

Системний червоний вовчак Systemic lupus erythematosus

Системна склеродермія Systemic scleroderma

Хвороба Рейно Raynaud's disease

194 / 1500
Жінка 40-ка років звернулася до лікаря зі скаргами на тупий ниючий біль в лівій поперековій ділянці, пітливість, зниження працездатності, субфебрильну температуру. У дитинстві перенесла туберкульоз шийних хребців. Лікувалася, знята з диспансерного обліку. При фізикальному обстеженні нирки не пальпуються, болючість в проекції лівої нирки. При лабораторному дослідженні в загальному аналізі сечі спостерігаються протеїнурія, піурія. Реакція сечі кисла. Яке дослідження найбільш імовірно дозволить уточнити діагноз цієї хворої? A 40-year-old woman consulted a doctor with complaints of dull, aching pain in the left lumbar region, sweating, reduced work capacity, low-grade fever. She suffered from tuberculosis of the cervical vertebrae as a child She was treated, removed from the medical records. The kidneys are not palpable. In the laboratory examination, proteinuria, pyuria are most likely to clarify the diagnosis of this patient.

Цистографія Cystography

Ультразвукове дослідження нирок Ultrasound study of kidneys

Проба Манту та повторний візит через 2 дні Mantoux test and return visit in 2 days

Екскреторна урографія Excretory urography

Полімеразна ланцюгова реакція (ПЛР) сечі Polymerase chain reaction (PCR) of urine

195 / 1500
Під час виробництва коксу концентрація пилу в повітрі робочої зони впродовж багатьох років перевищує ГДК у 4-8 разів. Розвиток якого захворювання є найімовірнішим у робітників цього виробництва? During the production of coke, the concentration of dust in the air of the working area for many years exceeds the MPC by 4-8 times. The development of which disease is most likely among the workers of this production?

Силікоз Silicosis

Сидероз Siderosis

Антракоз Anthracosis

Азбестоз Asbestosis

Бісиноз Bisinosis

196 / 1500
Дівчина 18 років скаржиться на напади чхання, переважно вранці, 15-20 разів поспіль, свербіння в носі, глотці, численні водянисті виділення з носа, нежить, біль в очах. Який із тестів є найінформативнішим для постановки діагнозу? An 18-year-old girl complains of sneezing attacks, mostly in the morning, 15-20 times in a row, itching in the nose, throat, numerous watery discharge from the nose, runny nose, pain in eyes. Which of the tests is the most informative for making a diagnosis?

Прик-тест Prick test

Крапельна проба Drop sample

Провокаційна проба Provocative test

Загальний IgE Total IgE

Внутрішньошкірна проба Intradermal test

197 / 1500
Пацієнтка віком 30 років звернулася до лікаря зі скаргами на відсутність місячних впродовж двох років після других пологів. З анамнезу відомо, що пологи ускладнилися масивною кровотечею. Після пологів пацієнтка почала відзначати випадіння волосся, втрату ваги. Об'єктивно спостерігається: жінка астенічна, зовнішні статеві органи гіпопластичні, шийка матки циліндричної форми, тіло матки маленьке, безболісне, додатки матки не визначаються. Який діагноз найімовірніший? A 30-year-old patient turned to the doctor complaining of the absence of menstruation for two years after the second childbirth. From the anamnesis, it is known that childbirth was complicated by massive bleeding. After childbirth, the patient began note hair loss, weight loss. Objectively observed: the woman is asthenic, the external genitalia are cylindrical, the uterine body is small, painless, the uterine appendages are not identified. What is the most likely diagnosis?

Первинна аменорея Primary amenorrhea

Синдром Штейна-Левенталя Stein-Leventhal syndrome

Пухлина гіпофіза (хвороба Іценка-Кушинга) Pituitary tumor (Itsenko-Cushing's disease)

Гіпофізарна аменорея (синдром Шихана) Pituitary amenorrhea (Sheehan syndrome)

Маткова вагітність Uterine pregnancy

198 / 1500
Пацієнтка віком 26 років, яка хворіє на ревматоїдний артрит впродовж 8 місяців, має набряки ліктьових, променево-зап'ясткових, колінних та гомілковостопних суглобів, ревматоїдні вузлики в ділянці ліктьових суглобів. У загальному аналізі крові спостерігається: ШОЕ - 57 мм/год., С-реактивний білок (+++). На рентгенограмах суглобів визначається виражений остеопороз. Що є базисною терапією в такій ситуації? A 26-year-old female patient, who has been suffering from rheumatoid arthritis for 8 months, has swelling of the elbow, wrist, knee and ankle joints, rheumatoid nodules in the area elbow joints. In the general blood test: ESR - 57 mm/h, C-reactive protein is determined on X-rays of the joints. What is the basic therapy in such a situation?

Диклофенак натрію Diclofenac sodium

Мелоксикам Meloxicam

Метилпреднізолон Methylprednisolone

Інфліксимаб Infliximab

Метотрексат Methotrexate

199 / 1500
До лікаря звернулися діти школи-інтернату за 4-5 годин після обіду із скаргами на нудоту, блювоту, холодний піт і невеликий біль в епігастрії. На обід діти їли картопляний суп на м'ясному бульйоні, млинці з м'ясом, компот із сухофрук-тів. Млинці начинялися вареним м'ясом, яке подрібнювалось на м'ясорубці працівником їдальні, хворим панарицієм, і термічній обробці не піддавались. Який найімовірніший діагноз у цьому разі? The children of the boarding school turned to the doctor 4-5 hours after lunch with complaints of nausea, vomiting, cold sweat and a slight pain in the epigastrium. For lunch, the children ate potato soup on meat broth, pancakes with dried fruits. The pancakes were filled with boiled meat, which was ground on a meat grinder by a canteen worker, and were not subjected to heat treatment. What is the most likely diagnosis in this if so?

Сальмонельоз Salmonellosis

Стафілококова інтоксикація Staphylococcal intoxication

Афлатоксикоз Aflatoxicosis

Ботулізм Botulism

Ешеріхіоз Escherichia

200 / 1500
Пацієнт скаржиться на запаморочення, блювання. Блювотні маси темного кольору. З анамнезу відомо, що пацієнт часто вживає алкоголь. Під час ЕФГС виявлено: вміст шлунку схожий на 'кавову гущу', в ділянці кардії чотири продольні тріщини слизової оболонки, з яких потрапляє кров у незначній кількості. Який найімовірніший діагноз? The patient complains of dizziness, vomiting. Vomiting masses are dark in color. It is known from the anamnesis that the patient often drinks alcohol. During EFGS it was found: the contents of the stomach are similar to 'coffee thick', in the area of ​​the cardia there are four longitudinal cracks of the mucous membrane, from which blood enters in small quantities. What is the most likely diagnosis?

Синдром Мелорі-Вейса Mallory-Weiss syndrome

Кровотеча з варикозно розширених вен шлунка Bleeding from varicose veins of the stomach

Синдром Золінгера-Елісона Zollinger-Ellison Syndrome

Ерозивний гастрит Erosive gastritis

Виразка кардіального відділу шлунка Gastric ulcer

201 / 1500
Чоловік 61 року звернувся до дільничного лікаря зі скаргами на напади стиснення за грудиною, які виникають під час ходьби до 200 м та зникають, якщо зупинитися. Уважає себе хворим близько року. Межі серця в нормі, тони помірно приглушені, ЧСС=Ps=76/хв., АТ - 130/80 мм рт. ст. Який найбільш імовириний діагноз у хворого? A 61-year-old man turned to the district doctor with complaints of chest compression attacks that occur while walking up to 200 m and disappear when he stops. He considers himself ill year. Heart rates are normal, heart rate=76/min., BP - 130/80 mmHg. What is the most likely diagnosis for the patient?

ІХС: Інфаркт міокарда без елевації сегменту ST CHD: Myocardial infarction without ST segment elevation

ІХС: Нестабільна стенокардія IHD: Unstable angina

ІХС: Стабільна стенокардія CHD: Stable angina

Хронічне обструктивне захворювання легень Chronic obstructive pulmonary disease

Тривожний розлад Anxiety disorder

202 / 1500
У вогнищах облисіння на голові волосся обламане на рівні 4-6 мм від поверхні шкіри, відзначається гіперемія, мукоподібне лущення. Імовірний попередній діагноз? In foci of baldness on the head, the hair is broken off at the level of 4-6 mm from the surface of the skin, there is hyperemia, muco-like desquamation. Possible preliminary diagnosis?

Дискоїдний червоний вовчак Discoid lupus erythematosus

Вторинний сифіліс Secondary syphilis

Мікроспорія волосистої частини голови Microsporia of scalp

Себорейне облисіння Seborrheic alopecia

Гніздова плішивість Nest baldness

203 / 1500
Головний лікар ЦПМСД має надати звіт щодо забезпеченості населення лікарями. За допомогою якого коефіціє-нту він може розрахувати даний показник? The chief physician of the CPMSD must provide a report on the provision of doctors to the population. By which coefficient can he calculate this indicator?

Спеціальної інтенсивності Special intensity

Екстенсивності Extensives

Наочності Visibility

Співвідношення Ratio

Інтенсивності Intensities

204 / 1500
Під час проведення комплексної оцінки мікроклімату приміщень необхідно визначити радіаційну температуру. Який прилад потрібно застосовувати? During the comprehensive assessment of the microclimate of the premises, it is necessary to determine the radiation temperature. What device should be used?

Кататермометр Catathermometer

Спиртовий термометр Alcohol thermometer

Термограф Thermograph

Кульовий термометр Ball thermometer

Ртутний термометр Mercury thermometer

205 / 1500
Під час профілактичного медичного огляду у одного з учнів технічного ліцею виявлені ознаки хейлозу, який проявляється в мацерації епітелію в місці змикання губ. Губи яскраво-червоного кольору з одиничними вертикально розташованими тріщинами, які вкриті кірочками червоно-бурого кольору. Дані клінічні прояви, найімовірніше, пов'язані з недостатнім надходженням з їжею в організм: During a preventive medical examination, one of the students of the technical lyceum showed signs of cheilosis, which manifests itself in the maceration of the epithelium in the place where the lips close. The lips are bright red with single vertical located cracks, which are covered with red-brown crusts. These clinical manifestations are most likely associated with insufficient intake of food into the body:

Тіаміну Thiamine

Аскорбінової кислоти Ascorbic acid

Кальциферолів Calciferol

Рибофлавіну Riboflavin

Ретинолу Retinol

206 / 1500
У шестирічної дитини, яка хворіє на екзантемну інфекцію, на 8-й день захворювання повторно підвищилася температура тіла до 39,6^oС, з'явився головний біль, блювання, спостерігалася хитка хода, скандована мова. Під час огляду виявлено залишкові елементи висипу (кірочки), атаксію, дискоординацію рухів, тремор кінцівок, горизонтальний ністагм, нестійкість у позі Ромберга. Вкажіть препарат вибору для лікування ускладення, що виникло у дитини. A six-year-old child suffering from an exanthem infection, on the 8th day of the disease, the body temperature rose again to 39.6^oС, a headache appeared, vomiting, wobbly gait, slurred speech were observed. During the examination, residual elements of the rash (crusts), ataxia, limb tremor, instability in Romberg's position were found. Specify the drug of choice for the treatment of the child's condition.

Занамівір Zanamivir

Варіцело-зостерний імуноглобулін Vicella-zoster immunoglobulin

Рибавірин Ribavirin

Ацикловір Acyclovir

Озельтамівір Oseltamivir

207 / 1500
Чоловік 23 років звернувся до поліклініки зі скаргами на головний біль, погіршення зору, задишку, одутлість обличчя, загальну слабкість, зменшення кількості виділеної за добу сечі та забарвлення її типу <<м'ясних зливів>>. Три тижні тому переніс ангіну. Об'єктивно встановлено: помірна блідість, обличчя одутле, повіки набряклі; серцева діяльність ритмічна, І тон на верхівці ослаблений, систолічний шум. Пульс - 66/хв. АТ - 175/105 мм рт. ст. Симптом Пастернацького слабко позитивний з обох боків. У сечі виявлено: білок - 1,48 г/л, еритроцити - 35-40 в п/з, змінені; лейкоцити - 8-10 в п/з. Який діагноз є найімовірнішим? A 23-year-old man came to the clinic with complaints of headache, visual impairment, shortness of breath, puffiness of the face, general weakness, decrease in the amount of urine released per day and the color of its type Three weeks ago, he suffered from angina: moderate pallor, swollen eyelids, rhythmic heart rate, systolic heart rate - 66/min. 175/105 mm Hg, weakly positive on both sides: protein - 1.48 g/l, erythrocytes - 8-10 g/l . What is the most likely diagnosis?

Амилоїдоз Amyloidosis

Есенціальна гіпертонія Essential hypertension

Інфекційно-алергічний міокардит Infectious-allergic myocarditis

Гострий гломерулонефрит Acute glomerulonephritis

Гострий пієлонефрит Acute pyelonephritis

208 / 1500
Хворий скаржиться на біль в горлі зліва, біль у лівому вусі, підвищення t^o до 39^oC, гугнявість голосу. Хворіє 5 днів. Виражений тризм, підвищена салівація. Голова нахилена до лівого плеча. Припухлість, гіперемія, інфільтрація лівої половини м'якого піднебіння. Защелепні лімфовузли зліва різко болючі при пальпації. Отоскопічна картина в нормі. Який найбільш імовірний діагноз? The patient complains of a sore throat on the left, pain in the left ear, an increase in t^o to 39^oC, hoarseness of the voice. He has been sick for 5 days. Pronounced trismus, increased salivation. The head is tilted to the left shoulder. Swelling, infiltration of the left half of the soft palate. The otoscopic picture is normal?

Паратонзиліт зліва Left paratonsillitis

Флегмона шиї зліва Phlegmon of the neck on the left

Заглотковий абсцес Pharyngeal abscess

Парафарингеальна флегмона Parapharyngeal phlegmon

Лівобічний паратонзилярний абсцес Left-sided paratonsillar abscess

209 / 1500
Жінка 42-х років звернулася до лікаря зі скаргами на м'язову слабкість у верхніх і нижніх кінцівках. Об'єктивно: у періорбітальних ділянках еритема з бузковим відтінком, макульозна еритема на розгинальних поверхнях пальців. В аналізі крові вия-влено суттєве підвищення рівня креатинфосфокінази. Яким є імовірний діа-гноз? A 42-year-old woman consulted a doctor with complaints of muscle weakness in the upper and lower limbs. Objectively: in the periorbital areas erythema with a lilac tint, macular erythema on the extensor surfaces of the fingers. A significant increase in the level of creatine phosphokinase was detected in the blood test. What is the probable diagnosis?

Гіпопаратиреоз Hypoparathyroidism

Хвороба Кушинга Cushing's disease

Дерматоміозит Dermatomyositis

Системна склеродермія Systemic scleroderma

Системний червоний вовчак Systemic lupus erythematosus

210 / 1500
Аналіз захворюваності жителів села, розташованого поблизу хімічного підприємства, показав, що протягом останніх років помітно зросла кількість захворювань на подагру і рак стравоходу. З яким забруднювачем навколишнього середовища можна пов'язати таку динаміку? An analysis of the morbidity of residents of a village located near a chemical plant showed that in recent years the number of diseases such as gout and esophageal cancer has increased significantly. What environmental pollutant can be associated with 'do such a dynamic?

Нікелем Nickel

Марганцем Manganese

Молібденом Molybdenum

Стронцієм Strontium

Ртуттю Mercury

211 / 1500
Пацієнт віком 30 років звернувся із скаргами на сильний нападоподібний біль у попереку справа, що віддає у пахову ділянку і внутрішню поверхню правого стегна, нудоту, здуття живота, блювання, порушення сечовиділення, що супроводжується різями і домішками крові у сечі. Об'єктивно спостерігається: пацієнт неспокійний, змінює положення в ліжку. З боку органів дихання і серцево-судинної системи патології не виявлено. Під час пальпації живота відмічається здуття, напруження м'язів і болючість у правому боці відповідно проекції правої нирки і за ходом правого сечоводу. Печінка і селезінка не збільшені. Симптоми запалення жовчного міхура негативні. Позитивний симптом Пастернацького справа. Який найімовірніший діагноз? A 30-year-old patient complained of severe attack-like pain in the lower back on the right, radiating to the inguinal area and the inner surface of the right thigh, nausea, abdominal distension, vomiting, violation of urination, accompanied by blood in the urine. Objectively observed: the patient changes the position in bed. On the part of the respiratory system and the cardiovascular system, bloating and muscle tension are noted pain in the right side of the right kidney and behind the right ureter. Symptoms of gallbladder inflammation are negative. What is the most likely diagnosis?

Сечокам'яна хвороба Urolithiasis

Туберкульоз нирки Kidney tuberculosis

Пухлина нирки Kidney tumor

Жовчокам'яна хвороба Cholelithiasis

Гломерулонефрит Glomerulonephritis

212 / 1500
У дівчинки 8 місяців під час огляду виявлений гучний інтенсивний систолічний шум над усією серцевою ділянкою з епіцентром у ІІІ-ІV міжребер'ї зліва від груднини та систолічне тремтіння; ІІ тон над легеневою артерією посиленої гучності. АТ - 110/70 мм рт. ст. Скарг немає. Під час рентгенологічного дослідження розміри і форма серцевої тіні не змінені. Який найімовірніший діагноз? In an 8-month-old girl, during the examination, a loud intense systolic murmur was detected over the entire cardiac region with an epicenter in the III-IV intercostal space to the left of the sternum and systolic tremor; II the tone over the pulmonary artery is 110/70 mmHg. No changes in the size and shape of the cardiac shadow?

Стеноз гирла легеневої артерії Stenosis of the mouth of the pulmonary artery

Дефект міжшлуночкової перетинки Defect of interventricular membrane

Відкрита артеріальна протока Open ductus arteriosus

Дефект міжпередсердної перетинки Atrial membrane defect

Коарктація аорти Coarctation of the aorta

213 / 1500
Чоловік 50 років скаржиться на наявність висипу на шкірі тулуба без суб'єктивних відчуттів, хворіє 3 місяці, не лікувався, висип прогресує. Об'єктивно встановлено: на шкірі тулуба дисемінована висипка у вигляді червоних негострозапальних лентикулярних та нумулярних папул із чіткими границями, укритих сріблястими лусочками, які легко знімаються при пошкрябуванні предметним склом, інтенсивніше пошкрябування призводить до появи крапкової кровотечі. Який імовірний діагноз? A 50-year-old man complains about the presence of a rash on the skin of the trunk without subjective sensations, he has been sick for 3 months, was not treated, the rash is progressing. Objectively established: on the skin body, a disseminated rash in the form of red non-acute inflammatory lenticular and nummular papules with clear borders, covered with silvery scales, which are easily removed when scraped with a glass slide, more intensive scraping leads to the appearance of spot bleeding. What is the probable diagnosis?

Псоріаз звичайний Psoriasis usual

Дерматит Dermatitis

Рожевий пітиріаз Pityriasis rosea

Екзема Eczema

Червоний плоский лишай Red lichen planus

214 / 1500
Жінці з терміном вагітності 34 тижні проведена доплерометрія пуповинного кровообігу - встановлений реверсний діа-столічний компонент. Об'єктивно: висота дна матки над лоном - 27 см, голівка плода рухома над входом в малий таз. Серцебиття плода - 136/хв. Піхвове дослідження: шийка матки закрита, довжиною 3 см. Якою має бути тактика акушера? A woman with a gestation period of 34 weeks underwent dopplerometry of the umbilical cord blood circulation - a reverse diastolic component was established. Objectively: the height of the bottom of the uterus above the womb is 27 cm, the head of the fetus moving over the entrance to the pelvis. Fetal heartbeat - 136/min. Vaginal examination: the cervix is ​​closed, 3 cm long. What should be the obstetrician's tactics?

Негайний кесарів розтин Immediate C-section

Родозбудження окситоцином Arousal by oxytocin

Біофізичний профіль плода Biophysical profile of the fetus

Ультразвукова фотометрія плода Fetal ultrasound photometry

Повторна доплерометрія наступної доби Repeated dopplerometry next day

215 / 1500
На підприємстві з виробництва синтетичних миючих речовин збільшилися випадки захворювання ринітами, вазоринітами, фарингітами, дерматитами. У повітрі робочої зони присутній пил миючих речовин. Фахівці вважають, що клінічні прояви й зростання захворюваності зумовлене впливом пилу. Які особливості дії пилу полягають в основі патогенезу? At the enterprise for the production of synthetic detergents, cases of rhinitis, vasorinitis, pharyngitis, and dermatitis have increased. There is dust from detergents in the air of the working area. Experts believe that the clinical manifestations and the increase in morbidity is due to the influence of dust. What features of the action of dust are the basis of pathogenesis?

Подразнююча дія Annoying action

Фіброгенна дія Fibrogenic action

Алергічна дія Allergic action

Канцерогенна дія Carcinogenic action

Загальнотоксична дія General toxic action

216 / 1500
У пацієнта, що хворіє на епілепсію, після самостійного припинення вживання протиепілептичних медикаментів раптово виник епілептичний статус генералізованих судомних нападів. Вкажіть препарати першої лінії для лікування цього стану. A patient with epilepsy suddenly developed status epilepticus with generalized seizures after self-discontinuation of antiepileptic drugs. Specify the first-line drugs for the treatment of this condition.

Габапентин, прегабалін, етосуксимід Gabapentin, Pregabalin, Ethosuximide

Левопромазин, клозапін, кветіапін Levopromazine, Clozapine, Quetiapine

Діазепам, лоразепам, мідазолам Diazepam, Lorazepam, Midazolam

Топірамат, окскарбазепін, карбамазепін Topiramate, oxcarbazepine, carbamazepine

Доксепін, амітриптилін, міансерин Doxepin, Amitriptyline, Mianserin

217 / 1500
Вагітна 22-х років звернулась до жіночої консультації зі скаргами на кровотечу зі статевих шляхів, що розпочалася 2 дні тому, сильну втому та запаморочення. Термін вагітності 13 тижнів. За день вона використала 6 гігієнічних прокладок. При фізикальному обстеженні: артеріальний тиск - 90/60 мм рт.ст., температура - 37,8^oC, пульс - 125/хв., частота дихання - 15/хв. Розпочата інфузійна терапія. При вагінальному дослідженні: кров у піхві, шийка матки згладжена та розкрита. На УЗД плідне яйце в порожнині матки, серцебиття і рухи плода не визначаються. Який з наступних кроків у веденні пацієнтки буде найбільш доречним? A 22-year-old pregnant woman came to the women's consultation with complaints of bleeding from the genital tract, which began 2 days ago, severe fatigue and dizziness. The pregnancy period is 13 weeks. During the day, she used 6 sanitary napkins.During physical examination: blood pressure - 90/60 mm Hg, temperature - 37.8°C, pulse - 15/min. Infusion therapy was started. During vaginal examination: blood in the vagina, the cervix is ​​smoothed and opened. On ultrasound, the fetal egg in the uterine cavity, the heartbeat and the movements of the fetus are not determined. Which of the following steps in the management of the patient will be most appropriate?

Магнію сульфат внутрішньовенно Magnesium sulfate intravenously

Метотрексат Methotrexate

Антибіотикотерапія Antibiotic therapy

Постільний режим та анальгетики Bed rest and analgesics

Дилатація та кюретаж матки Dilatation and curettage of the uterus

218 / 1500
На репрезентативній сукупності хворих на грип проведено вивчення зв'язку між температурою тіла і частотою пульсу. Розрахований коефіцієнт кореляції дорівнює +0,5. Яка характеристика сили і направленості зв'язку між ознаками, що вивчаються є найбільш точною? The relationship between body temperature and pulse rate was studied on a representative population of flu patients. The calculated correlation coefficient is +0.5. What is the characteristic of the strength and directionality of 'is the connection between the signs being studied the most accurate?

- -

Середній прямий Average straight

Слабкий прямий Weak direct

Сильний прямий Strong straight

Середній зворотній Average inverse

219 / 1500
Пацієнтка віком 38 років, друга вагітність терміном 23-24 тижні, хворіє на цукровий діабет 2 типу, з приводу чого отримує метформін в добовій дозі 2500 мг. Лабораторно виявлено: глікований гемоглобін (HbA_1c) - 7,2 %. Яка подальша тактика лікування цієї пацієнтки? 38-year-old female patient, second pregnancy at 23-24 weeks, suffers from type 2 diabetes, for which she receives metformin in a daily dose of 2500 mg. Laboratory revealed : glycated hemoglobin (HbA_1c) - 7.2%. What is the further treatment strategy for this patient?

Призначити інсулінотерапію та відмінити метформін Prescribe insulin therapy and cancel metformin

Збільшити дозу метформіну до 3000 мг/добу Increase metformin dose to 3000 mg/day

Залишити лікування без змін Leave treatment unchanged

Відмінити метформін і перевести на препарати сульфанілсечовини Cancel metformin and switch to sulfonylureas

Додати до метформіну аналоги ГПП-1 Add GLP-1 analogs to metformin

220 / 1500
У структурі населення N-ського регіону питома вага осіб віком від 0 до 14 років склала 31%, а питома вага осіб віком 50 років і старше склала 20%. Який склад населення найбільш влучно характеризує таку демографічну ситуацію? In the population structure of the N region, the share of people aged 0 to 14 was 31%, and the share of people aged 50 and over was 20%. What composition of the population most aptly characterizes such a demographic situation?

Стаціонарний склад населення Stationary composition of the population

Прогресивний склад населення Progressive composition of the population

Еміграція населення Population emigration

Регресивний склад населення Regressive composition of the population

Міграція населення Population migration

221 / 1500
Пацієнт віком 35 років четверту добу знаходиться у відділенні інтенсивної терапії після перенесеної кровотечі та геморагічного шоку. Добовий діурез - 50 мл. Другу добу відзначає блювання. ЦВТ - 159 мм вод. ст., під час аускультації в легенях останні 3 години відзначаються поодинокі вологі хрипи, ЧД - 32/хв. За результатами лабораторного аналізу крові виявлено: залишковий азот - 82 ммоль/л, К^+ - 7,1 ммоль/л, Cl^- - 78 ммоль/л, Na^+ - 130 ммоль/л, Ht - 0,32, Hb - 100г/л, креатинін крові - 0,9 ммоль/л. Яке ускладнення найімовірніше розвинулося у пацієнта? A 35-year-old patient is in the intensive care unit for the fourth day after suffering bleeding and hemorrhagic shock. Daily diuresis - 50 ml. Vomiting on the second day. CVT - 159 mm during auscultation in the lungs for the last 3 hours, single wet wheezes are noted, BH - 32/min. According to the results of the blood analysis: residual nitrogen - 82 mmol/l, K^+ - 7.1 mmol/l, Cl^- - 78 mmol/l, Na^+ - 130 mmol/l, Ht - 0.32, Hb - 100 g/l, blood creatinine - 0.9 mmol/l. What complication most likely developed in the patient?

Гостра серцева недостатність Acute heart failure

Гостра надниркова недостатність Acute adrenal insufficiency

Гостра ниркова недостатність Acute renal failure

Дегідратація у зв'язку з гіповолемією Dehydration due to hypovolemia

Пневмонія Pneumonia

222 / 1500
На 5-й день після пологів породілля скаржиться на біль в лівій молочній залозі, підвищення температури тіла до 38,1^oC. При огляді: молочна залоза збільшена, болюча при пальпації, є тріщини та набряк соска, гіперемія верхньо-зовнішнього квадранту залози. Яка профілактика ускладнення, що виникло? On the 5th day after childbirth, a woman in labor complains of pain in the left mammary gland, an increase in body temperature to 38.1^oC. On examination: the mammary gland is enlarged, painful on palpation, there are cracks and swelling of the nipple, hyperemia of the upper-external quadrant of the gland. What is the prevention of the complication that has arisen?

Годування не довше 10 хвилин через накладку Feeding no longer than 10 minutes through the pad

Припинення годування дитини при появі тріщин Stop feeding the baby when cracks appear

Постійне зціджування молочних залоз Constant breast pumping

Годування немовляти по годинах Feeding the baby by the hour

Годування дитини на вимогу, зціджування залишкового молока, попередження тріщин сосків Feeding a child on demand, expressing residual milk, preventing cracked nipples

223 / 1500
Під час скринінг-дослідження у двотижневого новонародженого було виявлено фенілкетонурію. Яке лікування необхідно призначити дитині для запобігання тяжким ускладненням у майбутньому? During the screening examination, phenylketonuria was detected in a two-week-old newborn. What treatment should be given to the child to prevent serious complications in the future?

Антибіотикотерапію Antibiotic therapy

Вітамінотерапію Vitamin therapy

Сонячні ванни Sunbaths

Гормональну терапію Hormone therapy

Спеціальну дієту Special diet

224 / 1500
Дитина 3-х місяців доставлена в лікарню зі скаргами на мляве смоктання та появу задишки, яка посилюється під час годування, неспокій. Симптоми з'явилися протягом останніх годин. Шкіра бліда, з попелястим відтінком, кінцівки холодні. Аксилярна температура - 36,8^oC. Частота дихання - 66/хв. Частоту серцевих скорочень підрахувати не вдає-ться. Пульс слабкий. Печінка пальпується на 4 см нижче реберної дуги. На ЕКГ: ЧСС - 230/хв., ритм правильний, зубець Р на Т, комплекси тривалістю 0,08 с. Який препарат вибору для надання невідкладної допомоги даній дитині? A 3-month-old baby was brought to the hospital with complaints of sluggish sucking and shortness of breath, which worsens during feeding, restlessness. Symptoms appeared during the last hours. The skin is ashen, the axillary temperature is 66/min. The pulse is weak at 4 cm below the costal arch. Heart rate - 230/min., rhythm is correct, wave P on T, complexes lasting 0.08 s. What is the drug of choice for providing emergency care to this child?

Аденозин Adenosine

Пропранолол Propranolol

Лідокаїн Lidocaine

Верапаміл Verapamil

Дигоксин Digoxin

225 / 1500
У хлопця 18 років за 2 тижня після ангіни з'явилися набряки під очима, на гомілках, підвищився артеріальний тиск, зменшилась кількість сечі і змінився її колір. Загальний аналіз сечі виявив: білок - 1,9 г/л, еритроцити - покривають усе поле зору, лейкоцити - 10-12 у полі зору. Для якого захворювання характерні такі симптоми? 2 weeks after angina, an 18-year-old boy developed swelling under his eyes, on his lower legs, increased blood pressure, decreased the amount of urine and changed its color. General analysis urine showed: protein - 1.9 g/l, erythrocytes - covering the entire field of vision, leukocytes - 10-12 in the field of vision. What disease is characterized by such symptoms?

Амілоїдоз нирок Kidney amyloidosis

Гострий пієлонефрит Acute pyelonephritis

Токсичне ураження нирок Toxic kidney damage

Гострий гломерулонефрит Acute glomerulonephritis

Тубуло-інтерстиційний нефрит Tubulo-interstitial nephritis

226 / 1500
Роділля 28-ми років доставлена до пологового будинку з бурхливою пологовою діяльністю. Пологи перші. Розміри таза: 23-25-28-18 см. Ознака Генкеля-Вастена позитивна. Роділля збуджена, живіт напружений, болісний в нижніх відділах. Контракційне кільце на рівні пупка, розташоване косо. Голівка плода притиснута до входу у малий таз. Серцебиття плоду - 140/хв. Яке ускладнення виникло у роділлі? A 28-year-old woman in labor was brought to the maternity hospital with violent labor. First delivery. Pelvic dimensions: 23-25-28-18 cm. Henkel-Wasten's sign Labor is excited, the abdomen is painful. The fetal head is pressed against the pelvic inlet.

Надмірна пологова діяльність Excessive reproductive activity

Загроза розриву матки Threat of uterine rupture

Завершений розрив матки Completed uterine rupture

Дискоординація пологової діяльності Discoordination of labor

Розпочатий розрив матки Beginning uterine rupture

227 / 1500
У двомісячної дитини спостерігаються судоми, рецидивуючі вірусно-бактеріальні інфекції верхніх дихальних шляхів. Виявлено деформований череп, гіпоплазію тимусу та паращитовидних залоз. Під час лабораторного імунологічного обстеження виявлено: лімфоцитопенія, зниження рівня та проліферативної відповіді Т-лімфоцитів, нормальний рівень імуноглобулінів. Який найімовірніший діагноз? A two-month-old child has convulsions, recurrent viral and bacterial infections of the upper respiratory tract. A deformed skull, hypoplasia of the thymus and parathyroid glands were detected. During the laboratory immunological examination, lymphocytopenia was detected , a decrease in the level and proliferative response of T-lymphocytes, a normal level of immunoglobulins. What is the most likely diagnosis?

Первинний імунодефіцит, синдром Чедіака-Хігасі Primary immunodeficiency, Chediak-Higashi syndrome

- -

Первинний імунодефіцит, хвороба Брутона Primary immunodeficiency, Bruton's disease

Загальний варіабельний імунодефіцит General variable immunodeficiency

Первинний імуннодефіцит, синдром Ді-Джорджі Primary immunodeficiency, Di-Georgi syndrome

228 / 1500
Жінка 32-х років скаржиться на посилення спастичного болю внизу живота після психоемоційного напруження. Випорожнення кишечника інтермітуючі: 2-3 випорожнення після пробудження чергуються із закрепами протягом 1-2 днів впродовж 6 місяців. Об'єктивно: маса тіла збережена, помірний біль під час пальпації сигмоподібної кишки. Hb- 130 г/л, лейкоцити - 5,2 Г/л, швидкість осідання еритроцитів - 9 мм/год. Ректороманоскопічне дослідження болісне через спастичний стан кишечника, його слизова оболонка не змінена. В просвіті кишечника багато слизу. Яке захворювання найбільш імовірне у даної пацієнтки? A 32-year-old woman complains of increased spastic pain in the lower abdomen after psycho-emotional stress. Intermittent bowel movements: 2-3 bowel movements after waking up alternate with constipation for 1-2 days for 6 months. Body weight is preserved, moderate pain during palpation of the sigmoid intestine. Hb - 5.2 G/l, erythrocyte sedimentation rate - 9 mm/h spastic condition of the intestine, its mucous membrane is not changed. There is a lot of mucus in the intestinal lumen. What disease is most likely in this patient?

Хвороба Крона Crohn's disease

Гостра ішемія кишківника Acute intestinal ischemia

Синдром мальабсорбції Malabsorption syndrome

Неспецифічний виразковий коліт Nonspecific ulcerative colitis

Синдром подразненого кишечника Irritable bowel syndrome

229 / 1500
Пацієнтка віком 12 років скаржиться на слабкість, запаморочення, головний біль, підвищення температури тіла до 38^oС. Об'єктивно спостерігається: температура тіла 37,8^oС, слизові та шкіра бліді, зів без змін. Пальпуються збільшені до 2-х см підщелепні та шийні лімфовузли, щільні, неболючі. Патологічних змін внутрішніх органів не виявлено. У загальному аналізі крові виявлено: еритроцити - 2,8·10^12/л, гемоглобін - 85 г/л, кольоровий показник - 0,9, лейкоцити - 10·10^9/л, еозинофіли - 0%, паличкоядерні - 1%, сегментоядерні - 8%, лімфоцити - 47%, ретикулоцити - 0,5%, тромбоцити - 60·10^9/л, бластні клітини - 44%. Який найімовірніший діагноз? A 12-year-old patient complains of weakness, dizziness, headache, an increase in body temperature up to 38°C. Objectively observed: body temperature 37.8°C , mucous membranes and skin are unchanged. Submaxillary and cervical lymph nodes are palpable, painless. No pathological changes in internal organs are detected: erythrocytes - 2.8·10^12/l , hemoglobin - 85 g/l, color index - 0.9, leukocytes - 10·10^9/l, eosinophils - 0%, rod-nuclear - 1%, segmentonuclear - 8%, lymphocytes - 47%, reticulocytes - 0.5 %, platelets - 60·10^9/l, blast cells - 44%. What is the most likely diagnosis?

Інфекційний мононуклеоз Infectious mononucleosis

Гострий еритромієлоз Acute erythromyelosis

Гострий лейкоз Acute leukemia

Лімфогрануломатоз Lymphogranulomatosis

Хронічний лімфолейкоз Chronic lymphocytic leukemia

230 / 1500
Жінка оперована з приводу дифузно-токсичного зобу ІІ ступеня. За 12 годин після операції скаржиться на захриплість та втрату голосу, утруднення дихання, задишку, тривогу. Післяопераційна рана без особливостей. Яке ускладення виникло у пацієнтки? A woman was operated on for a diffuse-toxic goiter of the II degree. 12 hours after the operation, she complains of hoarseness and loss of voice, difficulty breathing, shortness of breath, anxiety. Postoperative wound without features. What condition did the patient have?

Пошкодження трахеї Damage to the trachea

Тиреотоксичний криз Thyrotoxic crisis

Гіпопаратиреоз Hypoparathyroidism

Парез зворотніх гортанних нервів Reverse laryngeal nerve paresis

Післяопераційна кровотеча Postoperative bleeding

231 / 1500
Хлопчик 3 років доставлений до стаціонару в важкому стані. Об'єктивно: сомнолентність, гіперрефлексія, судоми, гіперестезія, невпинне блювання, температура тіла - 39,9^oC, ЧСС - 160/хв., АТ - 80/40 мм рт. ст. Яке дослідження слід провести насамперед? A 3-year-old boy was brought to the hospital in serious condition. Objectively: somnolence, hyperreflexia, convulsions, hyperesthesia, incessant vomiting, body temperature - 39.9^oC , heart rate - 160/min., blood pressure - 80/40 mm Hg. What research should be done first?

Реоенцефалографія Rheoencephalography

Рентгенографія черепа X-ray skull

Люмбальна пункція Lumbar puncture

Комп'ютерна томографія мозку Computed tomography of the brain

Ехоенцефалографія Echoencephalography

232 / 1500
Чоловік віком 40 років скаржиться на ниючий біль у м'язах спини, поперековій ділянці, ногах, плечових і кульшових суглобах, погіршення зору, прискорене серцебиття. Об'єктивно спостерігаються явища увеїту. Рентгенографічно виявлено поодинокі синдесмофіти між тілами хребців і розмитість контурів крижово–клубових суглобів. Під час лабораторного обстеження виявлено антитіла до антигенів HLA–B27, анемію, ШОЕ - 28 мм/год. Яке захворювання зумовлює таку картину? A 40-year-old man complains of aching pain in the muscles of the back, lumbar region, legs, shoulder and hip joints, impaired vision, rapid heartbeat. Objectively the phenomena of uveitis are observed. Single syndesmophytes between the vertebral bodies and blurring of the contours of the sacroiliac joints are detected. During the laboratory examination, antibodies to HLA-B27 antigens, ESR - 28 mm/h are found. What disease causes such a picture?

Синдром Рейтера Reiter syndrome

Деформуючий спондилоартроз Deforming spondyloarthrosis

Системний червоний вовчак Systemic lupus erythematosus

Ревматоїдний артрит Rheumatoid arthritis

Анкілозуючий спондиліт Ankylosing spondylitis

233 / 1500
Дитині 8 місяців. Тиждень тому перенесла ГРВІ. Фізичний та нервово-психічний розвиток відповідають вікові. Скарги на млявість, зниження апетиту, немотивоване повторне блювання, підвищення температури протягом останньої доби до 38^oC, блідість, часте сечовипускання малими порціями. Тахікардія. В аналізі сечі виявлено: білок - 0,099%, лейкоцити - 15-20 в полі зору, бактеріурія - +++, слиз - +++. Яке захворювання можна припустити? The child is 8 months old. A week ago he suffered SARS. Physical and neuropsychological development correspond to the age. Complaints of lethargy, decreased appetite, unmotivated repeated vomiting, increased temperature during the last days up to 38°C, frequent urination in small portions. Urinalysis revealed: protein - 15-20 in the field of vision, bacteriuria - +++. What disease can be assumed ?

Гострий гломерулонефрит Acute glomerulonephritis

Фосфат-діабет Phosphate-Diabetes

Дефекти догляду Care Defects

Гострий пієлонефрит Acute pyelonephritis

Дисметаболічна нефропатія Dysmetabolic nephropathy

234 / 1500
Пацієнт віком 29 років хворіє гормонозалежною бронхіальною астмою. Під час флюорографії в С2 правої легені виявлено округлу тінь з чіткими рівними контурами, середньої інтенсивності. Навколо неї кілька поліморфних вогнищевих тіней, у корені - кальцинат. Під час обстеження спостерігається: перкуторний звук над легенями з коробковим відтінком, вислуховуються розсіяні сухі хрипи. Аналіз крові без змін. Реакція на пробу Манту з 2 ТО ППД-Л - папула 22 мм. Який найімовірніший діагноз? A 29-year-old patient suffers from hormone-dependent bronchial asthma. During fluorography, a rounded shadow with clear even contours of medium intensity was detected in C2 of the right lung. Several polymorphic focal shadows were found around it , in the root - calcification. During the examination, there is a percussion sound over the lungs, scattered dry rales are heard. The reaction to the Mantoux test is 22 mm. What is the most likely diagnosis?

Периферичний рак Peripheral cancer

Еозинофільний інфільтрат Eosinophilic infiltrate

Аспергільома Aspergilloma

Пневмонія Pneumonia

Туберкульома Tuberculoma

235 / 1500
Пацієнтка віком 30 років на п'яту добу після фізіологічних пологів скаржиться на нагрубання лівої молочної залози, біль, почервоніння шкіри, підвищення температури тіла до 38^oС. Об'єктивно спостерігається: ліва молочна залоза збільшена у розмірі, шкіра червоного кольору, в верхньо-зовнішньому квадранті з ціанозом, пальпується без чітких меж інфільтрат, різко болісний. Зціджування молока не приносить полегшення. Який найімовірніший діагноз? On the fifth day after physiological childbirth, a 30-year-old patient complains of swelling of the left mammary gland, pain, redness of the skin, an increase in body temperature up to 38°C. 'objectively observed: the left mammary gland is enlarged, the skin is red, in the upper-external quadrant, the infiltrate is palpable, sharply painful. Expressing milk does not bring relief. What is the most likely diagnosis?

Бешиха Beshikha

Інфікована кіста молочної залози Infected breast cyst

Рак молочної залози Breast cancer

Гострий інфільтративний мастит Acute infiltrative mastitis

Лактостаз Lactostasis

236 / 1500
Чоловік 37 років скаржиться на гострий біль у лівій половині грудної клітки, задишку, що посилюється під час будь-яких рухів. Захворів раптово після значного фізичного навантаження. Під час обстеження встановлено: помірний ціаноз обличчя, ліва половина грудної клітки відстає в акті дихання. Перкуторно виявлено: зліва - тимпаніт, дихання ослаблене. ЧД - 24/хв. Тони серця ослаблені. ЧСС - 90/хв. На рентгенограмі видно лінія вісцеральної плеври. Назовні від неї легеневий малюнок відсутній. Який найімовірніший діагноз? A 37-year-old man complains of sharp pain in the left half of the chest, shortness of breath that worsens during any movement. He fell ill suddenly after significant physical exertion. During examination revealed: moderate cyanosis of the chest. Percussion revealed: heart rate - 24/min. Visceral pleura line she has no lung pattern. What is the most likely diagnosis?

Спонтанний пневмоторакс Spontaneous pneumothorax

Тромбоемболія легеневої артерії Thromboembolism of the pulmonary artery

Лівосторонній ексудативний плеврит Left-sided exudative pleurisy

Лівостороння пневмонія Left-sided pneumonia

Інфаркт міокарда Myocardial infarction

237 / 1500
Чоловік 70 років за 2 тижні до госпіталізації у стаціонар спостерігав підвищення температури до 38^oС. З діагнозом пневмонія був направлений до лікарні. Під час огляду встановлено: блідість шкіри, тахікардія. У нижніх відділах легенів - дрібнопухирчаті хрипи. У V точці - короткий протодіастолічний шум. АТ - 140/40 мм рт. ст., пальпується нижній край селезінки. Hb - 40 г/л., еритроцити - 2,5·10^12/л, лейкоцити - 12·10^9/л, ШОЕ - 35 мм/год. На ЕКГ спостерігається: депресія ST в V 5,6. У сечі - протеїнурія. Який найімовірніший діагноз? 2 weeks before hospitalization, a 70-year-old man observed an increase in temperature to 38°C. He was referred to the hospital with a diagnosis of pneumonia. During the examination, it was found: pale skin , tachycardia. In the lower parts of the lungs - a short protodiastolic murmur. Hb - 40 g/l ^12/l, leukocytes - 12·10^9/l, ESR - 35 mm/h. On the ECG, there is depression in V 5.6. What is the most likely diagnosis?

Хронічний мієлолейкоз Chronic myelogenous leukemia

Гострий міокардит Acute myocarditis

Ревматична вада серця Rheumatic heart disease

Гострий перикардит Acute pericarditis

Бактеріальний ендокардит Bacterial endocarditis

238 / 1500
Лікар оглядає недоношену новонароджену дитину. Об'єктивно спостерігалося одиничне апное тривалістю до 5 секунд. Якою основною функціональною особливістю дихальної системи можна пояснити приступи апное у недоношеної дитини? The doctor examines a premature newborn baby. A single apnea lasting up to 5 seconds was objectively observed. What main functional feature of the respiratory system can explain apnea attacks in a premature baby?

Схильність слизових оболонок до набряку та гіперсекреції Propensity of mucous membranes to edema and hypersecretion

Функціональна незрілість дихального центру Functional immaturity of the respiratory center

М'якість хрящів трахеї та бронхів Softness of tracheal and bronchial cartilages

Вузкість просвіту дихальних шляхів Narrowness of the airway

Недостатня дренажна та захисна функція бронхів Insufficient drainage and protective function of the bronchi

239 / 1500
Пацієнта віком 38 років турбує біль у ділянці плеснофалангових суглобів 1-2 пальців правої стопи. Під час огляду виявлено: шкіра над ураженими суглобами багряно-синюшного кольору, на дотик гаряча, у ділянці вушних раковин наявні вузликові утворення, вкриті тонкою блискучою шкірою. Який препарат для уратзнижувальної терапії потрібно призначити пацієнту? A 38-year-old patient is bothered by pain in the area of ​​the metatarsophalangeal joints of 1-2 toes of the right foot. During the examination, the skin over the affected joints is purple-bluish to the touch hot, there are nodular formations in the area of ​​the auricles, covered with thin shiny skin. What drug should be prescribed for urate-lowering therapy?

Метотрексат Methotrexate

Німесулід Nimesulide

Колхіцин Colchicine

Алопуринол Allopurinol

Фебуксостат Febuxostat

240 / 1500
До лікаря звернулася жінка 74 років зі скаргами на біль у пахвинній ділянці справа. Ці симптоми виникли раптово близько 2 годин тому. З анамнезу відомо: пацієнтка каже, що ці симптоми вже були у неї близько 3 тижнів тому, але самі зникли, після того як хвора лягла. Об'єктивно спостерігається: нижче пупартової зв'язки пальпується щільне, напружене утворення до 3,5 см у діаметрі, різко болюче, симптом <<кашльового поштовху>> негативний. Який діагноз найімовірніший? A 74-year-old woman came to the doctor with complaints of pain in the inguinal region on the right. These symptoms appeared suddenly about 2 hours ago. It is known from the anamnesis: the patient says that these she already had symptoms about 3 weeks ago, but they disappeared after the patient lay down. Objectively observed: a dense, tense mass up to 3.5 cm in diameter, sharply painful, is palpable below the Pupart ligament. shock>> negative. What is the most likely diagnosis?

Набута невправима пахова грижа Acquired inoperable inguinal hernia

Набута защемлена пахова грижа Acquired inguinal hernia

Паховий лімфаденіт Inguinal lymphadenitis

Набута невправима стегнова грижа Acquired inoperable femoral hernia

Набута защемлена стегнова грижа Acquired femoral hernia

241 / 1500
Серед населення, що мешкає поблизу підприємства з виробництва пестицидів, динамічно підвищується рівень вроджених вад розвитку, що виражаються в центральному паралічі, ідіотії та сліпоті новонароджених. Сполуки якої хімічної речовини можуть зумовити розвиток цієї патології? Among the population living near a pesticide production plant, the level of congenital malformations, expressed in central paralysis, idiocy and blindness in newborns, is dynamically increasing. Compounds of which chemical substance can cause the development of this pathology?

Кадмій Cadmium

Хром Chrome

Стронцій Strontium

Ртуть Mercury

Залізо Iron

242 / 1500
Чоловік 32-х років звернувся до сімейного лікаря зі скаргами на нав'язливі спогади про автомобільну аварію, яка була 3 місяці тому, порушення нічного сну, тривогу, зниження настрою і апетиту, загальну слабкість і спустошеність. Що найбільш доцільно призначити? A 32-year-old man presented to his family doctor with complaints of intrusive memories of a car accident 3 months ago, nighttime sleep disturbances, anxiety, decreased mood and appetite, general weakness and desolation. What is the most appropriate to prescribe?

Літію карбонат Lithium carbonate

Сертралін Sertraline

Амітриптилін Amitriptyline

Карбамазепін Carbamazepine

Хлорпромазин Chlorpromazine

243 / 1500
Хлопець 27-ми років під час купання в річці раптово знепритомнів та зник з поверхні води. Через 2 хвилини був доставлений до берега. Об'єктивно: непритомний, шкіра вираженого синюшного кольору, пінисті виділення з рота та носа, дихання та пульс на сонних артеріях відсутні. Який вид допомоги необхідно надати в цьому випадку? A 27-year-old boy while swimming in the river suddenly fainted and disappeared from the surface of the water. After 2 minutes, he was brought to the shore. Objectively: unconscious, skin pronounced bluish color, foamy discharge from the mouth and nose, breathing and pulse on the carotid arteries are absent. What kind of help should be provided in this case?

Паліативна медична допомога Palliative medical care

Екстрена медична допомога Emergency medical assistance

Вторинна медична допомога Secondary medical care

Третинна медична допомога Tertiary medical care

Первинна медична допомога Primary medical care

244 / 1500
Пацієнт віком 56 років скаржиться на зниження апетиту, слабкість, серцебиття, біль та відчуття печіння язика, відчуття важкості в надчеревній ділянці та почуття оніміння кінцівок. Об'єктивно спостерігається: блідність шкіри з лимонним відтінком, глосит Гантера, збільшення печінки, селезінки. У загальному аналізі крові виявлено: еритроцити - 2,8·10^12/л, гемоглобін - 100 г/л, кольоровий показник - 1,2. Еритроцити великі, часто овальні з тільцями Жоллі та кільцями Кебота. Який найімовірніший діагноз? A 56-year-old patient complains of decreased appetite, weakness, palpitations, pain and a burning sensation in the tongue, a feeling of heaviness in the epigastric area, and a feeling of numbness in the limbs. Objectively observed : pallor of the skin with a lemon hue, Ganter's glossitis, enlargement of the liver, spleen. In the general blood analysis, erythrocytes - 2.8·10^12/l, hemoglobin - 100 g/l, erythrocytes are large. often oval with Jolly bodies and Cabot rings. What is the most likely diagnosis?

Апластична анемія Aplastic anemia

Хронічний гепатит Chronic hepatitis

В_12(фолієво)-дефіцитна анемія B_12 (folic) deficiency anemia

Гемолітична анемія Hemolytic anemia

Залізодефіцитна анемія Iron deficiency anemia

245 / 1500
Жінка 35-ти років звернулася до лікаря зі скаргами на зростаючий біль під час менструації протягом 1-го року. Тривалість менструального циклу 28 днів. 2 роки тому була проведена лапароскопічна перев'язка маткових труб. При пальпації матка збільшена до 8 тижнів вагітності, м'якої консистенції, болюча. Придатки з обох сторін не збільшені, при пальпації безболісні. Який діагноз є найбільш імовірним? A 35-year-old woman consulted a doctor with complaints of increasing pain during menstruation during the 1st year. The duration of the menstrual cycle is 28 days. 2 years ago she was laparoscopic tubal ligation. On palpation, the uterus is soft, painful. On palpation, the appendages are painless. What is the most likely diagnosis?

Зовнішній ендометріоз External endometriosis

Полікістоз яєчників Polycystic ovary

Позаматкова вагітність Ectopic pregnancy

Гідросальпінкс Hydrosalpinx

Аденоміоз Adenomyosis

246 / 1500
До пологового відділення шпиталізовано жінку, яка народжує вперше, зі скаргами на перейми нерегулярного характеру та кров'янисті виділення зі статевих шляхів. Термін вагітності 39 тижнів. Під час зовнішнього акушерського дослідження встановлено: положення плода повздовжнє, головне передлежання. Визначається гіпертонус матки. Під час внутрішнього акушерського дослідження спостерігається: шийка матки довжиною 1 см, цервікальний канал відкритий на 2 см. Передлежить голівка плода, притиснута до входу в малий таз. Виділення кров'янисті, зі згортками. Серцебиття плода аритмічне, 80-100/хв. Яке ускладнення виникло у роділлі? A woman giving birth for the first time was hospitalized in the maternity ward with complaints of irregular periods and bleeding from the genital tract. The term of pregnancy is 39 weeks. During external Obstetric examination revealed: the position of the fetus is longitudinal, the uterus is hypertonic. During the internal obstetric examination, the cervix is ​​1 cm long. The fetal head is pressed against the entrance to the pelvis , with contractions. The heartbeat of the fetus is arrhythmic, 80-100/min. What complication occurred during childbirth?

Розрив матки Rupture of uterus

Передчасне відшарування плаценти Premature placental abruption

Розрив шийки матки Rupture of the cervix

Крайове передлежання плаценти Marginal placenta previa

Повне передлежання плаценти Complete placenta previa

247 / 1500
У чоловіка на різних ділянках шкірного покриву, переважно симетрично, розташовані депігментовані плями, що не лущаться, різного розміру й обрисів. Частина волосся на них знебарвлена. Захворювання почалося в дитинстві. Появі плям не передували будь-які елементи висипки. Який імовірний діагноз? The man has depigmented spots that do not peel, of different sizes and outlines, on different areas of the skin, mostly symmetrically. Some of the hair on them is discolored. The disease began in childhood. The appearance of spots was not preceded by any elements of the rash. What is the probable diagnosis?

Різнокольоровий лишай Multicolored lichen

Альбінізм Albinism

Вітиліго Vitiligo

Псоріаз Psoriasis

Рожевий лишай Pink lichen

248 / 1500
Хлопчик 5-ти років прийшов з матір'ю на прийом до лікаря зі скаргами на припухлість обличчя, біль у вухах та підвищену температуру протягом останніх 5 днів. Через релігійні переконання матері вакцинація дитини неповна. Під час фізикального обстеження виявлено двосторонній, болючий набряк у ділянці жувального м'яза та гарячку 39,3^oC. Що з перерахованого буде найбільш доцільно призначити цьому паціє-нту? A 5-year-old boy came with his mother to a doctor's appointment with complaints of facial swelling, ear pain, and high temperature for the past 5 days. Due to religious the mother's conviction that the child's vaccination is incomplete. During the physical examination, bilateral, painful swelling in the area of ​​the masticatory muscle and a fever of 39.3°C were detected. Which of the following would be most appropriate to prescribe for this patient?

Цефтриаксон Ceftriaxone

Парацетамол Paracetamol

Пеніцилін Penicillin

Імуноглобулін людини нормальний Human immunoglobulin is normal

Ацикловір Acyclovir

249 / 1500
Під час огляду новонародженого, який народився в сідничному передлежанні, педіатр установив обмеження пасивного відведення правого стегна і клацання у разі його відведення, асиметрію пахових і стегнових сідничних складок. Яка ознака достовірно свідчить про наявність вродженого вивиху стегна у дитини? During the examination of a newborn who was born in the breech presentation, the pediatrician established a limitation of passive abduction of the right hip and clicking in the event of its abduction, asymmetry of the inguinal and femoral buttock folds. Which does the sign reliably indicate the presence of congenital dislocation of the hip in the child?

Клацання у разі відведення стегна Click when abducting hip

Народження в сідничному передлежа-нні Birth in breech presentation

Достовірної ознаки немає There is no valid sign

Асиметрія пахових і стегнових сіднич-них складок Asymmetry of inguinal and femoral gluteal folds

Обмеження пасивного відведення стегна Limit passive abduction of hip

250 / 1500
Чоловік скаржиться на болючу припухлість у ділянці підборіддя, загальне нездужання, головний біль. У цій ділянці виявлено гостро запалений щільний вузол конусоподібної форми. Шкіра над ним напружена, червона. Усередині вузла є виразка з прямовисними краями і некротичним стрижнем брудно-зеленого кольору. Підщелепні лімфатичні вузли справа збільшені і болючі. Який діагноз найімовірніший? A man complains of painful swelling in the chin area, general malaise, headache. In this area, an acutely inflamed, dense, cone-shaped node was found. The skin above it is tense, red. Inside the node there is an ulcer with vertical edges and a necrotic rod of dirty green color. The submandibular lymph nodes on the right are enlarged and painful. What is the most likely diagnosis?

Третинний сифіліс (гумозний) Tertiary syphilis (humous)

Туберкульоз Tuberculosis

Глибока трихофітія Deep ringworm

Карбункул Carbuncle

Фурункул Furnish

251 / 1500
При вивченні середнього рівня та характеру різноманітності деяких лабораторних показників отримані такі дані: для загального білку крові - середнє квадратичне відхилення pm4 г/л, коефіцієнт варіації - 6%; для швидкість осідання еритроцитів відповідно pm2 мм/год, 23%. Яка з ознак, що вивчаються, є найбільш різноманітною? When studying the average level and nature of the diversity of some laboratory indicators, the following data were obtained: for total blood protein - mean square deviation of pm4 g/l, coefficient of variation - 6%; for the sedimentation rate of erythrocytes, respectively, pm2 mm/h, 23%. Which of the studied features is the most diverse?

Для вивчення різноманітності потрібні додаткові розрахунки Examination of diversity requires additional calculations

Загальний білок сироватки крові Total serum protein

Швидкість осідання еритроцитів (ШОЕ) Erythrocyte Sedimentation Rate (ESR)

Відмінності в різноманітності ознак відсутні There are no differences in the diversity of features

Для вивчення різноманітності потрібні додаткові дослідження More research is needed to study diversity

252 / 1500
Пацієнтка 48 років із матковою кровотечею. В анамнезі вказано порушення менструального циклу впродовж року. Під час огляду шийки матки патології не виявлено. Бімануально встановлено: матка нормальних розмірів, неболюча, рухома, придатки без особливостей. Виділення кров'янисті, рясні. Що треба виконати на цьому етапі насамперед? A 48-year-old female patient with uterine bleeding. The anamnesis indicated a violation of the menstrual cycle for a year. During the examination of the cervix, no pathology was found. It was established bimanually: the uterus is of normal size, painless , mobile, appendages without features. Bloody discharge, abundant. What should be done at this stage first?

Гістеректомію Hysterectomy

Уведення вітаміну К Vitamin K administration

Гормональну терапію Hormone therapy

Вишкрібання стінок матки Scraping the walls of the uterus

Уведення окситоцину Oxytocin administration

253 / 1500
Пацієнтка на 3-тю добу після штучного аборту госпіталізована до гінекологічного відділення у тяжкому стані з симптомами інтоксикації, болем у животі, гнійними виділеннями з піхви. Об'єктивно: стан жінки тяжкий, температура тіла - 38,8^oC, пульс - 100/хв., АТ- 110/70 мм рт.ст., матка м'яка, дно її на рівні пупка, симптоми подразнення очеревини позитивні. Який найбільш імовірний діагноз? On the 3rd day after the artificial abortion, the patient was hospitalized in the gynecological department in a serious condition with symptoms of intoxication, abdominal pain, purulent discharge from the vagina. Objectively: the woman's condition is severe, body temperature - 38.8°C, pulse - 100/min, blood pressure - 110/70 mm Hg, the uterus is soft, the bottom is at the level of the navel, the symptoms of peritoneal irritation are positive diagnosis?

Гострий метроендометрит Acute metroendometritis

Гострий гнійний сальпінгоофорит Acute purulent salpingo-oophoritis

Пельвіоперитоніт Pelvioperitonitis

Перфорація матки Uterine perforation

Позаматкова вагітність Ectopic pregnancy

254 / 1500
Чоловіку 42-х років виконано геміти-реоїдектомію з приводу вузлового еути-реоїдного зоба. Призначення якого лікування найбільш імовірно допоможе запобігти рецидиву захворювання? A 42-year-old man underwent hemit-rheoidectomy for a nodular euthy-rheoid goiter. Which treatment is most likely to help prevent recurrence of the disease?

L-тироксин L-thyroxine

Тиротропін Thyrotropin

Радіоактивний йод Radioactive iodine

Мерказоліл Mercazolil

Антиструмін Anticurrent

255 / 1500
Мати прийшла на прийом до педіатра зробити своїй 2-місячній дитині планові щеплення. Для зручності, вона бажає аби дитині було введено комбіновану 6-компонентну вакцину, що містить антигени усіх збудників, проти яких рекомендовано вакцинуватися у цьому віці. Під час збору анамнезу вона зазначила, що іноді після пробудження її дитина починає раптово напружуватися, згинати голівку, ручки та ніжки. Зазвичай такі напади тривають не більше декількох секунд з інтервалами ''спо-кою'' у 10 секунд. Останній напад був декілька днів тому. Температура тіла - 37,2^oC , частота дихання - 25/хв., пульс - 104/хв., артеріальний тиск - 100/70 мм рт.ст. Яке рішення має прийняти лікар щодо подальшої імунізації цієї дитини? A mother came to the pediatrician's appointment to give her 2-month-old child routine vaccinations. For convenience, she wants the child to be given a combined 6-component vaccine containing antigens of all the pathogens that are recommended to be vaccinated at this age. During the anamnesis, she noted that sometimes after waking up, her child begins to tense up, bend the head, arms and legs. Usually such attacks last no more than a few seconds with 'calm' intervals '' in 10 seconds. The last attack was several days ago. Body temperature - 25/min., pulse - 100/70 mm Hg should the doctor accept regarding further immunization of this child?

Провести вакцинацію за схемою АДП+поліо+гепатит B+Hib-інфекція Vaccinate according to the scheme ADP+polio+hepatitis B+Hib-infection

Введення будь-яких вакцин на даний момент протипоказано Introduction of any vaccines is currently contraindicated

Провести вакцинацію за схемою АКДП+гепатит В+Hib-інфекція Vaccinate according to the scheme AKDP+hepatitis B+Hib-infection

Провести вакцинацію за схемою гепатит В+Hib-інфекція Vaccinate according to the hepatitis B+Hib infection scheme

Ввести комбіновану 6-компонентну вакцину Enter a combined 6-component vaccine

256 / 1500
Пацієнта віком 62 роки шпиталізовано зі скаргами на сильний біль за грудниною, що триває 1 годину і не знімається нітрогліцирином. Хворіє на стенокардію, раніше напад знімався нітратами. Інших захворювань немає. Об'єктивно спостерігається: ціаноз губ. Тони серця глухі, ритмічні. На ЕКГ виявлено: елевація сегменту ST у V4-V6. Який лікарський засіб потрібно негайно ввести пацієнту? A 62-year-old patient was hospitalized with complaints of severe chest pain lasting 1 hour and not relieved by nitroglycerin. He suffers from angina pectoris, previously the attack was relieved by nitrates. Other diseases Objectively observed: cyanosis of the heart, rhythmic: elevation of the ST segment in V4-V6 is detected immediately.

Но-шпу No-shpu

Дімедрол Diphenhydramine

Актилізе Actilize

Аналгін Analgin

Корглікон Corglycon

257 / 1500
У новонародженої дитини на третій день життя з'явилася жовтяниця шкіри та склер, загальний стан не порушений, сон і апетит без відхилень. З анамнезу відомо: дитина від ІІ пологів, у матері група крові - А (ІІ) Rh(+), у дитини - А (ІІ) Rh(+). Який стан виник у новонародженої дитини? On the third day of life, a newborn child developed jaundice of the skin and sclera, the general condition is not impaired, sleep and appetite are normal. It is known from the anamnesis: the child from II childbirth, the mother's blood group is A (II) Rh(+), the child's blood group is A (II) Rh(+). What condition did the newborn have?

Фізіологічна жовтяниця Physiological jaundice

Пологова травма Birth trauma

Гемолітична хвороба Hemolytic disease

Сепсис Sepsis

Інфекційний гепатит Infectious hepatitis

258 / 1500
Чоловік віком 35 років був знайдений мертвим у власному гаражі. Під час судово-медичного дослідження виявлено: странгуляційна борозна горизонтальна, замкнута, петехіальні крововиливи шкіри голови та шиї, ціаноз обличчя, трупні плями добре виражені, множинні плями Тардьє. Які ознаки вказують на те, що чоловік помер не внаслідок повішення? A 35-year-old man was found dead in his garage. Forensic examination revealed: horizontal strangulation groove, closed, petechial hemorrhages of scalp and neck, cyanosis face, cadaver spots well defined, multiple Tardieu spots. What signs indicate that the man did not die as a result of hanging?

Петехіальні крововиливи шкіри Petechial skin hemorrhages

Добре виражені трупні плями Well defined corpse spots

Горизонтальна странгуляційна борозна Horizontal strangulation furrow

Множинні плями Тардьє Multiple Tardieu spots

Ціаноз обличчя Facial cyanosis

259 / 1500
Роділлю віком 27 років у терміні вагітності 38-39 тижнів шпиталізовано до пологового відділення з приводу кров'янистих виділень зі статевих шляхів, що з'явилися з початком пологової діяльності. Крововтрата склала - 90 мл. Загальний стан пацієнтки задовільний, АТ - 110/70 мм рт. ст. Під час піхвового дослідження спостерігається: шийка матки вкорочена до 0,5 см, розкриття шийки матки 3 см, спереду і праворуч від передлежачої голівки визначається губчаста тканина з нерівною поверхнею, яка доходить до внутрішнього вічка шийки матки. Плідний міхур цілий. Після обстеження кровотеча стала інтенсивнішою. Який найбільш імовірний діагноз? A 27-year-old woman in the 38-39th week of pregnancy was hospitalized in the maternity ward due to bloody discharge from the genital tract that appeared with the onset of labor The blood loss was 90 ml. The patient's blood pressure was 110/70 mm Hg. During the vaginal examination, the cervix was shortened to 3 cm, in front and to the right of the head. spongy tissue that reaches the inner cervix. After examination, the bleeding became more intense?

Бокове передлежання плаценти Lateral placenta previa

Крайове передлежання плаценти Marginal placenta previa

Центральне передлежання плаценти Central placenta previa

Розрив варикозно розширених вен піхви Rupture of varicose veins of the vagina

Відшарування нормально розташованої плаценти Detachment of a normally located placenta

260 / 1500
Під час проведення епідеміологічного дослідження на території міста згідно висунутою гіпотезою вивчалися причино-наслідкові зв'язки виникнення нових випадків захворювань залежно від визначених факторів ризику. Який вид епідеміологічного дослідження був використаний? During an epidemiological study on the territory of the city, according to the proposed hypothesis, the cause-and-effect relationships of the occurrence of new cases of diseases depending on the identified risk factors were studied. What type of epidemiological study was used?

Квазі-експериментальний Quasi-experimental

Описовий Descriptive

Експериментальний Experimental

Аналітичний Analytic

Клінічний Clinical

261 / 1500
Вагітна жінка віком 22 роки госпіталізована у важкому стані. Протягом останніх трьох днів з'явилися набряки, головний біль, нудота, одноразове блювання. Об'єктивно спостерігається: свідомість потьмарена, АТ - 160/130 мм рт. ст., дрібні фібрилярні посмикування м'язів обличччя, утруднене носове дихання. Під час транспортування почалося посмикування верхніх кінцівок, тіло жінки витяглося, хребет вигнувся, щелепи щільно стиснулися, дихання припинилося. Після цього з'явилися клонічні судоми, виражений ціаноз. Потім судоми припинилися, з'явився глибокий шумний вдих, на губах виступила піна, забарвлена кров'ю. Який діагноз найімовірніший? A 22-year-old pregnant woman was hospitalized in serious condition. Over the past three days, swelling, headache, nausea, and one-time vomiting appeared. Objectively observed: consciousness darkened, blood pressure - 160/130 mm Hg, small fibrillar twitching of the facial muscles, difficulty breathing 'clonic convulsions appeared, pronounced cyanosis. Then the convulsions stopped, a deep noisy breath appeared, foam appeared on the lips. What is the most likely diagnosis?

Епілепсія Epilepsy

Діабетична кома Diabetic coma

Еклампсія Eclampsia

Гіпертонічний криз Hypertensive crisis

Хорея Chorea

262 / 1500
Пацієнт скаржиться на нестерпний кашель з виділенням до 600 мл за добу гнійного мокротиння шоколадного кольору з гнилісним запахом. З анамнезу відомо що захворів гостро, температура тіла - 39^oС, лихоманка неправильного типу. Під час рентгенологічного дослідження виявлено: ділянка затемнення з порожниною у центрі, з неправильними контурами та з рівнем рідини. Який найімовірніший діагноз? The patient complains of a persistent cough with discharge of up to 600 ml per day of chocolate-colored purulent sputum with a putrid smell. It is known from the anamnesis that he was acutely ill, the body temperature is 39^ oC, fever of the wrong type. During the X-ray examination, an area of ​​darkening with a cavity in the center, with irregular contours and a level of fluid was found. What is the most likely diagnosis?

Бронхоектатична хвороба Bronchoectatic disease

Рак легені з розпадом Lung cancer with decay

Кавернозний туберкульоз Cavernous tuberculosis

Абсцес легені Lung abscess

Гангрена легені Gangrene of the lung

263 / 1500
Пацієнтка скаржиться на частi, ряснi, без патологiчних домiшок, пiнистi випорожнення, переймоподiбний бiль у навколопупковiй дiлянцi, бурчання в животi, лихоманку. З анамнезу відомо про вживання в їжу качиних яєць, зварених некруто. Який збудник захворювання найiмовiрніший? The patient complains of frequent, abundant, without pathological impurities, foamy stools, cramp-like pain in the peri-umbilical region, stomach rumbling, fever. From the anamnesis, it is known about eating hard-boiled duck eggs. What is the most likely causative agent of the disease?

Salmonella enteritidis Salmonella enteritidis

Salmonella typhi Salmonella typhi

Shigella sonnei Shigella sonnei

Escherichia coli Escherichia coli

V. cholerae V. cholerae

264 / 1500
Хлопчик 12-ти років звернувся до хірурга зі скаргою на нестерпний біль кінчика II пальця правої кисті. Лікарем з'ясовано, що 4 дні тому він вколов це місце дротом. Протягом останніх трьох днів з'явився різкий пульсуючий біль, набряк тканин та гіперемія дистальної фаланги пальця. Об'єктивно: температура тіла - 38,1^oC. Який з перерахованих діагнозів є найбільш імовірним? A 12-year-old boy turned to the surgeon complaining of excruciating pain in the tip of the II finger of the right hand. The doctor found out that 4 days ago he pricked this place with a wire During the last three days, sharp throbbing pain and swelling of the distal phalanx of the finger appeared. Objectively, the body temperature is 38.1°C.

Целюліт Cellulite

Пароніхія Paronychia

Панарицій Panaricius

- -

Еризипелоїд Erysipeloid

265 / 1500
Дитина народилася з терміном гестації 30 тижнів, із масою - 1100,0 г, через 3 години після народження з'явилися часті апное, дихання за типом <>, ЧСС - 98/хв. Оцінка за шкалою Сільвермана - 9 балів. Додаткова оксиногенація не привела до поліпшення стану. Що потрібно зробити? The child was born with a gestation period of 30 weeks, with a weight of 1100.0 g, frequent apnea appeared 3 hours after birth, breathing in the type <>, heart rate - 98/min. Score on the Silverman scale - 9 points. Additional oxygenation did not improve the condition?

Почати тактильну стимуляцію Start tactile stimulation

Почати наружний масаж серця Start external heart massage

Почати штучну вентиляцію легень під позитивним тиском Start positive pressure ventilation

Почати штучну вентиляцію легень Start CPR

Почати інтубувати трахею Start tracheal intubation

266 / 1500
До лікарні шпиталізовано постраждалого в ДТП чоловіка віком 42 роки з діагнозом: політравма, розрив селезінки, перелом правого стегна. Об'єктивно спостерігається: загальмований, шкіра вкрита липким холодним потом, бліда. Дихання поверхневе, часте. Пульс - 140/хв, слабкого наповнення, АТ - 65/40 мм рт. ст., ЦВТ - 0. Планується операційне втручання під внутрішньовенною анестезією зі штучною вентиляцією легень. Який анестетик необхідно вибрати для проведення загальної анестезії? A 42-year-old man injured in a road accident was admitted to the hospital with the diagnosis: polytrauma, rupture of the spleen, fracture of the right hip. Objectively observed: slowed down, skin covered with sticky cold sweat, shallow breathing. Pulse - 140/min, blood pressure - 65/40 mmHg. An operation under intravenous anesthesia is planned general anesthesia?

Кетамін Ketamine

Севофлуран Sevoflurane

Ізофлюран Isoflurane

Тіопентал натрію Sodium thiopental

Пропофол Propofol

267 / 1500
У чоловіка 59 років протягом місяця спостерігається короткочасне зниження сили у лівих кінцівках. Потім уранці після сну розвинулась стійка слабкість у цих кінцівках. Об'єктивно спостерігається: свідомість не втрачена, центральний парез VII і XII пар черепних нервів зліва. З того ж боку центральний геміпарез і гемігіперстезія. Які препарати вибору для диференційованного лікування? A 59-year-old man experienced a short-term decrease in strength in his left limbs during the month. Then in the morning after sleep, persistent weakness developed in these limbs. Objectively observed: consciousness is not lost , central paresis of the VII and XII pairs of cranial nerves on the left side. Central hemiparesis and hemihyperesthesia. What are the drugs of choice for differential treatment?

Антикоагулянти Anticoagulants

Диуретики Diuretics

Гемостатики Hemostatics

Гіпотензивні Hypotensive

Кортикостероїди Corticosteroids

268 / 1500
Жінка 35-ти років надійшла до відділення інтенсивної терапії з нападами судом з періодичністю 2-3 хвилини, між якими не відбувається покращення свідомості, реакція зіниць на світло відсутня. З дитинства хворіє на епілепсію. Зазвичай напади виникають 1-2 рази на місяць у нічний час та супроводжуються мимовільним сечовиділенням та дефекацією. Почастішання нападів та вищевказані ускладнення виникли на тлі перенесеного грипу. Який стан найбільш імовірно розвинувся у хворої? A 35-year-old woman came to the intensive care unit with convulsions every 2-3 minutes, between which there is no improvement in consciousness, and the reaction of the pupils to light is absent. She has been suffering from epilepsy since childhood. Usually, seizures occur 1-2 times a month and are accompanied by involuntary urination and defecation.

Епілептичний психоз Epileptic psychosis

Гіпокальціємічний криз Hypocalcemic crisis

Епілептичний статус Status epilepticus

- -

Істеричний невроз Hysterical neurosis

269 / 1500
У хворого 46-ти років з болем стискального характеру в ділянці серця настала зупинка кровообігу та дихання. На ЕКГ моніторі: великохвильова фібриляція шлуночків. Що потрібно зробити першочергово? A 46-year-old patient with constrictive pain in the heart area has stopped blood circulation and breathing. On the ECG monitor: large-wave ventricular fibrillation. What should be done first?

Внутрішньовенно ввести допамін Dopamine intravenously

Провести дефібриляцію Perform defibrillation

Імплантувати електрокардіостимулятор Implant pacemaker

Внутрішньовенно ввести лідокаїн Inject lidocaine intravenously

Внутрішньовенно ввести атропін Inject atropine intravenously

270 / 1500
Хлопчик 8-ми років, звернувся до лікаря зі скаргами на підвищену втомлюваність, зниження апетиту, іктеричність склер, біль у животі. У періоді новонародженості була затяжна жовтяниця. У фізичному розвитку не відстає. Об'єктивно: шкіра бліда, слизові оболонки і склери іктеричні. Печінка +2 см, злегка болюча при пальпації. Випорожнення і сечовиділення в нормі. При лабораторному дослідженні крові: еритроцити - 4,5·10^12/л, гемоглобін - 115 г/л, лейкоцити - 7·10^9/л, швидкість осідання еритроцитів - 8 мм/год, білірубін прямий - 10 мкмоль/л, непрямий - 39 мкмоль/л, аспартатамінотрансфераза (АСТ) - 17 Од/л, аланінамінотрансфераза (АЛТ) - 21 Од/л. Який діагноз є найбільш імовірним? An 8-year-old boy turned to the doctor with complaints of increased fatigue, decreased appetite, icterus of the sclera, abdominal pain. In the newborn period, he had prolonged jaundice. In Physical development is not lagging behind. Skin is pale, mucous membranes are icteric. Liver is slightly painful on palpation. In laboratory examination, erythrocytes are 4.5·12/l , hemoglobin - 115 g/l, leukocytes - 7·10^9/l, erythrocyte sedimentation rate - 8 mm/h, direct bilirubin - 10 μmol/l, indirect - 39 μmol/l, aspartate aminotransferase (AST) - 17 Units/ l, alanine aminotransferase (ALT) - 21 units/l. What is the most likely diagnosis?

Гемолітична анемія Hemolytic anemia

- -

Хронічний вірусний гепатит Chronic viral hepatitis

Синдром Жильбера Gilbert Syndrome

Дискінезія жовчовивідних шляхів Biliary tract dyskinesia

271 / 1500
У відділення надійшов хворий 30-ти років зі скаргами на часті рідкі випорожнення з кров'ю і слизом, підвищення температури до 37,7^oC, переймоподібний біль у животі. Пальпаторно відзначається болючість по ходу товстої кишки. При ректороманоскопії: стінка кишечника набрякла, легкоранима, ерозії і виразки, значна кількість крові та слизу. Імовірний попередній діагноз: A 30-year-old patient came to the department with complaints of frequent loose stools with blood and mucus, temperature rise to 37.7^oC, spasm-like pain in Abdominal tenderness is noted along the course of the colonoscopy: the intestinal wall is swollen, with erosions and ulcers. A preliminary diagnosis is:

Поліпоз товстої кишки Colon polyposis

Хвороба Крона Crohn's disease

Неспецифічний виразковий коліт Nonspecific ulcerative colitis

Дизентерія Dysentery

Целіакія Celiac

272 / 1500
У чоловіка 73 років, який упродовж 5 років хворіє на доброякісну гіперплазію простати, вранці настала гостра затримка сечі. Закатетеризувати сечовий міхур неможливо. Якою буде невідкладна допомога? A 73-year-old man who has been suffering from benign prostatic hyperplasia for 5 years has acute urinary retention in the morning. It is impossible to catheterize the bladder. What will be the emergency care?

Призначення адреноблокаторів Prescription of adrenoblockers

Призначення спазмолітиків та аналгетиків Prescription of antispasmodics and analgesics

Призначення теплових процедур Assignment of thermal procedures

Призначення сечогінних препаратів Prescription of diuretics

Пункція сечового міхура Bladder puncture

273 / 1500
Пацієнт віком 32 роки, дивлячись на малюнок на шпалерах, бачить, як лінії починають рухатися, утворюючи силуети химерних тварин. Замість люстри, що висить на стелі, бачить гігантського восьминога. Встановіть психопатологічний симптом. A 32-year-old patient, looking at a picture on the wallpaper, sees the lines begin to move, forming silhouettes of whimsical animals. Instead of a chandelier hanging from the ceiling, he sees a giant Octopus. Establish a psychopathological symptom.

Зорові галюцинації Visual hallucinations

Функціональні галюцинації Functional hallucinations

Парейдолічні ілюзії Pareidolic illusions

Псевдогалюцинації Pseudohallucinations

Дереалізація Derealization

274 / 1500
До жіночої консультації звернулась породілля на 18-й день після пологів зі скаргами на біль та ущільнення у правій молочній залозі. Температура тіла - 38,5^oС. Під час огляду встановлено: у ділянці соска - тріщина, болючість під час пальпації. Яке ускладнення виникло в післяпологовому періоді? On the 18th day after giving birth, a woman in labor came to the women's consultation with complaints of pain and tightness in the right mammary gland. The body temperature is 38.5°C. Under at the time of the examination it was established: there was a crack in the area of ​​the nipple, tenderness during palpation. What complication occurred in the postpartum period?

Рак молочної залози Breast cancer

Кіста правої молочної залози Right breast cyst

Лактостаз Lactostasis

Лактаційний мастит Lactation mastitis

Додаткова долька молочної залози Additional breast lobe

275 / 1500
Чоловік 48 років звернувся до лікаря зі скаргами на біль у попереку. Хворіє 3 дні після переохолодження. Після огляду лікар поставив діагноз: гострий попереково-крижовий радикуліт. Які ліки слід призначити хворому? A 48-year-old man went to the doctor with complaints of lower back pain. He has been sick for 3 days after hypothermia. After the examination, the doctor diagnosed acute lumbosacral sciatica. What medicines should be prescribed to the patient?

Нестероїдні протизапальні засоби Nonsteroidal anti-inflammatory drugs

Вітаміни Vitamins

Кортикостероїди Corticosteroids

Антибактеріальні Antibacterial

Десенсибілізуючі Desensitizing

276 / 1500
Хлопчик 15-ти років скаржиться на нападоподібний біль в животі під час дефекації, діарею до 6 разів на добу з домішками гною та темної крові у калових масах. Об'єктивно: відставання у фізичному та статевому розвитку. Шкіра бліда, суха. Живіт здутий, болісний біля пупка та у правій здухвинній ділянці. Припущено хворобу Крона. Яке дослідження треба провести для підтвердження діагнозу? A 15-year-old boy complains of acute abdominal pain during defecation, diarrhea up to 6 times a day with impurities of pus and dark blood in the feces. About' objectively: retardation in physical and sexual development. The skin is swollen, painful in the right navel area. What research should be done to confirm the diagnosis?

Колоноскопія Colonoscopy

УЗД органів черевної порожнини Ultrasound of abdominal organs

Ректороманоскопія Rectoromanoscopy

ФГДС FGDS

Копроцитограма Coprocytogram

277 / 1500
Хвора 42-х років скаржиться на наяв-ність болючого утворення у лівій сідниці, підвищення температури тіла до 38,2^oC. Тиждень тому завершила курс лікування з приводу поперекового радикуліту. У верхньо-зовнішньому квадранті лівої сідниці гіперемія, набряк. При пальпації - різко болюче ущільнення до 6 см в діаметрі з розм'якшенням у центрі. Ваш попередній діагноз: A 42-year-old patient complains of the presence of a painful lump in the left buttock, an increase in body temperature to 38.2^oC. A week ago, she completed a course of treatment for of lumbar radiculitis. In the upper-external quadrant of the left buttock, there is a sharp, painful induration up to 6 cm in diameter. Your previous diagnosis:

Карбункул сідниці Carbuncle of the buttock

Післяін'єкційний абсцес Post-injection abscess

Абсцедивний фурункул Abscess boil

Нагноєна атерома Suppurative atheroma

Підшкірний парапроктит Subcutaneous paraproctitis

278 / 1500
У породіллі вагою 70 кг за 40 хвилин після пологів великим плодом (4500 г) з'явилися значні кров'янисті виділення зі статевих шляхів. Проведено випорожнення сечового міхура та зовнішній масаж матки; після чого матка скоротилася, але за 5 хвилин кровотеча відновилася. Після введення утеротоніків, ручного обстеження порожнини матки крововтрата склала 1200 мл. Якою повинна бути подальша тактика? In a woman in labor weighing 70 kg, 40 minutes after giving birth to a large fetus (4500 g), significant bloody discharge appeared from the genital tract. The bladder was emptied and external massage of the uterus, after which the bleeding resumed in 5 minutes. After the administration of the uterine cavity, the blood loss was 1200 ml. What should be the next tactics?

Кюретаж стінок порожнини матки Curettage of the walls of the uterine cavity

Лапаротомія. Екстирпація матки без додатків Laparotomy. Extirpation of the uterus without attachments

Компресія черевного відділу аорти Compression of the abdominal aorta

Продовжити введення утеротоніків Continue administration of uterotonics

Локальна гіпотермія Local hypothermia

279 / 1500
У дитини 4,5 років з вираженими ознаками зневоднення свідомість на рівні сопору, загальний ціаноз, токсичне дихання, анурія, АТ- 60/20 мм рт.ст., ембріокардія, відсутність пульсу на променевій артерії. Який болюс розчину NaCl 0,9% необхідно ввести на першому етапі невідкладної допомоги? A 4.5-year-old child with pronounced signs of dehydration has consciousness at the level of sedation, general cyanosis, toxic breathing, anuria, blood pressure - 60/20 mm Hg. , embryocardia, absence of a pulse on the radial artery. What bolus of NaCl solution 0.9% should be administered at the first stage of emergency care?

60 мл/кг 60 ml/kg

80 мл/кг 80 ml/kg

50 мл/кг 50 ml/kg

30 мл/кг 30 ml/kg

100 мл/кг 100 ml/kg

280 / 1500
Дівчина 22 років, хвора на системний червоний вовчак, скаржиться на лихоманку, схуднення, біль у дрібних суглобах кистів. Об'єктивно встановлено: дифузна алопеція, генералізована лімфаденопатія, еритематозна висипка на щоках та спинці носа, ексудативний плеврит та перикардит. Температура тіла - 38,4^oC. У крові виявлено: лейкоцити - 3,1·10^9/л, Нb - 102 г/л, ШОЕ - 56 мм/год, С-реактивний протеїн - (++++). Яка доза преднізолону є доцільною у цьому разі? A 22-year-old girl with systemic lupus erythematosus complains of fever, weight loss, pain in the small joints of the hands. Objectively established: diffuse alopecia, generalized lymphadenopathy, erythematous rash on the cheeks and back of the nose, exudative pleurisy and pericarditis. Body temperature - 38.4°C. Leukocytes - 3.1·10^9/l, Hb - 102 g/l, ESR - 56 mm/l. hours, C-reactive protein - (++++). What dose of prednisolone is appropriate in this case?

0,75 мг/кг 0.75 mg/kg

0,1 мг/кг 0.1 mg/kg

0,25 мг/кг 0.25 mg/kg

0,5 мг/кг 0.5 mg/kg

1 мг/кг 1 mg/kg

281 / 1500
У 9-місячної дитини ВІЛ-інфіко-ваної жінки визначені гіпотрофія 2-го ступеня, полілімфаденопатія, гепатоспленомегалія, рецидивуючий орофарингеальний кандідоз, гіпохромна анемія (Нb - 80 г/л), нейтропенія; виявлено ДНК ВІЛ у крові. Поставте діагноз: In a 9-month-old child of an HIV-infected woman, hypotrophy of the 2nd degree, polylymphadenopathy, hepatosplenomegaly, recurrent oropharyngeal candidiasis, hypochromic anemia (Hb - 80 g /l), neutropenia; HIV DNA was detected in the blood. Diagnose:

Безсимптомне носійство ВІЛ Asymptomatic HIV carrier

ВІЛ-серопозитивна дитина HIV-seropositive child

- -

СНІД. Опортуністичні інфекції AIDS. Opportunistic infections

Персистуюча генералізована лімфаденопатія Persistent generalized lymphadenopathy

282 / 1500
У чоловіка 52 рокiв, який страждає на постхолецистектомiчний синдром, під час ретроградної холецистопанкреатографії виявлено стеноз сфiнктера Оддi з помiрною дилатацiею загального жовчного протока i рефлюксом контраста у Вiрсунгiв проток. Конкрементiв у протоках немає. Дiагноз хронiчного панкреатита безсумнiвний. Яка операцiя адекватна для корекцiї цієї патологiї? In a 52-year-old man suffering from post-cholecystectomy syndrome, stenosis of the sphincter of Oddi with moderate dilatation of the common bile duct and reflux of contrast into the duct of Wirsung was revealed during retrograde cholecystopancreatography. there are no ducts. The diagnosis of chronic pancreatitis is unquestionable. What operation is adequate for the correction of this pathology?

Продольна панкреатиконостомiя Longitudinal pancreaticostomy

Ендоскопiчна папiлосфiнктеротомiя Endoscopic papillosphincterotomy

Холецисто-єюностомія Cholecystojejunostomy

Холедохо-дуоденостомія Choledocho-duodenostomy

Трансдуоденальна папiлосфiнктеропластика Transduodenal papillosphincteroplasty

283 / 1500
У пацієнта віком 22 роки об'єктивно спостерігається: на шкірі грудей, шиї, бокових поверхонь тулуба, плечей - чисельні незапальні плями жовтувато-коричневого та блідо-рожевого кольору, у разі пошкрябування яких виявляється 'симптом стружки'. Хворіє протягом 2-х років, висипи помітив після літнього відпочинку на морі. Який найімовірніший діагноз? A 22-year-old patient objectively observes: on the skin of the chest, neck, lateral surfaces of the body, shoulders - numerous non-inflammatory spots of yellowish-brown and pale pink color , in the case of scraping, it turns out to be a 'chip symptom'. He has been sick for 2 years, he noticed rashes after a summer vacation at the sea. What is the most likely diagnosis?

Сифілітична розеола Syphilitic roseola

Рожевий лишай Жибера Zhiber's pink lichen

Еритразма Erythrasma

Стрептодермія Streptoderma

Висівкоподібний лишай Lichen bran

284 / 1500
Пацієнт віком 32 роки під час фізичних вправ зненацька відчув нестачу дихання, слабкість, біль в правій половині грудей з іррадіацією в праве плече, задишку, серцебиття. Об'єктивно спостерігається: стан тяжкий, тахікардія до 100/хв, АТ - 90/60 мм рт. ст. ЧД - 28/хв, права половина грудної клітки відстає при диханні. Перкуторно справа тимпанічний звук, дихальних шумів немає. Температура тіла - нормальна. Який найімовірніший діагноз? A 32-year-old patient suddenly experienced shortness of breath, weakness, pain in the right half of the chest with radiation to the right shoulder, shortness of breath, palpitations during physical exercises. Objectively the condition is severe, BP - 90/60 mmHg, right sided chest sound, no respiratory sounds the most likely diagnosis?

Інфаркт легені Pulmonary infarction

Пневмонія Pneumonia

Інфаркт міокарда Myocardial infarction

Спонтаний пневмоторакс Spontaneous pneumothorax

Судинний колапс Vascular collapse

285 / 1500
У сухому піщаному ґрунті був знайдений труп чоловіка 45-50 років, шкіра якого буро-брунатного кольору, пергаментної щільності. Під час обстеження трупа встановлено: травматичні ушкодження відсутні, довжина тіла 172 см, внутрішні органи зменшені, сухі, безформні, окремі органи плівчасті. В якому стані трупних змін перебуває труп? The corpse of a 45-50-year-old man was found in the dry sandy soil, the skin of which is brownish-brown in color, parchment-like. During the examination of the corpse, it was found: there are no traumatic injuries, body length 172 cm, internal organs are reduced, dry, shapeless, some organs are membranous. What state of cadaveric changes is the corpse in?

Штучного консервування Artificial preservation

Торф'яного дублення Peat tanning

Жировоску Tallow wax

Гниття Rot

Муміфікації Mummifications

286 / 1500
Мати 22-місячного хлопчика звернулася до лікаря зі скаргами на погане набирання ваги та худобу у дитини. Годує грудним молоком на вимогу, не менше 5 разів на день. Вага та зріст хлопчика нижче 3-го стандартного відхилення для його віку. У нього дугоподібна деформація ніг та потовщені зап'ястя. При лабораторному дослідженні підвищений рівень лужної фосфатази. Дефіцит якого вітаміну найбільш імовірно спричинив подібний стан у дитини? The mother of a 22-month-old boy went to the doctor with complaints of poor weight gain and cattle in the child. Breastfeeds on demand, at least 5 times a day. Weight and the boy's height is below the 3rd standard deviation. He has bowed legs and thickened wrists. On laboratory examination, the level of alkaline phosphatase is most likely to cause this condition in the child?

Вітаміну A Vitamin A

Вітаміну B_6 Vitamin B_6

Вітаміну PP Vitamin PP

Вітаміну E Vitamin E

Вітаміну D Vitamin D

287 / 1500
У лісі виявлено труп чоловіка 35 років, який висів у петлі на дереві. Тканини обличчя та шиї синюшного забарвлення, нижні кінцівки звичайного забарвлення. Странгуляційна борозна добре виражена, замкнута. Установити причину та рід смерті: The corpse of a 35-year-old man was found in the forest, hanging from a noose on a tree. The tissues of the face and neck are bluish in color, the lower limbs are of a normal color. The strangulation furrow is well defined, closed To establish the cause and type of death:

Механічна асфіксія внаслідок закриття дихальних шляхів, рід смерті - насильницька Mechanical asphyxiation due to closing of the respiratory tract, type of death - violent

Механічна асфіксія внаслідок здавлення грудей і живота, рід смерті - насильницька Mechanical asphyxiation due to compression of the chest and abdomen, the manner of death is violent

Механічна асфіксія внаслідок задушення петлею, рід смерті - насильницька Mechanical asphyxiation due to strangulation with a noose, type of death - violent

Механічна асфіксія внаслідок задушення петлею, рід смерті - ненасильницька Mechanical asphyxiation due to strangulation with a noose, type of death - non-violent

Механічна асфіксія внаслідок повішення, рід смерті - ненасильницька Mechanical asphyxiation due to hanging, type of death - non-violent

288 / 1500
64-річний чоловік прийшов до лікаря на щорічний профілактичний огляд. Скарг не має. Хворіє на артеріальну гіпертензію, ХОЗЛ та глаукому. Викурює одну пачку сигарет на день протягом 30-ти років (індекс паління - 30 пачко/років). При фізикальному обстеженні: температура - 37,2^oC, артеріальний тиск - 140/70 мм рт.ст., пульс - 79/хв., частота дихання - 16/хв., пряма, співдружня реакція зіниць на світло. При аус-культації серця акцент II тону над аортою, аускультація легень без патологічних змін, грудна клітка ''бочкоподібна''. Який з перерахованих методів скринінгу є найбільш доречним для цього пацієнта? A 64-year-old man came to the doctor for an annual preventive examination. He has no complaints. He suffers from arterial hypertension, COPD and glaucoma. He smokes one pack of cigarettes a day for 30 years (smoking index - 30 pack/years). During physical examination: temperature - 37.2°C, blood pressure - 140/70 mm Hg, pulse - 79/min, respiratory rate - 16/min ., direct pupillary reaction to light. During auscultation of the heart, the accent of the II tone over the aorta, auscultation of the lungs is 'barrel-shaped'. Which of the listed screening methods is the most appropriate for this patient?

МРТ легень Lung MRI

Низькодозова КТ легень Low-dose lung CT

Рентгенографія ОГК Roentgenography of OGK

Бронхоальвеолярний лаваж Bronchoalveolar lavage

Легеневі функціональні тести Pulmonary function tests

289 / 1500
Чоловік віком 63 роки звернувся до лікаря зі скаргами на виражену загальну слабкість, поганий апетит, схуднення, біль в суглобах, відчуття важкості в правому підребер'ї. У клінічному аналізі крові спостерігається: еритроцити - 3,4·10^12/л, Hb - 102 г/л, КП - 0,9, тромбоцити - 640·10^9/л, лейкоцити - 138·10^9/л, бласти -1 %, промієлоцити - 2 %, мієлоцити - 13 %, юні - 12 %, паличкоядерні - 16 %, сегментоядерні - 31 %, базофіли - 3 %, еозинофіли - 8 %, лімфоцити - 9 %, моноцити - 5 %, ШОЕ - 30 мм/год. Який попередній діагноз? A 63-year-old man consulted a doctor with complaints of severe general weakness, poor appetite, weight loss, pain in the joints, a feeling of heaviness in the right hypochondrium. In the clinical blood analysis shows: erythrocytes - 3.4·10^12/l, Hb - 102 g/l, CP - 0.9, platelets - 640·10^9/l, leukocytes - 138·10^9/l, blasts -1%, promyelocytes - 2%, myelocytes - 13%, young - 12%, rod-nuclear - 16%, segmentonuclear - 31%, basophils - 3%, eosinophils - 8%, lymphocytes - 9%, monocytes - 5%, ESR - 30 mm/h. What is the previous diagnosis?

Хронічний лімфолейкоз Chronic lymphocytic leukemia

Лейкемоїдна реакція Leukemoid reaction

Хронічний мієлолейкоз Chronic myelogenous leukemia

Гострий лейкоз Acute leukemia

Еритремія Erythremia

290 / 1500
Робітник цеху з виробництва акумуляторів для автомобілів звернувся до лікаря зі скаргами на нудоту, втрату апетиту, різкий біль у черевній порожнині, закрепи. Під час огляду лікар виявив підвищення артеріального тиску, брадикардію, збільшення печінки, біль в правому підребер'ї, сірувато-синю смужку на яснах, сірий колір шкірних покривів. Результати лабораторного аналізу крові свідчать про наявність еритроцитів з базофільною зерністістю та знижений вміст гемоглобіну, а у сечі виявлено амінолевулінову кислоту і копропорфірин. Який попередній діагноз найімовірніший? A worker at a workshop for the production of batteries for cars turned to the doctor with complaints of nausea, loss of appetite, sharp pain in the abdominal cavity, constipation. During the examination, the doctor found an increase in blood pressure pressure, bradycardia, liver enlargement, pain in the right hypochondrium, grayish-blue stripe on the gums, gray color of the skin. The results of the laboratory blood test indicate the presence of erythrocytes with basophilic granularity and a reduced hemoglobin content, and aminolevulinic acid and coproporphyrin were detected in the urine . What is the most likely preliminary diagnosis?

Отруєння кадмієм Cadmium poisoning

Харчове отруєння Food poisoning

Отруєння алюмінієм Aluminum poisoning

Отруєння ртуттю Mercury poisoning

Отруєння свинцем Lead poisoning

291 / 1500
Жінка, яка хворіє на полікістоз нирок, спостерігає зростання добового діурезу до 2-2,5 л. За даними динамічної нефросцинтиграфії виявлено, що загальна клубочкова фільтрація становить 34 мл/хв., креатинін сироватки крові - 84 мкмоль/л, сечовина - 8,0 ммоль/л. Яка стадія хронічної ниркової недостатності у пацієнтки? A woman suffering from polycystic kidney disease observes an increase in daily urine output to 2-2.5 liters. According to dynamic nephroscintigraphy, it was found that the total glomerular filtration rate is 34 ml /min., serum creatinine - 84 μmol/l, urea - 8.0 mmol/l. What is the patient's stage of chronic renal failure?

Інтермітуюча Intermittent

Поліурична Polyuric

Термінальна Terminal

Компенсована Compensated

Латентна Latent

292 / 1500
При проведенні медичного огляду учнів середнього та старшого шкільного віку лікарі визначали відповідність біологічного розвитку та календарного віку за наступними критеріями: щорічне збільшення довжини тіла, осифікація кісток кисті, кількість постійних зубів. Який додатковий показник розвитку у ці вікові періоди найбільш імовірно мають включити лікарі? During the medical examination of middle and high school students, doctors determined the correspondence of biological development and calendar age according to the following criteria: annual increase in body length, ossification of hand bones, number of permanent teeth. What additional developmental indicator should doctors most likely include in these age periods?

Розвиток вторинних статевих ознак Development of secondary sexual characteristics

Життєва ємність легень Vital lung capacity

Маса тіла Body weight

М'язова сила кисті Hand muscle strength

Обвід грудної клітки Chest Circumference

293 / 1500
Дівчина 21-го року звернулася до лікаря з грипоподібною картиною захворювання, субфебрильною температурою, нездужанням та легкою жовтяницею протягом 2-х днів. Лабораторні показники сироватки крові: аспартатамінотрансфераза (АСТ) - 456 Од/л, аланінамінотрансфераза (АЛТ) - 745 Од/л, IgM до HAV - позитивний результат. Яка тактика по відношенню до контактних осіб буде найбільш доречною? A 21-year-old girl consulted a doctor with a flu-like illness, low-grade fever, malaise, and mild jaundice for 2 days. Serum laboratory parameters: aspartate aminotransferase ( AST) - 456 units/l, alanine aminotransferase (ALT) - 745 units/l, IgM to HAV - a positive result. What tactics will be most appropriate in relation to contact persons?

Вакцинація від гепатиту В протягом першого тижня Hepatitis B vaccination during the first week

Введення інтерферону альфа-2b протягом першого тижня Introduction of interferon alfa-2b during the first week

Профілактичний прийом софосбувіру/ледіпасвіру протягом 1 місяця Prophylactic sofosbuvir/ledipasvir for 1 month

Введення імуноглобуліну людини нормального протягом 24 годин Injection of normal human immunoglobulin within 24 hours

Вакцинація від гепатиту А протягом першого тижня Vaccination against hepatitis A during the first week

294 / 1500
Контроль міжнародного нормалізованого співвідношення (МНС) потрібно проводити у пацієнтів із фібриляцією передсердь, які приймають препарат із групи антитромботичних засобів. Укажіть цей препарат. International normalized ratio (INR) control should be performed in patients with atrial fibrillation who are taking a drug from the group of antithrombotic drugs. Specify this drug.

Варфарин (антагоніст вітаміну К) Warfarin (vitamin K antagonist)

Ривароксабан (інгібітори фактора Ха) Rivaroxaban (factor Xa inhibitors)

Дабігатран (прямий інгібітор тромбіну) Dabigatran (direct thrombin inhibitor)

Аспірин (інгібітор ферменту циклооксигенази) Aspirin (cyclooxygenase enzyme inhibitor)

Клопідогрель (блокатор АДФ рецепторів тромбоцитів) Clopidogrel (platelet ADP receptor blocker)

295 / 1500
Чоловік 31-го року доставлений до відділення невідкладної допомоги зі скаргами на утруднене дихання, відчуття стороннього тіла в горлі, осиплість, сильний набряк обличчя та шиї. Вищевказані симптоми швидко наростають та з'явились протягом декількох хвилин після куштування меду. Який стан найбільш імовірно розвинувся у хворого? A 31-year-old man was brought to the emergency department with complaints of difficulty breathing, feeling of a foreign body in the throat, dizziness, severe swelling of the face and neck. The above symptoms quickly build up and appeared within a few minutes after tasting the honey. Which condition most likely developed in the patient?

Гострий ларинготрахеїт Acute laryngotracheitis

Ангіоневротичний набряк Квінке Angioedema of Quincke

Запальний набряк гортані Inflammatory swelling of the larynx

Токсичний епідермальний некроліз Toxic epidermal necrolysis

Синдром Стівенса-Джонсона Stevens-Johnson Syndrome

296 / 1500
Хвора 12 років із хронічним панкреатитом, період реконвалесценції. Стоїть питання про подальше продовження замісної терапії препаратом панкреатину та розширення режиму харчування. За яким методом дослідження можна найкоректніше оцінити екзокринну функцію підшлункової залози? 12-year-old patient with chronic pancreatitis, convalescent period. There is a question about further continuation of pancreatin replacement therapy and expansion of the diet. Which research method can be used to assess exocrine function most correctly pancreas?

Визначення еластази-1 у калі Determination of elastase-1 in feces

Визначення рівня амілази в сироватці крові та діастази в сечі Determining the level of amylase in blood serum and diastase in urine

Визначення рівня трипсину в сироватці крові Determination of trypsin level in blood serum

Визначення еластази-2 у сироватці крові Elastase-2 determination in blood serum

Рівень стеатореї під час копро логіч-ного дослідженя Level of steatorrhea during coprologic examination

297 / 1500
У хворого, що 1 рік тому переніс Q-інфаркт міокарда задньої стінки лівого шлуночка, протягом останніх 2-х тижнів щодня виникають напади фібриляції передсердь та епізоди брадикардії, що супроводжуються запамороченнями. Яка найбільш доцільна тактика? A patient who suffered a Q-myocardial infarction of the posterior wall of the left ventricle 1 year ago, during the last 2 weeks, attacks of atrial fibrillation and episodes of bradycardia occur every day, which are accompanied by dizziness. What is the most appropriate tactic?

Призначити новокаїнамід Prescribe novocaine

Призначити аміодарон Prescribe amiodarone

Призначити бісопролол Prescribe bisoprolol

Імплантація кардіостимулятора Implantation of a pacemaker

Призначити дигоксин Prescribe digoxin

298 / 1500
Протягом останніх трьох років хвора жінка страждає на цукровий діабет. Сімейний лікар, з метою профілактики ускладнень цукрового діабету, регулярно призначає їй превентивне лікування. До якого виду профілактики належать вказані заходи? For the past three years, a sick woman has been suffering from diabetes. The family doctor, in order to prevent complications of diabetes, regularly prescribes preventive treatment for her. What kind of prevention do the specified measures?

Вторинна профілактика Secondary prevention

Первинна профілактика Primary prevention

Громадська профілактика Public prevention

Третинна профілактика Tertiary prevention

Індивідуальна профілактика Individual prevention

299 / 1500
Хворому 65-ти років встановлено діагноз рак головного мозку IV стадії. Чоловік з вираженими епілептичними нападами та больовим синдромом. Який вид медичної допомоги необхідний для полегшення стану хворого? A 65-year-old patient has been diagnosed with stage IV brain cancer. A man with severe epileptic seizures and pain syndrome. What type of medical care is needed to alleviate the patient's condition?

Вторинна медична допомога Secondary medical care

Третинна медична допомога Tertiary medical care

Первинна медична допомога Primary medical care

Паліативна медична допомога Palliative medical care

Санаторно-курортна допомога Sanatorium and resort assistance

300 / 1500
У новонародженого на п'ятий день життя відмічається надмірне здуття живота з контуруванням кишкових петель на передній черевній стінці. Меконій відійшов після клізми. На оглядовій рентгенограмі органів черевної порожнини було виявлено розширення петель товстого кишківника. Яка вада розвитку ймовірно у дитини? A newborn on the fifth day of life has excessive abdominal distension with contouring of intestinal loops on the anterior abdominal wall. Meconium has passed after an enema. On the X-ray examination of the abdominal organs, there was the expansion of the loops of the large intestine was detected. What malformation is likely in the child?

Інвагінація кишківника Intussusception

Стеноз клубової кишки Ileal stenosis

Хвороба Гіршпрунга Hirschsprung's disease

Синдром Леда Led syndrome

Портальна гіпертензія Portal hypertension

301 / 1500
Для характеристики стану здоров’я населення в районі розраховувалися показники народжуваності, смертності, природного приросту, поширеності хвороб і первинної захворюваності, загальної інвалідності та інвалідизації населення. До якого виду статистичних величин відносяться ці показники? In order to characterize the state of health of the population in the district, indicators of birth rate, mortality, natural increase, prevalence of diseases and primary morbidity, general disability and disability of the population were calculated. To what type statistical values ​​to which these indicators belong?

Співвідношення Ratio

Екстенсивних Extensive

Наочності Visibility

Стандартизованих Standardized

Інтенсивних Intensive

302 / 1500
Хлопчик 9-ти років упав з дерева, вдарився потиличною ділянкою, спостерігалась короткочасна втрата свідомості. Стан дитини задовільний, турбують головний біль, запаморочення. На рентгенограмах черепа виявлено втиснений уламчастий перелом потиличної кістки в ділянці потиличного горба. Яка лікувальна тактика показана хворому? A 9-year-old boy fell from a tree, hit the occipital region, a short-term loss of consciousness was observed. The child's condition is satisfactory, he is bothered by headache, dizziness. X-rays of the skull revealed an imprinted fragmentary fracture of the occipital bone in the area of ​​the occipital hump. What treatment tactics are indicated for the patient?

Протизапальна терапія Anti-inflammatory therapy

Розвантажувальні люмбальні пункції Unloading lumbar punctures

Оперативне втручання Operative intervention

Гемостатична терапія Hemostatic therapy

Комплексне консервативне лікування Complex conservative treatment

303 / 1500
Дитина віком 6 років захворіла гостро: підвищилася температура тіла, з'явилися головний біль і біль під час ковтання. Через 3 години з'явилися яскраво-червоний дрібно-крапчастий висип на гіперемованій шкірі, рясніший на боковій поверхні тулуба та в природних складках, відмежована гіперемія слизової оболонки ротоглотки, гнійний наліт на мигдаликах. Яке захворювання можна припустити у дитини? A 6-year-old child became acutely ill: body temperature rose, headache and pain during swallowing appeared. After 3 hours, bright red small spotted rash on the hyperemic skin, more abundant on the lateral surface of the body and in the natural folds, limited hyperemia of the mucous membrane of the oropharynx, purulent plaque on the tonsils. What disease can be assumed in the child?

Інфекційний мононуклеоз Infectious mononucleosis

Краснуха Krasnukha

Скарлатина Scarlatina

Дифтерія Diphtheria

Кір Measles

304 / 1500
Дитині 7 років. Скаржиться на біль у животі нападоподібного характеру, який виникає після психічного навантаження, уживання холодних напоїв, морозива. Після клініко-інструментального обстеження поставлено діагноз: дискінезія жовчного міхура за гіпертонічним типом. Які препарати слід призначати для лікування насамперед? The child is 7 years old. He complains of abdominal pain of an attack-like nature, which occurs after mental stress, consumption of cold drinks, ice cream. After a clinical and instrumental examination, the diagnosis was made: dyskinesia gall bladder according to the hypertensive type. What drugs should be prescribed for treatment first of all?

Холеретики та холекінетики Choleretics and cholekinetics

Спазмолітики та холеретики Spasmolytics and choleretics

Антиоксиданти Antioxidants

Седативні та холекінетики Sedatives and cholekinetics

Антибіотики Antibiotics

305 / 1500
У хлопчика 3-х років під час ходьби та бігу збільшується права половина мошонки, яка в положенні лежачи зменшує-ться до нормальних розмірів. Діаг-ноз: In a 3-year-old boy, the right half of the scrotum increases during walking and running, which decreases to normal size when lying down. Diagnosis:

Цистоцеле Cystocele

Орхіепідидиміт Orchiepididymitis

Парафімоз Paraphimosis

Сполучна водянка оболонок правого яєчка Conjunctive dropsy of the membranes of the right testicle

Фімоз Phimosis

306 / 1500
Пацієнт віком 38 років скаржиться на кашель з виділенням гнійного мокротиння (до 60-80 мл на добу), підвищення температури тіла до 39^oС. Захворювання пов'язує з переохолодженням. Об'єктивно спостерігається: пульс - 96/хв, ритмічний. Артеріальний тиск - 110/60 мм рт. ст. Під час огляду виявлено: відставання правої половини в диханні. ЧД - 30/хв. Перкуторно локальне притуплення біля кута лопатки. Під час аускультації вислуховується: вологі різнокаліберні хрипи, амфоричне дихання. Який найімовірніший діагноз? A 38-year-old patient complains of a cough with the release of purulent sputum (up to 60-80 ml per day), an increase in body temperature to 39°C. The disease is associated with Objectively observed: pulse - 110/60 mm Hg. During the examination, the right side was found to be dull - 30/min . During auscultation: moist rales of various calibers, amphoric breathing are heard. What is the most likely diagnosis?

Бронхіальна астма Bronchial asthma

Гострий абсцес легені Acute lung abscess

Емпієма плеври Empyema of the pleura

Вогнищева пневмонія Focal pneumonia

Гострий бронхіт Acute bronchitis

307 / 1500
Одинадцятирічна дівчинка скаржиться на біль у суглобах, підвищення температури тіла до 38^oС, слабкість. Захворіла 5 днів тому. Об'єктивно спостерігається: плямисто-папульозний висип на переніссі та обличчі, суглоби не змінені, тони серця послаблені. АТ - 100/80 мм рт. ст. У загальному аналізі крові: еритроцити - 2,6 т/л, лейкоцити - 3,7 г/л, тромбоцити - 12г/л, ШОЕ - 45 мм/год, СРБ (+++). У загальному аналізі сечі: білок - 0,063 г/л, еритроцити - 15-20 в п/з, лейкоцити - 10-14 в п/з. Вкажіть імовірний діагноз. An eleven-year-old girl complains of pain in the joints, an increase in body temperature up to 38°C, weakness. She fell ill 5 days ago. Objectively observed: a spotted-papular rash on nose and face, the joints are not changed, blood pressure - 100/80 mm Hg: erythrocytes - 3.7 g/l, platelets - 12 g/l. , ESR - 45 mm/h, CRP (+++). In the general analysis of urine: protein - 0.063 g/l, erythrocytes - 15-20 in p/z. Specify the probable diagnosis .

Системний червоний вовчак Systemic lupus erythematosus

Геморагічний васкуліт Hemorrhagic vasculitis

Гостра ревматична лихоманка Acute rheumatic fever

Дерматоміозит Dermatomyositis

Вузликовий періартеріїт Nodular periarteritis

308 / 1500
Громадянин заключив декларацію з сімейним лікарем приватного закладу. Для проходження чергового медичного огляду, лікар дав направлення хворому на біохімічний аналіз крові. Хто є платником медичної послуги у цьому разі? The citizen made a declaration with the family doctor of a private institution. To undergo another medical examination, the doctor referred the patient for a biochemical blood test. Who is the payer of the medical service in this case?'

Благодійні внески Charity contributions

За рахунок місцевого бюджету At the expense of the local budget

Спеціальний фонд лікарні Special hospital fund

Пацієнт Patient

НСЗУ NSZU

309 / 1500
У пацієнта протягом декількох днів з'явилося обмеження периферичного зору ззовні, схоже на заслінку. Причину не знає. Око спокійне, оптичні середовища прозорі. Під час офтальмоскопії з носової сторони при рухах ока коливається сірий «парус» із судинами. Диск зорового нерва та судини не змінені. Який діагноз найімовірніший? For several days, the patient had a limitation of peripheral vision from the outside, similar to a valve. The cause is unknown. The eye is calm, the optical media is transparent. During ophthalmoscopy from the nose sides with eye movements, a gray 'sail' with vessels. The optic nerve disc and vessels are not changed. What is the most likely diagnosis?

Судинна патологія сітківки Retinal vascular pathology

Геміанопсія при неврологічній патології Hemianopsia in neurological pathology

Відшарування сітківки Retinal detachment

Підвивих кришталика Subluxation of the lens

Початкові прояви глаукоми Initial manifestations of glaucoma

310 / 1500
Пацієнта віком 30 років шпиталізовано до лікарні. З анамнезу відомо що пацієнт захворів гостро, хвороба почалась з частих водянистих випорожнень, потім з'явилось рясне блювання без нудоти, спрага. Об'єктивно спостерігається: температура тіла - 35,4^oC. Стан важкий, шкіра холодна. Тургор тканин та еластичність шкіри значно знижені. Риси обличчя загострені. Голос сиплий, акроціаноз. Пульс - 130/хв, слабкий. АТ - 60/30 мм рт. ст. Язик сухий. Живіт запалий, неболючий, під час пальпації виражене гуркотіння в кишечнику. Анурія. Який найімовірніший діагноз? A 30-year-old patient was admitted to the hospital. It is known from the anamnesis that the patient became acutely ill, the disease began with frequent watery stools, then profuse vomiting appeared without nausea, thirst Objectively observed: body temperature - 35.4 °C, skin turgor is increased. Voice - 130/min. 30 mm Hg. The tongue is inflamed, during palpation there is pronounced rumbling. What is the most likely diagnosis?

Ротавірусний гастроентерит Rotavirus gastroenteritis

Холера Cholera

Харчова токсикоінфекція Food poisoning

Сальмонельоз Salmonellosis

Шигельоз Shigelosis

311 / 1500
Чоловік 45-ти років, звернувся до лікаря зі скаргами на дратівливість, підвищену втомленість, схуднення, серцебиття, перебої в роботі серця. При пальпації лівої долі щитоподібної залози пальпується утворення, щільно-еластичної консистенції, безболісне, яке при ковтанні зміщується разом з щитоподібною залозою. Позитивні очні симптоми, екзофтальм, порушення конвергенції. Артеріальний тиск - 135/80 мм рт.ст., пульс - 110/хв. При ультразвуковому дослідженні щитоподібної залози у лівій долі візуалізується гіперехогенне округле утворення розмірами 2х3 см, з чіткими контурами, однорідної структури. Який висновок лікаря після проведеного обстеження буде найбільш коректним? A 45-year-old man consulted a doctor with complaints of irritability, increased fatigue, weight loss, palpitations, interruptions in the work of the heart. When palpating the left lobe of the thyroid gland, palpable formation, of a dense and elastic consistency, which is displaced with the thyroid gland. Positive eye symptoms, violation of convergence. Arterial pressure - 135/80 mm Hg. During ultrasound examination of the thyroid gland In the left lobe, a 2x3 cm hyperechoic mass with clear contours and uniform structure is visualized. What conclusion of the doctor will be the most correct after the examination?

Рак щитоподібної залози Thyroid cancer

Дифузно-токсичний зоб Diffuse toxic goiter

Вузол щитоподібної залози Thyroid nodule

Кіста щитоподібної залози Thyroid cyst

Гострий тиреоїдит Acute thyroiditis

312 / 1500
Пацієнт госпіталізований у нейрохірургічне відділення з приводу закритої черепно-мозкової травми, перелому скроневої кістки справа. Через 5 годин його стан різко погіршився, виникли порушення дихання, періодичні тонічні судоми, анізокорія (розширення правої зіниці). Яке ускладнення можна припустити у пацієнта? The patient was hospitalized in the neurosurgery department due to a closed craniocerebral injury, a fracture of the temporal bone on the right. After 5 hours, his condition deteriorated sharply, breathing disorders, periodic tonic convulsions occurred , anisocoria (enlargement of the right pupil). What complications can be assumed in the patient?

Ішемічний інсульт в ділянці правої ніжки мозку Ischemic stroke in the area of ​​the right leg of the brain

Субарахноїдальний крововилив Subarachnoid hemorrhage

Абсцес головного мозку Brain abscess

Виникнення епідуральної гематоми Emergence of epidural hematoma

Особливості перебігу струсу головного мозку Features of concussion

313 / 1500
Новонароджена доношена дитина з масою тіла 4500 г народилася в асфіксії з оцінкою за шкалою Апгар 4-6 балів. В пологах утруднене виведення плечового поясу. У неврологічному статусі: загальномозкові розлади, виявлений тотальний верхній млявий парез - ручка атонічна, пронована, не викликаються рефлекси - хапальний, Бабкіна. Вкажіть рівень ураження спинного мозку: A newborn full-term child with a body weight of 4500 g was born asphyxiated with an Apgar score of 4-6 points. During delivery, the removal of the shoulder girdle was difficult. In the neurological status: cerebral disorders, a total upper limb paresis was detected - the handle is atonic, pronated, reflexes are not evoked - grasping, Babkin. Specify the level of spinal cord damage:

Шийні сегменти СI-СII Cervical segments CI-CII

Шийно-грудні сегменти СV-TI Cervical-thoracic segments СV-TI

Грудні сегменти TVI-TVІІ Thoracic segments TVI-TVII

Шийні сегменти СIII-С IV Cervical segments CIII-C IV

Грудні сегменти TI-TV TI-TV Chest Segments

314 / 1500
У дівчинки 12 років упродовж 2 місяців відзначається періодичне підвищеня температури тіла до 39^oС, спостерігаються веретеноподібна припухлість міжфалангових суглобів, біль у верхній частині грудини і шиї, скутість зранку. Який діагноз найімовірніший? A 12-year-old girl has had a periodic increase in body temperature up to 39°C for 2 months, spindle-shaped swelling of the interphalangeal joints, pain in the upper part of the sternum and neck, morning stiffness . What is the most likely diagnosis?

Токсичний синовіт Toxic synovitis

Ювенільний ревматоідний артрит Juvenile rheumatoid arthritis

Остеоартрит Osteoarthritis

Септичний артрит Septic arthritis

Ревматизм Rheumatism

315 / 1500
Після видобування корисних копалин і швидкому піднятті на поверхню з глибини 80 м працівники відчули сильні болі у суглобах і м'язах, м'язах живота, парез нижніх кінцівок, судоми. Укажіть ймовірне захворювання: After extracting minerals and quickly rising to the surface from a depth of 80 m, the workers felt severe pains in the joints and muscles, abdominal muscles, paresis of the lower limbs, convulsions. Specify the probable disease:

Кесонна хвороба caisson disease

Тромбоемболія судин нижніх кінцівок Thromboembolism of vessels of the lower extremities

Шумова хвороба Noise sickness

Висотна хвороба Altitude sickness

Горна хвороба Mountain sickness

316 / 1500
Хлопчик 3-х років надійшов з вираженим набряковим синдромом. Об'єктивно: блідий. Артеріальний тиск - 90/60 мм рт.ст. Олігурія. Загальний аналіз сечі: колір - жовтий, відносна густина - 1020, білок - 3,5 г/л, еритроцити - вилужені, 4-5 в п/з, лейкоцити - 2-3 в п/з. Добова протеїнурія - 6,6 г. Холестерин - 9,8 ммоль/л. Який наступний крок у веденні пацієнта буде найбільш доречним? A 3-year-old boy was admitted with severe edematous syndrome. Objectively: pale. Blood pressure - 90/60 mm Hg. Oliguria. General analysis of urine : color - yellow, relative density - 1020, protein - 3.5 g/l, erythrocytes - leached, 4-5 in p/z, leukocytes - 2-3 in p/z. Daily proteinuria - 6.6 g. Cholesterol - 9.8 mmol/l. What would be the most appropriate next step in the management of the patient?

Аналіз сечі за Нечипоренко Urine analysis according to Nechiporenko

Біопсія нирки Kidney biopsy

Комп'ютерна томографія без контрасту Computed tomography without contrast

Спостереження протягом тижня Observations during the week

Проба Зимницького Zymnytsky's trial

317 / 1500
У хворого 40-ка років ранковий кашель з виділенням слизово-гнійного харкотиння, підвищення температури тіла до 37,6^oC. Тютюнопаління з 17-ти років. Об'єктивно: при аускультації в легенях жорстке дихання, розсіяні сухі хрипи. ЗАК: Л- 12 Г/л, ШОЕ- 18 мм/год. При бронхоскопії: катарально-гнійний ендобронхіт. Поставте діагноз: A 40-year-old patient has a morning cough with mucus-purulent sputum discharge, an increase in body temperature to 37.6^oC. He has been smoking since the age of 17. About Objectively: during auscultation in the lungs, scattered dry wheezes. АК - 12 G/l. At bronchoscopy: make a diagnosis of purulent endobronchitis.

Туберкульоз легень Pulmonary tuberculosis

Позагоспітальна пневмонія Community-acquired pneumonia

Рак легень Lung cancer

Хронічний бронхіт Chronic bronchitis

Бронхіальна астма Bronchial asthma

318 / 1500
Для проведення інтенсивної інфузійної терапії пацієнту з гострою дихальною недостатністю виконано катетеризацію підключичної вени за Сельдінгером. Після введення 600 мл інфузійного розчину стан пацієнта різко погіршився, збільшилося тахіпное з 26 до 40/хв., ДО зменшився з 400 мл до 250 мл. Під час аускультації дихання справа різко ослаблене. Перкуторно спостерігається тупість звуку. Яке ускладнення виникло у пацієнта? For intensive infusion therapy, a patient with acute respiratory failure underwent Seldinger's subclavian vein catheterization. After administration of 600 ml of infusion solution, the patient's condition worsened sharply, tachypnea increased from 26 to 40/min., DO decreased from 400 ml to 250 ml. During auscultation, the breath is sharply weakened. Dullness of sound is observed in the patient?

Набряк легенів Pulmonary edema

Набряк мозку Brain edema

Гідроторакс Hydrothorax

Тромбоемболія легеневої артерії Thromboembolism of the pulmonary artery

Гостра серцева недостатність Acute heart failure

319 / 1500
У новонародженого терміном гестації 31 тиждень спостерігаються гіпотонія та пригнічення свідомості. Гематокрит - 35%, а в загальному аналізі ліквору вия-влено підвищену кількість еритроцитів, білка та знижений вміст глюкози. Ці дані відповідають клінічній картині: A newborn with a gestational age of 31 weeks has hypotonia and depression of consciousness. Hematocrit is 35%, and in the general analysis of the cerebrospinal fluid, an increased number of erythrocytes, protein and a decreased content of glucose. These data correspond to the clinical picture:

Менінгіту Meningitis

Внутрішньоутробної інфекції Intrauterine infection

Анемії Anemia

Сепсису Sepsis

Внутрішньочерепного крововиливу Intracranial hemorrhage

320 / 1500
Жінка 53-х років, звернулася до лікаря зі скаргами на розпираючий біль в м'язах лівої гомілки та набряк, що з'явилися тиждень тому. Раніше подібного стану не виникало. Температура тіла - 37,6^oC. При огляді гомілка в середній третині збільшена в об'ємі на 5 см. Шкіра гомілки та ступні синюшна, блискуча, відмічається місцева гіпертермія. При пальпації гомілки - різка болючість. Позитивні симптоми Хоманса та Мозеса. Пульс на артеріях стоп задовільний. Який діагноз є найбільш імовірним? A 53-year-old woman turned to the doctor with complaints of excruciating pain in the muscles of the left leg and swelling that appeared a week ago. Previously, a similar condition did not occur. Body temperature - 37.6°C. When examining the lower leg, the volume is increased by 5 cm. The skin of the lower leg and foot is bluish. When palpating the lower leg, there is a sharp pain Moses. The pulse on the arteries of the feet is satisfactory. What is the most likely diagnosis?

Післятромбофлебітичний синдром Postthrombophlebitic syndrome

- -

Гострий тромбоз підколінної вени Acute thrombosis of popliteal vein

Гострий тромбоз підколінної артерії Acute thrombosis of popliteal artery

Хвороба Рейно Raynaud's disease

321 / 1500
Матір восьмимісячного хлопчика скаржиться на здуття живота у дитини, рясні, пінисті випорожнення 3-4 рази на добу з неприємним запахом, зниження маси тіла. Шкірні покриви дитини під час огляду бліді і сухі, визначається збільшення в об'ємі живота, гепатомегалія. У загальному аналізі крові анемія. У біохімічному дослідженні крові - гіпопротеїнемія, гіпоальбумінемія, зниження холестерину, загальних ліпідів і alpha-ліпопротеїдів. Встановіть діагноз за результатами дослідження. The mother of an eight-month-old boy complains of abdominal distension in the child, copious, foamy stools 3-4 times a day with an unpleasant odor, weight loss. The child's skin during the examination is pale and dry, an increase in the volume of the abdomen is determined. In the general blood analysis, hypoproteinemia, hypoalbuminemia, a decrease in total lipids and alpha-lipoproteins are established.

Хвороба Крона Crohn's disease

Кишкова форма муковісцидозу Intestinal form of cystic fibrosis

Лактозна недостатність Lactose deficiency

Виразковий ентерит Ulcerative enteritis

Целіакія Celiac

322 / 1500
У чоловіка 35 років виникає задишка під час фізичного навантаження, він періодично втрачає свідомість. Аускультативно встановлено: систолічний шум у ІІІ міжребер'ї зліва від грудини. Під час УЗД серця встановлено: симетрична гіпертрофія ЛШ, його діастолічна дисфункція з деяким зменшенням порожнини. Спостерігається передньо-систолічний рух передньої стулки мітрального клапану. Яка патологія у хворого? A 35-year-old man develops shortness of breath during physical exertion, he periodically loses consciousness. Auscultation revealed: systolic murmur in the third intercostal space to the left of the sternum. During ultrasound of the heart: symmetric LV hypertrophy, its diastolic dysfunction with some reduction of the cavity. Ante-systolic movement of the anterior leaflet of the mitral valve is observed. What is the patient's pathology?

Дилятаційна кардіоміопатія Dilated cardiomyopathy

Рестриктивна кардіоміопатія Restrictive cardiomyopathy

Гіпертрофічна кардіоміопатія Hypertrophic cardiomyopathy

Стеноз вустя аорти Stenosis of the mouth of the aorta

Постінфарктний міокардіосклероз Postinfarction myocardial sclerosis

323 / 1500
Дівчина 22 років скаржиться на дратівливість, плаксивість, головний біль, нудоту, іноді блювоту, серцебиття, нагрубання молочних залоз, набряки на руках і ногах, метеоризм. Скарги виникають за 6-10 днів до менструації та зникають із початком місячних. Бімануально встановлено: матка та додатки без змін. Який імовірний діагноз? A 22-year-old girl complains of irritability, tearfulness, headache, nausea, sometimes vomiting, palpitations, swelling of the mammary glands, swelling of the arms and legs, flatulence. Complaints occur 6-10 days before menstruation and disappear with the onset of menstruation. Bimanually established: the uterus and appendages are unchanged. What is the probable diagnosis?

Нейро-циркуляторна дистонія Neuro-circulatory dystonia

Дисменорея Dysmenorrhea

Синдром Шихана Sheehan Syndrome

Невроз Neurosis

Передменструальний синдром Premenstrual syndrome

324 / 1500
Потерпілий, робочий хімзаводу, пра-цюючи з агресивною рідиною, по необережності вилив її собі на спецодяг, отримавши хімічний опік правого стегна та гомілки. Після зняття одягу виявлено: на передньо-внутрішній поверхні правого стегна, передній поверхні правої гомілки з переходом на тил стопи ділянки жовтаво-сірого кольору, місцями обривки епідермісу. Тактильна та больова гіпестезія уражених ділянок шкіри. Який з наступних кроків невідкладної допомоги має бути виконаний першочергово? The victim, a worker at a chemical plant, while working with an aggressive liquid, carelessly spilled it on his work clothes, receiving a chemical burn on his right thigh and lower leg. After removing the clothes, it was found: on the front-inner surface of the right thigh, with the transition to the back of the foot, a patch of epidermis. Tactile and painful hypoesthesia of the affected skin. Which of the following steps of emergency care should be performed as a priority?

Обробка розчином соди Treatment with soda solution

Обробка етиловим спиртом Ethyl alcohol treatment

Накладання асептичної пов'язки Applying an aseptic bandage

Накладання жирових пов'язок Applying fatty bandages

Промивання проточною водою Rinsing with running water

325 / 1500
Хлопчик віком 10 років лікувався в кардіологічному відділенні з приводу ревматизму, І атака, активна фаза, активність ІІ ступеня. Виписаний у задовільному стані. Який препарат найдоцільніше призначити для профілактики вторинного ревматизму? A 10-year-old boy was treated in the cardiology department for rheumatism, 1st attack, active phase, 2nd degree activity. He was discharged in a satisfactory condition. What drug is most appropriate to prescribe for prevention secondary rheumatism?

Еритроміцин Erythromycin

Біцилін-1 Bicilin-1

Ампіцилін Ampicillin

Біцилін-5 Bicillin-5

Оксацилін Oxacillin

326 / 1500
У жінки віком 35 років за 30 хвилин після внутрішньо-м'язової ін'єкції ампіциліну виникли різка слабкість, свербіж обличчя та рук, нудота, кашель, задишка, біль у грудній клітці. Об'єктивно спостерігається: ціаноз, набряк повік, обличчя, шиї, шкіра волога з червоним висипом. Пульс - 120/хв, АТ - 70/20 мм рт. ст., тони серця глухі, дихання часте, поверхневе з великою кількістю різнокаліберних вологих хрипів. З введення якого препарату необхідно негайно розпочати терапію? A 35-year-old woman developed sudden weakness, itching of the face and hands, nausea, cough, shortness of breath, 30 minutes after an intramuscular injection of ampicillin pain in the chest. Objectively observed: cyanosis, swelling of the face, neck, skin with a red rash. Pulse - 70/20 mm Hg, heart sounds are shallow with a large number of wet rales. With the introduction of which drug should the therapy be started immediately?

Еуфіліну Euphilina

Адреналіну Adrenaline

Преднізолону Prednisolone

Астмопенту Astmopentu

Допаміну Dopamine

327 / 1500
Хвора 48-ми років доставлена в лікарню з неадекватною поведінкою. Зі слів чоловіка, постійно вживала знеболювальні препарати у зв'язку з мігренню та болем у спині. Об'єктивно: жовтяничність склер, чутливість у правому верхньому квадранті живота та астериксис. БАК: АЛТ- 649 Од/л, загальний білірубін - 95,8 мкмоль/л та МНС- 6,8. Який найбільш імовірний діа-гноз? A 48-year-old patient was brought to the hospital with inappropriate behavior. According to her husband, she was constantly taking painkillers due to migraines and back pain. About' objectively: sclera, sensitivity in the right upper quadrant and asterixis. BAC - 649 U/l, total bilirubin - 95.8 μmol/l. What is the most likely diagnosis?

Інтоксикація знеболювальними засобами Intoxication with painkillers

Гостра печінкова недостатність Acute liver failure

Гостра наднирникова недостатність Acute adrenal insufficiency

Гостра ниркова недостатність Acute renal failure

Гостре порушення мозкового кровообігу Acute cerebrovascular accident

328 / 1500
Жiнка віком 25 рокiв була госпіталізована в терміні вагітності 11 тижнів зі скаргами на біль внизу живота. З анамнезу відомо, що 2 тижнi тому вона перехворiла на краснуху в тяжкiй формi. Під час бiмануального дослiдження виявлено: шийка матки сформована, вiчко шийки матки закрите, матка збільшена до 11 тижнів вагiтності, придатки матки без особливостей. Яка лікарська тактика є найдоцiльнішою в цій ситуації? A 25-year-old woman was hospitalized at 11 weeks of pregnancy with complaints of pain in the lower abdomen. It is known from the anamnesis that 2 weeks ago she was ill with rubella in a severe form During the bimanual examination, it was found that the cervix is ​​formed, the cervix is ​​closed, the uterus is enlarged to 11 weeks of pregnancy, and the appendages are unremarkable.

Пролонгування вагiтностi Prolongation of pregnancy

Введення утеротонікiв Introduction of uterotonics

Переривання вагітності Termination of pregnancy

Введення спазмолiтикiв Introduction of antispasmodics

Малий кесарiв розтин Small caesarean section

329 / 1500
Чоловік 27-ми років звернувся до лікаря зі скаргами на біль та набряк у II та III пальцях лівої стопи, почервоніння та гнійні виділення з очей, різь та біль при виділенні сечі. Зі слів пацієнта має декількох статевих партнерів, зрідка користується презервативами. При фізикальному обстеженні лікарем виявлено двосторонній кон'юнктивіт та дактиліт пальців стопи. Обстеження інших органів і систем патології не виявило. У загальному аналізі крові ШЗЕ- 40 мм/год, біохімічний аналіз без відхилень. На рентгенограмах пальців стопи збільшення об'єму м'яких тканин, звуження суглобової щілини. Який діагноз є найбільш імовірним? A 27-year-old man consulted a doctor with complaints of pain and swelling in the II and III toes of the left foot, redness and purulent discharge from the eyes, tearing and pain in according to the patient, he occasionally uses condoms. During the physical examination, the doctor revealed dactylitis of the feet. In the general analysis of the blood, it was 40 mm/h analysis without deviations. On X-rays of the toes, an increase in the volume of soft tissues, narrowing of the joint space. What is the most likely diagnosis?

Ревматоїдний артрит Rheumatoid arthritis

Реактивний артрит Reactive arthritis

Синдром Шегрена Sjogren's syndrome

Подагра Gout

Сифіліс Syphilis

330 / 1500
Провідник потягу 39 років госпіталізований на 4-й день хвороби зі скаргами на головний біль, слабкість, запаморочення, пітливість, безсоння, лихоманку. Обличчя гіперемійоване, набрякле, кон'юнктивіт. На перехідній складці кон'юнктиви - поодинокі петехії. На шкірі тулуба, грудної клітки, живота, кінцівок - рясна розеольозно-петехіальна висипка. Тахікардія. АТ - 100/60 мм рт. ст. Спостерігається тремор язика. Пальпується печінка, селезінка. Випорожнення затримане. Який найімовірніший діагноз? A 39-year-old train conductor was hospitalized on the 4th day of illness with complaints of headache, weakness, dizziness, sweating, insomnia, fever. The face is hyperemic, swollen, con Conjunctivitis - single petechiae on the trunk, chest, extremities. Tachycardia - 100/60 mmHg. Palpable liver . Defecation is delayed. What is the most likely diagnosis?

Менінгококцемія Meningococcemia

Черевний тиф

Висипний тиф Typhoid

Грип Flu

Лептоспіроз Leptospirosis

331 / 1500
Дитина віком 5 років захворіла гостро, спостерігаються підвищення температури тіла до 39,2^oC, одноразове блювання, скарги на переймоподібний біль у животі, тенезми, часті випорожнення: малокалові, з великою кількістю слизу, домішками гною та прожилок крові. Під час огляду виявлено: ущільнена сигмоподібна кишка, болюча під час пальпації. Встановіть діагноз. A 5-year-old child became acutely ill, there was an increase in body temperature to 39.2^oC, one-time vomiting, complaints of spasmodic abdominal pain, tenesmus, frequent bowel movements: scanty, with a large amount of mucus, pus and streaks of blood. During the examination, it was found that the sigmoid colon was painful during palpation.

Ротавірусна інфекція Rotavirus infection

Шигельоз Shigelosis

Гострий апендицит Acute appendicitis

Сальмонельоз Salmonellosis

Холера Cholera

332 / 1500
Дитина 2 років була в контакті з хворим на оперізуючий герпес. На 15-й день контакту в неї підвищилась температура тіла до 39,3^oC, з'явився висип на тулубі, кінцівках, обличчі та волосистій частині голови, який проходив стадії: пляма, папула, визикула, кірочка. Який імовірний діагноз? A 2-year-old child was in contact with a patient with herpes zoster. On the 15th day of contact, her body temperature rose to 39.3^oC, appeared a rash on the trunk, limbs, face and scalp, which went through the stages: spot, papule, vesicle, crust. What is the probable diagnosis?

Оперізуючий герпес Herpes zoster

Ентеровірусна екзантема Enterovirus exanthema

Натуральна віспа Smallpox

Вітряна віспа Chicken Pox

Простий герпес Herpes simplex

333 / 1500
Чоловік 45 років скаржиться на втрату апетиту, запаморочення, виражену загальну слабкість, субфебрильну температуру. Шкіра бліда, синці на тулубі та кінцівках. ЗАК виявив: еритроцити - 2,9·10^12/л, Нb - 96 г/л, КП - 0,99, ретикулоцити - 0,02%, лейкоцити - 2,9·10^9/л, тромбоцити - 85·10^9/л, ШОЕ - 45 мм/год. Яке додаткове дослідження є найбільш доцільним для верифікації діагнозу? A 45-year-old man complains of loss of appetite, dizziness, pronounced general weakness, subfebrile temperature. The skin is pale, bruises on the trunk and limbs. ZAK revealed: erythrocytes - 2, 9·10^12/l, Hb - 96 g/l, CP - 0.99, reticulocytes - 0.02%, leukocytes - 2.9·10^9/l, platelets - 85·10^9/l, ESR - 45 mm/h. What additional research is most appropriate for diagnosis verification?

Коагулограма Coagulogram

Стернальна пункція Sternal puncture

Рівень сироваткового заліза Serum iron level

Вміст В_12 в крові B_12 content in blood

Осмотична резистентність еритроцитів Osmotic resistance of erythrocytes

334 / 1500
Пацiєнтка віком 45 рокiв скаржиться на дискомфорт пiд час читання, почервонiння краю повiк, бiлi пiнистi видiлення в кутах очних щiлин протягом останнiх двох мiсяцiв. Об'єктивно спостерiгається: гiперемiя та потовщення вiльного краю повiк, розширення вивiдних протокiв залоз хряща повiк. Який найімовірніший діагноз? A 45-year-old patient complains of discomfort while reading, reddening of the edge of the eyelids, white foamy discharge in the corners of the eye slits for the past two months. Objectively observed: hyperemia and thickening of the free edge of the eyelids, expansion of the excretory ducts of the eyelid cartilage. What is the most likely diagnosis?

Бленорейний кон'юнктивiт Blenorrhoeic conjunctivitis

Хронiчний каналiкулiт Chronic canaliculitis

Аденовiрусний кон'юнктивiт Adenovirus conjunctivitis

Гострий дакрiоаденiт Acute dacryoadenitis

Мейбомiєвий блефарит Meibomian blepharitis

335 / 1500
Пацієнт 72 років надійшов до відділення судинної хірургії зі скаргами на біль та мерзлякуватість нижніх кінцівок. УЗД артерій нижніх кінцівок виявило атеросклеротичне ураження судинної стінки. Який метод діагностики потрібний для уточнення локалізації та протяжності процесу? A 72-year-old patient came to the department of vascular surgery with complaints of pain and numbness of the lower extremities. Ultrasound of the arteries of the lower extremities revealed atherosclerotic lesions of the vascular wall. What diagnostic method is needed to clarify localization and duration of the process?

Ангіографія Angiography

Рентгенографія кінцівки Extremity X-ray

Термометрія Thermometry

Рентгенографія органів грудної порожнини X-ray of chest cavity

ЕКГ ECG

336 / 1500
Хворий 26-ти років звернувся до лікаря зі скаргами на озноб, нежить, сухий кашель, лихоманку до 38^oC. В ході огляду: стан середньої тяжкості, на шкірі спини, живота, та кінцівок блідо-рожеві дрібні незливні плями. Пальпуються збільшені потиличні, аксилярні лімфовузли. Відомостей про щеплення немає. Яка ймовірна етіологія даного захворювання? A 26-year-old patient turned to the doctor with complaints of chills, runny nose, dry cough, fever up to 38°C. During the examination: a condition of moderate severity, on the skin of the back, abdomen, and limbs are pale pink. There are enlarged occipital and axillary lymph nodes. There is no information about the etiology of this disease.

Менінгокок Meningococcus

Стрептокок Streptococcus

Вірус епідемічного паротиту Epidemic mumps virus

Вірус краснухи Ruellas virus

Вірус Епштейна-Барр Epstein-Barr virus

337 / 1500
Чоловік 30 років скаржиться на висип та свербіння шкіри стоп. Хворіє 3 роки. Об'єктивно в ділянці шкіри підошов спостерігаються групи пухирців, схожих на розварені сагові зерна, а також ерозії з уривками мацерованого епідермісу на периферії вогнищ. У міжпальцевих складках обох стоп видно тріщини, ерозії. Яка патологія найімовірніша? A 30-year-old man complains of a rash and itching of the skin of his feet. He has been ill for 3 years. Objectively, in the area of ​​the skin of the soles, groups of blisters similar to boiled sago grains are observed, and also erosions with fragments of macerated epidermis on the periphery of the foci. Cracks, erosions are visible in the interdigital folds of both feet. What is the most likely pathology?

Псоріаз Psoriasis

Вторинний сифіліс Secondary syphilis

Руброфітія стоп Rubrophytia of the feet

Дерматит Dermatitis

Епідермофітія стоп Epidermophyta of the feet

338 / 1500
Жінка 22-х років звернулася до лікаря з метою планування вагітності. В анамнезі цукровий діабет протягом двох років. Діа-бетичних ускладнень при опитуванні та огляді не виявлено. Глікемія натще в межах 6,0-7,0 ммоль/л. Який аналіз серед перерахованих слід призначити, щоб адекватно оцінити контроль глікемії? A 22-year-old woman consulted a doctor to plan a pregnancy. She has a history of diabetes for two years. Diabetic complications were not detected during the survey and examination. Glycemia fasting in the range of 6.0-7.0 mmol/l. Which analysis among the listed should be prescribed to adequately assess glycemic control?

Рівень С-пептиду C-peptide level

Постпрандіальна глікемія Postprandial glycemia

Глікемічний профіль Glycemic Profile

Глюкозуричний профіль Glucosuric profile

Рівень глікозильованого (глікованого) гемоглобіну (НbА1c) The level of glycosylated (glycated) hemoglobin (HbA1c)

339 / 1500
Під час проходження допризивної комісії у хлопця 17 років виявлена артеріальна гіпертензія - АТ на руках 190/110 мм рт. ст. Скарг немає. Звертає на себе увагу непропорційна будова тіла - добре розвинений плечовий пояс і недорозвинені нижні кінцівки. Який метод дослідження, найвірогідніше, найбільш інфор-мативний для постановки діагнозу? During the pre-conscription commission, a 17-year-old boy was diagnosed with hypertension - blood pressure in the arms of 190/110 mm Hg. There are no complaints. The disproportionate structure attracts attention bodies - a well-developed shoulder girdle and underdeveloped lower limbs. Which research method is most likely to be the most informative for making a diagnosis?

Реносцинтіграфія Renoscintigraphy

Визначення катехоламінів у сечі Determination of catecholamines in urine

УЗД нирок і наднирників Ultrasound of kidneys and adrenal glands

Вимірювання АТ на нижніх кінцівках BP measurement on lower limbs

Доплер-сонографія судин Doppler sonography of vessels

340 / 1500
Жінка 36 років скаржиться на біль у надчеревній ділянці, що виникає після вживання їжі, нудоту, відрижку, нестійкість випорожнень. Хвороба поступово прогресує впродовж 2 років. Об'єктивно спостерігаються блідість і сухість шкіри, язик обкладений і вологий з відбитками зубів по краях. Під час пальпації живота спостерігається розлитий біль у надчеревній ділянці. Який найінформативніший метод дослідження слід застосувати в цьому разі? A 36-year-old woman complains of pain in the epigastric region, which occurs after eating, nausea, belching, stool instability. The disease gradually progresses over 2 years. Objectively paleness and dryness of the skin are observed, the tongue is coated and wet with imprints of the teeth. During palpation of the abdomen, diffuse pain is observed in the epigastric region. What is the most informative method of examination in this case?

Рентгенологічне дослідження шлунка і кишечника X-ray examination of the stomach and intestines

Розгорнутий клінічний аналіз крові Expanded clinical blood analysis

Комп'ютерне дослідження черевної порожнини Computer examination of the abdominal cavity

Фракційне дослідження шлункової секреції Fractional study of gastric secretion

Фіброгастроскопія з біопсією слизової шлунка Fibrogastroscopy with gastric mucosa biopsy

341 / 1500
Хвора 45-ти років звернулася зі скаргами на прогресуюче збільшення маси тіла за останній рік. При огляді: місяцеподібне обличчя, ламке волосся, гірсутизм, на животі смуги розтягу, кінцівки непропорційно тонкі. Зріст - 162 см, маса тіла - 94 кг, ІМТ - 35,8 кг/м^2. Який тип ожиріння притаманний для хворої? A 45-year-old patient complained of a progressive increase in body weight over the past year. On examination: a moon-shaped face, brittle hair, hirsutism, stretch marks on the abdomen, limbs are disproportionately thin. Height - 162 cm, body weight - 94 kg, BMI - 35.8 kg/m^2. What type of obesity is characteristic of the patient?

Гіноїдне Gynoid

Аліментарно-конституційне Alimentary and constitutional

Диспластичне Dysplastic

Церебральне Cerebral

Андроїдне Android

342 / 1500
Жінка 55 років скаржиться на багаторазовий пронос, лущення та пігментацію відкритих ділянок тіла (шиї, кистей та ступней), роздратованість та занепокоєння. Про яку вітамінну недостат-ність йдеться? A 55-year-old woman complains of repeated diarrhea, peeling and pigmentation of open areas of the body (neck, hands and feet), irritability and anxiety. What vitamin deficiency is this? ?

Рибофлавіну Riboflavin

Тіаміну Thiamine

Пантотенової кислоти Pantothenic acid

Нікотинової кислоти Nicotinic acid

Ретинолу Retinol

343 / 1500
Дитині 4 доби. Стан після народження важкий (клоніко-тонічні судоми, рефлекси новонародженого не викликаються). Тонус м'язів асиметричний. Велике тім'ячко 3х3 см, пульсація його підвищена. ЧД - 32/хв., напади апное. Тони серця ясні, ритмічні, ЧСС - 122/хв. Нейросонограма показала: бокові шлуночки збільшені, з ехопозитивними включеннями в епендимі. Укажіть найімовірнішу причину порушення стану дитини: The child is 4 days old. The condition after birth is severe (clonic-tonic convulsions, reflexes of the newborn are not evoked). Muscle tone is asymmetric. Large parietal lobe 3x3 cm, its pulsation is increased. Heart rate is 32/min. Heart sounds are clear, heart rate is 122/min. The lateral ventricles are enlarged, with echopositive inclusions in the ependyma. State the most likely cause of the child's condition:

Спінальна пологова травма Spinal childbirth injury

Менінгіт Meningitis

Гідроцефалія Hydrocephalus

Синдром дихальних розладів Syndrome of respiratory disorders

Внутрішньошлуночковий крововилив Intraventricular hemorrhage

344 / 1500
У чоловіка віком 42 роки на тлі психотравмуючих обставин (втрата роботи, хвороба дружини) поступово з'явилося безсоння, пригнічений настрій, зниження апетиту. Втратив інтерес до повсякденної діяльності, перестав справлятися з роботою, з’явилися думки про небажання жити. Які лікарські засоби потрібно йому призначити? A 42-year-old man, against the background of psycho-traumatic circumstances (loss of job, illness of his wife), gradually developed insomnia, depressed mood, decreased appetite. He lost interest in everyday activities , stopped coping with work, thoughts about unwillingness to live appeared. What medicines should he be prescribed?

Снодійні Sedatives

Антипсихотики Antipsychotics

Нейролептики Neuroleptics

Анксіолітики Anxiolytics

Антидепресанти Antidepressants

345 / 1500
Перед випискою додому на третій день життя у доношеної новонародженої дівчинки спостерігались кількаразове блювання з домішкою крові і випорожнення у вигляді ''малинового желе''. Відомо, що матір відмовилась від внутрішньом'язових ін'єкцій своїй дитині. Виберіть найбільш інформативний лабораторний показник для підтвердження діагнозу: Before discharge home on the third day of life, a full-term newborn girl had repeated vomiting with blood and stools in the form of 'raspberry jelly'. It is known that the mother refused from intramuscular injections to your child. Choose the most informative laboratory indicator to confirm the diagnosis:

Протромбіновий час Prothrombin time

Тривалість кровотечі Duration of bleeding

Кількість тромбоцитів Number of platelets

Парціальний (частковий) тромбопластиновий час Partial thromboplastin time

Тромбіновий час Thrombin time

346 / 1500
Дільничному лікарю доручили підготувати план проведення комплексу лікувально-профілактичних заходів серед населення на підпорядкованій території. Які заходи щодо вторинної профілактики захворювань він повинен внести до цього плану? The district doctor was instructed to prepare a plan for carrying out a complex of medical and preventive measures among the population in the subordinate territory. What measures for the secondary prevention of diseases should he include in this plan?

Попередження ускладнень захворювань Prevention of complications of diseases

Поліпшення умов життя населення Improving living conditions of the population

Усунення чинників виникнення захворювань Elimination of factors causing diseases

Попередження виникнення захворювань Disease prevention

Проведення реабілітаційних заходів Carrying out rehabilitation measures

347 / 1500
Збуджений пацієнт весь час намагається зазирнути за двері та в сусіднє приміщення, бо переконаний, що там знаходяться його приятелі. Стверджує, що чує розмову друзів та сторонніх людей, хоча в цей час нікого поблизу немає. Намагається переконати лікаря, що за стіною відбувається суперечка з приводу 'його покарання', голосно повторює фрази, які ніби то почув з-за стіни. Який патологічний стан спостерігається у пацієнта? Agitated patient constantly tries to look outside the door and into the next room, because he is convinced that his friends are there. Claims that he can hear the conversation of friends and strangers, although at this time there is no one nearby. He tries to convince the doctor that there is a dispute about his punishment, loudly repeating the phrases that he heard from behind the wall. What pathological condition is observed in the patient?

Конфабуляції Confabulations

Маячення Delusion

Вербальні ілюзії Verbal illusions

Гострий галюциноз Acute hallucinosis

Нав'язливі ідеї Obsessions

348 / 1500
Пацієнтка віком 14 років скаржиться на наявність плямисто-папульозного висипу вздовж тіла, підвищення температури тіла до 38,5^oС та біль у горлі. Об'єктивно спостерігається: збільшені шийні та підщелепні лімфатичні вузли, слизова ротоглотки гіперемована, язик та мигдалики обкладені білим нальотом. Пальпаторно виявлено: гепато- та спленомегалія. Який найімовірніший діагноз? A 14-year-old patient complains of the presence of a spotted-papular rash along the body, an increase in body temperature to 38.5°C, and a sore throat. Objectively observed: enlarged cervical and submandibular lymph nodes, the oropharyngeal mucosa is hyperemic, the tongue and tonsils are coated with white plaque. Palpation reveals: hepatomegaly and splenomegaly. What is the most likely diagnosis?

Скарлатина Scarlatina

Вірусний гепатит А Viral hepatitis A

Хронічний мієлолейкоз Chronic myelogenous leukemia

Ентеровірусна інфекція Enterovirus infection

Інфекційний мононуклеоз Infectious mononucleosis

349 / 1500
Жінка 52-х років прийшла до свого лікаря на щорічний профілактичний огляд. Вона ніколи не госпіталізувалася до лікарні та виконує легкі аеробні вправи чотири рази на тиждень. У її батька було захворювання серцево-судинної системи. Її зріст - 172 см, вага - 77 кг, ІМТ- 26 кг/м^2. При фізикальному обстеженні температура - 36,8^oC, пульс - 81/хв., частота дихання - 16/хв., артеріальний тиск сидячи - 160/100 мм рт.ст. на обох руках. Офтальмоскопія очного дна виявила звуження судин сітківки. Лабораторні показники без відхилень. Що з перерахованого буде найбільш доречним призначенням з метою зниження ризику смерті цієї пацієнтки від серцево-судинних захворювань? A 52-year-old woman comes to her doctor for an annual check-up. She has never been hospitalized and does light aerobic exercise four times a week. Her father had cardiovascular disease. Her height - 172 cm, weight - 26 kg/m^2. During physical examination, temperature - 36.8°C, pulse - 81/min., respiratory rate - 16/ min., blood pressure while sitting - 160/100 mm Hg. Ophthalmoscopy revealed narrowing of the retinal vessels. Which of the following would be the most appropriate appointment for the purpose of reducing the risk of cardiovascular death in this patient diseases?

Еналаприл Enalapril

Варфарин Warfarin

Дипіридамол Dipyridamole

Аспірин Aspirin

Нітрогліцерин Nitroglycerin

350 / 1500
Чоловік 25 років скаржиться на швидку втому, серцебиття, напади задишки переважно вночі. Хворіє з дитинства. Шкіра бліда. Індекс маси тіла - 17,5. Тони серця ритмічні, 98/хв, у ІІ міжреберному проміжку ліворуч від грудини вислуховується високочастотний <<машинний>> шум, який починається після І тону, досягає максимуму під час ІІ тону і стихає в діастолі. Який найбільш імовірний діагноз? A 25-year-old man complains of rapid fatigue, palpitations, attacks of shortness of breath mainly at night. He has been ill since childhood. The skin is pale. Body mass index is 17.5. Heart sounds are rhythmic , 98/min, in the II intercostal space to the left of the sternum, a high-frequency <> noise is heard, which begins after the I tone, reaches a maximum during the II tone and subsides in diastole. What is the most likely diagnosis?

Первинний дефект міжпередсердної перетинки Primary atrial septal defect

Коарктація аорти Coarctation of the aorta

Вторинний дефект міжпередсердної перетинки Secondary atrial septal defect

Аномалія Ебштейна Ebstein anomaly

Відкрита аортальна протока Open aortic duct

351 / 1500
До лікаря звернувся чоловік 36-ти років зі скаргами на пекучий загрудинний біль, що зазвичай виникає через 1-1,5 години після вживання їжі. Зазначає, що біль підсилюється в горизонтальному положенні. При верхній ендоскопії виявлені 2 вогнищевих ураження нижньої третини слизової оболонки стравоходу до 5 мм в межах однієї складки. Яка тактика лікаря буде найбільш доречною? A 36-year-old man went to the doctor with complaints of burning chest pain, which usually occurs 1-1.5 hours after eating. He notes that the pain increases in the horizontal position. During the upper endoscopy, 2 focal lesions of the lower third of the esophagus were detected within one fold. What would be the most appropriate tactics of the doctor?

Хірургічне лікування Surgical treatment

Призначення кларитроміцину Clarithromycin Prescription

Призначення ненаркотичних анальгетиків Prescription of non-narcotic analgesics

Призначення інгібіторів протонної помпи Prescription of proton pump inhibitors

- -

352 / 1500
Пацієнт віком 47 років, звернувся до лікаря-невролога зі скаргами на біль у поперековій ділянці. Встановлено попередній діагноз: радикуліт, та проведено курс фізіотерапевтичного лікування. Однак, стан пацієнта не покращився. На рентгенограмі хребта і таза виявлено остеопороз та значні кісткові дефекти. У загальному аналізі крові спостерігається нормохромна анемія, загальний білок - 107 г/л. За результатами загального аналізу сечі виявлено протеїнурію - 4,0 г/л. Яке дослідження необхідно провести для встановлення остаточного діагнозу? A 47-year-old patient turned to a neurologist with complaints of pain in the lumbar region. A preliminary diagnosis of sciatica was made, and a course of physiotherapy treatment was carried out. However, the condition The patient did not improve. Osteoporosis and significant bone defects were detected in the general blood analysis. According to the results of the general urine analysis, proteinuria was detected - 4.0 g/l conduct to establish a final diagnosis?

Цитохімічне дослідження клітин крові

Стернальну пункцію Sternal puncture

Радіоізотопне дослідження нирок Radioisotope examination of kidneys

Розгорнутий аналіз крові Expanded blood analysis

УЗД органів черевної порожнини Ultrasound of abdominal organs

353 / 1500
У дівчини 19-ти років скарги на головний біль пульсуючого характеру в лівій лобно-скронево-тім'яній ділянці, який турбує протягом п'яти років. Головний біль виникає нападоподібно і супроводжується нудотою, блюванням, фонофобією та фотофобією. Тривалість нападу від кількох годин до 2-3 діб. Напад цефалгії часто виникає при перевтомі, емоційному перенавантаженні. Частота нападів 1-2 рази на місяць. Неврологічний статус без патологічних змін. Схожі головні болі були у матері хворої. Який діагноз є найбільш імовірним? A 19-year-old girl complains of a throbbing headache in the left frontal-temporal-parietal area, which has been bothering her for five years. Headache occurs in an attack-like manner and is accompanied by nausea, vomiting and photophobia. Attacks of cephalgia often occur during fatigue, emotional overload. Neurological status without pathological changes The patient's mother had pain. What is the most likely diagnosis?

Хронічна цервікокраніалгія Chronic cervicocranialgia

Кластерний головний біль Cluster Headache

Головний біль напруги Tension headache

Мігрень Migraine

Лікворно-динамічний головний біль Liquor-dynamic headache

354 / 1500
Хворий скаржиться на почервоніння, печіння, відчуття стороннього тіла у правому оці. Захворів раптово. Під час огляду: гострота зору правого та лівого ока - 1,0. У правому оці - гіперемія та набряк кон'юнктиви, поверхнева ін'єкція. У кон'юнктивальному мішку гнійне виділення. Рогівка прозора. Колір та малюнок райдужки не змінені, зіниця рухлива. Який найбільш імовірний діагноз? The patient complains of redness, burning, feeling of a foreign body in the right eye. He fell ill suddenly. During the examination: the visual acuity of the right and left eye is 1.0. in the right eye - hyperemia and swelling of the conjunctiva. Purulent discharge in the conjunctival sac. The color and pattern of the iris are not changed. What is the most likely diagnosis?

Гострий іридоцикліт Acute iridocyclitis

Гострий напад глаукоми Acute attack of glaucoma

Стороннє тіло рогівки Corneal foreign body

Гострий дакріоцистит Acute dacryocystitis

Гострий кон'юнктивіт Acute conjunctivitis

355 / 1500
У жінки 25 років за два тижні після проведення антибактеріальної терапії з приводу гарячкового захворювання розвинулася тяжка водяниста діарея, з'явився переймоподібний біль у животі, підвищилася температура тіла. Під час проктосигмоскопії виявлені вогнища пошкодження слизової з блідо-жовтим налітом. Який діагноз найімовірніший? A 25-year-old woman developed severe watery diarrhea, spasm-like pain in the abdomen, and increased body temperature two weeks after antibacterial therapy for a febrile illness. during proctosigmoscopy foci of mucosal damage with pale yellow plaque were detected. What is the most likely diagnosis?

Псевдомембранозний коліт Pseudomembranous colitis

Хвороба Крона Crohn's disease

Гастроентерит Gastroenteritis

Виразковий коліт Ulcerative colitis

Ішемічний коліт Ischemic colitis

356 / 1500
Жінка 34 років звернулася до лікаря з приводу м'язової слабкості, спраги, підвищення сечовиділення в нічний час, парестезій, нападів судом. Об'єктивно встановлено: загальний стан задовільний, пастозність обличчя, ніг. Ps - 80/хв. АТ - 200/110 мм рт. ст., акцент ІІ тону над аортою. У крові виявлено: К^+ - З,1 ммоль/л, Na^+ - 165 ммоль/л. На ЕКГ: інверсія зубця Т, депресія сегменту S-T. Під час УЗД спостерігалася: гіперплазія правої надниркової залози. Який попередній діагноз? A 34-year-old woman consulted a doctor about muscle weakness, thirst, increased urination at night, paresthesias, seizures. Objectively established: general condition Satisfactory, blood pressure of the legs - 80/min, pressure of II tone above the aorta: K^+ - 165 mmol/l. On the ECG: inversion of the ST segment. During the ultrasound: hyperplasia of the right adrenal gland was observed?

Глюкостерома Glucosteroma

Есенціальна гіпертонія Essential hypertension

Гіпопаратиреоз Hypoparathyroidism

Феохромоцитома Pheochromocytoma

Первинний гіперальдостеронізм Primary hyperaldosteronism

357 / 1500
Жінку віком 50 років шпиталізовано зі скаргами на серцебиття, перебої в роботі серця, запаморочення, задишку, що виникли вранці. Під час огляду спостерігається: стан важкий, шкірні покриви бліді, кінцівки холодні. Пульс - 150/хв, малого наповнення, АТ - 80/50 мм рт. ст. Тони серця послаблені, аритмія. На ЕКГ виявлено: зубець Р відсутній, f-хвилі, інтервали R-R різні. Яке порушення ритму серця розвинулося у пацієнтки? A 50-year-old woman was hospitalized with complaints of palpitations, heart failure, dizziness, shortness of breath that occurred in the morning. During the examination, the following was observed: the condition was severe, the skin pale, pulse - 150/min, blood pressure - 80/50 mm Hg. Heart rhythm is absent, f-waves are different developed in the patient?

Надшлуночкова тахікардія Supraventricular tachycardia

Шлуночкова тахікардія Ventricular tachycardia

Фібриляція шлуночків Ventricular fibrillation

Фібриляція передсердь Atrial fibrillation

Екстрасистолія Extrasystole

358 / 1500
Чоловік віком 40 років скаржиться на висипання по всьому тілу, незначне свербіння. Хворіє 3 місяці, висип пов'язує з нервово-психічною травмою. Об'єктивно спостерігається: на шкірі тулуба, розгинальних поверхнях кінцівок і волосистої частини голови наявні множинні рожеві папули, вкриті сріблястими лусочками. Яким буде попередній діагноз? A 40-year-old man complains of a rash all over his body, slight itching. He has been sick for 3 months, the rash is associated with a neuropsychiatric injury. Objectively observed: there are multiple pink papules covered with silvery scales on the skin of the trunk, extensor surfaces of the limbs and scalp. What will be the preliminary diagnosis?

Розповсюджений псоріаз Extensive psoriasis

Папульозний сифілід Papular syphilis

Червоний плескатий лишай Red lichen planus

Себорейний дерматит Seborrheic dermatitis

Рожевий лишай Жибера Zhiber's pink lichen

359 / 1500
У чотириденого хлопчика з'явились прояви геморагічної хвороби новонароджених у вигляді мелени. Який препарат повинен був призначити лікар в перші години життя для профілактики цього захворювання? A four-day-old boy developed symptoms of hemorrhagic disease of newborns in the form of melena. What drug should the doctor have prescribed in the first hours of life to prevent this disease?

Амінокапронову кислоту Aminocaproic acid

Вітамін С Vitamin C

Глюконат кальцію Calcium gluconate

Етамзилат Etamsylate

Вітамін К Vitamin K

360 / 1500
У пацієнта раптово з'явився різкий біль у правій половині грудної клітки. Швидко наросла задишка. Об'єктивно спостерігається виражений акроціаноз. Стан пацієнта важкий. Визначається підшкірна емфізема в ділянці шиї і верхніх відділів грудної клітки. Над правою легенею прослуховується коробковий звук, дихання відсутнє. Межі серця зсунуті вліво. ЧСС - 110/хв., АТ - 100/60 мм рт. ст. Яке захворювання найімовірніше у пацієнта? The patient suddenly developed a sharp pain in the right half of the chest. Shortness of breath quickly increased. Objectively, pronounced acrocyanosis is observed. The patient's condition is severe. Subcutaneous emphysema is detected in in the neck and upper chest. There is no chest sound. The heart rate is 110/min. What is the most likely disease in the patient?

Ексудативний плеврит Exudative pleurisy

Спонтанний пневмоторакс Spontaneous pneumothorax

Інфаркт легені Pulmonary infarction

Позагоспітальна пневмонія Community-acquired pneumonia

Інфаркт міокарда Myocardial infarction

361 / 1500
У хлопчика 7 років раптово виник біль, набрякло праве коліно. Напередодні він брав участь у кросі пересічною місцевістю. У сімейному анамнезі даних про гемофілію та підвищену кровоточивість немає. Об'єктивно спостерігається: температура тіла - 37,5^oC. Коліно болісне під час пальпації, гаряче на дотик, набрякле з локальним напруженням тканин над ним. У крові виявлено: Нb - 123 г/л, лейкоцити - 5,6·10^9/л, тромбoцити - 354·10^9/л, протромбіновий час - 12 сек (норма - 10-15 сек), частково активований тромбопластиновий час - 72 с (норма - 35-45 с). Час кровотечі нормальний, фактор VIII: - 5% від норми. Поставте діагноз. A 7-year-old boy suddenly developed pain, his right knee was swollen. The day before, he participated in cross-country skiing. There is no family history of hemophilia or increased bleeding. About objectively observed: body temperature - 37.5°C. Knee is painful during palpation, swollen with local tension of tissues above it. Blood - 123 g/l, leukocytes - 5.6·10 9/l, platelets - 354·10^9/l, prothrombin time - 12 sec (normal - 10-15 sec), partially activated thromboplastin time - 72 s (normal - 35-45 s). Bleeding time is normal, factor VIII : - 5% of the norm. Make a diagnosis.

Хвороба Шенлейна-Геноха Schönlein-Henoch disease

Гемофілія В Hemophilia B

Нестача вітаміну К Vitamin K deficiency

Тромбоцитопенія Thrombocytopenia

Гемофілія А Hemophilia A

362 / 1500
Жінка 38 років скаржиться на висип упродовж року після сильного стресу. На шкірі волосистої частини голови, попереку, розгинальних поверхонь ліктьових та колінних суглобів бляшки та папули яскраво-червоного кольору, укриті сріблясто-білими лусочками. Під час застосування методу пошкрябування - симптоми <<стеаринової плями>>, <<термінальної плівки>>, <<кров'яної роси>>. Який імовірний діагноз? A 38-year-old woman complains of a rash for a year after severe stress. Bright red plaques and papules on the scalp, lower back, extensor surfaces of the elbow and knee joints , covered with silvery-white scales. When using the scraping method - symptoms of <>, <>. What is the probable diagnosis?

Піодермія Pyoderma

Червоний плескатий лишай Red lichen planus

Вторинний сифіліс Secondary syphilis

Псоріаз Psoriasis

Мікроспорія Microsporia

363 / 1500
Хвора 55-ти років скаржиться на наяв-ність пухлинного утворення, що пальпується в зоні ниркової ділянки зліва, помірний тупий біль у лівому боці, періодичну появу яскраво-червоної крові при сечовипусканні. Об'єктивно: хвора схудла, шкіра бліда, суха. При пальпації живота в лівій нирковій ділянці наявне еластичне утворення, що зміщується, безболісне. Аналіз сечі: макрогематурія, атипові клітини. Hb крові - 87 г/л, ШОЕ- 70 мм/год. Яка найбільш імовірна патологія, що зумовлює таку картину? A 55-year-old patient complains of the presence of a palpable tumor in the area of ​​the left kidney, moderate dull pain in the left side, periodic appearance of bright red of blood during urination. The patient has lost weight, the skin is dry. On palpation, there is an elastic formation in the left renal area. Urine analysis: Hb - 87 g/l. 70 mm/h. What is the most likely pathology that causes such a picture?

Хронічний панкреатит Chronic pancreatitis

Гострий пієлонефрит Acute pyelonephritis

Пухлина лівої нирки Left kidney tumor

Пухлина лівого сечоводу Left ureter tumor

Пухлина товстої кишки Colon tumor

364 / 1500
У пацієнта віком 23 роки об’єктивно спостерігається: шкірні покриви жовтяничні, склери іктеричні, пульс - 66/хв, АТ - 120/80 мм рт. ст., АЛТ, АСТ, тимолова проба - норма. Загальний білірубін - 34 мкмоль/л за рахунок непрямого. Для якої патології характерні ці симптоми? A 23-year-old patient objectively observes: jaundiced skin, icteric sclera, pulse - 66/min, blood pressure - 120/80 mm Hg. , ALT, AST, thymol test - normal. Total bilirubin - 34 μmol/l due to indirect. For which pathology are these symptoms characteristic?

Синдрому холестазу Cholestasis syndrome

Цирозу печінки Liver cirrhosis

Хронічного криптогенного гепатиту Chronic cryptogenic hepatitis

Синдрому Жильбера Gilbert syndrome

Хронічного вірусного гепатиту С Chronic viral hepatitis C

365 / 1500
Хворий 30-ти років надійшов в клініку зі скаргами на задишку, гарячку, біль за грудиною, який посилюється при закиданні голови назад. 3 дні тому хворому проведена екстракція сьомого зуба нижньої щелепи праворуч і розкрита флегмона дна порожнини рота. Незважаючи на це, стан хворого прогресивно погіршувався. При рентгенологічному обстеженні в клініці виявлено розширення тіні середостіння і зниження прозорості його відділів. Яке захворювання виникло у даного хворого? A 30-year-old patient came to the clinic with complaints of shortness of breath, fever, pain behind the sternum, which worsens when the head is thrown back. 3 days ago, the patient underwent extraction of the seventh tooth of the lower jaw on the right and an open phlegmon of the floor of the mouth. Despite this, the patient's condition progressively worsened. During the X-ray examination in the clinic, an expansion of the mediastinum and a decrease in the transparency of the patient's departments were found?

Гострий гнійний медіастиніт Acute purulent mediastinitis

Гнійний тиреоїдит Suppurative thyroiditis

Заглотковий абсцес Pharyngeal abscess

Інфекційний ендокардит Infective endocarditis

Ексудативний перикардит Exudative pericarditis

366 / 1500
Матір шестирічного хлопчика скаржиться на підвищення температури тіла у дитини до 37,9^oС, вологий малопродуктивний кашель, інспіраторну задишку під час фізичного навантаження, загальну слабкість. З анамнезу відомо, що захворів гостро 3 дні тому, після контакту із хворою на ГРВІ сестрою. Під час обстеження лікар-педіатр діагностувала позагоспітальну двобічну вогнищеву пневмонію, ІІ ступеня тяжкості, гострий перебіг, неускладнену форму, ДН І ступеня. Який антибактеріальний засіб потрібно призначити дитині? The mother of a six-year-old boy complains of an increase in the child's body temperature up to 37.9^oС, a wet low-productive cough, inspiratory shortness of breath during physical exertion, general weakness. From the anamnesis It is known that he became acutely ill 3 days ago, after contact with a sister with SARS. During the examination, the pediatrician diagnosed an out-of-hospital bilateral focal pneumonia, acute, uncomplicated form, DN of the 1st degree.

Цефалоспорини ІІІ покоління Third generation cephalosporins

Аміноглікозиди Aminoglycosides

Макроліди Macrolides

Цефалоспорини ІІ покоління II generation cephalosporins

Амоксицилін Amoxicillin

367 / 1500
На прийом до лікаря акушера-гінеколога жіночої консультації звернулася жінка зі строком вагітності 10 тижнів (перше звертання). Який з медичних документів повинен бути заведений лікарем у цьому випадку для контролю за перебігом вагітності? A 10-week pregnant woman applied for an appointment with an obstetrician-gynecologist at the women's consultation (first visit). Which of the medical documents should be prepared by the doctor in this case for pregnancy control?

Медична карта амбулаторного хворого Medical card of an outpatient

Карта обліку диспансеризації Dispensary accounting card

Контрольна карта диспансерного спостереження Dispensary monitoring control card

Історія пологів Birth history

Індивідуальна карта вагітної, породіллі Individual card of a pregnant woman, a woman giving birth

368 / 1500
Пацієнта віком 32 роки шпиталізовано до хірургічного відділення за 2 години після закритої травми живота з клінічними ознаками геморагічного шоку. Під час оперативного втручання діагностовано: розрив селезінки та до 1,5 л рідкої крові в черевній порожнині, пошкодження порожнистих органів не виявлено. Який вид трансфузії потрібно застосувати в цьому випадку? A 32-year-old patient was hospitalized in the surgical department 2 hours after a closed abdominal injury with clinical signs of hemorrhagic shock. During surgery, the following diagnosis was made: rupture of the spleen and up to 1, 5 liters of liquid blood in the abdominal cavity, no damage to the hollow organs was detected. What type of transfusion should be used in this case?

Пряму гемотрансфузію Direct blood transfusion

Непряму гемотрансфузію Indirect hemotransfusion

Замінну гемотрансфузію Replacement blood transfusion

Трансфузія не показана Transfusion not shown

Реінфузію Reinfusion

369 / 1500
У дівчинки віком 14 років виявлено низький зріст, широкі плечі, крилоподібні шкірні складки на шиї, відсутні ознаки статевого дозрівання. Інтелект у нормі. Під час ультразвукового дослідження органів малого тазу спостерігаються гіпоплазія матки, відсутність яєчників. Каріотип дитини: 45, Х0. Наявність якого патологічного синдрому можна припустити? A 14-year-old girl has short stature, broad shoulders, wing-like skin folds on the neck, no signs of puberty. Intelligence is normal. During an ultrasound examination of the small organs hypoplasia of the uterus, absence of ovaries. The child's karyotype: 45, X0. The presence of which pathological syndrome can be assumed?

Синдром Патау Patau syndrome

Синдром Шерешевського-Тернера Shereshevsky-Turner syndrome

Синдром Дауна Down Syndrome

Синдром Едвардса Edwards syndrome

Синдром Клайнфельтера Klinefelter syndrome

370 / 1500
Жінка 28 років поступила зі скаргами на різкі болі внизу живота, короткочасну втрату свідомості вдома. Остання менструація була 12 днів тому. Під час вагінального обстеження встановлено: матка звичайної форми, неболюча, додатки зліва дещо збільшені, болючі під час пальпації. Заднє склепіння нависає, напружене, різко болюче. Який попередній діагноз? A 28-year-old woman came in with complaints of sharp pains in the lower abdomen, short-term loss of consciousness at home. The last menstruation was 12 days ago. During a vaginal examination, it was found: a uterus of normal shape , painless, the appendages on the left are slightly enlarged, painful during palpation. The posterior arch is overhanging, tense, painful. What is the previous diagnosis?

Загострення хронічного аднекситу Exacerbation of chronic adnexitis

Перекрут ніжки кісти Cyst stem twist

Піосальпінкс праворуч Piosalpinx on the right

Апоплексія правого яєчника Apoplexy of the right ovary

Позаматкова вагітність Ectopic pregnancy

371 / 1500
Бригада ШМД прибула до хворого 45-ти років. Зі слів родичів, захворів раптово, після повернення з гірськолижного курорту, коли підвищилась температура тіла до 38,7^oC, з'явився головний біль, блювання. Об'єктивно: шкіра бліда з ціа-нотичним відтінком, рясна геморагічна висипка по всьому тілу, місцями з некрозом у центрі. АТ- 45/0 мм рт.ст., пульс - 126/хв., слабкого наповнення. Виражена ригідність м'язів потилиці, позитивний симптом Керніга. Поставте попередній діаг-ноз: SMD brigade arrived at a 45-year-old patient. According to relatives, he fell ill suddenly, after returning from a ski resort, when his body temperature rose to 38.7^oC , a headache, vomiting appeared. Objectively: the skin is pale with a cyanotic shade, a profuse hemorrhagic rash in places with necrosis in the center. Blood pressure - 45/0 mmHg, pulse - 126/min ., weak filling. Pronounced stiffness of the occipital muscles, positive Kernig's symptom. Make a preliminary diagnosis:

Менінгококова інфекція Meningococcal infection

Грип Flu

Поліомієліт Polio

Везикульозний рикетсіоз Vesicular rickettsiosis

Висипний тиф Typhoid

372 / 1500
Жінка віком 55 років звернулася до лікаря з приводу деформації суглобів пальців рук та болю в них під час рухів. Об'єктивно спостерігається: невеликі кісткові утворення на бічних поверхнях дистальних міжфалангових суглобів, трохи болючі під час пальпації. Як називаються ці утворення? A 55-year-old woman consulted a doctor about deformation of the joints of the fingers and pain in them during movements. Objectively observed: small bone formations on the lateral surfaces of the distal interphalangeal joints, slightly painful during palpation. What are these formations called?

Вузлики Гебердена Heberden's nodes

Вузлики Бушара Bouchard knots

Тофуси Tofus

Нодулярна еритема Erythema nodular

Ревматоїдні вузлики Rheumatoid nodules

373 / 1500
Пацієнтка віком 22 роки збуджена, поведінка неадекватна, свідомість сплутана. В анамнезі - цукровий діабет 1-го типу впродовж 4 років, перебіг лабільний. Отримує інсулінотерапію 54 ОД/добу. Об'єктивно спостерігається: шкіра волога, холодна на дотик, гіперрефлексія, зіниці розширені. Дихання везикулярне. АТ - 140/90 мм рт. ст., пульс - 88/хв. У аналізі крові виявлено: глікемія - 2,3 ммоль/л, аглюкозурія. Який найімовірніший діагноз? A 22-year-old female patient is excited, her behavior is inadequate, her consciousness is confused. She has type 1 diabetes in her history for 4 years, the course is labile. She receives insulin therapy 54 units/ Objectively, the skin is cold to the touch, the pupils are dilated, blood pressure - 88/min /l, aglucosuria. What is the most likely diagnosis?

Гіперосмолярна кома Hyperosmolar coma

Лактацидотична кома Lactacidotic coma

Гостре порушення мозкового кровообігу Acute cerebrovascular accident

Кетоацидотична кома Ketoacidotic coma

Гіпоглікемічна кома Hypoglycemic coma

374 / 1500
Хворий 19-ти років під час пірнання у воду вдарився головою у дно. Скаржиться на біль у шиї, обмеженість і болючість рухів головою. При огляді: голову, нахилену вперед і вправо, підтримує руками. При пальпації визначається напруження м'язів шиї, виступає остистий відросток ІV шийного хребця. При натискуванні на нього та голову (навантаження вздовж осі) - посилення болю. Який попередній діагноз? A 19-year-old patient hit his head on the bottom while diving into the water. He complains of neck pain, limited and painful head movements. On examination: head tilted forward and to the right, supports with the hands. During palpation, the spinous process of the 4th cervical vertebra protrudes. When pressing on it and the head (load along the axis), the pain increases. What is the preliminary diagnosis?

Забій шийного відділу хребта Contusion of the cervical spine

Неускладнений перелом шийного від-ділу хребта Uncomplicated cervical spine fracture

Ушкодження корінців спинного мозку Damage of the roots of the spinal cord

Ушкодження м'язів шиї Neck muscle damage

Ускладнений перелом шийного від-ділу хребта Complicated fracture of the cervical spine

375 / 1500
Хворий 20-ти років надійшов у лікарню на 9-й день хвороби. Захворювання пов'язує з вживанням недостатньо просмаженої свинини. Захворювання почалось з появи періорбітальних набряків, лихоманки. Об'єктивно: температура тіла - 38,5^оC) Обличчя одутле. Різко виражений набряк повік. Пальпація литкових м'язів різко болюча. В крові: гіпереозинофілія. Яка етіологія захворювання? A 20-year-old patient was admitted to the hospital on the 9th day of illness. The disease is associated with the consumption of insufficiently fried pork. The disease began with the appearance of periorbital edema, fever Objectively: the body temperature is 38.5°C. The face is swollen. Palpation of the calf muscles is painful. What is the etiology of the disease?

Ехінококи Echinococci

Лептоспіри Leptospira

Аскариди Ascarids

Трихінели Trichinella

Волосоголовці Hairheads

376 / 1500
До лікарні шпиталізовано пацієнта віком 51 рік зі скаргами на пекучі болі за грудиною тривалістю близько 2 годин. На ЕКГ спостерігається: елевація сегмента ST у II, III, aVF відведеннях. Артеріальний тиск - 150/90 мм рт. ст., пульс - 90/хв. Який із наведених нижче препаратів відноситься до патогенетичної терапії цього захворювання? A 51-year-old patient was admitted to the hospital with complaints of burning pains behind the sternum lasting about 2 hours. The ECG shows: ST segment elevation in leads II, III, aVF Arterial pressure - 150/90 mm Hg, pulse - 90/min.

Омепразол Omeprazole

Предуктал Preductal

Альтеплаза Alteplase

Мілдронат Mildronate

Метронідазол Metronidazole

377 / 1500
У хворої 31-го року після проведеної субтотальної резекції щитоподібної залози з приводу токсичного зоба виникли серцебиття, пітливість, психомоторне збудження. Пацієнтка знепритомніла. Об'єктивно: температура тіла - 39,7^oC, частота дихання - 38/хв.; ЧСС- 135/хв., миготлива аритмія. АТ- 190/80 мм рт.ст. Імовірне ускладнення: A 31-year-old patient developed palpitations, sweating, psychomotor agitation after subtotal resection of the thyroid gland for toxic goiter. The patient fainted. Objectively: body temperature - 39.7°C, heart rate - 135/min. Blood pressure - 190/80 mm Hg.

Рецидив дифузного токсичного зоба Recurrence of diffuse toxic goiter

Тиреотоксичний криз Thyrotoxic crisis

Гіпертонічний криз Hypertensive crisis

Бактеріальний шок Bacterial shock

Істеричний напад Hysterical attack

378 / 1500
В пологовий будинок доставлена вагітна 32-х років. Зі слів родичів дома мала судоми, втратила свідомість. Турбує головний біль, загальна слабкість, свідомість дещо затьмарена. Вагітність ІІ, 34 тижні. АТ- 170/120 мм рт.ст., пульс - 100/хв. Спостерігаються генералізовані набряки, що виникли 3 тижні тому. Серцебиття плода - ритмічне, приглушене, 124/хв. Яка тактика? A 32-year-old pregnant woman was taken to the maternity ward. According to relatives, she had convulsions at home and lost consciousness. She is troubled by a headache, general weakness, consciousness is somewhat clouded. Pregnancy II , 34 weeks. BP- 170/120 mm Hg. Generalized swellings are observed, which occurred 3 weeks ago. The heartbeat is muffled, 124/min.

Інтенсивна терапія, родорозрішення впродовж доби Intensive therapy, delivery during the day

Інтенсивна терапія, родорозрішення в терміні 37-40 тижнів Intensive therapy, delivery within 37-40 weeks

Інтенсивна терапія, родорозрішення впродовж тижня Intensive therapy, delivery within a week

Інтенсивна терапія, родорозрішення в терміні 36 тижнів Intensive therapy, delivery within 36 weeks

Інтенсивна терапія та негайне родорозрішення Intensive therapy and immediate delivery

379 / 1500
Стан доношеної новонародженої дитини погіршився у 1-шу добу життя. З анамнезу відомо, що дитина народилася від 3 вагітності на тлі гестозу другої половини, група крові матері 0(I) Rh(-). Під час огляду спостерігається: дитина млява, шкіра та слизові оболонки жовтушні, сеча та випорожнення звичайного кольору. Білірубін сироватки крові - 248 мкмоль/л за рахунок непрямого. Укажіть найімовірнішу причину патологічного стану: The condition of a full-term newborn child worsened on the 1st day of life. It is known from the anamnesis that the child was born from the 3rd pregnancy against the background of gestosis of the second half, the mother's blood group is 0( I) Rh(-). During the examination, the child is lethargic, the urine and stools are normal in color. The blood serum is 248 μmol/L. State the most likely cause of the pathological condition:

Rh-несумісніть Rh-incompatible

Фізіологічна жовтяниця Physiological jaundice

АВО-несумісність AVO-incompatibility

Атрезія жовчовивідних шляхів Biliary atresia

Фетальний гепатит Fetal hepatitis

380 / 1500
У пацієнта раптово виник гострий біль за грудиною з іррадіацією в ліву руку. Об'єктивно спостерігалося: пацієнт збуджений, шкірні покриви бліді, ЧД - 38/хв., АТ - 180/ 110 мм рт. ст. Потім він втратив свідомість, впав, пульс на магістральних судинах не визначався, зіниці були рівномірно розширені. Встановіть діагноз. The patient suddenly developed sharp pain behind the sternum with radiation to the left arm. Objectively it was observed: the patient is agitated, the skin is pale, BH - 38/min., Blood pressure - 180/110 mm Hg. Then he lost consciousness, the pulse was not determined, the pupils were evenly dilated.

Серцевий напад Heart attack

Агональний стан Agonal State

Клінічна смерть Clinical death

Кома Comma

Порушення мозкового кровообігу Disruption of cerebral circulation

381 / 1500
Хворий скаржиться на наявність запального інфільтрату в середній третині лівого передпліччя. Захворів вперше. У середній третині лівого передпліччя на зовнішній його поверхні є запальний інфі-льтрат до 3 см в діаметрі, який конусоподібно виступає над поверхнею шкіри. Шкіра над ним гіперемована, набрякла, пальпація різко болюча. На вершині інфільтрату невелике скупчення гною з чорною точкою в центрі. Температура тіла - 37,6^oC. Яке захворювання описано? The patient complains about the presence of an inflammatory infiltrate in the middle third of the left forearm. He got sick for the first time. In the middle third of the left forearm on its outer surface, there is an inflammatory infiltrate up to 3 cm in in diameter, which protrudes above the surface of the skin. The skin is hyperemic, palpation is sharp. A small accumulation of pus with a black dot in the center. What disease is described?

Флегмона передпліччя Forearm phlegmon

Запальний інфільтрат передпліччя Inflammatory infiltrate of forearm

Карбункул передпліччя Forearm carbuncle

Еризипелоїд передпліччя Erysipeloid of forearm

Фурункул передпліччя Forearm furuncle

382 / 1500
Чоловіка 56 років турбують постійні напади ядухи вдень та 3-4 рази за тиждень уночі. Напади приводять до порушення фізичної активності та сну. Пікова об'ємна швидкість видиху (ПОШ) - 65% від належних, добові коливання ПОШ (30%). Визначте ступінь важкості бронхіальної астми: A 56-year-old man is bothered by constant attacks of wheezing during the day and 3-4 times a week at night. The attacks lead to disruption of physical activity and sleep. Peak expiratory volume velocity ( POH) - 65% of the proper, daily fluctuations of POH (30%). Determine the severity of bronchial asthma:

Персистуюча середньої важкості перебігу Persistent average run difficulty

Астматичний статус Asthmatic status

Персистуюча легкого перебігу Persistent light run

Інтермітуюча Intermittent

Персистуюча важкого перебігу Persistent heavy run

383 / 1500
Хворий 21-го року госпіталізований на 2-й день хвороби. Загальний стан тяжкий, температура тіла - 39^oC. На шкірі рясні геморагічні елементи неправильної форми. Діагноз: менінгококцемія. Наступного дня температура раптово знизилась, АТ- 80/40 мм рт.ст., частота пульсу - 120/хв. Акроціаноз. Яке ускладнення розвинулося у хворого? A 21-year-old patient was hospitalized on the 2nd day of illness. The general condition is serious, the body temperature is 39^oC. There are abundant hemorrhagic elements of irregular shape on the skin. Diagnosis : meningococcemia. The next day the temperature dropped, blood pressure - 80/40 mmHg. Acrocyanosis developed in the patient?

Гостра серцева недостатність Acute heart failure

Церебральна кома Cerebral coma

Гостра печінкова недостатність Acute liver failure

Гостра надниркова недостатність Acute adrenal insufficiency

Гостра кровотеча Acute bleeding

384 / 1500
Чоловік 23-х років звернувся до лікаря зі скаргами на наявність набряків обличчя, головні болі, запаморочення, зменшення виділення сечі, бурий колір сечі. Наведені скарги з'явились після перенесеного фарингіту. При фізикальному обстеженні температура тіла - 37,4^oC, артеріальний тиск - 170/110 мм рт.ст., пульс - 86/хв., набряки на обличчі, шкірні покриви бліді. Аускультативно серцеві тони приглушені, акцент ІІ тону над аортою. Які зміни найбільш імовірно будуть спостерігатись у загальному аналізі сечі? A 23-year-old man went to the doctor with complaints of facial swelling, headaches, dizziness, decreased urine output, brown urine. These complaints appeared after pharyngitis. On physical examination, body temperature - 37.4°C, blood pressure - 170/110 mmHg, pulse - 86/min., edema on the face, pale skin tone above the aorta. What changes are most likely to be observed in the general analysis of urine?

Еритроцитурія, оксалатурія, еритроцитарні циліндри Erythrocyturia, oxalaturia, erythrocyte cylinders

Еритроцитурія, протеїнурія, гіалінові циліндри Erythrocyturia, proteinuria, hyaline cylinders

Лейкоцитурія, протеїнурія, епітеліальні циліндри Leukocyturia, proteinuria, epithelial cylinders

Гемоглобінурія, білірубінурія, зернисті циліндри Hemoglobinuria, bilirubinuria, granular cylinders

Еритроцитурія, лейкоцитурія, лейкоцитарні циліндри Erythrocyturia, leukocyturia, leukocyte cylinders

385 / 1500
Жінка віком 27 років, вагітність І, пологи І, була госпіталізована в пологове відділення. З анамнезу відомо: первинне безпліддя впродовж трьох років. Перейми відбуваються кожні 4-5 хвилин, по 20-25 секунд, тривалість - 9 годин. 2,5 години тому відійшли навколоплідні води. Серцебиття плода - 136/хв. Голівка малим сегментом розташована в площині входу в малий таз. Шийка матки згладжена, відкриття - 4 см. Плодовий міхур відсутній. Яке ускладнення виникло під час пологів? A 27-year-old woman, pregnancy I, childbirth I, was hospitalized in the maternity ward. It is known from the anamnesis: primary infertility for three years. Menstruation occurs every 4-5 minutes, duration - 9 hours. Fetal heartbeat - 136/min. Cervix is ​​smoothed, opening - 4 cm the bladder is absent. What complication occurred during childbirth?

Первинна слабкість пологової діяльності Primary weakness of labor activity

Дискоординована пологова діяльність Discoordinated birth activity

Нормальна пологова діяльність Normal labor activity

Патологічний прелімінарний період Pathological preliminary period

Вторинна слабкість пологової діяльності Secondary weakness of labor activity

386 / 1500
У малюка 9 місяців спостерігається затримання розвитку зубів та подовжується строк зарощення тім'ячка, слабкість та пітнівість. Який вид гіповітамінозу може бути у малюка? A 9-month-old baby has a delay in the development of teeth and a prolonged period of hair growth, weakness and sweating. What kind of hypovitaminosis can a baby have?

Гіповітаміноз А Hypovitaminosis A

Гіповітаміноз D Hypovitaminosis D

Гіповітаміноз В_6 Hypovitaminosis B_6

Гіповітаміноз С Hypovitaminosis C

Гіповітаміноз В_1 Hypovitaminosis B_1

387 / 1500
Вагітна в термiнi 35 тижнiв, стан тяжкий. Скаржиться на головний бiль, порушення зору, миготiння <<мушок>> перед очима. Під час огляду виявлено: загальний набряк, АТ - 180/120 мм рт. ст. Раптово з'явилися фібрилярнi посмикування м'язiв обличчя, тонiчнi судоми. Дихання припинилося. За 1 хвилину дихання відновилося. З рота виділилася значна кiлькiсть пiни. Амнезiя. Протеінурія - 7 г/л. Який імовірний діагноз? She is 35 weeks pregnant, her condition is serious. She complains of a headache, impaired vision, flickering of <> in front of her eyes. During the examination, it was found: general swelling , BP - 180/120 mm Hg. Sudden twitching of the facial muscles. Breathing stopped. A large amount of foam was released from the mouth - 7 g/l. What is the probable diagnosis?

Епiлепсiя Epilepsy

Передеклампсiя тяжкого ступеню Severe preeclampsia

Гiпертонiчний криз Hypertensive crisis

Еклампсiя Eclampsia

Черепно-мозкова травма Traumatic brain injury

388 / 1500
Роділлю віком 20 років шпиталізовано до пологового будинку з регулярною пологовою діяльністю. Під час вагінального обстеження відійшли навколоплідні води густо забарвлені меконієм. Шийка матки згладжена, розкрита до 6 см, голівка прижата до входу в малий таз, серцебиття плода 190/хв. Який діагноз найімовірніший? A 20-year-old woman in labor was hospitalized in a maternity hospital with regular labor. During a vaginal examination, amniotic fluid thickly stained with meconium was removed. The cervix was smoothed, opened up to 6 cm, the head is pressed against the entrance to the small pelvis, the heart rate of the fetus is 190/min. What is the most likely diagnosis?

Відшарування нормально розташованої плаценти Detachment of a normally located placenta

Слабкість пологової діяльності Weak labor activity

Надмірна пологова діяльність Excessive reproductive activity

Загроза розриву матки Threat of uterine rupture

Дистрес плода Fetal distress

389 / 1500
Чоловік 68 років скаржиться на тупий біль у череві, схуднення, слабкість, закрепи межують із рідким стільцем, багато темної крові у калі. Об'єктивно встановлено: шкіра землиста, суха. Під час пальпації живота в правій здухвинній ділянці - інфільтрат 6х9 см, який майже не зміщується. Hb крові - 68 г/л. Яка найімовірніша патологія може зумовлювати таку картину? A 68-year-old man complains of dull pain in the abdomen, weight loss, weakness, constipation bordering on liquid stool, a lot of dark blood in the stool. Objectively established: the skin is earthy , dry. During palpation of the abdomen, there is an infiltrate of 6x9 cm. Blood Hb is 68 g/l. What is the most likely cause of such a picture?

Поліпоз ободової кишки, ускладнений кровотечею Polyposis of the colon complicated by bleeding

Пухлина сліпої кишки, кишкова кровотеча Cecal tumor, intestinal bleeding

Неспецифічний виразковий коліт, ускладнений кровотечею Nonspecific ulcerative colitis complicated by bleeding

Хвороба Крона, ускладнена кровотечею Crohn's disease complicated by bleeding

Дивертикулярна хвороба, ускладнена кровотечею Diverticular disease complicated by bleeding

390 / 1500
Жінка 58-ми років перебуває на стаціонарному лікуванні з приводу гіпертонічної хвороби. Вранці, після прийому каптоприлу та празозину різко піднялася з ліжка та короткочасно знепритомніла. Об'єктивно: шкіра бліда, волога. Пульс - 100/хв., слабкий, артеріальний тиск - 70/40 мм рт.ст. На ЕКГ: ритм синусовий, правильний, ознаки перенавантаження лівого шлуночка. Змін з боку центральної нервової системи не виявлено. Яка причина стану хворої є найбільш імовірною? A 58-year-old woman is undergoing inpatient treatment for hypertension. In the morning, after taking captopril and prazosin, she suddenly got out of bed and briefly fainted. Objectively: pale skin, pulse - 100/min, blood pressure - 70/40 mm Hg: sinus rhythm, signs of left ventricular overload. What is the cause of the condition patient is the most likely?

Анафілактичний шок Anaphylactic shock

Напад Морганьї-Адамса-Стокса Morganhi-Adams-Stokes Attack

- -

Кардіогенний шок Cardiogenic shock

Ортостатична гіпотензія Orthostatic hypotension

391 / 1500
У жінки 65 років, яка вживала воду з криниці, поступово підвищилась температура тіла до 39,9^oС. Об'єктивно встановлено: загальмована, язик вкритий сіро-бурим нальотом із відбитками зубів по краях, на шкірі живота поодинокі розеоли, живіт здутий. Реакція Відаля - 1:400. Який із препаратів етіотропної хіміотерапії необхідно призначити? A 65-year-old woman, who drank water from a well, gradually increased her body temperature to 39.9^oC. It was objectively established: she was inhibited, her tongue was covered with gray a brown coating with teeth marks on the edges, single roseolae on the abdomen, swollen abdomen. Which of the etiotropic chemotherapy drugs should be prescribed?

Пеніцилін Penicillin

Еритроміцин Erythromycin

Цефазолін Cefazolin

Ніфуроксазид Nifuroxazide

Ципрофлоксацин Ciprofloxacin

392 / 1500
У породіллі на 12-ту добу післяпологового періоду раптово підвищилася температура тіла до 38,2^oС. Спостерігає-ться загальна слабкість, біль в ділянці правої молочної залози протягом 1 доби. Під час огляду встановлено: молочна залоза напружена, гаряча, у правому верхньому квадранті пальпується інфі-льтрат, болючий, щільної консистенції. Який діаг-ноз найбільш імовірний? On the 12th day of the postpartum period, the mother's body temperature suddenly rose to 38.2°C. There is general weakness, pain in the area of ​​the right mammary gland during 1 day. During the examination, the mammary gland is tense, hot, and the infiltrate is palpable in the right upper quadrant. What is the most likely diagnosis?

Мастопатія Mastopathy

Лактаційний мастит Lactation mastitis

Лактостаз Lactostasis

Пухлина молочної залози Breast tumor

Аномалії розвитку молочних залоз Mammary gland development anomalies

393 / 1500
Жінка 55-ти років скаржиться на пронос, лущення і пігментацію відкритих ділянок тіла (шиї, рук та стоп), дратівливість і занепокоєння. З анамнезу відомо, що основним продуктом харчування для неї є кукурудза. Салатні овочі і бобові жінка вживає рідко, м'ясо і рибу в їжу не вживає зовсім. Про яке захворювання йдеться? A 55-year-old woman complains of diarrhea, peeling and pigmentation of open areas of the body (neck, hands and feet), irritability and anxiety. From the anamnesis, it is known that the main the food for her is corn. She rarely eats salad vegetables and legumes, she does not eat meat and fish at all. What disease is it about?

Пелагра Pellagra

Псоріаз Psoriasis

Дерматит Свіфта Swift's dermatitis

Бері-бері Beri-beri

Цинга Scurvy

394 / 1500
Під час обстеження у юнака віком 14 років високого зросту виявлено: арахнодактилія, килеподібна деформація грудної клітки, кіфосколіоз, плоскостопість, вальгусна деформація стопи, міопічний астигматизм, енофтальм, пролапс мітрального клапана, розширення кореня аорти, позитивний тест великого пальця та зап'ястя. Який найімовірніший діагноз? During the examination, a 14-year-old tall boy was found to have: arachnodactyly, bull-like deformity of the chest, kyphoscoliosis, flat feet, valgus foot deformity, myopic astigmatism, enophthalmos, prolapse mitral valve, aortic root dilatation, positive thumb and wrist test. What is the most likely diagnosis?

Гомоцистинурія Homocystinuria

Синдром Марфана Marfan syndrome

Синдром Елерса-Данлоса Ehlers-Danlos syndrome

Синдром Білса Beals syndrome

Трисомія 8 Trisomy 8

395 / 1500
Пацієнтка віком 23 роки скаржиться на тупий біль у правій та лівій поперекових ділянках. Під час об’єктивного обстеження в лівому підребер'ї та мезогастрії пальпується безболісна, горбиста нирка еластичної консистенції, права нирка не пальпується. Під час УЗД виявлено множинні чіткі порожнисті утвори з ехонегативним умістом у паренхімі правої нирки, розмірами від 1,2х1,7 см до 2х2,5 см, та аналогічні утвори у паренхімі лівої нирки розмірами 2,5х3,7 см, одна з яких локалізується у ділянці нижнього полюса і має розмір 8,3х11,7 см. Встановіть імовірний діагноз. A 23-year-old female patient complains of dull pain in the right and left lumbar regions. During the objective examination, a painless, lumpy kidney is palpated in the left hypochondrium and mesogastric area elastic consistency, the right kidney is not palpable. During ultrasound, multiple clear hollow formations with echonegative contents in the parenchyma of the right kidney, measuring from 1.2x1.7 cm to 2x2.5 cm, and similar formations in the parenchyma of the left kidney, measuring 2.5x3, were detected. 7 cm, one of which is localized in the area of ​​the lower pole and has a size of 8.3x11.7 cm. Establish a probable diagnosis.

Солітарні кісти нирок Solitary kidney cysts

Полікістоз нирок Polycystic kidney disease

Двобічний гідронефроз Bilateral hydronephrosis

Мегакалікоз Megacalicosis

Мультикістоз Multicystosis

396 / 1500
У дитини 10 мiсяцiв на тлi легкого перебiгу ГРВІ з'явилися повторнi клонiчнi судоми. Під час огляду констатовано чiткi прояви рахiту середньої важкостi. Рiвень кальцiю кровi - 1,6 ммоль/л, iнтервал Q-T на ЕКГ подовжений. Даних про перинатальне ушкодження ЦНС немає. Спинно-мозкова рiдина iнтактна, витiкала пiд тиском. Вигодовування штучне, без овочевих страв. Яке захворювання проявилося на тлi ГРВI? A 10-month-old child had repeated clonic convulsions against the background of a mild acute respiratory viral infection. During the examination, clear manifestations of moderate rickets were noted. Blood calcium level - 1.6 mmol/l, the QT interval on the ECG is prolonged. There is no evidence of CNS damage. The cerebrospinal fluid was leaking under pressure. What disease was manifested against the background of GERD?

Енцефалiтична реакцiя Encephalitic reaction

Менiнгiт Meningitis

Нейротоксикоз Neurotoxicosis

Енцефалiт Encephalitis

Спазмофiлiя Spasmophilia

397 / 1500
Хлопець віком 15 років хворіє протягом тижня. Скаржиться на лихоманку, яка з субфебрильної у перші дні підвищилася до 39-40^oС у наступні 2 дні, загальну слабкість, зниження апетиту, періодичну нудоту, біль у животі, м'язах, суглобах. Спостерігаються прояви фарингіту, язик яскравий із набряклими сосочками, ін'єкція склер, на шкірі плямисто-папульозний висип зі згущенням навколо суглобів, гіперемія і набряклість кистей і стоп. Пульс - 110/хв. Під час УЗД виявлено гепатоспленомегалію, мезаденіт. Фізіологічні випорожнення в нормі. У загальному аналізі крові - нейтрофільний лейкоцитоз. Встановіть попередній діагноз. A 15-year-old boy has been ill for a week. He complains of fever, which rose from subfebrile in the first days to 39-40°C in the next 2 days, general weakness, loss of appetite, periodic nausea, pain in the stomach, muscles, joints. There are manifestations of pharyngitis, a bright tongue with swollen papillae, a spotted-papular rash on the skin around the joints, hyperemia and swelling of the hands and feet - 110/min. Hepatosplenomegaly, mesadenitis was detected. In the general blood analysis, neutrophilic leukocytosis was established.

Псевдотуберкульоз, тяжка форма Pseudotuberculosis, severe form

Скарлатина, тяжка форма Scarlet fever, severe form

Лептоспіроз, тяжка форма Leptospirosis, severe form

Хвороба Кавасакі Kawasaki disease

Інфекційний мононуклеоз Infectious mononucleosis

398 / 1500
Для населення, яке проживає на радіаційно-забрудненій території, для виведення з організму радіонуклідів рекомендується включити до раціону харчування пектини. Які з наведених продуктів є основним джерелом пектинів? For the population living in a radiation-contaminated territory, to remove radionuclides from the body, it is recommended to include pectins in the diet. Which of the following products are the main source of pectins?

М'ясо Meat

Хліб Bread

Молоко Milk

Макарони Pasta

Овочі та фрукти Vegetables and fruits

399 / 1500
У пацієнтки 23-х років, хворої на цукровий діабет І типу, на другому тижні позалікарняної пневмонії виникли нудота, блювання. Ввечері втратила свідомість. Госпіталізована. Об'єктивно: шкіра суха та бліда. Дихання шумне, язик сухий, з нашаруванням коричневого кольору. ЧСС - 129/хв., АТ - 85/50 мм рт.ст. На пальпацію живота не реагує. Печінка +3 см. Реакція на ацетон різко позитивна, глюкоза крові - 26 ммоль/л. Попередній діагноз: A 23-year-old female patient with type 1 diabetes developed nausea and vomiting in the second week of community-acquired pneumonia. She lost consciousness in the evening. She was hospitalized. Objectively : the skin is dry and pale. The breathing is dry, with a brown coating. Heart rate - 85/50 mm Hg. The liver does not react to the palpation of +3 cm , blood glucose - 26 mmol/l. Preliminary diagnosis:

Печінкова кома Hepatic coma

Гіперосмолярна кома Hyperosmolar coma

Лактацидемічна кома Lactacidemic coma

Інфекційно-токсичний шок Infectious-toxic shock

Кетоацидотична кома Ketoacidotic coma

400 / 1500
Пологи II, термінові, у вагітної 23 років. Три години тому відійшли світлі навколоплідні води. Пологова діяльність регулярна. Перейми з інтервалом 4-5 хвилин, тривають 25 - 30 секунд, положення плоду поздовжнє, передлежить голівка плода, притиснута до входу у малий таз. Серцебиття плода - 136/хв. Внутрішнє дослідження показало: шийка матки згладжена, відкриття маткового зіва - 3 см, плодовий міхур відсутній, нижній полюс голівки на рівні l. terminalis. Який це період пологів? Labor II, urgent, in a 23-year-old pregnant woman. Light amniotic fluid passed three hours ago. Labor activity is regular. Contractions with an interval of 4-5 minutes, lasting 25 - 30 seconds, the position of the fetus is longitudinal, the fetal head is pressed against the entrance to the pelvis. The fetal heart rate is 136/min. The internal examination showed that the cervix is ​​smooth, the uterine opening is 3 cm, the fetal bladder is absent, the lower pole of the head is at the level of l . terminalis. What is the period of childbirth?

I період пологів I period of childbirth

Передвісники пологів Forerunners of childbirth

III період пологів III birth period

II період пологів II period of childbirth

Прелімінарний період Preliminary period

401 / 1500
У хлопчика 2 років на тлі лакунарної ангіни погіршилось дихання носом і ротом. Під час мезофарингоскопії спостерігаються гіперемія піднебінних мигдаликів і задньої стінки глотки, жовтуваті плівки в устях лакун мигдаликів, а також випинання задньої стінки ротоглотки, що флюктуює під час пальпації. Якого ускладнення слід запобігати під час хірургічного втручання? A 2-year-old boy with lacunar angina has worsened breathing through the nose and mouth. During mesopharyngoscopy, hyperemia of the palatine tonsils and the back wall of the pharynx, yellowish films in the mouths of the tonsil lacunae, as well as protrusion of the posterior wall of the oropharynx, which fluctuates during palpation. What complication should be prevented during surgery?

Асфіксія гноєм Asphyxia with pus

Порушення лімфовідтоку Disruption of lymphatic drainage

Медіастиніт Mediastinitis

Ретроезофагіт Retroesophagitis

Травма магістральних судин Injury of the main vessels

402 / 1500
Чоловік 30 років скаржиться на біль у гомілках обох ніг, який посилюється під час ходьби. Хворий змушений зупинятися через 150-200 м. Переміжна кульгавість з'явилась рік тому, відстань між зупинками скоротилася. Обидві ступні бліді, прохолодні. Пульс на тильних артеріях ступнів відсутній. Шкіра суха, на підошвах гіперкератоз, нігті потовщені, легко ламаються, матового кольору. На гомілках - ділянки облисіння. Яка стадія облітеруючого ендартеріїту у цього пацієнта? A 30-year-old man complains of pain in the lower legs of both legs, which worsens when walking. The patient is forced to stop after 150-200 m. Intermittent lameness appeared a year ago , the distance between the stops has decreased. The pulse on the back of the feet is absent. The skin is thickened, the nails break easily. What is the stage of alopecia in this patient?

І стадія - стадія функціональної компенсації I stage - stage of functional compensation

ІV стадія - стадія деструктивних змін IV stage - the stage of destructive changes

ІІ стадія - стадія субкомпенсації II stage - subcompensation stage

- -

ІІІ стадія - стадія декомпенсації III stage - decompensation stage

403 / 1500
У жінки 36 років після фізичного навантаження з'явилася ядуха, кашель із виділенням розового харкотіння. Страждає на ревматизм та мітральну ваду серця. Аускультативно виявлено над серцем та нижніми відділами легенів дрібно- та середньопухирчаті вологі хрипи. Який механізм погіршення стану є провідним? A 36-year-old woman developed dyspnea after physical exertion, a cough with pink sputum. She suffers from rheumatism and mitral valve disease. Auscultation revealed over the heart and lower parts small- and medium-vesicular wet rales in the lungs. What is the leading mechanism of deterioration?

Зниження лімфотичного відтоку Decreased lymphatic outflow

Підвищення онкотичного тиску крові Increased blood oncotic pressure

Підвищення гідростатичного тиску крові Increased hydrostatic blood pressure

Зниження онкотичного тиску крові Reduction of oncotic blood pressure

Підвищення агрегації еритроцитів та тромбоцитів Increased aggregation of erythrocytes and platelets

404 / 1500
Чоловік 52 років захворів гостро після переохолодження 3 тижні тому. Скаржиться на кашель, підвищення температури тіла до 39,5^oС, біль у правій половині грудної клітки, різку задишку. Пульс - 120/хв., АТ - 90/60 мм рт. ст. Акроціаноз. ЧД - 48/хв. Об'єктивно спостерігається відставання правої половини грудної клітки в акті дихання. Перкуторно праворуч визначається притуплення легеневого звуку. Аускультативно дихання над правою легенею не вислуховується. Рентгенологічно праворуч спостерігається затемнення з нечіт-ким верхнім контуром до рівня II ребра. У ексудаті до 90% нейтрофілів. Який діагноз найімовірніший? A 52-year-old man became acutely ill after hypothermia 3 weeks ago. He complains of a cough, an increase in body temperature to 39.5°C, pain in the right half of the chest, sharp Shortness of breath. Pulse - 90/60 mm Hg. Objectively, the right half of the chest is dull the right lung is not heard. Radiologically, there is an obscuration on the right side up to the level of the II rib. What is the most likely diagnosis?

Крупозна пневмонія Croup pneumonia

Емпієма плеври Empyema of the pleura

Рак легені Lung cancer

Ексудативний туберкульозний плеврит Exudative tuberculous pleurisy

Інфільтративний туберкульоз правої легені Infiltrative tuberculosis of the right lung

405 / 1500
У пацієнта віком 29 років 3 дні тому з'явився біль пульсуючого характеру в ділянці прямої кишки, підвищення температури тіла, загальна слабість. Пальпаторно виявлено: локальна болючість в анальній ділянці на 6 годині. Під час пальцевого дослідження прямої кишки виявлено: болючий інфільтрат, що досягає не вище гребінцевої лінії. Який найімовірніший діагноз? 3 days ago, a 29-year-old patient developed pain of a pulsating nature in the region of the rectum, increased body temperature, general weakness. Palpation revealed: local soreness in the anus area at 6 o'clock. During a digital examination of the rectum, a painful infiltrate was found, reaching no higher than the comb line. What is the most likely diagnosis?

Пухлина прямої кишки Rectal tumor

Гостра анальна тріщина Acute anal fissure

Гострий геморой Acute hemorrhoids

Гострий простатит Acute prostatitis

Гострий парапроктит Acute paraproctitis

406 / 1500
Хворий 58-ми років скаржиться на слабкість, набряки на обличчі, попереку та ногах, задишку, вологий кашель. Протягом багатьох років хворіє на ХОЗЛ. Останні 5 років відзначає посилення виділення харкотиння, часто гнійного характеру. Об'єктивно: ЧСС- 80/хв., АТ- 120/80 мм рт.ст. Розповсюджені набряки, шкіра бліда, суха, тургор знижений. Добова протеїнурія - 6,6 г. У крові: гіпоальбумінемія, підвищення альфа-2 і гаммаглобулінів, ШОЕ- 50 мм/год. Креатинін крові - 88 мкмоль/л. Який діагноз правильний? A 58-year-old patient complains of weakness, swelling of the face, lower back and legs, shortness of breath, wet cough. He has been suffering from COPD for many years. For the last 5 years, he has Increased sputum, often purulent : hypoalbuminemia, increased alpha-2 and gammaglobulins, ESR - 50 mm/h. Blood creatinine - 88 μmol/l. What is the correct diagnosis?

Вторинний амілоїдоз нирок, азотемічна стадія Secondary renal amyloidosis, azotemic stage

Первинний амілоїдоз Primary amyloidosis

Вторинний амілоїдоз нирок, нефротичний синдром Secondary kidney amyloidosis, nephrotic syndrome

- -

Вторинний амілоїдоз нирок, протеїнурична стадія Secondary renal amyloidosis, proteinuric stage

407 / 1500
Мати привела до лікаря хлопчика 7-ми років на плановий медичний огляд. Вона скаржиться, що дитина дуже багато часу проводить за ґаджетами та дуже мало грає з однолітками на свіжому повітрі. Фізичний розвиток відповідає віку дитини. Згідно з карткою профілактичних щеплень до 6-ти років включно отримав усі щеплення, передбачені Національним календарем. При фізикальному обстеженні температура тіла - 37,1^oC, пульс - 88/хв., артеріальний тиск - 110/80 мм рт.ст., частота дихання - 16/хв. Які подальші дії по веденню хлопчика будуть найбільш доречними? A mother brought a 7-year-old boy to the doctor for a routine medical examination. She complains that the child spends a lot of time on gadgets and plays very little with his peers outdoors Physical development corresponds to the child's age. According to the preventive vaccination card, he received all the vaccinations prescribed by the National Calendar. During the physical examination, the body temperature is 88/min., blood pressure is 110 /80 mm Hg, respiratory rate - 16/min. What further actions for the boy's management would be most appropriate?

Провести бесіду щодо важливості активного способу життя Have a conversation about the importance of an active lifestyle

Призначити вакцину БЦЖ Prescribe BCG vaccine

Призначити вакцину КПК Prescribe PDA vaccine

Провести пробу Манту Perform Mantoux test

Призначити оральну поліомієлітну вакцину (ОПВ) Prescribe oral polio vaccine (OPV)

408 / 1500
Пацієнт скаржиться на біль, світлобоязнь, сльозотечу, зниження зору в правому оці. Гострота зору правого ока 0,5 не корегується, гострота зору лівого ока 1,0. Об'єктивно спостерігається: в правому оці очна щілина звужена, перикорнеальна ін'єкція очного яблука. Преципітати на задній поверхні рогівки. Зіниця звужена, реакція на світло уповільнена. В передньому відділі склистого тіла плаваючі помутніння. Очне дно в нормі. Який найімовірніший діагноз? The patient complains of pain, photophobia, lacrimation, reduced vision in the right eye. The visual acuity of the right eye is 0.5 uncorrected, the visual acuity of the left eye is 1.0. Objectively, the eye slit is narrowed, the eyeball is injected, the pupil is narrowed, the reaction to light is slowed down. What is the most likely diagnosis?

Бактеріальний кон'юнктивіт Bacterial conjunctivitis

Гострий кератит Acute keratitis

Задній увеїт Posterior uveitis

Гострий іридоцикліт Acute iridocyclitis

Гострий напад глаукоми Acute attack of glaucoma

409 / 1500
Жінка віком 23 роки скаржиться на підвищення температури тіла до 37,4^oС, появу геморагічного висипу на нижніх кінцівках, біль у попереку, появу червоної сечі. Захворіла 3 дні тому після переохолодження. Об'єктивно спостерігається: шкіра бліда, на поверхні гомілок і стегон - дрібний геморагічний симетричний висип. ЧСС - 90/хв, АТ-115/90 мм рт. ст. Симптом Пастернацького - слабопозитивний з обох боків. В аналізі крові визначається: лейкоцити - 9,6·10^9/л, тромбоцити - 180·10^9/л, ШОЕ - 31 мм/год. У сечі спостерігається: білок - 0,33 г/л, еритроцити змінені - 3-40 в п/з, лейкоцити - 5-8 в п/з. Який найімовірніший діагноз? A 23-year-old woman complains of an increase in body temperature to 37.4^oС, the appearance of a hemorrhagic rash on the lower extremities, pain in the lower back, the appearance of red urine. She became ill 3 days after hypothermia: pale skin, symmetric hemorrhagic rash on both sides, BP-115/min blood is determined: leukocytes - 9.6·10^9/l, platelets - 180·10^9/l, ESR - 31 mm/h. In urine: protein - 0.33 g/l, erythrocytes changed - 3- 40 in p/z, leukocytes - 5-8 in p/z. What is the most likely diagnosis?

Системний червоний вовчак Systemic lupus erythematosus

Тромбоцитопенічна пурпура Thrombocytopenic purpura

Геморагічний васкуліт Hemorrhagic vasculitis

Гострий інтерстиціальний нефрит Acute interstitial nephritis

Вузликовий періартеріїт Nodular periarteritis

410 / 1500
Повторнороділля 34-х років надійшла до пологового відділення з регулярною пологовою діяльністю. Розміри тазу: 26-29-32-22 см. При піхвовому дослідженні: відкриття шийки матки 6 см, плодовий міхур цілий. Передлежать сідниці плода, притиснуті до входу у малий таз. Мис недосяжний, екзостозів немає. ЧСС плода - 140/хв., передбачувана маса - 2800 г. Якою має бути тактика пологів? A 34-year-old woman who gave birth again came to the maternity ward with regular labor. Pelvis dimensions: 26-29-32-22 cm. During vaginal examination: opening of the cervix 6 cm, the fetal bladder is present, pressed against the entrance to the pelvis. The fetal heart rate is 140/min. What should be the delivery tactics?

Розродження через природні пологові шляхи Birth through natural birth canal

Екстракція плоду за тазову частину Fetal extraction for the pelvic part

Класичний комбінований зовнішньо-внутрішній поворот плода Classic combined external-internal rotation of the fetus

Зовнішній акушерський поворот плода External obstetric rotation of the fetus

Кесарів розтин в ургентному порядку Caesarean section in urgent order

411 / 1500
Жінка 43-х років скаржиться на біль в животі, що посилюється нападами, нудоту, багаторазове блювання застійним кишковим вмістом, здуття живота, затримку газів. Хворіє 7 годин. Пульс - 116/хв. Язик сухий, коричневий. Живіт симетрично здутий, м'який, болючий. Перкуторно: тимпаніт. Аускультативно: кишкові шуми з металевим відтінком, шум сплеску, шум падаючої краплі. Який найбільш імовірний діагноз? A 43-year-old woman complains of abdominal pain that worsens with attacks, nausea, repeated vomiting of stagnant intestinal contents, abdominal distension, gas retention. She has been ill for 7 hours. Pulse - 116/min. Tongue is symmetric, soft, painful. Auscultation: guttural sounds, noise of a falling drop?

Гострий неспецифічний коліт Acute nonspecific colitis

Гострий некротичний панкреатит Acute necrotizing pancreatitis

Гострий ерозивний гастрит Acute erosive gastritis

Гостра кишкова непрохідність Acute intestinal obstruction

Гострий деструктивний холецистит Acute destructive cholecystitis

412 / 1500
35-річного чоловіка доставлено до приймального відділення у непритомному стані. Його дружина повідомила, що він втратив свідомість після відкриття крану з водою. За день до цього протягом тижня чоловік скаржився на запаморочення, слабкість та порушення координації. Працює спелеологом, проводить екскурсії печерами. Під час лікування у стаціонарі, спостерігалися постійні парестезії, дисфагія, дезорієнтація та атаксія. Стан швидко прогресував, пацієнт почав нерозбірливо говорити, приєдналися галюцинації, ажитація, що потребували седації та інтубації пацієнта. На 14-й день госпіталізації пацієнт помер. Вакцинація від якого збудника найбільш імовірно попередила б смерть цього пацієнта? A 35-year-old man was brought to the reception department in an unconscious state. His wife reported that he lost consciousness after opening the water tap. A day before during the week, the man complained of dizziness, weakness and incoordination.Works as a speleologist, conducts tours of the caves.Persistent paresthesias, disorientation and ataxia were observed.The patient began to speak unintelligibly, accompanied by hallucinations, which required sedation and intubation of the patient. On the 14th day of hospitalization, the patient died. Vaccination from which pathogen would most likely prevent the death of this patient?

Вірусу поліомієліту Polio virus

Вірусу японського енцефаліту Japanese encephalitis virus

Вакцина від цього збудника не розроблена A vaccine against this pathogen has not been developed

Вірусу кліщового енцефаліту Tick-borne encephalitis virus

Вірусу сказу Rabies virus

413 / 1500
Пацієнт віком 29 років через 2 тижні після перенесеної ангіни помітив набряки обличчя, слабкість, зниження працездатності, поступово з'явилися задишка, набряки нижніх кінцівок та поперекового відділу хребта. Об'єктивно спостерігається: блідість шкірних покривів, серцеві тони ослаблені, гідроторакс, анасарка, АТ - 150/100 мм рт. ст. В клінічному аналізі сечі виявлено: питома вага сечі - 1021, білок - 9 г/л, еритроцити 40-50 в п/з, гіалінові циліндри - 4-6 в п/з. Який попередній діагноз? A 29-year-old patient noticed swelling of the face, weakness, reduced work capacity, shortness of breath, swelling of the lower extremities and the lumbar spine 2 weeks after having angina. Objectively observed: pallor of the skin, weakened heart tones, hydrothorax, blood pressure - 150/100 mm Hg. In the clinical analysis of urine: specific gravity - 9 g/l, erythrocytes 40-50 in p/z, hyaline cylinders - 4-6 in p/z. What is the previous diagnosis?

Мікседема Myxedema

Гострий гломерулонефрит Acute glomerulonephritis

Гострий пієлонефрит Acute pyelonephritis

Серцева недостатність Heart failure

Загострення хронічного гломерулонефриту Exacerbation of chronic glomerulonephritis

414 / 1500
Пацієнт віком 42 роки скаржиться на періодичний біль в епігастрії, частіше вночі і зранку, натще печію, відрижку кислим. З'явилася загальна слабкість, серцебиття, головокружіння, потім - блювання <<кавовою гущею>>. У загальному аналізі крові визначається: гемоглобін - 92 г/л, лейкоцити - 7,5·10^9/л, ШОЕ - 22 мм/год. Яке ускладнення виникло у пацієнта? A 42-year-old patient complains of periodic pain in the epigastrium, more often at night and in the morning, on an empty stomach, heartburn, sour belching. General weakness, palpitations, dizziness appeared, then - Vomiting with <>. In the general blood analysis, it is determined: hemoglobin - 92 g/l, leukocytes - 7.5·10^9/l, ESR - 22 mm/h. What complication did the patient have?

Шлунково-кишкова кровотеча Gastrointestinal bleeding

Стеноз пілоруса Stenosis of the pylorus

Перфорація виразки дванадцятипалої кишки Perforation of duodenal ulcer

Пенетрація виразки Ulcer penetration

Малігнізація виразки Ulcer malignancy

415 / 1500
У жінки цукровий діабет 1-го типу середнього ступеня тяжкості. Захворювання ускладнилося ретинопатією та полінейропатією. Крім того, під час неодноразового дослідження добової сечі на екскрецію альбуміна виявлена мікроальбумінурія (200-300 мг/добу). Швидкість клубочкової фільтрації - 105 мл/хв. АТ в межах норми. Для вторинної профілактики діабетичної нефропатії потрібно насамперед прагнути до нормалізації такого показника: The woman has type 1 diabetes mellitus of moderate severity. The disease was complicated by retinopathy and polyneuropathy. In addition, microalbuminuria was detected during repeated examination of daily urine for albumin excretion ( 200-300 mg/day. The glomerular filtration rate is within the normal range. For the secondary prevention of diabetic nephropathy, it is necessary to strive for the following indicator:

Інсуліну крові Blood insulin

Глікемії натщесерце Fasting blood glucose

Глікемії за 2 години після їжі Glycemia 2 hours after eating

С-пептида C-peptide

Глікозильованного гемоглобіну Glycosylated hemoglobin

416 / 1500
Сімейний лікар визначає у пацієнта наявність шкідливих факторів та проводить профілактичну роботу щодо них. Який це вид медичної профілактики? The family doctor determines the presence of harmful factors in the patient and carries out preventive work on them. What kind of medical prevention is this?

Третинна Tertiary

Соціальна Social

Первинна Primary

Вторинна Secondary

Долікарська Dolikarska

417 / 1500
У новонародженого на 3 добу виявлено деформацію, набряк та гематому м'яких тканин у надключичній ділянці ліворуч. Верхня кінцівка приведена до тулуба, пасивні рухи супроводжуються неспокоєм дитини. Який імовірний діагноз? On the 3rd day, the newborn was found to have deformation, edema and hematoma of soft tissues in the supraclavicular area on the left. The upper limb is brought to the trunk, passive movements are accompanied by restlessness of the child. What probable diagnosis?

Підокістний перелом лівої ключиці без зсуву відламків Subosseous fracture of the left clavicle without displacement of fragments

Акушерський параліч Ерба Erb's Obstetric Palsy

Флегмона новонародженого Phlegmon of a newborn

Остеомієліт лівої ключиці Osteomyelitis of the left clavicle

Перелом лівої ключиці зі зсувом відламків Fracture of the left clavicle with displacement of fragments

418 / 1500
Дівчинка 4-х років захворіла гостро після перенесеної 3 тижні тому стрептококової інфекції на шкірі. З'явилися набряки обличчя, нижніх кінцівок, головний біль, темний колір сечі (колір ''кока-коли''). АТ- 125/60 мм рт.ст. Аналіз сечі: білок - 3,3^o/_oo, питома вага - 1012, лейкоцити - 1-3 в п/з, еритроцити вкри-вають все п/з. Добова протеїнурія - 980 мг. Діу-рез - 550 мл. Загальний білок крові - 60 г/л. Альбуміни - 55%. Холестерин - 4,5 ммоль/л. Креатинін сироватки - 89 мкмоль/л. Який найбільш імовірний діагноз? A 4-year-old girl became acutely ill after suffering a streptococcal infection on the skin 3 weeks ago. Swelling of the face, lower limbs, headache, dark urine color appeared ( 'Coca-cola' color). Blood pressure - 125/60 mmHg. Urine analysis: protein - 3.3^o/_oo, specific gravity - 1-3 in p/z, erythrocytes. - daily proteinuria - 550 ml. Serum albumin - 4.5 mmol/l . What is the most likely diagnosis?

Гостре ураження нирок Acute kidney injury

Гломерулонефрит, нефротичний синдром Glomerulonephritis, nephrotic syndrome

Сечокам'яна хвороба Urolithiasis

Пієлонефрит Pyelonephritis

Гломерулонефрит, нефритичний синдром Glomerulonephritis, nephritic syndrome

419 / 1500
У немовляти 7 днів від народження спостерігаються виражена млявість, повторне блювання, іноді фонтаном, рідкі випорожнення, ексикоз, що наростає, виражена гіпотонія. Прибавка маси тіла відсутня. Шкіра землисто-сіра, соски пігментовані. Визначається збільшення клітора, неповне зрощення статевих губ, неповне розділення уретри та вагіни. Виражені гіперкаліємія і гіпонатріємія, метаболічний ацидоз, гіпоглікемія. Рівень альдостерону крові знижений, підвищена активність реніну плазми. Який попередній діагноз? 7 days after birth, the baby has pronounced lethargy, repeated vomiting, sometimes with a fountain, liquid stools, increasing exicosis, pronounced hypotonia. There is no weight gain. Skin Earthy-gray, pigmented nipples. An enlarged clitoris, incomplete separation of the urethra and vagina. Pronounced hyperkalemia, hypoglycemia. The blood aldosterone level is elevated. What is the preliminary diagnosis?

Адреногенітальний синдром, гіпертонічна форма Adrenogenital syndrome, hypertensive form

Адреногенітальний синдром, проста вірильна форма Adrenogenital syndrome, simple viral form

Адреногенітальний синдром, солевтратна форма Adrenogenital syndrome, soleutrate form

Гермафродитизм Hermaphroditism

Синдром Шерешевського-Тернера Shereshevsky-Turner syndrome

420 / 1500
Чоловік віком 28 років під час проходження чергового медичного огляду скаржився на періодичні напади серцебиття. Об'єктивно спостерігається: AT - 130/80 мм рт. ст., пульс - 65/хв. За результатами ЕКГ реєструється вкорочення інтервалу P-Q, поява у складі комплексу QRS додаткової хвилі збудження, деформація комплексу QRS та негативний зубець Р. Про яку патологію свідчать скарги пацієнта та зміни на ЕКГ? A 28-year-old man complained of periodic attacks of palpitations during a regular medical examination. Objectively observed: AT - 130/80 mm Hg, pulse - 65/min. According to the results of the ECG, a shortening of the PQ interval, the appearance of an additional excitation wave in the QRS complex, a deformation of the QRS complex and a negative R wave are indicated by the patient's complaints and changes on the ECG?

Синдром WPW WPW Syndrome

Пароксизмальну шлуночкову тахікардію Paroxysmal ventricular tachycardia

Шлуночкову екстрасистолію Ventricular extrasystole

Пароксизмальну надшлуночкову тахікардію Paroxysmal supraventricular tachycardia

Блокаду ніжки пучка Гіса His bundle leg blockade

421 / 1500
Жінка 56-ти років звернулася до лікаря зі скаргами на підвищення температури до 38,5^oC, підвищену пітливість вночі та збільшення лімфовузлів в лівій пахвовій та лівій надключичній ділянках. У сімейному анамнезі рак молочної залози у бабусі. При фізикальному обстеженні в зазначених ділянках пальпуються збільшені, безболісні, щільні лімфовузли. При рентгенологічному дослідженні органів грудної порожнини розширення тіні середостіння за рахунок збільшених лімфовузлів. Після ексцизійної біопсії лімфовузла надключичної ділянки, виявлені атипові багатоядерні клітини Рід-Штернберга. Який діагноз є найбільш імовірним? A 56-year-old woman consulted a doctor with complaints of a temperature rise to 38.5^oC, increased sweating at night and an increase in lymph nodes in the left axillary and left supraclavicular areas In the family history of breast cancer, enlarged, painless lymph nodes are palpated in the indicated areas. On X-ray examination of the chest cavity due to enlarged lymph nodes, atypical multinucleated cells are found. - Sternberg. Which diagnosis is most likely?

Лімфома Ходжкіна Hodgkin lymphoma

Метастази раку молочної залози Breast cancer metastases

Туберкульоз Tuberculosis

Інфекційний мононуклеоз Infectious mononucleosis

Бронхогенна карцинома Bronchogenic carcinoma

422 / 1500
Під час зовнішнього акушерського дослідження вагітної сімейним лікарем висоту стояння дна матки визначено на рівні пупка. Який передбачуваний термін вагітності? During an external obstetric examination of a pregnant woman, the family doctor determined the height of the uterine fundus at the level of the navel. What is the expected term of pregnancy?

32 тижні 32 weeks

16 тижнів 16 weeks

8 тижнів 8 weeks

40 тижнів 40 weeks

24 тижні 24 weeks

423 / 1500
Дівчинка 10 років потрапила до відділення із проявами кардиту. В анамнезі вказано: два тижні тому було загострення хронічного тонзиліту. Який етіологічний фактор найімовірніший у цьому разі? A 10-year-old girl was admitted to the department with manifestations of carditis. The anamnesis indicated: there was an exacerbation of chronic tonsillitis two weeks ago. What is the most likely etiological factor in this case?

Стафілокок Staphylococcus

Стрептокок Streptococcus

Клебсієла Klebsiella

Протей Proteus

Пневмокок Pneumococcus

424 / 1500
Чоловік 27-ми років 3 роки тому переніс важку черепно-мозкову травму. Останній рік у нього з'явилися стани, коли він під час будь-якої роботи раптово застигає, не втрачаючи м'язового тонусу, вираз обличчя стає маскоподібним, що триває 1-2 хвилини. Після цього він повертається до попередньої діяльності, але подібні ''відключення'' не пам'ятає. Препарати якої групи доцільно призначити? A 27-year-old man suffered a severe brain injury 3 years ago. In the last year, he has had conditions in which, during any work, he suddenly stiffens, without losing muscle tone, the facial expression becomes mask-like, which lasts 1-2 minutes. After that, he returns to his previous activity, but he does not remember such 'disconnections'. Which group of drugs is appropriate to prescribe?

Антигіпертензивні засоби Antihypertensive drugs

Антиконвульсанти Anticonvulsants

Ноотропні засоби Nootropics

Антидепресанти Antidepressants

Антипсихотики Antipsychotics

425 / 1500
У хворого 47-ми років на восьмий день після операції з приводу панкреонекрозу, перитоніту з'явилось підвищення температури тіла до 39^oC, анемія, лейкоцитоз, гіпопротеїнемія, спленомегалія, токсична енцефалопатія. Яке ускладнення має розвиток у цій ситуації? On the eighth day after surgery for pancreatic necrosis, peritonitis, a 47-year-old patient developed an increase in body temperature to 39°C, anemia, leukocytosis, hypoproteinemia, splenomegaly, toxic encephalopathy. What complication does this situation have?

Гостра ниркова недостатність Acute renal failure

Гостра печінкова недостатність Acute liver failure

Гострий менінгіт Acute meningitis

Внутрішньоочеревинна кровотеча Intraperitoneal bleeding

Загальна гнійна інфекція (сепсис) General purulent infection (sepsis)

426 / 1500
Жінка 35 років, шліфувальниця, скаржиться на мерзлякуватість, похолодання та посиніння кінчиків пальців, тугорухомість у суглобах кистей, відчуття стягнутості шкіри обличчя та кистей. Під час огляду: амімія обличчя, звуження ротової щілини за типом <<кисета>>, шкіра на щоках та кистях потовщена, кінчики пальців рук бліді, холодні. Над легенями прослуховується коробковий звук, поодинокі сухі хрипи. результат аналізу крові: еритроцити - 3,8·10^12/л, лейкоцити - 4,8·10^9/л, ШОЕ - 45 мм/год. СРБ++. Який діагноз найбільш імовірний? A 35-year-old woman, a grinder, complains of frostbite, coldness and blueness of the fingertips, stiffness in the joints of the hands, a feeling of tightness of the skin of the face and hands. During the examination: amymia face, narrowing of the oral cavity, the skin on the cheeks and hands is thickened, the tips of the fingers are cold, a box sound is heard above the lungs, the result of the blood test is 3.8·10^12 /l, leukocytes - 4.8·10^9/l, ESR - 45 mm/h. What is the most likely diagnosis?

Мікседема Myxedema

Системна склеродермія Systemic scleroderma

Хвороба Рейно Raynaud's disease

Вібраційна хвороба Vibration disease

Облітеруючий ендартеріїт кінцівок Endarteritis obliterans of the limbs

427 / 1500
Хвора 21-го року скаржиться на часті позиви до сечовиділення, біль та печію при сечовипусканні. Захворювання пов'язує з випадковим статевим актом 4 дні тому. При огляді: уретра пальпується у вигляді потовщеного болючого тяжа, набряк та гіперемія зовнішнього отвору сечівника, при натискуванні на уретру виділяється краплина гною. При бактеріоскопічному дослідженні виділень з уретри виявлено розміщені всередині лейкоцитів грамнегативні диплококи. Який діагноз є найбільш імовірним? A 21-year-old patient complains of frequent urges to urinate, pain and heartburn during urination. The disease is associated with accidental sexual intercourse 4 days ago. On examination: the urethra is palpable in the form of a thickened painful mass, swelling and hyperemia of the urethra, a drop of pus is released when the urethra is pressed. Gram-negative diplococci are found inside the urethra. What is the most likely diagnosis?

Неспецифічний уретрит Nonspecific urethritis

Уретрит викликаний Chlamidiya trachomatis Urethritis caused by Chlamydia trachomatis

Уретрит викликаний Neisseria gonorrhoeae Urethritis caused by Neisseria gonorrhoeae

Гострий цистит Acute cystitis

Вагініт Vaginitis

428 / 1500
Завiдувач вiддiлення стацiонару хоче провести експертну оцiнку виконання лiкарями-ординаторами медико-технологiчних стандартiв обслуговування пацiєнтiв. Яку документацiю вiн повинен перевiрити з цiєю метою? The head of a hospital department wants to conduct an expert assessment of the fulfillment of medical and technological standards of patient care by resident doctors. What documentation should he check for this purpose?

Медичнi карти стацiонарних хворих Medical charts of inpatients

Статистичнi карти пацiєнтiв, що вибули iз стацiонару Statistical maps of patients discharged from the hospital

Карти лiкарських призначень Maps of medical appointments

Рiчний звiт лiкувально-профiлактичного закладу Annual report of the treatment and prevention facility

Журнал облiку оперативних втручань Operational interventions accounting log

429 / 1500
Роділля в пологах 8 годин. Пологова діяльність слабка, перейми тривають 25 секунд, 2 перейми за 10 хвилин. Під час піхвового дослідження встановлено: розкриття шийки матки до 4 см, передлежить голівка. Під час огляду вилились навколоплідні води з домішками меконію. Серцебиття плоду - 90/хв., глухе. Яка тактика ведення пологів? Labor in labor for 8 hours. Labor activity is weak, contractions last 25 seconds, 2 contractions in 10 minutes. During the vaginal examination, it was found: opening of the cervix up to 4 cm , the head is presented. During the examination, amniotic fluid with impurities of meconium was poured out. The heart rate of the fetus is 90/min. What is the delivery strategy?

Призначити родостимуляцію із застосуванням розчину окситоцину Prescribe birth stimulation using oxytocin solution

Спостереження, лікування дистресу плода Observation, treatment of fetal distress

Продовжити консервативне ведення пологів із постійним моніторингом стану плода Continue conservative management of childbirth with constant monitoring of the condition of the fetus

Накласти акушерські щипці Apply obstetric forceps

Термінове розродження шляхом операції кесаревого розтину Emergency delivery by caesarean section

430 / 1500
Під час проведення медичного огляду хворого чоловіка лікар звернув увагу на перекорнеальну ін'єкцію судин склери очей (<<віночок фіалково-блакитного кольору>>), кон'юктивіт, хейлоз, ангулярний стоматит, гіпертрофічний глосит. Яке із наведених захворювань можна припустити? During the medical examination of a sick man, the doctor drew attention to the percorneal injection of the vessels of the sclera of the eyes (<>), conjunctivitis , cheilosis, angular stomatitis, hypertrophic glossitis. Which of the following diseases can be assumed?

Гіповітаміноз тіаміну Thiamin hypovitaminosis

Гіповітаміноз ціанокобаламіну Cyanocobalamin hypovitaminosis

Гіповітаміноз рибофлавіну Riboflavin hypovitaminosis

Гіповітаміноз піридоксину Pyridoxine hypovitaminosis

Гіповітаміноз ніацину Niacin hypovitaminosis

431 / 1500
За 2 дні після вживання холодної їжі у чоловіка виникли скарги на різкий біль у горлі та ядуху. Дихання шумне, ЧД - 26/хв, температура тіла 39^oС. За результатами ларингоскопії: голосова щілина не проглядається, прикрита набряклими тканинами. Які термінові лікувальні заходи потрібно вжити? 2 days after eating cold food, the man complained of a sharp pain in the throat and pharynx. Breathing is noisy, BH - 26/min, body temperature 39^oС According to the results of laryngoscopy: the glottis is not visible, it is covered by swollen tissues. What urgent medical measures should be taken?

Дезінтоксикаційну терапію Detoxification therapy

Трахеостомію Tracheostomy

Уведення спазмолітиків Introduction of antispasmodics

Внутрішньовенне уведення гормонів Intravenous administration of hormones

Штучну кисневу вентиляцію Artificial oxygen ventilation

432 / 1500
Недоношена дитина від першої вагітності в терміні 37 тижнів народилась з масою 2400 г та зростом 51 см. Немовля збуджене, з тремором кінцівок, не смокче, спостерігаються порушення дихання, гепатоспленомегалія. Наприкінці першого дня з'явилась жовтяниця шкіри і слизових, на другий день - висипання на шкірі: як окремі пухирці, так і цілі їх скупчення в ділянці грудної клітки. Поставте попередній діагноз: A premature baby from the first pregnancy was born at 37 weeks with a weight of 2400 g and a height of 51 cm. The baby is excited, with tremors of the limbs, does not suck, breathing disorders are observed, hepatosplenomegaly. At the end of the first day, jaundice of the skin and mucous membranes appeared, on the second day - a rash on the skin: both individual blisters and their accumulation in the chest area. Make a preliminary diagnosis:

Пухирчатка новонароджених Neonatal pemphigus

Везикулопустульоз Vesiculopustulosis

Токсоплазмоз Toxoplasmosis

Вроджена герпетична інфекція Congenital herpes infection

Краснуха Krasnukha

433 / 1500
Хлопчик 7-ми років звернувся по допомогу до педіатра зі скаргами на повторювані епізоди нудоти і блювання протягом останніх 3-х днів. Блювання без домішок крові і жовчі, виникає переважно вранці і повторюється до 10-ти разів на день. Вживає здорову їжу. Це третій епізод блювання за останні 6 місяців. Об'єктивно: запах ацетону з рота, легкий ступінь дегідратації. Інші показники в нормі. Який діагноз найімовірніший? A 7-year-old boy sought help from a pediatrician with complaints of repeated episodes of nausea and vomiting during the past 3 days. Vomiting without blood and bile impurities, occurs mainly in the morning and is repeated up to 10 times a day. This is the third episode of vomiting in the last 6 months.

Виразкова хвороба шлунку Gastric ulcer disease

Синдром циклічного блювання Syndrome of cyclic vomiting

Гострий панкреатит Acute pancreatitis

Хронічний гастрит Chronic gastritis

Гострий гастрит Acute gastritis

434 / 1500
Чоловік 53-х років хворіє на цукровий діабет протягом 5-ти років. Звертається до лікаря зі скаргами на болі та судоми в нижніх кінцівках, відчуття оніміння в них, мерзлякуватість. При обстеженні нижніх кінцівок ступні звичайного кольору, теплі на дотик, усі види чутливості значно знижені, гіперкератози, пульсація на артеріях кінцівок збережена. Яке ускладнення найбільш імовірно виникло у пацієнта? A 53-year-old man has been suffering from diabetes for 5 years. He goes to the doctor with complaints of pain and cramps in the lower limbs, a feeling of numbness in them, frostbite. When examining the lower extremities, the feet are normal in color, all types of sensitivity are significantly reduced, pulsation in the arteries of the extremities is preserved. What complication most likely occurred in the patient?

Синдром діабетичної стопи, нейропатична форма Diabetic foot syndrome, neuropathic form

Синдром діабетичної стопи, змішана форма Diabetic foot syndrome, mixed form

Синдром діабетичної стопи, ішемічна форма Diabetic foot syndrome, ischemic form

Облітеруючий ендартеріїт Endarteritis obliterans

Хронічна венозна недостатність Chronic venous insufficiency

435 / 1500
Мікроклімат виробничого приміщення характеризується такими показниками: середня температура повітря - +35^oС, радіаційна температура - +30^oС, відносна вологість повітря - 50%, швидкість руху повітря - 0,01 м/с. Вкажіть основний шлях тепловіддачі за таких мікрокліматичних умов. The microclimate of the production premises is characterized by the following indicators: average air temperature - +35^oС, radiation temperature - +30^oС, relative air humidity - 50%, speed of movement air - 0.01 m/s. Specify the main path of heat transfer under such microclimatic conditions.

Випаровування Evaporation

Кондукція Conduction

Конвекція Convection

Випромінювання Radiation

- -

436 / 1500
Десятирічна дитина скаржиться на млявість та зниження апетиту. Під час об’єктивного обстеження виявлено: блідість шкіри, легка жовтяниця, субіктеричність склер, глосит, помірна гепатомегалія. З анамнезу відомо про хірургічне втручання - резекцію частини тонкого кишківника. У загальному аналізі крові виявлено: еритроцити - 3,0·10^12/л, гемоглобін - 90 г/л, кольоровий показник - 1,15, тромбоцити - 180·10^9/л, лейкоцити - 45,3·10^9/л, паличкоядерні 4%, сегментоядерні 36%, еозинофіли 3%, лімфоцити 52%, моноцити 5%, ШОЕ 4 мм/год, макроцитоз, анізоцитоз, пойкілоцитоз. Встановіть імовірний діагноз. A ten-year-old child complains of lethargy and decreased appetite. During the objective examination, the following were found: pale skin, mild jaundice, subicteric sclera, glossitis, moderate hepatomegaly. From the history it is known about surgical intervention - resection of a part of the small intestine. The general blood analysis revealed: erythrocytes - 3.0·10^12/l, hemoglobin - 90 g/l, color index - 1.15, platelets - 180·10^9/ l, leukocytes - 45.3·10^9/l, rod nuclear 4%, segmentonuclear 36%, eosinophils 3%, lymphocytes 5%, ESR 4 mm/h, macrocytosis, anisocytosis, poikilocytosis.

Залізодефіцитна анемія Iron deficiency anemia

Апластична анемія Aplastic anemia

Постгеморагічна анемія Posthemorrhagic anemia

В_12 -фолієводефіцитна анемія B_12 - folic acid deficiency anemia

Гемолітична анемія Hemolytic anemia

437 / 1500
Жінка 23-х років. Скарги на ріжучий біль внизу живота, почастішання сечовипускання, підвищення температури тіла до 37,7^oC, збільшення виділень зі статевих шляхів жовтуватого кольору. Остання менструація 18 днів назад. В анамнезі: погіршення стану через 5-6 днів після незахищеного статевого акту. Симптом Щоткіна негативний. Бімануально: двобічне збільшення додатків. Діагноз: 23-year-old woman. Complaints of cutting pain in the lower abdomen, increased frequency of urination, increase in body temperature to 37.7^oC, increased secretions from the genital tract of yellowish color Last menstruation 18 days ago: worsening of condition after unprotected intercourse. Bimanual: bilateral enlargement.

Кандидозний вульвовагініт Candida vulvovaginitis

Двобічний сальпінгоофорит Bilateral salpingo-oophoritis

Ендометрит Endometritis

Апендицит Appendicitis

Бактеріальний вагіноз Bacterial vaginosis

438 / 1500
У недоношеної дитини, народженої в терміні гестації 34 тижні, за 4 години після народження спостерігається: тахіпное, дихання по типу гойдалки, западання грудини, експіраторні шуми. Частота дихання - 80/хв. Аускультативно над легенями прослуховується послаблене дихання з непостійними різнокаліберними хрипами. Під час рентгенологічного дослідження легенів виявлено: повітряна бронхограма та нодозно-ретикулярна сітка. Який найімовірніший діагноз? In a premature baby born at 34 weeks' gestation, 4 hours after birth, the following symptoms are observed: tachypnea, swing-like breathing, sternum depression, expiratory noises. Respiratory rate - 80/min. Weak breathing is heard over the lungs. During the X-ray examination of the lungs, an air bronchogram and a reticular mesh are found. What is the most likely diagnosis?

Ателектази легенів Atelectasis of the lungs

Синдром масивної меконіальної аспірації Massive meconial aspiration syndrome

Хвороба гіалінових мембран Disease of hyaline membranes

Пологова травма Birth trauma

Пневмонія новонароджених Pneumonia of newborns

439 / 1500
Жінка звернулась зі скаргами на мажучі кров'янисті виділення, біль в попереку. Остання менструація 2,5 місяці тому. При об'єктивному обстеженні: шийка матки ціа-нотична, бочкоподібна, розширена, зовнішнє вічко розташоване ексцентрично, матка збільшена як до 5 тижнів вагітності (пісочний годинник), придатки з обох боків не пальпуються, виділення з цервікального каналу кров'я-нисті, незначні. Яку патологію можна припустити? A woman complained of spotting bleeding, lower back pain. The last menstruation was 2.5 months ago. On objective examination: the cervix was notic, barrel-shaped, expanded, the external eye is located eccentrically, the uterus is enlarged as before 5 weeks of pregnancy (hourglass), the appendages on both sides are not palpable, the discharge from the cervical canal is bloody, insignificant. What pathology can be assumed?

Субмукозний міоматозний вузол, що народжується Submucosal myomatous nodule at birth

Поліп цервікального каналу Cervical canal polyp

Міома шийки матки Cervical myoma

Рак шийки матки Cervical cancer

Шийкова вагітність Cervical pregnancy

440 / 1500
Пацієнтка віком 62 роки лікується з приводу двобічного гонартрозу, регулярно приймає диклофенак натрію протягом 2-х років. Останні півроку відмічає підйоми АТ до 160/100 мм рт. ст. Під час рентгенологічного дослідження колінних суглобів виявлено: зниження висоти міжсуглобової щілини, крайові розростання суглобових поверхонь та епіфізарний остеопороз. У загальному аналізі сечі спостерігається: питома вага - 1010, еритроцити - 5-6 в п/з, лейкоцити - 10-12 в п/з, циліндри - 0-1 в п/з, глюкоза - не виявлено, білок - 0,22 г/л. Глюкоза плазми натще - 6,3 ммоль/л. Яка ймовірна причина виявлених змін у загальному аналізі сечі? A 62-year-old patient is being treated for bilateral gonarthrosis, has been regularly taking diclofenac sodium for 2 years. For the past six months, she has been experiencing blood pressure rises to 160/100 mm Hg During the X-ray examination of the knee joints, a decrease in the height of the joint space, marginal growths of the articular surfaces, and epiphyseal osteoporosis were observed. /z, cylinders - 0-1 in p/z, glucose - not detected, protein - 0.22 g/l. Fasting plasma glucose - 6.3 mmol/l. What is the probable cause of the detected changes in the general analysis of urine?

Тубулоінтерстиціальний нефрит Tubulointerstitial nephritis

Сечокам'яна хвороба Urolithiasis

Діабетична нефропатія Diabetic nephropathy

Гіпертензивна нефропатія Hypertensive nephropathy

Хронічний гломерулонефрит Chronic glomerulonephritis

441 / 1500
У хворого 56-ти років, що знаходиться в палаті інтенсивної терапії з приводу гострого інфаркту міокарда у ділянці передньої стінки лівого шлуночка, посилилась загальна слабкість та з'явилось відчуття ''завмирання'' у роботі серця. При аналізі ЕКГ виявлено: ритм синусовий, частота серцевих скорочень - 76/хв. Періодично з'являються поширені шлуночкові комплекси тривалістю 0,22 с., неправильної форми, після яких виявляються повні компенсаторні паузи. Яке ускладнення виникло у даного хворого? A 56-year-old patient, who is in the intensive care unit due to an acute myocardial infarction in the area of ​​the front wall of the left ventricle, has increased general weakness and a feeling of 'Stillness' in the work of the heart. During the ECG analysis, the rhythm of the heart is 76/min. Periodically, there are widespread ventricular complexes of 0.22 s., after which complete compensatory pauses are found did this patient have a complication?

Шлуночкова екстрасистолія Ventricular extrasystole

Передсердна екстрасистолія Atrial extrasystole

Атріовентрикулярна екстрасистолія Atrioventricular extrasystole

Повна блокада правої ніжки пучка Гіса Complete blockade of the right leg of the bundle of His

Повна блокада лівої ніжки пучка Гіса Complete blockade of the left leg of the bundle of His

442 / 1500
У хірургічному відділенні лікується жінка віком 65 років із приводу флегмони сідничної ділянки. Під час мікробіологічного дослідження рани збудником хірургічної інфекції виявлені неклостридіальні анаероби - протей та бактероїди. Яку лікувальну тактику треба застосувати насамперед у лікуванні цієї інфекції? In the surgical department, a 65-year-old woman is being treated for phlegmon of the buttocks. During the microbiological examination of the wound, non-clostridial anaerobes - proteus and bacteroids - were found to be the causative agent of the surgical infection. What are the treatment tactics should be used primarily in the treatment of this infection?

Антибіотикотерапію + профілактичне щеплення Antibiotic therapy + preventive vaccination

Застосування протигангренозної сиворотки Application of anti-gangrene serum

Радикальне висічення уражених тканин Radical excision of affected tissues

Антибактеріальну терапію Antibacterial therapy

Промивання гнійної порожнини антисептиками Washing of purulent cavity with antiseptics

443 / 1500
Чоловік віком 44 роки, перед зверненням за допомогою до психіатра 6 днів щоденно вживав 0,5-0,8 л горілки. На 3 день вночі не міг заснути, став тривожним, скаржився на те, що бачить <<фільм жахів, який показують на стіні>>, чує крики з вулиці, з погрозами вбити його, вважає, що він знаходиться <<на заводі>>. Який найімовірніший діагноз? A 44-year-old man used 0.5-0.8 liters of vodka daily for 6 days before seeking help from a psychiatrist. On the 3rd day, he could not fall asleep, became anxious, complained that he sees a <>, hears screams from the street, with threats to kill him, believes that he is in a <>. What is the most likely diagnosis?

Алкогольний галюциноз Alcoholic hallucinosis

Соматогений психоз Somatogenic psychosis

Обсесивно-фобічний невроз Obsessive-phobic neurosis

Професійний делірій Professional delirium

Алкогольний делірій Alcoholic delirium

444 / 1500
Жінка 38 років скаржиться на тягучий біль внизу живота і в попереку протягом останнього місяця, який посилюється напередодні менструації; передменструальні темні кров'яні виділення. В анамнезі зазначено: 4 штучних аборти, 1 пологи. Під час УЗД виявлено окремі осередки підвищеної ехогенності у міометрії, збільшення передньо-заднього розміру матки, округлі гіпоехогенні включення діаметром 2 мм. Який діагноз найімовірніший? A 38-year-old woman complains of pulling pain in the lower abdomen and lower back during the last month, which worsens on the eve of menstruation; premenstrual dark bloody discharge. The anamnesis indicates: 4 artificial abortions, 1 delivery. During the ultrasound scan, some hyperechoic cells were detected in the myometrium, an increase in the anterior-posterior size of the uterus, and rounded hypoechoic inclusions with a diameter of 2 mm. What is the most likely diagnosis?

Аденоміоз Adenomyosis

Гормонопродукуюча пухлина яєчників Hormone-producing ovarian tumor

Ендометріоз яєчників Ovarian endometriosis

Ретроцервікальний ендометріоз Retrocervical endometriosis

Хоріонепітеліома Chorioepithelioma

445 / 1500
Чоловік страждає на попереково-крижовий радикуліт близько 10 років. Останнє загострення сталося близько 5 місяців тому. Болі по зовнішній поверхні стегна, гомілки, через тил стопи до великого пальця. Під час огляду встановлено: парез гомілкового м'яза, слабкість тильного згинання великого пальця. Рефлекси збережені. Який імовірний діагноз? A man has been suffering from lumbosacral sciatica for about 10 years. The last exacerbation occurred about 5 months ago. Pain on the outer surface of the thigh, lower leg, through the back of the foot to the big toe During the examination, it was found: paresis of the tibial muscle, weakness of the dorsiflexion of the thumb. What is the probable diagnosis?

Дисциркуляторна мієлопатія Dyscirculatory myelopathy

Мієліт Myelitis

Пухлина спинного мозку Spinal cord tumor

Грижа диска L5-S1 L5-S1 disc herniation

Грижа диска L4-L5 L4-L5 disc herniation

446 / 1500
Дівчина 22-х років звернулася до лікаря зі скаргами на задишку, загрудинний біль, відчуття страху та ''стороннього тіла'' у горлі. Зазначені симптоми з'явилися в момент перегляду фільму з друзями і вже протягом 40 хвилин зберігаються і не зникають. Пацієнтка стверджує, що подібний напад у неї не вперше, але раніше вона могла опанувати цей стан та заспокої-тися. При фізикальному обстеженні перед лікарем тремтяча та спітніла дівчина, температура тіла - 37,5^oC, артеріальний тиск - 130/90 мм рт.ст., пульс - 112/хв., частота дихання - 18/хв., SpO_2 - 99% при кімнатному повітрі. Лікар не може заспокоїти пацієнтку. Вона стверджує, що помирає та благає про допомогу. Який наступний крок лікаря буде найбільш доречним? A 22-year-old girl consulted a doctor with complaints of shortness of breath, chest pain, a feeling of fear and a 'foreign body' in her throat. The indicated symptoms appeared at the time of watching a movie with friends, they persist for 40 minutes and do not disappear. The patient claims that this is not the first time she has had such an attack, but she was able to control this condition and calm down. During the physical examination in front of the doctor, the girl shivered and sweated body temperature - 37.5°C, blood pressure - 130/90 mmHg, pulse rate - 18/min, SpO_2 - 99% on room air. The doctor cannot calm the patient down claims to be dying and begs for help. What would be the most appropriate next step for the doctor?

Ввести метопролол внутрішньовенно Enter metoprolol intravenously

Розпочати курс флуоксетину перорально Start oral fluoxetine course

Призначити нітрогліцерин перорально та спостерігати мінімум 5 хвилин Give nitroglycerin orally and observe for at least 5 minutes

Призначити аспірин перорально та доставити до найближчого реперфузійного центру Prescribe aspirin orally and transport to the nearest reperfusion center

Ввести феназепам внутрішньовенно Inject phenazepam intravenously

447 / 1500
За даними лабораторного контролю якості питної води з водогону отримані наступні результати: каламутність - 1,5 мг/м^3, запах - 3 бали, присмак - металевий - 2 бали, колір - світло-жовтий, колірність - 20^o, температура - 12^oС. Який із показників не відповідає гігієнічним вимогам? According to the laboratory quality control of drinking water from the tap, the following results were obtained: turbidity - 1.5 mg/m^3, smell - 3 points, aftertaste - metallic - 2 points, color - light yellow, chroma - 20°C, temperature - 12°C. Which of the indicators does not meet the hygienic requirements?

Температура Temperature

Запах Smell

Колірність Color

Каламутність Turbidity

Присмак Taste

448 / 1500
Пацієнт віком 65 років з артеріальною гіпертензією в анамнезі скаржиться на запаморочення, серцебиття, що виникли протягом 1 години. Об'єктивно спостерігається: АТ - 80/40 мм рт. ст., ЧСС - 150/хв, пульс - 106/хв. На ЕКГ виявлено: зубець Р відсутній, інтервали R-R різні, частота шлуночкових скорочень 136-148/хв. Яку допомогу необхідно надати пацієнту першочергово? A 65-year-old patient with a history of arterial hypertension complains of dizziness, palpitations that occurred within 1 hour. Objectively observed: BP - 80/40 mm Hg Art., heart rate - 106/min. On the ECG, the P wave is absent, the ventricular rate is 136-148/min. What is the priority for the patient?

Призначення внутрішньовенно аміодарону Administration of intravenous amiodarone

Невідкладна електрична кардіоверсія Emergency electrical cardioversion

Імплантація кардіостимулятора Implantation of a pacemaker

Призначення внутрішньовенно блокаторів кальцієвих каналів Prescription of intravenous calcium channel blockers

Призначення внутрішньовенно beta-блокаторів Prescription of intravenous beta-blockers

449 / 1500
При плановому обстеженні 22-річної вагітної (30 тижнів) двічі у сечі виявлена ізольована бактеріурія. Вагітність перебігає без патології. Визначте тактику ведення вагітної: During a routine examination of a 22-year-old pregnant woman (30 weeks), isolated bacteriuria was detected twice in the urine. The pregnancy proceeds without pathology. Determine the tactics of managing the pregnant woman:

Динамічне спостереження Dynamic Observation

Фізіотерапевтичне лікування Physiotherapy treatment

Призначення ампіциліну Prescription of ampicillin

Призначення ципрофлоксацину Ciprofloxacin Prescription

Фітотерапія Phytotherapy

450 / 1500
Пацієнт віком 45 років скаржиться на висип в ділянці шкіри тулуба, верхніх та нижніх кінцівок. Появу висипу пов'язує з нещодавно перенесеною стресовою ситуацією. Об'єктивно спостерігається: на шкірі запальні папули, що мають тенденцію до розповсюдження та вкриті рихлими сріблясто-жовтими лусочками. Під час пошкрябування елеметів висипу спостерігається симптом 'стеаринової плями'. Генеалогічній анамнез не обтяжений. Який найімовірніший діагноз? A 45-year-old patient complains of a rash on the skin of the trunk, upper and lower limbs. He associates the appearance of the rash with a recently experienced stressful situation. Objectively observed: inflammatory papules with a tendency to spread and covered with loose silver-yellow scales. When scraping the rash, the symptom of a 'stearin spot' is observed. What is the most likely diagnosis?

Алергічний дерматит Allergic dermatitis

Дерматофітія Dermatophytia

Екзема Eczema

Червоний плоский лишай Red lichen planus

Псоріаз Psoriasis

451 / 1500
Пацієнт віком 24 роки скаржиться на гострий біль та висип у вигляді пухирців на лівій половині тулуба. Захворювання пов'язує з переохолодженням. Об'єктивно спостерігається: на шкірі тулуба зліва за ходом нерва згруповані дрібні пухирці, по периферії пухирців - запальна еритема. Який найімовірніший діагноз? A 24-year-old patient complains of acute pain and a rash in the form of blisters on the left half of the body. The disease is associated with hypothermia. Objectively observed: on the skin of the body On the left side of the course of the nerve are grouped small bubbles, on the periphery of the bubbles - inflammatory erythema. What is the most likely diagnosis?

Герпетиформний дерматоз Дюрінга Dühring's herpetiform dermatosis

Оперізуючий лишай Zingles

Простий пухирчатий лишай Liquor simplex

Екзема Eczema

Дерматит Dermatitis

452 / 1500
Жінка 65 років доставлена до лікарні зі скаргами на слабкість у лівих кінцівках, яка виникла вранці і поступово наростала протягом дня. Об'єктивно встановлено: свідомість ясна, АТ - 190/100 мм рт. ст., ЧСС - 80/хв., ритмічний, систолічний шум на шиї в проєкції біфуркації правої загальної сонної артерії. Під час дослідження неврологічного статусу встановлено: зглаженість носо-губної складки зліва, девіація язика вліво, зниження м'язової сили в лівих кінцівках до 3 балів, сухожилкові рефлекси S>D, позитивний симптом Бабінського зліва, лівобічна гемігіпестезія. Який найімовірніший діагноз у пацієнтки? A 65-year-old woman was brought to the hospital with complaints of weakness in the left limbs, which arose in the morning and gradually increased during the day. Objectively established: consciousness is clear, blood pressure - 190/100 mmHg, heart rate - 80/min., rhythmic, systolic murmur on the neck in the projection of the right common carotid artery. During the examination of the neurological status, it was established: smoothing of the nasolabial fold on the left, deviation of the tongue to the left 'ulcer strength in the left limbs up to 3 points, tendon reflexes S>D, positive Babinski's symptom on the left, left-sided hemiesthesia. What is the most likely diagnosis for the patient?

Геморагічний інсульт Hemorrhagic stroke

Дисциркуляторна енцефалопатія Dyscirculatory encephalopathy

Ішемічний інсульт Ischemic stroke

Розсіяний склероз Multiple sclerosis

Гостра гіпертензивна енцефалопатія Acute hypertensive encephalopathy

453 / 1500
Через глузування однолітків пацієнт-ка для схуднення протягом 6 місяців відмовлялася від їжі. Різко виснажена, із землистого кольору обличчям, з трофічними виразками на шкірі. Цукор у крові знижений, білок присутній у сечі, анемія, астено-адинамічні порушення. Визначте психічний розлад: Due to peer ridicule, the patient refused to eat for 6 months to lose weight. She is severely exhausted, with an earthy complexion, with trophic ulcers on the skin. Low blood sugar , protein is present in the urine, anemia, astheno-adynamic disorders. Define mental disorder:

Психічна анорексія Anorexia nervosa

Психічна астенія Mental asthenia

Психічна булімія Bulimia Psychiatry

Психогенна депресія Psychogenic depression

Психічна адинамія Mental adynamia

454 / 1500
Хворий 60-ти років скаржиться на інтенсивні болі стискаючого характеру, локалізовані за грудниною з іррадіацією у ліву лопатку та нижню щелепу. Біль тривалий, не знімається нітрогліцерином. Об'єктивно: холодний піт. Хворий збуджений. Аускультативно: тахікардія, приглушеність тонів. Артеріальний тиск - 100/70 мм рт.ст. ЕКГ-комплекси QS та підйом S-T вище ізолінії в І, аVL, V1, V2, V3, V4. Який діагноз є найбільш імовірним? A 60-year-old patient complains of intense squeezing pains, localized behind the sternum with radiation to the left scapula and lower jaw. The pain is long-lasting, not relieved by nitroglycerin. Ob' Objectively: the patient is agitated. Tachycardia, muffled tones. ECG-complexes above I, aVL, V3. What is the diagnosis is the most likely?

Міжреберна невралгія Intercostal neuralgia

Стенокардія Принцметала Prinzmetal Angina

Тромбоемболія легеневої артерії Thromboembolism of the pulmonary artery

Інфаркт міокарда Myocardial infarction

Лівосторонній плеврит Left-sided pleurisy

455 / 1500
Чоловік 35 років надійшов до клініки з явищами хірургічного сепсису, джерелом якого, ймовірно, був великий карбункул лопаткової ділянки. Під час обстеження виявлено вторинні гнійні вогнища в печінці і правій легені. Укажіть фазу хірургічного сепсису: A 35-year-old man came to the clinic with symptoms of surgical sepsis, the source of which was probably a large carbuncle of the scapula. During the examination, secondary purulent foci were found in the liver and right lungs. Specify the phase of surgical sepsis:

Термінальна Terminal

Токсемія Toxemia

Септицемія Septicemia

Септикопіемія Septicopemia

Гнійно-резорбтивна лихоманка Suppurative resorptive fever

456 / 1500
У чоловіка, який добу тому повернувся з Африки, у пахвовій ділянці було виявлено різко болючий пакет лімфовузлів, шкіра над яким гіперемована. Припускається чума, бубонна форма. Чим проводити екстрену профілактику контактним особам? In a man who returned from Africa a day ago, a sharply painful package of lymph nodes was found in the armpit, the skin over which is hyperemic. Plague, bubonic form is assumed. What to do emergency prevention for contact persons?

Доксициклін Doxycycline

Флуконазол Fluconazole

Гетерологічна сироватка Heterologous serum

Гомологічний імуноглобулін Homologous immunoglobulin

Празіквантел Praziquantel

457 / 1500
До ендокринолога звернулася мати чотирирічної дівчинки зі скаргами на неправильну будову зовнішніх статевих органів дитини: гіпертрофію клітора, великі статеві губи, що нагадують мошонку, а також прискорений ріст, появу аксилярного та лобкового оволосіння, зниження тембру голосу. Яке обстеження необхідно провести для підтвердження діагнозу? The mother of a four-year-old girl turned to an endocrinologist with complaints about the abnormal structure of the child's external genitalia: hypertrophy of the clitoris, large labia resembling a scrotum, as well as accelerated growth, the appearance axillary and pubic hair, decreased voice tone. What examination is necessary to confirm the diagnosis?

Визначення 17-ОН-прогестерону сироватки крові та екскреції 17-кетостероїдів з сечею Determination of serum 17-OH-progesterone and excretion of 17-ketosteroids with urine

Визначення кісткового віку Determining bone age

Каріотипування Karyotyping

Визначення ТТГ у крові Determination of TSH in blood

Визначення АКТГ у крові Determination of ACTH in blood

458 / 1500
Дівчина 18 років скаржиться на тяжкість і розпирання в правому підребер'ї. Стан посилюється після їжі чи прийнятті но-шпи. Живіт мякий, безболісний, печінка не збільшена. АсТ - 35 ОД, АлТ - 40 ОД. Аналіз крові показав: лейкоцити - 5,2 Г/л, ШОЕ - 7 мм/год. УЗД встановлено: печінка не збільшена. Жовчний міхур натщесерце 68 мл, після жовчогінного сніданку на 45 хв. - 64 мл. Стінка не потовщена. Який найбільш імовірний діагноз? An 18-year-old girl complains of heaviness and distention in the right hypochondrium. The condition worsens after eating or taking no-shpa. The abdomen is soft, painless, the liver is not enlarged. AsT - 40 U. Blood analysis showed: leukocytes - 7 mm/h. The liver was not enlarged, 68 ml after a choleretic breakfast. - 64 ml. The wall is not thickened. What is the most likely diagnosis?

Функціональна диспепсія Functional dyspepsia

Дискінезія жовчного міхура за гіпотонічним типом Gall bladder dyskinesia of the hypotonic type

Дискінезія жовчного міхура за гіпертонічним типом Gall bladder dyskinesia of the hypertensive type

Синдром подразненого кишечника Irritable bowel syndrome

Хронічний холецистит у стадії нестійкої ремісії Chronic cholecystitis in the stage of unstable remission

459 / 1500
У дитини 3-х місяців з ознаками рахіту мають місце симптоми Хвостека, Труссо, Маслова. Добу тому батьки спостерігали напад ціанозу, витрішкуватість, дитина не дихала, вкрилась липким потом. Через одну хвилину настав гучний видих, після чого стан нормалізувався. Яка основа виникнення вищезазначених симптомів захворювання дитини? A 3-month-old child with signs of rickets has symptoms of Khvostek, Trousseau, Maslov. A day ago, the parents observed an attack of cyanosis, lethargy, the child was not breathing, covered with sticky After one minute, there was a loud exhalation, after which the condition normalized. What is the basis of the above-mentioned symptoms of the child?

Підвищення рівня кальцію крові Increased blood calcium level

Метаболічний ацидоз Metabolic acidosis

Зниження рівня кальцію крові Decreased blood calcium level

Зниження рівня фосфору крові Decrease in blood phosphorus

Підвищення рівня фосфору крові Increased blood phosphorus level

460 / 1500
У недоношеного новонародженого хлопчика, який народився від V вагітності, I пологів, у терміні гестації 27 тижнів, після народження дихання нерегулярне, становить <30/хв., SpO_2 - 70 %. Спостерігаються втягнення податливих ділянок грудної клітки, експіраторний стогін. Без кисневої підтримки виникає генералізований ціаноз. Аускультативно виявлено крепітуючі хрипи в базальних відділах. Дані про профілактику РДС відсутні. Яку тактику лікування потрібно вибрати? In a premature newborn boy born from V pregnancy, I delivery, at 27 weeks of gestation, after birth breathing is irregular, is <30/min., SpO_2 - 70%. There are retractions of the chest, expiratory wheezing. There is no evidence of wheezing in the basal regions. What treatment tactics should be chosen?

Введення препарату сурфактанту інтратрахеально через 2 години після народження Intratracheal surfactant administration 2 hours after birth

Введення препарату сурфактанту інтратрахеально в перші 15 хвилин після народження Intratracheal surfactant administration in the first 15 minutes after birth

Киснева терапія за допомогою кисневого намету Oxygen therapy using an oxygen tent

Призначення антибактеріальної терапії Prescription of antibacterial therapy

ШВЛ мішком Амбу та маскою Ventilation with Ambu bag and mask

461 / 1500
Пацієнтка віком 35 років скаржиться на біль у правій гомілці, який посилюється під час ходьби. Об'єктивно спостерігається: на правій гомілці за ходом варикозно розширеної малої підшкірної вени шкіра червона, гаряча на дотик, під час пальпації у верхній третині гомілки визначається довгасте болюче ущільнення розмірами 3х2 см. Яке ускладнення виникло у пацієнтки? A 35-year-old female patient complains of pain in her right shin, which worsens when walking. Objectively observed: on the right shin, along the course of a varicose small saphenous vein, the skin red, hot to the touch, during palpation in the upper third of the lower leg, an oblong painful induration measuring 3x2 cm is determined. What complication did the patient experience?

Фурункул Furnish

Тромбофлебіт Thrombophlebitis

Тромбоз глибоких вен Deep vein thrombosis

Лімфаденіт Lymphadenitis

Лімфангіт Lymphangitis

462 / 1500
У породіллі спостерігається масивна кровотеча після народження двійні через природні пологові шляхи. Дитяче місце та родові шляхи цілі. Дно матки вище пупка, матка під час пальпації м'яка, не реагує на введення утеротоніків. Яка найімовірніша причина кровотечі? A woman in labor has massive bleeding after the birth of twins through the natural birth canal. The placenta and birth canal are intact. The bottom of the uterus is above the navel, the uterus is soft on palpation, does not respond to the administration of uterotonics. What is the most likely cause of bleeding?

Атонія матки Uterine atony

Пошкодження шийки матки Cervical damage

Затримка частки плаценти Delayed part of the placenta

Гіпотонія матки Uterine hypotonia

Розрив матки Rupture of uterus

463 / 1500
Під час огляду плаценти, яка щойно народилася, встановлено: наявність дефекту розміром 2х3 см. Кровотечі немає. Яка подальша тактика ведення пацієнтки? During the examination of the newly born placenta, it was established: the presence of a defect measuring 2x3 cm. There is no bleeding. What are the patient's further management tactics?

Екстирпація матки Uterus extirpation

Інструментальна ревізія порожнини матки Instrumental revision of the uterine cavity

Призначення утеротонічних препаратів Prescription of uterotonic drugs

Ручна ревізія порожнини матки Manual revision of the uterine cavity

Зовнішній масаж матки External uterine massage

464 / 1500
Пацієнтка віком 24 роки скаржиться на папульозний висип в ділянці зовнішніх статевих органів. Висип неболючий, без свербіння, різко відмежований від здорової шкіри. З анамнезу відомо, що два місяці тому на великій статевій губі самостійно, без лікування, зникла виразка округлої форми з твердим гладеньким дном. Який імовірний діагноз? A 24-year-old patient complains of a papular rash in the area of ​​the external genitalia. The rash is painless, without itching, sharply separated from healthy skin. It is known from the anamnesis that two months therefore, a round ulcer with a hard smooth bottom disappeared on the labia majora independently, without treatment. What is the probable diagnosis?

Вторинний сифіліс Secondary syphilis

Токсидермія Toxidermy

Висівкоподібний лишай Ban-like lichen

Кір Measles

Висипний тиф Typhoid

465 / 1500
Хвора 55-ти років звернулась до лікаря зі скаргами на відчуття важкості в лівій гомілці, періодичні нічні судоми литкових м'язів. Хворіє близько 5 років, відколи виникли вищевказані скарги. Не лікувалась. Об'єктивно: на медіальній поверхні лівої гомілки та стегна наявні щільні, варикозно розширені підшкірні вени, не болючі при пальпації. Встановіть діагноз: A 55-year-old patient turned to the doctor with complaints of a feeling of heaviness in the left leg, periodic night cramps of the calf muscles. She has been sick for about 5 years, since the above complaints. It was not treated. Objectively: on the medial surface of the leg and thigh there are dense, varicose veins, not painful during palpation:

Посттромботичний синдром лівої нижньої кінцівки Postthrombotic syndrome of the left lower limb

Гострий тромбоз глибоких вен лівої нижньої кінцівки Acute deep vein thrombosis of the left lower limb

Варикозне розширення підшкірних вен лівої нижньої кінцівки Varicose dilatation of the subcutaneous veins of the left lower limb

Облітеруючий атеросклероз судин лівої нижньої кінцівки Obliterating atherosclerosis of vessels of the left lower limb

Гострий висхідний тромбофлебіт підшкірних вен лівої нижньої кінцівки Acute ascending thrombophlebitis of the subcutaneous veins of the left lower limb

466 / 1500
У хворого частота дихання - 28/хв., тахікардія - до 100/хв., ознаки інтоксикації. В зоні правої лопатки притуплення перкуторного звуку, бронхіальне дихання, поодинокі дрібноміхурчасті та крепітуючі хрипи. Рентгенологічно: масивна запальна інфільтрація в середньому легеневому полі. Через три доби на фоні лікування, що проводилось, виник напад кашлю з виділенням 200 мл гнійної мокроти, після чого температура тіла знизилась до субфебрильної, стан покращився. На рівні кута лопатки на фоні інфільтрації легені виявлено округле просвітлення з горизонтальним рівнем. Встановіть діагноз: The patient's breathing rate is 28/min., tachycardia - up to 100/min., signs of intoxication. In the area of ​​the right scapula, dulling of the percussion sound, bronchial breathing, single X-ray: massive inflammatory infiltration in the middle lung field. After three days of treatment, a cough with 200 ml of purulent sputum appeared. The condition improved at the level of the scapula against the background of infiltration of the lung, a rounded illumination with a horizontal level was detected. Establish a diagnosis:

Абсцес легені Lung abscess

Емпієма плеври Empyema of the pleura

Правобічний бронхіт Right-sided bronchitis

Гангрена правої легені Gangrene of the right lung

Бронхоектатична хвороба Bronchoectatic disease

467 / 1500
Жінка 32-х років звернулася до лікаря жіночої консультації зі скаргами на хронічний тазовий біль, який підсилюється під час менструацій, диспареунію, кровомазання напередодні та після менструації. Остання менструація 3 тижні потому. При огляді у дзеркалах: на шийці матки 2 кісти діаметром 3 і 5 мм синьо-багрового кольору, з яких виділяється темно-коричнева рідина. При бімануальному дослідженні: тіло матки шароподібної форми, збільшене до 6 тижнів вагітності, болісне при пальпації. Придатки з обох боків без особливостей. Лікаря проінформовано, що у найближчий час народження дитини не планується. Яка тактика лікування цієї пацієнтки є найбільш доречною? A 32-year-old woman turned to a gynecologist with complaints of chronic pelvic pain that worsens during menstruation, dyspareunia, bleeding before and after menstruation. Last menstruation 3 weeks later. On examination in mirrors: 2 cysts with a diameter of 3 and 5 mm, from which a dark brown liquid is released. On bimanual examination: the body of the uterus is spherical, enlarged to 6 weeks of pregnancy, painful on palpation. Appendices on both sides. The doctor is informed that the birth of a child is not planned in the near future. What is the most appropriate treatment for this patient?

Призначення андрогенів Androgens prescription

Хірургічне втручання Surgery

Призначення комбінованих оральних контрацептивів Prescription of combined oral contraceptives

Призначення антагоністів гонадотропін-рилізинг гормону Prescription of gonadotropin-releasing hormone antagonists

Контрольована гіперстимуляція яєчників Controlled ovarian hyperstimulation

468 / 1500
На третю добу після первинної хірургічної обробки вогнепального кульового поранення м'яких тканин стегна, стан пацієнта раптово погіршився. Скарги на біль розпираючого характеру в рані, що наростає протягом останніх 5 годин. Наявний набряк шкіри і підшкірної клітковини швидко збільшується. Краї рани зіяють, м'язи мають вигляд <<вареного м'яса>>, тьмяні, з сіро-брудним нальотом, у разі захоплення пінцетом - розпадаються. Який вид інфекції розвинувся у рані стегна? On the third day after the initial surgical treatment of a gunshot wound to the soft tissues of the thigh, the patient's condition suddenly worsened. Complaints of excruciating pain in the wound, which has been increasing during the last 5 hours. The swelling of the skin and subcutaneous tissue increases rapidly. The edges of the wound look like 'boiled meat', and in case of grasping with tweezers, they fall apart sore hips?

Гнійна інфекція рани Suppurative wound infection

Анаеробна інфекція Anaerobic infection

Бешиха Beshikha

Дифтерія рани Diphtheria wound

Аеробна грам-негативна інфекція рани Aerobic gram-negative wound infection

469 / 1500
Хворий 39-ти років страждає на хронічну ревматичну хворобу серця. Скаржиться на задишку під час фізичного навантаження, кашель з харкотинням, серцебиття. Аускультативно: І тон підсилений, діастолічний шум, тон відкриття мітрального клапану вислуховується на верхівці серця. Відзначається акцент ІІ тону над легеневою артерію. Ціаноз. На рентгенограмі - розширення кореня легенів, збільшення правого шлуночка та лівого передсердя. Який найбільш імовірний діаг-ноз? A 39-year-old patient suffers from chronic rheumatic heart disease. He complains of shortness of breath during physical exertion, cough with sputum, palpitations. Auscultation: I tone is increased, diastolic murmur, the opening of the mitral valve is heard at the apex of the heart. Cyanosis is noted on the lung root, enlargement of the right ventricle and the left atrium?

Стеноз легеневої артерії Pulmonary artery stenosis

Відкрита артеріальна протока Open ductus arteriosus

Мітральний стеноз Mitral stenosis

Стеноз гирла аорти Stenosis of the mouth of the aorta

Коарктація аорти Coarctation of the aorta

470 / 1500
Пацієнтка 31 року протягом 14 років страждає на системну склеродермію. Неодноразово лікувалася в стаціонарах. Скаржиться на періодичний тупий біль у ділянці серця, серцебиття, задишку, головний біль, набряки повік, схуднення, біль та деформацію суглобів кінцівок. Ураження якого органа погіршує прогноз цього захворювання? A 31-year-old patient has been suffering from systemic scleroderma for 14 years. She has been treated repeatedly in hospitals. She complains of periodic dull pain in the heart area, palpitations, shortness of breath, headache, swelling eyelids, weight loss, pain and deformation of the joints of the limbs. Damage to which organ worsens the prognosis of this disease?

Серця Hearts

Шлунково-кишкового тракту Gastrointestinal tract

Шкіри та суглобів Skin and joints

Легень Lung

Нирок Kidney

471 / 1500
Хворий 30-ти років звернувся до сімейного лікаря через 2 місяці після операції з приводу відкритого перелому плечової кістки. При огляді стан хворого задовільний, в ділянці операційної рани є нориця з незначним гнійним виділенням, почервоніння, флюктуація. На рентгенограмі: деструкція плечової кістки з секвестрами. Яке ускладнення виникло у хворого в післяопераційному періоді? A 30-year-old patient consulted a family doctor 2 months after surgery for an open fracture of the humerus. On examination, the patient's condition is satisfactory, there is a fistula in the area of ​​the surgical wound with slight purulent discharge, redness, fluctuation. On the X-ray: destruction of the humerus with sequestrations. What complications did the patient have in the postoperative period?

Гематогенний остеомієліт Hematogenous osteomyelitis

Післятравматична флегмона Post-traumatic phlegmon

Нагноєння рани Wound suppuration

Лігатурна нориця Ligatary fistula

Посттравматичний остеомієліт Post-traumatic osteomyelitis

472 / 1500
Під час судово-медичної експертизи трупа судово-медичний експерт описав у тім'яно-скроневій ділянці праворуч рану лінійної форми, розміром 6,4 см при зведених краях; краї нерівні, осаднені, в глибині рани видно тканинні перетинки. Дайте назву описаної рани: During the forensic medical examination of the corpse, the forensic medical expert described a linear wound measuring 6.4 cm with folded edges in the parietal-temporal area on the right; the edges are uneven, sedimented, tissue membranes are visible in the depth of the wound. Give the name of the described wound:

Розсічена рана Cut wound

Рубана рана Cutting wound

Забита рана Stacked wound

Різана рана Cut wound

Колота рана Puncture wound

473 / 1500
Дочка викликала сімейного лікаря до своєї матері 77 років. У хворої онко-логічне захворювання в термінальній стадії. Вона скаржиться на постійні болі та інші виснажливі симптоми. До якого закладу охорони здоров'я найдоцільніше госпіталізувати цю жінку? The daughter called the family doctor to her 77-year-old mother. The patient has an oncological disease in the terminal stage. She complains of constant pain and other debilitating symptoms. To which institution of health care, is it most expedient to hospitalize this woman?

Багатопрофільна територіальна лікарня Multidisciplinary territorial hospital

Хоспіс Hospice

Соціальний пансіонат Social boarding house

Територіальний центр соціального обслуговування пенсіонерів Territorial center of social service for pensioners

Онкологічний диспансер Oncological dispensary

474 / 1500
Чоловік 64 років поступив до інфекційного відділення. Під час обстеження діагностовано вірусний гепатит А. З анамнезу відомо, що він страждає на цукровий діабет 2-го типу, з приводу чого він протягом останніх 2 років отримував глібенкламід 20 мг на добу. Глікемія натще - 15,6 ммоль/л. Яка повинна бути подальша тактика лікування? A 64-year-old man was admitted to the infectious disease department. During the examination, viral hepatitis A was diagnosed. From the anamnesis, it is known that he suffers from type 2 diabetes, due to that he has been receiving glibenclamide 20 mg per day for the past 2 years. Fasting blood glucose - 15.6 mmol/l. What should be the further treatment tactics?

Перевести хворого на похідні сульфонілсечовини ІІІ генерації Transfer the patient to sulfonylureas of the III generation

Збільшити дозу глібенкламіду до 20 мг на добу Increase the dose of glibenclamide to 20 mg per day

Перевести хворого на інсулінотерапію Transfer the patient to insulin therapy

Додатково призначити бігуаніди Additionally assign biguanides

Додатково призначити акарбозу Additionally prescribe acarbose

475 / 1500
Дівчина віком 19 років скаржиться на напади ядухи до 4-5 разів на тиждень, нічні напади 1-2 рази на тиждень. З анамнезу відомо про наявність у пацієнтки бронхіальної астми, поліпоз носа, часті синусити. Упродовж останніх 5 років приймає симбікорт. Об'єктивно спостерігається: у легенях дихання везикулярне, послаблене, поодинокі розсіяні сухі свистячі хрипи. Додаванням якого з наведених нижче лікарських засобів можна покращити контроль перебігу бронхіальної астми у цієї пацієнтки? A 19-year-old girl complains of asthma attacks up to 4-5 times a week, nocturnal attacks 1-2 times a week. From the anamnesis, it is known that the patient has bronchial Asthma, nasal polyposis, frequent sinusitis. Over the past 5 years, she has been taking symbicort. Objectively, her breathing is vesicular, with scattered dry wheezing. Which of the following drugs can improve the course of bronchial asthma in this patient?

Теофіліни уповільненого виділення Theophylline delayed release

Монтелукаст Montelukast

Системні кортикостероїди Systemic corticosteroids

Дезлоратадин Desloratadine

Омалізумаб Omalizumab

476 / 1500
Пацієнт скаржиться на висип і свербіж, який посилюється у вечірній та нічний час і турбує протягом 2-х тижнів. Об'єктивно спостерігається: на бокових ділянках грудей, живота, в міжпальцьових складках визначається папульозно-везикульозний висип, що розташований попарно. Який найімовірніший діагноз? The patient complains of a rash and itching that worsens in the evening and at night and bothers him for 2 weeks. Objectively observed: on the sides of the chest, abdomen , a papular-vesicular rash located in pairs is determined in the interdigital folds. What is the most likely diagnosis?

Екзема Eczema

Токсикодермія Toxicoderma

Псоріаз Psoriasis

Нейродерміт Neurodermatitis

Короста Scabies

477 / 1500
Жінка 50 років госпіталізована до стаціонару на 5-й день хвороби зі скаргами на сильний головний біль, температуру тіла 39^oС, безсоння. Об'єктивно встановлено: стан тяжкий, трохи збуджена, шкіра обличчя гіперемована, склери ін'єктовані. Позитивний симптом Говорова-Годельє. На тулубі та кінцівках рясний розеольозно-петехіальний висип; тахікардія, АТ - 140/70 мм рт. ст. Для невідкладної хіміопрофілактики у вогнищі цієї інфекції призначають: A 50-year-old woman was hospitalized on the 5th day of illness with complaints of severe headache, body temperature of 39°C, insomnia. Objectively established: condition severe, hyperemic skin, positive Govorov-Petechial rash on the trunk, BP - 140/70 mmHg :

Левоміцетин Levomycetin

Доксициклін Doxycycline

Канаміцин Kanamycin

Норфлоксацин Norfloxacin

Аугментин Augmentin

478 / 1500
Хвора 46-ти років звернулася до жіночої консультації зі скаргами на помірні кров'янисті виділення з піхви, які з'явилися після затримки чергової менструа-ції на 1,5 місяці. Під час піхвового дослідження шийка матки чиста, матка не збільшена, рухома, неболюча, додатки не змінені. Ваш діагноз: A 46-year-old patient turned to a women's consultation with complaints of moderate vaginal bleeding, which appeared after the next menstruation was delayed by 1, 5 months. During the vaginal examination, the cervix is ​​clean, the uterus is not enlarged, it is not painful, the appendages have not changed:

Підслизова міома матки Submucosal myoma of the uterus

Аденоміоз Adenomyosis

Позаматкова вагітність Ectopic pregnancy

Дисфункційна маткова кровотеча Dysfunctional uterine bleeding

Рак тіла матки Cancer of the uterine body

479 / 1500
Під час судово-медичного розтину трупа чоловіка 59-ти років, що помер раптово вдома без наявних ознак насильницької смерті, встановлені рожевий колір шкіри та слизових оболонок, рідка кров яскраво-червоного кольору, повнокрів'я та яскраво-червоне забарвлення внутрішніх органів. В результаті судово-токсикологічного аналізу в крові був вия-влений етиловий спирт в концентрації 1,44^o/_oo та карбоксигемоглобін - 55%. Яка причина смерті? During the forensic autopsy of the corpse of a 59-year-old man who died suddenly at home without signs of violent death, the pink color of the skin and mucous membranes, liquid blood bright red color, full blood and bright red staining of internal organs. As a result of forensic toxicological analysis, ethyl alcohol was found in the blood at a concentration of 1.44% and carboxyhemoglobin - 55%. What was the cause of death?

Отруєння чадним газом Monoxide poisoning

Отруєння ціаністим калієм Poisoning with potassium cyanide

Отруєння миш'яком Arsenic poisoning

Отруєння аніліном Aniline poisoning

Отруєння алкоголем Alcohol poisoning

480 / 1500
До стаціонару госпіталізовано вагітну 28-ми років з наступним діагнозом: вагітність ІІІ, 7-8 тижнів; ревматизм, неактивна фаза, мітральний стеноз ІІІ ступеня. Який план ведення вагітної? A 28-year-old pregnant woman was hospitalized with the following diagnosis: pregnancy III, 7-8 weeks; rheumatism, inactive phase, mitral stenosis III degree. What is the management plan pregnant?

Призначення кортикостероїдів та імунодепресантів Prescription of corticosteroids and immunosuppressants

Переривання вагітності після проведення оперативного лікування вади (мітральної комісуротомії) Termination of pregnancy after surgical treatment of the defect (mitral commissurotomy)

Збереження вагітності, оперативне лікування (мітральна комісуротомія) Pregnancy preservation, operative treatment (mitral commissurotomy)

Переривання вагітності за медичними показаннями Termination of pregnancy for medical reasons

Збереження вагітності, планова госпіталізація в критичні терміни, консервативна терапія Preserving pregnancy, planned hospitalization in critical periods, conservative therapy

481 / 1500
У жінки віком 29 років захворювання розпочалося раптово з підвищення температури тіла до 39^oС, появи на шкірі обличчя, чола, навколо очей, шиї та передньої поверхні грудної клітки висипань темно-бузкового кольору. Скелетні м'язи в'ялі, пальпаторно болючі, сила у них знижена. Відмічає затруднення ковтання. Лабораторно виявлено: різкопозитивні гострофазові показники, ШОЕ - 39 мл/год, еритроцити - 2,9·10^12, Нb - 72 г/л. Визначення рівня якого ферменту буде мати найважливіше значення для верифікації діагнозу? In a 29-year-old woman, the disease began suddenly with an increase in body temperature to 39^oC, the appearance of rashes on the skin of the face, forehead, around the eyes, neck and front surface of the chest dark purple rash. Skeletal muscles are weak, palpable, difficulty in swallowing. Laboratory findings: sharp positive ESR - 2.9·10^12, Hb - 72 g/l. Determination of the level of which enzyme will be most important for the verification of the diagnosis?

Трансферину Transferin

Трансамінази Transaminases

Лужної фосфатази Alkaline phosphatase

Креатинфосфокінази Creatine phosphokinase

Мідьоксидази Copper oxidases

482 / 1500
Пацієнт скаржиться на підвищення температури тіла до 39^oС, озноб, головний біль, поганий сон і апетит. Об'єктивно спостерігається: на задній поверхні шиї великий багряно-ціанотичний різко болючий щільний інфільтрат. У центрі інфільтрату кілька отворів, крізь які виділяється густий жовто-зелений гній. Який найімовірніший діагноз? The patient complains of an increase in body temperature up to 39^oС, chills, headache, poor sleep and appetite. Objectively observed: on the back of the neck, a large purple cyanotic sharply painful infiltrate. In the center of the infiltrate, there are several openings through which thick yellow-green pus is released. What is the most likely diagnosis?

Абсцес Abscess

Фурункул Furnish

Гідраденіт Hydradenitis

Бешиха Beshikha

Карбункул Carbuncle

483 / 1500
У пацієнтки 29-ти років скарги на відсутність менструацій впродовж року. В анамнезі два аборти. Об'єктивно: вторинні статеві ознаки розвинені правильно, оволосіння за жіночим типом, галакторея з молочних залоз. Рівень пролактину значно підвищений. Яке дообстеження необхідно зробити для верифікації діагнозу? A 29-year-old patient complains of a lack of menstruation for a year. She has two abortions in her history. Objectively: secondary sexual characteristics are developed correctly, female-type hair growth, galactorrhea from the mammary glands. The level of prolactin is significantly increased. What additional examination is necessary to verify the diagnosis?

Мамографію Mammography

Визначення рівня пролактину в динаміці Determination of prolactin level in dynamics

УЗД молочних залоз Ultrasound of mammary glands

Генетичне обстеження Genetic examination

МРТ головного мозку MRI brain

484 / 1500
Хвора 32-х років скаржиться на втрату маси тіла, незважаючи на підвищений апетит, знервованість, тремор кінцівок. Об'єктивно: шкіра волога, щитоподібна залоза дифузно збільшена, безболісна, м'яка, рухома. Аналіз крові: підвищений рівень Т3, Т4 та знижений рівень ТТГ. Який найбільш імовірний діагноз? A 32-year-old patient complains of weight loss, despite increased appetite, nervousness, tremors of the limbs. Objectively: the skin is moist, the thyroid gland is diffusely enlarged, painless, soft, mobile. Blood test: elevated T3, T4 and decreased TSH levels. What is the most likely diagnosis?

Дифузний токсичний зоб Diffuse toxic goiter

Дифузний нетоксичний зоб Diffuse non-toxic goiter

Аденома щитоподібної залози Thyroid adenoma

Рак щитоподібної залози Thyroid cancer

Аутоімунний тиреоїдит Хашимото Hashimoto's autoimmune thyroiditis

485 / 1500
Чоловік 25-ти років, звернувся до лікаря зі скаргами на появу висипань в міжпальцевих проміжках, в ділянці променевозап'ястних суглобів, паховій ділянці та живота, які супроводжуються сильним свербежем, особливо в нічний час. Пацієнт зазначає, що після вечірнього душу покращення не наступає. Також йому здається, що як тільки висип проходить в одному місці, він одразу з'являється в іншому. При фізикальному обстеженні на фоні неураженої шкіри множинні еритематозні папули та екскоріації. Який діагноз є найбільш імовірним? A 25-year-old man turned to the doctor with complaints about the appearance of rashes in the interdigital spaces, in the area of ​​the radiocarpal joints, inguinal area and abdomen, which are accompanied by strong itching, especially at night. The patient notes that after the evening shower, it seems that as soon as the rash disappears, it appears in another. On physical examination, multiple erythematous papules and excoriations. What is the most likely diagnosis?

Епідермофітія Epidermophytia

- -

Обсесивно-компульсивний розлад Obsessive-compulsive disorder

Екзема Eczema

Короста Scabies

486 / 1500
Жінка за 10 днів після травми ділянки стопи відчула утруднення під час відкриття рота. Наступного дня з важкістю могла приймати їжу, з'явилося напруження м'язів потилиці, спини, живота. Під час прийому в стаціонар на 3 день хвороби встановлено тонічне напруження всіх груп м'язів, генералізовані судоми кожні 10-15 хвилин. Який найбільш імовірний діагноз? 10 days after the injury to the foot, the woman felt difficulty opening her mouth. The next day, she could hardly take food, there was tension in the muscles of the back of the head and back , abdomen. During admission to the hospital on the 3rd day of the illness, tonic tension of all muscle groups was established, generalized convulsions every 10-15 minutes. What is the most likely diagnosis?

Геморагічний інсульт Hemorrhagic stroke

Епілепсія Epilepsy

Правець Tetanus

Тетанія Tetany

Менінгітоенцефаліт Meningoencephalitis

487 / 1500
Хвора 20 років скаржиться на затримку менструації на десять днів, порушення менструальної функції відмічає вперше. Статеве життя регулярне. Об'єктивно встановлено: стан задовільний, живіт м'який, безболісний. АТ - 120/80 мм рт. ст., пульс - 72/хв. Під час УЗД виникла підозра на наявність прогресуючої трубної вагітності. Яка тактика лікаря жіночої консультації? A 20-year-old patient complains of a ten-day delay in menstruation, notices a violation of menstrual function for the first time. Sex life is regular. Objectively established: the condition is satisfactory, the abdomen is soft , painless. Blood pressure - 120/80 mm Hg. During the ultrasound, the presence of a progressive tubal pregnancy was suspected.

Терміново госпіталізувати хвору для уточнення діагнозу Urgently hospitalize the patient to clarify the diagnosis

Рекомендувати повторну явку за 1 тиждень для контрольного УЗД Recommend repeat appearance in 1 week for control ultrasound

Провести обстеження за тестами функціональної діагностики Conduct an examination using functional diagnostic tests

Видати лікарняний лист та направити хвору на визначення титру ХГ сечі Issue a sick letter and send the patient to determine the titer of hCG urine

Провести пункцію черевної порожнини через заднє склепіння піхви Perform a puncture of the abdominal cavity through the posterior vault of the vagina

488 / 1500
На другий день після тиреоїдектомії у пацієнтки з'явилися судоми. Яке лікування вона потребує насамперед? On the second day after thyroidectomy, the patient developed convulsions. What treatment does she need first?

Внутрішньовенне введення калія хлорида Intravenous administration of potassium chloride

Магній В_6+ калій внутрішньо Magnesium B_6+ potassium internally

Внутрішньовенне введення 10%-го кальція хлорида Intravenous administration of 10% calcium chloride

Кетамін внутрішньовенно Intravenous Ketamine

Внутрішньовенне введення 40%-го розчину глюкози Intravenous administration of 40% glucose solution

489 / 1500
Під час судово-медичного дослідження тіла немовляти виявлено: маса - 3500 г, довжина тіла - 50 см, пуповина гладенька, волога, блискуча, без ознак висихання, плавальні легеневі проби позитивні. Про що свідчать результати плавальних проб у цій ситуації? During the forensic examination of the baby's body, it was found: weight - 3500 g, body length - 50 cm, umbilical cord smooth, moist, shiny, without signs of drying, floating the lung tests are positive. What do the results of the swimming tests indicate in this situation?

Хвороба гіалінових мембран Disease of hyaline membranes

Дитина народилася мертвою The child was stillborn

Вторинний ателектаз Secondary atelectasis

Первинний ателектаз Primary atelectasis

Дитина народилася живою The child was born alive

490 / 1500
Жінка віком 25 років скаржиться на схуднення, нудоту, утруднене дихання, болі внизу живота, що виникли за два місяці після медикаментозного переривання вагітності. Під час бімануального обстеження встановлено: матка дещо збільшена, щільна, обмеженої рухомості, придатки незмінені. Визначення яких гормонів необхідно провести для диференційної діагностики? A 25-year-old woman complains of weight loss, nausea, difficulty breathing, pain in the lower abdomen, which occurred two months after medical termination of pregnancy. During a bimanual examination, it was found: the uterus is slightly enlarged, dense, with limited mobility, the appendages are unchanged. What hormones should be determined for differential diagnosis?

Прогестерону Progesterone

Кортизолу Cortisol

Тестостерону Testosterone

Хоріонічного гонадотропіну Chorionic gonadotropin

Гормонів щитовидної залози Thyroid hormones

491 / 1500
Чоловіка 48-ми років доставлено до лікаря у непритомному стані. В анамнезі зловживання алкоголем. При обстеженні шкіра бліда, на тулубі ''судинні зірочки'', печінковий запах з рота, язик обкладений коричневим нальотом. Підшкірні вени передньої стінки живота розширені у вигляді (caput medusae). При пальпації органів черевної порожнини позитивний симптом флуктуації, гепатомегалія, спленомегалія. Набряки нижніх кінцівок. Який стан у хворого є найбільш імовірним? A 48-year-old man was brought to the doctor in an unconscious state. He has a history of alcohol abuse. On examination, the skin is pale, there are 'vascular stars' on the body, liver odor from the mouth, the tongue is covered with a brown coating. The subcutaneous veins of the anterior wall are enlarged in the form of caput medusae. A positive symptom of fluctuation, hepatomegaly, is the patient's condition.

Перфоративна виразка шлунка Perforative gastric ulcer

Гостра печінкова недостатність Acute liver failure

Гостра шлунково-кишкова кровотеча Acute gastrointestinal bleeding

Гіпоглікемічна кома Hypoglycemic coma

Гіперглікемічна кома Hyperglycemic coma

492 / 1500
Чоловік 52 років страждає на виражену задишку під час фізичного навантаження, непродуктивний кашель. Хворіє 8 місяців. Палить 30 років. Під час аускультації в легенях з обох боків вислуховуються крепітуючі хрипи - <<тріск целофану>>. ЧДД - 26/хв., SpO_2 - 92%. Під час спірометрії виявили: помірні порушення функції зовнішнього дихання за рестриктивним типом. Запідозрили інтер-стиційний процес. Який метод дослідження найбільш ефективно допоможе верифікувати діагноз та прогноз? A 52-year-old man suffers from pronounced shortness of breath during physical exertion, a non-productive cough. He has been ill for 8 months. He has been smoking for 30 years. During auscultation, crackles are heard in the lungs on both sides Wheezing - 26/min. During spirometry, we suspected an interstitial process forecast?

Рентгенографія органів грудної клітки X-ray of chest organs

Торакоскопія з біопсією Thoracoscopy with biopsy

Комп'ютерна томографія Computed tomography

Бактеріологічне дослідження харкоти-ння Bacteriological study of sputum

Фібробронхоскопія Fibrobronchoscopy

493 / 1500
Жінка віком 40 років з 15 років хворіє на епілепсію з генералізованими епілептичними нападами, які виникають 2-3 рази на місяць у нічний час та супроводжуються мимовільним сечовиділенням та дефекацією. Після психотравми (смерть батька) напади почастішали, виникають з періодичністю 2-3 хвилини, між нападами до свідомості не повертається, реакція зіниць на світло відсутня. Виберіть найімовірніший діагноз. A 40-year-old woman has been suffering from epilepsy since the age of 15 with generalized epileptic seizures that occur 2-3 times a month at night and are accompanied by involuntary urination and defecation. After the psychotrauma (father's death), the attacks became more frequent, occur with a frequency of 2-3 minutes, there is no return to consciousness between the attacks, the reaction of the pupils to light is absent. Choose the most likely diagnosis.

Епілептичний психоз Epileptic psychosis

Збільшення кількості великих епінападів Increasing the number of large epiattacks

Істеричний напад Hysterical attack

Абсанс Absence

Епілептичний статус Status epilepticus

494 / 1500
Хлопець віком 18 років у крамниці купив салат з копченою рибою, грибами і майонезом. Через 6 годин після споживання салату відчув слабкість, що прогресувала, погіршення зору, «туман в очах», розлади ковтання. Був госпіталізований до лікарні. Під час обстеження встановлено: температура тіла - 36,0^oC, шкіра і слизові оболонки бліді, хлопець адинамічний, на запитання відповідає мляво, зіниці розширені, реакція на світло знижена, голос осиплий, слизова оболонка рота суха. Який вид харчового отруєння можна припустити у хлопця? An 18-year-old boy bought a salad with smoked fish, mushrooms and mayonnaise in a store. 6 hours after eating the salad, he experienced progressive weakness, blurred vision, 'fog in the eyes', swallowing disorders. He was hospitalized. During the examination, it was found that the body temperature was 36.0 °C, the skin and mucous membranes were pale, the boy answered sluggishly, the pupils were dilated, the reaction to light was hoarse. , the mucous membrane of the mouth is dry. What type of food poisoning can be assumed in the boy?

Ботулізм Botulism

Отруєння грибами Mushroom poisoning

Отруєння рибою Fish poisoning

Сальмонельоз Salmonellosis

Стафілококова інтоксикація Staphylococcal intoxication

495 / 1500
У дівчинки 11-ти років (в анамнезі часті ангіни) захворювання мало поступовий розвиток. З'явились дратівливість, неуважність, гіперкінези. При обстеженні виявлено порушення координації рухів, гіпотонію м'язів, гіперрефлексію. Для якого захворювання характерні такі зміни зі сторони центральної нервової системи? In an 11-year-old girl (with a history of frequent angina), the disease had a gradual development. Irritability, inattention, hyperkinesis appeared. During the examination, a violation of movement coordination was found, muscle hypotonia, hyperreflexia. What disease is characterized by such changes on the part of the central nervous system?

Вегето-судинна дисфункція Vegeto-vascular dysfunction

Системна склеродермія Systemic scleroderma

Системний червоний вовчак Systemic lupus erythematosus

Гостра ревматична лихоманка Acute rheumatic fever

Вузликовий поліартеріїт Polyarteritis nodosa

496 / 1500
Чоловік 58-ми років звернувся до лікаря зі скаргами на значне підсилення загальної слабкості за останні кілька тижнів, ціаноз, болі у животі, нудоту, періодичне блювання, запах ацетону з роту, гіперпігментацію шкірних складок. З анамнезу відомо, що пацієнт хворіє на туберкульоз. При фізикальному обстеженні артеріальний тиск - 80/40 мм рт.ст., пульс - 124/хв. При лабораторному дослідженні крові: натрій - 125 ммоль/л, хлориди - 74 ммоль/л, калій - 5,7 ммоль/л, глюкоза - 3,5 ммоль/л, сечовина - 14 ммоль/л. У загальному аналізі сечі: білок - 0,66 г/л, лейкоцити - 10-12 в п/з, еритроцити - 5-6 в п/з, циліндри - поодинокі. Який наступний крок у веденні пацієнта буде найбільш доречним? A 58-year-old man turned to the doctor with complaints of a significant increase in general weakness over the past few weeks, cyanosis, abdominal pain, nausea, periodic vomiting, the smell of acetone from the mouth, hyperpigmentation of the skin folds. From the anamnesis, it is known that the patient has tuberculosis. During the physical examination, the blood pressure is 80/40 mm Hg. During the laboratory examination, sodium is 125 mmol/l. chlorides - 74 mmol/l, potassium - 5.7 mmol/l, glucose - 3.5 mmol/l, urea - 14 mmol/l. In the general analysis of urine: protein - 0.66 g/l, leukocytes - 10 12 in p/z, erythrocytes - 5-6 in p/z, cylinders - single. What next step in the management of the patient will be most appropriate?

Визначення концентрації кортизолу в крові Determination of cortisol concentration in blood

Визначення концентрації кетонових тіл в сечі Determining the concentration of ketone bodies in urine

Визначення концентрації креатиніну в крові Determination of creatinine concentration in blood

Визначення концентрації діастази в сечі Determination of diastase concentration in urine

Визначення концентрації лужної фосфатази в крові Determining the concentration of alkaline phosphatase in the blood

497 / 1500
До лікаря-педіатра звернулася мати з дитиною 3,5 років. Дитина 4-у добу хворіє, лихоманка в межах 38,7-39,2^oС. Захворювання почалося гостро з сухого кашлю. На другу добу захворювання з'явилися: одутлість обличчя, світлобоязнь та гнійний двобічний кон'юнктивіт. На четверту добу на тлі температури тіла 39,4^oС виникли плямисто-папульозні висипання за вухами та на обличчі. Дихання через ніс утруднене, у нижніх носових ходах невеликі слизисті виділення. У сім'ї є молодший брат віком 7 місяців. Які заходи захисту для цієї дитини повинен рекомендувати сімейний лікар? A mother with a 3.5-year-old child consulted a pediatrician. The child has been ill for the 4th day, with a fever of 38.7-39.2°C. The disease began acutely with a dry cough. On the second day, swelling of the face, photophobia, and purulent conjunctivitis appeared on the background of a body temperature of 39.4°C it is difficult through the nose, there is a small mucous discharge in the lower nasal passages. The family has a younger brother aged 7 months. What protective measures should the family doctor recommend for this child?

Виконати дом'язеву ін'єкцію 1,5 мл протикоревого імуноглобуліну Perform an intramuscular injection of 1.5 ml of measles immunoglobulin

Провести хіміопрофілактику Conduct chemotherapy

Провести вакцинацію від кору, краснухи, паротиту (КПК) Vaccinate against measles, rubella, mumps (KPK)

Призначити вітамін А 100 000 МО Prescribe vitamin A 100,000 IU

Профілактики не потребує, оскільки дитина захищена трансплацентарно Prophylaxis is not required, as the child is transplacentally protected

498 / 1500
У 45-річного чоловіка, хворого на артеріальну гіпертензію (АТ - 150/100 мм рт. ст.) виявлено ішемічну хворобу серця, серцеву недостатність, цукровий діабет II типу. Який із наведених гіпотензивних засобів слід застосувати? A 45-year-old man with hypertension (BP - 150/100 mm Hg) was diagnosed with coronary heart disease, heart failure, and diabetes II type. Which of the following hypotensive agents should be used?

Верапаміл Verapamil

Атенолол Atenolol

Гідралазин Hydralazine

Лозартан Losartan

Гіпотіазид Hypotiazide

499 / 1500
Дівчинка віком 15 років скаржиться на затримку росту, відсутність менструацій і вторинних статевих ознак. Об'єктивно спостерігається: зріст - 153 см, антимонголоїдний розріз очей, широка шия, крилоподібні складки шиї, низька лінія росту волосся на шиї, плечовий пояс переважає над тазовим, молочні залози нерозвинені, відсутнє оволосіння на лобці. Виявлено гіпоплазію матки. Який найімовірніший діагноз? A 15-year-old girl complains of stunted growth, lack of menstruation and secondary sexual characteristics. Objectively observed: height - 153 cm, anti-Mongoloid cut of the eyes, wide neck, wing-like folds of the neck, low growth line of the neck, shoulder girdle prevails over the pelvic, mammary glands are absent, pubic hair is found. What is the most likely diagnosis?

Нейрофіброматоз Neurofibromatosis

Синдром Шерешевського-Тернера Shereshevsky-Turner syndrome

Нанізм Nanism

Синдром Клайнфельтера Klinefelter syndrome

Гіпогеніталізм Hypogenitalism

500 / 1500
В організм людини з атмосферного повітря надходять декілька хімічних речовин. Як називається тип комбінованої дії, де сукупний ефект перевищує суму ефектів кожної окремої речовини, що входить у комбінацію, в разі її ізольованої дії на організм? Several chemical substances enter the human body from atmospheric air. What is the type of combined action called, where the cumulative effect exceeds the sum of the effects of each individual substance included in the combination, in in case of its isolated effect on the body?

Потенціювання Potentiation

Ізольована дія Isolated Action

Комплексна дія Complex Action

Поєднана дія Combined Action

Антагонізм Antagonism

501 / 1500
Родина з трьох осіб поступила до стаціонару з приводу захворювання, яке пов'язане із вживанням рибної консерви домашнього виготовлення. У пацієнтів була така клініка: порушення зору, туман перед очима, неможливість читати дрібний шрифт, із приводу чого звернулись до окуліста. Який патогенез нервових порушень? A family of three was admitted to the hospital due to an illness associated with the consumption of home-made canned fish. The patients had the following clinic: visual impairment, fog before eyes, inability to read fine print, about which they consulted an oculist. What is the pathogenesis of nervous disorders?

Блокування виділення ацетилхоліну в синапсах Blocking of acetylcholine release in synapses

Підвищення виділення ацетилхоліну в синапсах Increased release of acetylcholine in synapses

Стимуляція синтезу аденілатциклази Stimulation of adenylate cyclase synthesis

Демієлінізація нервових волокон Demyelination of nerve fibers

Розвиток гіпокоагуляції Development of hypocoagulation

502 / 1500
Пацієнтка віком 29 років скаржиться на рясні кров'янисті виділення зі статевих шляхів. Відмічає затримку менструації протягом 3-х місяців. З анамнезу відомо: одні пологи та один медичний аборт. Під час піхвового дослідження виявлено: із шийки матки значні геморагічні виділення з пухирцями, тіло матки збільшене до 15-16 тижнів вагітності, м'якувате, неболюче. Обидва яєчники збільшені, рухомі, неболючі. Хоріонічний гонадотропін в крові - 200 000 ОД. Який найімовірніший діагноз? A 29-year-old patient complains of profuse bleeding from the genital tract. She notes a delay in menstruation for 3 months. From the anamnesis it is known: one childbirth and one medical During the vaginal examination, significant hemorrhagic discharge with blisters was found, the uterus was enlarged up to 15-16 weeks of pregnancy, both ovaries were enlarged, mobile, painless. What is the most likely diagnosis?

Вагітність 12 тижнів, мимовільний аборт в ходу Pregnancy 12 weeks, spontaneous abortion in progress

Шийкова вагітність Cervical pregnancy

Вагітність 12 тижнів, загрозливий аборт Pregnancy 12 weeks, threatened abortion

Міхуровий занесок Bubble Drift

Позаматкова вагітність Ectopic pregnancy

503 / 1500
Чоловік віком 42 роки скаржиться на слабкість, серцебиття, носові кровотечі, появу крововиливів на шкірі. Стан прогресивно погіршується протягом місяця. Об'єктивно спостерігається: стан тяжкий, на шкірі кінцівок і тулуба петехіальні та плямисті крововиливи, лімфовузли не пальпуються, пульс - 116/хв., печінка +2 см, селезінка не пальпується. В загальному аналізі крові виражена панцитопенія. Яке захворювання можна припустити в першу чергу? A 42-year-old man complains of weakness, palpitations, nosebleeds, the appearance of hemorrhages on the skin. The condition progressively worsens over the course of a month. Objectively observed: the condition is severe, on petechial and spotted hemorrhages on the skin of the limbs, lymph nodes are not palpable, the pulse is +2 cm, the spleen is not palpable. In the general blood test, pancytopenia can be assumed first of all?

Гострий агранулоцитоз Acute agranulocytosis

Гострий лейкоз Acute leukemia

Гіпопластична анемія Hypoplastic anemia

Хвороба Верльгофа Werlhof's disease

Геморагічний васкуліт Hemorrhagic vasculitis

504 / 1500
Робітниця прядильного цеху фабрики з виробництва льону зі стажем роботи 15 років скаржиться на слабкість, задишку, кашель, відчуття стислості у грудях, приступи утрудненого дихання. Зазвичай найбільш тяжкі приступи ядухи з'являються при поверненні на роботу у понеділок (<<симптом понеділка>>), тоді як у вихідні дні самопочуття поліпшується. Під час аускультації над легенями прослуховуються розсіяні сухі хрипи з обох сторін. У крові - незначний лейкоцитоз. Назвіть захворювання пацієнтки: A worker in the spinning shop of a flax factory with 15 years of experience complains of weakness, shortness of breath, cough, a feeling of tightness in the chest, attacks of difficulty breathing. Usually the most severe attacks wheezing appears when returning to work (<>), while on the weekend, the patient's health improves.

Антракоз Anthracosis

Азбестоз Asbestosis

Сидероз Siderosis

Силікоз Silicosis

Бісиноз Bisinosis

505 / 1500
У пацієнта віком 36 років, після вживання напередодні алкоголю та жирної їжі, вночі виник напад гострого артриту 1-го плесне-фалангового суглоба правої стопи. Під час огляду спостерігається: перший палець набряклий, шкіра синюшна, гаряча на дотик. У загальному аналізі крові виявлено: L - 12,0·10^12/л, зсув вліво, ШОЕ - 42 мм/год. Виберіть варіант стартової терапії. A 36-year-old patient, after consuming alcohol and fatty food the night before, developed an attack of acute arthritis of the 1st metatarsophalangeal joint of the right foot at night. During the examination, it is observed : the first finger is swollen, the skin is bluish, hot to the touch. The general blood test revealed: L - 12.0·10^12/l, shift to the left, ESR - 42 mm/h. Choose the starting therapy option.

Колхіцин Colchicine

Преднізолон Prednisone

НПЗП NSAIDs

Дипроспан внутрішньосуглобово Diprospan intra-articularly

Мазь з НПЗП місцево Ointment with NSAIDs locally

506 / 1500
Чоловік 45-ти років, доставлений у відділення невідкладної допомоги зі скаргами на раптовий інтенсивний біль у поперековій ділянці, часте болісне сечовиділення, блювоту. Температура тіла - 36,8^oC, пульс - 82/хв., артеріальний тиск - 130/80 мм рт.ст. При фізикальному обстеженні болісність при пальпації поперекової ділянки, позитивний симптом Пастернацького. При лабораторному дослідженні у загальному аналізі сечі білок - 0,06 г/л, реакція - слабко кисла, лейкоцити - 3-4 в полі зору, еритроцити - 30-40 в полі зору. Який діагноз є найбільш імовірним? A 45-year-old man was brought to the emergency department with complaints of sudden intense pain in the lumbar region, frequent painful urination, vomiting. Body temperature - 36.8 ^oC, pulse - 82/min., blood pressure - 130/80 mmHg. During physical examination, pain during palpation of the lumbar area, positive Pasternaksky's symptom. In the laboratory examination, protein - 0.06 g/l, reaction - weakly acidic, leukocytes - 3-4 in the field of vision, erythrocytes - 30-40 in the field of vision. What diagnosis is most likely?

Сечокам'яна хвороба Urolithiasis

Полікістоз нирок Polycystic kidney disease

Гостра ниркова недостатність Acute renal failure

Гострий пієлонефрит Acute pyelonephritis

Гострий гломерулонефрит Acute glomerulonephritis

507 / 1500
Чоловік 58-ми років скаржиться на появу пухлиноподібних утворень на передній поверхні шиї та в пахвинній ділянці, слабкість. Пальпуються м'які, безболісні, рухливі шийні та пахвинні лімфовузли до 2 см в діаметрі. Печінка виступає на 2 см з-під краю реберної дуги, нижній полюс селезінки на рівні пупка. У крові: еритроцити - 3,5·10^12/л, Hb - 88 г/л, лейкоцити - 86·10^9/л, п/я - 1%, с/я - 10%, л - 85%, е - 2%, б - 0%, м - 2%, швидкість осідання еритроцитів - 15 мм/год, тіні Боткіна-Гумпрехта. Який найбільш імовірний діагноз? A 58-year-old man complains of the appearance of tumor-like formations on the front surface of the neck and in the groin area, weakness. Soft, painless, mobile cervical and inguinal lymph nodes are palpated up to 2 cm in diameter. The liver protrudes 2 cm from the edge of the costal arch, the lower pole of the spleen is at the level of the navel. In the blood: erythrocytes - 3.5·10^12/l, Hb - 88 g/l. 10^9/l, p/y - 1%, s/y - 10%, l - 85%, e - 2%, b - 0%, m - 2%, erythrocyte sedimentation rate - 15 mm/h, Botkin-Gumprecht shadows. What is the most likely diagnosis?

Лімфогранулематоз Lymphogranulomatosis

Хронічний мієлолейкоз Chronic myelogenous leukemia

Гострий лейкоз Acute leukemia

Хронічний лімфолейкоз Chronic lymphocytic leukemia

Лейкемоїдна реакція лімфоцитарного типу Lymphocyte type leukemoid reaction

508 / 1500
Пацієнтку віком 42 роки шпиталізовано зі скаргами на тупий ниючий біль в попереку більше справа, інколи підвищення температури тіла до субфебрильної. З анамнезу відомо: 10 років тому під час вагітності був напад болю в правій половині попереку, підвищення температури тіла до 39^oС. Лікувалась антибіотиками. В останні роки почувала себе задовільно. Остані 5 років спостерігалося підвищення артеріального тиску. В аналізі сечі виявлено: білок - 0,66 г/л, лейкоцити - 10-15 в п/з, еритроцити - 2-3 в п/з. Який найімовірніший діагноз? A 42-year-old patient was hospitalized with complaints of a dull, aching pain in the lower back, more on the right side, sometimes with a rise in body temperature to subfebrile. It is known from the anamnesis: 10 years ago during pregnancy There was an attack of pain in the lower back, an increase in body temperature up to 39°C. In the last 5 years, an increase in blood pressure was observed: protein - 0.66 g/l. 10-15 in p/z, erythrocytes - 2-3 in p/z. What is the most likely diagnosis?

Туберкульоз нирок Kidney tuberculosis

Уролітіаз нирок Kidney urolithiasis

Хронічний гломерулонефрит Chronic glomerulonephritis

Гіпертонічна хвороба Hypertensive disease

Хронічний пієлонефрит Chronic pyelonephritis

509 / 1500
Десятимісячний хлопчик погано набирає вагу тіла, мати скаржиться на його постійний нав'язливий кашель. Мокротиння густе, в'язке. Тричі переніс пневмонію. Рівень хлоридів поту понад 80 мекв/л. Який найімовірніший діагноз? A ten-month-old boy is not gaining body weight well, his mother complains of his constant intrusive cough. The sputum is thick, viscous. He has had pneumonia three times. The sweat chloride level is over 80 meq/l. What is the most likely diagnosis?

Муковісцидоз Cystic Fibrosis

Природжена вада легень Congenital lung defect

Хронічний бронхіт Chronic bronchitis

Інородне тіло у бронхах Foreign body in bronchi

Бронхіальна астма Bronchial asthma

510 / 1500
Хворий 45-ти років має скарги на постійний головний біль, біль у серці, колінних суглобах. Три роки тому був укус кліща з кільцевою еритемою на шкірі. Об'єктивно: акродерматит, двобічна пірамідна недостатність, розлади координації. Який діагноз найбільш імовірний? A 45-year-old patient complains of constant headache, pain in the heart, and knee joints. Three years ago, there was a tick bite with annular erythema on the skin. About' objectively: acrodermatitis, bilateral pyramidal insufficiency, coordination disorders. What is the most likely diagnosis?

Розсіяний склероз Multiple sclerosis

Ішемічна енцефалопатія Ischemic encephalopathy

Нейросифіліс Neurosyphilis

Вірусний енцефаліт Viral encephalitis

Нейробореліоз Neuroborreliosis

511 / 1500
Пацієнт віком 38 років скаржиться на біль в попереку, лівих колінному та гомілковостопному суглобах. За результатами аналізу крові виявлено: лейкоцити - 11·10^9/л, ШОЕ - 38 мм/год, СРП - ++, Титр АСЛ-'О' - 125 од, сечова кислота - 375 мкмоль/л, ревматоїдний фактор - негативний. У зіскобі з уретри виявлено хламідії. Під час рентгенологічного дослідження виявлено: зліва суглобові поверхні - нерівні, нечіткі, суглобова щілина звужена. Який найімовірніший діагноз? A 38-year-old patient complains of pain in the lower back, left knee and ankle joints. According to the results of a blood test, leukocytes - 11·10^9/l, ESR - 38 mm/h, SRP - ++, Titer ASL-'O' - 125 units, uric acid - 375 μmol/l, chlamydia was detected in the scraping from the urethra - uneven, indistinct, the joint space is narrowed. What is the most likely diagnosis?

Остеохондроз хребта Osteochondrosis of the spine

Реактивний поліартрит Reactive polyarthritis

Ревматоїдний поліартрит Rheumatoid polyarthritis

Подагричний артрит Gouty arthritis

Анкілозуючий спондиліт Ankylosing spondylitis

512 / 1500
Хворий 60-ти років скаржиться на легкі порушення пам'яті, координації, ходи. Підтверджений церебральний атеросклероз, АТ в межах норми. МРТ: перивентрикулярно - лейкоареоз. Найбільш імовірний діагноз: A 60-year-old patient complains of mild impairments in memory, coordination, and gait. Cerebral atherosclerosis is confirmed, blood pressure is within the normal range. MRI: periventricular leukoaraiosis. The most probable diagnosis:

Хвороба Альцгеймера Alzheimer's disease

Хвороба Бінсвангера Binswanger's disease

Лакунарний інфаркт мозку Lacunar cerebral infarction

Транзиторні ішемічні атаки Transient ischemic attacks

Гіпоксично-ішемічна енцефалопатія Hypoxic-ischemic encephalopathy

513 / 1500
Хворий на гемофілію А 18-ти років звернувся із скаргами на гострий біль у лівому колінному суглобі та збільшення його в об'ємі. Суглоб різко болючий під час рухів та пальпації, шкіра над ним гаряча на дотик. Яке лікування слід застосувати? An 18-year-old hemophiliac A patient complained of sharp pain in the left knee joint and an increase in its volume. The joint is sharply painful during movements and palpation, the skin over it is hot to the touch. What treatment should be applied?

Накласти холод та іммобілізувати суглоб Apply cold and immobilize the joint

Провести лікувальну пункцію суглоба Perform medical puncture of the joint

В/в введення концентрату чинника IX IV administration of factor IX concentrate

В/в введення концентрату чинника VIII IV administration of factor VIII concentrate

В/в введення свіжозамороженої плазми IV administration of fresh frozen plasma

514 / 1500
Дитина, що народилась 1 день тому, проходить огляд у зв'язку з виникненням висипу. Її мати на облік у жіночу консультацію не ставала. Після народження дитина оцінена за шкалою Апгар у 7 та 9 балів на 1 та 5 хвилині відповідно. При фізикальному обстеженні температура - 37^oC, пульс - 145/хв., частота дихання - 33/хв. При аускультації серця вислуховується систоло-діастолічний шум у лівій підключичній ділянці. При огляді по всьому тілу розсіяні петехії. Який збудник найбільш імовірно викликав інфікування дитини? A child born 1 day ago is being examined due to the appearance of a rash. Her mother was not registered at the gynecological clinic. After birth, the child was assessed for Apgar score of 7 and 9 at 1 and 5 minutes, respectively. On physical examination, the temperature is 37°C, the heart rate is 145/min. On auscultation of the heart, a systolic-diastolic murmur is heard in the left subclavian area. On examination, petechiae are scattered all over the body. What pathogen most likely caused the child's infection?

Краснуха Krasnukha

Цитомегаловірус Cytomegalovirus

Токсоплазмоз Toxoplasmosis

Сифіліс Syphilis

Вірус простого герпесу Herpes simplex virus

515 / 1500
Чоловік 32-х років госпіталізований на 5-й день хвороби зi скаргами на пiдвищення температури до 38,4^oC, загальну слабкiсть, появу в лiвiй пахвовій дiлянцi щiльного утвору 5х6 см, малоболючого, рухомого. Шкiра над утвором не змiнена. Обличчя хворого гiперемоване, пастозне. Частота серцевих скорочень - 95/хв., артеріальний тиск - 100/80 мм рт.ст. В загальному аналiзi кровi - помiрна лейкопенiя. Працює ветсанітаром. Який дiагноз є найбільш імовірним? A 32-year-old man was hospitalized on the 5th day of illness with complaints of an increase in temperature to 38.4^oC, general weakness, the appearance of a dense mass in the left axillary region 5x6 cm. The skin over the lesion is not changed. The heart rate is 95/min. In the general blood analysis, he has moderate leukopenia What is the most likely diagnosis?

Ієрсиніоз Yersiniosis

Гiдраденiт Hydradenite

Доброякiсний лiмфоретикульоз Benign lymphoreticulosis

Чума Plague

Туляремiя Tularemia

516 / 1500
До сімейного лікаря звернулася жінка 35-ти років зі скаргами на рясні менструації, які тривають до 10 днів. При гінекологічному огляді виявлено: шийка матки не змінена, матка в anterflexio, звичайних розмірів, рухома, неболюча. Придатки з обох боків без особливостей. Сімейний лікар поставив попередній діагноз: аномальна маткова кровотеча. Який із вказаних інструментальних методів є першочерговим у діагностиці цієї патології? A 35-year-old woman came to the family doctor with complaints of heavy menstruation that lasts up to 10 days. During a gynecological examination, it was found that the cervix is ​​unchanged, the uterus in anterflexio, mobile, painless. The family doctor made a preliminary diagnosis: abnormal uterine bleeding. Which of the indicated instrumental methods is the first priority in the diagnosis of this pathology?

Лапароскопія Laparoscopy

УЗД трансабдомінальне Transabdominal ultrasound

УЗД трансвагінальне Transvaginal ultrasound

Кульдоскопія Culdoscopy

Кольпоскопія Colposcopy

517 / 1500
У жінки, яка хворіє на ІХС, щотижня виникають напади серцебиття, задишки, перебої у роботі серця. На ЕКГ під час нападу спостерігається: інтервали R-R мають різну тривалість, зубці P відсутні, хвилі f, зубці R різної амплітуди. На ЕхоКГ виявлено ФВ - 38%. Призначена схема лікування (аміодарон, варфарин). Який із наведених нижче показників у цьому разі потребує постійного моніторування? A woman suffering from coronary heart disease has attacks of palpitations, shortness of breath, heart failure every week. On the ECG during the attack, the following is observed: RR intervals have different durations, There are no P waves, R waves of varying amplitude. On echocardiography, PV is detected (amiodarone, warfarin). In this case, which of the following indicators needs constant monitoring?

Протромбіновий індекс Prothrombin index

Гемоглобін Hemoglobin

Рівень фібриногену Fibrinogen level

Рівень тромбоцитів Platelet level

Міжнародне нормалізоване співвідношення International Normalized Ratio

518 / 1500
Чоловік 48-ми років скаржиться на стискаючий біль за грудиною, що виник вперше 1,5 години тому після фізичного навантаження та не знімається нітрогліцерином. Об'єктивно: ЧСС- 75/хв., ЧД- 16/хв., АТ- 140/80 мм рт.ст. При ЕКГ дослідженні сегмент ST зміщений донизу від ізолінії на 1-2 мм у відведеннях V4-V6. Дослідження якого показника допоможе встановити діагноз у даному випадку? A 48-year-old man complains of squeezing pain behind the sternum, which first appeared 1.5 hours ago after physical exertion and is not relieved by nitroglycerin. Objectively: Heart rate - 75/min., BH- 16/min., BP- 140/80 mm Hg. During the ECG examination, the ST segment is shifted downwards from the isoline by 1-2 mm in the V4-V6 leads in this case?

АсАТ АСАТ

Натрій уретичного пептиду Uretic peptide sodium

Тропоніну Troponin

D-димеру D-dimer

КФК KFC

519 / 1500
Пацієнтка віком 14 років скаржиться на кров'янисті виділення з піхви протягом 10 днів, що з'явилися після 3-місячної затримки менструації. З анамнезу відомо: менархе - у 13 років, менструальний цикл - нерегулярний. У загальному аналізі крові спостерігається: гемоглобін - 90 г/л. Який найімовірніший діагноз? A 14-year-old patient complains of vaginal bleeding for 10 days, which appeared after a 3-month delay in menstruation. It is known from the anamnesis: menarche - at the age of 13, the menstrual cycle is irregular. The general blood test shows: hemoglobin - 90 g/l. What is the most likely diagnosis?

Поліп шийки матки Cervical polyp

Ювенільна маткова кровотеча Juvenile uterine bleeding

Гормонопродукуюча пухлина матки Hormone-producing uterine tumor

Аномалія розвитку матки Anomaly of uterine development

Хвороба Верльгофа Werlhof's disease

520 / 1500
До ортопеда звернулись батьки 3-мі-сячної дитини зі скаргами на асиметрію шкірних складок правого стегна. Під час огляду виявлено: різна кількість складок шкіри на правому та лівому стегні, їх асиметрія, обмеження розведення правої нижньої кінцівки зігнутої під кутом 90^o в кульшовому та колінному суглобах. Який діагноз є найбільш імовірним? Parents of a 3-month-old child turned to the orthopedist with complaints about the asymmetry of the skin folds of the right thigh. During the examination, it was found: a different number of skin folds on the right and left thigh , their asymmetry, limitation of extension of the right lower limb bent at an angle of 90° in the hip and knee joints. What diagnosis is most likely?

Вроджений вивих стегна Congenital hip dislocation

Ушкодження кульшового суглоба під час пологів Injury of the hip joint during childbirth

Дисплазія кульшового суглоба Hip dysplasia

- -

Контрактура кульшового суглоба Hip joint contracture

521 / 1500
У хлопця 18 років скарги на переймоподібний біль у животі, рідкі випорожнення до 6 разів на добу з домішками слизу та свіжої крові. Хворіє впродовж року. Схуд на 10 кг. Об'єктивно встановлено: живіт м'який, болючий по ходу товстої кишки, особливо зліва. Сигмовидна кишка спазмована. У крові виявлено: еритроцити - 3,2·10^12/л, Нb - 92 г/л, лейкоцити - 10,6·10^9/л, ШОЕ - 34 мм/год. Іригоскопія показала: товста кишка звужена, гаустри відсутні, контури нечіткі, симптом <<водопровідної труби>>. Який найбільш ймовірний діагноз? An 18-year-old boy complains of spasm-like abdominal pain, loose stools up to 6 times a day with impurities of mucus and fresh blood. He has been ill for a year. He lost 10 kg Objectively, the abdomen is soft, painful in the course of the colon, especially the left sigmoid colon. , 6·10^9/l, ESR - 34 mm/h. Irigoscopy showed: the colon is narrowed, the contours are unclear, the symptom of a 'water pipe'. What is the most likely diagnosis?

Неспецифічний виразковий коліт Nonspecific ulcerative colitis

Туберкульоз кишечника Intestinal tuberculosis

Хронічний ентероколіт Chronic enterocolitis

Амебна дизентерія Amoebic dysentery

Хвороба Крона Crohn's disease

522 / 1500
Робітник цвяхового цеху 35 років, стаж на цьому виробництві - 10 років, працює в умовах високочастотного інтенсивного шуму. На черговому періодичному огляді поставлено діагноз: <<професійна тугоухість>>. Що є підставою для такого діагнозу? A 35-year-old nail workshop worker, 10 years of experience in this industry, works in conditions of high-frequency intense noise. At the next periodic examination, the diagnosis was made: < >. What is the basis for such a diagnosis?

Характеристика шуму на цьому виробництві Noise characteristic of this production

Результати дослідження стану внутрішнього вуха Results of the study of the condition of the inner ear

Результати дослідження показників ЦНС Results of research of central nervous system indicators

Дані аудіометрії та гігієнічна оцінка умов праці Audiometry data and hygienic assessment of working conditions

Стаж роботи на цьому виробництві Work experience at this production

523 / 1500
Хлопчик 10-ти рокiв скаржиться на підвищення температури тіла до 38,5^oC, припухлість суглобів пальців рук та ніг, ранкову скутість, біль у шийному відділі хребта. Проведено обстеження. У кровi: еритроцити - 2,6·10^12/л, Hb- 85 г/л, лейкоцити - 16,5·10^9/л, е.- 2%, п.- 8%, с.- 68%, л.- 16%, м.- 6%, швидкiсть осiдання еритроцитів - 28 мм/год. Позитивний ревматоїдний фактор. Диспротеїнемія з гіперглобуліне-мією. Встановіть діагноз: A 10-year-old boy complains of an increase in body temperature up to 38.5°C, swelling of the joints of fingers and toes, morning stiffness, pain in the cervical spine. An examination was carried out in the blood: erythrocytes - 2.6·10^12/l, Hb - 85 g/l, leukocytes - 16.5·10^9/l, e. - 2%, p. - 8%, .- 68%, l.- 6%, erythrocyte sedimentation rate - 28 mm/h. Dysproteinemia with hyperglobulinemia.

Остеоартрит Osteoarthritis

Токсичний синовiїт Toxic synovitis

Реактивний артрит Reactive arthritis

Ювенiльний ревматоїдний артрит Juvenile rheumatoid arthritis

Гостра ревматична лихоманка Acute rheumatic fever

524 / 1500
Жінка 65-ти років, вранці після сну відмітила затерплість в правих кінцівках, запаморочення, гикавку, зміну голосу, похлинання при вживанні рідини, затерплість та біль у лівій половині обличчя, хиткість ходи. Останні показники артеріального тиску - 160/100 мм рт.ст. У анамнезі гіпертонічна хвороба (систематично не лікується). При фізикальному обстеженні горизонтальний середньоамплітудний ністагм, більше виражений вліво, синдром Горнера ліворуч, гіпестезія лівої половини обличчя за цибулинним типом, правобічна гемігіпестезія, атаксія в позі Ромберга. Мова змінена - нечітка артикуляція, проте зміст і розуміння мови не порушені. Пацієнтці важко висунути язик з рота. Парезів кінцівок немає. Який метод буде найбільш доречним у підтверджені діагнозу цієї пацієнтки? A 65-year-old woman, in the morning after sleep noted numbness in the right limbs, dizziness, hiccups, change in voice, swallowing when drinking liquid, numbness and pain in the left half face, unsteadiness of gait. Recent blood pressure readings - 160/100 mm Hg. History of hypertension (not systematically treated). On physical examination, horizontal nystagmus, more pronounced on the left side, hypoesthesia of the left half of the face. , right-sided hemiesthesia, ataxia in Romberg's position. The language is unclear, but the meaning and understanding of speech is not disturbed. There are no paresis of the limbs. Which method would be most appropriate in this patient's diagnosis?

Реоенцефалографія Rheoencephalography

Комп'ютерна томографія (КТ) головного мозку Computed tomography (CT) of the brain

Електроенцефалографія Electroencephalography

Ангіографія Angiography

Магнітно-резонансна томографія (МРТ) головного мозку Magnetic resonance imaging (MRI) of the brain

525 / 1500
У регулювальника дорожнього руху з'явились скарги на головний біль, запаморочення, порушення сну, зниження пам'яті та уваги, задишку, біль у ділянці серця. У крові під час лабораторного обстеження виявлено підвищений вміст карбоксигемоглобіну. Який чинник навколишнього середовища найімовірніше міг спричинити такі зміни в організмі? The traffic controller complained of headache, dizziness, sleep disturbance, decreased memory and attention, shortness of breath, pain in the heart area. In the blood During the laboratory examination, an increased content of carboxyhemoglobin was detected. What environmental factor could most likely cause such changes in the body?

Оксид вуглецю Carbon monoxide

Оксид азоту Nitrogen oxide

3,4-безпірен 3,4-bezpyrene

Діоксид азоту Nitrogen dioxide

Діоксид вуглецю Carbon dioxide

526 / 1500
Тривале вживання питної води з високою жорсткістю (20 мг-екв/дм^3) негативно впливає на стан здоров'я населення. Яка з наведених нижче патологій не пов'язана з жорсткістю питної води? Long-term use of drinking water with high hardness (20 mg-eq/dm^3) negatively affects the health of the population. Which of the following pathologies is not 'related to the hardness of drinking water?

Жовчнокам'яна хвороба Cholelithiasis

Диспепсичні явища Dyspeptic symptoms

Дерматит Dermatitis

Сечокам'яна хвороба Urolithiasis

Кальциноз Calcinosis

527 / 1500
Хворий 38-ми років доставлений машиною швидкої допомоги до хірургічного відділення зі скаргами на загальну слабкість, нездужання, чорний кал. Під час огляду хворий блідий, спостерігаються крапкові крововиливи на шкірі тулуба та кінцівок. При пальцевому дослідженні на рукавичці чорний кал. У крові: Hb - 108 г/л, тромбоцитопенія. В анамнезі: подібний стан спостерігався 1 рік назад. Встановіть діагноз: A 38-year-old patient was taken by ambulance to the surgical department with complaints of general weakness, malaise, black stool. During the examination, the patient is pale, spotty hemorrhages are observed on on the skin of the trunk and limbs. Black stool in the blood: Hb - 108 g/l. In the anamnesis: a similar condition was observed 1 year ago;

Тромбоцитопенічна пурпура Thrombocytopenic purpura

Виразкова кровотеча Ulcer bleeding

Гемофілія Hemophilia

Неспецифічний виразковий коліт Nonspecific ulcerative colitis

Пухлина прямої кишки Rectal tumor

528 / 1500
Доношений новонароджений вагою 3900 г. За даними УЗД: вроджена вада серця, коарктація аорти без гіпоплазії дуги аорти, м'язовий дефект міжшлуночкової перегородки діаметром 2 мм. Яка тактика лікування? A full-term newborn weighing 3900 g. According to ultrasound: congenital heart disease, coarctation of the aorta without hypoplasia of the aortic arch, muscular defect of the interventricular septum with a diameter of 2 mm. What are the tactics treatment?

Пластика дефекту міжшлуночкової перегородки та резекція коарктації аорти Plastic of the defect of the interventricular septum and resection of coarctation of the aorta

Резекція коарктації аорти Resection of coarctation of the aorta

Пластика дефекту міжшлуночкової перегородки Ventricular septal defect plastic

Ушивання дефекту міжшлуночкової перегородки Suture of ventricular septal defect

Пластика дуги аорти Aortic arch plastic

529 / 1500
У пацієнта віком 78 років з хронічною венозною недостатністю раптово з'явилася задишка, <<кинджальний>> біль в грудній клітині, сухий кашель, серцебиття, температура тіла - 37,1^oС, короткочасна непритомність. Під час обстеження виявлено: стан важкий, дифузний ціаноз, розширення яремних вен, ЧД - 35/хв, везикулярне дихання, акцент ІІ тону над легеневою артерією, ЧСС - 130/хв, АТ - 80/60 мм рт. ст., сатурація знижена. Рівень D-димеру високий. ЕКГ: синусова тахікардія, Р-pulmonale, негативний Т в ІІІ та в V1-V2. Ro-ОГК: високе стояння куполу діафрагми ліворуч. Який стан ймовірніше розвився у пацієнта? A 78-year-old patient with chronic venous insufficiency suddenly developed shortness of breath, <> pain in the chest, dry cough, palpitations, body temperature - 37.1°C, short-term unconsciousness. During the examination, it was found: severe condition, diffuse cyanosis, expansion of the jugular veins, BH - 35/min, vesicular breathing, emphasis on the II tone over the pulmonary artery, heart rate - 130/min, BP - 80/min 60 mm Hg, the level of D-dimer is high, P-pulmonale T in V1-V2: high position of the diaphragm on the left the patient?

Гострий коронарний синдром Acute coronary syndrome

Гостра дихальна недостатність Acute respiratory failure

Пневмоторакс Pneumothorax

Тромбоемболія легеневої артерії Thromboembolism of the pulmonary artery

Напад бронхіальної астми Bronchial asthma attack

530 / 1500
Хворій 48 років. Після загибелі чоловіка місяць тому настало безсоння, погіршився настрій, з'явилися неспокій, страх, думки про самогубство. У лікарні: розмовляє тихим голосом, настрій пригнічений, тривожна, не спить, відмовляє-ться від прийому їжі. Препарати якої групи доцільніше призначити? The patient is 48 years old. After the death of her husband a month ago, she developed insomnia, her mood worsened, restlessness, fear, and thoughts of suicide appeared. In the hospital: she speaks in a low voice, the mood is depressed, anxious, does not sleep, refuses to eat. What group of drugs is more appropriate to prescribe?

Антиконвульсанти Anticonvulsants

Ноотропи Nootropics

Нейролептики Neuroleptics

Антидепресанти Antidepressants

Вітаміни групи В B vitamins

531 / 1500
Мати хлоптичка 3 років під час купання дитини виявила пухлиноподібний утвір у животі. Під час обстеження визначається щільний, неболючий, із рівною поверхнею, малорухомий пухлиноподібний утвір великих розмірів, розташований під лівою реберною дугою. Температура тіла - 37,5^oС, ЧСС - 110/хв., АТ - 150/90 мм рт. ст. У загальному аналізі крові виявлено: еритроцити - 3,2·10^12/л, Hb - 90 г/л, лейкоцити - 10,4·10^9/л, ШОЕ - 35 мм/год. У загальному аналізі сечі виявлено: еритроцити на все поле зору. Яке захворювання можна запідозрити у дитини? The mother of a 3-year-old boy discovered a tumor-like mass in the abdomen while the child was bathing. During the examination, a dense, painless, flat surface, slow-moving tumor-like mass of large sizes, located under the left costal arch. Body temperature - 37.5°C, heart rate - 150/90 mmHg. In the general blood analysis: erythrocytes - 3.2·10^12/l - 90 g/l, leukocytes - 10.4·10^9/l, ESR - 35 mm/h. In the general analysis of urine, erythrocytes are found in the entire field of view. What disease can be suspected in the child?

Пухлина Вільямса Williams Tumor

Гострий піелонефрит Acute pyelonephritis

Вроджений гідронефроз Congenital hydronephrosis

Крижово-куприкова тератома Sacral-coccygeal teratoma

Карбункул нирки Kidney carbuncle

532 / 1500
Чоловік підібраний на вулиці в непритомному стані та доставлений до санпропускника. У нього спостерігаються повторні напади тонічних і клонічних судом рук і ніг з невеликими проміжками, мимовільне сечовипускання. Зіниці широкі, не реагують на світло. На язиці - сліди прикушування. Вогнищева неврологічна симптоматика не виявлена. АТ - 140/90 мм рт. ст. Голова хворого може бути вільно приведена до грудей. Визначіть найбільш імовірну патологію: A man was picked up on the street in an unconscious state and taken to the hospital. He has repeated attacks of tonic and clonic convulsions of the arms and legs with small intervals, involuntary urination. Pupils are wide , do not react to light. No focal neurological symptoms are detected. The patient's head can be brought to the chest:

Паренхіматозний крововилив Parenchymal hemorrhage

Епілептичний статус Status epilepticus

Інфаркт мозку Cerebral infarction

Гострий менінгоенцефаліт Acute meningoencephalitis

Правець Tetanus

533 / 1500
У жінки віком 25 років з'явився висип на шкірі обличчя, шиї, біль у суглобах. Об'єктивно спостерігається: стан важкий, еритема обличчя у вигляді <<метелика>>, температура тіла - 38,7^oС, суглоби кистей та променево-зап'ясткові суглоби набряклі. Тони серця приглушені, систолічний шум на верхівці серця, ЧСС = 102/хв. У нижніх відділах легень вислуховується шум тертя плеври. У загальному аналізі сечі виявлено білок - 0,36 г/л. Визначення рівня якого показника найінформативніше для верифікації діагнозу? A 25-year-old woman developed a rash on the skin of the face, neck, and joint pain. Objectively observed: the condition is severe, facial erythema in the form of << butterfly>>, body temperature - 38.7°C, wrist and carpal joints are swollen, systolic murmur at the top of the heart, heart rate = 102/min the general analysis of urine revealed protein - 0.36 g/l. Determination of the level of which indicator is the most informative for the verification of the diagnosis?

Загального білка та білкових фракцій крові Total protein and protein fractions of blood

Aнтитіл до двоспіральної ДНК Antibody to double-helical DNA

Рівні імуноглобулінів A, M, G, E Levels of immunoglobulins A, M, G, E

С-реактивного протеїна C-reactive protein

Рівні циркулюючих імунних комплексів Levels of circulating immune complexes

534 / 1500
Хлопчик 7 років перебуває на лікуванні протягом місяця. Під час госпіталізації спостерігалися виражені набряки, білок у добовій сечі - 4,2 г. У біохімічному аналізі крові утримується гіпопротеїнемія (43,2 г/л), гіперхолестеринемія (9,2 ммоль/л). Установіть провідний синдром гломерулонефриту, який, найвірогідніше, має місце у пацієнта? A 7-year-old boy has been receiving treatment for a month. During hospitalization, severe edema was observed, protein in the daily urine was 4.2 g. In the biochemical blood analysis, hypoproteinemia was observed ( 43.2 g/l), hypercholesterolemia (9.2 mmol/l). Establish the leading syndrome of glomerulonephritis, which is most likely to occur in the patient?

Змішаний Mixed

Нефритичний Nephritic

Гематуричний Hematuric

Ізольований сечовий Isolated urinary

Нефротичний Nephrotic

535 / 1500
Хлопчик 10 років, який спостерігає-ться гематологом із приводу гемофілії, під час фізичних вправ пошкодив правий колінний суглоб. Під час огляду за годину після травми суглоб значно збільшений у розмірах, симптом балотування надколінка позитивний. Яку початкову тактику лікування слід обрати? A 10-year-old boy, who is being observed by a hematologist for hemophilia, injured his right knee joint during exercise. During the examination one hour after the injury, the joint is significantly enlarged in sizes, the patellar prolapse symptom is positive. What initial treatment tactics should be chosen?

Фізіотерапевтичне лікування Physiotherapy treatment

Спиртово-фураціліновий компрес Alcohol-furacilin compress

Артротомія Arthrotomy

Пункція суглоба Joint puncture

Гемостатична терапія, іммобілізація Hemostatic therapy, immobilization

536 / 1500
Жінка скаржиться на набряк нижнiх кiнцiвок, ламкiсть нігтiв, сухiсть шкiри, зниження пам'ятi, слабкiсть. Мiсячнi вiдсутнi з моменту пологiв. Під час зовнiшнього огляду встановлено: гiпотрофiя статевих органiв та молочних залоз. З анамнезу вiдомо, що 2 роки тому пiд час пологiв виникла масивна кровотеча понад 2000 мл. Який імовірний дiагноз? A woman complains of swelling of the lower extremities, brittle nails, dry skin, memory loss, weakness. Menstruation has been absent since childbirth. During the external examination, it was found: hypotrophy genitals and mammary glands. It is known from the anamnesis that 2 years ago during childbirth there was a massive bleeding of more than 2000 ml. What is the probable diagnosis?

Синдром Шихана Sheehan Syndrome

Пiсляпологове ожирiння Postpartum obesity

Iнфантилiзм Infantilism

Гiпотiреоз Hypothyroidism

Серцево-судинна недостатнiсть Cardiovascular failure

537 / 1500
Пацієнтка віком 30 років скаржиться на дискомфорт у животі зліва, біль в суглобах, лихоманку, періодичні крововиливи. Об'єктивно спостерігається: гепатолієнальний синдром. У загальному аналізі крові виявлено: лейкоцити - 200·10^9/л, велика кількість гранулоцитів різної ступені зрілості, мієлобласти - < 5% у кістковому мозку, Рh-хромосома позитивна. Який найімовірніший діагноз? A 30-year-old patient complains of abdominal discomfort on the left side, pain in the joints, fever, periodic hemorrhages. Objectively observed: hepato-ileal syndrome. A general blood test revealed : leukocytes - 200·10^9/l, a large number of granulocytes of various degrees of maturity, myeloblasts - < 5% in the bone marrow, the Rh-chromosome is positive. What is the most likely diagnosis?

Хронічний мієлолейкоз Chronic myelogenous leukemia

Злоякісна пухлина Malignant tumor

Мієлофіброз Myelofibrosis

Лейкемоїдна реакція Leukemoid reaction

Гострий лейкоз Acute leukemia

538 / 1500
Чоловік 28 років, скарги на біль у лівій нижній кінцівці протягом 6 місяців, переміжну кульгавість через 50-100 м, періодично нічні болі. Під час огляду кінцівка бліда, холодна. Артеріальна пульсація відсутня на гомільці. Який найімовірніший діагноз? 28-year-old man, complaints of pain in the left lower limb for 6 months, intermittent lameness after 50-100 m, occasional night pains. During examination, the limb is pale, cold. Arterial pulsation is absent in the lower leg. What is the most likely diagnosis?

Синдром Рейно Raynaud's syndrome

Емболія стегнової артерії Femoral embolism

Облітеруючий атеросклероз Obliterating atherosclerosis

Облітеруючий ендартеріїт Endarteritis obliterans

Хвороба Токоясу Tokoyasu disease

539 / 1500
Під час огляду новонародженої дівчинки виявлено крововилив на голові, який не виходить за межі однієї кістки, не пульсує, не болить. Який стан розвинувся у дитини? During the examination of a newborn girl, a hemorrhage was found on the head, which does not go beyond one bone, does not pulsate, does not hurt. What condition has developed in the child?

Внутрішньочерепна пологова травма Intracranial birth injury

Пухирчатка новонародженого Neonatal pemphigus

Водянка мозку Hydrosis

Пологова пухлина Obstetric tumor

Кефалогематома Cephalohematoma

540 / 1500
Чоловік 52-х років надійшов до відділення невідкладної допомоги зі скаргами на раптову задишку та прискорене серцебиття. Протягом місяця лікувався у стаціонарі з приводу септичного шоку, що розвинувся після інфекції сечовивідних шляхів. При фізикальному обстеженні артеріальний тиск - 100/60 мм рт.ст., пульс - 128/хв., частота дихання - 28/хв., SpO_2 - 89%. При лабораторному дослідженні у сироватці крові креа-тинін - 96 мкмоль/л, тропонін І - 1,0 нг/мл (норма <0,1 нг/мл). На рентгенограмі органів грудної порожнини незначний гідроторакс праворуч та субсегментарний ателектаз. На ЕКГ: синусова тахікардія, блокада правої ніжки пучка Гіса та інверсія зубців Т у відведеннях III та V1. Який перший крок у веденні пацієнта буде найбільш доречним? A 52-year-old man presented to the emergency department with complaints of sudden shortness of breath and palpitations. He had been hospitalized for a month for septic shock that developed after an infection During physical examination, blood pressure - 100/60 mm Hg, pulse rate - 28/min, SpO_2 - 89%. In laboratory examination, creatinine - 96 μmol /l, troponin I - 1.0 ng/ml (norm <0.1 ng/ml). On the X-ray of the chest organs, there is slight hydrothorax and subsegmental atelectasis. On the ECG: sinus tachycardia, right bundle branch block and inversion of T waves in leads III and V1. What would be the most appropriate first step in the management of the patient?

Ехокардіографія Echocardiography

Призначення аспірину та внутрішньовенне введення метопрололу Prescription of aspirin and intravenous metoprolol

Невідкладна катетеризація серця Urgent cardiac catheterization

Компресійна ультрасонографія вен Compression ultrasonography of veins

КТ-ангіографія CT-angiography

541 / 1500
У жінки 20-ти років головний біль, запаморочення, плаксивість, блювання, біль в ділянці серця, тахікардія, яка наростає за 6-7 днів до менструації. У перші дні менструації симптоми зникають. Поставте діагноз: A 20-year-old woman has a headache, dizziness, tearfulness, vomiting, heart pain, tachycardia, which increases 6-7 days before menstruation. the first days of menstruation, the symptoms disappear. Make a diagnosis:

Діенцефальний синдром Diencephalic syndrome

Альгоменорея Algomenorrhea

Метаболічна краніопатія Metabolic craniopathy

Синдром Штейна-Левенталя Stein-Leventhal syndrome

Передменструальний синдром Premenstrual syndrome

542 / 1500
Чоловік 64-х років знаходиться на амбулаторному лікуванні з приводу ішемічної хвороби серця, дифузного кардіосклерозу, постійної форми фібриляції передсердь, серцевої недостатності ІII функціонального класу. Фармакотерапія складається з кордарону 2 рази на день, торасеміду через день, триметазидину 2 рази на день. Сімейний лікар з метою профілактики тромбоутворення рекомендував хворому прийом варфарину (3 мг на день). Який метод контролю ефективності та безпеки антикоагулянтної терапії в даному випадку буде найбільш доречним? A 64-year-old man is undergoing outpatient treatment for ischemic heart disease, diffuse cardiosclerosis, persistent atrial fibrillation, functional class III heart failure. Pharmacotherapy consists of cordarone 2 times a day, torasemide every other day, trimetazidine 2 times a day. In order to prevent thrombosis, the patient was advised to take warfarin (3 mg a day). What method of monitoring the effectiveness and safety of anticoagulant therapy would be most appropriate in this case?

Міжнародне нормалізоване співвідношення International Normalized Ratio

Час згортання крові за Лі-Уайтом Lee-White blood clotting time

Швидкість зсідання еритроцитів Red blood cell sedimentation rate

Рівень фібриногену Fibrinogen level

Кількість тромбоцитів Number of platelets

543 / 1500
Жінка 50-ти років, яка страждає на системний червоний вовчак, отримує в якості базисної терапії метотрексат 10 мг на тиждень та метипред у дозі 8 мг на добу впродовж останнього року. Не вакцинована з підліткового віку, бажає отримати щеплення від дифтерії. Яка тактика імунопрофілактики у даному випадку? A 50-year-old woman with systemic lupus erythematosus receives as basic therapy methotrexate 10 mg per week and metipred 8 mg per day during the last year. Not vaccinated since adolescence, wants to get vaccinated against diphtheria. What are the tactics of immunoprophylaxis in this case?

При відсутності антитіл щеплення після відміни імуносупресивної терапії In the absence of vaccination antibodies after withdrawal of immunosuppressive therapy

Щеплення за стандартною схемою Vaccination according to the standard scheme

При відсутності антитіл щеплення за стандартною схемою In the absence of antibodies, vaccination according to the standard scheme

Відмінити імуносупресивну терапію та провести щеплення Cancel immunosuppressive therapy and carry out vaccination

Пасивна імунопрофілактика протидифтерійним імуноглобуліном Passive immunoprophylaxis with diphtheria immunoglobulin

544 / 1500
Дівчина 22-х років прийшла у жовтні до свого сімейного лікаря на профілактичний огляд. Самопочуття задовільне, скарг не має. Протягом 3-х років хворіє на добре контрольований цукровий діабет I типу. Її 70-річна бабуся хворіє на рак грудей. Останній ПАП-тест було зроблено рік тому, відхилень не зафіксовано. Її зріст - 172 см, вага - 61 кг, ІМТ - 20 кг/м^2. Показник глікованого гемоглобіну A1c складає 6,5%, останній показник глюкометра - 6,54 ммоль/л. Яка рекомендація лікаря цій пацієн-тці буде найбільш доречною? A 22-year-old girl came to her family doctor for a preventive check-up in October. She is feeling well, has no complaints. She has been suffering from well-controlled diabetes for 3 years Type I diabetes. Her 70-year-old grandmother has breast cancer. The last Pap test was done a year ago. Her height is 172 cm, and her BMI is 20 kg/m2 A1c is 6.5%, the last reading of the glucometer is 6.54 mmol/l. Which recommendation of the doctor would be most appropriate for this patient?

Пройти тест на вірус папіломи людини (ВПЛ-тест) Take a test for the human papilloma virus (HPV test)

Вакцинуватися від грипу Get vaccinated against the flu

Модифікація дієти з метою зниження ваги Diet modification for weight loss

Легкі аеробні вправи щодня Light aerobic exercise every day

Пройти мамографію Get a mammogram

545 / 1500
Пацієнт віком 48 років скаржиться на частий стискальний біль за грудиною з іррадіацією в ліве плече і ліву лопатку. Напади виникають вночі, у повному спокої, тривають 10-15 хв. Змін загального стану та порушень на ЕКГ під час звертання до поліклініки не виявлено. Яке обстеження є найнеобхіднішим для уточнення діагнозу? A 48-year-old patient complains of frequent squeezing pain behind the sternum with radiation to the left shoulder and left scapula. The attacks occur at night, at complete rest, and last 10-15 minutes No changes in the general condition and abnormalities were detected during the visit to the clinic. What examination is most necessary to clarify the diagnosis?

Холтерівське моніторування ЕКГ Holter ECG monitoring

Повторна ЕКГ через тиждень Repeat ECG in a week

Велоергометрія Cycle ergometry

Ехокардіографія Echocardiography

Катетеризація порожнин серця Catheterization of heart cavities

546 / 1500
У дитини 3 років під час гри в манежі раптово з'явилися нападоподібний кашель і утруднене дихання. Об'єктивно спостерігаються сухий кашель, задишка змішаного характеру. У легенях аускультативно виявлено невелика кількість сухих хрипів. Справа дихання ослабл-ене. Дитячий колектив не відвідує. Щеплена за віком. Який патологічний стан можна припустити? A 3-year-old child suddenly developed a paroxysmal cough and difficulty breathing while playing in the playpen. Objectively, a dry cough and dyspnea of ​​a mixed nature are observed. The lungs are auscultated A small amount of wheezing is detected. The child does not attend. What pathological condition can be assumed?

Гостра респіраторна вірусна інфекція Acute respiratory viral infection

Стороннє тіло дихальних шляхів Foreign body of respiratory tract

Коклюш Whooping cough

Пневмонія Pneumonia

Бронхіальна астма Bronchial asthma

547 / 1500
Лікар при обстеженні хворого встановив ураження очей (гемералопія, синдром Біто), шкіри та її придатків, слизових оболонок та травного тракту. Попередній діагноз: хвороба Прасада. Чим зумовлений розвиток вказаної патології? During the patient's examination, the doctor found damage to the eyes (hemeralopia, Bito's syndrome), skin and its appendages, mucous membranes, and the digestive tract. Preliminary diagnosis: Prasad's disease. What is the cause the development of the specified pathology?

Дефіцитом ванадію Vanadium deficiency

Дефіцитом марганцю Manganese deficiency

Дефіцитом цинку Zinc deficiency

Дефіцитом міді Copper deficiency

Дефіцитом заліза Iron deficiency

548 / 1500
Пацієнт віком 38 рокiв працює на будівництві відбійним молотком протягом 8 рокiв. Скаржиться на рiзкий бiль у плечовому поясi, особливо в нiчний час, оніміння пальцiв рук у разі охолодження. Об'єктивно встановлено: кистi набряклi, холоднi, цiанотичнi, рiзко зниженi всi види чутливостi (больова, температурна, вiбрацiйна). Під час проведення проби на холод - симптом <<мертвих пальцiв>>. Вiдзначається слабкiсть у привiдних м'язах V пальця, змiнена електрозбудженiсть м’язів кистей рук. Сухожилковi та перiостальнi рефлекси наявні. Який попередній дiагноз? A 38-year-old patient has been working in construction with a jackhammer for 8 years. He complains of sharp pain in the shoulder girdle, especially at night, and numbness of the fingers when cold. Objectively established: the hands are swollen, cold, cyanotic, all types of sensitivity (pain, temperature, vibration) are sharply reduced. During the cold test, the symptom of 'dead fingers' is noted. altered electrical excitability of the hand muscles. Tendon and periosteal reflexes are present. What is the previous diagnosis?

Вiбрацiйна хвороба локальної дiї Vibration disease of local effect

Вібраційна полiнейропатiя Vibration polyneuropathy

Синдром Рейно Raynaud's syndrome

Вiбрацiйна хвороба загальної дiї Vibration disease of general effect

Вузликовий периартерiїт Nodular periarteritis

549 / 1500
Жінка 28 років скаржиться на слабкість, запаморочення, носові кровотечі, крововиливи на тулубі. Хворіє 4 місяці. Об'єктивно спостерігається: стан середньої важкості. У ділянках живота та спини наявні крововиливи розміром 1-2 см, різного кольору, неболючі. Периферійні лімфовузли не збільшені. Печінка (-), селезінка (+). Аналіз крові виявив: Нb - 120 г/л, еритроцити - 3,4·10^12/л, КП - 0,9, ретикулоцити - 0,9%, залізо сироватки - 15,01 мкмоль/л, лейкоцити - 4,2·10^9/л, еозинофіли - 2%, базофіли - 0%, паличкоядерні - 7%, сегментоядерні - 40%, моноцити - 6%, лімфоцити - 45%, тромбоцити - 47,1·10^9/л, ШОЕ - 27 мм/год. Який діагноз імовірний? A 28-year-old woman complains of weakness, dizziness, nosebleeds, hemorrhages on the body. She has been ill for 4 months. Objectively observed: a condition of moderate severity. In the areas of the abdomen and Hemorrhages of different colors are present on the back. Peripheral lymph nodes are not enlarged. Blood analysis revealed: Hb - 120 g/l, erythrocytes - 3.4·10^12 l, KP - 0.9, reticulocytes - 0.9%, serum iron - 15.01 μmol/l, leukocytes - 4.2·10^9/l, eosinophils - 2%, basophils - 0%, rods - 7 %, segmentonuclear cells - 40%, monocytes - 6%, lymphocytes - 45%, platelets - 47.1·10^9/l, ESR - 27 mm/h. What is the probable diagnosis?

Ідіопатична тромбоцитопенічна пурпура Idiopathic thrombocytopenic purpura

Хронічна залізодефіцитна анемія Chronic iron deficiency anemia

Хронічний лімфолейкоз Chronic lymphocytic leukemia

Гіпопластична анемія Hypoplastic anemia

Гемолітична анемія Hemolytic anemia

550 / 1500
Приблизно у 40 % пацієнтів з бактеріальними пневмоніями розвивається супутній плевральний випіт. Діагноз підтверджується рентгенографією органів грудної клітки в прямій проекції вертикально за наявності не менш ніж: Approximately 40% of patients with bacterial pneumonia develop a concomitant pleural effusion. The diagnosis is confirmed by X-ray of the chest organs in a direct vertical projection with the presence of at least:

200 мл рідини 200 ml of liquid

300 мл рідини 300 ml of liquid

500 мл рідини 500 ml of liquid

100 мл рідини 100 ml of liquid

- -

551 / 1500
Юнак 17 років вимагає зробити йому пластичну операцію. Вважає, що з таким носом, як у нього, неможливо жити, де б він не з'явився, усі з нього сміються, кепкують за його спиною. Ходить, низько насунувши кашкета, низько опускає голову, до очей замотується шарфом. Об'єктивних підстав для ринопластики немає, ніс у хлопця майже класичної форми. Визначте психопатологічний стан: A 17-year-old boy demands plastic surgery. He believes that it is impossible to live with a nose like his, no matter where it appears, everyone with they laugh at him behind his back. He walks with his cap down, wraps his head with a scarf. There are no objective reasons for rhinoplasty. Define a psychopathological condition:

Синдром Капгра Capgra syndrome

Дисморфоманія Dysmorphomania

Порушення схеми тіла Body schema violation

Іпохондричний невроз Hypochondriac neurosis

Нав'язливі думки Intrusive thoughts

552 / 1500
Для пацієнта віком 64 роки з пухлиною сигмоподібної кишки та хронічним тромбофлебітом глибоких вен правої нижньої кінцівки планується оперативне втручання. Виберіть оптимальний препарат для профілактики тромбозу глибоких вен у цього пацієнта. A 64-year-old patient with a tumor of the sigmoid colon and chronic thrombophlebitis of the deep veins of the right lower limb is scheduled for surgery. Choose the optimal drug for the prevention of deep vein thrombosis in this patient.

Фенілін Feniline

Ацетилсаліцилова кислота Acetylsalicylic acid

Звичайний гепарин Plain heparin

Низькомолекулярний гепарин Low molecular weight heparin

Реополіглюкін Rheopoliglyukin

553 / 1500
Чотиримісячна дитина захворіла гостро. З'явилися такі симптоми: підвищення температури тіла до 38,5^oC, одноразове блювання, в'ялiсть. Через 10 годин з'явилося висипання на сідницях та нижнiх кiнцiвках у виглядi петехій, плям та папул. Деякі геморагічні елементи з некрозом у центрі. Яке захворювання ймовірно у дитини? A four-month-old child became acutely ill. The following symptoms appeared: an increase in body temperature to 38.5^oC, one-time vomiting, lethargy. After 10 hours from there was a rash on the buttocks and lower extremities in the form of petechiae, spots and papules. What disease is the child likely to have?

Грип Flu

Скарлатина Scarlatina

Геморагічний васкуліт Hemorrhagic vasculitis

Менінгококцемія Meningococcemia

Краснуха Krasnukha

554 / 1500
Пацієнта віком 18 років шпиталізовано до лікарні після бійки з ознаками внутрішньої кровотечі. З анамнезу відомо, що хворіє на гемофілію А. Діагностовано позаочеревну гематому. Що першочергово потрібно призначити пацієнту? An 18-year-old patient was hospitalized after a fight with signs of internal bleeding. From the anamnesis, it is known that he suffers from hemophilia A. An extraperitoneal hematoma is diagnosed. What should be prescribed to the patient as a priority ?

Суху плазму Dry plasma

Кріопреципітат Cryoprecipitate

Амінокапронову кислоту Aminocaproic acid

Тромбоцитарну масу Platelet mass

Свіжу кров Fresh blood

555 / 1500
У породіллі через 4 тижні після термінових пологів підвищилась температура тіла до 39^oC, з'явилися слабкість та біль у правій молочній залозі, озноб. Молочна залоза нагрубла, збільшена, чутлива при пальпації. Флуктуації у ділянці інфільтрату немає. В аналізі крові помірний лейкоцитоз. Який діагноз є найбільш імовірним? 4 weeks after the emergency delivery, the mother's body temperature rose to 39^oC, weakness and pain in the right mammary gland appeared, chills. The mammary gland thickened, enlarged, sensitive on palpation. There is no fluctuation in the infiltrate. In the blood analysis, what is the most likely diagnosis?

Гангренозний мастит Gangrenous mastitis

Лактостаз Lactostasis

Серозний мастит Serous mastitis

Мастопатія Mastopathy

Абсцедуючий мастит Abscessing mastitis

556 / 1500
Давність виникнення синців можна визначити за: The age of bruising can be determined by:

Температурою шкіри в синці Skin temperature in the bruise

Швидкістю поширення крові по підшкірній клітковині By the speed of blood spreading through the subcutaneous tissue

За кількістю синців By number of bruises

Зміною форми синця By changing the shape of the bruise

Забарвленням синця The color of the bruise

557 / 1500
Пацієнтка 49 років скаржиться на нерегулярність циклу впродовж 18 місяців, міжменструальні кровотечі та <<приливи>>, які її дуже турбують; наполягає на негайному проведенні лікування. Яку процедуру треба провести перед призначенням терапії? A 49-year-old patient complains of cycle irregularity for 18 months, intermenstrual bleeding and <>, which bother her a lot; she insists on immediate treatment. What procedure should be performed before prescribing therapy?

Призначення медроксипрогестерону ацетату Medroxyprogesterone acetate prescription

Біопсія ендометрію Endometrial biopsy

Гістеректомія Hysterectomy

Застосування естрогенового крему Estrogen cream application

Послідовна терапія естрогенами та прогестином Sequential estrogen and progestin therapy

558 / 1500
Хлопчику 15 років. Непокоїть періо-дичний головний біль, стомлюваність. При огляді дитина достатньо активна, психо-емоційний розвиток відповідає вікові, шкіра бліда, волога на дотик, з боку внутрішніх органів відхилень не виявлено. Артеріальний тиск - 120/80 мм рт.ст. Звертає на себе увагу надмірний розвиток жирової підшкірної клітковини, розподіленої рівномірно. Лікарем висловлене припущення про наявність у дитини ожиріння. Який показник першочергово повинен бути врахований для підтвердження діагнозу? The boy is 15 years old. He is concerned about periodic headaches, fatigue. During examination, the child is quite active, psycho-emotional development corresponds to his age, the skin is pale, moist to the touch, on the part of internal organs, no abnormalities were detected. Blood pressure - 120/80 mmHg. Attention is drawn to the excessive development of subcutaneous tissue. The doctor suggested that the child is obese ?

Індекс маси тіла Body mass index

Відношення маси тіла до зросту Ratio of body weight to height

Товщина підшкірної жирової складки Thickness of the subcutaneous fat fold

Наявність ожиріння у членів родини Obesity in family members

Маса тіла Body weight

559 / 1500
Пацієнтка віком 19 років скаржиться на відсутність менструацій, млявість, зменшення ваги. З анамнезу відомо: рік тому - патологічні пологи зі значною крововтратою. Після пологів лактації не було. Під час піхвового дослідження виявлено: піхва вузька, матка зменшена, яєчники не пальпуються. Лабораторно виявлено гіпоестрогенемію. Який наймовірніший діагноз? A 19-year-old patient complains of the absence of menstruation, lethargy, and weight loss. It is known from the anamnesis: a year ago - pathological childbirth with significant blood loss. There was no lactation after childbirth. During the vaginal examination, it was found that the vagina is narrow, the uterus is not palpable. Hypoestrogenemia was detected in the laboratory. What is the most likely diagnosis?

Синдром Шихана Sheehan Syndrome

Астено-вегетативний синдром Astheno-vegetative syndrome

Гіпотиреоз Hypothyroidism

Туберкульоз геніталій Genital tuberculosis

Синдром Штейна-Левенталя Stein-Leventhal syndrome

560 / 1500
Чоловік 58-ми років, протягом 20-ти років страждає на варикозне розширення поверхневих вен правої нижньої кінцівки. Рік тому у нього по медіальній поверхні в нижній третині правої гомілки утворилася трофічна виразка, яка після консервативного лікування загоїлась. Хворому запропоновано оперативне лікування - видалення поверхневих варикозних вен та перев'язка перфорантних. Яке інструментальне обстеження перед операцією буде найбільш доречним? A 58-year-old man has been suffering from varicose veins of the superficial veins of the right lower limb for 20 years. A year ago, he had a rash on the medial surface in the lower third of the right lower leg a trophic ulcer was formed, which healed after conservative treatment. The patient was offered surgical treatment - removal of superficial varicose veins and ligation of perforating veins. What instrumental examination would be most appropriate?

Осцилографія Oscillography

Флебоманометрія Phlebomanometry

Плетизмографія Plethysmography

Ультразвукове дуплексне сканування вен Ultrasound duplex scanning of veins

Реовазографія Rheovasography

561 / 1500
У жінки віком 20 років протягом 8 років у серпні-вересні виникають риніт і кон'юнктивіт, а минулого року додалися напади бронхіальної астми. Під час шкірного тестування виявлена гіперчутливість до пилку амброзії. До якого класу імуноглобулінів належать антитіла, що реалізують загострення захворювання? A 20-year-old woman has had rhinitis and conjunctivitis for 8 years in August-September, and last year, attacks of bronchial asthma were added. Hypersensitivity was detected during skin testing to ragweed pollen. To which class of immunoglobulins belong the antibodies that implement the exacerbation of the disease?

Імуноглобулін Е Immunoglobulin E

Імуноглобулін А Immunoglobulin A

Імуноглобулін М Immunoglobulin M

Імуноглобулін D Immunoglobulin D

Імуноглобулін G Immunoglobulin G

562 / 1500
У хворого 48-ми років під час профілактичного огляду на рентгенографії ОГК у паренхімі верхньої частки правої легені виявлена промениста тінь до 7 см у діаметрі, яка прилягає до грудної стінки. Який діагностичний метод найбільш інформативний? In a 48-year-old patient, during a preventive X-ray examination, a radiant shadow up to 7 cm in diameter was found in the parenchyma of the upper lobe of the right lung, which is adjacent to the chest wall . Which diagnostic method is the most informative?

Визначення онкомаркерів Definition of tumor markers

Діагностична торакотомія Diagnostic thoracotomy

Аналіз мокроти на клітини злоякісного новоутворення Analysis of sputum for malignant neoplasm cells

Трансторакальна пункція Transthoracic puncture

Фібробронхоскопія Fibrobronchoscopy

563 / 1500
Дівчинка 5-ти днів, від першої вагітності. Маса при народженні - 3100 г, довжина тіла - 51 см. Оцінка за шкалою Апгар - 8/9 балів. На 3-тю добу з'явилася іктерич-ність шкірних покривів. При огляді на 4-й день життя стан задовільний, смокче добре, крик гучний. Пупкова ранка чиста. В легенях дихання пуерильне, серцеві тони звучні. Живіт м'який, печінка +1 см, селезінка ''-''. Випорожнення жовтого кольору. Група крові матері А (II) Rh(+). Група крові дитини 0 (I) Rh(+). Білірубін на 4-ту добу: непрямий - 140 мкмоль/л, прямий - 0, АЛТ- 25 ммоль/л, ACT- 18 ммоль/л. Визначте діагноз: The girl is 5 days old, from the first pregnancy. Birth weight - 3100 g, body length - 51 cm. Apgar score - 8/9 points. On the 3rd day, jaundice appeared. On the 4th day of life, the condition is satisfactory, the umbilical wound is clear. The lungs are soft, the heart sounds are soft +1 cm, stool yellow. Blood group A (II) Rh(+). Bilirubin on the 4th day - 140 μmol /l, direct - 0, ALT - 25 mmol/l, ACT - 18 mmol/l. Determine the diagnosis:

Гемолітична анемія Hemolytic anemia

Фізіологічна жовтяниця Physiological jaundice

Вроджений гепатит Congenital hepatitis

Атрезія жовчовивідних шляхів Biliary atresia

Гемолітична хвороба новонароджених Hemolytic disease of the newborn

564 / 1500
У чоловіка 51 року після переохолодження гостро з'явився біль унизу живота, різь наприкінці сечовипускання. Частота сечовипускання до 15 разів на добу. Сеча каламутна, з домішками крові. У клінічному аналізі сечі виявлено: лейкоцити на все поле зору, поодинокі еритроцити. Яким буде діагностичне припущення? A 51-year-old man developed acute pain in the lower abdomen after hypothermia, cutting at the end of urination. The frequency of urination is up to 15 times a day. The urine is cloudy, with blood impurities. In the clinical analysis of urine, leukocytes were found throughout the field of vision, single erythrocytes. What will be the diagnostic assumption?

Гострий гломерулонефрит Acute glomerulonephritis

Сечокам'яна хвороба Urolithiasis

Гострий уретрит Acute urethritis

Гострий цистит Acute cystitis

Гострий пієлонефрит Acute pyelonephritis

565 / 1500
Жінка 21 року звернулась до хірурга за 5 годин після опіку окропом лівого передпліччя. Об'єктивно спостерігається: на ділянці 17x10 см були виявлені в'ялі пухирі, місцями розірваний і зсунутий епідерміс, дно рани білувато-сіре, сухувате, больова чутливість знижена. Після лікування пов'язками з розчином фурациліну з левосином та з гіпозолем на 29-ту добу після опіку некротичний струп відділився, у ділянці опіку рожевий поверхневий рубець. Якого ступеня опік був у пацієнтки? A 21-year-old woman consulted a surgeon 5 hours after a scald burn of her left forearm. Objectively observed: flaccid blisters were found on an area of ​​17x10 cm, torn in places and the epidermis is shifted, the bottom of the wound is whitish-gray, pain sensitivity is reduced. After treatment with dressings with a solution of furacilin and with hyposol, on the 29th day after the burn, the necrotic scab separated, in the area of ​​the burn, what is the degree of the burn was at the patient's?

І ступеня I degree

ІІ ступеня II degree

ІІІ Б ступеня III B degree

ІV ступеня IV degree

ІІІ А ступеня III A degree

566 / 1500
Пацієнт віком 67 років скаржиться на запаморочення, загальну слабкість, двічі була втрата свідомості. З анамнезу відомо, що 4 роки тому він переніс інфаркт міокарда. Об'єктивно спостерігається: стан середньої важкості, шкіра та слизові оболонки звичайного кольору, у легенях дихання везикулярне, хрипів немає. Тони серця приглушені, аритмічні, ЧСС - 62/хв. АТ - 140/90 мм рт. ст. Живіт м'який, безболісний. Периферичних набряків немає. На ЕКГ виявлено: інтервал PQ - 200 мс, QRS - 80 мс, раптове випадіння комплексу QRS без попереднього збільшення інтервалу PQ, з періодичністю - 3:1, 4:1. Яке порушення ритму серця виникло у пацієнта? A 67-year-old patient complains of dizziness, general weakness, twice lost consciousness. It is known from the anamnesis that he suffered a myocardial infarction 4 years ago. Objectively observed : condition of medium severity, lungs are vesicular, heart sounds are muffled, heart rate - 140/90 mm Hg. Peripheral pain There is no swelling. The ECG showed: PQ interval - 80 ms, sudden loss of the QRS complex, with a frequency of 3:1, 4:1. What kind of heart rhythm disorder did the patient have?

АВ блокада ІІ ступеня, Мобітц І AB blockade II degree, Mobitz I

АВ блокада ІІ ступеня, Мобітц ІІ AB blockade II degree, Mobitz II

Повна АВ блокада Complete AV blockade

АВ блокада І ступеня AV blockade of the 1st degree

СА блокада SA blockade

567 / 1500
У постраждалого в стані травматичного шоку після значной крововтрати, артеріальний тиск - 70 мм рт. ст., ЧСС - 140/хв. Під час гіповолемічного шоку: The victim is in a state of traumatic shock after significant blood loss, blood pressure - 70 mm Hg, heart rate - 140/min. During hypovolemic shock:

Знижується серцевий індекс Decreasing heart index

Знижується системний судинний опір Systemic vascular resistance decreases

Підвищується рН плазми Raising pH of plasma

Збільшується центральний венозний тиск Central venous pressure increases

Виникають набряки на нижніх кінцівках Swelling occurs on the lower limbs

568 / 1500
Пацієнтка 29 років скаржиться на болючі, тривалі менструації. З анамнезу відомо: менархе з 13 років помірно болючі, але рясні. З 19 років болючість менструацій посилилась, подовжилась їх тривалість. У шлюбі і статеве життя з 25 років, протизаплідних засобів не застосовує, не вагітніє. Під час гістероскопії у ділянці дна матки спостерігаються темно-червоні утвори, з яких виділяється темна кров. Який діагноз відповідає клінічній картині? A 29-year-old patient complains of painful, prolonged menstruation. It is known from the anamnesis: menarche since the age of 13 has been moderately painful, but abundant. Since the age of 19, the pain of menstruation has increased, they have become longer duration. In marriage and sexual life since 25 years, does not use contraceptives. During hysteroscopy, dark red formations from which dark blood is secreted are observed?

Залозисто-кістозна гіперплазія ендометрію Glandular-cystic endometrial hyperplasia

Субмукозна міома матки Submucosal myoma of the uterus

Внутрішній ендометріоз Internal endometriosis

Аденокарцинома Adenocarcinoma

Поліп ендометрію Endometrial polyp

569 / 1500
Юнаку віком 17 років встановлено діагноз: хвороба Прасада, для якої характерні низький зріст, недостатній статевий розвиток, збільшення печінки та селезінки, залізодефіцитна анемія в крові. Недостатність якого мікроелемента в харчовому раціоні викликає це захворювання? A 17-year-old young man was diagnosed with Prasad disease, which is characterized by short stature, insufficient sexual development, an increase in the liver and spleen, and iron deficiency anemia in the blood. Deficiency of which trace element in the diet causes this disease?

Йоду Jodu

Цинку Zinc

Селену Selena

Заліза Iron

Міді Copper

570 / 1500
Жінка 52-х років звернулася до лікаря зі скаргами на швидку втомлюваність та задишку протягом останнього року з періодичним сухим кашлем. Пацієнтка відзначає, що останнім часом їй стало важко піднімати обидві руки та розчісувати волосся. При фізикальному обстеженні температура тіла - 37,4^оC, пульс - 76/хв., артеріальний тиск - 130/85 мм рт.ст., SpO_2 - 95% при кімнатному повітрі. При неврологічному дослідженні незначне зниження сили дельтоподібного м'яза. При аускультації легень - розповсюджені сухі хрипи. На щоках, навколо орбіт та на ліктях еритематозний висип. Який метод найбільш імовірно підтвердить діагноз у цієї пацієнтки? A 52-year-old woman came to the doctor with complaints of rapid fatigue and shortness of breath for the past year with a periodic dry cough. The patient notes that recently it has become difficult for her to lift both hands and comb hair.Body temperature - 37.4°C, pulse - 76/min, blood pressure - 130/85 mmHg, SpO_2 - 95% in room air the strength of the deltoid muscle. On auscultation of the lungs - diffused rales. On the cheeks, around the orbits and on the elbows. Which method is most likely to confirm the diagnosis in this patient?

Виявлення антинуклеарних антитіл Detection of antinuclear antibodies

КТ легень CT lung

Біопсія шкіри Skin biopsy

Рентгенографія ОГП Roentgenography of OHP

Біопсія м'яза Muscle biopsy

571 / 1500
Пацієнт віком 32 роки хворіє на епілепсію. Раптово, без будь-яких причин, став збудженим. Об'єктивно спостерігається: дезорієнтований у власній особі та навколишньому середовищі, зорові та слухові галюцинації загрозливого змісту, маячні ідеї стосунку і переслідування; на обличчі вираз страху, гніву, люті; поведінка агресивна, з руйнівними діями. Визначте психопатологічний синдром. A 32-year-old patient suffers from epilepsy. Suddenly, without any reason, he became excited. Objectively observed: disoriented in his own person and environment, vision and auditory hallucinations of a threatening content, delusional ideas of the relationship and persecution; the expression of fear, anger, rage; define the psychopathological syndrome.

Деліріозний синдром Delirious syndrome

Гебефренічний синдром Hebephrenic syndrome

Параноїдний синдром Paranoid Syndrome

Маніакальний синдром Manic syndrome

Сутінковий синдром Twilight Syndrome

572 / 1500
У жінки віком 70 роки в ході флюорографії ОГК над лівим куполом діафрагми виявлено тінь неоднорідної структури. Рентгеноскопічне дослідження з контрастуванням виявило наявність у грудній порожнині абдомінального сегмента стравоходу. Який діагноз найімовірніший? In a 70-year-old woman, a shadow of a heterogeneous structure was detected during fluoroscopy of the OGK above the left dome of the diaphragm. X-ray examination with contrast revealed the presence of an abdominal segment of the esophagus in the chest cavity. What is the diagnosis the most likely?

Езофагіт Esophagitis

Грижа стравохідного отвору діафрагми Hernia of the esophageal orifice of the diaphragm

Ахалазія кардії Achalasia cardia

Дивертикул стравоходу Esophageal diverticulum

Доброякісна пухлина стравоходу Benign tumor of esophagus

573 / 1500
Через 1,5 години після народження на 32 тижні у дитини спостерігаються роздування крил носа, хрюкаючий видих, тахіпное та втяжіння міжреберних проміжків. Забруднення амніотичної рідини меконієм виявлено не було. Температура новонародженого - 37,4^oC, пульс - 180/хв., частота дихання - 80/хв. Шкіра ціанотична. Аускультативно в легенях послаблене дихання з обох боків. РаО_2 - 32 мм рт.ст., РаСО_2 - 48 мм рт.ст. На рентгенограмі органів грудної клітки дифузний сітчасто-зернистий рисунок, повітряна ''бронхограма''. Який діагноз є найбільш імовірним? In 1.5 hours after birth at 32 weeks, the baby has flaring of the wings of the nose, grunting exhalation, tachypnea, and retractions of the intercostal spaces. Meconium contamination of the amniotic fluid was not detected The temperature of the newborn is 37.4°C, the respiratory rate is 80/min Art. Diffuse mesh-granular pattern on chest X-ray, air 'bronchogram'. What is the most likely diagnosis?

Респіраторний дистрес-синдром Respiratory distress syndrome

Ідіопатичний легеневий фіброз Idiopathic pulmonary fibrosis

Транзиторне тахіпное новонароджених Transient tachypnea of ​​newborns

Крововилив у легені Hemorrhage in the lungs

Аспіраційна пневмонія Aspiration pneumonia

574 / 1500
Для організації водопостачання жителів селища вибрано вододжерело з умістом сульфатів, хлоридів, нітратів, заліза, що відповідає гігієнічним вимогам, проте з підвищеним умістом фтору (3 мг/дм^3). До розвитку якого захворювання може призвести вживання води з таким хімічним складом? For the organization of water supply to the residents of the village, a water source with a content of sulfates, chlorides, nitrates, iron, which meets hygienic requirements, but with an increased content of fluorine (3 mg/dm^ 3). What disease can be caused by the use of water with such a chemical composition?

Карієсу Caries

Уролітіазу Urolithiasis

Флюорозу Fluorosis

Жовчно-кам'яної хвороби Cholelithiasis

Подагри Gout

575 / 1500
У водія під час клінічного обстеження у сечі виявлено підвищений вміст кальціє-вих солей фосфорної кислоти. Його раціон харчування містить житній та пшеничний хліб, макаронні вироби, вершкове масло, олія, картопляне пюре, молоко, сир, кава, чай, відвар шипшини, кисіль зі смородини. Енергоцінність раціону відповідає енерговитратам. Що потрібно обмежити у раціоні? During a clinical examination, the driver was found to have an elevated content of calcium salts of phosphoric acid in his urine. His diet includes rye and wheat bread, pasta, butter, oil, mashed potatoes, milk, cheese, tea, rosehip broth, currant jelly. The energy value of the diet corresponds to the energy expenditure. What should be limited in the diet?

Кисіль зі смородини Currant jelly

Відвар шипшини і кисіль Decoction of rose hips and jelly

Молоко і сир Milk and cheese

Макаронні вироби і хліб Pasta and bread

Каву та чай Coffee and tea

576 / 1500
Чоловік 32-х років звернувся до лікаря зі скаргами на висип, що з'явився на колінах 4 місяці тому і зараз вперше починає з'являтися на ліктях. Зі слів пацієнта болю не відчуває, але місце висипу злегка свербить та кровоточить при розчісуванні. Він відзначив, що на ліктях висип з'явився після подряпин, які йому наніс його домашній кіт. При фізикальному обстеженні лікарем виявлено на колінах та ліктях еритематозні бляшки діаметром від 2 до 3 см із чітким контуром, щільні, вкриті сріблястими лусочками. Який діагноз є найбільш імовірним? A 32-year-old man went to the doctor complaining of a rash that appeared on his knees 4 months ago and is now beginning to appear on his elbows for the first time. With According to the patient, he does not feel pain, but the rash is slightly itchy and bleeds when combed. He noted that the rash appeared on his elbows after being scratched by his pet cat. On physical examination, erythematous plaques with a diameter of 2 up to 3 cm with a clear outline, dense, covered with silvery scales. What diagnosis is most likely?

Хвороба котячих подряпин Cat Scratch Disease

Себорейний дерматит Seborrheic dermatitis

Контактний дерматит Contact dermatitis

Псоріаз Psoriasis

Екзема Eczema

577 / 1500
У лікарню швидкої допомоги поступив хворий, що скаржиться на нудоту, багаторазове блювання, пронос, розлади зору - поява сітки перед очима, нечітке бачення ближніх предметів. З анамнезу хворого з'ясувалося, що напередодні ввечері він споживав м'ясну консерву домашнього приготування зі смаженою картоплею. Перші симптоми з'явились вночі. Який діагноз найбільш імовірний? A patient was admitted to the emergency hospital complaining of nausea, repeated vomiting, diarrhea, visual disturbances - the appearance of a net in front of the eyes, blurred vision of nearby objects. From the patient's history it turned out that he had eaten canned meat with fried potatoes the night before. The first symptoms appeared at night. What is the most likely diagnosis?

Стафілококовий токсикоз Staphylococcal toxicosis

Шигельоз Shigelosis

Дизентерія Dysentery

Ботулізм Botulism

Сальмонельоз Salmonellosis

578 / 1500
Чоловік 51 року доставлений до лікарні зі скаргами на біль у лівому боці, загальну слабість, сухість у роті, спрагу. За годину до госпіталізації, переходячи залізничну колію, упав лівим боком на рельси. Шкірні покриви та видимі слизові облонки бліді, холодний піт. Пульс слабкого наповнення, 100/хв. АТ - 85/55 мм рт. ст. Болючість та незначне напруження м'язів у лівому підребер'ї, слабо позитивний симптом Щоткіна-Блюмберга. Що можна запідозрити у хворого? A 51-year-old man was taken to the hospital with complaints of pain in the left side, general weakness, dry mouth, thirst. An hour before hospitalization, he fell while crossing the railway tracks on the left side.The skin and mucous membranes are pale. The pulse is 100/55 mm Hg. Pain and slight tension in the left hypochondrium Shttkin-Blumberg. What can be suspected in the patient?

Пошкодження селезінки Spleen damage

Пошкодження шлунка Stomach damage

Пошкодження печінки Liver damage

Пошкодження підшлункової залози Pancreas damage

Пошкодження товстої кишки Colon damage

579 / 1500
Хворий 55-ти років скаржиться на біль у грудині, поперековому відділі хребта, ребрах. Анамнестично-патологічний перелом кісток правої гомілки. У крові: загальний білок - 110 г/л, позитивний М-градієнт. У сечі: білок Бенс-Джонса. Ваш діагноз: A 55-year-old patient complains of pain in the sternum, lumbar spine, ribs. Anamnestic pathological fracture of the bones of the right tibia. Blood: total protein - 110 g /l, positive M-gradient. In urine: Bence-Jones protein. Your diagnosis:

Гломерулонефрит Glomerulonephritis

Остеохондроз Osteochondrosis

Невралгія Neuralgia

Мієломна хвороба Myeloma

Стенокардія напруги 2 ФК Angina of voltage 2 FC

580 / 1500
Пацієнту віком 43 роки діагностовано гострий респіраторний дистрес-синдром. На який показник необхідно орієнтуватися під час оцінки ступеня тяжкості цього захворювання? A 43-year-old patient was diagnosed with acute respiratory distress syndrome. What indicator should be used to assess the severity of this disease?

Респіраторний індекс Respiratory index

Центральний венозний тиск Central venous pressure

Сатурація гемоглобіну артеріальної крові Hemoglobin saturation of arterial blood

Дихальний об'єм Respiratory volume

Парціальний тиск кисню в артеріальній крові Partial pressure of oxygen in arterial blood

581 / 1500
Чоловік 54 років поступив до стаціонару зі скаргами на загальну слабкість, підвищення температури тіла до 38,6^oС, задишку під час значного фізичного навантаження, часті носові кровотечі. Працює на виробництві пластмаси, де контактує з ароматичними з'єднаннями. Об'єктивно встановлено: шкірні покриви бліді, сухі. У крові виявлено: еритроцити - 2,1·10^12/л, Hb - 90 г/л, лейкоцити - 2,2·10^9/л, еозинофіли - 1%, паличкоядерні - 1%, сегментоядерні - 75%, лімфоцити - 20%, мієлоцити - 3%, тромбоцити - 30·10^9/л, ШОЕ - 32 мм/год. Який найімовірніший діагноз? A 54-year-old man was admitted to the hospital with complaints of general weakness, an increase in body temperature to 38.6^oC, shortness of breath during significant physical exertion, frequent nosebleeds. He works in the production of plastics, where he comes in contact with aromatic compounds. ,2·10^9/l, eosinophils - 1%, rod-nuclear cells - 1%, segmentonuclear cells - 75%, lymphocytes - 20%, myelocytes - 3%, platelets - 30·10^9/l, ESR - 32 mm/h . What is the most likely diagnosis?

Хронічна інтоксикація бензолом Chronic benzene intoxication

Хронічна інтоксикація аніліном Chronic aniline intoxication

Хронічна інтоксикація тетраетилсвинцем Chronic tetraethyl lead intoxication

Хронічна інтоксикація свинцем Chronic lead intoxication

Хронічна інтоксикація нітробензолом Chronic nitrobenzene intoxication

582 / 1500
Хворий 34-х років перебуває на лікуванні в психіатричній лікарні з приводу загострення шизофренії. Об'єктивно: лежить в ліжку, рухливо загальмований, контакт відсутній. На запитання не відповідає. Поза одноманітна, гіпомімічний, наявний симптом ''хоботка'', воскова гнучкість м'язів, симптом ''повітряної подушки''. В такому стані перебуває близько тижня. Харчування парентеральне. Визначте наявний синдром розладу рухово-вольової сфери: A 34-year-old patient is being treated in a psychiatric hospital due to an exacerbation of schizophrenia. Objectively: he is lying in bed, immobilized, there is no contact. When asked, no responds. Monotonous, hypomimic, the present symptom of 'air cushion'.

Екзогенний ступор Exogenous stupor

Депресивний ступор Depressive stupor

Психогенний ступор Psychogenic stupor

Кататонічний ступор Catatonic stupor

Апатичний ступор Apathetic stupor

583 / 1500
Протягом останніх трьох місяців пацієнтку віком 68 років турбує біль у серці тривалістю 10 хвилин, що виникає при найменшому фізичному навантаженні. Нітрогліцерином не користується через сильний головний біль. Неодноразово лікувалася з приводу ІХС, перенесла інфаркт міокарда. АТ періодично підвищується до 160/80 мм рт. ст. На ЕКГ спостерігаються рубцеві зміни ділянки задньої стінки лівого шлуночка. Аускультативно виявлено систолічний шум над аортою. Який діагноз найімовірніший? For the past three months, a 68-year-old patient has been troubled by heart pain lasting 10 minutes, which occurs with the slightest physical exertion. She does not use nitroglycerin because of severe headaches. She has been treated several times about myocardial infarction. Blood pressure rises to 160/80 mm Hg. On the ECG, a systolic murmur is detected over the aorta.

Аневризма аорти з розшаруванням Aortic aneurysm with dissection

Стабільна стенокардія напруги ФК ІІ Stable angina pectoris II

Рецидивний інфаркт міокарда Recurrent myocardial infarction

Стабільна стенокардія напруги ФК IV Stable angina pectoris IV

Нестабільна стенокардія Unstable angina

584 / 1500
У дитини 7 років після контакту з котом раптово з'явилось утруднення дихання. Об'єктивно спостерігається: хлопчик блідий, переляканий, сидить, спираючись на руки. Температура тіла - 36,6^oC, ЧСС - 120/хв., ЧД - 42/хв., говорить по складах, видих подовжений, свистячі хрипи під час видиху, над легенями перкуторно вислуховується коробковий звук. Розпочато інгаляцію розчину вентоліну з використанням небулайзера. Що з наведеного допоможе оцінити потребу в оксигенотерапії у цієї дитини? A 7-year-old child suddenly developed difficulty breathing after contact with a cat. Objectively observed: the boy is pale, frightened, sitting, leaning on his arms. Body temperature - 36.6°C, heart rate - 120/min., speaking in syllables, exhalation is prolonged, whistling sound is heard over the lungs. Inhalation of Ventolin solution using a nebulizer was started of the following will help assess the need for oxygen therapy in this child?

Пневмотахометрія Pneumotachometry

Пульсоксиметрія Pulse oximetry

Оцінка участі допоміжної мускулатури у диханні Assessment of the participation of auxiliary muscles in breathing

Спірометричне дослідження Spirometric study

Рентгенологічне обстеження органів грудної клітки X-ray examination of chest organs

585 / 1500
У дівчинки з тетрадою Фалло, після психоемоційного збудження виник задухо-ціанотичний напад. У цьому стані мати з дитиною звернулася до педіатра, який проводив прийом у дитячій поліклініці. Який препарат невідкладної допомоги необхідно ввести першим? A girl with tetrad of Fallot, after psycho-emotional excitement, had a suffocating-cyanotic attack. In this state, the mother and the child turned to the pediatrician, who held an appointment at the children's clinic. What should an emergency drug be administered first?

Провести інтубацію трахеї і дати кисень Intubate the trachea and give oxygen

Дигоксін Digoxin

Папаверін Papaverine

Пропранолол Propranolol

Калію хлорид Potassium chloride

586 / 1500
Пацієнтка, що хворіє на системний червоний вовчак у складі комплексної терапії отримувала лікування метилпреднізолоном у дозі 10 мг на добу протягом останніх 6 місяців. Яке ускладнення може розвинутися у результаті довготривалого прийому кортикостероїдів? A patient with systemic lupus erythematosus as part of complex therapy was treated with methylprednisolone at a dose of 10 mg per day for the past 6 months. What complications can develop as a result of long-term taking corticosteroids?

Остеопороз Osteoporosis

Гіпоглікемія Hypoglycemia

Артеріальна гіпотонія Arterial hypotension

Гіпонатріємія Hyponatremia

Кахексія Cachexia

587 / 1500
Чоловік 42 років унаслідок ДТП отримав політравму: закриті переломи правої плечової кістки та костей лівого передпліччя із зміщенням відламків, закрита тупа травма живота. Доставлений до приймального відділення за 30 хвилин після травми. Шкірни покриви бліді. Артеріальний тиск - 90/20 мм рт. ст., у місцях переломів деформація, біль. Живіт напружений, під час пальпації виникає різкий біль, позитивний симптом Щьоткіна-Блюмберга. Які заходи будуть провідними під час надання кваліфікованої медичної допомоги? A 42-year-old man received polytrauma as a result of a road accident: closed fractures of the right humerus and bones of the left forearm with displacement of fragments, closed blunt trauma to the abdomen. He was taken to the receiving department in 30 minutes after the injury. Blood pressure - 90/20 mmHg. Abdomen tense, during palpation there is a positive symptom of Schotkin medical assistance?

Термінова лапаротомія Urgent laparotomy

Додаткове обстеження з метою визначення точного діагнозу Additional examination to determine the exact diagnosis

Накладення іммобілізації на переломи, знеболення Immobilization on fractures, anesthesia

Інфузійна терапія з метою стабілізації артеріального тиску Infusion therapy to stabilize blood pressure

Блокади переломів місцевим анестетиком Blocks of fractures with local anesthetic

588 / 1500
Чоловік 38 років, вживає алкоголь впродовж 3 років. За 3 дні після запою став відчувати тривогу, страх. Бачив навколо себе павуків, черв'яків, стали переслідувати <<голоси>> осудливого характеру, поводив себе агресивно. У власній особі орієнтований правильно, у місці та часі - дезорієнтований. Який діагноз найімовірніший? A 38-year-old man has been drinking alcohol for 3 years. 3 days after drinking, he began to feel anxiety and fear. He saw spiders and worms around him, began to chase him < > of a judgmental nature, behaved aggressively. In his own person, disoriented in place and time. What is the most likely diagnosis?

Алкогольний делірій Alcoholic delirium

Алкогольний галюциноз Alcoholic hallucinosis

Алкогольна енцефалопатія Alcoholic encephalopathy

Алкогольний параноїд Alcoholic Paranoid

Патологічне сп'яніння Pathological intoxication

589 / 1500
У двомісячної дитини, спостерігається втрата маси тіла, блювота <<фонтаном>> після кожного годування, затримка стільця (1 раз у 2 доби), під час огляду - симптом пісочного годинника. Блювотні маси мають неприємний запах, без вмісту жовчі. Знаходиться на природному вигодовуванні. Який найбільш імовірний діагноз? In a two-month-old child, there is a loss of body weight, vomiting <> after each feeding, stool retention (1 time in 2 days), during examination - hourglass symptom. Vomiting masses are foul-smelling, without bile. What is the most likely diagnosis?

Кишкова непрохідність Intestinal obstruction

Звичне зригування Habitual vomiting

Вроджений пілоростеноз Congenital pylorostenosis

Інвагінація кишечника Intussusception

Пілороспазм Pilorospasm

590 / 1500
Чоловік скаржиться на відчуття важкості за грудиною та періодичне відчуття зупинки їжі, дисфагію. Під час рентгенологічного обстеження барієвий контраст виявляє поодиноке мішкоподібне випинання передньоправої стінки стравоходу з рівними контурами та чітко окресленою шийкою. Який імовірний діагноз? A man complains of a feeling of heaviness behind the sternum and a periodic feeling of stopping food, dysphagia. During X-ray examination, barium contrast reveals a single sac-like protrusion of the anterior right wall of the esophagus with smooth contours and clearly outlined neck. What is the probable diagnosis?

Рак стравоходу Esophageal cancer

Дивертикул стравоходу Esophageal diverticulum

Поліп стравоходу Esophageal polyp

Кила стравохідного отвору діафрагми Hernia of the esophageal orifice of the diaphragm

Варикозне розширення вен стравоходу Esophageal varicose veins

591 / 1500
У хлопчика 3 років раптово під час гри з іграшками з'явився напад кашлю, задишка. Об'єктивно встановлено: ЧД - 45/хв., ЧСС - 130/хв. Перкуторно виявлено: вкорочення перкуторного звуку з правого боку в нижніх відділах. Аускультативно виявлено: з правого боку послаблене дихання с бронхіальним відтінком. На рентгенограмі видно: затемнення нижньої частки легень праворуч. У крові запальних змін не виявлено. Поставлено діагноз: стороннє тіло правого бронху. Укажіть яке ускладнення викликало таку клінічну картину: A 3-year-old boy suddenly developed a cough and shortness of breath while playing with toys. Objectively established: BH - 45/min., HR - 130 Percussion revealed: shortness of sound on the right side. Auscultation revealed: darkening of the lower lobe of the lungs on the right. No foreign body was detected of the right bronchus. Specify which complication caused the following clinical picture:

Ателектаз Atelectasis

Пневмонія Pneumonia

Емфізема Emphysema

Бронхіт Bronchitis

Пневмоторакс Pneumothorax

592 / 1500
Добовий раціон харчування дівчини 14 років містить вітамін А - 1,5 мг, вітамін B_1 - 1,8 мг, вітамін B_2 - 2,0 мг, вітамін B_6 - 2,0 мг, вітамін С - 20 мг. Під час медичного огляду встановлено: дівчина середньої ваги, фізичний розвиток гармонійний, біологічний вік відповідає календарному, слизові оболонки та шкіра без змін. Дівчина часто хворіє гострими респіраторно-вірусними захворюваннями, після яких спостерігаються численні точкові крововиливи у місцях тертя одягу (манжети, гумки, пояси). З найбільшою вірогідністю у дівчини має місце: The daily diet of a 14-year-old girl contains vitamin A - 1.5 mg, vitamin B_1 - 1.8 mg, vitamin B_2 - 2.0 mg, vitamin B_6 - 2.0 mg, vitamin C - 20 mg. During the medical examination, it was found that the girl is of average weight, her physical development corresponds to the calendar age, her mucous membranes and skin are unchanged numerous spot hemorrhages in places of friction of clothes (cuffs, elastics, belts). The girl is most likely to have:

С-гіповітаміноз C-hypovitaminosis

B_2-гіповітаміноз B_2-hypovitaminosis

B_1-гіповітаміноз B_1 hypovitaminosis

А-гіповітаміноз A hypovitaminosis

B_6-гіповітаміноз B_6 hypovitaminosis

593 / 1500
Гіпертонічна хвороба як важливе неепідемічне захворювання є найпоширенішою формою артеріальної гіпертензії, за якої спостерігається стійке підвищення артеріального тиску до 140/90 мм рт. ст. і на яку впливають екзогенні та ендогенні чинники ризику. Що належить до ендогенних чинників? Hypertensive disease as an important non-epidemic disease is the most common form of arterial hypertension, in which there is a persistent increase in blood pressure up to 140/90 mm Hg and which is influenced by exogenous and endogenous risk factors. What belongs to endogenous factors?

Ожиріння Obesity

Метеозалежність Weather dependence

Стрес Stress

Вік (понад 40 років), стать, спадкова схильність Age (over 40 years old), gender, hereditary predisposition

Емоційно-нервові перенапруження Emotional and nervous overstrain

594 / 1500
Дівчина 17-ти років хворіє на цироз печінки впродовж 3-х років. Останніми днями у дівчини періоди збудження змінюються депресією, спить мало. Об'єктивно: стан тяжкий, дівчина загальмована, говорить поодинокі слова, тремор кінцівок, шкірні покриви іктеричні, на шкірі поодинокий геморагічний висип. Найімовірніше ускладнення хвороби: A 17-year-old girl has been suffering from cirrhosis of the liver for 3 years. In recent days, the girl's periods of excitement are replaced by depression, she sleeps little. Objectively: the condition is serious , the girl is retarded, speaks single words, tremors of the limbs, icteric skin, a single hemorrhagic rash on the skin. The most likely complication of the disease:

Біполярний афективний розлад Bipolar affective disorder

Сепсис Sepsis

Синдром Рея Ray syndrome

Ниркова недостатність Kidney failure

Печінкова енцефалопатія Hepatic encephalopathy

595 / 1500
Дівчина 19-ти років скаржиться на помірний свербіж та випадіння волосся на голові. Об'єктивно: на шкірі потиличної ділянки голови одиничне еритематозне вогнище округлої форми з чіткими межами, діаметром 3 см, з азбестоподібним лущенням на поверхні. Волосся у вогнищі обламане на висоті 6-8 мм. Який діагноз є найбільш імовірним? A 19-year-old girl complains of moderate itching and hair loss on the head. Objectively: on the skin of the occipital part of the head, a single round erythematous focus with clear borders, 3 cm in diameter, with asbestos-like peeling on the surface. The hair in the focus is broken off at a height of 6-8 mm. What is the most likely diagnosis?

Мікроспорія Microsporia

Трихофітія Trichophytia

Себорея Seborrhea

Псоріаз Psoriasis

Короста Scabies

596 / 1500
У дитини віком 6 років підвищилася температура тіла до 37,5^oC, з'явилися закладеність носа, нерясні слизові виділення, дертя в горлі. На четвертий день захворювання температура нормалізувалася, проте з'явилися скарги на біль у ногах. Дитина стала накульгувати та «тягнути» ліву ногу. Знизилися м'язовий тонус та рефлекси, чутливість збережена. Яке захворювання може припустити лікар? A 6-year-old child's body temperature rose to 37.5^oC, nasal congestion, sparse mucous secretions, and a scratchy throat appeared. On the fourth day of the disease the temperature normalized, but there were complaints of pain in the legs. The child began to limp and 'pull' the left leg. Muscle tone and reflexes were reduced. What disease can the doctor assume?

Грип, енцефалічна реакція Flu, encephalic reaction

Ботулізм Botulism

Дифтерійний поліневрит Diphtheria polyneuritis

Полірадикулоневрит Polyradiculoneuritis

Поліомієліт, паралітична форма Polio, paralytic form

597 / 1500
Пацієнт на прийомі у лікаря скаржився на болі у суглобах. Із дієтологічного анамнезу відомо, що пацієнт віддає перевагу м'ясній та жирній їжі. Після відповідних досліджень лікар встановив діагноз: подагра. Які з перерахованих продуктів рекомендується вживати пацієнту? The patient at the doctor's appointment complained of pain in the joints. It is known from the dietary anamnesis that the patient prefers meat and fatty food. After the relevant studies, the doctor diagnosed : gout. Which of the listed products is recommended for the patient?

Молокопродукти Dairy

Бобові Beans

Субпродукти Offal

М'ясо молодих тварин і птахів Meat of young animals and birds

Жири тваринного походження Fats of animal origin

598 / 1500
У хворої 34-х років сильний біль глибоко в орбіті, головний біль, підвищення температури тіла. Захворювання виникло гостро. Об'єктивно: повіки набряклі, шкіра червоного кольору, екзофтальм, обмеження рухів очного яблука. Ваш діагноз: A 34-year-old patient has severe pain deep in the orbit, headache, increased body temperature. The disease occurred acutely. Objectively: the eyelids are swollen, the skin is red , exophthalmos, restriction of eyeball movements. Your diagnosis:

Ретробульбарний неврит Retrobulbar neuritis

Виразка рогівки Corneal ulcer

Блефарит Blepharitis

Абсцес повіки Eyelid abscess

Флегмона орбіти Phlegmon of the orbit

599 / 1500
Під час дослідження проби молока виявлено: колір - білуватий, запах - без особливостей, смак - характерний для молока, густина - 1,038, кислотність - 35^o Тернера, жирність - 3,2%. Визначте ступінь якості молока. During the examination of the milk sample, the following was found: color - whitish, odor - without any features, taste - characteristic of milk, density - 1.038, acidity - 35^o Turner, fat content - 3.2%. Determine the degree of milk quality.

Молоко умовно придатне Milk is conditionally suitable

Молоко фальсифіковане Milk is falsified

Молоко доброякісне Good quality milk

Молоко зниженої якості Low quality milk

Молоко недоброякісне Milk is of poor quality

600 / 1500
Породілля 22-х років на 12 добу після нормальних пологів відзначає підвищення температури тіла до 39^oC протягом 3-х днів, біль в правій молочній залозі. Права молочна залоза збільшена, гаряча, напружена, гіперемована, болюча. При пальпації визначається щільний інфільтрат 8х8 см, в центрі якого - флюктуація. Який найбiльш імовiрний дiагноз? A 22-year-old woman in labor on the 12th day after a normal delivery notes an increase in body temperature to 39^oC for 3 days, pain in the right mammary gland. Right mammary the gland is enlarged, hot, tense, painful. During palpation, a dense infiltrate of 8x8 cm is determined, in the center of which there is a fluctuation. What is the most likely diagnosis?

Післяпологовий період, 12 доба. Правобічний лактостаз Postpartum period, 12 days. Right-sided lactostasis

Післяпологовий період, 12 доба. Правобічний гангренозний мастит Postpartum period, 12 days. Right-sided gangrenous mastitis

Післяпологовий період, 12 доба. Правобічний серозний мастит Postpartum period, 12 days. Right-sided serous mastitis

Післяпологовий період, 12 доба. Правобічний інфільтративно-гнійний мастит Postpartum period, 12 days. Right-sided infiltrative-purulent mastitis

Післяпологовий період, 12 доба. Правобічний флегмонозний мастит Postpartum period, 12 days. Right-sided phlegmonous mastitis

601 / 1500
На території міста відзначено підвищений рівень марганцю в атмосферному повітрі, питній воді та продуктах харчування сільськогосподарського походження. Як можна охарактеризувати дію цих чинників на здоров'я населення? An elevated level of manganese in the atmospheric air, drinking water and food products of agricultural origin was noted on the territory of the city. How can the effect of these factors on the health of the population be characterized?

Синергічна Synergistic

Комбінована Combined

Поєднана Combined

Комплексна Comprehensive

Роздільна Separate

602 / 1500
Чоловік 42 років унаслідок ДТП отримав політравму: закриті переломи правої плечової кістки та кісток лівого передпліччя із зміщенням відламків, закриту тупу травму живота. Доставлений до приймального відділення за 30 хвилин після травми. Шкірні покриви бліді. Артеріальний тиск - 90/20 мм рт. ст., у місцях переломів деформація, біль. Живіт напружений, під час пальпації виникає різкий біль, позитивний симптом Щоткіна-Блюмберга. Яких лікувальних заходів слід вжити насамперед? A 42-year-old man received polytrauma as a result of a road accident: closed fractures of the right humerus and bones of the left forearm with displacement of fragments, closed blunt trauma to the abdomen. He was taken to the emergency room in 30 minutes after the injury. Blood pressure - 90/20 mm Hg. Abdomen tense, during palpation there is a positive symptom of Shtokkin. What treatment should be taken first?

Блокада переломів місцевим анестетиком Block of fractures with local anesthetic

Додаткове обстеження для встановлення точного діагнозу Additional examination to establish an accurate diagnosis

Термінова лапаротомія Urgent laparotomy

Інфузійна терапія для стабілізації артеріального тиску Infusion therapy to stabilize blood pressure

Накладення іммобілізації на переломи, знеболення Immobilization on fractures, anesthesia

603 / 1500
До лікаря звернувся чоловік, у якого було діагностовано коросту і який пройшов специфічне лікування. Яких заходів слід вжити щодо профілактики рецидивів корости? A man who was diagnosed with scabies and who underwent specific treatment came to the doctor. What measures should be taken to prevent relapses of scabies?

Корекція імунного статусу Correction of immune status

Дезінфекція білизни Disinfection of linen

Ліквідація харчових алергенів Elimination of food allergens

Дезінфекція взуття Shoe disinfection

Корекція функції шлунково-кишково-го тракту Correction of the function of the gastrointestinal tract

604 / 1500
У дитини 2-х років на тлі відносного соматичного здоров'я після гри з ''мозаї-кою'' раптово виникли кашель, стридорозне дихання, потяг до блювання, ціаноз. При огляді дитини першочергово лікар має припустити: A 2-year-old child, against the background of relative somatic health, suddenly developed a cough, stridorous breathing, urge to vomit after playing with a 'mosaic' , cyanosis. When examining a child, the doctor should first assume:

Гострий обструктивний бронхіт Acute obstructive bronchitis

Аспірацію стороннього тіла Foreign body aspiration

Гострий ларинготрахеїт Acute laryngotracheitis

Пневмонію Pneumonia

Кашлюк Whooping cough

605 / 1500
У дитини 3-х діб із гіпербілірубіне-мією (428 мкмоль/л) з'явилися порушення у вигляді періодичного збудження та судом на тлі млявості, гіпотонії, гіподинамії, пригніченості безумовних рефлексів, а також збіжна косоокість, ротаторний ністагм, симптом ''призахідного'' сонця. Чим пояснити цю симптоматику? A 3-day-old child with hyperbilirubinemia (428 μmol/l) developed disturbances in the form of periodic excitement and convulsions against the background of lethargy, hypotension, hypodynamia , depression of unconditioned reflexes, as well as converging strabismus, rotatory nystagmus, a symptom of the ``set'' sun. How can this symptom be explained?

Черепно-мозкова травма Traumatic brain injury

Білірубінова енцефалопатія Bilirubin encephalopathy

Дитячий церебральний параліч Cerebral palsy

Пухлина головного мозку Brain tumor

Гідроцефалія Hydrocephalus

606 / 1500
Хвора 55-ти років скаржиться на біль і хруст в лівому колінному суглобі, які виникають при ходьбі по сходах, періодичне ''заклинювання'' суглобу при рухах. 5 років тому була травма лівого коліна. Клінічний та біохімічний аналізи крові без особливих змін. Рентгенологічно виражений остеосклероз, остеофіти. Звуження суглобової щілини. Який попередній діагноз? A 55-year-old patient complains of pain and creaking in the left knee joint, which occur when walking up the stairs, periodic 'jamming' of the joint during movements. 5 years ago there was an injury to the left knee. Clinical and biochemical blood tests without special changes. Radiologically expressed osteosclerosis, osteophytes. Narrowing of the joint space. What is the previous diagnosis?

Подагричний артрит Gouty arthritis

Псоріатичний артрит Psoriatic arthritis

Остеоартроз Osteoarthrosis

Реактивний артрит Reactive arthritis

Ревматоїдний артрит Rheumatoid arthritis

607 / 1500
У 200 пацієнтів з гіпертонічною хворобою досліджувалась величина артеріального тиску та вік пацієнта. Якою статистичною величиною потрібно скористатися, щоб виміряти силу зв'язку між вказаними ознаками? In 200 patients with hypertension, the value of blood pressure and the age of the patient were studied. What statistical value should be used to measure the strength of the relationship between the specified characteristics?

Коефіцієнтом варіації Coefficient of variation

Коефіцієнтом Ст'юдента Student coefficient

Коефіцієнтом кореляції Correlation coefficient

Помилкою репрезентативності Representativeness error

Сигмальним відхиленням By sigmal deviation

608 / 1500
Жінка 78-ми років, яка страждає на артеріальну гіпертензію, мала три епізоди раптової транзиторної втрати зору на ліве око. При аускультації сонних артерій в точках біфуркації з обох боків вислуховується шум. Яке додаткове дослідження найбільш доцільно призначити? A 78-year-old woman suffering from arterial hypertension had three episodes of sudden transient loss of vision in the left eye. On auscultation of the carotid arteries at the bifurcation points on both sides noise is heard. What additional research is most appropriate to prescribe?

Люмбальна пункція Lumbar puncture

Дуплексне сканування судин шиї Duplex scan of neck vessels

МРТ головного мозку MRI brain

КТ головного мозку CT brain

Нейросонографія Neurosonography

609 / 1500
На прийомі у сімейного лікаря дитина, яка народилася доношеною, здоровою і дотепер нічим не хворіла. Дитина може гратися іграшками годину і більше. Ходить вздовж меблів при підтримці за одну руку або самостійно. Має мовний запас із 8-12 слів. За проханням обнімає батьків та прагне схвалення, підтвердження свого успіху. Визначте імовірний вік дитини: At the family doctor's appointment, a child who was born full-term, healthy and has not been sick so far. The child can play with toys for an hour or more. He walks along the furniture with support for one hand or independently. Has a language reserve of 8-12 words. Upon request, hugs parents, seeks confirmation of his success. Determine the probable age of the child:

8 місяців 8 months

9 місяців 9 months

6 місяців 6 months

7 місяців 7 months

12 місяців 12 months

610 / 1500
В районі відзначається зростання серцево-судинних захворювань серед населення. Сімейний лікар (лікар загальної практики) планує вжити заходів з профілактики серцево-судинних захворювань. Визначте, які з наведених заходів належать до первинних: There is an increase in cardiovascular diseases among the population in the district. The family doctor (general practitioner) plans to take measures to prevent cardiovascular diseases. Determine which of the following measures belong to the primary:

Направлення на стаціонарне лікування Referral to inpatient treatment

Комплексне обстеження Comprehensive survey

Направлення на санаторно-курортне лікування Direction for sanatorium-resort treatment

Модифікація факторів ризику Modification of risk factors

Направлення на лікування в профільний диспансер Referral for treatment to a specialized dispensary

611 / 1500
Пацієнт віком 45 рокiв захворiв раптово з появи ознобу i пiдвищення температури тіла до 39,2^oС. Ввечерi з'явились iнтенсивнi болi в животi та литкових м'язах. За 2 днi помiтив жовтяничнiсть шкiри i склер. Об'єктивно спостерігається: стан важкий, Т - 39,9^oС. Виражена в'ялiсть. Шкiра i склери помiрно жовтi. На тулубi множиннi петехiї. Дихання везикулярне. ЧД - 20 /хв, ЧСС - 102/хв, АТ - 100/60 мм рт. ст. Живiт м'який, болючий в епiгастрiї, печiнка на 3 см виступає з-пiд реберної дуги. Добовий діурез - 300 мл сечi темного кольору. Який попередній дiагноз? A 45-year-old patient fell ill suddenly with the onset of chills and an increase in body temperature to 39.2°C. In the evening, intense pains in the abdomen and calf muscles appeared For 2 days, he observed jaundice of the skin, T - 39.9 °C, moderate vesicular respiration , heart rate - 100/60 mm Hg. Abdomen is soft, painful, 3 cm below the costal arch. Daily urine output is 300 ml. What is the previous diagnosis?

Сепсис Sepsis

Єрсинiоз Yersiniosis

Лептоспiроз Leptospirosis

Вiрусний гепатит В Viral hepatitis B

Iнфекцiйний мононуклеоз Infectious mononucleosis

612 / 1500
Лікар медичного кабінету загальноосвітньої школи повинен провести оцінку фізичного розвитку дітей та підлітків. Ця оцінка проводиться на підставі зіставлення індивідуальних даних з: The doctor of the medical office of the secondary school must conduct an assessment of the physical development of children and adolescents. This assessment is carried out on the basis of comparing individual data with:

Дані не порівнюють Data does not compare

Даними наукових досліджень в періодичних фахових виданнях Data of scientific research in periodical specialist publications

Регіональними стандартами фізичного розвитку By regional standards of physical development

Даними засобів масової інформації Media data

Загальносвітовими стандартами фізичного розвитку Global standards of physical development

613 / 1500
Жінці віком 35 років через пневмонію призначено ампіцилін. За 30 хвилин після внутрішньом'язової ін'єкції препарату пацієнтка відчула різку слабкість, з'явився кашель, задишка, біль у грудні клітці. Об'єктивно спостерігається: ціаноз, набряк повік, обличчя з червоним висипом. Пульс - 120/хв, АТ - 70/20 мм рт. ст. Тони серця глухі, дихання часте, поверхневе, з різнокаліберними вологими хрипами. Варикозне розширення вен на правій гомілці. Яка найімовірніша причина раптового погіршення стану жінки? A 35-year-old woman was prescribed ampicillin due to pneumonia. 30 minutes after the intramuscular injection of the drug, the patient felt sharp weakness, cough, shortness of breath, pain appeared Objectively observed: cyanosis, red rash, heart rate - 70/min varicose veins on the right leg. What is the most likely cause of the sudden deterioration of the woman's condition?

Набряк Квінке Quincke edema

Тромбоемболія легеневої артерії Thromboembolism of the pulmonary artery

Анафілактичний шок Anaphylactic shock

Кропив'янка Hives

Астматичний напад Asthma attack

614 / 1500
Під час оглядової рентгенографії черевної порожнини в пацієнта спостерігається декілька ділянок просвітлення напівсферичної форми, розташованих над чіткими горизонтальними рівнями. Чим зумовлена така рентгенологічна картина? During the X-ray examination of the abdominal cavity, the patient has several areas of hemispherical light located above clear horizontal levels. What is the reason for such X-ray picture?

- -

Метеоризмом Meteorism

Кишковою непрохідністю Intestinal obstruction

Раком товстого кишечника Colon cancer

Перфоративною виразкою Perforating ulcer

615 / 1500
Пацієнтка віком 28 років скаржиться на відчуття нереальності, зміненності власного тіла. Коли стоїть перед дзеркалом, впізнає себе, але її руки, ноги, обличчя здаються їй чужими, що не належать їй. Який найімовірніший психопатологічний розлад виник у пацієнтки? A 28-year-old patient complains of a feeling of unreality, changes in her own body. When she stands in front of the mirror, she recognizes herself, but her hands, legs, and face seem alien to her, which do not belong to her. What is the most probable psychopathological disorder that the patient has?

Дереалізація Derealization

Деперсоналізація Depersonalization

Сенестопатії Senestopathy

Ілюзії Illusions

Галюцинації Hallucinations

616 / 1500
У жінки 18-ти років, яка перебуває на лікуванні в стаціонарі з приводу системного червоного вовчака, виявлена позитивна реакція мікропреципітації на сифіліс. Клінічних проявів хвороби не виявлено. При дообстеженні - ІФА трепонемний - реакція негативна. Найбільш імовірний діагноз? An 18-year-old woman, who is being treated in a hospital for systemic lupus erythematosus, has a positive microprecipitation reaction for syphilis. No clinical manifestations of the disease were detected. additional examinations - treponemal ELISA - the reaction is negative. The most likely diagnosis?

Сифіліс прихований Syphilis hidden

Сифіліс вторинний Secondary syphilis

Сифіліс третинний Tertiary syphilis

Хибнопозитивна реакція на сифіліс False positive reaction to syphilis

Сифіліс первинний Primary syphilis

617 / 1500
Жінка віком 65 років скаржиться на загальну слабкість, підвищену втомлюваність, оніміння і відчуття поколювання в руках протягом останніх 4 місяців. Об'єктивно спостерігається: температура тіла - 36,6^oС, ЧД - 19/хв, пульс - 96/хв, АТ - 115/70 мм рт. ст. Шкіра та видимі слизові оболонки бліді з незначною жовтушністю. Під час неврологічного огляду виявлено симетричне зменшення чутливості на обох верхніх кінцівках. В загальному аналізі крові: eритроцити - 2,4· 10^12/л, гемоглобін - 105 г/л, лейкоцити - 2,5· 10^9/л, ШОЕ - 28 мм/год, тромбоцити - 180· 10^9/л. Під час мікроскопії мазка виявлено: мегалоцитоз, гіперхромію еритроцитів, гіперсегментацію нейтрофілів, анізоцитоз, пойкілоцитоз. Який найімовірніший діагноз? A 65-year-old woman complains of general weakness, increased fatigue, numbness and a tingling sensation in her hands for the past 4 months. Objectively observed: body temperature - 36, 6^oС, pulse - 96/min. Skin and mucous membranes are pale with slight yellowness general blood analysis: erythrocytes - 2.4 10^12/l, hemoglobin - 105 g/l, leukocytes - 2.5 10^9/l, ESR - 28 mm/h, platelets - 180 10^9/ l. During smear microscopy, the following were found: megalocytosis, hyperchromia of erythrocytes, anisocytosis, poikilocytosis?

Аутоімунна гемолітична анемія Autoimmune hemolytic anemia

В_12 фолієводефіцитна анемія B_12 foliodeficiency anemia

Залізодефіцитна анемія Iron deficiency anemia

Сидероахрестична анемія Sideroachrestic anemia

Анемія хронічного захворювання Anemia of chronic disease

618 / 1500
У недоношеної дитини на 32 тижні гестації за годину після народження з'явилися дихальні розлади, які оцінюються за шкалою Довнеса у 6 балів. Аускультативно над легенями вислуховується ослаблене диханняослаблене дихання, розсіяні крепітуючі хрипи. ЧД - 66/хв. На рентгенограмі органів грудної порожини виявлено: зниження прозорості легеневої тканини за рахунок дифузного сітчасто-зернистого малюнка. Встановіть попередній діагноз. A premature baby at the 32nd week of gestation, an hour after birth, developed respiratory disorders, which are evaluated according to the Dovenes scale at 6 points. Weak breathing is heard over the lungs by auscultation, weak breathing , scattered rales. BH - 66/min. A chest X-ray showed a decrease in the transparency of the lung tissue. Establish a preliminary diagnosis.

Вроджена пневмонія Congenital pneumonia

Діафрагмальна кила Diaphragmatic hernia

Респіраторний дистрес-синдром новонародженого Respiratory distress syndrome of a newborn

Трахео-стравохідна нориця Tracheoesophageal fistula

Набряково-геморагічний синдром Oedema-hemorrhagic syndrome

619 / 1500
У пацієнта віком 19 років після ГРВІ з'явилась стійка макрогематурія, задишка, кровохаркання. Під час рентгенологічного дослідження грудної клітини виявлено: ознаки двобічного дисемінованого ураження. Впродовж 2 тижнів стан пацієнта різко погіршився, рівень креатиніну зріс до 327 ммоль/л. Який найімовірніший діагноз? A 19-year-old patient developed persistent macrohematuria, shortness of breath, and hemoptysis after acute respiratory viral infection. X-ray examination of the chest revealed signs of bilateral disseminated lesions. For 2 weeks the patient's condition has worsened, the creatinine level has increased to 327 mmol/l. What is the most likely diagnosis?

Паранеопластична нефропатія Paraneoplastic nephropathy

Гранулематоз Вегенера Wegener's granulomatosis

Гострий гломерулонефрит Acute glomerulonephritis

Синдром Гудпасчера Goodpasture syndrome

Геморагічний васкуліт Hemorrhagic vasculitis

620 / 1500
Хвора 27-ми років надійшла до гінекологічного відділення зі скаргами на різкий біль в ділянці зовнішніх статевих органів, що посилюється при ходьбі, підвищення температури тіла до 39,2^oC, озноб. Захворіла тиждень тому, після переохолодження. Остання менструація 3 тижні тому. При огляді зовнішніх статевих органів в ділянці великої статевої губи визначається болюче пухлиноподібне утворення розмірами 3х4 см, шкіра над ним гіперемована, гаряча на дотик, при пальпації відмічається флуктуація. Діагноз: A 27-year-old patient came to the gynecology department with complaints of sharp pain in the area of ​​the external genitalia, which worsens when walking, an increase in body temperature to 39.2^ oC, chills. Last menstruation 3 weeks ago. A painful tumor-like mass in the area of ​​the labia majora is determined, the skin over it is hot to the touch. Diagnosis :

Абсцес бартолінієвої залози Bartholin gland abscess

Кіста бартолінієвої залози Bartholin's cyst

Фурункул великої статевої губи Furnish of labia majora

Ліпома зовнішніх статевих органів Lipoma of the external genitalia

Рак вульви Vulvar cancer

621 / 1500
Жінка 40 років після безрезультат-ного лікування у невропатолога була направлена до гінеколога зі скаргами на нагрубання молочних залоз, депресію, що чергується з агресивністю, слабкість, плаксивість, оніміння рук, метеоризм, які з'являються за 2-3 дні до початку менструації і зникають після неї. Уважає себе хворою протягом 2 років. Під час гінекологічного обстеження патологічних змін з боку статевих органів не виявлено. Поставлено діагноз: передменструальний синдром. Яка клінічна форма передменструального синдрому найімовірніша в цьому разі? A 40-year-old woman, after unsuccessful treatment by a neurologist, was referred to a gynecologist with complaints of engorgement of the mammary glands, depression alternating with aggressiveness, weakness, tearfulness, numbness hands, flatulence, which appear 2-3 days before the onset of menstruation. She considers herself ill for 2 years. During the gynecological examination, no pathological changes were detected. What is the clinical form premenstrual syndrome is most likely in this case?

Набрякова Nabryakova

Нервово-психічна Neuro-psychic

Кризова Crisis

Змішана Mixed

Цефалгічна Cephalgic

622 / 1500
При розслідуванні випадку харчового отруєння встановлено діагноз харчової токсикоінфекції, що спричинена парагемолітичним вібріоном. Який із перелічених продуктів і страв може бути найімовірнішою причиною даного отруєння? When investigating a case of food poisoning, a diagnosis of food poisoning caused by parahemolytic vibrio was established. Which of the listed products and dishes could be the most likely cause of this poisoning?

Устриці Oysters

Гриби консервовані Canned mushrooms

Виноград Grapes

Молоко пастеризоване Milk pasteurized

Ковбаса сирокопчена Raw-smoked sausage

623 / 1500
Пацієнта віком 72 роки шпиталізовано до приймального відділення зі скаргами на біль в лівому кульшовому суглобі. Травму отримав на вулиці внаслідок падіння на лівий бік. Під час огляду: пацієнт лежить на спині, ліва нижня кінцівка ротована назовні, дещо приведена, вкорочена. Лівий вертлюг пальпується вище лінії Розер-Нелатона, лінія Шемакера проходить вище пупка. Позитивний симптом <<прилиплої п’яти>> зліва. Який попередній діагноз? A 72-year-old patient was hospitalized with complaints of pain in the left hip joint. He was injured on the street as a result of falling on his left side. During the examination: the patient is lying on the back, the left lower extremity is rotated outward, slightly shortened. The left acetabulum is palpated above the Roser-Nelaton line. A positive symptom of 'sticky heel' on the left side?

Перелом лобкової кістки Fracture of pubic bone

Перелом шийки стегнової кістки Fracture of femoral neck

Вивих стегна Dislocation of the hip

Забій кульшового суглоба Hip contusion

Пошкодження зв’язки кульшового суглобу Hip ligament damage

624 / 1500
Пацієнтка 42 років звернулася до гінеколога зі скаргами на періодичні пекучі болі в середній частині молочної залози зліва. Больові відчуття не залежать від фази менструального циклу. Під час огляду, пальпації патології не виявлено. З якого інструментального методу необхідно почати обстеження? A 42-year-old patient consulted a gynecologist with complaints of periodic burning pains in the middle part of the left mammary gland. The pain does not depend on the phase of the menstrual cycle. During the examination, palpation No pathology was detected. What instrumental method should be used to begin the examination?

Дуктографія Ductography

КТ молочних залоз CT breast

МРТ молочних залоз MRI breast

Біопсія молочної залози Breast biopsy

УЗД молочних залоз Ultrasound of mammary glands

625 / 1500
Місто забезпечується питною водою з підземних артезіанських джерел, яка характеризується високою якістю і чистотою. Який з перерахованих методів обробки води, отриманої з підземних міжпластових напірних джерел, передбачається санітарними вимогами? The city is provided with drinking water from underground artesian springs, which is characterized by high quality and purity. Which of the listed methods of treatment of water obtained from underground interlayer pressure springs is provided by sanitary requirements ?

Дефторування Defluorination

Знезараження Disinfection

Коагуляція Coagulation

Опріснення Desalination

Знебарвлення Fading

626 / 1500
Пацієнтка віком 12 років захворіла гостро з підвищення температури тіла до 38,5^oС та появи висипу на шкірі. Об'єктивно спостерігається: стан середнього ступеня тяжкості, гепатоспленомегалія, дрібнокрапчастий та плямисто-папульозний висип на згинальних поверхнях кінцівок, бічних поверхнях тулуба, внизу живота, відмежоване рожево-синюшне забарвлення кистей та стоп. Який найімовірніший діагноз? A 12-year-old patient became acutely ill with an increase in body temperature to 38.5^oС and the appearance of a rash on the skin. Objectively observed: a condition of moderate severity, hepatosplenomegaly , a small-spotted and papular rash on the flexor surfaces of the limbs, the lateral surfaces of the trunk, the lower part of the abdomen, a demarcated pinkish-bluish coloration of the hands and feet. What is the most likely diagnosis?

Кір Measles

Вітряна віспа Chicken Pox

Інфекційний мононуклеоз Infectious mononucleosis

Псевдотуберкульоз Pseudotuberculosis

Скарлатина Scarlatina

627 / 1500
Пацієнт віком 20 років скаржиться на почервоніння очей, сльозотечу, помірні гнійні виділення з кон'юнктивальної порожнини, відчуття стороннього тіла в очах. Об'єктивно спостерігається: гіперемія кон'юнктиви повік. На очних яблуках кон'юнктивальна ін'єкція. Рогівка прозора. Зіниця 3 мм в діаметрі, на світло реагує жваво. Кришталик, склисте тіло прозорі. Очне дно в нормі. Який найімовірніший діагноз? A 20-year-old patient complains of eye redness, lacrimation, moderate purulent discharge from the conjunctival cavity, the sensation of a foreign body in the eyes. Objectively observed: hyperemia of the Conjunctival injection. The pupil is 3 mm in diameter. The lens is clear. What is the most likely diagnosis?

Аденовірусний кератокон'юнктивіт Adenoviral keratoconjunctivitis

Гострий іридоцикліт Acute iridocyclitis

Гострий бактеріальний кон'юнктивіт Acute bacterial conjunctivitis

Алергічний кон'юнктивіт Allergic conjunctivitis

Гонобленорея Honoblenorrhea

628 / 1500
У дитини після падіння на потилицю з'явилися глухота на праве вухо, периферичний параліч правого лицьового нерва, витікання прозорої рідини з правого слухового проходу, розсіяна неврологічна симптоматика. Встановіть попередній діагноз. After falling on the back of the head, the child developed deafness in the right ear, peripheral paralysis of the right facial nerve, leakage of clear fluid from the right auditory canal, scattered neurological symptoms. Set previous diagnosis.

Забій головного мозку, перелом основи черепа Brain contusion, skull base fracture

Посттравматичний менінгоенцефаліт Post-traumatic meningoencephalitis

Струс головного мозку Concussion

Забій головного мозку, субарахноїдальний крововилив Cerebral contusion, subarachnoid hemorrhage

Менінгоенцефаліт Meningoencephalitis

629 / 1500
Дитина 1 місяця зригує, які виникають щоразу, щойно її після годування грудьми вкладають у ліжечко. Огляд педіатра відхилень у стані дитини не виявив. Виникнення зригувань лікар пояснив особливостями травної системи у цьому віці та дав поради щодо догляду за дитиною. Які це поради? A 1-month-old child vomits, which occur every time as soon as she is put in a crib after breastfeeding. The pediatrician's examination did not reveal any abnormalities in the child's condition. The doctor explained the occurrence of vomiting by the peculiarities of the digestive system system at this age and gave advice on caring for the child. What are these tips?

Вертикальне положення дитини одразу після годування Vertical position of the child immediately after feeding

Масаж живота Abdominal massage

Теплові процедури на живіт Heat procedures on the stomach

Дати соску одразу після годування грудьми Give nipple immediately after breastfeeding

Укладання дитини на живіт після годування Putting baby on stomach after feeding

630 / 1500
Жінка 32 років поскаржилась на раптовий різкий головний біль, втратила свідомість, упала. Лікар швидкої допомоги відзначив важкий стан пацієнтки, сопор, менінгеальний синдром. У ста-ціонарі під час люмбальної пункції отримано кров'янистий ліквор, лікворний тиск - 260 мм водн. ст. Поставте попередній діагноз: A 32-year-old woman complained of a sudden sharp headache, lost consciousness, and fell. The emergency doctor noted the patient's serious condition, sopor, meningeal syndrome. In the hospital under at the time of the lumbar puncture, the cerebrospinal fluid was obtained, the cerebrospinal fluid pressure was 260 mm Hg. Make a preliminary diagnosis:

Ішемічний інсульт Ischemic stroke

Розрив аневризми, субарахноідальний крововилив Aneurysm rupture, subarachnoid hemorrhage

Черепно-мозкова травма Traumatic brain injury

Менінгеома Meningioma

Менінгоенцефаліт Meningoencephalitis

631 / 1500
Пацієнт віком 58 років скаржиться на профузну макрогематурію з відходженням безформних згустків крові, странгурію. Макрогематурія, що не супроводжувалась болями та дизурією, вперше з'явилась 5 місяців тому без видимих причин, через декілька днів кровотеча самостійно припинилась. Який найімовірніший діагноз? A 58-year-old patient complains of profuse macrohematuria with the discharge of shapeless blood clots, stranguria. Macrohematuria, which was not accompanied by pain and dysuria, first appeared 5 months ago without for obvious reasons, after a few days the bleeding stopped on its own. What is the most likely diagnosis?

Дивертикул сечового міхура Bladder diverticulum

Пухлина сечового міхура Bladder tumor

Конкремент сечового міхура Bladder calculus

Пухлина нирки Kidney tumor

Гострий цистит Acute cystitis

632 / 1500
У дітей населеного пункту спостерігається крихкість зубів, неправильний прикус, ерозії емалі зубів, пігментація зубів у вигляді жовто-коричневих плям. Який чинник може бути причиною такої симптоматики? The children of the settlement have fragility of teeth, malocclusion, erosion of tooth enamel, pigmentation of teeth in the form of yellow-brown spots. What factor could be the cause of such symptoms?

Високий вміст нітратів High nitrate

Високий вміст сульфатів у воді High sulfate content in water

Низький вміст фтору у воді Low fluoride content in water

Високий вміст фтору у воді High fluoride content in water

Низький вміст сульфатів у воді Low sulfate content in water

633 / 1500
Рідні чоловіка 36-ти років звернулися до лікаря зі скаргами на зміни у його поведінці. Під час опитування чоловік повідомив, що чує голоси у своїй голові. Його мовою хтось керує та примушує говорити. Він зазначає, що бачить події, які трапляються за межами кімнати у якій він знаходиться. Думки його плинуть самі по собі, всупереч його бажанням. Інколи навіть два потоки думок одночасно. Під час подальшого опитування він повідомив лікаря, що має підозру щодо свого перебування під наглядом засекреченої наукової організації, яка ставить над ним експерименти. Який першочерговий висновок щодо стану пацієнта буде найбільш точним? Relatives of a 36-year-old man went to the doctor with complaints about changes in his behavior. During the interview, the man reported that he hears voices in his head. In his language, someone controls and compels him to speak. He notes that he sees events occurring outside the room he is in. His thoughts flow by themselves. Sometimes he even tells the doctor that he has two streams at the same time suspicion of being under the supervision of a classified scientific organization that is conducting experiments on him. What primary conclusion about the patient's condition will be the most accurate?

Синдром психічного автоматизму Psychic automatism syndrome

Синдром слухового галюцинозу Syndrome of auditory hallucinosis

Параноїдний синдром Paranoid Syndrome

Парафренний синдром Paraphrenic syndrome

Психосенсорний розлад Psychosensory disorder

634 / 1500
Доношена дитина від 1-ї вагітності у вікової першороділлі, 1-х стрімких на 37-38 тижні пологів, народилась із масою 3000 г, зростом - 49 см. Під час оцінювання неврологічного статусу визначається зниження м'язового тонусу в правій руці, праве плече повернуте всередину, лікоть розігнутий, спостерігається пронація передпліччя, кисть зігнута - симптом ляльки. Рефлекс Моро не викликається. Рухи в пальцях збережені, хапальний рефлекс позитивний. Який попередній діагноз? A full-term child from the 1st pregnancy in an age-matched primiparous child, 1st fast at 37-38 weeks of childbirth, was born with a weight of 3000 g, a height of 49 cm. During the evaluation of the neurological status, a decrease in muscle tone is determined in the right arm, the elbow is extended, the forearm is bent - the Moro reflex is not elicited. The movements in the fingers are positive. What is the preliminary diagnosis ?

Перелом ключиці clavicle fracture

Пологова травма: парез Дюшена-Ерба справа Birth trauma: Duchenne-Erb paresis on the right

Параліч діафрагмального нерва Phrenic nerve palsy

Внутрішньочерепна пологова травма Intracranial birth injury

Параліч Клюмпке Klumpke's palsy

635 / 1500
Дитина народилася від 1-ї вагітності, що протікала з гестозом 1 і 2-ї половини. Пологи відбулися шляхом кесаревого розтину. Оцінка за шкалою Апгар - 6 балів. Вага дитини - 3000 г. Груди смоктала активно, не відригувала. За два тижні з'явилося блювання <<фонтаном>> із домішками створоженого молока, об'єм перевищував з'їдене. Дитина стала в'ялою, погано набирає вагу, зменшилася кількість сечі та випорожнень. Який імовірний діагноз? The child was born from the 1st pregnancy, which proceeded with gestosis of the 1st and 2nd half. The delivery took place by caesarean section. Apgar score - 6 points. The child's weight is 3000 g. She was actively sucking, did not regurgitate. In two weeks, vomiting appeared with milk impurities, the volume exceeded what was eaten. The child became sluggish, gained weight poorly, the amount decreased urine and stools. What is the likely diagnosis?

Пілоростеноз Pylorostenosis

Пілороспазм Pilorospasm

Адреногенітальний синдром Adrenogenital syndrome

Кишечна непрохідність Intestinal obstruction

Гостра кишечна інфекція Acute intestinal infection

636 / 1500
У пацієнта неодноразово відзначалися напади короткочасного <<відключення>> свідомості, на мову звернену до нього не реагував, дивився прямо перед собою. Який метод обстеження необхідно призначити пацієнтові? The patient had repeated bouts of short-term ``disconnection'' of consciousness, he did not respond to the speech addressed to him, he looked straight ahead. What examination method should be prescribed to the patient?

Реоенцефалографія Rheoencephalography

Електроенцефалографія Electroencephalography

Ультразвукова доплерографія Ultrasound dopplerography

Люмбальна пункція Lumbar puncture

Ехоенцефалоскопія Echoencephaloscopy

637 / 1500
Пацієнтка віком 47 років скаржиться на вкрай неприємні відчуття в ділянці живота: свербіж, поколювання, печіння. Пояснює це тим, що в неї <<дірка в шлунку>>, просить її обстежити. Під час об'єктивного дослідження жодної соматичної патології не виявлено. Яке порушення сприйняття у пацієнтки? A 47-year-old patient complains of extremely unpleasant sensations in the abdomen: itching, tingling, burning. She explains this by the fact that she has a <> , asks her to be examined. During the objective examination, no somatic pathology was detected. What kind of perception disorder does the patient have?

Парестезії Paraesthesia

Синестезії Synesthesia

Сенестопатії Senestopathy

Ілюзії Illusions

Гіперестезії Hyperesthesia

638 / 1500
Мати хлопчика 6-ти місяців прийшла на прийом до педіатра стурбована тим, що її дитина не отримувала жодних щеплень. Скарг не мають. Об'єктивно: температура - 37,1^oC, частота дихання - 20/хв., артеріа-льний тиск - 100/70 мм рт.ст. Зріст та вага відповідають діапазону між 50 та 75 перцентилем. Відповідно до нормативно-правових документів МОЗ, якої тактики має дотримуватися лікар щодо імунізації цієї дитини? The mother of a 6-month-old boy came to see a pediatrician worried that her child had not received any vaccinations. They have no complaints. Objectively: temperature - 37 ,1^oC, respiratory rate - 20/min., arterial pressure - 100/70 mm Hg. Height and weight correspond to the range between 50 and 75 percentile about the immunization of this child?

Ввести вакцину БЦЖ Enter BCG vaccine

Провести пробу Манту та записати на прийом через 2 дні Conduct a Mantoux test and make an appointment in 2 days

- -

Ввести вакцину БЦЖ, КПК та АКДС Inject BCG, CPC and DPT vaccine

Відкласти вакцинацію та записати на прийом через тиждень Postpone the vaccination and make an appointment in a week

639 / 1500
Породілля на 11-ту добу після пологів поскаржилась на різкий біль у лівій молочній залозі, підвищення температури тіла до 39,0^oC. Патології з боку внутрішніх та статевих органів не виявлено. Ліва молочна залоза гаряча на дотик, болюча. У верхньо-зовнішньому квадранті шкіра гіперемована, дещо ціанотична, набрякла. При пальпації визначається інфільтрат 6х8 см з флуктуацією посередині. Ваша тактика ведення: On the 11th day after giving birth, the woman in labor complained of sharp pain in the left mammary gland, an increase in body temperature to 39.0^oC. Internal and genital pathologies No organs are detected. The left mammary gland is hot to the touch. The skin is hyperemic, slightly swollen. On palpation, an infiltrate of 6x8 cm is detected. Your management tactics:

Рекомендувати зціджування молока Recommend expressing milk

Хірургічне лікування маститу Surgical treatment of mastitis

Рекомендувати продовжити грудне годування Recommend to continue breastfeeding

Припинити лактацію агоністами дофаміну Stop lactation with dopamine agonists

Антибактеріальна терапія Antibacterial therapy

640 / 1500
Пацієнтка віком 60 років скаржиться на обмеження рухливості в дистальних міжфалангових суглобах обох кистей впродовж 12 років, періодичний біль у спині. Об'єктивно спостерігається: вузлуваті потовщення дистальних міжфалангових суглобів обох кистей, пальці деформовані, рухливість обмежена. Іншої патології не виявлено. Аналізи крові і сечі в межах норми. Який найімовірніший діагноз? A 60-year-old patient complains of limited mobility in the distal interphalangeal joints of both hands for 12 years, periodic back pain. Objectively observed: nodular thickening of the distal interphalangeal joints both hands, fingers are deformed, no other pathology is detected. Blood and urine tests are within normal limits?

Остеоартроз Osteoarthrosis

Реактивний артрит Reactive arthritis

Подагра Gout

Ревматоїдний артрит Rheumatoid arthritis

Анкілозуючий спонділоартрит Ankylosing spondylitis

641 / 1500
Мати п'ятирічної дитини звернулася до лікаря-гінеколога з приводу появи у дівчинки відчуття печіння у піхві та сильного свербежу. З анамнезу відомо, що дитина приймала антибіотики для лікування пневмонії. Під час огляду виявлено: гіперемія та набряк вульви, густі, білі, сирнисті виділення без запаху. Яке ускладнення виникло у дівчинки? The mother of a five-year-old child consulted a gynecologist about the girl's burning sensation in the vagina and severe itching. It is known from the anamnesis that the child took antibiotics for treatment pneumonia. During the examination, hyperemia and swelling of the vulva, thick, white, cheesy discharge were found. What complication did the girl have?

Бактеріальний вульвовагініт Bacterial vulvovaginitis

Гонорейний вульвовагініт Gonorrhea vulvovaginitis

Трихомонадний вульвовагініт Trichomonas vulvovaginitis

Бактеріальний вагіноз Bacterial vaginosis

Кандидозний вульвовагініт Candida vulvovaginitis

642 / 1500
У дівчинки віком 14 років з'явилися лихоманка в ранкові години, хейліт, стоматит, фотосенсибілізація, лейкоцитоз до 24·10^9/л, тромбоцитопенія. В ході лабораторних досліджень виявлено антинуклеарні антитіла у високому титрі. Встановіть попередній діагноз. A 14-year-old girl developed fever in the morning hours, cheilitis, stomatitis, photosensitivity, leukocytosis up to 24·10^9/l, thrombocytopenia. During laboratory studies revealed antinuclear antibodies in a high titer. Make a preliminary diagnosis.

Системна склеродермія Systemic scleroderma

Сепсис Sepsis

Системний червоний вовчак Systemic lupus erythematosus

Ювенільний ідіопатичний артрит Juvenile idiopathic arthritis

Дерматоміозит Dermatomyositis

643 / 1500
Чоловік віком 64 роки впав на лівий бік на підлогу. Об'єктивно спостерігається: вкорочення лівої кінцівки, зовнішня ротація стегна, симптом прилиплої п'яти. Під час пальпації та постукування по п'яті пацієнт відчуває біль у кульшовому суглобі. Який імовірний діагноз? A 64-year-old man fell on his left side on the floor. Objectively observed: shortening of the left limb, external rotation of the hip, the symptom of stuck heel. During palpation and tapping the heel, the patient feels pain in the hip joint. What is the likely diagnosis?

Забій кульшового суглоба Hip contusion

Перелом верхньої третини стегнової кістки Fracture of the upper third of the femur

Перелом шийки стегна Fracture of femoral neck

Вивих стегна Dislocation of hip

Перелом великого вертлюга Fracture of the greater acetabulum

644 / 1500
Чоловіка 42-х років, шахтаря, через 12 годин звільнено з-під завалу. Об'єктивно: обидві гомілки і ступні блідого кольору. Пульсації периферійних судин немає. Чутливість та пасивні рухи в суглобах відсутні. Доставлений із джгутами на обох кінцівках. Які невідкладні заходи першої лікарської допомоги необхідно провести для запобігання міоглобінурії та гострої ниркової недостатності? A 42-year-old man, a miner, was freed from the rubble after 12 hours. Objectively: both lower legs and feet are pale in color. There is no pulsation of peripheral vessels. Tenderness and passive movements in the joints are absent. Delivered with tourniquets on both limbs. What emergency medical measures should be taken to prevent myoglobinuria and acute renal failure?

Зняти джгут, гіпербарична оксигенація Remove tourniquet, hyperbaric oxygenation

Джгут не знімати, дезінтоксикаційна терапія Do not remove tourniquet, detoxification therapy

Джгут не знімати, гіпербарична оксигенація Do not remove tourniquet, hyperbaric oxygenation

Зняти джгут, дезінтоксикаційна терапія Remove tourniquet, detoxification therapy

Джгут не знімати, ампутація кінцівок вище джгута Do not remove the tourniquet, amputation of limbs above the tourniquet

645 / 1500
Чоловік 68-ми років звернувся до лікаря зі скаргами на сильний головний біль, нудоту, відчуття дискомфорту за грудиною. Протягом 2-х років страждає на гіпертонічну хворобу. Свідомість збережена. Ознак неврологічного дефіциту немає. АТ- 220/100 мм рт.ст., ЧСС- 92/хв. ЕКГ: ритм синусовий, ознаки гіпертрофії лівого шлуночка. Яке ускладнення розвилося? A 68-year-old man consulted a doctor with complaints of a severe headache, nausea, a feeling of discomfort behind the sternum. He has been suffering from hypertension for 2 years. Consciousness preserved. No signs of neurological deficit. BP - 92/min. ECG, signs of left ventricular hypertrophy?

Розшаровуюча аневризма аорти Dissecting aortic aneurysm

Гостре порушення мозкового кровообігу Acute cerebrovascular accident

Неускладнений гіпертензивний криз Uncomplicated hypertensive crisis

Ускладнений гіпертензивний криз Complicated hypertensive crisis

Інфаркт міокарда Myocardial infarction

646 / 1500
Чоловік 39 років звернувся до лікаря зі скаргами на біль у лівій нижній кінцівці. Захворів 2 дні тому. Об'єктивно спостерігається: температура тіла - 37,8^oC, на внутрішній поверхні лівого стегна та гомілки наявні варикозно розширені підшкірні вени, шкіра над ними підвищеної температури, почервоніла. Симптоми Мозеса та Хоманса негативні. Який імовірний діагноз? A 39-year-old man consulted a doctor complaining of pain in the left lower limb. He fell ill 2 days ago. Objectively observed: body temperature - 37.8^oC , there are varicose veins on the inner surface of the left thigh, the skin above them is reddened. Moses' and Homans' symptoms are negative. What is the probable diagnosis?

Варикозне розширення підшкірних вен лівої нижньої кінцівки Varicose dilatation of the subcutaneous veins of the left lower limb

Гострий висхідний тромбофлебіт підшкірних вен лівої нижньої кінцівки Acute ascending thrombophlebitis of the subcutaneous veins of the left lower limb

Тромбоз гомілкових артерій зліва Thrombosis of left tibial arteries

Посттромботичний синдром лівої нижньої кінцівки Postthrombotic syndrome of the left lower limb

Гострий тромбоз глибоких вен лівої нижньої кінцівки Acute deep vein thrombosis of the left lower limb

647 / 1500
Вагітність І, 38 тижнів. Положення плоду поздовжнє, передлежить голівка, притиснута до входу в малий таз. Очікувана маса плода - 3500,0 г. Перейми через 5 хв., по 25-30 сек. Серцебиття плода ясне, ритмічне, 130/хв. Під час вагінального дослідження виявлено: шийка матки вкорочена до 1 см, цервікальний канал пропускає 1 п/п (2 см). Плідний міхур цілий. Яка тактика ведення пологів? Pregnancy I, 38 weeks. The position of the fetus is longitudinal, the head is in front, pressed against the entrance to the small pelvis. The expected weight of the fetus is 3500.0 g. Take it in 5 minutes ., for 25-30 seconds. The heartbeat is clear, 130/min. During the vaginal examination, the cervix is ​​shortened to 1 cm. The amniotic sac is intact gave birth?

Токолітична терапія Tocolytic therapy

Кесарів розтин Caesarean section

Пологи вести через природні пологові шляхи Give birth through the natural birth canal

Амніотомія Amniotomy

Стимуляція пологової діяльності Stimulation of labor activity

648 / 1500
Чоловік 62 років скаржиться на слабкість, жар у язиці, заніміння нижніх кінцівок. Об'єктивно встановлено: шкіра бліда з субіктеричним відтінком, язик яскраво-червоний, смакові сосочки згладжені. Печінка - +3 см. Під час гастроскопії виявлена атрофія слизової оболонки. У крові виявлено: еритроцити - 1,2·10^12/л, Нb - 56 г/л, КП - 1,4, макроцитоз, лейкоцити - 2,8·10^9/л, еозинофіли - 2%, паличкоядерні - 4%, сегментоядерні - 50%, лімфоцити - 42%, моноцити - 2%, ретикулоцити - 0,1%, тромбоцити - 120·10^9/л, ШОЕ - 36 мм/год, білірубін - 29 ммоль/л, непрямий - 26 ммоль/л. Який найбільш імовірний діагноз? A 62-year-old man complains of weakness, heat in the tongue, numbness of the lower extremities. Objectively established: the skin is pale with a subicteric shade, the tongue is bright red, taste buds Liver - +3 cm. Atrophy of the mucous membrane was detected in the blood: erythrocytes - 1.2·10^12/l, KP - 1.4, macrocytosis - 2. ,8·10^9/l, eosinophils - 2%, rod-nuclear cells - 4%, segmentonuclear cells - 50%, lymphocytes - 42%, monocytes - 2%, reticulocytes - 0.1%, platelets - 120·10^9/l , ESR - 36 mm/h, bilirubin - 29 mmol/l, indirect - 26 mmol/l. What is the most likely diagnosis?

Залізодефіцитна анемія Iron deficiency anemia

Гіпопластична анемія Hypoplastic anemia

Фолієво-дефіцитна анемія Folate deficiency anemia

В_12-дефіцитна анемія B_12-deficiency anemia

Гемолітична анемія Hemolytic anemia

649 / 1500
Породілля 25 років госпіталізована з переймами впродовж 12 годин. Перейми тривають 25 секунд з інтервалом mbox3-4-7 хвилин, нерегулярні, різко болючі, біль поширюється від нижнього сегмента матки вгору. Передлежить голівка плода, притиснута до входу в малий таз, матка в гіпертонусі. Внутрішнє акушерське дослідження виявило: шийка матки згладжена, розкриття маткового вічка 3 см. Плодовий міхур цілий. Поставте попередній діагноз: A 25-year-old woman in labor was hospitalized with seizures for 12 hours. Seizures last 25 seconds with an interval of mbox3-4-7 minutes, are irregular, sharply painful, pain spreads from the lower segment The uterus is up. The head of the fetus is pressed against the entrance to the pelvis. The internal obstetric examination revealed: the cervix is ​​smooth, the opening of the uterine cavity is intact. Make a preliminary diagnosis:

Вторинна слабкість пологової діяльності Secondary weakness of labor activity

Дискоординована пологова діяльність Discoordinated birth activity

Первинна слабкість пологової діяльності Primary weakness of labor activity

Дистоція шийки матки Cervical dystocia

Фізіологічний перебіг пологів Physiological course of childbirth

650 / 1500
Після багаторазової обробки операційного інструменту слабоконцентрованим розчином дезинфікуючої рідини у медичної сестри поступово з'явилось запалення шкіри долоневої поверхні кистей рук. Об'єктивно встановлено: долонні поверхні набряклі, на них визначається дифузна еритема без чітких меж, на фоні якої є поодинокі папульозні висипання. Шкірні алергічні проби негативні. Який попередній діагноз? After repeated treatment of the surgical instrument with a weakly concentrated solution of disinfectant liquid, the nurse gradually developed inflammation of the skin of the palmar surface of the hands. Objectively established: the palmar surfaces are swollen, on they are defined as diffuse erythema with isolated papular rashes. What is the previous diagnosis?

Токсидермія Toxidermy

Сенсибілізаційний дерматит Sensitizing dermatitis

Екзема Eczema

Хімічний опік Chemical burn

Контактний дерматит Contact dermatitis

651 / 1500
До гінекологічного відділення звернулася жінка 25 років із скаргами на підвищення температури тіла до 38,6^oС, біль внизу живота, дизуричні розлади. Захворіла 3 доби тому, коли після штучного аборту з'явилися вказані симптоми. Під час гінекологічного дослідження виявлено: шийка матки циліндрична, вічко закрите. Тіло матки дещо збільшене, болюче, м'яке. Додатки матки не пальпуються. Виділення гнійно-кров'яні. За результатами аналізу крові встановлено: лейкоцитоз зі зміщенням формули крові вліво, прискорена ШОЕ. Який діагноз найбільш імовірний? A 25-year-old woman came to the gynecological department with complaints of an increase in body temperature to 38.6°C, pain in the lower abdomen, dysuric disorders. She became ill 3 days ago, when after an artificial abortion, the following symptoms appeared: the cervix was closed, the uterus was slightly enlarged, and the uterine appendages were not palpable : leukocytosis with a shift of the blood formula to the left, accelerated ESR. What is the most likely diagnosis?

Піосальпінкс Piosalpinx

Гострий цистит Acute cystitis

Гострий ендометрит Acute endometritis

Гострий ендоцервіцит Acute endocervicitis

Гострий сальпiнгоофорит Acute salpingo-oophoritis

652 / 1500
Чоловік 42 років госпіталізований до клініки з діагнозом: бронхоектатична хвороба, загострення. На рентген знімках бронхоектази локалізовані в межах окремих сегментів однієї долі. Яка подальша тактика лікування? A 42-year-old man is hospitalized with a diagnosis of bronchiectasis, exacerbation. On X-rays, bronchiectasis are localized within separate segments of one lobe. What is the further treatment strategy?

Диспансерне спостереження Dispensary observation

Консервативне лікування Conservative treatment

Дихальна гімнастика Breathing gymnastics

Антибактеріальна терапія Antibacterial therapy

Хірургічне лікування Surgical treatment

653 / 1500
Хворий 45-ти років звернувся на прийом до гематолога зі скаргами на загальну слабкість, підвищення температури тіла, пітливість, збільшення шийних лімфатичних вузлів. Об'єктивно: температура тіла - 37,5^oC, шкіра бліда, суха, задньошийні лімфатичні вузли щільно-еластичної консистенції, до 2 см в діаметрі, рухомі. Серце та легені без особливостей. Гепатоспленомегалія. Яке дослідження необхідно провести хворому, щоб оцінити поширеність процесу? A 45-year-old patient consulted a hematologist with complaints of general weakness, increased body temperature, sweating, and enlarged cervical lymph nodes. Objectively: body temperature - 37.5°C, the skin is pale, the cervical lymph nodes are dense and elastic, up to 2 cm in diameter, and the heart and lungs are unremarkable.

УЗД шийних лімфатичних вузлів Ultrasound of cervical lymph nodes

Загальний аналіз крові General blood test

Рентгенографію ОЧП X-ray radiography

КТ KT

Остеосцинтіграфію Osteoscintigraphy

654 / 1500
Підліток 15-ти років напередодні був у контакті із хворим на кір. При огляді - здоровий. Не щеплений за переконаннями батьків. Які заходи рекомендовані для профілактики виникнення захворювання? A 15-year-old teenager was in contact with a measles patient the day before. On examination, he is healthy. He has not been vaccinated according to his parents' beliefs. What measures are recommended to prevent the occurrence of the disease?

Спостереження протягом 5 діб, при появі проявів захворювання - вакцинація проти кору Observation for 5 days, when symptoms of the disease appear - vaccination against measles

Вакцинація проти кору через 4 доби після контакту з хворим Measles vaccination 4 days after contact with the patient

Вакцинація проти кору протягом перших 72 годин після контакту з хворим Measles vaccination within the first 72 hours after contact with the patient

Антибіотики широкого спектру дії Broad-spectrum antibiotics

Ніякі None

655 / 1500
Семимісячна дитина шпиталізована до хірургічного відділення за 8 годин після захворювання зі скаргами на напади неспокою, біль у животі, одноразове блювання. Об'єктивно спостерігається: у правій половині живота пальпується пухлиноподібне утворення. Під час ректального дослідження виявлено кров у вигляді <<малинового желе>>. Яке захворювання можна припустити? A seven-month-old child was hospitalized in the surgical department 8 hours after the illness with complaints of anxiety attacks, abdominal pain, one-time vomiting. Objectively observed: in the right half of the abdomen a tumor-like formation is palpable. Blood in the form of 'raspberry jelly' was detected during the rectal examination. What disease can be assumed?

Подвоєння кишечника Intestine doubling

Глистну інвазію Worm infestation

Ентерокістому Enterocystoma

Інвагінацію кишечника Intussusception

Пухлину черевної порожнини Abdominal tumor

656 / 1500
Дівчина 17 років скаржиться на біль і набряк 2-го пальця правої руки. 3 доби тому зробила манікюр. Біль з'явився другого дня. Об'єктивно спостерігається: навколонігтьовий валик набряклий, гіперемований, нависає над нігтьовою пластинкою, болючий під час пальпації. Який діагноз найімовірніший? A 17-year-old girl complains of pain and swelling of the 2nd finger of her right hand. She had a manicure 3 days ago. The pain appeared on the second day. Objectively observed: the nail plate is swollen, hyperemic, overhanging the nail plate, painful during palpation. What is the most likely diagnosis?

Еризипелоїд Erysipeloid

Підшкірний панарицій Subcutaneous panaritium

Пароніхія Paronychia

Піднігтьовий панарицій Subungual panaritium

Шкірний панарицій Skin panaritium

657 / 1500
Новонароджена дитина народилася на 37 тижні вагітності. Дитина млява, шкіра іктерична, на шкірі тулуба незначний геморагічний висип, рефлекси пригнічені, виявлений ністагм та хоріоретиніт. Гепатоспленомегалія. В ПЛР крові виявлено ДНК цитомегаловірусу. Який етіотропний препарат є найбільш ефективним? A newborn baby was born at the 37th week of pregnancy. The baby is lethargic, the skin is icteric, there is a slight hemorrhagic rash on the skin of the body, reflexes are depressed, nystagmus and chorioretinitis are detected. Hepatosplenomegaly. In PCR cytomegalovirus DNA was detected in the blood. Which etiotropic drug is the most effective?

Ремантадин Remantadine

Ганцикловір Ganciclovir

Ацикловір Acyclovir

Озельтамівір Oseltamivir

Рибавірин Ribavirin

658 / 1500
Дівчинка 8 років після хвороби на гостру респіраторну інфекцію впродовж останніх 2 тижнів скаржиться на біль у ділянці серця, відчуття серцебиття. Об'єктивно спостерігається: фебрильна температура тіла, блідість шкірних покривів, розширення меж серця вліво, ЧСС - 142/хв., парні передсердні екстрасистоли, глухість серцевих тонів, неінтенсивний систолічний шум, максимальний у 5-й точці. Який попередній діагноз? An 8-year-old girl after an illness with an acute respiratory infection for the past 2 weeks complains of pain in the area of ​​the heart, a feeling of a heartbeat. Objectively observed: febrile body temperature, pallor of the skin, expansion of the heart's borders to the left, heart rate - 142/min., paired atrial extrasystoles, deafness of heart sounds, mild systolic murmur, maximum at the 5th point. What is the previous diagnosis?

Міокардит Myocarditis

Ендокардит Endocarditis

Вроджена вада серця Congenital heart defect

Функціональні зміни в серці Functional changes in the heart

Вегетативна дисфункція Vegetative dysfunction

659 / 1500
У пацієнта, який хворіє на хронічну хворобу нирок протягом останнього місяця з'явилися скарги на швидку втомлюваність, порушення сну, відчуття нудоти, свербіж шкіри. Об’єктивно спостерігається: свідомість загальмована, шкірні покриви бліді, одутловатість обличчя. Під час аускультації серця спостерігається: тони аритмічні, глухі, ЧСС - 105/хв, АТ - 170/100 мм рт. ст. Під час аускультації легень спостерігається: дрібно-пухирчасті хрипи в нижніх відділах. У біохімічному налізі крові виявлено: креатинін - 0,65 ммоль/л. Який рівень швидкості клубочкової фільтрації є показанням для замісної терапії? A patient suffering from chronic kidney disease complained of rapid fatigue, sleep disturbances, nausea, skin itching during the last month. Objectively observed : consciousness is inhibited, the skin is bloated. During auscultation of the heart, heart rate is 105/min, blood pressure is 170/100 mmHg departments. The biochemical analysis of blood revealed: creatinine - 0.65 mmol/l. What level of glomerular filtration rate is an indication for replacement therapy?

20 мл/хв 20 ml/min

35 мл/хв 35 ml/min

15 мл /хв 15 ml/min

30 мл/хв 30 ml/min

25 мл/хв 25 ml/min

660 / 1500
Для клінічно здорової дитини 6 місяців, яка перебуває на природному вигодовуванні потрібно призначити перший прикорм. Яку страву найбільш доцільно використати у цьому разі? For a clinically healthy child of 6 months who is on natural feeding, it is necessary to prescribe the first complementary food. Which dish is the most appropriate to use in this case?

Овочеве пюре Vegetable puree

Гречана каша Buckwheat porridge

Печиво Cookie

Терте яблуко Grated apple

Ячнева каша Barley porridge

661 / 1500
Який прилад використовується для визначення рівня природного освітлення? What device is used to determine the level of natural light?

Кататермометр Catathermometer

Актинометр Actinometer

Психрометр Psychrometer

Люксметр Luxmeter

Анемометр Anemometer

662 / 1500
Пацієнт скаржиться на біль в попереку, слабкість правої стопи, порушення ходіння. Під час огляду виявлено: болісна пальпація поперекових паравертебральних точок, позитивні симптоми кашльового поштовху та Ласега справа під кутом 70^o, ахіловий рефлекс справа відсутній, слабкість розгиначів правої стопи, погано стоїть на правій п'яті. У пацієнта діагностували попереково-крижовий радикуліт з парезом стопи справа. Яку групу лікарських засобів необхідно йому призначити? The patient complains of pain in the lower back, weakness of the right foot, impaired walking. During the examination, painful palpation of the lumbar paravertebral points, positive symptoms of cough impulse and Lasega on the right under at an angle of 70°, the Achilles reflex is absent, the extensors of the right foot are weak, the patient is diagnosed with lumbosacral radiculitis with paresis of the right foot.

Вітаміни Vitamins

Ноотропи Nootropics

Анальгетики Analgesics

Нестероїдні протизапальні Nonsteroidal anti-inflammatory drugs

Антихолінестеразні Anticholinesterases

663 / 1500
Чоловік 43 років скаржиться на значну задишку у спокої, відчуття тяжкості у грудях. Захворювання пов'язує з перенесеним 2 тижня тому грипом. Об'єктивно встановлено: положення вимушене - сидить, нахилившись у перед. Обличчя одутле, ціанотичне, шийні вени набряклі. Межі серця поширені в обидві сторони, тони серця глухі, ЧСС - 112/хв., АТ - 135/80 мм рт. ст. У крові виявлено: ШОЕ - 46 мм/год. На ЕКГ видно низький вольтаж, на рентгенограмі - трапецевидна тінь серця, ознаки застою у легенях. Який препарат треба вживати з метою попередження тампонади серця? A 43-year-old man complains of significant shortness of breath at rest, a feeling of heaviness in the chest. The disease is associated with the flu he experienced 2 weeks ago. Objectively established: the position is forced - sitting, leaning forward. The face is swollen, the neck veins are swollen, heart rate - 112/min., blood pressure - 135/80 mmHg. 46 mm/h. Low voltage is visible on the X-ray, a trapezoidal shadow of the heart, signs of stasis in the lungs. What drug should be used to prevent cardiac tamponade?

Фуросемід Furosemide

Гіпотиазід Hypothiazide

Еналаприл Enalapril

Метопролол Metoprolol

Верошпірон Verospiron

664 / 1500
Хворого 46-ти років турбують часті напади ядухи, порушення сну через пароксизми експіраторної задишки 3-5 разів на тиждень, обмеження фізичної активності. Об'єктивно: везикулярне дихання з подовженим видихом, сухі свистячі хрипи на видиху. Спірометрія: ОФВ1- 67%, позитивна проба з бета-2-агоністом, добові коливання ПОШ_вид більше 30%. Якою має бути контролююча терапія даного хворого? A 46-year-old patient is bothered by frequent attacks of dyspnea, sleep disturbances due to paroxysms of expiratory shortness of breath 3-5 times a week, physical activity limitations. Objectively: vesicular breathing with prolonged exhalation, dry wheezing on exhalation. Spirometry: FEV1- 67%, positive test with beta-2-agonist, daily fluctuations of POH_vid more than 30%. What should be the controlling therapy of this patient?

Фенспірид + сальбутамол Fenspiride + Salbutamol

Тіотропій + формотерол Tiotropium + formoterol

Іпратропій + фенотерол Ipratropium + Fenoterol

Флютиказон + сальметерол Fluticasone + Salmeterol

Іпратропій + сальбутамол Ipratropium + salbutamol

665 / 1500
У жінки вагою 70 кг відразу ж після народження посліду розпочалась маткова кровотеча. Під час огляду посліду виявлені дефекти плаценти та оболонок. Крововтрата досягла 400 мл і продовжується. Яка дія лікаря? A woman weighing 70 kg started uterine bleeding immediately after the birth of the litter. During the examination of the litter, defects of the placenta and membranes were found. Blood loss reached 400 ml and continues. What is the action doctor?

Ручна ревізія порожнини матки Manual revision of the uterine cavity

Уведення 800 мкг мізопростолу ректально Introduction of 800 μg of misoprostol rectally

Термінове УЗД матки Urgent uterine ultrasound

Уведення 10 ОД окситоцину внутрішньовенно Introduction of 10 units of oxytocin intravenously

Інструментальна ревізія порожнини матки Instrumental revision of the uterine cavity

666 / 1500
У дитини віком 2 роки відзначаються часті та тривалі респіраторні захворювання та панкреатогенна форма мальабсорбції. Було припущено муковісцидоз. Яке дослідження необхідно провести для підтвердження цього діагнозу? A 2-year-old child has frequent and long-lasting respiratory diseases and a pancreatogenic form of malabsorption. Cystic fibrosis was suspected. What research should be done to confirm this diagnosis?

Бронхоскопію Bronchoscopy

Вміст хлоридів в потовій рідині Chloride content in sweat fluid

Імунограму Immunogram

Каріотипування Karyotyping

Рентгенограму органів грудної клітки Rentgenogram of chest organs

667 / 1500
В слабкорозвинених країнах Африки та Латинської Америки у дітей грудного віку можливий розвиток захворювання, для якого характерне відставання у фізичному та психічному розвитку, атрофія м'язів, жирова дистрофія печінки, депігментація волосся, гормональні порушення. Зміни у стані здоров'я розвиваються у випадку переводу дитини на штучне вигодовування переважно вуглеводною їжею з нестачею тваринних білків. Дане захворювання називається: In underdeveloped countries of Africa and Latin America, infants may develop a disease characterized by physical and mental retardation, muscle atrophy, fatty liver disease , hair depigmentation, hormonal disorders. Changes in the state of health develop in the case of transferring a child to artificial feeding with mainly carbohydrate food with a lack of animal proteins. This disease is called:

Хвороба Кешана Keshan's disease

Хвороба Прасада Prasad Disease

Аліментарна дистрофія Alimentary dystrophy

Рахіт Rickets

Квашіоркор Kwashiorkor

668 / 1500
Під час огляду триденної дитини виявлено криваве блювання, петехіально-плямистий висип, часті рідкі випорожнення чорного кольору. У крові виявлено: еритроцити - 4,2·10^12/л, Нb - 128 г/л, тромбоцити - 200·10^9/л, протромбіновий індекс - 40%. Результат тесту Апта: після додавання лугу колір розчину не змінився. Назвіть провідний механізм геморагічного синдрому у дитини: During the examination of a three-day-old child, bloody vomiting, petechial-spotted rash, frequent liquid black stools were found. In the blood, erythrocytes - 4.2·10^12 /l, Hb - 128 g/l, platelets - 200·10^9/l, prothrombin index - 40%. The result of the Apta test: the color of the solution did not change. Name the leading mechanism of the hemorrhagic syndrome in the child:

Інфекційний вплив на кістковомозкове кровотворення Infectious effect on bone marrow hematopoiesis

Наявність материнських антитіл до тромбоцитів дитини Presence of maternal antibodies to the child's platelets

Коагулопатія споживання Consumption coagulopathy

Низький рівень вітамін К-залежних факторів Low level of vitamin K-dependent factors

Низький рівень VIII і IX факторів згортання крові Low level of clotting factors VIII and IX

669 / 1500
15-річна дівчина доставлена до лікаря зі скаргами на біль у суглобах з обмеженням рухів, ранкову скутість. Зі слів пацієнтки все розпочалося з болю та припухання обох колінних суглобів. Протягом 2-х років подібні симптоми поступово розвивались у гомілкових, ліктьових, п'ястково-фалангових та проксимальних міжфалангових суглобах. При обстеженні лімфаденопатія, гепатоспленомегалія та висип відсутні. При лабораторному дослідженні ШЗЕ- 42 мм/год, антинуклеарні антитіла (ANA) не виявлено, ревматоїдний фактор не виявлено. Який діагноз є найбільш імовірним у цієї пацієнтки? A 15-year-old girl was taken to the doctor with complaints of pain in the joints with restriction of movement, morning stiffness. According to the patient, it all started with pain and swelling of both knee joints. Over the course of 2 years, similar symptoms gradually developed in the tibial, ulnar, and proximal interphalangeal joints. Lymphadenopathy, hepatosplenomegaly, and rash were absent during the laboratory examination, and antinuclear antibodies (ANA) were not detected , no rheumatoid factor was detected. What is the most likely diagnosis for this patient?

Системний червоний вовчак Systemic lupus erythematosus

Ювенільний ідіопатичний артрит Juvenile idiopathic arthritis

Анкілозивний спондилоартрит Ankylosing spondylitis

Реактивний артрит Reactive arthritis

Гостра ревматична лихоманка Acute rheumatic fever

670 / 1500
При огляді дитини на 4-ту добу життя в ділянці шиї, потилиці і сідниць з'явилися пухирці з серозно-гнійним вмістом. Стан задовільний, дитина активна, рефлекси новонароджених викликаються в повному обсязі, пуповина на стадії муміфікації, пупочна ділянка без особливостей. Яке захворювання можна припустити? When examining the child on the 4th day of life, blisters with serous-purulent contents appeared in the neck, back of the head, and buttocks. The condition is satisfactory, the child is active, reflexes newborns are caused in full, the umbilical cord is at the stage of mummification, the umbilical area is without features. What disease can be assumed?

Везикулопустульоз Vesiculopustulosis

Бульозний епідермоліз Epidermolysis bullosa

Пухирчатка новонароджених Neonatal pemphigus

Пітниця Potnitsa

Флегмона Phlegmon

671 / 1500
Чоловік 32 років госпіталізований із нападами гарячки, які супроводжувалися потовиділенням і повторювалися кожні 48 годин. 12 років тому проходив військову службу в Таджикистані, де хворів на малярію. Об'єктивно спостерігаються субіктеричність склер, блідість шкіри, збільшення печінки на 2 см та її ущільнення, збільшення селезінки на 5 см. Яке дослідження треба провести для уточнення і підтвердження діагнозу? A 32-year-old man was hospitalized with febrile attacks, which were accompanied by sweating and recurred every 48 hours. 12 years ago, he served in the military in Tajikistan, where he suffered from malaria. Ob' subicteric sclera, pallor of the skin, enlargement of the liver by 2 cm and its compaction, enlargement of the spleen by 5 cm are objectively observed. What research should be conducted to clarify and confirm the diagnosis?

Біохімічний аналіз крові Biochemical blood analysis

Дослідження активності сироваткових ферментів АлАТ і АсАТ Study of activity of serum enzymes AlAT and AsAT

Мікроскопія товстої краплі та мазка крові Thick drop and blood smear microscopy

Серологічне дослідження крові Serological blood test

Визначення рівня білірубіну в сироватці крові Determination of bilirubin level in blood serum

672 / 1500
Пацієнтка віком 20 років скаржиться на підвищення температури тіла до 39^oC, головний біль, що локалізується в лобній ділянці, біль в очних яблуках, світлобоязнь, біль у м'язах та сухий кашель. Симптоми з'явилися 2 дні тому. Об'єктивно спостерігається: стан важкий, обличчя гіперемоване, слизова ротоглотки гіперемована, зерниста, судини розширені. Пульс - 96/хв, ритмічний, тони серця послаблені. Аускультативно в легенях прослуховуються розсіяні сухі хрипи. Менінгеальні симптоми відсутні. У загальному аналізі крові виявлено: лейкоцити - 3·10^9/л, еозинофіли - 1%, паличкоядерні - 6%, сегментоядерні - 51%, лімфоцити - 35%, моноцити - 7%. Вкажіть імовірний діагноз. A 20-year-old patient complains of an increase in body temperature up to 39^oC, a headache localized in the frontal area, pain in the eyeballs, photophobia, pain in the Cough and dry cough. Objectively, the condition is severe, the mucous membrane of the oropharynx is hyperemic, the pulse is 96/min, the heart sounds are auscultated scattered dry wheezes. The general blood analysis revealed: leukocytes - 1%, eosinophils - 6%, lymphocytes - 35%, monocytes - 7% probable diagnosis.

Кір Measles

Грип Flu

Пневмонія Pneumonia

Висипний тиф Typhoid

Менінгококова інфекція Meningococcal infection

673 / 1500
Дитині 2,5 років. Хворіє другу добу. Захворювання почалось з підвищення температури до 37,8^oC, одноразового блювання, водянистої діареї до 5 разів на добу. Сьогодні блювання було двічі, температура тіла - 38,0^oC, апетит знижений, продовжується водяниста діарея. З чого потрібно почати лікування дитини? The child is 2.5 years old. He has been ill for the second day. The illness began with an increase in temperature to 37.8^oC, one-time vomiting, watery diarrhea up to 5 times a day. Vomiting occurred twice today, the body temperature is 38.0°C, the appetite is reduced, watery diarrhea continues. What is the need to start treating the child?

Призначити ніфуроксазид Prescribe nifuroxazide

Призначити поліміксин Prescribe polymyxin

Призначити оральну регідратацію Prescribe oral rehydration

Призначити лоперамід Prescribe loperamide

Призначити цефтріаксон Prescribe ceftriaxone

674 / 1500
У пацієнта віком 48 років, який хворіє на артеріальну гіпертензію, раптово виникло запаморочення, бiль у потиличнiй дiлянцi, нудота та світлобоязнь. Об'єктивно спостерігається: сопор, гiперемiя обличчя, АТ - 190/100 мм рт. ст., пульс - 70/хв, температура тiла 36,8^oС. Горизонтальний нiстагм. Сухожильнi рефлекси - без чіткої різниці сторін, посилені. Ригiднiсть потиличних м'язiв - 4 поперечні пальці, двобiчний симптом Кернiга (+). Який попереднiй дiагноз? A 48-year-old patient suffering from arterial hypertension suddenly developed dizziness, pain in the occipital region, nausea and photophobia. Objectively observed: sopor, hyperemia face, blood pressure - 190/100 mmHg, body temperature 36.8°C. bilateral Kernig sign (+). What is the previous diagnosis?

Субарахноїдальний крововилив Subarachnoid hemorrhage

Субдуральна гематома Subdural hematoma

Гостра гiпертонiчна енцефалопатiя Acute hypertensive encephalopathy

Абсцес головного мозку Brain abscess

Інсульт-гематома гіпертензивна Stroke-hematoma hypertensive

675 / 1500
Пацієнтка віком 13 років скаржиться на біль у ділянці серця, серцебиття, відчуття жару, запаморочення, слабкість. З анамнезу відомо: скарги з'явилися 2 місяці тому, схудла на 4 кг, погіршилася успішність в школі, сон. Об'єктивно спостерігається: плаксива, дратівлива, тремор пальців рук, шкіра підвищеної вологості, тургор тканин знижений. ЧСС - 104/хв. Під час пальпації виявлено: щитовидна залоза збільшена до зоба ІІІ ступеня, ущільнена, неболюча. Який найімовірніший діагноз? A 13-year-old patient complains of heart pain, palpitations, a feeling of heat, dizziness, weakness. It is known from the anamnesis: the complaints appeared 2 months ago, she lost weight by 4 kg, performance at school has deteriorated. Objectively observed: tearful, irritable fingers, increased tissue turgor. Heart rate - 104/min. During palpation, the thyroid gland is enlarged to III degree , compacted, painless. What is the most likely diagnosis?

Ревматична хорея Rheumatic chorea

Неревматичний кардит Non-rheumatic carditis

Вегето-судинна дисфункція Vegeto-vascular dysfunction

Дифузний токсичний зоб Diffuse toxic goiter

Ендемічний зоб Endemic goiter

676 / 1500
До лікаря в ОКЛ звернулась жінка 35 років зі скаргами на біль за грудиною, утруднення проходження твердої та рідкої їжі стравоходом, підвищену слюнотечу, зригування їжі, похудання. Такі симптоми турбують близько року після перенесеного психоемоційного стресу. Який попередній діагноз? A 35-year-old woman came to the doctor in OKL with complaints of pain behind the sternum, difficulty passing solid and liquid food through the esophagus, increased salivation, regurgitation of food, weight loss. Such symptoms bother about a year after the psycho-emotional stress. What is the previous diagnosis?

Песляопіковий рубцевий стеноз Post-acute cicatricial stenosis

Рак стравоходу Esophageal cancer

Атрезія стравоходу Esophageal atresia

Дивертикул стравоходу Esophageal diverticulum

Ахалазія стравоходу Esophageal achalasia

677 / 1500
Дівчина віком 18 років звернулася до сімейного лікаря з метою вибору методу контрацепції. Статевий розвиток відповідає віку. Зріст 150 см, вага 75 кг. Статевий партнер один. Менархе з 14 років. Менструальний цикл не регулярний, відмічає затримки до 3 місяців. Пацієнтка має надлишковий ріст волосся на стегнах, акне на підборідді та чолі. Який метод контрацепції найбільш прийнятний для цієї дівчини? An 18-year-old girl consulted a family doctor to choose a contraceptive method. Sexual development corresponds to age. Height 150 cm, weight 75 kg. One sexual partner. Menarche with 14 years old. Menstrual cycle is irregular, notices delays of up to 3 months. The patient has excessive growth of hair on the thighs, chin and forehead. Which method of contraception is most acceptable for this girl?

Метод лактаційної аменореї Method of lactational amenorrhea

Комбіновані оральні контрацептиви (КОК) Combined oral contraceptives (COCs)

Внітрішньоматкові контрацептиви Intrauterine contraceptives

Методи розпізнавання фертильності за календарем овуляції Methods of recognizing fertility by ovulation calendar

Прогестагенові ін’єкційні кoнтрацептиви Progestogen injectable contraceptives

678 / 1500
Хворий 35-ти років впродовж 14-ти років працює в ливарному цеху, де концентрація кварцевого пилу в 4 рази перевищує ГДК. Впродовж 4-х років його турбує кашель, задишка при фізичному навантаженні. При аускультації дихання ослаблене. Рентгенографія ОГК: емфізема, дрібноплямисті тіні по всім легеневим полям. Найбільш імовірний діагноз? A 35-year-old patient has been working in a foundry for 14 years, where the concentration of quartz dust is 4 times higher than the MPC. For 4 years, he has been troubled by a cough , shortness of breath during physical exertion. Breathing is weakened. X-ray: emphysema, small shadows in all lung fields?

ХОЗЛ COPD

Силікоз Silicosis

Хронічний бронхіт Chronic bronchitis

Туберкульоз легенів Pulmonary tuberculosis

Азбестоз Asbestosis

679 / 1500
Чоловік 36 років скаржиться на постійний тупий біль у лівому підребер'ї після прийому жирної та копченої їжі, блювоту, яка не приносить полегшення. Калові маси блискучі, з неприємним запахом. Хворіє протягом 8 років; зловживає алкоголем, багато курить. Об'єктивно встановлено: астенічної тілобудови, шкіра бліда і суха, язик з білим налітом, живіт помірно здутий. Під час пальпації - біль у зонах Шоффара, Губергриця-Скульського, точках Дежардена, Мейо-Робсона. Про яке захворювання можна подумати насамперед? A 36-year-old man complains of constant dull pain in the left hypochondrium after eating fatty and smoked food, vomiting that does not bring relief. The stools are shiny, with unpleasant He has been sick for 8 years, he is a heavy smoker, he has an asthenic body, a white plaque, and pain in the areas of Shoffar, Gubergritsa, and Skulsky. , Mayo-Robson. What disease can you think of first?

Хронічний холецистит у стадії загострення Chronic cholecystitis in the acute stage

Хронічний панкреатит у стадії загострення Chronic pancreatitis in the acute stage

Рак голівки підшлункової залози Cancer of the head of the pancreas

Виразкова хвороба у стадії загострення Ulcer disease in the acute stage

Хронічний гастродуоденіт у стадії загострення Chronic gastroduodenitis in the acute stage

680 / 1500
У жінки 75 років, яка веде помірно активний спосіб життя, під час профілактичного огляду виявлена сироваткова концентрація загального холестерину на рівні 5,1 ммоль/л (208 мг/дл) та ЛПВЩ - 70 мг/дл. ЕКГ без патології. Яка з наведених дієтичних рекомендацій показана жінці? In a 75-year-old woman who leads a moderately active lifestyle, during a preventive examination, a serum concentration of total cholesterol at the level of 5.1 mmol/l (208 mg/ dl) and HDL - 70 mg/dl. ECG without pathology. Which of the following dietary recommendations is shown to the woman?

Зменшити прийом холестерину Reduce cholesterol intake

Збільшити вживання клітковини Increase fiber intake

Зменшити прийом простих вуглеводів Reduce intake of simple carbohydrates

Зменшити прийом насичених жирів Reduce saturated fat intake

Ніяких змін у дієті No change in diet

681 / 1500
45-річний чоловік звернувся до лікаря зі скаргами на біль у горлі та лихоманку протягом 10-ти днів. В анамнезі: хворів на виразковий коліт та хронічний біль у спині. Викурює 1 пачку сигарет на день протягом 10-ти років. Батько пацієнта помер у 50 років від раку товстої кишки. При фізикальному обстеженні температура тіла - 38,6^oC, артеріальний тиск - 130/80 мм рт.ст., пульс - 72/хв., частота дихання - 18/хв., при огляді ротової порожнини глотка еритематозна, у криптах мигдаликів ексудат. Антигенний тест на СГА (стрептокок групи А) позитивний. Яка рекомендація лікаря зі вторинної профілактики онкологічних захворювань найбільш імовірно буде доречною цьому пацієнту? A 45-year-old man went to the doctor complaining of a sore throat and fever for 10 days. He had a history of ulcerative colitis and chronic back pain . Smokes 1 pack of cigarettes a day. The patient's father died at the age of 50. During the physical examination, the body temperature was 38.6 °C, the blood pressure was 130/80 mmHg. 72/min., respiratory rate - 18/min., when examining the oral cavity, the pharynx is erythematous, exudate in the tonsil crypts. The antigen test for SGA (streptococcus group A) is most likely to be appropriate for this patient ?

Зробити аналіз калу на приховану кров Do stool analysis for occult blood

Провести низькодозову КТ легень Perform a low-dose CT scan of the lungs

Провести колоноскопію Perform colonoscopy

Зробити ПСА-тест та пройти пальцеве ректальне дослідження Do a PSA test and undergo a digital rectal examination

Провести гнучку сигмоскопію Perform flexible sigmoscopy

682 / 1500
Пiд час профогляду у жiнки 50-ти рокiв у правiй молочнiй залозi виявлена пухлина дiаметром 5 см, щiльної консистенцiї, без чiтких контурiв. Шкiра над пухлиною має вигляд лимонної кiрки. У пахвовiй дiлянцi пальпується лiмфатичний вузол. Який найбiльш iмовiрний дiагноз? During the physical examination of a 50-year-old woman, a tumor with a diameter of 5 cm was found in the right mammary gland, with a dense consistency, without clear contours. The skin above the tumor has the appearance of a lemon peel A lymph node is palpated in the armpit. What is the most likely diagnosis?

Кiста молочної залози Breast gland cyst

Мастит Mastitis

Лiпома молочної залози Lipoma of the breast

Дифузна мастопатiя Diffuse mastopathy

Рак молочної залози Breast cancer

683 / 1500
Для вивчення фізичного розвитку дітей та підлітків використовують антропометричні дослідження. Який показник не відноситься до антропометричних? Anthropometric studies are used to study the physical development of children and adolescents. Which indicator is not anthropometric?

- -

Вимірювання маси тіла Body weight measurement

Вимірювання ЖЕЛ GEL measurement

Вимірювання обводу грудної клітки Chest measurement

Вимірювання зросту Height measurement

684 / 1500
До дерматолога звернувся чоловік віком 43 роки у зв'язку із наявністю висипки на шкірі кистей, яка супроводжує-ться свербежем. Хворіє 2 тижні. Об'єктивно: у вказаних місцях відзначається гіперемія та набряк шкіри, везикули, мокнуття, екскоріації на місцях розчухування. Який найбільш імовірний діагноз у хворого? A 43-year-old man consulted a dermatologist due to the presence of a rash on the skin of the hands, which is accompanied by itching. He has been ill for 2 weeks. Objectively: in hyperemia and swelling of the skin, vesicles, wetting, excoriations at the places of scratching are noted. What is the most likely diagnosis for the patient?

Екзема справжня Eczema is true

Екзема мікробна Microbial eczema

Дерматит алергічний Allergic dermatitis

Дерматит контактний Contact dermatitis

Токсикодермія Toxicoderma

685 / 1500
Пацієнт віком 76 років скаржиться на утруднене сечовипускання вночі, до 3-х разів, відчуття неповного випорожнення сечового міхура. Хворіє протягом 2-х років. Об'єктивно спостерігається: після акту сечовипускання перкуторно над лоном визначається притуплення. Симптом Пастернацького негативний. Зовнішні статеві органи без особливостей. Під час дослідження через пряму кишку: передміхурова залоза збільшена у 2-3 рази, гладка, еластична, симетрична з чіткими контурами. Слизова кишечника над нею рухома. Який найімовірніший діагноз? A 76-year-old patient complains of difficulty urinating at night, up to 3 times, a feeling of incomplete emptying of the bladder. He has been ill for 2 years. Objectively observed : after urination, dullness is determined. Pasternaksky's symptom is negative. During the examination through the rectum: the prostate gland is enlarged by 2-3 times. The mucous membrane above it is mobile . What is the most likely diagnosis?

Рак простати Prostate cancer

Хронічний простатит Chronic prostatitis

Абсцес передміхурової залози Prostate abscess

Туберкульоз простати Prostate tuberculosis

Аденома простати Prostate adenoma

686 / 1500
Чоловік 55 років звернувся до сімейного лікаря зі скаргами на слабкість, спрагу, сухість у роті, підвищення АТ. ІМТ - 35,6 кг/м^2, АТ - 140/90 мм рт. ст., шкіра суха, підшкірно жирова клітковина розподілена за абдомінальним типом. Глікемія натщесерце - 6,0 ммоль/л, постпрандіальна - 11,4 ммоль/л, HbA1c - 6,8%. Яка доцільна тактика лікування? A 55-year-old man went to the family doctor with complaints of weakness, thirst, dry mouth, increased BP. BMI - 35.6 kg/m^2, BP - 140/90 mmHg, subcutaneous fat distributed by abdominal type - fasting blood glucose - 11.4 mmol/L, HbA1c - 6.8% treatment?

Інсулінотеапія Insulin therapy

Призначити статини Prescribe statins

Призначити похідні сульфонілсечовини Assign sulfonylurea derivatives

Дієтотерапія та фізичні навантаження Diet therapy and exercise

Призначити метформін Prescribe metformin

687 / 1500
Чоловік 47 років хворіє 3 доби. У правому підребер'ї пальпується болючий запальний інфільтрат. Температура тіла - 38,9^oC. Під час сонографічного дослідження діагностовано калькульозний деструктивний холецистит. Клінічних та лабораторних даних на користь холедохолітіазу немає. Визначте тактику лікування хворого: A 47-year-old man has been ill for 3 days. A painful inflammatory infiltrate is palpated in the right hypochondrium. Body temperature is 38.9°C. During sonographic examination, calculous destructive Cholecystitis. There are no clinical and laboratory data in favor of choledocholithiasis. Determine the patient's treatment tactics:

Жовчогінні препарати, гепатопротектори, кортикостероїди Cholesterol drugs, hepatoprotectors, corticosteroids

Оперативне лікування - холецистектомія Operative treatment - cholecystectomy

Лапароцентез, дренування черевної порожнини Laparocentesis, drainage of the abdominal cavity

Спостереження, холецистектомія в разі розвитку перитоніту Observation, cholecystectomy in case of peritonitis

Комплексна протизапальна терапія Complex anti-inflammatory therapy

688 / 1500
Чоловік 45 років скаржиться на появу на шкірі поперека та кінцівок безболісних вузликових елементів, схильних до периферичного росту та злиття. Хворіє 2 роки. Загострення переважно весною. В анамнезі вказано: у батька пацієнта було подібне ураження шкіри. Об'єктивно спостерігається: патологічні елементи представлені краплевидними та монетовидними вузликами і бляшками, укритими білими лусочками. Який попередній діагноз? A 45-year-old man complains of the appearance of painless nodular elements on the skin of the lower back and limbs, prone to peripheral growth and fusion. He has been ill for 2 years. Exacerbations are mainly in the spring. The anamnesis indicates : the patient's father had a similar skin lesion. Objectively, the pathological elements are represented by teardrop-shaped and coin-shaped nodules, covered with white scales. What is the previous diagnosis?

Себорейна екзема Seborrheic eczema

Червоний плескатий лишай Red lichen

Рожевий лишай Pink lichen

Псоріаз Psoriasis

Атопічний дерматит Atopic dermatitis

689 / 1500
Роділля 22-х років доставлена машиною швидкої допомоги зі скаргами на різко болючі перейми по 25-30 секунд, через 4-5 хвилин. Амніотична рідина не виливалася. Положення плода поперечне, серцебиття не страждає. Контракційне кільце різко болюче, розташоване косо на 10 см вище лона. Який найбільш імовірний діагноз? A 22-year-old woman in labor was delivered by ambulance with complaints of sharp painful contractions lasting 25-30 seconds, after 4-5 minutes. Amniotic fluid did not spill out. Conditions the fetus is transverse, the heartbeat does not suffer. The contraction ring is sharply painful, located obliquely 10 cm above the womb. What is the most likely diagnosis?

Дискоординована пологова діяльність Discoordinated birth activity

Загроза розриву матки Threat of uterine rupture

Тетанія матки Uterine tetany

Розрив матки, що відбувся Uterine rupture occurred

Надмірно бурхлива пологова діяльність Excessive labor activity

690 / 1500
Чоловік 22 років лежить у ліжку, високо підвівши голову, не відчуває незручності в такій позі. У бесіду вступає неохоче, реагує на шепітну мову, відповіді однослівні. Обличчя байдуже, гіпомімічне, чоло наморщене, губи витягнуті трубочкою. Рухається дуже мало, часто надовго застигає в малозручній позі. Такий стан розвинувся поступово, протягом тижня без видимих причин. Діагностуйте психічний стан хворого: A 22-year-old man lies in bed with his head raised high, he does not feel discomfort in this position. He enters the conversation reluctantly, responds to whispered speech, answers are monotonous. The face is indifferent , hypomimic, wrinkled forehead. Moves very little, often stays in an uncomfortable position for a long time. Diagnose the patient's mental state:

Психогенний ступор Psychogenic stupor

Екзогенний ступор Exogenous stupor

Депресивний субступор Depressive substupor

Кататонічний субступор Catatonic substupor

Апатичний субступор Apathetic substupor

691 / 1500
У пацієнта віком 17 років об'єктивно спостерігається: ріст волосся на обличчі відсутній, гінекомастія, відзначається відкладення жиру на стегнах, високий голос. Пацієнт високого зросту за рахунок подовжених нижніх кінцівок при відносно короткому тулубі. Відзначається розумова відсталість. У букальному епітелії виявлено статевий хроматин. Який найімовірніший діагноз? A 17-year-old patient is objectively observed: there is no facial hair growth, gynecomastia, there is fat deposition on the thighs, a high voice. The patient is tall due to elongated lower limbs with a relatively short trunk. Mental retardation is noted. Sexual chromatin is found in the buccal epithelium. What is the most likely diagnosis?

Синдром Едвардса Edwards Syndrome

Синдром Патау Patau syndrome

Синдром Дауна Down Syndrome

Синдром Шерешевського-Тернера Shereshevsky-Turner syndrome

Синдром Клайнфельтера Klinefelter syndrome

692 / 1500
Хвора 30-ти років скаржиться на свербіж шкіри, переважно у вечірній та нічний час. Хворіє 2 тижні. На шкірі міжпальцевих складок, молочних залоз, живота, сідниць та стегон множинні дрібнопапульозні та папуловезикульозні висипки, розташовані попарно, екскоріації. Висипки відсутні на обличчі та шиї. Подібну висипку має чоловік хворої. Найбільш імовірний діагноз: A 30-year-old patient complains of skin itching, mainly in the evening and at night. She has been ill for 2 weeks. On the skin of the interdigital folds, mammary glands, abdomen, buttocks and multiple small papular and papulovesicular rashes, located in pairs. There are no rashes on the face and neck. The patient's husband has the most likely diagnosis:

Епідермофітія Epidermophyta

Екзема Eczema

Короста Scabies

Герпес Herpes

Нейродерміт Neurodermatitis

693 / 1500
Вагітна віком 25 років шпиталізована у зв'язку з матковою кровотечею. Загальна крововтрата - 250 мл. Після проведеного обстеження встановлено: вагітність 10-11 тижнів, самовільний викидень в ходу. Укажіть лікувальну тактику у цьому разі. A 25-year-old pregnant woman was hospitalized due to uterine bleeding. The total blood loss was 250 ml. After the examination, it was established: pregnancy of 10-11 weeks, spontaneous abortion in move. Indicate the treatment tactics in this case.

Провести гемотрансфузію Perform blood transfusion

Провести видалення плодового яйця Remove the fetal egg

Призначити спазмолітики Prescribe antispasmodics

Призначити утеротонічні препарати Prescribe uterotonic drugs

Суворий ліжковий режим Strict bed rest

694 / 1500
Лікар швидкої допомоги під час надання допомоги пацієнту з кровотечею проколов палець. З'ясувалося, що пацієнт є ін'єкційним наркоманом, обстежувався на ВІЛ, результат не відомий. Яких заходів слід вжити лікарю? An emergency physician punctured a finger while treating a bleeding patient. It turned out that the patient is an injection drug user, he was tested for HIV, the result is unknown. What measures should the doctor take?

Обробити забруднену ділянку шкіри, обстежитися на ВІЛ, розпочати хіміопрофілактику Treat the contaminated skin area, get tested for HIV, start chemoprophylaxis

Обробити забруднену ділянку шкіри, увести донорський імуноглобулін Treat the contaminated skin area, inject donor immunoglobulin

Обробити забруднену ділянку шкіри, обстежитись на TORCH-інфекції, розпочати лікування антибіотиками Treat contaminated area of ​​skin, examine for TORCH infection, start antibiotic treatment

Нічого не робити Do nothing

Обробити забруднену ділянку шкіри, розпочати лікування противірусними препаратами та антибіотиками Treat the contaminated skin area, start treatment with antiviral drugs and antibiotics

695 / 1500
Жінка 39-ти років вперше звернулась до лікаря зі скаргами на значні менструації протягом 10-12 днів. Остання менструація почалась 3 тижні тому та триває дотепер. З анамнезу: цикл порушився 3 місяці тому, пологів - 2, абортів - 5. При дослідженні: шийка матки циліндричної форми, вічко зімкнуте. Матка та придатки без патологічних змін. Виділення кров'яні, надмірні. Яка тактика лікаря буде найбільш доречною? A 39-year-old woman first consulted a doctor with complaints of heavy menstruation for 10-12 days. The last menstruation began 3 weeks ago and continues until now. From the anamnesis: the cycle was broken 3 months ago, childbirth - 5. On examination: the cervix is ​​cylindrical, the eye is closed. The discharge is bloody, excessive. Which doctor's tactics will be most appropriate?

Надпіхвова ампутація матки Supravaginal amputation of the uterus

Гемостатична терапія Hemostatic therapy

Кріокоагуляція ендометрію Endometrial cryocoagulation

Гормональна терапія Hormonal therapy

Вишкрібання порожнини матки Uterine scraping

696 / 1500
Пацієнтка віком 32 роки скаржиться на виражену задуху, сухий кашель, підвищення температури тіла до 39^oС, пітливість. Під час бактеріоскопічного аналізу мокротиння виявлено: КСБ [+]. Проба Манту з 2 ТО - папула 21 мм. Під час рентгенологічно обстеження виявлено: в обох легенях візуалізуються численні симетрично розташовані вогнищеві тіні розмірами 1-2 мм, з нечіткими контурами, малої інтенсивності. Який найімовірніший діагноз? A 32-year-old patient complains of severe shortness of breath, a dry cough, an increase in body temperature up to 39°C, sweating. During bacterioscopic analysis of sputum, it was found: KSB [+] . Mantoux test with 2 TO - papule 21 mm. During the X-ray examination, numerous symmetrically located shadows of 1-2 mm size, with indistinct contours, are visualized. What is the most likely diagnosis?

Казеозна пневмонія Case pneumonia

Вогнищевий туберкульоз Focal tuberculosis

Міліарний туберкульоз легень Miliary pulmonary tuberculosis

Хронічний дисемінований туберкульоз легень Chronic disseminated pulmonary tuberculosis

Саркоїдоз Sarcoid

697 / 1500
58-річний чоловік 10 днів тому переніс помірний ішемічний інсульт в правій півкулі головного мозку (під час госпіталізації оцінка дефіциту за шкалою NIHSS становить 9 балів). Хворіє на помірну артеріальну гіпертензію, фібриляцію передсердь, сечокам'яну хворобу, подагру (ремісія). Що потрібно призначити пацієнту для вторинної профілактики інсульту? A 58-year-old man suffered a moderate ischemic stroke in the right hemisphere of the brain 10 days ago (at the time of hospitalization, the deficit score on the NIHSS scale was 9 points). He suffers from moderate arterial hypertension, atrial fibrillation, urolithiasis, gout (remission). What should be prescribed to the patient for secondary stroke prevention?

Клопідогрель в дозі 75 мг на добу Clopidogrel at a dose of 75 mg per day

Аспірин в дозі 100 мг на добу Aspirin in a dose of 100 mg per day

Високі дози (наприклад, аторвастатин - 80 мг на добу) ліпідознижувальних засобів High doses (for example, atorvastatin - 80 mg per day) of lipid-lowering drugs

Оральні антикоагулянти (варфарин або НОАК - дабігатран, рівароксабан, апіксабан) Oral anticoagulants (warfarin or NOAC - dabigatran, rivaroxaban, apixaban)

Препарати, що покращують мозковий кровообіг Drugs that improve cerebral circulation

698 / 1500
У жінки віком 26 років 6 місяців тому відбулися пологи. Звернулась в жіночу консультацію зі скаргами на відсутність менструацій. Дитина на грудному вигодовуванні. Під час вагінального обстеження виявлено: матка звичайних розмірів, щільної консистенції. Який найімовірніший діагноз? A 26-year-old woman gave birth 6 months ago. She went to a women's consultation with complaints about the absence of menstruation. The child is breastfed. During a vaginal examination, the following was found: uterus of normal size, dense consistency. What is the most likely diagnosis?

Синдром Шихана Sheehan Syndrome

Псевдоаменорея Pseudoamenorrhea

Вагітність Pregnancy

Фізіологічна аменорея Physiological amenorrhea

Синдром Ашермана Asherman syndrome

699 / 1500
У жінки 26-ти років відсутні менструа-ції. 10 місяців тому були другі пологи, в ранньому післяпологовому періоді була масивна гіпотонічна кровотеча. Грудьми не годує. Останнім часом схудла, почало випадати волосся, турбує нездужання. При гінекологічному обстеженні відзначається атрофія зовнішніх статевих органів, матка менша від норми, придатки не визначаються. Який найбільш імовірний діагноз? A 26-year-old woman has no menstruation. 10 months ago she had her second birth, in the early postpartum period there was massive hypotonic bleeding. She does not breastfeed. Recently she has lost weight, her hair has started to fall out, she is worried about malaise. During a gynecological examination, the uterus is smaller than normal, the appendages are not identified. What is the most likely diagnosis?

Синдром галактореї-аменореї Syndrome of galactorrhea-amenorrhea

Підозра на прогресуючу позаматкову вагітність Suspicion of progressive ectopic pregnancy

Фізіологічна аменорея Physiological amenorrhea

Синдром Штейна-Левенталя Stein-Leventhal syndrome

Синдром Шихана Sheehan Syndrome

700 / 1500
Хвора 35-ти років скаржиться на перебої в роботі серця, зниження ваги, біль передньої поверхні шиї, що почалися після ГРВІ. Пульс - 110/хв., АТ- 130/60 мм рт.ст. Щитоподібна залоза незначно збільшена, болюча при пальпації. ШОЕ- 40 мм/год. ТТГ- 0,05 мОд/л, вТ4 - 34,3 пмоль/л. Який найбільш імовірний діагноз? A 35-year-old patient complains of heart failure, weight loss, pain in the front surface of the neck, which began after SARS. Pulse - 110/min., BP - 130/60 mm Hg. Thyroid gland is slightly enlarged. ESR - 0.05 mU/l. What is the most likely diagnosis?

Тиреоїдит Хашимото Hashimoto's thyroiditis

Підгострий тиреоїдит Subacute thyroiditis

Дифузний токсичний зоб Diffuse toxic goiter

Пароксизмальна тахікардія Paroxysmal tachycardia

Гострий тиреоїдит Acute thyroiditis

701 / 1500
Дівчинка віком 14 років хворіє на хронічний гломерулонефрит, хронічну ниркову недостатність. У дівчинки продовжує прогресувати анемічний синдром. Який препарат для патогенетичної терапії цієї анемії необхідно призначити? A 14-year-old girl suffers from chronic glomerulonephritis, chronic kidney failure. The girl's anemic syndrome continues to progress. What drug should be prescribed for the pathogenetic therapy of this anemia?

Фолієва кислота Folic acid

Еритропоетин Erythropoietin

Ціанокобаламін Cyanocobalamin

Препарат заліза Iron drug

Еритроцитарна маса Erythrocyte mass

702 / 1500
При відходженні у жінки навколоплідних вод помічено, що вони значно забруднені меконієм. При народженні дитина не дихає, неактивна, шкіра ціанотична, забруднена меконієм, ЧСС- 98/хв. Яких реанімаційних заходів слід вжити після народження дитини? When a woman's amniotic fluid is discharged, it is noticed that it is significantly contaminated with meconium. At birth, the child is not breathing, inactive, the skin is cyanotic, contaminated with meconium, heart rate - 98/min What resuscitation measures should be taken after the birth of a child?

Внутрішньовенне введення адреналіну Intravenous administration of epinephrine

Тактильну стимуляцію новонародженого Tactile stimulation of the newborn

Санацію ВДШ гумовим балоном Sanitation of VDSH with a rubber balloon

ШВЛ маскою та мішком Амбу Ventilator with mask and Ambu bag

Пряму ларингоскопію, інтубацію, санацію трахеї Direct laryngoscopy, intubation, tracheal rehabilitation

703 / 1500
Третя доба після 1-х термінових, нормальних пологів. Дитина на сумісному перебуванні, природному вигодовуванні. Об'єктивно: загальний стан задовільний. Температура - 36,4^oC, пульс - 80/хв., артеріальний тиск - 120/80 мм рт.ст. Молочні залози м'які, безболісні. Лактація помірна, відтік молока не утруднений. Матка щільна, дно матки на 3 п/п нижче пупка. Лохії кров'янисто-серозні, у помірній кількості. Оцініть динаміку зворотного розвитку матки: The third day after the 1st term, normal delivery. The child is in a joint stay, natural feeding. Objectively: the general condition is satisfactory. Temperature - 36.4^ oC, pulse - 120/80 mmHg. Lactation is moderate, milk flow is not difficult bloody-serous, in moderate amount. Evaluate the dynamics of the reverse development of the uterus:

Лохіометра Lochiometer

Гематометра Hematometer

Фізіологічна інволюція Physiological involution

Субінволюція Subinvolution

Патологічна інволюція Pathological involution

704 / 1500
Під час сільськогосподарських робіт в полі тракторист отримав відкриту травму кисті. Планового повного курсу щеплення проти правцю не проходив. В чому полягатиме екстрена специфічна профілактика правцю в даному випадку? During agricultural work in the field, the tractor driver received an open hand injury. He did not complete the planned full course of vaccination against tetanus. What will be the emergency specific prevention of tetanus in this case?

1,0 мл АП-анатоксину, 3000 МО ППС 1.0 ml AP toxoid, 3000 IU PPP

1,0 мл АП-анатоксину, 50000 МО ППС 1.0 ml AP toxoid, 50000 IU PPP

3000 МО ППС 3000 MO PPS

0,5 мл АП-анатоксину, 3000 МО ППС 0.5 ml AP toxoid, 3000 IU PPS

Не проводиться Not held

705 / 1500
У повторнонароджуючої з початком термінових пологів з'явились кров'янисті виділення з піхви. Під час внутрішнього акушерського дослідження виявлено: шийка матки згладжена, відкриття вічка на 6 см, внутрішнє вічко на 1/3 перекрито губчастою тканиною. На решті ділянки пальпується плідний міхур. Пологова діяльність активна. Вкажіть подальшу тактику ведення пологів. With the onset of term labor, a woman who gave birth again had vaginal bleeding. During an internal obstetric examination, it was found that the cervix was smoothed, the opening of the eye was 6 cm, the inner eye is 1/3 covered by spongy tissue. The amniotic sac is palpable. Indicate the subsequent delivery tactics.

Амніотомія Amniotomy

Гемостатична терапія Hemostatic therapy

Кесарський розтин Caesarean section

Стимуляція пологової діяльності Stimulation of labor activity

Зняття активної пологової діяльності Withdrawal of active maternity activity

706 / 1500
Під час дослідження згортувальної системи крові пацієнта перед операцією виявлено дефіцит VIII фактора - антигемофільного глобуліну А. Яке захворювання у пацієнта? During the examination of the patient's blood coagulation system before the operation, a deficiency of factor VIII - antihemophilic globulin A was detected. What disease does the patient have?

Гемофілія В Hemophilia B

Гемофілія А Hemophilia A

Геморагічний ангіоматоз Hemorrhagic angiomatosis

Гемофілія С Hemophilia C

Геморагічний васкуліт Hemorrhagic vasculitis

707 / 1500
Під час проведення очисних робіт у шахті концентрація вугільного пилу становить 450 мг/м3 (ПДК - 10 мг/м^3). Розвиток якого професійного захворювання органів дихання можливе у гірників очисного забою? During cleaning operations in the mine, the concentration of coal dust is 450 mg/m3 (PDA - 10 mg/m^3). The development of which occupational respiratory disease is possible at the miners of the cleaning slaughter?

Сидерозу Siderosis

Алергічного ринофарингіту Allergic nasopharyngitis

Талькозу Talcos

Бісинозу Bisinosis

Антракозу Anthracosis

708 / 1500
До лікаря-гінеколога звернулася вагітна жінка (термін вагітності 20 тижнів), в якої було діагностовано гостру гонорею. Який лікарський препарат із нижченаведених потрібно застосовувати для лікування цієї пацієнтки? A pregnant woman (20 weeks pregnant) who was diagnosed with acute gonorrhea consulted a gynecologist. Which drug from the following should be used to treat this patient?

Бісептол Biseptol

Тетрациклін Tetracycline

Лікування антибіотиками провести після пологів Treatment with antibiotics should be carried out after childbirth

Стрептоміцин Streptomycin

Бензилпеніцилін Benzylpenicillin

709 / 1500
Вагітна 26-ти років хворіє на ревматизм із дитинства. Вагітність перша, термін - 10 тижнів. Із настанням вагітності стан здоров'я погіршився: посилилась задишка, з'явились набряки на нижніх кінцівках. Пульс - 86/хв., ціаноз губ, систолічний і діастолічний шуми на верхівці серця. Печінка виступає з-під реберної дуги на 4 см. Діагностовано СН ІІІ ступеня, фракція викиду лівого шлуночка менше 40%. Яка подальша тактика ведення вагітної? A 26-year-old pregnant woman has been suffering from rheumatism since childhood. It is her first pregnancy, the term is 10 weeks. With the onset of pregnancy, her health condition worsened: shortness of breath increased, with' there were swellings on the lower extremities. Pulse - 86/min., systolic and diastolic murmurs at the top of the heart. The liver protrudes from under the costal arch by 4 cm. The left ventricular ejection fraction is less than 40% tactics of managing a pregnant woman?

Медикаментозне переривання вагітності Medical abortion

Пролонгування вагітності, періодична госпіталізація до акушерського стаціонару в критичні терміни Prolongation of pregnancy, periodic hospitalization at an obstetric hospital in critical periods

Переривання вагітності шляхом малого кесаревого розтину Termination of pregnancy by small caesarean section

Переривання вагітності методом ва-куум-аспірації Termination of pregnancy by vacuum aspiration

Пролонгування вагітності з періодичним лікуванням у кардіологічному відділенні Prolongation of pregnancy with periodic treatment in the cardiology department

710 / 1500
Хворий 46-ти років госпіталізований в інфекційне відділення з діагнозом: гострий вірусний гепатит В. На цукровий діабет хворіє 9 років, дотримується діє-ти, отримує метформін - 2 г/добу, глімепірид - 2 мг вранці. Цукор крові - 7,3 ммоль/л. Гліколізований гемоглобін - 6,8%. Яка подальша тактика лікування хворого? A 46-year-old patient is hospitalized in the infectious disease department with a diagnosis of acute viral hepatitis B. He has been suffering from diabetes for 9 years, follows the diet, receives metformin - 2 g/day, glimepiride - 2 mg in the morning. Blood sugar - 7.3 mmol/l. Glycolized hemoglobin - 6.8%.

Перевести хворого на глібенкламід Transfer the patient to glibenclamide

Перевести хворого на інсулін Transfer the patient to insulin

Збільшити дозу метформіну Increase metformin dose

Збільшити дозу глімепіриду Increase glimepiride dose

Залишити терапію без змін Leave therapy unchanged

711 / 1500
Чоловіка віком 34 роки шпиталізовано до неврологічного відділення зі скаргами на інтенсивний головний біль, двоїння при погляді прямо, світлобоязнь. Захворів гостро, напад стався під час підняття ваги. Об'єктивно спостерігається: оглушення, помірна розбіжна косоокість, диплонія. Позитивні симптоми Керніга з обох боків. Парезів немає. Ліквор геморагічний. Який лікарський засіб необхідно призначити насамперед? A 34-year-old man was hospitalized in the neurological department with complaints of intense headache, diplopia when looking straight ahead, photophobia. He became acutely ill, the attack occurred while lifting weights. About objectively observed: strabismus, positive Kernig's symptoms on both sides. What drug should be prescribed first?

Ацетилсаліцилову кислоту Acetylsalicylic acid

Нікотинову кислоту Nicotinic acid

Гепарин Heparin

varepsilon-амінокапронову кислоту varepsilon-aminocaproic acid

Глютамінову кислоту Glutamic acid

712 / 1500
Хвора 37-ми років звернулась зі скаргами на виражену загальну слабкість, набряк обличчя, кистей, швидку втому під час ходьби, затруднене ковтання, перебої в роботі серця. Дані симптоми з'явились через 11 днів після відпочинку на морі. Об'єктивно: еритема обличчя, симптом ''окулярів'', набряк м'язів гомілки. Тони серця приглушені, АТ - 100/70 мм рт.ст. В крові: активність АсАТ- 0,95 ммоль/год·л, АлАТ- 1,3 ммоль/год·л, підвищена активність альдолази та креатинфосфокінази. Яке дослідження найбільш інформативне? A 37-year-old patient complained of pronounced general weakness, swelling of the face and hands, rapid fatigue while walking, difficulty swallowing, heart failure. Data Symptoms appeared after 11 days of rest at sea AsAT - 0.95 mmol/h·L, AlAT - 1.3 mmol/h·L, increased activity of aldolase and creatine phosphokinase. Which study is the most informative?

Електрокардіографія Electrocardiography

Визначення циркулюючих імунних комплексів Determination of circulating immune complexes

Електроміографія Electromyography

Гастродуоденофіброскопія Gastroduodenofibroscopy

Біопсія м'язів Muscle biopsy

713 / 1500
Пацієнт скаржиться на головний біль, утруднення носового дихання, гнійні виділення з носу протягом 3 місяців. 4 місяці тому йому лікували верхній премоляр справа. Рентгенологічно визначається затінення правої верхньощелеп-ної пазухи. Під час діагностичної пункції гайморової пазухи отримано велику кількість густого крихтоподібного гною з неприємним запахом. Який діагноз імовірний? The patient complains of a headache, difficulty in nasal breathing, and purulent discharge from the nose for 3 months. 4 months ago, he was treated with the right upper premolar. Radiographically, the shadowing of the right maxilla is determined During the diagnostic puncture of the maxillary sinus, a large amount of thick pus with an unpleasant odor was obtained. What is the probable diagnosis?

Хронічний гнійний одонтогенний гайморит Chronic purulent odontogenic sinusitis

Гострий гнійний гайморит Acute purulent sinusitis

Хронічний гнійний гайморит Chronic purulent sinusitis

Пухлина верхньощелепної пазухи Tumor of maxillary sinus

Хронічний атрофічний гайморит Chronic atrophic sinusitis

714 / 1500
Дівчинку 7-ми років доставлено до відділення невідкладної допомоги у важкому стані. Зі слів матері дитина почала скаржитися на біль у спині та шиї, після чого згодом їй було дуже важко вимовляти слова та випити склянку води. Об'єктивно: спастичність жувальних м'язів. Під час збору анамнезу лікар дізнався, що тиждень тому дитина гуляла надворі з друзями та уколола палець дерев'яною палицею, після якої у шкірі залишилася скіпка. Матір про інцидент завчасно повідомлена не була і видалила стороннє тіло декілька днів тому, коли випадково його помітила. Дівчинка не отримала жодного щеплення через особисті переконання матері. Скільки доз входить у первинний вакцинальний комплекс від збудника, що найбільш імовірно викликав захворювання у дівчинки? A 7-year-old girl was brought to the emergency department in critical condition. According to the mother, the child began to complain of back and neck pain, after which she was very it is difficult to pronounce words and drink a glass of water the incident was not reported in advance and she removed the foreign body a few days ago when she accidentally noticed it. The girl did not receive any vaccinations due to her mother's personal beliefs. How many doses are included in the primary vaccine complex that most likely caused the disease in the girl?

Дві Two

Три Three

Чотири Four

Шість Six

П'ять Five

715 / 1500
Жінка 49-ти років звернулася до лікаря зі скаргами на головний біль, припливи жару до голови, шиї, підвищену пітливість, серцебиття, підвищення артеріального тиску до 170/100 мм рт.ст., дратівливість, безсоння, плаксивість, послаблення пам'яті, рідкі мізерні менструації, збільшення маси тіла на 5 кг протягом останніх півроку. Який діагноз є найбільш імовірним? A 49-year-old woman consulted a doctor with complaints of headache, hot flushes to the head, neck, increased sweating, palpitations, blood pressure increase to 170/100 mmHg, irritability, insomnia, tearfulness, memory loss, scanty menstruation, weight gain of 5 kg during the last six months. What is the most likely diagnosis?

Передменструальний синдром Premenstrual syndrome

Соматоформний розлад Somatoform disorder

Посткастраційний синдром Post-castration syndrome

Артеріальна гіпертензія Hypertension

Клімактеричний синдром Climacteric syndrome

716 / 1500
Чоловікові 69 років проведена правостороня нефректомія. Інтраопераційна одномоментна крововтрата - 800 мл, після якої протягом 40 хвилин спостерігалась гіпотонія. Надалі розвинулась анурія. Укажіть вірогідну причину анурії: A 69-year-old man underwent right-sided nephrectomy. Intraoperative one-time blood loss was 800 ml, after which hypotension was observed for 40 minutes. Later, anuria developed. Specify the probable cause of anuria:

Хронічна ниркова недостатність Chronic renal failure

Гостра ниркова недостатність Acute renal failure

Обтурація сечового катетера Urinary catheter obstruction

Синдром дисемінованого внутришньосудинного згортання Disseminated intravascular coagulation syndrome

Тампонада сечового міхура Bladder tamponade

717 / 1500
У чоловіка маса тіла вище середньої норми на 40% при <<індексі Кетле>> - 29. Вкажіть ступінь ожиріння у пацієнта. The man's body weight is 40% above the average norm with <> - 29. Specify the degree of obesity in the patient.

Третій Third

Перший First

Другий Second

Четвертий Fourth

Надлишкова маса тіла Overweight

718 / 1500
У хлопця 19 років після лікування двобічної пневмонії з'явилися та наростали спрага, підвищення апетиту, зниження маси тіла, сухість у роті. Під час обстеження виявлено: рівень глікемії натще - 19,7 ммоль/л, глюкозурія - 2,8 г/л, ознаки ацетонурії. Яке лікування слід призначити? A 19-year-old boy, after treatment for bilateral pneumonia, developed and increased thirst, increased appetite, decreased body weight, dry mouth. During the examination, the following was found: blood glucose level fasting - 19.7 mmol/l, glucosuria - 2.8 g/l, signs of acetonuria. What treatment should be prescribed?

Розчин глюкози Glucose solution

Інсулін Insulin

Метформін Metformin

Бігуаніди Biguanides

Фізіологічний розчин Saline

719 / 1500
Чоловік 65 років раптово знепритомнів, посинів. Прід час обстеження встановлено: пульс та АТ на периферичних судинах не визначаються, тони серця не вислуховуються. На ЕКГ спостерігаєть-ся: нерегулярні, різної величини і форми хвилі, які реєструються без інтервалів. Які заходи необхідно провести насамперед? A 65-year-old man suddenly fainted, turned blue. During the examination, it was found that the pulse and blood pressure in the peripheral vessels were not determined, heart sounds were not heard. The ECG showed: Irregular waves of different sizes and shapes, which are recorded without intervals. What measures should be taken first?

Штучне дихання Artificial respiration

В/в введення адреналіну IV administration of epinephrine

В/в введення строфантину IV administration of strophanthin

Тимчасова кардіостимуляція Temporary cardiac stimulation

Електрична дефібриляція Electrical defibrillation

720 / 1500
У хлопчика 2 років із вираженими порушеннями у фізичному і психомоторному розвитку та деформаціями скелета запідозрено синдром Дебре-де Тоні-Фанконі. Які зміни в сечі підтверджують таке припущення? A 2-year-old boy with marked impairments in physical and psychomotor development and skeletal deformities is suspected of having Debre-de-Tony-Fanconi syndrome. What changes in urine confirm this assumption?

Галактозурія, лактозурія Galactosuria, lactosuria

Глюкозурія, гіпераміноацидурія Glucosuria, hyperaminoaciduria

Циліндрурія, бактеріурія Cylindruria, bacteriuria

Протеїнурія, гематурія Proteinuria, hematuria

Ацетонурія, ізостенурія Acetonuria, isosthenuria

721 / 1500
Пацієнт віком 27 років упродовж 5 років лікувався з приводу НВК. Чоловік звернувся зі скаргами на виражений біль у животі, що з'явився 3 години тому, гази не відходять. Об'єктивно спостерігається: сухий язик, живіт не бере участі в акті дихання, позитивні симптоми подразнення очеревини. Пульс - 120/хв. Встановіть імовірний діагноз. A 27-year-old patient was treated for CKD for 5 years. The man complained of severe abdominal pain that appeared 3 hours ago, gas does not pass Objectively observed: dry tongue, abdomen does not participate in the act of breathing. Pulse - 120/min. Establish a probable diagnosis.

Гостра шлунково-кишкова кровотеча Acute gastrointestinal bleeding

Інвагінація товстої кишки Intussusception of the colon

Гострий апендицит Acute appendicitis

Механічна кишкова непрохідність Mechanical intestinal obstruction

Перфорація товстої кишки. Перитоніт Perforation of the colon. Peritonitis

722 / 1500
Юнак 18 років під час госпіталізації до стаціонару скаржиться на загальну слабкість, підвищення температури тіла до 37,5^oC, зниження апетиту, нудоту, важкість у правому підребер'ї, зміну кольору сечі та калу. Хворіє протягом 5 днів. Об'єктивно спостерігається незнач-на жовтяниця шкіри та склер. Печінка виступає на 3 см, чутлива під час пальпації. Сеча темно-коричневого кольору, кал світлий. Уживає водопровідну воду. Який діагноз найімовірніший? An 18-year-old boy, during hospitalization, complains of general weakness, an increase in body temperature to 37.5^oC, decreased appetite, nausea, heaviness in the right hypochondrium' , change of color of urine and stool. Objectively, the liver is 3 cm thick. Urine is light brown the most likely diagnosis?

Лептоспіроз Leptospirosis

Вірусний гепатит А Viral hepatitis A

Малярія Malaria

Черевний тиф Typhoid

Вірусний гепатит В Viral hepatitis B

723 / 1500
Чоловік 45 років доставлений до клініки у тяжкому стані. До госпіталізації хворів протягом 3 тижнів пневмонією. Об'єктивно встановлено: шкіра та слизові темно-землистого кольору, температура тіла - 38^oС, задишка у спокої, дихання зліва різко ослаблено. Спостерігається кашель із рясним харкотанням. Рентгенологічно виявлено затемнення лівого геміторакса. Який діагноз найімовірніший? A 45-year-old man was brought to the clinic in serious condition. Before hospitalization, he had been ill for 3 weeks with pneumonia. Objectively established: dark earthy skin and mucous membranes, body temperature - 38°C, shortness of breath on the left side. There is a cough with profuse sputum. X-ray revealed the darkening of the left hemithorax. What is the most likely diagnosis?

Пухлина легені Lung tumor

Пневмоторакс Pneumothorax

Бронхіт Bronchitis

Емпієма плеври Empyema of the pleura

Пневмонія Pneumonia

724 / 1500
Пацієнтка віком 42 роки скаржиться на ранкову скутість у суглобах кистей, відчуття стягнутості шкіри обличчя, утруднення під час ковтання їжі. Об'єктивно спостерігається: амімія обличчя, звуження ротової порожнини по типу 'кисета', кінчики пальців рук бліді, холодні на дотик. Під час аускультації вислуховується: тони серця аритмічні, ослаблені, систолічний шум на верхівці. У аналізі крові виявлено: еритроцити - 3,2·10^12/л, лейкоцити - 6,7·10^9/л, ШОЕ - 35 мм/год. Який найімовірніший діагноз? A 42-year-old patient complains of morning stiffness in the joints of the hands, a feeling of tightness of the skin of the face, difficulty swallowing food. Objectively observed: facial emymia, narrowing of the mouth cavities of the 'pouch' type, fingertips are pale, cold to the touch. During auscultation, heart sounds are arrhythmic, weakened, systolic murmur at the apex. Blood analysis revealed: erythrocytes - 3.2·10^12/l - 6.7·10^9/l, ESR - 35 mm/h. What is the most likely diagnosis?

Гостра ревматична лихоманка Acute rheumatic fever

Склеродермія Scleroderma

Системний червоний вовчак Systemic lupus erythematosus

Ревматоїдний артрит Rheumatoid arthritis

Синдром Шегрена Sjogren's syndrome

725 / 1500
Хлопчик 2 років захворів гостро: скарги на підвищення температури тіла до 37,8^oС, осиплий голос, <<гавкаючий кашель>>, утруднене дихання. Об'єктивно встановлено: шкіра бліда, чиста. ЧД - 30/хв., інспіраторна задишка під час хвилювання. Над легенями жорстке дихання, хрипів немає. Перкуторно виявлено: легеневий звук. Рівень Са^+ в крові в нормі. У загальному аналізі крові спостерігається: лейкопенія, лімфоцитом. Насамперед слід думати про: A 2-year-old boy became acutely ill: complaints of an increase in body temperature up to 37.8^oС, hoarse voice, <>, difficulty breathing. Ob' objectively established: skin is pale, respiratory rate is 30/min. Over the lungs, there is no wheezing. The level of Ca^+ in the blood is normal. leukopenia, lymphocytoma. First of all, you should think about:

Гострий обструктивний бронхіт Acute obstructive bronchitis

Гострий обструктивний ларинготрахеіт Acute obstructive laryngotracheitis

Бронхіальна астма Bronchial asthma

Спазмофілія Spasmophilia

Гострий простий бронхіт Acute simple bronchitis

726 / 1500
Породілля 27-ми років, пологи ІІ, термінові, нормальні. 3 доба післяпологового періоду. Температура тіла - 36,8^oC, пульс - 72/хв., артеріальний тиск - 120/80 мм рт.ст. Молочні залози помірно нагрублі, соски чисті. Живіт м'який, безболісний. Дно матки на 3 п/п нижче пупка. Лохії кров'янисті, помірні. Який можна встановити діагноз? 27-year-old woman in labor, labor II, urgent, normal. 3 days postpartum. Body temperature - 36.8^oC, heart rate - 72/min. , blood pressure - 120/80 mm Hg. The mammary glands are soft, painless. The bottom of the uterus is bloody, moderate. ;

Фізіологічний перебіг післяпологового періоду Physiological course of the postpartum period

Субінволюція матки Subinvolution of uterus

Залишки плацентарної тканини після пологів Remains of placental tissue after childbirth

Післяпологовий метроендометрит Postpartum metroendometritis

Лактостаз Lactostasis

727 / 1500
Чоловіку 42-х років, з діагнозом спастичний парапарез призначено міорелаксант тизанідин за схемою поступового збільшення дози. Хворий порушив рекомендацію і вжив відразу добову дозу, після чого відчув загальну слабість, запаморочення, зниження сили в ногах та на короткий час втратив свідомість. При фізикальному обстеженні артеріальний тиск - 75/55 мм рт.ст., пульс - 69/хв., температура тіла - 36,6^oC, загальна м'язова гіпотонія. Який невідкладний стан найбільш імовірно виник у хворого? A 42-year-old man with a diagnosis of spastic paraparesis was prescribed the muscle relaxant tizanidine according to the scheme of gradually increasing the dose. The patient violated the recommendation and immediately took the daily dose, after which he felt general weakness , dizziness, loss of strength in the legs and lost consciousness for a short time. During physical examination, blood pressure - 75/55 mm Hg, pulse - 69/min, body temperature - 36.6°C, general muscle hypotonia What emergency condition most likely occurred in the patient?

Симпато-адреналовий криз Sympatho-adrenal crisis

Синкопальний стан Syncopal state

Транзиторна ішемічна атака Transient ischemic attack

Панічна атака Panic attack

Ваго-інсулярний криз Vago-insular crisis

728 / 1500
Жінка 35 років оперована з приводу гострого холециститу. Перебіг раннього післяопераційного періоду був нормальним, виписана в задовільному стані. За 3 тижні після операції звернулася повторно. Скарги на жовтушність склер, підвищення температури тіла до 38,8^oС, озноб, помірний біль у правому підребер'ї. Живіт м'який, перитонеальних симптомів немає. АЛТ - 2,3 ммоль/л. Білірубінемія - 66 ммоль/л через пряму фракцію. Визначте найраціональнішу тактику ведення хворої: A 35-year-old woman was operated on for acute cholecystitis. The course of the early postoperative period was normal, she was discharged in a satisfactory condition. She returned 3 weeks after the operation. Complaints of sclera yellowness , an increase in body temperature up to 38.8°C, moderate pain in the right hypochondrium. ALT - 2.3 mmol/l. Determine the most rational tactics of managing the patient:

Лапароскопічна ендоскопічна холедохоскопія Laparoscopic endoscopic choledochoscopy

Ендоскопічна ретроградна холангіографія, літоекстракція Endoscopic retrograde cholangiography, lithoextraction

3D-комп'ютерна томографія, дистанційна холедохолітотрипсія 3D computer tomography, remote choledocholithotripsy

Антибактеріальна, протизапальна терапія Antibacterial, anti-inflammatory therapy

Лапаротомія, ревізія гепатикохоледоха Laparotomy, revision of hepaticocholedochus

729 / 1500
Пацієнтка 22-х років із первинним безпліддям скаржиться на виділення молозива із молочних залоз, нерегулярні менструації. УЗД органів малого тазу: матка гіпоплаcтична, яєчники без особливостей. Рентген-дослідження турецького сідла: патології не виявлено. Концентрація пролактину у 3 рази перевищує норму. Рівні ФСГ, ЛГ, естрадіолу нижчі за норму. Кортизол, тестостерон відповідають нормі. Який медикаментозний засіб слід обрати для лікування цієї патології? A 22-year-old patient with primary infertility complains of colostrum discharge from the mammary glands, irregular menstruation. Pelvic ultrasound: hypoplastic uterus, ovaries without features. X-ray Examination of the Turkish saddle: no pathology is detected. FSH, estradiol levels are below the norm. Which drug should be chosen for the treatment of this pathology?

Естрогени Estrogens

Прогестагени Progestogens

Агоністи дофаміну Dopamine agonists

Глюкокортикоїди Glucocorticoids

Комбіновані оральні контрацептиви Combined oral contraceptives

730 / 1500
Виявлення яких структур під час патоморфологічного дослідження є патогномонічним для сказу? The detection of which structures during pathological examination is pathognomonic for rabies?

Тілець Негрі Taurus Negri

Тілець Талалаєва Talalayev Taurus

Кристалів Шарко-Лейдена Charcot-Leyden crystals

Тілець Гварнері Taurus Guarneri

Вузликів Ашофа Ashof's knots

731 / 1500
У пацієнтки віком 18 років після укусу бджоли спостерігається: набряк губи, обличчя, шиї, відчуття жару та задишка. Дихання утруднене, шумне, кашель. Шкіра бліда, холодна на дотик. АТ - 75/50 мм рт. ст. Пульс ниткоподібний - 98/хв. Тахікардія, тони серця глухі, ритмічні. Який найімовірніший діагноз? An 18-year-old female patient after a bee sting has: swelling of the lip, face, neck, feeling of heat and shortness of breath. Breathing is difficult, noisy, cough. The skin is pale, cold to the touch. Blood pressure - 75/50 mm Hg. Tachycardia, dull, rhythmic heart sounds?

Анафілактичний шок Anaphylactic shock

Гіпотонічний криз Hypotonic crisis

Набряк Квінке Quincke edema

Кропив'янка Hives

Астматичний статус Asthmatic status

732 / 1500
Жінка 52 років надійшла до стаціонару зі скаргами на підвищену кровоточивість зі слизових оболонок, обширні крововиливи в шкіру у вигляді екхімозів, плям, носові та шлункові кровотечі. Після клінічних обстежень було поставлено діагноз: тромбоцитопенічна пурпура. Яка найбільш вірогідна причина виникнення цього захворювання? A 52-year-old woman came to the hospital with complaints of increased bleeding from the mucous membranes, extensive skin hemorrhages in the form of ecchymoses, spots, nasal and stomach bleeding. After clinical examinations a diagnosis was made: thrombocytopenic purpura. What is the most likely cause of this disease?

Дефіцит заліза в сироватці крові, кістковому мозку і депо Iron deficiency in blood serum, bone marrow and depot

Утворення антитромбоцитарних аутоантитіл Formation of antiplatelet autoantibodies

Порушення гемостазу Disruption of hemostasis

Дефіцит VІІІ фактору згортання крові VIII blood coagulation factor deficiency

Спадкова недостатність плазмових факторів згортання крові Hereditary deficiency of plasma clotting factors

733 / 1500
Гірничий робітник очисного вибою 37 років після довготривалого вимушеного зігнутого положення тулуба у шахті відчув інтенсивний, стріляючого характеру біль у поперековому відділі хребта, який поширювався вниз у ліву ногу до підколінної ямки. Рухи хребта у поперековій ділянці різко обмежені. Позитивний симптом Ласега зліва. Пальпаторна болючість паравертебральних точок L5-S1. Сухожилкові рефлекси на нижніх кінцівках - знижений лівий ахіловий рефлекс. Гіпотонія м'язів лівого стегна і гомілки. Поставте попередній клінічний діагноз: A 37-year-old mine worker in a clean-out pit, after a long-term forced bent position in the mine, felt an intense, shooting pain in the lumbar spine, which spread down the left leg to the popliteal Spine movements are sharply limited. Palpatory tenderness of the L5-S1 points. Hypotonia of the left thigh and lower leg.

Перелом поперекового хребця Lumbar fracture

Спінальний інсульт Spinal stroke

Транзиторна ішемічна атака Transient ischemic attack

Попереково-крижова радикулопатія Lumbosacral radiculopathy

Ниркова коліка Renal colic

734 / 1500
Для Прикарпатського регіону притаманно постійно висока (більше 80%) вологість атмосферного повітря. У холодний період року за помірно низьких температур повітря населення цього регіону відчуває сильний холод. Який шлях тепловіддачі тоді збільшується? The Carpathian region is characterized by constantly high (more than 80%) atmospheric air humidity. In the cold period of the year, with moderately low air temperatures, the population of this region feels very cold. What a way heat transfer then increases?

Випромінювання Radiation

Радіація Radiation

Конвекція Convection

Випаровування Evaporation

Кондукція Conduction

735 / 1500
Пацієнт 20-ти років звернувся в клініку із скаргами на загальну слабкість, підвищення температури тіла, припухлість та болючість суглобів, висипання на гомілках. На рентгенограмі ОГК збільшені внутрішньогрудні лімфатичні вузли. Найбільш імовірний діагноз: A 20-year-old patient came to the clinic with complaints of general weakness, increased body temperature, swelling and pain in the joints, a rash on the lower legs. On the radiograph of the OGK, enlarged intrathoracic lymphatics nodes. The most likely diagnosis:

Лімфогрануломатоз Lymphogranulomatosis

Рак легень Lung cancer

Туберкульоз легень Pulmonary tuberculosis

Медіастиніт Mediastinitis

Саркоїдоз Sarcoid

736 / 1500
Після тривалого перебування на сонці у восьмирічної дівчинки розвинувся тепловий удар середнього ступеня тяжкості. У дитини об'єктивно спостерігається: виражений пульсуючий головний біль, гіпертермія до 37,5^oС, АТ - 110/65 мм рт. ст., гіперемія шкіри, рясне потовиділення, дратівливість. Яка тактика лікування дитини? After a long stay in the sun, an eight-year-old girl developed heat stroke of moderate severity. The child objectively observed: severe throbbing headache, hyperthermia up to 37.5^ oC, blood pressure - 110/65 mm Hg, hyperemia of the skin, profuse sweating. What are the tactics of treating the child?

Уведення розчину седуксену 0,5 мг/кг в/м Introduction of seduxen solution 0.5 mg/kg IV

Оксигенотерапія Oxygenotherapy

Уведення 2%-го розчину папаверину 1 мг/кг в/м Introduction of 2% papaverine solution 1 mg/kg IV

Пероральна регідратація Oral rehydration

Уведення розчину преднізолону 2 мг/кг в/м Introduction of prednisone solution 2 mg/kg IV

737 / 1500
Дівчина віком 17 років звернулася до сімейного лікаря зі скаргами на кволість, постійну стомлюваність, дратівливість, підвищену сонливість, погіршення концентрації, погіршення сутінкового зору. Своє харчування вважає задовільним, вживає хлібо-булочні, макаронні та ковбасні вироби, консерви, солодощі, каву, безалкогольні газовані напої. Добовий харчовий раціон фактично не містить гарячі страви, салати, свіжі фрукти та овочі, натуральні фруктові соки. Під час огляду дівчини лікар звернув увагу на кілька синців на руках, колінах, ціаноз носа, губ, нігтів, ясен, поодинокі петехіальні шкірні крововиливи, блідість і сухість шкіри, виражений гіперкератоз ліктьового суглоба, ороговіння волосяних фолікулів. Яких інгредієнтів, що викликали скарги та симптоми у дівчини, не вистачає у харчовому раціоні? A 17-year-old girl turned to her family doctor with complaints of frailty, constant fatigue, irritability, increased sleepiness, impaired concentration, impaired twilight vision. She considers her diet satisfactory, she consumes bakery products, pasta and sausages, canned goods, sweets, coffee, soft drinks. The daily diet does not include hot dishes, salads, fresh fruits and vegetables. During the examination of the girl, the doctor noticed several bruises on the hands, knees, cyanosis of the nose, lips, nails, gums, isolated petechial hemorrhages, paleness and dryness of the skin, pronounced hyperkeratosis of the elbow joint, keratinization of the hair follicles.

Білків Proteins

Са і Р Sa and P

Вітамінів D і К Vitamins D and K

Вітамінів групи В B vitamins

Вітамінів А і С Vitamins A and C

738 / 1500
У лікарні з середньорічною кількістю ліжок 330, лікувалося - 4250 пацієнтів, виписано - 4540 пацієнтів, померло в стаціонарі - 42 пацієнти. Який показник можна розрахувати? In a hospital with an average annual number of beds of 330, 4250 patients were treated, 4540 patients were discharged, 42 patients died in the hospital. What indicator can be calculated?

Загальну смертність Total mortality

Подобову летальність Daily mortality

Частоту розбіжності діагнозів Frequency of discrepancy of diagnoses

Лікарняну летальність Hospital mortality

Середнє число днів роботи ліжка на рік Average number of bed days per year

739 / 1500
27-річний хлопець вперше за останні декілька років звернувся до свого лікаря занепокоєний результатами ліпідограми. Він повідомив, що не хоче захворіти на атеросклероз та почав робити такий аналіз щорічно з 22-х років. Цього року у нього вперше зафіксовано показник (5,0 ммоль/л), що є вищим у порівнянні з попередніми чотирма роками (не більше 4,1 ммоль/л). Артеріальної гіпертензії, або ранньої ішемічної хвороби серця у сімейному анамнезі немає. Його бабуся хворіє на цукровий діабет II типу з 58-ми років. Його 64-річний дідусь переніс радикальну простатектомію з приводу раку простати. Окрім інформування щодо основних факторів ризику цереброваскулярної патології та рекомендацій по збалансованому харчуванню, який наступний крок у веденні пацієнта буде найбільш доречним? A 27-year-old guy went to his doctor for the first time in the last few years, worried about the results of a lipidogram. He said that he did not want to get atherosclerosis and began to do such an analysis annually from 22 years old. For the first time, he had an indicator (5.0 mmol/l), which is higher than in the previous four years (no more than 4.1 mmol/l). Arterial hypertension, or early ischemic heart disease no family history His grandmother has type II diabetes since age 58. His 64-year-old grandfather underwent a radical prostatectomy.In addition to informing about the main risk factors for cerebrovascular disease, what is the next step in management? patient will be most appropriate?

Пройти глюкозотолерантний тест Pass glucose tolerance test

Визначити рівень простатспецифічного антигену (ПСА-тест) Determine the level of prostate-specific antigen (PSA test)

Повторити визначення рівня загального холестерину через 5 років Retest total cholesterol level after 5 years

- -

Розпочати терапію статинами Start statin therapy

740 / 1500
Пацієнтка віком 45 років скаржиться на наявність ущільнення в лівій молочній залозі. Об'єктивно спостерігається: в верхньо-наружньому квадраті лівої молочної залози пухлиноподібне утворення приблизно 2,5 см в діаметрі без чітких контурів, рухоме, безболісне. Позитивний симптом 'лимонної кірки'. В лівій пахвовій ділянці збільшений до 1,5 см лімфовузол. Який найімовірніший діагноз? A 45-year-old patient complains of a lump in the left mammary gland. Objectively observed: in the upper-outer square of the left mammary gland, a tumor-like formation of approximately 2.5 cm in diameter without clear contours, painless. Positive symptom of 'lemon peel'. Lymph node enlarged to 1.5 cm. What is the most likely diagnosis?

Нелактаційний мастит Nonlactational mastitis

Рак молочної залози Breast cancer

Абсцес молочної залози Breast abscess

Ліпогранульома Lipogranuloma

Фіброаденома молочної залози Breast fibroadenoma

741 / 1500
У чоловіка 37 років 10 років тому був перелом середньої третини лівої стегнової кістки, після чого протягом останіх 7 років у ділянці колишнього перелому з'являється гостре запалення з утворенням нориці, через яку виділяється гнійний вміст із дрібними фрагментами кісткової тканини. Через деякий час нориця закривається. Про яке ускладнення перелому можна думати? A 37-year-old man had a fracture of the middle third of the left femur 10 years ago, after which acute inflammation with the formation of a fistula has appeared in the area of ​​the former fracture for the past 7 years , through which purulent contents with small fragments of bone tissue are released. After a while, the fistula closes. What complication of the fracture can be thought of?

Трофічна виразка Tropical ulcer

Флегмона м'яких тканин Phlegmon of soft tissues

Хронічний остеомієліт Chronic osteomyelitis

Туберкульоз кісток Bone tuberculosis

Несправжній суглоб False joint

742 / 1500
На четвертій добі життя у здорової новонародженої дитини з'явилися мелена та кривава блювота. За даними коагулограми виявлено подовжений протромбіновий час, знижений протромбіновий індекс, дефіцит ІІ, VІІ, ІХ і Х плазменних факторів. Назвіть найімовірніше захворювання у дитини. On the fourth day of life, a healthy newborn child had ground and bloody vomit. According to the coagulogram, prolonged prothrombin time, reduced prothrombin index, deficiency of II, VII, IX and X plasma factors. Name the most likely disease in the child.

Синдром дисемінованого внутрішньосудинного згортання Syndrome of disseminated intravascular coagulation

Геморагічна хвороба новонародженого Hemorrhagic disease of the newborn

Гемолітична хвороба новонародженого Hemolytic disease of the newborn

Гемофілія А Hemophilia A

Сепсис новонародженого Newborn sepsis

743 / 1500
Під часрозслідування випадку масового отруєння з'ясувалося, що у автомеханіків, які проводили випробування дизельних двигунів у боксі майстерні, де була зіпсована витяжна вентиляція, наприкінці робочого дня з'явились такі симптоми: головний біль, нудота, блювання, шум у вухах, лабільність пульсу. Об'єктивно виявлено, що шкіра та слизові оболонки мають вишнево-червоний колір. Який токсичний чинник став причиною масового отруєння автомеханіків? During the investigation of the case of mass poisoning, it was found that at the end of the working day, auto mechanics, who were testing diesel engines in the workshop box, where the exhaust ventilation was damaged, had the following symptoms appeared: headache, nausea, vomiting, ringing in the ears, lability of the pulse. Objectively, it was found that the skin and mucous membranes have a cherry-red color. What toxic factor caused the mass poisoning of auto mechanics?

Сірковуглець Carbon disulfide

Діоксид вуглецю Carbon dioxide

Оксид азоту Nitrogen oxide

Оксид вуглецю Carbon monoxide

Діоксид сірки Sulfur dioxide

744 / 1500
У приймальне відділення звернулися батьки з двомісячним хлопчиком, який напередодні ввечері впав із дивана на підлогу. Об'єктивно: дитина не контактує, млява, мали місце тоніко-клонічні судоми, в скроневій ділянці зліва напружена гематома. Яке із досліджень слід провести в першу чергу? Parents of a two-month-old boy who fell from the sofa to the floor the night before came to the reception department. Objectively: the child is unresponsive, lethargic, tonic-clonic seizures occurred convulsions, tense hematoma in the left temporal area. Which of the tests should be performed first?

Оглядова рентгенографія голови Overview X-ray of the head

Дослідження ліквору Liquor Research

Комп'ютерна томограма голови Computer tomography of the head

Іонограма сироватки крові Ionogram of blood serum

Електроенцефалографія Electroencephalography

745 / 1500
Пацієнтка віком 30 років скаржиться на раптове почервоніння правого ока, світлобоязнь, 'відчуття піску', сльозотечу, що з'явилися 3 дні тому, нежить, підвищення температури тіла до 38^oC. Об'єктивно спостерігається: виражений набряк повік, гіперемія кон'юнктиви перехідних складок, дрібні точкові крововиливи в кон'юнктиві верхньої повіки, незначні слизово-гнійні виділення. Який найімовірніший діагноз? A 30-year-old female patient complains of sudden redness of the right eye, photophobia, 'feeling of sand', lacrimation that appeared 3 days ago, runny nose, fever up to 38°C. Objectively observed: pronounced swelling of the eyelids, hyperemia of the conjunctiva of the transition folds, small point hemorrhages in the conjunctiva of the upper eyelid, minor purulent discharge. What is the most likely diagnosis?

Туберкульозний кон'юнктивіт правого ока Tuberculous conjunctivitis of the right eye

Герпетичний кон'юнктивіт правого ока Herpetic conjunctivitis of the right eye

Аденовірусний кон'юнктивіт правого ока Adenoviral conjunctivitis of the right eye

Бактеріальний кон'юнктивіт правого ока Bacterial conjunctivitis of the right eye

Алергічний кон'юнктивіт правого ока Allergic conjunctivitis of the right eye

746 / 1500
У жінки віком 38 років на тлі ясної свідомості виникли клонічні судоми у м'язах правої кисті, що поширилися на праву руку і тривали протягом декількох хвилин. Під час огляду пацієнтки після нападу спостерігається: центральний правобічний верхній монопарез. Укажіть тип епілептичного нападу. A 38-year-old woman developed clonic convulsions in the muscles of the right hand against the background of clear consciousness, which spread to the right hand and lasted for several minutes. During the examination after the attack, the patient has: central right-sided upper monoparesis. Specify the type of epileptic attack.

Генералізований тоніко-клонічний Generalized tonic-clonic

Простий фокальний Simple Focal

Генералізований абсанс Generalized absence

Фокальний міоклонічний Focal myoclonic

Фокальний моторний із усвідомленням (джексонівський) Focal motor with awareness (Jacksonian)

747 / 1500
Під час проведення медогляду в дитячому колективі у 30% дітей було виявлено надлишкову масу тіла. Які з наведених продуктів треба обмежити в раціоні дітей? During the medical examination in the children's team, 30% of the children were found to be overweight. Which of the following products should be limited in the children's diet?

Овочі, фрукти Vegetables, fruits

Яйця Eggs

Кондитерські та борошняні вироби Confectionery and flour products

М'ясо, рибу та продукти з них Meat, fish and their products

Молоко, сир Milk, cheese

748 / 1500
Пацієнт 28-ми років скаржиться на періодичні болі в ногах та руках, мерзлякуватість стоп, зміну кольору 3 пальців кистей. Болі тривають вже протягом 1 року, але зміна кольору пальців виникла вперше. Він викурює по 20 сигарет в день протягом 12-ти років. Об'єктивно: шкіра ніг бліда, прохолодна, тургор знижений, гіпотрихоз. Який діагноз є найбільш імовірним? A 28-year-old patient complains of periodic pains in the legs and arms, frostbite of the feet, and discoloration of 3 fingers. The pain has lasted for 1 year, but the discoloration He has been smoking 20 cigarettes a day for 12 years. Objectively, the skin of the legs is pale, the turgor is reduced. What is the most likely diagnosis?

- -

Діабетична нейропатія Diabetic neuropathy

Системна склеродермія Systemic scleroderma

Облітеруючий атеросклероз Obliterating atherosclerosis

Облітеруючий тромбангіїт Thrombangiitis obliterans

749 / 1500
Чоловіку 40 років. Доставлений з місця катастрофи в оглушеному стані. Об'єктивно встановлено: відсутні активні рухи в лівих кінцівках, пригнічені сухожильні рефлекси з обох боків S>D, тонус м'язів знижений. Симптом Бабінського зліва. Затримка сечовипуску, брадикардія, що змінюється тахікардією. Ліквор прозорий, безбарвний, тиск - 300 мм вод. ст. На КТ спостерігається осередок підвищеної щільності округлої форми у правій тім'яно-скроневій ділянці, загальний об'єм - 60 см^3. Який імовірний ді-агноз? The man is 40 years old. He was brought from the scene of the accident in a stunned state. Objectively established: there are no active movements in the left limbs, depressed tendon reflexes on both sides S>D , muscle tone is reduced. Urinary retention, changing to tachycardia. Cerebrospinal fluid is 300 mm of water. On the CT scan, a center of increased density is observed in the right parietal region. total volume - 60 cm^3. What is the probable diagnosis?

Забій головного мозку тяжкого ступеню Severe cerebral contusion

Забій головного мозку середнього ступеню тяжкості Brain contusion of medium severity

Забій головного мозку легкого ступеню тяжкості Mild cerebral contusion

Дифузне аксональне ушкодження Diffuse axonal injury

Струс головного мозку Concussion

750 / 1500
У пацієнтки на другий день після тиреоїдектомії спостерігаються: судоми, 'оніміння' та 'повзання мурах' у кистях та стопах, позитивні симптоми Хвостека та Труссо. Яке ускладнення розвинулося у пацієнтки? On the second day after thyroidectomy, the patient has: convulsions, 'numbness' and 'ants crawling' in the hands and feet, positive Khvostek and Trousseau symptoms. What complication developed in the patient?

Гіпопаратиреоз Hypoparathyroidism

Тиреотоксична криза Thyrotoxic crisis

Травма гортанних нервів Laryngeal nerve injury

Залишкові явища тиреотоксикозу Residual symptoms of thyrotoxicosis

Гіпотиреоз Hypothyroidism

751 / 1500
Робітниці вуглезбагачувальної фабрики протягом 12-годинної робочої зміни виконують роботу стоячи. Маса вантажу, що піднімається і переміщується, становить 1-10 кг. Працівниці скаржаться на набряки, біль у нижніх кінцівках, які частіше з'являються в другій половині зміни. З яким захворюванням можна пов'язати характер праці та скарги робітниць? Coal beneficiation factory workers perform standing work during a 12-hour work shift. The weight of the load being lifted and moved is 1-10 kg. The workers complain of swelling, pain in the lower limbs, which more often appear in the second half of the shift. What disease can be associated with the nature of work and complaints of female workers?

Варикозне розширення вен Varicose veins

Артрит Arthritis

Подагра Gout

Міозит Myositis

Бурсит Bursitis

752 / 1500
Чоловік 36-ти років звернувся до лікаря зі скаргами на серцебиття, пітливість, схуднення на 5-6 кг за останні 3 місяці. Апетит збережений. При фізикальному обстеженні: артеріальний тиск - 130/60 мм рт.ст., пульс - 140/хв., шкіра волога, тепла. При пальпації рівномірне збільшення щитоподібної залози. Призначення якого дослідження буде найбільш доречним? A 36-year-old man consulted a doctor with complaints of palpitations, sweating, weight loss of 5-6 kg over the past 3 months. Appetite is preserved. During physical examination: blood pressure - 130/60 mmHg, pulse - 140/min. When palpating, the thyroid gland is uniformly enlarged. Which study would be most appropriate?

Рівень глюкози крові Blood glucose level

Загальний аналіз крові General blood test

Загальний аналіз сечі General urinalysis

Рівень тиреотропних гормонів у крові Level of thyroid-stimulating hormones in the blood

Рівень тропонінів у крові Level of troponins in blood

753 / 1500
Хвора 38-ми років звернулась до лікаря зі скаргами на слабкість, сонливість, біль у суглобах, збільшення ваги при зниженому апетиті, закрепи. Шкіра суха, потовщена. Обличчя одутле, амімічне. Очні щілини звужені, язик потовщений. Низький, дещо хрипкий голос. Тони серця - послаблені, пульс - 56/хв. Т4 вільний - знижений. Хворій треба постійно приймати: A 38-year-old patient turned to the doctor with complaints of weakness, drowsiness, joint pain, weight gain with reduced appetite, constipation. The skin is dry, thickened. Face Swollen, amicable. The eye slits are thickened. The heart sounds are weakened, the pulse is 56/min. The patient must take:

Тироксин Tyroxin

Карбонат літію Lithium Carbonate

Глюконат кальцію Calcium gluconate

Фуросемід Furosemide

Мерказоліл Mercazolil

754 / 1500
Наркоман 31-го року скаржиться на кашель з кровохарканням, задишку, персистуючу лихоманку, набряки нижніх кінцівок. Визначається набухання яремних вен, грубий пансистолічний шум над основою мечоподібного відростка та у другому міжребер'ї зліва, біля краю грудини. Серцеві тони ясні, аритмія, ЧСС- 128/хв., пульс - 82/хв., АТ- 100/70 мм рт.ст. Який найбільш імовірний діагноз? A 31-year-old drug addict complains of cough with hemoptysis, shortness of breath, persistent fever, swelling of the lower extremities. There is swelling of the jugular veins, a rough pansystolic murmur over the base of the xiphoid process and in the second intercostal space on the left, near the edge of the sternum. Heart sounds are clear, heart rate - 128/min., pulse - 100/70 mm Hg. What is the most likely diagnosis?

Коарктація аорти Coarctation of the aorta

Тромбоемболія легеневої артерії Thromboembolism of the pulmonary artery

Негоспітальна пневмонія Community-acquired pneumonia

Інфекційний ендокардит Infective endocarditis

Синдром Лютембаше Lutembache Syndrome

755 / 1500
Чоловік віком 52 роки звернувся зі скаргами на періодичні болі за грудиною, порушення проходження твердої їжі, схуднення на 5 кг за 3 тижні. Під час фіброезофагогастроскопії виявлено: вільна прохідність стравоходу до рівня 37,5-38 см від краю верхніх різців, де визначається верхній полюс екзофітного утворення, що звужує просвіт до 1/2 від норми. За результатами біопсії: аденокарцинома. Яке передракове захворювання найімовірніше у пацієнта? A 52-year-old man complained of periodic pains behind the sternum, impaired passage of solid food, weight loss of 5 kg in 3 weeks. Fibroesophagogastroscopy revealed: free passage esophagus to the level of 37.5-38 cm from the edge of the upper incisors, where the upper pole of the exophytic formation is determined, which narrows the lumen to 1/2 of the norm. According to the results of the biopsy: adenocarcinoma. What is the most likely precancerous disease in the patient?

Стан після опіку стравоходу Condition after esophageal burn

Стравохід Баррета Barrett's Esophagus

Поліп стравоходу Esophageal polyp

Ліпома стравоходу Esophageal lipoma

Дивертикул стравоходу Esophageal diverticulum

756 / 1500
У 72-річного чоловіка на сьомий день після операції хірургічної репозиції міжвертлюгового перелому стегна раптово з'явилися задишка та інтенсивний біль у лівій половині грудної клітки. Під час огляду спостерігається: шийні вени набряклі. Ціаноз. ЧДР - 26/хв. Аускультативно вислуховується: послаблене дихання над лівою легенею. ЧСС - 98/хв. АТ - 120/70 мм рт. ст. На комп'ютерній томограмі видно значне зменшення легеневого малюнку з лівого боку. Під час ехокардіографії ознак перевантаження правого шлуночка не виявлено. Який із наступних кроків є найдоцільнішим у цьому разі? A 72-year-old man suddenly developed shortness of breath and intense pain in the left half of the chest on the seventh day after surgical reduction of an intertrochanteric hip fracture. During the examination, Cyanosis - 26/min. Heart rate - 120/min side. No signs of right ventricular overload were detected during echocardiography. Which of the following steps is most appropriate in this case?

Установлення кава-фільтра Installing coffee filter

Уведення тромболітика в ліву легеневу артерію Introduction of a thrombolytic into the left pulmonary artery

Хірургічна емболектомія Surgical embolectomy

Призначення аспірина Aspirin Prescription

Призначення низькомолекулярного гепарину Prescription of low molecular weight heparin

757 / 1500
Пацієнта віком 28 років без постійного місця проживання шпиталізовано з попереднім діагнозом: грип. На 5 день хвороби з'явився розеолезно-петехіальний висип на тулубі та внутрішніх поверхнях кінцівок. Об'єктивно спостерігається: температура тіла - 40^oС, ейфорія, гіперемія обличчя, почервоніння склер, тремор язика, тахікардія, спленомегалія, збудження. Який найімовірніший діагноз? A 28-year-old patient without a permanent place of residence was hospitalized with a preliminary diagnosis of influenza. On the 5th day of the illness, a roseolae-petechial rash appeared on the trunk and inner surfaces of the limbs. Objectively observed: body temperature - 40°C, euphoria, facial hyperemia, redness of the sclera, tremor of the tongue, tachycardia, splenomegaly, excitement. What is the most likely diagnosis?

Черевний тиф

Висипний тиф Typhoid

Кір Measles

Алкогольний делірій Alcoholic delirium

Лептоспіроз Leptospirosis

758 / 1500
Жінка 45 років, яка рік тому перенесла резекцію щитоподібної залози з приводу багатовузлового зобу, скаржиться на загальну слабкість, сонливість, постійне відчуття втоми, зниження працездатності, закрепи, сонливість. Температура тіла - 36^oC. Шкірні покрови сухі, морщиністі, почало випадати волосся. Аменорея. Який стан виник у пацієнтки? A 45-year-old woman who underwent resection of the thyroid gland a year ago due to a multinodular goiter complains of general weakness, drowsiness, a constant feeling of fatigue, reduced work capacity, constipation, drowsiness The body temperature is 36°C. The skin is wrinkled, the hair has started to fall out. What condition did the patient have?

Хронічний тиреоідит Chronic thyroiditis

Клімакс Climax

Гіпопаратиреоз Hypoparathyroidism

Тиреотоксикоз Thyrotoxicosis

Гіпотиреоз Hypothyroidism

759 / 1500
Чоловік 41 року хворіє на виразку шлунку, яка 7 днів тому ускладнилась кровотечею. Турбує виражена слабкість, серцебиття, відзначається блідість шкіри та слизових. ЧСС - 98/хв. Результат налізу крові: Hb - 86 г/л, еритроцити - 3,0·10^12/л, КП - 0,86, тромбоцити - 420·10^9/л, ретикулоцити - 18%, лейкоцити - 4,0·10^9/л, еозинофіли - 2%, паличкоядерні - 6%, сегментоядерні - 52%, лімфоцити - 37%, моноцити - 3%. Яке ускладнення розвинулось у хворого? A 41-year-old man is suffering from a stomach ulcer, which 7 days ago was complicated by bleeding. He is concerned about severe weakness, palpitations, pallor of the skin and mucous membranes. Heart rate - 98/min. The result of the blood analysis: Hb - 86 g/l, erythrocytes - 3.0·10^12/l, CP - 0.86, platelets - 420·10^9/l, reticulocytes - 18%, leukocytes - 4.0· 10^9/l, eosinophils - 2%, eosinophils - 6%, segmentonuclear - 52%, lymphocytes - 37%, monocytes - 3%. What complication developed in the patient?

Хронічна залізодефіцитна анемія Chronic iron deficiency anemia

Лейкемоїдна реакція Leukemoid reaction

Набута гемолітична анемія Acquired hemolytic anemia

Постгеморагічна анемія Posthemorrhagic anemia

Мікросфероцитоз Microspherocytosis

760 / 1500
При проведенні профогляду робітниць промислового підприємства була виявлена група жінок з лабораторно підтвердженою залізодефіцитною анемією. Окрім медикаментозного лікування, які продукти, як основні носії засвоюваного заліза, повинен рекомендувати лікар для споживання? During a professional examination of female workers at an industrial enterprise, a group of women with laboratory-confirmed iron-deficiency anemia was identified. In addition to drug treatment, which products, as the main carriers of absorbed iron, should the doctor recommend for consumption?

Хліб та хлібобулочні вироби Bread and bakery products

Овочі та фрукти Vegetables and fruits

Хліб та рибу Loaves and fishes

Молоко та молокопродукти Milk and milk products

М'ясо та м'ясопродукти Meat and meat products

761 / 1500
Вагітна у терміні 32 тижня, вагітність II, скаржиться на лихоманку, озноб, нудоту, блювання, біль у попереку, дизурію. Симптом Пастернацького позитивний з обох боків. Аналіз сечі: піурія, бактеріурія. В крові: лейкоцитоз. Найбільш імовірний діагноз: Pregnant at 32 weeks, pregnancy II, complains of fever, chills, nausea, vomiting, lower back pain, dysuria. Pasternaksky's symptom is positive on both sides. Analysis urine: pyuria, bacteriuria. In the blood: leukocytosis. The most likely diagnosis:

Гестаційний пієлонефрит Gestational pyelonephritis

Безсимптомна бактеріурія Asymptomatic bacteriuria

Гострий цистит Acute cystitis

Хронічний гломерулонефрит Chronic glomerulonephritis

Гострий пієліт Acute pyelitis

762 / 1500
У пацієнта віком 58 років протягом 2-3 годин з'явилися множинні плями перед очима, після чого потемнішало перед правим оком. Під час обстеження виявлено: гострота зору становить 0,02 ексцентрично, зіниця помірно розширена, пряма реакція на світло знижена. В ході офтальмоскопії спостерігається: на очному дні множинні крововиливи різної величини і форми (симптом «розчавленого помідора»), диск зорового нерва набряклий, гіперемований. З анамнезу відомо про наявність гіпертонічної хвороби ІІ В ступеня. Який діагноз найімовірніший? A 58-year-old patient developed multiple spots in front of the eyes for 2-3 hours, after which it darkened in front of the right eye. During the examination, it was found that the visual acuity is 0.02 eccentrically, the pupil is moderately dilated, the direct reaction to light is reduced. During ophthalmoscopy, multiple hemorrhages of various sizes and shapes are observed (symptom of 'crushed tomato'), the optic nerve is swollen, hyperemic II B degree of disease. What is the most likely diagnosis?

Діабетична ретинопатія Diabetic retinopathy

Гіпертонічна ангіопатія Hypertensive angiopathy

Гіпертонічна ангіонейропатія Hypertensive angioneuropathy

Емболія центральної артерії сітківки Central retinal artery embolism

Тромбоз центральної вени сітківки Central retinal vein thrombosis

763 / 1500
Чоловік 60 років скаржиться на відчуття переповнення, розпирання в епігастрії після їжі, відрижку тухлим, блювоту їжею, з'їденою напередодні. В анамнезі вказана виразкова хвороба шлунка. Маса тіла знижена, шкіра бліда зі зниженим тургором та еластичністю. Опущення нижньої межі шлунку, болючість в епігастрії, позитивний симптом <<шуму плескоту>>. У крові виявлено: Нb - 102 г/л, К - 3,3 ммоль/л, Na - 134 ммоль/л. Яке ускладнення є найбільш імовірним? A 60-year-old man complains of a feeling of fullness, distension in the epigastrium after eating, belching rotten food, vomiting of food eaten the day before. The anamnesis indicates gastric ulcer disease. Mass the body is reduced, the skin is pale with reduced turgor and elasticity, the lower border of the stomach is lowered, pain in the epigastrium, a positive symptom of 'cluttering noise'. - 134 mmol/l. What complication is most likely?

Малігнізація Malignancy

Пенетрація виразки Ulcer penetration

Кровотеча з виразки Bleeding from an ulcer

Перфорація виразки Ulcer perforation

Пілородуоденальний стеноз Pyloroduodenal stenosis

764 / 1500
Чоловік 56 років протягом тривалого часу страждає на цироз печінки з ознаками портальної гіпертензії. Відмічалися явища кровотечі з варикозно розширенних вен стравоходу. Під впливом терапії кровотечу було зупинено. Який із препаратів найбільш доцільно призначити для профілактики цього ускладнення? A 56-year-old man has been suffering from cirrhosis of the liver with signs of portal hypertension for a long time. Bleeding from varicose veins of the esophagus was noted. Under the influence of therapy, the bleeding was stopped. Which of the following which drugs are the most appropriate to prescribe for the prevention of this complication?

Пропранолол Propranolol

Вазопресин Vasopressin

Контрикал Contrical

Препарати заліза Iron preparations

Гепарін Heparin

765 / 1500
Десятирічний хлопчик скаржиться на біль та припухлість колінних суглобів, ранкову скутість > 30 хвилин, субфебрильну температуру тіла, загальну слабкість. Під час огляду виявлено: колінні суглоби дефігуровані, теплі на дотик, порушена рухова активність уражених суглобів. Попередній діагноз: ювенільний ревматоїдний артрит. Вкажіть специфічний показник цього захворювання. A ten-year-old boy complains of pain and swelling of the knee joints, morning stiffness > 30 minutes, subfebrile body temperature, general weakness. During the examination, it was found: the knee joints are deformed, warm to the touch, impaired motor activity of the affected joints. Preliminary diagnosis: juvenile rheumatoid arthritis. Specify the specific indicator of this disease.

Диспротеїнемія, зниження вмісту альбумінів, гіперглобулінемія Dysproteinemia, decreased albumin content, hyperglobulinemia

Підвищення рівня С-реактивного протеїну Increased C-reactive protein

Підвищений вміст циркулюючих імунних комплексів (ЦІК) Increased content of circulating immune complexes (CIC)

Підвищення рівня ревматоїдного фактору Elevated rheumatoid factor

Підвищення антитіл до модифікованого цитрулінованого віментину (anti-MCV до SA-антигену) Elevation of antibodies to modified citrullinated vimentin (anti-MCV to SA-antigen)

766 / 1500
Жінка віком 25 років скаржиться на біль у правій здухвинній ділянці протягом 10-12 днів, затримку менструації на 7-8 тижнів. Під час пальпації спостерігається біль у правій здухвинній ділянці. Під час гінекологічного дослідження виявлено: матка та яєчники не збільшені, болючість у правому склепінні піхви. Припускається правостороння позаматкова вагітність. Виберіть оптимальний метод дослідження. A 25-year-old woman complains of pain in the right pubic area for 10-12 days, menstruation is delayed for 7-8 weeks. During palpation, there is pain in the right pubic area area. During the gynecological examination, the uterus and ovaries are not enlarged, the right vaginal vault is suspected. Select the optimal method of examination.

Термографія Thermography

Рентгенографія тазу Pelvis X-ray

Ультразвукове дослідження Ultrasound examination

Метросальпінгографія Metrosalpingography

Пневмогінекографія Pneumogynecography

767 / 1500
Педіатру потрібно проаналізувати рівні показника смертності немовлят. Що він повинен взяти за одиницю спостереження? A pediatrician needs to analyze the levels of the infant mortality rate. What should he take as the unit of observation?

Випадок смерті дитини у перші 7 діб життя Case of death of a child in the first 7 days of life

Випадок смерті дитини віком до одного року Case of death of a child under the age of one year

Випадок смерті дитини після 28 днів життя Case of child death after 28 days of life

Випадок смерті дитини на першому місяці життя Case of death of a child in the first month of life

Випадок смерті дитини під час пологів Case of child death during childbirth

768 / 1500
Жінка 60-ти років з нормальною масою тіла отримує з добовим раціоном 50 г білка, 70 г жиру, 300 г вуглеводів. У раціоні переважають зернобобові, достатньо овочів, але обмежена кількість молока і молочних продуктів. Весною щоденно вживає щавель і ревінь. Ризиком якого захворювання для жінки у першу чергу може бути такий ра-ціон? A 60-year-old woman with a normal body weight receives 50 g of protein, 70 g of fat, 300 g of carbohydrates in her daily diet. The diet is dominated by legumes, enough vegetables, but a limited amount of milk and dairy products. In the spring, she consumes sorrel and rhubarb. What kind of disease can a woman face in the first place?

Гіпертонічної хвороби Hypertensive disease

Атеросклерозу Atherosclerosis

Ожиріння Obesity

Сечокам'яної хвороби Urolithiasis

Остеопорозу Osteoporosis

769 / 1500
У хворої 65-ти років протягом 5 тижнів прогресує безбольова жовтяниця, шкірне свербіння, схудла на 10 кг, ахолія. Пальпується позитивний симптом Курвуа-зьє. Який попередній діагноз? A 65-year-old patient developed painless jaundice, skin itching, weight loss of 10 kg, acholia for 5 weeks. A positive Courvoisier symptom is palpated. What is the preliminary diagnosis ?

Рак жовчного міхура Gall bladder cancer

Вірусний гепатит Viral hepatitis

Малярія Malaria

Рак підшлункової залози Pancreatic cancer

Рак печінки Liver cancer

770 / 1500
Пацієнт віком 37 років скаржиться на загальну слабкість, спастичний біль у нижніх відділах живота, переважно у лівій здухвинній ділянці, рідкі випорожнення до 18 разів на добу з домішками слизу та крові. Захворювання почалося гостро три дні тому з ознобу, відчуття жару, головного болю. Загальний стан середньої важкості, температура тіла - 37,8^oC. Під час пальпації виявлено: сигмовидна кишка спазмована і болюча. Який найімовірніший діагноз? A 37-year-old patient complains of general weakness, spastic pain in the lower abdomen, mainly in the left iliac region, loose stools up to 18 times a day with impurities of mucus and of blood. The disease began three days ago with a feeling of fever, a general condition of 37.8°C. During palpation, the sigmoid colon was found to be spasmodic and painful.

Ієрсиніоз Yersiniosis

Неспецифічний виразковий коліт Nonspecific ulcerative colitis

Сальмонельоз Salmonellosis

Шигельоз Shigelosis

Амебіаз Amebiasis

771 / 1500
Дівчинка віком 13 років скаржиться на підвищення температури тіла до 37,4^oС протягом останніх 2 місяців після перенесеної ГРВІ. Під час огляду: худа статура, дифузне збільшення щитоподібної залози ІІ ступеня (щільна під час пальпації), екзофтальм, тахікардія. Який патологічний синдром спостерігається у пацієнтки? A 13-year-old girl complains of an increase in body temperature up to 37.4^oC during the last 2 months after having ARVI. During the examination: thin build, diffuse enlargement of the thyroid glands of the II degree (dense during palpation), exophthalmos, tachycardia. What pathological syndrome is observed in the patient?

Гіпотиреоз Hypothyroidism

Тиреотоксикоз Thyrotoxicosis

Тимомегалія Thymomegaly

Гіперпаратиреоз Hyperparathyroidism

Гіпопаратиреоз Hypoparathyroidism

772 / 1500
Жінка 29 років скаржиться на загальну слабкість, підвищену втомлюваність, зниження ваги, рідкі менструації малого обсягу. Рік тому були пологи, які ускладнилися масивною кровотечею. Об'єктивно спостерігається: пацієнтка астенічної будови, шкіра бліда і суха, волосся на голові рідке, під пахвами відсутнє. Молочні залози і статеві органи в стані гіпотрофії. Який попередній діагноз? A 29-year-old woman complains of general weakness, increased fatigue, weight loss, infrequent menstruation of small volume. A year ago, she gave birth, which was complicated by massive bleeding. Objectively observed : the patient is asthenic, the skin is pale and dry, the hair on the head is thin, the armpits are absent. The mammary glands and genitals are in a state of hypotrophy. What is the previous diagnosis?

Гіпопластична анемія Hypoplastic anemia

Нервова анорексія Anorexia nervosa

Пухлина гіпофізу Pituitary tumor

Синдром Шихана Sheehan Syndrome

Астеноневротичний синдром Asthenoneurotic syndrome

773 / 1500
Жінка 46-ти років знайшла при пальпації вузол в ділянці щитоподібної залози розміром 2-3 см, який зміщується при ковтанні, з чіткими межами. При скануванні визначається ''холодний вузол'' у правій частці щитоподібної залози. Лімфовузли шиї не збільшені. При пункції отримано клітини проліферуючого епітелію. Яка лікувальна тактика? During palpation, a 46-year-old woman found a 2-3 cm nodule in the area of ​​the thyroid gland, which shifts when swallowing, with clear borders. During the scan, it is determined '' 'cold node' in the right lobe of the thyroid gland. The lymph nodes of the neck are not enlarged. Proliferating epithelial cells were obtained during the puncture.

Правобічна гемітиреоїдектомія з терміновим гістологічним дослідженням Right-sided hemithyroidectomy with urgent histological examination

Тиреоїдектомія з висіченням клітковини шиї за Крайлем Thyroidectomy with neck tissue excision according to Crail

Спостереження Observations

Променеве лікування Radiation treatment

Геміструмектомія Hemistrumectomy

774 / 1500
Хворий 51-го року був збитий автомобілем. Скаржиться на біль у лівій половині тазу. При огляді з'ясовано, що конфігурація тазу не порушена, визначається припухлість лівої пахвинної ділянки. При пальпації: різка болючість в цій ділянці. Позитивні симптоми Ларрея, Габая і ''прилиплої п'яти''. Поставте попередній діагноз: The 51-year-old patient was hit by a car. He complains of pain in the left half of the pelvis. During the examination, it was found that the configuration of the pelvis is not disturbed, swelling of the left inguinal areas. On palpation: sharp pain in this area. Positive symptoms of Larray and 'sticky heel

Перелом дна вертлюгової западини Fracture of the bottom of the acetabulum

Перелом сідничної кістки Fracture of the hip bone

Перелом верхньої ості клубової кістки Fracture of the upper spine of the iliac bone

Перелом верхньої гілки лобкової кістки Fracture of the upper branch of the pubic bone

Ушкодження лобкового симфізу Damage of pubic symphysis

775 / 1500
Хворий 51-го року скаржиться на блювання з домішками крові. Зловживав алкоголем. Хворіє з 40-ка років, коли вперше виникла жовтяниця. При огляді: шкіра та видимі слизові оболонки жовтушні, ''судинні зірочки''. Зниженого живлення. Живіт збільшений в об'ємі, пупкова грижа, асцит. Край печінки гострий, неболючий, +3 см, селезінка +2 см. Аналіз крові: Hb - 80 г/л, лейкоцити - 3·10^9/л, тромбоцити - 85·10^9/л. Причиною портальної гіпертензії у хворого є: A 51-year-old patient complains of vomiting with blood impurities. He abused alcohol. He has been ill since the age of 40, when jaundice first appeared. On examination: the skin and visible mucous membranes are yellow, 'vascular asterisks'. Abdomen enlarged in volume, umbilical hernia, liver edge, +3 cm, spleen +2 cm. Blood analysis: Hb - 80 g/l. , leukocytes - 3·10^9/l, platelets - 85·10^9/l. The cause of portal hypertension in the patient is:

Синдром Бадда-Кіарі Budd-Chiari Syndrome

Тромбоз вени селезінки Thrombosis of the splenic vein

Констриктивний перикардит Constrictive pericarditis

Гемохроматоз Hemochromatosis

Цироз печінки на тлі зловживання алкоголю Liver cirrhosis due to alcohol abuse

776 / 1500
Дитина 3-х років надійшла зі скаргами матері на набряклість ніг, задишку, кашель, біль в животі. З раннього віку виявлений шум в серці, від обстеження батьки відмовилися. Після перенесеної ГРВІ 1 місяць тому мати помітила, що дитина стала малорухомою, періодично неспокійною, стала з'являтися набряклість на гомілках. Стан тяжкий. ЧД- 40/хв. Набряки гомілок і стоп. У легенях в задньонижніх відділах вологі хрипи. Ліва межа відносної серцевої тупості проходить вздовж лівої передньої пахвової лінії. Тони серця глухі, аритмічні. ЧСС- 120/хв. Печінка +5 см. Діу-рез понижений. Який патогенез набряків у дитини? A 3-year-old child came with his mother's complaints of swollen legs, shortness of breath, cough, stomach pain. A heart murmur was detected from an early age, the parents refused the examination 1 month ago, the mother noticed that the child became restless, the condition was severe. Swelling of the lower legs and feet. Left border relative dullness passes along the left axillary line. Heart rate - 120/min. Diuresis is decreased in the child?

Зниження серцевого викиду і венозний застій Decreased cardiac output and venous stasis

Порушення ниркової гемодинаміки Disturbance of renal hemodynamics

Зниження білковосинтетичної функції печінки Decreased protein synthetic function of the liver

Підвищення проникності судинної стінки Increased vascular wall permeability

Втрата білку з калом Fecal protein loss

777 / 1500
В оперативно-диспетчерську службу Центру екстреної медичної допомоги і медицини катастроф зателефонував житель міста. Скаржився на загрудиний біль стискального характеру з іррадіацією в ліву руку. Біль не знімається нітрогліцерином. Вкажіть термін прибуття бригади екстреної медичної допомоги у цьому випадку: A resident of the city called the operational dispatch service of the Center for Emergency Medical Aid and Disaster Medicine. He complained of chest pain of a compressive nature with radiation to the left arm. The pain is not relieved by nitroglycerin. Indicate the arrival date of the emergency medical assistance team in this case:

15 хвилин з моменту звернення 15 minutes since the request

60 хвилин з моменту звернення 60 minutes from the moment of application

20 хвилин з моменту звернення 20 minutes since the request

10 хвилин з моменту звернення 10 minutes since the request

30 хвилин з моменту звернення 30 minutes from the moment of application

778 / 1500
У пацієнта 64-х років протягом останніх 2 місяців з'явилося відчуття утрудненого ковтання твердої їжі з затримкою її посередині стравоходу. Втрата ваги - 4 кг. Батько хворого помер від раку шлунка. Запідозрено рак стравоходу. За допомогою якого дослідження найбільш імовірно можна верифікувати цю патологію? A 64-year-old patient had a feeling of difficulty swallowing solid food with its retention in the middle of the esophagus during the last 2 months. Weight loss - 4 kg. The patient's father died from stomach cancer. Esophageal cancer is suspected. Which test is most likely to verify this pathology?

Рентгенконтрастне дослідження стравоходу і шлунка X-ray contrast examination of esophagus and stomach

Комп'ютерна томограма грудної клітки Computer tomography of the chest

Внутрішньостравохідна рН-метрія Esophageal pH-metry

ФЕГДС з біопсією FEGDS with biopsy

Дихальний уреазний тест Breathing urease test

779 / 1500
Пацієнтку віком 25 років шпиталізовано до пологового будинку з терміном вагітності 34 тижні та скаргами на яскраві кров'янисті виділення зі згустками, що з'явились після акту дефекації. Об'єктивно спостерігається: голівка плода пальпується біля дна матки. Серцебиття плода - 140/хв. Пологової діяльності немає. Під час піхвового дослідження виявлено: шийка матки довжиною 3 см, зів пропускає кінчик пальця, скрізь склепіння піхви пальпується утворення м'якої консистенції. Виділення геморагічні, яскраві. Який найімовірніший діагноз? A 25-year-old patient was hospitalized in the maternity ward with a 34-week pregnancy and complaints of bright bloody discharge with clots that appeared after defecation. About The fetal head is palpated near the bottom of the uterus. There is no labor activity during the vaginal examination hemorrhagic, bright. What is the most likely diagnosis?

Розрив матки Rupture of uterus

Передлежання плаценти Placenta previa

Відшарування нормально розташованої плаценти Detachment of a normally located placenta

Передчасні пологи Premature birth

Низьке прикріплення плаценти Low placental attachment

780 / 1500
Хворий 35-ти років звернувся до сімейного лікаря з раптовим кинджальним болем в епігастрії; в анамнезі виразка шлунку. Об'єктивно: дошкоподібне напруження м'язів передньої черевної стінки, позитивний симптом Щоткіна-Блюмберга. При рентгенологічному дослідженні виявлено серпоподібне просвітлення під склепінням діафрагми. Яке ускладнення розвинулося? A 35-year-old patient went to the family doctor with a sudden stabbing pain in the epigastrium; a history of stomach ulcer. Objectively: plate-like tension of the muscles of the anterior abdominal wall , a positive Shtokkin-Blumberg symptom. A sickle-shaped light was detected under the diaphragmatic vault.

Виразкова кровотеча Ulcer bleeding

Перфорація виразки шлунку Perforation of gastric ulcer

Пенетрація виразки шлунку Stomach ulcer penetration

Стеноз воротаря Stenosis of the goalkeeper

Малігнізація виразки шлунку Malignancy of gastric ulcer

781 / 1500
Пацієнт віком 50 років скаржиться на біль у пальцях ступні, м'язах, особливо під час ходіння, головний біль, свербіж шкіри. Об'єктивно спостерігається: шкіра обличчя з червоно-ціанотичним відтінком. Лімфатичні вузли не пальпуються. Пульс - 76/хв. АТ - 180/100 мм рт. ст. Легені - без особливостей. Межі серця зміщені вліво на 2 см. Печінка - +2 см, селезінка біля краю реберної дуги, щільна, безболісна. У загальному аналізі крові виявлено: еритроцити - 6,3·10^12/л, гемоглобін - 201 г/л, КП - 0,8, лейкоцити - 10,5·10^9/л, еозинофіли - 4%, паличкоядерні - 7%, сегментоядерні - 62%, лімфоцити - 22%, моноцити - 5%, тромбоцити - 500·10^9/л, ШОЕ - 1 мм/год, гематокрит - 55%. Який найімовірніший діагноз? A 50-year-old patient complains of pain in the toes, muscles, especially when walking, headache, itchy skin. Objectively observed: facial skin with a red tinge. The pulse is 76/min. The heart is shifted to the left by 2 cm. arcs, dense, painless. The general blood analysis revealed: erythrocytes - 6.3·10^12/l, hemoglobin - 201 g/l, CP - 0.8, leukocytes - 10.5·10^9/l, eosinophils - 4%, rod cells - 7%, segment cells - 62%, lymphocytes - 22%, monocytes - 5%, platelets - 500·10^9/l, ESR - 1 mm/h, hematocrit - 55%. What is the most likely diagnosis?

Справжня поліцитемія True polycythemia

Вторинний еритроцитоз Secondary erythrocytosis

Гіпертонічна хвороба Hypertensive disease

Хвороба Іценко-Кушинга Itsenko-Cushing disease

Облітеруючий ендоартеріїт Endoarteritis obliterans

782 / 1500
Чоловік 52-х років звернувся до лікаря зі скаргами на безсоння, роздратованість, підвищену збудливість, підвищену пітливість, кровоточивість ясен. Працює на виробництві медичного обладнання у відділі, що спеціалізується на конструюванні сфігмоманометрів та термометрів. При фізикальному обстеженні неритмічний та асиметричний тремор пальців витягнутих рук, підвищена збудливість вегетативної нервової системи, червоний дермографізм, гінгівіт, стоматит, ціанотична смуга вздовж країв ясен. При лабораторному дослідженні крові виявлено лімфоцитоз і моноцитоз. Хронічне отруєння якою речовиною є найбільш імовірним у пацієнта? A 52-year-old man consulted a doctor with complaints of insomnia, irritability, increased excitability, increased sweating, bleeding gums. He works in the production of medical equipment in the department that specializes in the construction of sphygmomanometers and thermometers. During the physical examination, non-rhythmic and asymmetric tremor of the outstretched hands, increased excitability of the autonomic nervous system, gingivitis, stomatitis along the edges of the gums, lymphocytosis and monocytosis were found in the laboratory is most likely in the patient?

Ртуттю Mercury

Бензолом Benzol

Свинцем Lead

Марганцем Manganese

Чотирихлористим вуглецем Carbon tetrachloride

783 / 1500
17-річний хлопець на уроці фізичного виховання раптово втратив свідомість. На місці була проведена серцево-легенева реа-німація. На момент приїзду швидкої свідомість до пацієнта повернулася. При зборі анамнезу стало відомо, що батько і дід дитини проходили обстеження та лікування у кардіолога, проте помирали в молодому віці. При фізикальному огляді дитини встановлено наявність систолічного шуму в серці, гучність якого збільшується при зміні положення тіла. При ехокардіо-графічному дослідженні зареєстровано потовщення стінки лівого шлуночка більше 15 мм. Яке захворювання можна припустити у даного хворого? A 17-year-old boy suddenly lost consciousness during a physical education class. Cardiopulmonary resuscitation was performed on the spot. When the ambulance arrived, the patient regained consciousness. it became known that the child's father and grandfather were examined and treated by a cardiologist, but they died at a young age. During the physical examination of the child, the presence of a systolic murmur was established, the volume of which increases when the body position is changed. During the echocardiographic examination, wall thickening was registered the left ventricle is more than 15 mm. What disease can be assumed in this patient?

Дилатаційна кардіоміопатія Dilated cardiomyopathy

Вегетативна дисфункція Vegetative dysfunction

Гостра ревматична лихоманка Acute rheumatic fever

Гіпертрофічна кардіоміопатія Hypertrophic cardiomyopathy

Неревматичний кардит Non-rheumatic carditis

784 / 1500
У хворої 19-ти років слабкість, помірний головний біль, біль у горлі при ковтанні. Хворіє 3 дні. Об'єктивно: температура тіла - 38,2^oC, генералізована лімфоаденопатія. Ознаки тонзилофарингіту. Помірна гепатоспленомегалія. ЗАК: 35% атипових мононуклеарів. Яке дослідження необхідно призначити для верифікації діагнозу? A 19-year-old patient has weakness, moderate headache, sore throat when swallowing. She has been ill for 3 days. Objectively: body temperature - 38.2^ oC, generalized lymphadenopathy. Moderate hepatosplenomegaly. 35% of mononuclear cells. What test should be used to verify the diagnosis?

Дослідження крові на сумарні антитіла до ВІЛ Blood test for total HIV antibodies

Дослідження крові на IgM аnti-HHV-7 Blood test for IgM anti-HHV-7

Дослідження крові на IgM аnti-CMV Blood test for IgM anti-CMV

Дослідження крові на IgM аnti-HHV-6 Blood test for IgM anti-HHV-6

Дослідження крові на IgM аnti-VCA EBV Blood test for IgM anti-VCA EBV

785 / 1500
Чоловік 25-ти років звернувся до лікаря зі скаргами на наявність висипки протягом 2 тижнів, яка з'явилась несподівано. Під час збору анамнезу пацієнт повідомив, що почуває себе повністю здоровим, але зазначив що 2 місяці тому влітку мав ''жахливу, грипоподібну'' застуду. При фізикальному обстеженні на шкірі лобка, геніталій та стегон розташовані сферичні папули, рожевого та тілесного кольору, без запалення, з центральним заглибленням. Який діагноз є найбільш імовірним? A 25-year-old man came to the doctor complaining of a rash for 2 weeks that appeared unexpectedly. During history taking, the patient reported that he felt completely healthy, but noted that 2 months ago he had a 'terrible, flu-like' cold. On physical examination, there are spherical papules, pink and flesh-colored, with a central indentation probable?

Вторинний сифіліс Secondary syphilis

Контагіозний молюск Mollus contagion

Фурункул Furnish

Оперізуючий герпес Herpes zoster

Папілома Papilloma

786 / 1500
До дерматолога звернулася мати з дівчинкою 6 років. Дитина хворіє 5 днів. У дитини на тлі нежиті з'явилася велика кількість гнійної висипки. Об'єктивно встановлено: на шкірі обличчя є численні фліктени, наповнені серозно-гнійним вмістом, на деяких сформовані золотисто-жовті кірки. Який імовірний діагноз? A mother with a 6-year-old girl consulted a dermatologist. The child has been sick for 5 days. The child has a large amount of purulent rash against the background of a runny nose. Objectively established: on the skin of the face has numerous spots filled with serous-purulent contents, some have golden-yellow crusts. What is the probable diagnosis?

Стрептококове імпетиго Streptococcal impetigo

Дитяча екзема Children's eczema

Атопічний дерматит Atopic dermatitis

Фотодерматит Photodermatitis

Токсико-алергічний дерматит Toxic-allergic dermatitis

787 / 1500
Жінка 68 років доставлена швидкою допомогою з носовою кровотечею. З анамнезу встановлено, що протягом 15 років вона хворіє на гіпертонічну хворобу, не лікувалась, АТ - 210/120 мм рт. ст. Періодичні носові кровотечі турбують декілька років. Об'єктивно встановлено: у носовій порожнині кров'янисті згустки, дихання вільне. Під час видалення згустків з'явилась кровотеча з носової перегородки. Яку першу допомогу потрібно надати? A 68-year-old woman was delivered by ambulance with a nosebleed. From the anamnesis, it was established that she has been suffering from hypertension for 15 years, was not treated, blood pressure - 210/120 mm Art. Periodic nosebleeds. Objectively established: blood clots in the nasal cavity. Bleeding from the nasal septum appeared during the removal of the clots.

Перев'язка сонної артерії на боці кровотечі Ligation of the carotid artery on the bleeding side

Задня тампонада Rear tamponade

Тушування слизової носа розчином азотнокислого срібла або ваготілом Putting the nasal mucosa with a solution of silver nitrate or vagotyl

Відшарування слизової оболонки носової перегородки Defoliation of the mucous membrane of the nasal septum

Передня тампонада носа з судинозвужувальними препаратами, контроль за артеріальним тиском Anterior nasal tamponade with vasoconstrictor drugs, blood pressure control

788 / 1500
Дівчина 19 років скаржиться на виражений біль у поперековій ділянці, більше справа, підвищення температури тіла до 38,5^oC, часте сечовиділення, симптоми з'явились 5 днів тому після переохолодження (похід у гори). Об'єктивно встановлено: шкіра бліда, волога. Пульс - 100/хв., АТ - 120/65 мм рт. ст. Живіт м'який, симптом Пастернацького позитивний більше справа. ЗАК виявив: еритроцити - 3,5·10^12/л, Hb - 115 г/л, лейкоцити - 10·10^9/л, ШОЕ - 22 мм/год. ЗАС виявив: щільність - 1018 г/л, білок - 0,099 г/л, лейкоцити - 35-45 в п/з, переважають сегментоядерні, еритроцити - 3-4 в п/з, велика кількість слизу, бактерій. Який попередній діагноз? A 19-year-old girl complains of severe pain in the lumbar region, more on the right, an increase in body temperature to 38.5^oC, frequent urination, symptoms appeared for 5 days Therefore, after hypothermia, the skin is pale, the blood pressure is 120/min. The abdomen is more positive. erythrocytes - 3.5·10^12/l, Hb - 115 g/l, leukocytes - 10·10^9/l, ESR - 22 mm/h. ZAS revealed: density - 1018 g/l, protein - 0.099 g /l, leukocytes - 35-45 in p/z, predominately segmented, erythrocytes - 3-4 in p/z, large amount of mucus, bacteria. What is the previous diagnosis?

Хронічний гломерулонефрит Chronic glomerulonephritis

Гострий пієлонефрит Acute pyelonephritis

Хронічний пієлонефрит Chronic pyelonephritis

Гострий гломерулонефрит Acute glomerulonephritis

Туберкульоз нирок Kidney tuberculosis

789 / 1500
У квартирі знайдено труп чоловіка 56 років без ознак насильницької смерті. Під час дослідження трупа встановлено, що трупні плями розташовані на задній поверхні тіла, у разі натискання повністю блідніють та повністю відновлюють своє забарвлення за 55 секунд, трупне заклякання добре виражене у м'язах щелепи, шиї та верхніх кінцівок, ректальна температура складає 27^oC, ознак гниття на трупі не виявлено. Яка давність настання смерті? The corpse of a 56-year-old man was found in the apartment without signs of violent death. During the examination of the corpse, it was found that corpse spots are located on the back surface of the body, when pressed, they turn completely pale and completely recover their color in 55 seconds, the cadaveric convulsion is well expressed in the muscles of the jaw, neck and upper limbs, the rectal temperature is 27°C, no signs of decay are found on the corpse. How long ago did death occur?

Від 2 до 6 годин From 2 to 6 hours

Від 6 до 12 годин From 6 to 12 hours

Від 24 до 36 годин From 24 to 36 hours

Від 18 до 24 годин From 18 to 24 hours

Від 12 до 18 годин From 12 to 18 hours

790 / 1500
Жінка 36-ти років звернулася до лікаря зі скаргами на загальну слабкість, біль у животі та пожовтіння шкіри. Зазначені скарги поступово наростали протягом останніх 3-х місяців. Пацієнтка повідомила, що не мала серйозних захворювань в анамнезі. Вживання наркотиків та незахищені статеві стосунки заперечує. При фізикальному обстеженні іктеричність склер та жовтушність шкіри, болісність при пальпації правої підреберної ділянки. При лабораторному дослідженні сироватки крові: загальний білірубін - 64,5 мкмоль/л, прямий - 22,7 мкмоль/л, АСТ- 822 Од/л, АЛТ- 1237 Од/л, HBsAg - позитивно, HBeAg - позитивно. Яка тактика лікування цієї хворої є найбільш доречною? A 36-year-old woman consulted a doctor with complaints of general weakness, abdominal pain, and yellowing of the skin. These complaints gradually increased over the past 3 months. The patient reported that she had no history of serious diseases. Denies drug use and unprotected sex. During physical examination, sclera and yellowness of the skin, pain during palpation of the right subcostal area. In laboratory examination, total bilirubin - 64.5 μmol/l direct - 22.7 μmol/l, AST- 822 Units/l, ALT- 1237 Units/l, HBsAg - positive, HBeAg - positive. What is the most appropriate treatment strategy for this patient?

Вакцина для профілактики гепатиту В та ламівудин Vaccine for the prevention of hepatitis B and lamivudine

Імуноглобулін людини нормальний Human immunoglobulin is normal

Специфічний імуноглобулін проти вірусу гепатиту В Specific immunoglobulin against hepatitis B virus

Преднізолон та рибавірин Prednisone and Ribavirin

ПЕГ-інтерферон альфа-2а PEG interferon alfa-2a

791 / 1500
Хвора 68-ми років із застійною серцевою недостатністю, ФВ ЛШ textless40%, отримує наступну схему фармакотерапії: раміприл, торасемід, бісопролол, клопідогрель, дигоксин. Під час чергового обсте-ження була виявлена часта поліморфна шлуночкова екстрасистолія. Який з призначених препаратів необхідно вилучити зі схеми лікування? A 68-year-old patient with congestive heart failure, LVEF textless40%, receives the following regimen of pharmacotherapy: ramipril, torasemide, bisoprolol, clopidogrel, digoxin. During the next the examination revealed frequent polymorphic ventricular extrasystole. Which of the prescribed drugs should be removed from the treatment regimen?

Дигоксин Digoxin

Клопідогрель Clopidogrel

Раміприл Ramipril

Торасемід Torasemide

Бісопролол Bisoprolol

792 / 1500
Хворий скаржиться на підвищення температури тіла до 39,4^oC, головний біль, блювання. Ригідність м'язів потилиці, симптом Керніга позитивний, вогнищевої симптоматики немає. Ліквор: цитоз - 19600 мкл, лімфоцити - 27%, нейтрофіли - 73%, білок - 6,3 г/л. Імовірний діагноз: The patient complains of an increase in body temperature up to 39.4^oC, headache, vomiting. Neck muscle stiffness, Kernig's symptom is positive, there are no focal symptoms. Liquor : cytosis - 19600 μl, lymphocytes - 27%, neutrophils - 73%, protein - 6.3 g/l. Probable diagnosis:

Менінгококовий менінгіт Meningococcal meningitis

Герпетичний менінгіт Herpetic meningitis

Туберкульозний менінгіт Tuberculous meningitis

Субарахноїдальний крововилив Subarachnoid hemorrhage

Ентеровірусний менінгіт Enterovirus meningitis

793 / 1500
Робітник, чоловік 46 років, упродовж 19 років контактував із парами ртуті в концентрації 0,09-0,18 мг/м^3. Обстежений невропатологом. Виявлено: астеніч-ний синдром, мікроорганічна симптоматика з інтенційним тремтінням, позитивними субкортикальними симптомами, наявність депо ртуті (відповідно 0,01 і 0,04 до і після провокації унітіолом), гіпотонічний тип РЕГ. Розвиток якого захворювання можна припустити? The worker, a 46-year-old man, was in contact with mercury vapors at a concentration of 0.09-0.18 mg/m^3 for 19 years. He was examined by a neurologist. It was found: asthenic syndrome, microorganism symptoms with intentional tremors, positive subcortical symptoms, mercury depot (0.01 and 0.04 before and after unitiol provocation, respectively), hypotonic type of REG. What kind of disease can be assumed to develop?

Церебральний атеросклероз Cerebral atherosclerosis

Паркінсонічний синдром Parkinsonian syndrome

Неврастенія Neurasthenia

Залишкові явища нейроінфекції Residual effects of neuroinfection

Хронічне отруєння ртуттю Chronic mercury poisoning

794 / 1500
Чоловік 28 років упродовж декількох років спостерігає різке обмеження рухливості в хребті та біль у поперековому відділі хребта, який зберігається тривалий час. Рентгенографія попереково-крижового відділу хребта і крижово-клубового зчленування показує звуження суглобових щілин, остеопороз. Назвіть важливу ланку вторинної профілактики цієї хвороби: For several years, a 28-year-old man observes a sharp limitation of mobility in the spine and pain in the lumbar spine, which persists for a long time. X-ray of the lumbosacral spine and sacral of the hip joint shows the narrowing of the joint spaces, osteoporosis. Name an important link in the secondary prevention of this disease:

Щоденна лікувальна гімнастика Daily therapeutic gymnastics

Приймання цитостатиків Taking cytostatics

Іммобілізація хребта (корсет) Immobilization of the spine (corset)

Постійне тривале приймання малих доз препаратів золота Constant long-term intake of small doses of gold drugs

Лише хірургічна корекція Surgical correction only

795 / 1500
Чоловікові 72 років у зв'язку з миготливою аритмією та проявами недостатності кровоообігу призначено фуросемід, дігоксин, інгібітори АПФ. На 5-й день із початку лікування з'явились ниючі болі в животі, нудота, одноразове блювання. На ЕКГ виявлено: шлуночкова бігемінія, коритоподібне зміщення сегменту ST нижче ізолінії. Яке ускладнення найімовірніше виникло у хворого? A 72-year-old man was prescribed furosemide, digoxin, and ACE inhibitors in connection with atrial fibrillation and manifestations of circulatory failure. On the 5th day from the start of treatment, Abdominal pain, nausea, one-time vomiting. The ECG revealed: ventricular bigeminia, trough-like shift of the ST segment below the isoline. What complication most likely occurred in the patient?

Медикаментозний гастрит Medicated gastritis

Інтоксикація фуросемідом Furosemide intoxication

Дигіталісна інтоксикація Digital intoxication

Гіперкаліємія Hyperkalemia

Інфаркт міокарду (абдомінальний варіант) Myocardial infarction (abdominal variant)

796 / 1500
Хворий 38-ми років скаржиться на періодичну висипку в ділянці бороди і вусів, що супроводжується помірною болючістю. Хворіє впродовж 2-х років. Об'єктивно: в ділянці бороди та вусів на фоні гіперемії та помірної інфільтрації - множинні згруповані пустульозні елементи, дрібні ерозії та гнійні кірки. На яке супутнє захворювання слід обстежити паціє-нта в першу чергу? A 38-year-old patient complains of a periodic rash in the area of ​​the beard and mustache, which is accompanied by moderate pain. He has been ill for 2 years. Objectively: in the area beards and mustaches against the background of hyperemia and moderate infiltration - multiple grouped pustular elements, small erosions and purulent crusts. What concomitant disease should the patient be examined for first?

Туберкульозна інфекція Tuberculosis infection

Дисбіоз кишечника Intestinal dysbiosis

Гепатит С Hepatitis C

Цукровий діабет Diabetes

Захворювання щитоподібної залози Thyroid disease

797 / 1500
Пацієнтка віком 18 років скаржиться на біль внизу живота, підвищення температури тіла до 37,5^oС, гнійні виділення зі статевих шляхів. Під час гінекологічного обстеження спостерігається: уретра інфільтрована, шийка матки гіперемована, набрякла, виділення рясні, гнійні, матка нормальних розмірів, болюча при пальпації, додатки з обох сторін тяжисті, болючі. Під час бактеріоскопічного дослідження виділень виявлено: грамнегативні диплококи внутрішньоклітинно. Яке захворювання спостерігається у пацієнтки? An 18-year-old patient complains of pain in the lower abdomen, an increase in body temperature to 37.5°C, purulent discharge from the genital tract. During a gynecological examination, the following is observed: urethra infiltrated, the cervix is ​​hyperemic, the discharge is abundant, purulent, the uterus is painful on palpation, the appendages on both sides are painful. During the bacterioscopic examination of the discharge, gram-negative diplococci are found intracellularly. What disease is observed in the patient?

Трихомонадний кольпіт Trichomonad colpitis

Гостра висхідна гонорея Acute ascending gonorrhea

Бактеріальний вагіноз Bacterial vaginosis

Підгостра висхідна гонорея Subacute ascending gonorrhea

Хронічна гонорея Chronic gonorrhea

798 / 1500
Чоловік 18-ти років звернувся до лікаря зі скаргами на переймоподібний біль в животі, часті рідкі випорожнення з домішками слизу та свіжої крові, що тривають 4 місяці. За цей час схуд на 10 кг. При фізикальному обстеженні живіт м'який, болючий по ходу товстої кишки зліва, сигмоподібна кишка спазмована. При лабораторному дослідженні крові еритроцити - 3,2·10^12/л, гемоглобін - 92 г/л, лейкоцити - 10,6·10^9/л, ШЗЕ- 34 мм/год. Результат бак. посіву випорожнень негативний. На колоноскопії виявлено запалення із втратою судинного малюнку, ділянка підвищеної кровоточивості 25 см з чіткими межами починаючи від краю анусу. Яка лікувальна тактика буде найбільш доречною? An 18-year-old man consulted a doctor with complaints of spasmodic abdominal pain, frequent loose stools with mucus and fresh blood, lasting 4 months. lost weight by 10 kg. During physical examination, the abdomen is soft, painful in the course of the colon, the sigmoid colon is spasmodic. During the laboratory examination of blood, erythrocytes - 3.2·10^12/l, hemoglobin - 92 g/l 10.6·10^1/L, 34 mm/h. Colonoscopy revealed inflammation with a 25 cm bleeding area starting from the edge of the anus most appropriate?

Призначення метронідазолу Prescription of metronidazole

Хірургічне лікування Surgical treatment

Переливання еритроцитарної маси Red blood cell transfusion

Повторний забір випорожнень на бактеріологічне дослідження Re-sampling of feces for bacteriological examination

Призначення преднізолону та месаламіну Prednisone and mesalamine prescription

799 / 1500
Жінка 25 років скаржиться на висипку на шкірі міжпальцевих ділянок кистей, бокових поверхонь живота, що супроводжується свербежем, особливо у нічний час. Хворіє 4-й день. На цих ділянках шкіри є маленькі папуло-везикули у вигляді спарених цяточок, між ними - тонкі смужки сіруватого кольору. Який найбільш імовірний діагноз? A 25-year-old woman complains of a rash on the skin of the interdigital areas of the hands, the lateral surfaces of the abdomen, which is accompanied by itching, especially at night. She has been ill for the 4th day. there are small papulo-vesicles in the form of paired spots in areas of the skin, between them - thin strips of gray color. What is the most likely diagnosis?

Псоріаз Psoriasis

Червоний плаский лишай Lichen planus

Нейродерміт Neurodermatitis

Алергічний дерматит Allergic dermatitis

Короста Scabies

800 / 1500
Дівчина 20 років поступила до лікарні зі скаргами на серцебиття, біль у грудях, запаморочення, задуху, які виникають періодично і тривають 10-15 хв. Під час такого стану має місце підвищена тривожність, страх смерті. Який найбільш імовірний діагноз? A 20-year-old girl was admitted to the hospital with complaints of palpitations, chest pain, dizziness, shortness of breath, which occur periodically and last 10-15 minutes. During such a condition there is increased anxiety, fear of death. What is the most likely diagnosis?

Панічний розлад Panic Disorder

Тривожний розлад Anxiety disorder

Іпохондричний розлад Hypochondriacal disorder

Фобічний розлад Phobic disorder

Дисоціативний розлад Dissociative disorder

801 / 1500
1-місячна дитина доставлена матір'ю до відділення невідкладної допомоги зі скаргами на блювання з домішками жовчі після годування та випорожнення із домішками крові. Мати народила дитину у 30 років на 32 тижні гестації. При народженні стан дитини на 5-й хвилині 6 балів за шкалою Апгар, маса тіла - 1300 г. При фізикальному дослідженні дитина млява, значне здуття живота. Який наступний крок лікаря буде найбільш доречним? A 1-month-old child was brought to the emergency department by his mother complaining of bilious vomiting after feeding and bloody stools. The mother gave birth to a child at the age of 30 at 32 weeks of gestation. At birth, the child's condition is 6 points on the Apgar scale, the body weight is 1300 g. During the physical examination, the child is lethargic, with significant abdominal distension. What would be the most appropriate step for the doctor?

Направити до дитячого хірурга Refer to pediatric surgeon

Рекомендувати перейти на безлактозне штучне вигодовування Recommend switching to lactose-free artificial feeding

Направити на езофагогастроскопію Send to esophagogastroscopy

Призначити емпіричну антибіотикотерапію Prescribe empiric antibiotic therapy

Направити на рентгенографію черевної порожнини Send for abdominal X-ray

802 / 1500
Пацієнтка віком 65 років з цукровим діабетом лікувалась метформіном. У зв'язку з підвищеним АТ приймала сечогінні препарати. Поступово ефективність діуретика знизилась, з’явилась нудота, блювання. Об'єктивно спостерігається: пацієнтка не реагує на подразники, шкіра суха, запах ацетону відсутній. АТ - 180/100 мм рт. ст. Тони серця глухі, пульс - 98/хв, дихання везикулярне. Печінка - +4 см. У аналізі крові виявлено: глюкоза - 48 ммоль/л, Nа - 156 ммоль/л, К - 5,2 ммоль/л, сечовина - 15 ммоль/л. Яке ускладнення розвинулось у пацієнтки? A 65-year-old patient with diabetes was treated with metformin. Due to high blood pressure, she took diuretics. Gradually, the effectiveness of the diuretic decreased, nausea and vomiting appeared. Objectively observed: the patient does not react to stimuli, the smell of acetone is absent. Blood pressure is 180/100 mm Hg, the heart rate is 98/min. The liver is +4 cm revealed: glucose - 48 mmol/l, Na - 156 mmol/l, K - 5.2 mmol/l, urea - 15 mmol/l. What complication developed in the patient?

Лактацидотична кома Lactacidotic coma

Гіпоглікемічна кома Hypoglycemic coma

Дисциркуляторна кома Dyscirculatory coma

Кетоацидотична кома Ketoacidotic coma

Гіперосмолярна кома Hyperosmolar coma

803 / 1500
У чоловіка з хронічним гнійним отитом виник сильний головний біль, блювання, підвищилася температура тіла. Наявні менінгеальні симптоми. Вогнищева неврологічна симптоматика відсутня. Які першечергові заходи для ведення хворого? A man with chronic purulent otitis has a severe headache, vomiting, and increased body temperature. Meningeal symptoms are present. There are no focal neurological symptoms. What are the first-line measures for managing the patient?

Амбулаторне спостереження за хворим Ambulatory monitoring of the patient

Консультація отоларинголога Otolaryngologist consultation

Призначення протизапальних ліків Prescription of anti-inflammatory drugs

Призначення рентгенографії черепа Cranial x-ray appointment

Госпіталізація та діагностична люмбальна пункція Hospitalization and diagnostic lumbar puncture

804 / 1500
У чоловіка хворого на гіпертонію поступово розвинулися ознаки серцевої недостатності: задишка, вологі хрипи у легенях, збільшення печінки. Яке інструментальне дослідження має більшу діагностичну цінність для встановлення варіанту дисфункції міокарда? A man with hypertension gradually developed signs of heart failure: shortness of breath, wet rales in the lungs, enlarged liver. Which instrumental examination has a greater diagnostic value for establishing the variant of myocardial dysfunction ?

Фонокардіографія Phonocardiography

Коронарографія Coronarography

Тетраполярна реографія Tetrapolar rheography

Електрокардіографія Electrocardiography

Ехокардіографія Echocardiography

805 / 1500
Жінка 25 років госпіталізована до гінекологічного відділення зі скаргами на біль унизу живота, підвищення температури до 39,7^oC. Об'єктивно спостерігається: АТ - 120/80 мм рт. ст., пульс - 108/хв., задовільного наповнення і напруги. Живіт помірно здутий, різко болючий у нижніх відділах. Симптом Щоткіна-Блюмберга позитивний в гіпогастральній ділянці. Піхвове обстеження виявило: матка та придатки не пальпуються через напруження передньої черевної стінки. Заднє склепіння піхви нависає, різко болюче. Який діагноз найімовірніший? A 25-year-old woman was hospitalized in the gynecology department with complaints of pain in the lower abdomen, an increase in temperature to 39.7^oC. Objectively observed: blood pressure - 120/80 mmHg, pulse - 108/min. Abdomen is moderately bloated, sharply painful in the hypogastric area. Vaginal examination revealed that the uterus and appendages are not palpable walls. The posterior vault of the vagina hangs down, sharply painful. What is the most likely diagnosis?

Апоплексія яєчника Ovarian apoplexy

Пельвіоперитоніт Pelvioperitonitis

Позаматкова вагітність Ectopic pregnancy

Гострий ендометрит Acute endometritis

Гострий аднексит Acute adnexitis

806 / 1500
Хворий 42-х років доставлений до травматологічного відділення 2 години тому. Рентгенологічно: перелом кісток тазу. Об'єктивно: виведений з шоку. Самостійно не мочиться. Уретрорагія. Пальпується збільшений сечовий міхур. Болісна припухлість в ділянці промежини. Який попередній діагноз? A 42-year-old patient was brought to the trauma department 2 hours ago. X-ray: pelvic bone fracture. Objectively: brought out of shock. He does not urinate on his own. Urethrorrhagia. An enlarged bladder is palpated. Painful swelling in the perineum. What is the previous diagnosis?

Рефлекторна затримка сечовипускання Reflex micturition delay

Травма сечового міхура Bladder injury

Травма уретри Urethra trauma

Гостра ниркова недостатність Acute renal failure

Забій промежини Perineal bruise

807 / 1500
Жінка 36-ти років звернулася до лікаря зі скаргами на часте сечовипускання, біль у лівому боці, нудоту та підвищену температуру. При фізикальному обстеженні температура тіла - 39,4^oC, артеріальний тиск - 120/80 мм рт.ст., пульс - 89/хв., частота дихання - 18/хв., SpO_2 - 98% при кімнатному повітрі. При пальпації болючість у лівому костовертебральному куті та лівому боці. Який наступний крок лікаря у веденні пацієн-тки буде найбільш доречним? A 36-year-old woman consulted a doctor with complaints of frequent urination, pain in the left side, nausea, and high temperature. During a physical examination, the body temperature was 39.4 ^oC, blood pressure - 120/80 mm Hg, pulse - 18/min, SpO_2 - 98% on palpation, pain in the left costovertebral angle will the next step of the doctor in the management of the patient be the most appropriate?

Призначити триметоприм-сульфаметоксазол на 10 днів Prescribe trimethoprim-sulfamethoxazole for 10 days

Призначити нітрофурантоїн перорально на 10 днів Prescribe nitrofurantoin orally for 10 days

Рекомендувати пройти тест на вагітність Recommend to take a pregnancy test

Здати загальний аналіз сечі та виконати бактеріологічне дослідження сечі Pass a general urinalysis and perform a bacteriological examination of urine

Розпочати внутрішньовенне введення цефтриаксону Start intravenous ceftriaxone

808 / 1500
Під час кесаревого розтину у повторновагітної 37-38 тижнів із діагнозом передчасного відшарування нормально розташованної плаценти на фоні пізнього гестозу виявлено: матка м'яка, ціанотична, тотально імбібована кров'ю (матка Кувелера). Яка подальша тактика? During cesarean section in a 37-38-week pregnant woman with a diagnosis of premature detachment of a normally located placenta against the background of late gestosis, it was found: the uterus is soft, cyanotic, totally imbibed with blood 'yu (Kuveler's uterus). What is the next tactic?

Екстирпація матки без додатків Extirpation of the uterus without attachments

Перев'язка маточної артерії Uterine artery ligation

Розширена екстирпація матки Extended uterine extirpation

Надпіхвова ампутація матки Supravaginal amputation of the uterus

Перев'язка здухвинної артерії Ligation of the iliac artery

809 / 1500
Пацієнт віком 32 роки скаржиться на біль в лівій нижній кінцівкі протягом року, переміжну кульгавість через 50-100 м, періодично є нічні болі. Об'єктивно спостерігається: ліва кінцівка бліда, холодна. Артеріальна пульсація відсутня на лівій гомілці. Який найімовірніший діагноз? A 32-year-old patient complains of pain in the left lower limb for a year, intermittent lameness after 50-100 m, occasional night pains. Objectively observed: left the limb is pale, cold. There is no arterial pulsation on the left leg. What is the most likely diagnosis?

Тромбоз аорти Aortic thrombosis

Синдром Леріша Lerisch syndrome

Облітеруючий ендартеріїт Endarteritis obliterans

Емболія аорти Aortic embolism

Синдром Рейно Raynaud's syndrome

810 / 1500
Протягом 12 годин лісоруб працював на відкритому повітрі у вітряну погоду при температурі повітря +4-5^oС. Взуття здавлювало дистальні частини ступнів. Скарги на колючий, пекучий біль у 1-3 пальцях обох стоп, ломоту у міжфалангових суглобах зниження чутливості шкіри на 1-3 пальцях. Під час огляду встановлено: пальці стоп дещо набряклі, шкіра синюшна. Рухи у міжфалангових суглобах обмежені, болючі. Чутливість шкіри дещо знижена. Пульсація артерій тилу стоп збережена. Який попередній діагноз? For 12 hours, the woodcutter worked outdoors in windy weather at an air temperature of +4-5^oС. The shoes crushed the distal parts of the feet. Complaints of stabbing, burning pain in 1-3 toes, pain in the interphalangeal joints. During the examination, the toes are slightly swollen, the movements in the interphalangeal joints are somewhat reduced stop is preserved. What is the previous diagnosis?

Гострий артрит міжфалангових суглобів Acute arthritis of the interphalangeal joints

Відмороження І ступеня I degree frostbite

Критична ішемія стоп Critical foot ischemia

Синдром тривалого здавлення Prolonged squeeze syndrome

Облітеруючий ендартеріїт Endarteritis obliterans

811 / 1500
Під час розслідування групового захворювання ботулізмом (захворіло 5 осіб), було встановлено, що ці постраждалі вживали різні страви, серед яких були: декілька видів сирів, тушкована свинина, смажена картопля, риба домашнього посолу, салати з свіжих овочів. Яка з перелічених страв є найвірогіднішою причиною захворювання? During the investigation of a group case of botulism (5 people fell ill), it was established that these victims consumed various foods, among which were: several types of cheese, stewed pork, fried potatoes, homemade salted fish, salads from fresh vegetables. Which of the listed dishes is the most likely cause of the disease?

Салати Salads

Тушкована свинина Pork Stew

Сири Raw

Смажена картопля Fried potatoes

Риба домашнього посолу Home Ambassador Fish

812 / 1500
У пацієнта 47-ми років мітральний стеноз після гострої ревматичної лихоманки. Скаржиться на прогресуючу задишку, кашель, виражену загальну слабкість. На даний момент не може виконувати легку побутову роботу. Найбільш доцільна тактика лікування? A 47-year-old patient has mitral stenosis after acute rheumatic fever. He complains of progressive shortness of breath, cough, severe general weakness. At the moment, he cannot perform light household work . The most appropriate treatment tactic?

Препарати наперстянки Dingular drugs

Антибактеріальна терапія Antibacterial therapy

Призначення антикоагулянтів Prescription of anticoagulants

Призначення діуретиків Prescription of diuretics

Хірургічне лікування Surgical treatment

813 / 1500
54-річний чоловік звернувся до лікаря зі скаргами на гострий біль за грудниною та задишку. В положенні лежачи біль посилює-ться. В анамнезі гіперхолестеринемія протягом 4-х років (лікує симвастатином). Задишку пов'язує із нещодавнім захворюванням на грип. При фізикальному обстеженні пульс - 92/хв., частота дихання - 16/хв., артеріальний тиск - 140/80 мм рт.ст. Під час огляду помітно розширені шийні вени. Аускультативних змін в легенях та серці не виявлено. На ЕКГ у всіх відведеннях елевація сегменту ST. На рентгенограмі збільшення тіні серця. Який діагноз є найбільш імовірним? A 54-year-old man consulted a doctor with complaints of sharp chest pain and shortness of breath. The pain worsens when lying down. He has a history of hypercholesterolemia for 4 years (treats with simvastatin).During the physical examination, pulse - 16/min, blood pressure - 140/80 mmHg veins. No changes in the lungs and heart are detected. On the ECG, the ST segment is elevated. What is the most likely diagnosis?

Перикардит Pericarditis

Міокардит Myocarditis

- -

Гострий інфаркт міокарда Acute myocardial infarction

Тромбоемболія легеневої артерії (ТЕЛА) Thromboembolism of the pulmonary artery (PEA)

814 / 1500
Жінка 60 років, скаржиться на ниючий біль у колінних суглобах, який виникає під час рухів та підсилюється увечері, рухи супроводжуються хрускотом. Об'єктивно встановлено: вага - 120 кг, зріст - 160 см. Колінні суглоби деформовані, визначається крепітація. На рентгенограмі спостерігається: звуження суглобової щілини, бокові остеофіти. Який із наведених діагнозів є найімовірнішим? A 60-year-old woman complains of aching pain in the knee joints, which occurs during movements and worsens in the evening, movements are accompanied by a crunch. Objectively established: weight - 120 kg, height - 160 cm. Knee joints are deformed, crepitus is detected. On the radiograph: narrowing of the joint space, lateral osteophytes. Which of the following diagnoses is the most likely?

Подагричний артрит Gouty arthritis

Реактивний артрит Reactive arthritis

Ревматоїдний артрит Rheumatoid arthritis

Псоріатичний артрит Psoriatic arthritis

Остеоартроз Osteoarthrosis

815 / 1500
Через порушення техніки безпеки під час роботи з джерелами іонізуючого випромінювання робітник упродовж години піддавався загальному опроміненню в дозі близько 8 Гр. Яка форма променевої хвороби розвинулася у нього? Due to a violation of safety techniques while working with sources of ionizing radiation, the worker was exposed to general radiation at a dose of about 8 Gy for an hour. What form of radiation sickness did he develop?

Гостра променева хвороба, токсемічна форма Acute radiation sickness, toxemic form

Гостра променева хвороба, церебральна форма Acute radiation sickness, cerebral form

Хронічна променева хвороба, кістковомозкова форма Chronic radiation sickness, bone marrow form

Гостра променева хвороба, кишкова форма Acute radiation sickness, intestinal form

Гостра променева хвороба, кістковомозкова форма Acute radiation sickness, bone marrow form

816 / 1500
У хворої жінки наявність ушитої рани лівої підколінної ямки, постійний біль середньої інтенсивності, гнійні виділення, підвищення температури тіла до 38,5^oС. 5 діб тому отримала поранення лівої підколінної артерії холодною зброєю. На артерію було накладено судинний шов. За 3 доби підвищилася температура тіла, з'явились болі в нозі, набряк і гіперемія країв ушитої рани, гнійні виділення з неї. Після розпускання швів виникла інтенсивна артеріальна кровотеча, яку було зупинено шляхом накладання джгута <<Омега>> на стегно. Яка подальша тактика? The sick woman has a sutured wound of the left popliteal fossa, constant pain of medium intensity, purulent secretions, an increase in body temperature to 38.5°C. She was injured 5 days ago of the left popliteal artery with a cold weapon. In 3 days, the body temperature increased, swelling and purulent discharge appeared from it. After the sutures were dissolved, intense arterial bleeding occurred, which was stopped by applying the <> tourniquet. What is the next tactic?

Діатермокоагуляція кровоточивої судини Diathermocoagulation of a bleeding vessel

Перев'язка судини на протязі Vessel ligation during

Перев'язка судини в рані Vessel ligation in a wound

Туга тампонада рани Tight wound tamponade

Перев'язка судини на протязі та прошивання її в рані Ligading of a blood vessel in a draft and stitching it in a wound

817 / 1500
У пацієнта віком 27 років після переохолодження виникли підвищення температури тіла до 38,7^oС, біль в поперековій ділянці, слабкість, головний біль. Під час огляду виявлено позитивний симптом Пастернацького зліва. У загальному аналізі сечі спостерігаються піурія, бактеріурія. Який діагноз найімовірніший? A 27-year-old patient after hypothermia experienced an increase in body temperature to 38.7°C, pain in the lumbar region, weakness, headache. During the examination, a positive Pasternak's symptom on the left. Pyuria, bacteriuria are observed in the general analysis of urine. What is the most likely diagnosis?

Гострий гломерулонефрит Acute glomerulonephritis

Паранефральний абсцес Paranephric abscess

Гострий цистит Acute cystitis

Ниркова колька Renal colic

Гострий пієлонефрит Acute pyelonephritis

818 / 1500
Жінка 25-ти років, яка хворіє на цукровий діабет з 9-річного віку, надійшла у нефрологічне відділення із значними набряками на ногах, обличчі, руках. Артеріальний тиск - 200/110 мм рт.ст.; Нb - 90 г/л, креатинін крові - 850 мкмоль/л, білок сечі - 1,0 г/л, лейкоцити - 10-15 в полі зору. Клубочкова фільтрація - 10 мл/хв. Яка тактика лікаря? A 25-year-old woman, who has been suffering from diabetes since the age of 9, was admitted to the nephrology department with significant edema on her legs, face, and hands. Blood pressure - 200/110 mm Hg; Hb - 90 g/l, blood creatinine - 850 μmol/l, leukocytes - 10-15 in the field of vision. Glomerular filtration - 10 ml/ min. What are the doctor's tactics?

Переведення у ендокринологічний диспансер Transfer to the endocrinological dispensary

Активна консервативна терапія діабетичної нефропатії Active conservative therapy of diabetic nephropathy

Дієтотерапія Dietotherapy

Переведення у відділення гемодіалізу Transfer to the hemodialysis department

Трансплантація печінки Liver transplant

819 / 1500
Чоловік 72-х років звернувся до лікаря зі скаргами на задишку, кашель, набряки гомілок, збільшення живота. Хворіє більше ніж 20 років на хронічне обструктивне захворювання легень (ХОЗЛ). Останні 3 роки є інвалідом II групи у зв'язку з патологією серця. При фізикальному обстеженні пульс - 92/хв, артеріальний тиск - 120/70 мм рт.ст., частота дихання - 24/хв. Шкірні покриви ціанотичні, набряки гомілок, при пальпації органів черевної порожнини позитивний симптом флуктуації. При аускультації акцент II тону над легеневою артерією, сухі хрипи над всією поверхнею легень. Який механізм розвитку змін з боку серця у цього хворого є найбільш імовірним? A 72-year-old man went to the doctor with complaints of shortness of breath, cough, swelling of the legs, an enlarged abdomen. He has been suffering from chronic obstructive pulmonary disease (COPD) for more than 20 years For the last 3 years, he has been disabled due to heart disease. On physical examination, the pulse is 92/min, the respiratory rate is 24/min, and the skin is edematous shins, during palpation of the abdominal organs, a positive symptom of fluctuation. During auscultation, the accent of the II tone over the pulmonary artery, dry rales over the entire surface of the lungs. What is the most likely mechanism of the development of changes on the part of this patient?

Рефлекс Китаєва Reflex Kitaev

Дихальний рефлекс Respiratory reflex

Кардіоваскулярний рефлекс Cardiovascular reflex

Рефлекс Бейнбріджа Bainbridge Reflex

Вторинна легенева гіпертензія Secondary pulmonary hypertension

820 / 1500
Підліток госпіталізований зі скаргами на обмеженість рухів у нижніх кінцівках. Захворювання розпочалось 2 дні тому з підвищення температури тіла до 38,3^oC, рідких випорожнень 3-4 рази на добу. Об'єктивно: температура тіла - 36,8^oC, активні рухи в нижніх кінцівках відсутні, у зоні ураження - арефлексія, гіпотонія м'язів, чутливість збережена. Менінгеальні симптоми слабко позитивні. Яке захворювання можна припустити? A teenager was hospitalized with complaints of limited movement in the lower limbs. The disease began 2 days ago with an increase in body temperature to 38.3^oC, loose stools 3-4 times per day. Objectively: body temperature - 36.8°C, no active movements in the affected area, muscle hypotonia. Meningeal symptoms are weakly positive?

Поліомієліт Polio

Герпетичний менінгоенцефаліт Herpetic meningoencephalitis

Ботулізм Botulism

Cубарахноїдальний крововилив Cubarachnoid hemorrhage

Травма поперекового відділу хребта Injury of the lumbar spine

821 / 1500
Чоловік 30 років, пресувальник кабелів, скаржиться на млявість, погіршення пам'яті, біль у кінцівках. Об'єктивно спостерігаються блідість шкірних покривів, анемія, ретикулоцитоз, базофільна зернистість еритроцитів, підвищений вміст порфіріну в сечі. Симптоми якого захворювання спостерігаються у пацієнта? A 30-year-old man, a cable crimper, complains of lethargy, memory impairment, pain in the limbs. Objectively, pallor of the skin, anemia, reticulocytosis, basophilic granularity of erythrocytes, increased content of porphyrin in the urine. Symptoms of which disease are observed in the patient?

Азбестоз Asbestosis

Сатурнізм Saturnism

Сидероз Siderosis

Меркуріалізм Mercurialism

Бериліоз Berylliosis

822 / 1500
Пацієнтка віком 30 років скаржиться на відсутність менструацій впродовж року. З анамнезу відомо про наявність масивної післяпологової кровотечі. Об'єктивно спостерігається: вторинні статеві органи розвинені правильно, оволосіння за жіночим типом. Під час бімануального дослідження виявлено: матка та придатки без особливостей. Яка можлива причина аменореї? A 30-year-old patient complains of the absence of menstruation for a year. From the anamnesis, it is known about the presence of massive postpartum bleeding. Objectively observed: the secondary genitals are developed correctly, hair growth is female type. During the bimanual examination, the uterus and appendages were found without features. What is the possible cause of amenorrhea?

Гіпоталамічна недостатність Hypothalamic insufficiency

Недостатність яєчників Ovarian insufficiency

Порушення щитоподібної залози Thyroid disorder

Некроз кори наднирників Necrosis of the adrenal cortex

Некроз гіпофізу Necrosis of the pituitary gland

823 / 1500
Під час проведення медичного огляду студентів, лікар звернув увагу на студента віком 18 років зростом 176 см та масою тіла - 68 кг, у якого спостерігалась сухість шкіри та гіперкератоз. Під час спеціального обстеження виявлено суттєве зниження гостроти сутінкового зору. Дефіцит якого нутрієнта призводить до виникнення цього стану? During the medical examination of students, the doctor noticed an 18-year-old student with a height of 176 cm and a body weight of 68 kg, who had dry skin and hyperkeratosis. During a special examination, a significant decrease in the acuity of twilight vision was detected. What nutrient deficiency leads to this condition?

Вітаміну А Vitamin A

Селену Selena

Вітаміну С Vitamin C

Вітаміну В 1 Vitamin B 1

Фосфору Phosphorus

824 / 1500
63-річний чоловік звернувся до лікаря зі скаргами на пекучий та ниючий біль шкіри грудної клітки протягом 5-ти днів. Три дні тому на цій ділянці виникло почервоніння та макулопапульозна висипка, яка через день прогресувала у везикули. По буднях він доглядає двох онуків віком 1 та 3 роки, які щеплені згідно Національного календаря профілактичних щеплень. Фізикальне обстеження виявило везикулярний висип на грудній клітці ззаду в межах дерматомів T6-T8 . Неврологічний статус без змін. Який метод постконтактної профілактики дітям буде найбільш ефективним та доречним? A 63-year-old man consulted a doctor with complaints of burning and aching pain in the skin of the chest for 5 days. Three days ago, redness and maculopapular swelling appeared in this area a rash that progressed to vesicles every day.He takes care of two grandsons aged 1 and 3 who are vaccinated according to the National Immunization Schedule.Physical examination revealed a vesicular rash on the posterior chest within dermatomes T6-T8. What method of post-contact prevention for children will be the most effective and appropriate?

Призначити терапію ацикловіром дідусеві Prescribe acyclovir therapy to grandfather

Призначити дітям специфічний імуноглобулін до збудника Varicella Zoster Prescribe specific immunoglobulin for Varicella Zoster to children

Уникати контакту з дідусем до зникнення у нього висипу Avoid contact with grandfather until his rash disappears

Призначити Varicella Zoster-вмісну вакцину дітям Prescribe Varicella Zoster-containing vaccine to children

Призначити терапію ацикловіром дітям Prescribe acyclovir therapy for children

825 / 1500
У хворого 30-ти років через 3 доби після розкриття флегмони шиї з'явилися скарги на задишку, гарячку, біль за грудиною, що посилювався під час закидання голови назад. Стан хворого прогресивно погіршувався. В ході рентгенологічного обстеження виявлено розширення тіні середостіння й зниження прозорості його відділів. Яке ускладнення розвинулося в даного хворого? A 30-year-old patient complained of shortness of breath, fever, pain behind the sternum, which worsened when the head was thrown back, 3 days after opening the phlegmon of the neck The patient's condition progressively worsened. During the X-ray examination, an expansion of the mediastinum and a decrease in the transparency of its parts were found.

Заглотковий абсцес Pharyngeal abscess

Ексудативний перикардит Exudative pericarditis

Інфекційний ендокардит Infective endocarditis

Гострий гнійний медіастиніт Acute purulent mediastinitis

Гнійний тиреоїдит Suppurative thyroiditis

826 / 1500
Дівчинка 13-ти років скаржиться на головний біль, запаморочення, відчуття пульсації в голові, шум у вухах, носові кровотечі, серцебиття, біль в ділянці серця, задишку. При огляді блідість шкірних покривів. При пальпації виявлено ослаблення пульсу на стегнових артеріях. Артеріальний тиск на верхніх кінцівках - 125/60 мм рт.ст., на нижніх - 110/60 мм рт.ст. Аускультативно гучний систолічний шум, що іррадіює у сонні артерії. Який з перерахованих діагнозів є найбільш імовірним? A 13-year-old girl complains of headache, dizziness, pulsation in the head, tinnitus, nosebleeds, palpitations, pain in the heart area, shortness of breath. Palpation of the skin showed a weakening of the pulse in the upper extremities - 125/60 mm Hg arteries. Which of the listed diagnoses is the most probable?

Коарктація аорти Coarctation of the aorta

Стеноз устя легеневої артерії Stenosis of the mouth of the pulmonary artery

Стеноз устя аорти Stenosis of the mouth of the aorta

Дефект міжшлуночкової перетинки Defect of interventricular membrane

Відкрита артеріальна протока Open ductus arteriosus

827 / 1500
Чоловік 47 років скаржиться на безсоння, важкість в усьому тілі, постійно пригнічений настрій. Вважає себе нікчемним, неспроможним. Гадає, що є тягарем для своїх рідних, воліє померти. Пригнічений, малорухливий, обличчя гіпомімічне, скорботне. Мова тиха, монотонна, відповіді короткі. Який діагноз є найбільш імовірним? A 47-year-old man complains of insomnia, heaviness in the whole body, constantly depressed mood. He considers himself worthless, incapable. He thinks that he is a burden to his relatives, he prefers to die . Depressed, sedentary, hypomimic, speechless, short answers. What is the most likely diagnosis?

Невротична депресія Neurotic depression

Атеросклеротична депресія Atherosclerotic depression

Початкова стадія хвороби Альцгеймера Initial stage of Alzheimer's disease

Великий депресивний розлад Major Depressive Disorder

Шизофренія з пізнім дебютом Schizophrenia with late onset

828 / 1500
У дитини віком 2 доби, народженої на 32-ому тижні вагітності з вагою 1700 г наростають зміни з боку дихальної системи, що з'явилися через 8 годин після народження. У матері дитини - ІІІ вагітність, 2 пологи, абортів не було. Попередня дитина загинула від синдрому дихальних розладів. Об'єктивно спостерігається: оцінка за шкалою Сільвермана 6 балів, дихання з порушеним ритмом, апное, звучний видих, зниження тонусу м'язів. Під час аускультацїі: дихання помірно послаблене, вологі хрипи з обох боків. Рентгенологічно виявлено: наявність нодозно-ретикулярної сітки. Чим зумовлений синдром дихальних розладів у дитини? A 2-day-old child born at the 32nd week of pregnancy with a weight of 1700 g has increasing changes in the respiratory system that appeared 8 hours after birth The child's mother - 3rd pregnancy, no abortions. The previous child died from respiratory disorders. Objectively observed: 6 points on the Silverman scale, breathing with a disturbed rhythm, apnea, decreased muscle tone. During auscultation: breathing is moderately weakened, wheezing on both sides. X-ray revealed: the presence of a reticular mesh. What is the cause of the child's respiratory disorders?

Набряково-геморагічним синдромом Oedema-hemorrhagic syndrome

Ателектазами легенів Lung atelectasis

Синдромом гіалінових мембран Hyaline membrane syndrome

Внутрішньоутробною пневмонією Intrauterine pneumonia

Діафрагмальною грижею Diaphragmatic hernia

829 / 1500
Жінка 36 років скаржиться на головний біль, парестезії, м'язову слабкість, судоми, спрагу, поліурію. Об'єктивно спостерігається: температура - 36,6^oС, ЧДР - 18/хв., пульс - 92/хв., АТ - 180/110 мм рт. ст. На ЕКГ видно: зниження сегменту ST. Вміст калію у сироватці крові знижений, натрію - підвищений, активність реніну плазми значно знижена, концентрація альдостерону підвищена. Питома вага сечі протягом доби не перевищує 1,008-1,011, реакція лужна. Спіронолактоновий тест позитивний. КТ встановлено: правий наднирник збільшений. Який діагноз, найімовірніше, можна поставити пацієнтці? A 36-year-old woman complains of headache, paresthesias, muscle weakness, cramps, thirst, polyuria. Objectively observed: temperature - 36.6^oС , HR - 18/min., BP - 180/110 mm Hg. Decreased potassium content in blood serum, increased plasma renin activity. the concentration of aldosterone is increased. The specific gravity of the urine does not exceed 1.008-1.011. The spironolactone test is positive. The right adrenal gland is enlarged. What is the most likely diagnosis for the patient?

Андростерома Androsteroma

Синдром Іценка-Кушинга Itsenko-Cushing syndrome

Феохромоцитома Pheochromocytoma

Синдром Конна Conn Syndrome

Нецукровий діабет Diabetes insipidus

830 / 1500
Пацієнт віком 36 років скаржиться на напади задухи з переважно утрудненим видихом, які відбуваються до 2-3 разів на день і знімаються інгаляціями beta_2-адреноміметиків. Хворіє протягом 10 років. Об'єктивно спостерігається: грудна клітка розширена, над легенями коробковий перкуторний тон, під час аускультації вислуховується жорстке дихання з подовженим видихом. Під час лабораторного дослідження мокроти виявлено багато еозинофілів, кристали Шарко-Лейдена та спіралі Куршмана. Який попередній діагноз? A 36-year-old patient complains of attacks of suffocation with mainly difficult exhalation, which occur up to 2-3 times a day and are relieved by inhalation of beta_2-adrenomimetics. He has been sick for 10 years . Objectively observed: the chest is expanded, a percussion sound is heard during auscultation. During the laboratory examination of the sputum, many eosinophils, Charcot-Leyden crystals and Kurschman's spirals are found?

Карциноїдний синдром Carcinoid syndrome

Спонтанний пневмоторакс Spontaneous pneumothorax

Бронхіальна астма Bronchial asthma

Серцева астма Cardiac Asthma

Хронічний обструктивний бронхіт Chronic obstructive bronchitis

831 / 1500
Робітник очисної лави вугільної шахти зі стажем роботи 15 років звернувся до лікаря зі скаргами на біль у грудній клітці, задишку, сухий кашель, швидку втомлюваність. На рентгенограмі легень дифузний інтерстиціальний фіброз, емфізема, дрібновогнищеві утворення 3-5 мм у нижніх і середніх відділах. На спірограмі порушення функції зовнішнього дихання відсутні. За даними санітарно-гігієнічної характеристики робочої зони запиленість повітря кам'я-ним вугіллям становить 138 мг/м^3, вміст силіцію (IV) оксиду 2% (норма - 10 мг/м^3, якщо вугільний пил вміщує до 5% вільного діоксиду кремнію). Який діагноз є найбільш імовірним? A 15-year-old worker at a coal mine scrubber went to the doctor with complaints of chest pain, shortness of breath, dry cough, and rapid fatigue. On the X-ray, the lungs are diffuse interstitial fibrosis, emphysema, small foci of 3-5 mm in the lower and middle parts. According to the sanitary and hygienic characteristics of the working area, the dust content of the air is 138 mg/m^3. silicon (IV) oxide 2% (norm - 10 mg/m^3, if coal dust contains up to 5% free silicon dioxide). What diagnosis is most likely?

Коніотуберкульоз Coniotuberculosis

Антракоз Anthracosis

Силікоз Silicosis

Аденокарцинома Adenocarcinoma

Саркоїдоз Sarcoidosis

832 / 1500
Чоловік ремонтував огорожу і травмував дротом лікоть правої руки. Через 3 дні з'явилися набряк, гіперемія, обмеження обсягу рухів у суглобі, сильна болючість, місцеве та загальне підвищення температури. Об'єктивно спостерігається: правий лікоть збільшений в розмірах, шкіра над ним набрякла, гіперемована, регіонарні лімфатичні вузли збільшені, ущільнені, болючі. Під час пальпації правого ліктя спостерігаються різкий біль, флуктуація. Встановіть попередній діагноз. A man was repairing a fence and injured his right elbow with a wire. After 3 days, swelling, hyperemia, limitation of range of motion in the joint, severe pain, local and general elevation appeared Objectively, the right elbow is enlarged, the skin over it is swollen, the regional lymph nodes are enlarged, and painful. During palpation of the right elbow, there is a sharp pain, fluctuation. Establish a preliminary diagnosis.

Абсцес Abscess

Гідраденіт Hydradenitis

Бешиха Beshikha

Лімфаденіт Lymphadenitis

Бурсит Bursitis

833 / 1500
Пацієнтка віком 29 років скаржиться на безпліддя та порушення менструального циклу по типу олігоменореї. Об'єктивно спостерігається: зріст - 160 см, маса тіла - 91 кг, відзначається ріст волосся на обличчі, стегнах. Під час бімануального дослідження з обох сторін виявлено: збільшені щільної консистенції яєчники розмірами 5x6 см. Ці ж дані підтверджені під час УЗД. Який найімовірніший діагноз? A 29-year-old female patient complains of infertility and irregular menstrual cycle type oligomenorrhea. Objectively observed: height - 160 cm, body weight - 91 kg, growth is noted hair on the face, thighs. During a bimanual examination on both sides, enlarged ovaries measuring 5x6 cm were found. The same data were confirmed during ultrasound. What is the most likely diagnosis?

Передменструальний синдром Premenstrual syndrome

Адреногенітальний синдром Adrenogenital syndrome

Синдром склерокістозних яєчників (Штейна-Левенталя) Syndrome of sclerocystic ovaries (Stein-Leventhal)

Андробластома яєчників Androblastoma of the ovaries

Хронічний двохсторонній аднексит Chronic bilateral adnexitis

834 / 1500
У чоловіка 55-ти років 2 роки тому верифіковано цироз печінки. Впродовж 2-х тижнів стан погіршився, з'явились сонливість, запаморочення, втрата орієнтації у просторі і часі. Напередодні відзначав день народження в ресторані. Який синдром зумовив погіршення стану хворого? A 55-year-old man was diagnosed with liver cirrhosis 2 years ago. Over the course of 2 weeks, his condition worsened, drowsiness, dizziness, loss of orientation in space and The day before, he celebrated his birthday in a restaurant. What syndrome caused the patient's condition to deteriorate?

Печінкової енцефалопатії Hepatic encephalopathy

Жовтяниці Jaundice

Гіперспленізму Hypersplenism

Цитолізу Cytolysis

Холестазу Cholestasis

835 / 1500
Жінка 37-ми років звернулася до лікаря зі скаргами на слабкість, швидку стомлюваність, запаморочення, поганий апетит, утруднене ковтання їжі. В анамнезі під час вагітності 5 років тому у хворої відзначалося зниження гемоглобіну до 86 г/л. При лабораторному дослідженні в загальному аналізі крові: еритроцити - 3,4·10^12/л, Hb- 70 г/л, колірний показник - 0,6, ретикулоцити - 2%, лейкоцити - 4,7·10^9/л, е- 2%, п- 3%, с- 64%, лімф.- 26%, мон.- 5%, швидкість осідання еритроцитів - 40 мм/год. Сироваткове залізо - 6,3 мкмоль/л. Який діагноз є найбільш імовірним? A 37-year-old woman consulted a doctor with complaints of weakness, rapid fatigue, dizziness, poor appetite, difficulty swallowing food. History during pregnancy 5 years ago the patient had a decrease in hemoglobin to 86 g/l. During the laboratory examination in the general blood test: erythrocytes - 3.4·10^12/l, Hb - 70 g/l, color index - 0.6, reticulocytes - 2%, leukocytes - 4.7·10^9/l, e - 2%, p - 64%, lymph - 26%, mon. - 5%, erythrocyte sedimentation rate - 40 mm/h. Serum iron - 6.3 μmol/l. What diagnosis is most likely?

Фолієводефіцитна анемія Folate deficiency anemia

Аутоімунна гемолітична анемія Autoimmune hemolytic anemia

Анемія Мінковського-Шоффара Minkowski-Shofar Anemia

Гіпопластична анемія Hypoplastic anemia

Залізодефіцитна анемія Iron deficiency anemia

836 / 1500
Жінка віком 64 роки хворіє на цукровий діабет 2 типу, отримує цукрознижуючу терапію: метформін по 850 мг 3 рази на добу. Глікемія натщесерце - 7,8 ммоль/л, через дві години після їжі - 10,5 ммоль/л, глікозильований гемоглобін - 8,7%. Під час огляду лікарем-офтальмологом було діагностовано діабетичну проліферативну ретинопатію сітківки обох очей. Яка подальша тактика лікування? A 64-year-old woman has type 2 diabetes, receives hypoglycemic therapy: metformin 850 mg 3 times a day. Fasting blood glucose - 7.8 mmol/l , two hours after a meal - 10.5 mmol/l, glycosylated hemoglobin - 8.7%. During the examination by an ophthalmologist, diabetic retinopathy of both eyes was diagnosed. What are the further treatment tactics?

До цукрознижуючої терапії додати гліклазид 160 мг в день Add gliclazide 160 mg per day to hypoglycemic therapy

Перевести на інсулінотерапію Transfer to insulin therapy

Збільшити дозу метформіну до 3 500 мг на добу Increase metformin dose to 3,500 mg per day

До цукрознижуючої терапії додати дапагліфлозин 10 мг в день Add dapagliflozin 10 mg per day to hypoglycemic therapy

Не змінювати терапію Do not change therapy

837 / 1500
Чоловік 45 років був доставлений машиною швидкої допомоги з підприємства, яке виготовляє анілін, з діагнозом: транзиторна ішемична атака. Об'єктивно встановлено: шкірні покриви та слизові оболонки синюшні. Мова дизартрична. Наявна дезорієнтація в просторі. У крові виявлено: еритроцити - 4,6·10^12/л, Нb - 143 г/л, КП - 0,9, лейкоцити - 5,6·10^9/л, тільця Гейнца - 14%, ретикулоцити - 18%, метгемоглобін - 36%, ШОЕ - 5 мм/год. Пацієнту встановлено діагноз: гостра інтоксикація аніліном середньої тяжкості. Який засіб антидотної терапії є найефективнішим в цьому разі? A 45-year-old man was brought by ambulance from an aniline manufacturing company with a diagnosis of transient ischemic attack. Objectively established: the skin and mucous membranes are bluish Disorientation in space was found: erythrocytes - 4.6·10^12/l, KP - 0.9, leukocytes - 5.6·10^1/l, Heinz bodies - 14%, reticulocytes - 36%, ESR - 5 mm/h. The patient is diagnosed with acute aniline intoxication. What is the most effective antidote therapy in this case?

Сукцимер Succimer

Пентацин Pentacin

Натрія тіосульфат Sodium thiosulfate

Десферал Desferal

Метиленовий синій Methylene Blue

838 / 1500
Дівчина 10 років скаржиться на вогнища облисіння шкіри голови. Хворіє протягом 3 тижнів. Об'єктивно встановлено: на шкірі волосистої частини голови кілька невеликих вогнищ овальної форми з нечітко окресленими краями. Шкіра у вогнищах рожево-червоного кольору, волосся зламане на рівні 4-5 мм або на рівні шкіри. Під лампою Вуда не відмічається зеленого кольору вогнищ. Яка найімовірніша патологія? A 10-year-old girl complains of foci of baldness on the scalp. She has been sick for 3 weeks. Objectively established: on the scalp, several small oval-shaped foci with vaguely defined by the edges. The skin in the foci is red, the hair is broken at the level of 4-5 mm. Under the Wood's lamp, the foci are not green. What is the most likely pathology?

Фавус Favus

Мікроспорія волосистої частини голови Microsporia of scalp

Ідеопатична алопеція Ideopathic alopecia

Тріхофітія волосистої частини голови Trichophytia of the scalp

Сифілітична алопеція Syphilitic alopecia

839 / 1500
Молода жінка звернулася до лікаря зі cкаргами на задишку при незначному фізичному навантаженні, інколи набряки на ногах, слабкість. В анамнезі часті запальні захворювання легень. Задишка вперше з'явилась у дитинстві та поступово зростала. При фізикальному обстеженні пульс - 78/хв., артеріальний тиск - 130/80 мм рт.ст., шкіра ціанотична, набряки на ногах. Аус-культативно тони серця ритмічні, грубий систолічний шум уздовж лівого краю груднини, акцент та розщеплення II тону над легеневою артерією. Пальпаторно печінка збільшена. Вибір лікарем якого діагностичного методу буде найбільш доречним? A young woman consulted a doctor with complaints of shortness of breath during minor physical exertion, sometimes swelling of the legs, weakness. She has a history of frequent inflammatory lung diseases. Shortness of breath appeared for the first time in childhood and gradually increased. During physical examination, pulse - 78/min., blood pressure - 130/80 mm Hg, cyanotic skin, rhythmic heart sounds along the left sternal edge, accent and splitting of the II tone over the pulmonary artery. The liver is enlarged. Which diagnostic method will be the most appropriate?

Катетеризація серця Heart Catheterization

Біопсія міокарда Myocardial biopsy

Ехокардіографія Echocardiography

Електрокардіограма Electrocardiogram

Коронароангіографія Coronary angiography

840 / 1500
Пацієнтці 32-х років під час профогляду проведено кольпоскопію і виявлено наступне: ділянка пунктуації в І зоні шийки матки, цитологічно визначається дисплазія 3 ступеня. Після дообстеження встановлено діагноз: Ca in situ шийки матки. Який метод лікування слід обрати? A 32-year-old patient underwent a colposcopy during a professional examination and the following was found: a punctuation area in the 1st zone of the cervix, cytologically determined grade 3 dysplasia. After further examination, the diagnosis was established: Ca in situ of the cervix. What method of treatment should be chosen?

Конізація шийки матки Conization of the cervix

Екстирпація матки з додатками Extirpation of the uterus with attachments

Операція Вертгайма Operation Wertheim

Екстирпація матки без додатків Extirpation of the uterus without attachments

Променева терапія Radiotherapy

841 / 1500
Пацієнт страждає на сечокам'яну хворобу. Періодично спостерігається оксалурія. Які продукти слід обмежити в дієті хворого? The patient suffers from urolithiasis. Oxaluria is observed periodically. What foods should be limited in the patient's diet?

Щавель, шпинат, томати Sorrel, spinach, tomatoes

М'ясо, риба Meat, fish

Жирна їжа Fat food

Молочні продукти Dairy products

Бульйонні консерви Canned broth

842 / 1500
Хворий 72-х років звернувся до лікаря зі скаргами на тремор рук, більше справа, загальну скутість, сповільнення ходи, мовлення, утруднення при виконанні звичайної домашньої роботи. При фізикальному обстеженні загальна брадикінезія, гіпомімія, емпростотонус, хода ''човгаюча'', ахейрокінез, тремор спокою в руках, D>S, підвищений м'язовий тонус за пластичним типом, феномен ''зубчастого колеса'', виражена постуральна нестійкість. Які ліки, насамперед, мають бути у схемі лікування цього хворого? A 72-year-old patient turned to the doctor with complaints of tremors in the hands, more in the right, general stiffness, slowness of gait, speech, difficulty in performing usual housework. on physical examination, general bradykinesia, hypomimia, emprostotonus, 'shuffling' gait, acheirokinesis, rest tremor in the hands, D>S, increased muscle tone according to the plastic type, 'cogwheel' phenomenon, marked postural instability , first of all, should be in the treatment scheme of this patient?

Препарати, що містять леводопу Drugs containing levodopa

Препарати, що покращують мікроциркуляцію Drugs that improve microcirculation

Холінолітики Cholinolytics

Антихолінестеразні препарати Anticholinesterase drugs

Антидепресанти Antidepressants

843 / 1500
У новонародженого хлопчика вагою 2200 г, 38 тижнів гестації, виявлено дефект міжшлуночкової перетинки, катаракту обох очей, нейросенсорну глухоту. Мати дитини в 12 тижнів вагітності перехворіла на грипоподібне захворювання, яке супроводжувалося висипом. Який діагноз найімовірніший у новонародженого? A newborn boy weighing 2,200 g, 38 weeks gestation, had a defect in the interventricular membrane, cataracts in both eyes, and sensorineural deafness. The child's mother fell ill with a flu-like illness at 12 weeks of pregnancy , which was accompanied by a rash. What is the most likely diagnosis in a newborn?

Синдром вродженої краснухи Congenital rubella syndrome

Вроджений лістеріоз Congenital listeriosis

Вроджена вітряна віспа Congenital chicken pox

Токсоплазмоз Toxoplasmosis

Цитомегаловірусна інфекція Cytomegalovirus infection

844 / 1500
У чоловіка 73 років на тлі гострого трансмурального переднього інфаркту міокарда спостерігається зниження систолічного тиску (70 мм рт. ст), діурезу (100 мл/добу), підвищення креатиніну крові (480 мкмоль/л). У сечі спостерігається протеїнурія (0,066 г/добу). Чим зумовлене зниження діурезу? A 73-year-old man with an acute transmural anterior myocardial infarction has decreased systolic pressure (70 mm Hg), diuresis (100 ml/day), increased creatinine blood (480 μmol/l). Proteinuria is observed in the urine (0.066 g/day). What is the reason for the decrease in diuresis?

<<Застійна>> нирка <> kidney

Гостра преренальна ниркова недостатність Acute prerenal renal failure

Гломерулонефрит, що швидко прогресує Rapidly progressive glomerulonephritis

Гострий тубулоінтерстиціальний нефрит Acute tubulointerstitial nephritis

Гостра ренальна ниркова недостатність Acute renal failure

845 / 1500
Чоловік 68 років, що перебуває в реанімаційному відділенні з діагнозом: гострий інфаркт міокарда, раптово втратив свідомість. Об'єктивно пульс та АТ не визначаються. На ЕКГ спостерігаються часті нерегулярні хвилі різної форми та амплітуди. Яке ускладнення розвинулося у хворого? A 68-year-old man, who is in the intensive care unit with a diagnosis of acute myocardial infarction, suddenly lost consciousness. Objectively, the pulse and blood pressure are not determined. On the ECG, frequent irregular waves of different shape and amplitude. What complication has developed in the patient?

Асистолія Asystole

Фібриляція шлуночків Ventricular fibrillation

Тромбоемболія легеневої артерії Thromboembolism of the pulmonary artery

Гостра серцева недостатність Acute heart failure

Пароксизмальна шлуночкова тахікардія Paroxysmal ventricular tachycardia

846 / 1500
Пацієнтка віком 27 років скаржиться на біль та ранкову скутість в дрібних суглобах рук, їх метеозалежність. Об'єктивно спостерігається: припухлість та деформація проксимальних міжфалангових та II, III п'ястково-фалангових суглобів. Під час рентгенологічного дослідження кистей виявлено: узури, ознаки остеопорозу. Який найімовірніший діагноз? A 27-year-old patient complains of pain and morning stiffness in the small joints of the hands, their weather dependence. Objectively observed: swelling and deformation of the proximal interphalangeal and II, III p 'of the costo-phalangeal joints. During the X-ray examination of the hands, there were signs of osteoporosis. What is the most likely diagnosis?

Ревматоїдний артрит Rheumatoid arthritis

Ревматичний поліартрит Rheumatoid polyarthritis

Системна склеродермія Systemic scleroderma

Системний червоний вовчак Systemic lupus erythematosus

Псоріатичний артрит Psoriatic arthritis

847 / 1500
Підліток 16 років скаржиться на слабкість, запаморочення, відчуття важкості в лівому підребер'ї. Об'єктивно спостерігається: шкіра та видимі слизові жовтяничні. Баштовий череп. Печінка +2 см, нижній край селезінки на рівні пупка. У крові виявлено: еритроцити - 2,7·10^12/л, Hb - 88 г/л, лейкоцити - 5,6·10^9/л, ШОЕ - 15 мм/год. Укажіть найвірогіднішу зміну рівня білірубіну в цього пацієнта: A 16-year-old teenager complains of weakness, dizziness, a feeling of heaviness in the left hypochondrium. Objectively observed: skin and visible mucous membranes are jaundiced. Turret skull. Liver + 2 cm, the lower edge of the spleen at the level of the navel. In the blood: erythrocytes - 2.7·10^12/l, Hb - 88 g/l, leukocytes - 5.6·10^9/l, ESR - 15 mm/ h. Indicate the most likely change in the bilirubin level in this patient:

Підвищення вільного та зв'язаного білірубінів Increased free and bound bilirubin

Підвищення вільного білірубіну Increased free bilirubin

Зниження вільного білірубіну Decreased free bilirubin

Зниження зв'язаного білірубіну Decreased bound bilirubin

Підвищення зв'язаного білірубіну Increased bound bilirubin

848 / 1500
У дитини віком 18 місяців з'явилося облисіння потилиці, неспокійний сон, підвищена пітливість. Про яку хворобу можна думати? An 18-month-old child developed baldness on the back of the head, restless sleep, increased sweating. What disease can you think about?

Рахіт Rickets

Хондродістрофія Chondrodystrophy

Анемія Anemia

Спазмофілія Spasmophilia

Фосфат-діабет Phosphate-Diabetes

849 / 1500
П'ятирічна дитина скаржиться на нападоподiбний спазматичний кашель. З анамнезу відомо: хворiє 2 тижнi з появи сухого кашлю, після лiкування, що було неефективним, появилися репризи. Пiд час нападу кашлю обличчя дитини червонiє, шийнi вени набухають. Об'єктивно спостерігається: пацієнтка блiда, набрякле обличчя , крововиливи в склери. Аускультативно вислуховується: над легенями коробковий перкуторний звук, сухi хрипи. Під час рентгенологiчного дослідження виявлено: пiдвищення прозоростi легеневих полiв, посилення бронхiального малюнку. У аналізі кровi спостерігається: лейкоцити - 16·10^9/л, лiмфоцити - 72%, ШОЕ - 4 мм/год. Який найімовірніший діагноз? A five-year-old child complains of an attack-like spasmodic cough. It is known from the anamnesis: he has been sick for 2 weeks since the onset of a dry cough, after treatment, which was ineffective, relapses appeared. Under during a coughing attack, the child's face turns red, the neck veins swell. Objectively, the patient is pale, hemorrhagic in the sclera. Auscultation is heard: a percussive sound over the lungs. During the X-ray examination, an increase in the transparency of the lung fields is detected in the blood analysis: leukocytes - 16·10^9/l, lymphocytes - 72%, ESR - 4 mm/h. What is the most likely diagnosis?

Ларинготрахеїт Laryngotrachitis

Туберкульозний бронхоаденiт Tuberculous bronchoadenitis

Кашлюк Whooping cough

Стороннє тiло в дихальних шляхах Foreign body in respiratory tract

Аденовiрусна iнфекцiя Adenovirus infection

850 / 1500
Чоловік 27 років скаржиться на сухий кашель, задишку під час найменшого навантаження, біль у грудній кліт-ці, підвищення температури до 37,3^oС упродовж 3 тижнів. Колись зловживав наркотиками. Дихання жорстке, хрипів немає, тахікардія - 120/хв. Рентгенологічно з обох сторін легенів спостеріга-ються інтерстиціальні зміни. Під час бронхоскопії в лаважній рідині виявлені Pneumocista carinii. Які препарати будуть найефективнішими для лікування цього пацієнта? A 27-year-old man complains of a dry cough, shortness of breath during the slightest exertion, pain in the chest, an increase in temperature up to 37.3°C for 3 weeks. Used to abuse drugs. Breathing is hard, tachycardia - 120/min. Interstitial changes are observed on both sides of the lungs. During bronchoscopy, Pneumocista carinii is found to be the most effective for the treatment of this patient.

Еритроміцин, рифампіцин Erythromycin, Rifampicin

Ампіцилін, нізорал Ampicillin, Nizoral

Інтерферон, ремантадин Interferon, Rimantadine

Цефамізин, нітроксолін Cefamycin, nitroxoline

Бісептол, кліндаміцин Biseptol, clindamycin

851 / 1500
Жінка віком 62 роки скаржиться на відчуття перебоїв у роботі серця, періодичні напади серцебиття, зниження працездатності, загальну кволість. Погіршення стану відзначає протягом останніх місяців. Звернулася до лікаря після запаморочення в голові з нетривалою втратою свідомості напередодні. Об'єктивно спостерігається: пульс - 52/хв, аритмічний. Під час аускультації шумів не виявлено. На ЕКГ спостерігається: ритм синусовий, неправильний, інтервал PQ - 0,20 сек. QRS - 0,08 сек. Поступове зменшення інтервалу RR з наступним випадінням PQRST. Яка найімовірніша причина такого стану пацієнтки? A 62-year-old woman complains of a feeling of interruptions in the work of the heart, periodic attacks of palpitations, reduced work capacity, general weakness. She notes the worsening of her condition during the last months. She consulted a doctor after dizziness with brief loss of consciousness the day before. Objectively observed: pulse - 52/min. During auscultation: sinus rhythm, irregular, QRS interval - 0. 08 sec. Gradual decrease in RR interval followed by drop in PQRST. What is the most likely cause of this condition of the patient?

Синоаурикулярна блокада Sino-auricular blockade

Трифасцикулярна блокада ніжок пучка Гіса Trifascicular blockade of the legs of the bundle of His

Атріовентрикулярна блокада ІІІ ступеня III degree atrioventricular block

Атріовентрикулярна блокада ІІ ступеня II degree atrioventricular block

Атріовентрикулярна блокада І ступеня Atrioventricular blockade of the 1st degree

852 / 1500
Хворий 36-ти років скаржиться на виражену задишку, біль у ділянці серця. Своє захворювання пов'язує з перенесеним 2 тижні тому грипом. Об'єктивно: сидить, нахилившись уперед. Обличчя набрякле, ціанотичне, також спостерігається набряк шийних вен. Межі серця розширені у обидві сторони, тони глухі, частота серцевих скорочень=пульс - 118/хв., артеріальний тиск - 90/60 мм рт.ст. У крові: швидкість осідання еритроцитів - 46 мм/год. На ЕКГ - низький вольтаж. На рентгенограмі - трапецієподібна тінь серця, ознаки застою у легенях. Яка лікувальна тактика? A 36-year-old patient complains of pronounced shortness of breath, pain in the heart area. He associates his illness with the flu he had 2 weeks ago. Objectively: he is sitting, The face is swollen, cyanotic, the borders of the heart are enlarged on both sides, heart rate is 118/min, blood pressure is 90/60 mmHg sedimentation of erythrocytes - 46 mm/h. On the ECG - a trapezoidal shadow of the heart, signs of stasis in the lungs?

Призначення глюкокортикостероїдів Prescription of glucocorticosteroids

Призначення антибіотиків Prescription of antibiotics

Призначення сечогінних Prescription of diuretics

Проведення перикардектомії Pericardectomy

Проведення пункції перикарда Pericard puncture

853 / 1500
Чоловік 65-ти років надійшов до стаціо-нару з приводу хірургічного лікування гострого некротичного панкреатиту. Оперативне втручання пройшло без ускладнень. На 5-й день після операції стан погіршився. При фізикальному обстеженні температура тіла підвищена до 38,5^oC, артеріальний тиск - 110/65 мм рт.ст., пульс - 118/хв., частота дихання - 28/хв., SpO_2 - 92%. При аускультації легень звучні хрипи у правій нижній ділянці легень. Післяопераційна рана загоюється добре, живіт при пальпації безболісний, ненапружений. Який діагноз є найбільш імовірним? A 65-year-old man was admitted to the hospital for surgical treatment of acute necrotizing pancreatitis. The operation went without complications. On the 5th day after the operation, the condition worsened During physical examination, the body temperature is elevated to 38.5°C, blood pressure - 110/65 mm Hg, pulse - 28/min, SpO_2 - 92% wheezing in the right lower part of the lungs. The postoperative wound is healing well, the abdomen is painless on palpation. What is the most likely diagnosis?

- -

Сепсис Sepsis

Медикаментозна лихоманка Medicine fever

Злоякісна гіпертермія Malignant hyperthermia

Синдром системної запальної відповіді неінфекційної природи Systemic inflammatory response syndrome of a non-infectious nature

854 / 1500
Чоловік віком 35 років звернувся до лікаря зі скаргами на різке збільшення ваги на 27 кг протягом 2 років, слабкість, випадіння волосся на голові, зниження потенції. Об'єктивно спостерігається: зріст - 174 см, вага - 104 кг. Відкладення жиру переважно на шиї та тулубі. Шкіра суха, багряно-ціанотичного кольору. АТ- 160/110 мм рт. ст. Під час лабораторного дослідження виявлено підвищення рівня АКТГ. На комп’ютерній томографії - мікроаденома гіпофіза. Який найбільш імовірний діагноз? A 35-year-old man consulted a doctor with complaints of a sharp weight gain of 27 kg over the course of 2 years, weakness, hair loss on the head, decreased potency. Objectively observed: height - 104 kg. Skin of a purple color - 160/110 mm Hg. computed tomography - pituitary microadenoma. What is the most likely diagnosis?

Синдром Іценка-Кушинга Itsenko-Cushing syndrome

Хвороба Аддісона Addison's disease

Хвороба Іценка-Кушинга Itsenko-Cushing disease

Аліментарне ожиріння Alimentary obesity

Гіпертонічна хвороба Hypertensive disease

855 / 1500
У жінки 32-х років скарги на двосторонній біль у руках і припухлість пальців, відчуття скутості впродовж кількох годин після пробудження, втомлюваність. Об'єктивно: набряк другого та третього п'ястково-фалангових і проксимальних міжфалангових суглобів, позитивні симптоми поперечного стискання кисті. Яке захворювання найбільш імовірне? A 32-year-old woman complains of bilateral pain in the hands and swelling of the fingers, a feeling of stiffness for several hours after waking up, fatigue. Objectively: swelling of the second and third metacarpal-phalangeal and proximal interphalangeal joints, positive symptoms of transverse compression of the hand. What disease is most likely?

Подагричний артрит Gouty arthritis

Ревматичний артрит Rheumatic arthritis

Остеоатрит Osteoarthritis

Реактивний артрит Reactive arthritis

Ревматоїдний артрит Rheumatoid arthritis

856 / 1500
Пацієнт 37-ми років надійшов у нефрологічне відділення з діагнозом: хронічна хвороба нирок, І ст. - мезангіопроліферативний гломерулонефрит. АТ- 140/90 мм рт.ст. Добова втрата білка із сечею становить - 5,1 г. У крові: загальний білок - 48,2 г/л; сечовина - 5,7 ммоль/л; креатинін - 76,3 мкмоль/л; холестерин - 8,1 ммоль/л. Яку групу препаратів необхідно призначити? A 37-year-old patient was admitted to the nephrology department with a diagnosis of chronic kidney disease, stage I - mesangioproliferative glomerulonephritis. Blood pressure - 140/90 mmHg. The daily loss of protein in the urine is 5.1 g. In the blood: total protein - 5.7 mmol/l; creatinine - 8.1 mmol/l. l. What group of drugs should be prescribed?

Нітрофурани Nitrofurans

Сульфаніламідні Sulfonamides

Антибіотики Antibiotics

Нестероїдні протизапальні Nonsteroidal anti-inflammatory drugs

Глюкокортикостероїди Glucocorticosteroids

857 / 1500
У хлопчика, який страждає на бронхіальну астму, напади задухи почали спостерігатися кілька разів на день. Під час останнього нападу інгаляція астмопенту ефекту не дала. Інтенсивна терапія напад не зняла. Дитину переведено в реанімаційне відділення з діагнозом «астматичний статус II ступеня». Що було провідним механізмом розвитку цього стану у дитини? In a boy suffering from bronchial asthma, attacks of suffocation began to be observed several times a day. During the last attack, inhalation of astmopent had no effect. Intensive therapy did not relieve the attack The child was transferred to the intensive care unit with a diagnosis of 'asthmatic status II'. What was the leading mechanism for the development of this condition in the child?

Посилена секреція тучними клітинами біологічно активних речовин Increased secretion of biologically active substances by mast cells

Набряк слизової оболонки бронхів Swelling of the bronchial mucosa

Повна рефрактерність (блокада) beta_2-адренорецепторів Complete refractoriness (blockade) of beta_2-adrenoceptors

Запалення слизової оболонки бронхів Inflammation of the bronchial mucosa

Спазм гладеньких м'язів бронхів Spasm of bronchial smooth muscles

858 / 1500
До гінеколога звернулася жінка 36-ти років зі скаргами на нерегулярні мізерні менструації впродовж останніх 6 місяців, посилений ріст волосся на обличчі та ''ог-рубіння'' голосу. В анамнезі два викидні, одні передчасні пологи. За даними УЗД матка та яєчники без особливостей. Яке обстеження доцільно провести для встановлення причини гормональних порушень? A 36-year-old woman came to the gynecologist with complaints of irregular scanty menstruation for the past 6 months, increased growth of facial hair and hoarse voice 'There is a history of two miscarriages, one premature birth. According to the ultrasound of the uterus and ovaries, it is advisable to conduct an examination to determine the cause of hormonal disorders.

Проведення рентген-дослідження турецького сідла Carrying out an X-ray examination of the Turkish saddle

Проведення гістероскопії Hysteroscopy

Каріотипування Karyotyping

Взяття біопсії ендометрію Endometrial biopsy

Гормональне обстеження Hormonal examination

859 / 1500
Під час огляду у дванадцятирічної дівчинки виявлено ущільнення і незначну припухлість у ділянці лівої грудної залози, гіперемія відсутня. Вкажіть подальші рекомендації. During the examination, a twelve-year-old girl was found to have thickening and slight swelling in the area of ​​the left mammary gland, there is no hyperemia. Specify further recommendations.

Динамічне спостереження без лікування Dynamic observation without treatment

УВЧ на грудну залозу UHF on breast gland

Параареолярний розріз із дренуванням Paraareolar incision with drainage

Антибіотикотерапія Antibiotic therapy

Радіарний розріз над припухлістю з наступним дренуванням Radial incision over swelling followed by drainage

860 / 1500
До лікарні машиною швидкої допомоги доставлено хворого у непритомному стані. Об'єктивно: температура тіла - 39^oC, судоми, шкіра червона, суха. Відомо, що потерпілий працює кочегаром в котельній. Який імовірний діагноз? An unconscious patient was brought to the hospital by ambulance. Objectively: body temperature - 39°C, convulsions, red, dry skin. It is known that the victim works as a stoker in the boiler room. What is the probable diagnosis?

Гостре харчове отруєння Acute food poisoning

Гіпертонічний криз Hypertensive crisis

Тепловий удар Heatstroke

Отруєння CO CO poisoning

ГРВІ ARV

861 / 1500
Дівчинка вагою 3100 г народилася від третьої вагітності в домашніх умовах. На 3-й день мати помітила дрібні крововиливи у пупковій ранці, а також ''червоні цятки'' на склерах очей. Мати на обліку по вагітності у допологовому періоді не перебувала. Вигодовує дитину грудним молоком за потребою. При фізикальному обстеженні субкон'юнктивальні геморагії. При лабораторному дослідженні концентрація гемоглобіну - 155 г/л, група крові у матері В (ІІІ) Rh-позитивна, у дитини - А (ІІ) Rh-негативна. Яка тактика з метою попередження прогресування цього стану у дитини буде найбільш доречною? A girl weighing 3100 g was born from the third pregnancy at home. On the 3rd day, the mother noticed small hemorrhages in the umbilical wound, as well as 'red spots' on the sclera of the eyes. The mother was not registered during the prenatal period. During the physical examination, the hemoglobin concentration in the mother was 155 g/l -positive, the child has A (II) Rh-negative. Which tactic to prevent the progression of this condition in the child would be the most appropriate?

Переливання еритроцитарної маси Red blood cell transfusion

Перехід на штучне вигодовування залізовмісними сумішами Transition to artificial feeding with iron-containing mixtures

Призначити пероральний прийом препаратів заліза Prescribe oral iron preparations

Ввести вітамін K Enter vitamin K

Введення імуноглобуліну антирезус Rh0 (D) людини Introduction of human anti-rhesus Rh0 (D) immunoglobulin

862 / 1500
Жінка хворіє на бронхіальну астму із супутнім цукровим діабетом І типу, отримує в комплексній терапії: сальметерол, беклометазон, еуфілин, кромолін та преднізолон. Після призначення якої з наведених груп препаратів потрібна корекція дози інсуліну? A woman suffers from bronchial asthma with concomitant type 1 diabetes mellitus, receives in complex therapy: salmeterol, beclomethasone, euphylline, cromolyn and prednisolone. After the appointment of which of the following groups do you need insulin dose correction?

Системні глюкокортикоїди Systemic glucocorticoids

Пролонговані beta_2-агоністи Prolonged beta_2-agonists

Кромони Kromoni

Метилксантини Methylxanthines

Інгаляційні глюкокортикоїди Inhaled glucocorticoids

863 / 1500
Під час вивчення захворюваності на туберкульоз легень були отримані дані про соціально-економічні умови життя та шкідливі звички хворих. Який з наведених методів дозволяє оцінити міру впливу цих чинників на захворюваність туберкульозом? During the study of the incidence of pulmonary tuberculosis, data were obtained on the socio-economic living conditions and bad habits of the patients. Which of the following methods allows you to assess the degree of influence of these factors on the incidence tuberculosis?

Обчислення коефіцієнта регресії Calculation of regression coefficient

Обчислення показника відповідності Calculation of compliance indicator

Обчислення коефіцієнта кореляції Calculation of correlation coefficient

Обчислення коефіцієнта достовірності Calculation of reliability coefficient

Обчислення стандартизованих показників Calculation of standardized indicators

864 / 1500
Під час організації санаторно-курортного харчування осіб, які постраждали внаслідок Чорнобильської катастрофи, було запропоновано забезпечити в раціонах регламентовану кількість нутрієнтів, що вступають у конкурентні взаємовідносини з радіонуклідами та запобігають їхньому всмоктуванню у шлунково-кишковому тракті. Які нутрієнти мають пріоритетне значення для забезпечення такої біологічної дії їжі? During the organization of sanatorium-resort nutrition for persons affected by the Chernobyl disaster, it was proposed to provide in the diets a regulated amount of nutrients that enter into a competitive relationship with radionuclides and prevent to their absorption in the gastrointestinal tract. What nutrients are of priority for ensuring such a biological effect of food?

Калій, кальцій, харчові волокна Potassium, calcium, dietary fiber

Магній, фосфор, рослинні білки Magnesium, phosphorus, vegetable proteins

Рослинні жири, харчові волокна Vegetable fats, dietary fibers

Жиророзчинні вітаміни, тваринні білки Fat-soluble vitamins, animal proteins

Залізо, цинк, легкозасвоювані вуглеводи Iron, zinc, easily digestible carbohydrates

865 / 1500
Жінка віком 67 років, скаржиться на слабкість у руках, відчуття важкості в ногах, посіпування у м'язах кінцівок. Захворіла близько 10 місяців тому, коли вперше відмітила слабкість у руках. Об'єктивно спостерігається: виражена гіпотрофія м'язів верхніх кінцівок, дифузні м'язові фасцикуляції в кінцівках, переважно в руках. Знижена м'язова сила у верхніх кінцівках до 2 балів. У ногах м'язова сила збережена. Глибокі рефлекси на кінцівках значно посилені, клонуси стоп із обох боків. Патологічний рефлекс Бабінського з обох боків. Дисфункції тазових органів, чутливих і координаторних порушень не виявлено. Встановіть діагноз. A 67-year-old woman complains of weakness in her hands, a feeling of heaviness in her legs, twitching in the muscles of her limbs. She got sick about 10 months ago, when she first noticed weakness In the hands, there is marked hypotrophy of the muscles of the upper limbs, mainly in the hands. Muscle strength is preserved in the legs on the limbs, clonus on both sides. Pathological reflex of the pelvic organs, sensory and coordination disorders were not detected.

Міастенія Myasthenia

Аміотрофія Вердніга-Гофмана Werdnig-Hoffman amyotrophy

Сифілітичний менінгоенцефаліт Syphilitic meningoencephalitis

Бічний аміотрофічний склероз Amyotrophic lateral sclerosis

Туберкульозний менінгіт Tuberculous meningitis

866 / 1500
Робоче місце коректора друкарні обладнане письмовим столом та кріслом із регульованими параметрами сидіння і спинки. Робочий день у нього 8 годин, рівень завантаження робочого дня 96%. Розвиток якого професійного захворювання можливо у коректора? The workplace of a printing proofreader is equipped with a desk and a chair with adjustable seat and back parameters. His working day is 8 hours, the workload of a working day is 96%. The development of which professional Is it possible for the proofreader to have a disease?

Міопії Myopia

Варикозного розширення вен Varicose veins

Координаторного неврозу Coordinator neurosis

Ішемічної хвороби серця Ischemic heart disease

Остеохондрозу Osteochondrosis

867 / 1500
Під час огляду пацієнтки 64-річного віку лікар діагностував ожиріння (індекс маси тіла - 36 кг/м^2, об'єм талії - 118 см), артеріальну гіпертензію (170/105 мм рт. ст), порушення толерантності до вуглеводів. Під час обстеження виявлено підвищення рівню холестерину крові. Який діагноз хворої? During the examination of a 64-year-old female patient, the doctor diagnosed obesity (body mass index - 36 kg/m^2, waist circumference - 118 cm), arterial hypertension (170/105 mmHg), violation of carbohydrate tolerance. During the examination, an increase in blood cholesterol was detected. What is the patient's diagnosis?

Цукровий діабет Diabetes

Метаболічний синдром Metabolic syndrome

Гіпоталамічний синдром Hypothalamic syndrome

Артеріальна гіпертензія Hypertension

Аліментарне ожиріння Alimentary obesity

868 / 1500
Чоловік 24 років впав на ноги з балкону 4-го поверху. Відчув різкий біль у спині, через який не зміг встати самостійно. Під час огляду відзначається згладженість лордозу в поперековому відділі хребта. Осьове навантаження на хребет позитивне. Пальпація остистих відростків болюча, особливо 12-го грудного. Чутливість та рухи нижніх кінцівок не порушені. Яким буде діагноз? A 24-year-old man fell on his feet from the balcony of the 4th floor. He felt a sharp pain in his back, due to which he was unable to stand up on his own. During the examination, the smoothing of the lordosis in the lumbar spine. The axial load on the spine is painful, especially the 12th thoracic. What will be the diagnosis?

Забій поперекового відділу хребта Bruise of the lumbar spine

Перелом 12-грудного хребця з порушенням функції спинного мозку Fracture of the 12th thoracic vertebra with spinal cord dysfunction

Перелом поперекових відростків хребта Fracture of the lumbar processes of the spine

Компресійний перелом 12-грудного хребця без порушення функції спинного мозку Compression fracture of the 12th thoracic vertebra without spinal cord dysfunction

Перелом кісток тазу з порушенням функції органів тазу Fracture of pelvic bones with dysfunction of pelvic organs

869 / 1500
Чоловік 58 років скаржиться на болі в правій здухвинній ділянці, нечасті до 4-6 разів на добу, випорожнення, підвищення температури тіла до 37,5-38^oС. Об'єктивно встановлено: стан середньої важкості зниженого харчування, шкіра та слизові оболонки бліді. АТ - 100/60 мм рт. ст., PS - 98/хв. Пальпується болючий інфільтрат у правій здухвинній ділянці. Дослідження крові показало: еритроцити - 2,5·10^12/л, Нb - 80 г/л, лейкоцити - 11,2·10^9/л, еозинофіли - 3%, паличкоядерні - 6%, сегментоядерні - 68%, моноцити - 7%, лімфоцити - 14%, загальний білок - 61 г/л, альбуміни - 47%, глобуліни - 53%, С-реактивний білок - 24. Під час рентгенографічного дослідження з пасажем барію по тонкій кишці виявлений запальний стеноз термінального відділу тонкої кишки з депонуванням барію у дилятованій петлі тонкої кишки. Поставлено діагноз: хвороба Крона з ураженням термінального відділу тонкої кишки, стенозуюча форма середньої важкості. Яку групу препаратів слід призначити насамперед? A 58-year-old man complains of pain in the right iliac region, infrequent up to 4-6 times a day, defecation, an increase in body temperature to 37.5-38^oС Objectively, the condition of reduced nutrition, blood pressure - 100/60 mm Hg, palpable infiltrate in the right sinus showed: erythrocytes ... 14%, total protein - 61 g/l, albumins - 47%, globulins - 53%, C-reactive protein - 24. During an X-ray examination with the passage of barium through the small intestine, inflammatory stenosis of the terminal part of the small intestine with barium deposition in the dilated loops of the small intestine. A diagnosis was made: Crohn's disease with damage to the terminal part of the small intestine, a stenotic form of moderate severity. Which group of drugs should be prescribed first?

Ферменти Enzymes

Спазмолітики Spasmolytics

Антибіотики Antibiotics

Пробіотики Probiotics

Сульфаніламіди Sulfonamides

870 / 1500
Дівчина 18-ти років звернулася ввечері до лікаря зі скаргами на сильний біль ''знизу справа'', нудоту та одиничний епізод блювання. Зранку відвідувала фестиваль вуличної їжі. Біль розпочався 4 години тому, почався з епігастрію і поступово перейшов у праву здухвинну ділянку. При фізикальному обстеженні артеріальний тиск - 110/70 мм рт.ст., частота серцевих скорочень - 79/хв., частота дихання - 16/хв., температура - 37,4^oC. При пальпації лівого нижнього квадранту черевної стінки, біль виникає у правому. Гінекологічний та ректальний огляд без патологічних змін. Дані лабораторних досліджень: лейкоцити - 12·10^9/л, еритроцити - 4,4·10^12/л, гемоглобін - 130 г/л, гематокрит - 39%, тромбоцити - 225·10^9/л, beta-ХГЛ - 2 мОд/мл (норма - менше 5 мОд/л у невагітних жінок). Який діагноз є найбільш імовірним? An 18-year-old girl consulted a doctor in the evening with complaints of severe pain in the lower right, nausea and a single episode of vomiting. In the morning, she attended a street food festival. The pain started 4 hours ago, started from the epigastrium and gradually moved to the right thoracic region. During the physical examination, blood pressure - 110/70 mm Hg, heart rate - 79/min, respiratory rate - 16/min., temperature - 37.4°C. On palpation of the left lower quadrant, pain occurs in the right side. Laboratory data: leukocytes - 12·10^9/l, erythrocytes - 4.4·10^. 12/l, hemoglobin - 130 g/l, hematocrit - 39%, platelets - 225·10^9/l, beta-hCG - 2 mU/ml (norm - less than 5 mU/l in non-pregnant women). most likely?

Харчова токсикоінфекція Food poisoning

Кишкова непрохідність Intestinal obstruction

Гострий апендицит Acute appendicitis

Емболія мезентеріальних судин Mesenteric vessel embolism

Трубна вагітність Tapal pregnancy

871 / 1500
Пацієнт віком 28 років скаржиться на високу температуру 38-39^oС, загальну слабкість, головний біль. У середній третині лівої гомілки на місці незначного пошкодження шкіри відмічається набряклість та почервоніння шкіри з чіткими контурами. Яке захворювання найімовірніше спостерігається у пацієнта? A 28-year-old patient complains of a high temperature of 38-39^oC, general weakness, headache. In the middle third of the left lower leg, at the site of minor skin damage, there is swelling and redness of the skin with clear contours. What disease is most likely observed in the patient?

Бешиха Beshikha

Флегмона гомілки Phlegmon of the shin

Облітеруючий ендартеріїт Endarteritis obliterans

Лімфангоїт Lymphangitis

Гострий тромбофлебіт Acute thrombophlebitis

872 / 1500
Чоловік 43-х років скаржиться на поя-ву набряку обличчя, задухи та утрудненого ковтання через 3 години після укусу комахи. Об'єктивно: шкіра повік, щік та губ різко гіперемована й набрякла. Які засоби невідкладної допомоги слід призначити хворому? A 43-year-old man complains of swelling of the face, suffocation and difficulty swallowing 3 hours after an insect bite. Objectively: the skin of the eyelids, cheeks and the lips are hyperemic and swollen. What emergency aid should be prescribed to the patient?

Діуретики Diuretics

Глюкокортикостероїди Glucocorticosteroids

Нестероїдні протизапальні Nonsteroidal anti-inflammatory drugs

Антигістамінні Antihistamines

Міорелаксанти Myorelaxants

873 / 1500
Пацієнтка віком 20 років скаржиться, що впродовж трьох років відмічає похолодання пальців рук, вони набувають синюшно-білого кольору та стають нечутливими. За 5-10 хвилин виникає почервоніння шкіри і пальці теплішають, що супроводжується різким болем. Який найімовірніший діагноз? A 20-year-old patient complains that for three years she notices the coldness of her fingers, they acquire a bluish-white color and become insensitive. In 5-10 minutes, the skin becomes red and the fingers feel warm, accompanied by sharp pain. What is the most likely diagnosis?

Облітеруючий ендартериїт Endarteritis obliterans

Хвороба Бюргера Burger's disease

Поліневрит Polyneuritis

Хвороба Рейно Raynaud's disease

Облітеруючий атеросклероз Obliterating atherosclerosis

874 / 1500
Жінка звернулася до гінеколога з приводу планування вагітності і одержала рекомендацію збільшити споживання продуктів із високим вмістом фолієвої кислоти, зокрема сої, хліба з борошна грубого помелу, фруктів, зелених листяних овочів. Такі зміни в раціоні харчування вагітної сприятимуть профілактиці: A woman consulted a gynecologist about pregnancy planning and was advised to increase her intake of foods high in folic acid, including soy, wholemeal bread, fruits, green leafy vegetables vegetables. Such changes in the diet of a pregnant woman will contribute to prevention:

Багатоводдя Bagotovdya

Незарощення тім'ячка No overgrowth of crown

Рахіту Rickets

Залізодефіцитної анемії у вагітної Iron deficiency anemia in a pregnant woman

Незарощення невральної трубки у плода Neural tube failure in fetus

875 / 1500
У кар'єрі видобувається гранітний матеріал і подрібнюється за допомогою вибухів і ручних пневмомолотків. Під час буріння шпурів для вибухівки і подрібнення брил граніту робітники зазнають дії середньої і високочастотної локальної вібрації. Вимірювання рівнів вібрації показало, що вони перевищують гранично допустимі. Які специфічні зміни можуть виникати у робітників унаслідок тривалої роботи в таких умовах? Granite material is mined in the quarry and crushed using explosions and manual pneumatic hammers. During the drilling of holes for explosives and crushing of granite blocks, workers are exposed to medium and high-frequency local vibration. Measurements of vibration levels have shown that they exceed the maximum permissible. What specific changes can occur in workers as a result of long-term work in such conditions?

Психастенія Psychasthenia

Вегетосудинна дистонія Vegetovascular dystonia

Нейросенсорна приглухуватість Neurosen weed deafness

Дерматити кистей Dermatize hands

Сенсорна полінейропатія з ангіоспастичним синдромом Sensory polyneuropathy with angiospastic syndrome

876 / 1500
Вагітній 20-ти років 2 дні тому в терміні 18 тижнів вагітності виконано кримінальний аборт на дому. Об'єктивно: температура тіла - 39,4^oC, лихоманка, загальний стан тяжкий, пульс - 120/хв., артеріальний тиск - 80/60 мм рт.ст. Дно матки на 4 см вище лона, матка болюча. Сечі за останні 6 годин - 100 мл. Який наступний крок є найбільш доречним? A 20-year-old pregnant woman underwent a criminal abortion at home 2 days ago at 18 weeks of pregnancy. Objectively: body temperature - 39.4^oC, fever , the general condition is severe, the blood pressure is 80/60 mm Hg, the uterus is painful, the urine in the last 6 hours is the most appropriate?

Антибіотики широкого спектру дії внутрішньовенно, дилатація і кюретаж матки Broad-spectrum antibiotics intravenously, dilation and curettage of the uterus

Бактеріологічний посів крові, інфузійна терапія, гістеректомія з придатками Bacterial culture of blood, infusion therapy, hysterectomy with appendages

Бактеріологічний посів крові, форсований діурез, гістеректомія з придатками Bacterial culture of blood, forced diuresis, hysterectomy with appendages

Антибіотики широкого спектру дії внутрішньовенно, гістеректомія з придатками Broad-spectrum antibiotics intravenously, hysterectomy with appendages

Антибіотики широкого спектру дії внутрішньовенно, інфузійна терапія, дилатація та кюретаж матки Broad-spectrum antibiotics intravenously, infusion therapy, uterine dilation and curettage

877 / 1500
Чоловік віком 38 років звернувся до лікаря зі скаргами на появи болю та дискомфорту за грудниною, що тривають протягом останніх 4-х годин. Симптоми виникли після вечері, біль то посилюється, то вщухає, іррадіацію в ліву лопатку та руку пацієнт не відзначає. З анамнезу відомо, що батько пацієнта помер у 55 років через гострий інфаркт міокарда. Об'єктивно спостерігається: температура тіла 37,3^oС, АТ - 138/85 мм рт. ст., пульс - 115/хв, ЧД - 16/хв. Під час аускультації визначається везикулярне дихання. На ЕКГ виявлено синусову тахікардію. Попередньо пацієнт прийняв 3 таблетки нітрогліцерину з інтервалом 5 хвилин та аспірин, що покращило його стан. Яка подальша тактика у веденні цього пацієнта? A 38-year-old man turned to the doctor complaining of pain and discomfort behind the sternum, which has lasted for the past 4 hours. The symptoms appeared after dinner, the pain increases, then subsides, the patient does not note irradiation in the left shoulder blade. It is known from the anamnesis that the patient's father died at the age of 55. Objectively observed: body temperature 37.3°C, BP ​​- 138/85 mm Heart rate - 115/min. During auscultation, sinus tachycardia was detected. Previously, the patient took 3 tablets of nitroglycerin, which improved his condition tactics in the management of this patient?

Дати пацієнту знеболюючий засіб Give the patient painkiller

Визначення серцевих біомаркерів (КФК-МВ, тропонін І, N-кінцевий пептид натрійуретичного гормону) Determination of cardiac biomarkers (CFC-MV, troponin I, N-terminal peptide of natriuretic hormone)

КТ грудної клітки та ФГДС CT chest and FGDS

Рентгенографія з барієвою сумішшю X-ray with barium mixture

Провести пробу з фізичним навантаженням Conduct an exercise test

878 / 1500
Дівчина 23-х років без очевидних причин стала конфліктною на роботі, вважала, що її недооцінює керівництво, стверджувала, що вона спроможна займати посаду заступника директора, оскільки володіє чотирма мовами, є дуже привабливою й може покращити контакти компанії. Екстравагантно вдягалася, фліртувала з колегами, голосно співала у кабінеті. Має освіту оператора ЕОМ, іноземними мовами не володіє. Який імовірний клінічний діагноз? A 23-year-old girl became conflicted at work for no apparent reason, believed that she was underestimated by management, claimed that she was capable of holding the position of deputy director, as she has four languages, is very attractive and can improve company contacts. Dressed extravagantly, sang loudly in the office. She has an education as a computer operator, does not speak foreign languages.

Епілепсія Epilepsy

Депресивний розлад Depressive disorder

Шизофренія Schizophrenia

Маніакальний епізод Manic episode

Легка розумова недорозвиненість Slight mental retardation

879 / 1500
Жінка 56 років із надмірною масою тіла скаржиться на біль у колінних суглобах, який посилюється під час ходіння, особливо по східцях, під час довгого стояння на ногах. Хворіє 5 років. Об'єктивно встановлено: колінні суглоби деформовані, набряклі, під час руху - болючі. Рентгенограма суглобів показала: суглобова щілина звужена, субхондральний склероз, краєві остеофіти. Який діагноз найбільш імовірний? A 56-year-old overweight woman complains of pain in the knee joints, which worsens when walking, especially on stairs, when standing on her feet for a long time. Patient 5 years. Objectively established: knee joints are swollen, painful during movement. Joint gap is narrowed, subchondral sclerosis. What is the most likely diagnosis?

Остеоартроз Osteoarthrosis

Подагра Gout

Реактивний артрит Reactive arthritis

Ревматоїдний артрит Rheumatoid arthritis

Ревматичний артрит Rheumatic arthritis

880 / 1500
Пацієнтка віком 31 рік звернулася до лікаря-гінеколога зі скаргами на затримку менструації на 2 тижні, ранкову нудоту, мажучі кров'янисті виділення з піхви. Тест на вагітність виявився позитивним. Під час УЗД виявлено: плідного яйця в порожнині матки немає. Які обстеження потрібно призначити пацієнтці? A 31-year-old patient turned to a gynecologist with complaints of a delay in menstruation for 2 weeks, morning sickness, smearing bloody discharge from the vagina. The pregnancy test showed positive. During the ultrasound, it was found that there is no fertile egg in the uterine cavity. What examinations should be prescribed to the patient?

Визначення рівня прогестерону в крові Determining the level of progesterone in the blood

Визначення ХГ в крові в динаміці Determination of hCG in blood in dynamics

ХГ крові Hg blood

Визначення РАРР+ХГ в крові Determination of PAPP+ChG in blood

МРТ MRI

881 / 1500
Мама скаржиться на висипи та інтенсивний свербіж шкіри у дитини 11 років, яка захворіла на 1-му році життя. Шкірні покриви сухі. Переважно на згинах колінних і локтьових суглобів, передньої поверхні шиї, обличчі видні вогнища ліхенізації з великою кількістю ексоріацій, плоскі вузлові висипання, кольору нормальної шкіри, лущення, гіперпігментація. Поставте імовірний діагноз: The mother complains of rashes and intense itching of the skin in an 11-year-old child who got sick in the 1st year of life. The skin is dry. Mainly on the bends of the knee and elbow joints , on the front surface of the neck, on the face there are visible foci of lichenization with a large number of excoriations, flat nodular rashes, normal skin color, peeling, hyperpigmentation. Make a probable diagnosis:

Хронічна істинна екзема Chronic true eczema

Псоріаз Psoriasis

Червоний плоский лишай Red lichen planus

Атопічний дерматит Atopic dermatitis

Кропив'янка Hives

882 / 1500
Хлопчик 12-ти років надійшов до клініки зі скаргами на відчуття серцебиття, перебої та біль у серці, задишку. Ці симптоми найчастіше виникали після фізичних навантажень і закінчувались синкопальними станами. Згодом лікарем було встановлено діагноз гіпертрофічної обструктивної кардіоміопатії. Який препарат textbfПРОТИПОКАЗАНИЙ при лікуванні цієї патології? A 12-year-old boy came to the clinic with complaints of palpitations, interruptions and pain in the heart, shortness of breath. These symptoms most often occurred after physical exertion and ended with syncopal states Subsequently, the doctor diagnosed hypertrophic obstructive cardiomyopathy. Which drug is CONTRAINDICATED in the treatment of this pathology?

Дигоксин Digoxin

Верапаміл Verapamil

Еналаприл Enalapril

Бісопролол Bisoprolol

- -

883 / 1500
На зупинці автобуса чоловік похилого віку схопився за серце, а потім втратив свідомість. Декілька разів глибоко вдихнув з інтервалом у 8-10 секунд, а потім перестав дихати. Шкіра бліда, холодна. Пульс на сонних артеріях не визначається. Зіниці розширені, реагують на світло. Діагностована раптова коронарна смерть. З чого варто почати серцево-легеневу реанімацію? At the bus stop, an elderly man clutched his heart and then lost consciousness. He took several deep breaths at intervals of 8-10 seconds and then stopped breathing. Skin pale, cold. The pulse on the carotid arteries is not determined. The pupils react to light. What should I start cardiopulmonary resuscitation with?

Штучна вентиляція легень Artificial lung ventilation

Массаж каротидного синуса Carotid sinus massage

Непрямий масаж серця Indirect heart massage

Прийом Вальсальви Valsalva Reception

Трахеостомія Tracheostomy

884 / 1500
Хвора 42-х років після відпочинку, встала з ліжка і відчула загальну слабкість, запаморочення, потемніння в очах, втратила свідомість. Об'єктивно: хвора непритомна, шкіра бліда, холодна, зіничні та сухожильні рефлекси збережені. АТ- 75/50 мм рт.ст., пульс - 100/хв. Який діагноз найбільш імовірний? A 42-year-old patient after rest got out of bed and felt general weakness, dizziness, darkening of the eyes, lost consciousness. Objectively: the patient is unconscious, the skin pale, cold, pupillary and tendon reflexes preserved. Blood pressure - 75/50 mmHg, pulse - 100/min. What is the most likely diagnosis?

Ішемічний інсульт Ischemic stroke

Ортостатичний колапс Orthostatic collapse

Гіпоглікемічна кома Hypoglycemic coma

Епілептичний синдром Epileptic syndrome

Істеричний напад Hysterical attack

885 / 1500
Хворий 43-х років страждає на виразкову хворобу 12-палої кишки протягом 8-ми років. Під час роботи на садовій дільниці відчув різкий біль у епігастральній дільниці, слабкість, запаморочення. Об'єктивно: шкіра бліда, пульс - 102/хв., слабкий, артеріальний тиск - 100/60 мм рт.ст., живіт не приймає участі у акті дихання, напружений, болісний у епігастрії, печінкова тупість відсутня. Яке захворювання є найбільш імовірним? A 43-year-old patient has been suffering from duodenal ulcer disease for 8 years. While working in the garden area, he felt sharp pain in the epigastric area, weakness , dizziness. Objectively: the skin is pale, pulse - 102/min., blood pressure - 100/60 mmHg, abdomen does not take part in the act of breathing, tense, painful in the epigastrium. No liver dullness the disease is most likely?

Перфорація виразки Ulcer perforation

Загострення панкреатиту Exacerbation of pancreatitis

Шлункова кровотеча Gastric bleeding

Інфаркт міокарда Myocardial infarction

Кишкова непрохідність Intestinal obstruction

886 / 1500
На прохідників під час виконання робіт у гірничих виробітках діє мінеральний пил у вигляді аерозолів дезінтеграції. Для визначення рівня небезпеки виникнення професійних пилових захворювань вивчали хімічні та фізичні властивості пилу. Яка із властивостей пилу визначає загалом глибину його проникнення в дихальні шляхи? Pedestrians are exposed to mineral dust in the form of disintegration aerosols during mining operations. To determine the level of danger of occupational dust diseases, the chemical and physical properties of dust were studied. What from the properties of dust determines in general the depth of its penetration into the respiratory tract?

Електрозарядженість Electrical charge

Вміст діоксиду кремнію Content of silicon dioxide

Форма пилових часток Shape of dust particles

Дисперсність Variance

Розчинність Solubility

887 / 1500
Підлітка 16 років покусав собака. Укуси розташовані в ділянці кінців пальців та обличчя. Собака бездомний, встановити за ним спостереження неможливо. Проведена первинна хірургічна обробка рани без видалення країв рани. Якою буде тактика лікаря щодо профілактики сказу? A 16-year-old teenager was bitten by a dog. The bites are located in the area of ​​the ends of the fingers and the face. The dog is homeless, it is impossible to monitor it. Primary surgical treatment of the wound was carried out without removing the edges of the wound . What will the doctor's tactics be for the prevention of rabies?

Призначити імуноглобулін людини та рибавірин Prescribe human immunoglobulin and ribavirin

Призначити антирабічний імуноглобулін та рибавірин Prescribe anti-rabies immunoglobulin and ribavirin

Призначити антирабічну вакцину та імуноглобулін людини Prescribe anti-rabies vaccine and human immunoglobulin

Призначити антирабічну вакцину та рибавірин Prescribe rabies vaccine and ribavirin

Призначити антирабічний імуноглобулін та антирабічну вакцину Prescribe anti-rabies immunoglobulin and anti-rabies vaccine

888 / 1500
У хлопчика, який хворіє на гемофілію А, за 7 годин після травми з'явився біль у колінному суглобі. Об'єктивно спостерігається: суглоб збільшився у розмірі, шкіра над ним гіперемована, гаряча на дотик, рухи у коліні різко обмежені. Що потрібно призначити хлопчику насамперед? A boy suffering from hemophilia A developed pain in the knee joint 7 hours after the injury. Objectively observed: the joint increased in size, the skin over him, hyperemic, hot to the touch, movements in the knee are sharply limited. What should be prescribed to the boy first?

Уведення Е-АКК Entering E-ACC

Уведення вікасолу Vikasol administration

Пункцію суглоба Joint puncture

Уведення кріопреципітату Introduction of cryoprecipitate

Уведення діцинону Introduction of dizinon

889 / 1500
Чоловік 65 років, який понад 17 років страждає на хронічний обструктивний бронхіт, скаржиться на задуху з переважним утрудненням вдиху, відчуття важкості у правому підребер'ї, набряки гомілок та стоп. Під час аускультації над легенями спостерігається жорстке дихання, сухі хрипи; акцент та розщеплення ІІ тону в ІІ міжребер'ї. Яких змін слід очікувати на ЕКГ? A 65-year-old man, who has been suffering from chronic obstructive bronchitis for more than 17 years, complains of shortness of breath with predominant difficulty breathing, a feeling of heaviness in the right hypochondrium, swelling of the lower legs and stop. During auscultation over the lungs, there is hard breathing, accentuation and splitting of the II intercostal sound. What changes should be expected on the ECG?

Гіпертрофія лівого передсердя Left atrial hypertrophy

Екстрасистолія Extrasystole

Відхилення ЕВС вліво EVS deviation to the left

Миготлива аритмія Atrial fibrillation

Гіпертрофія міокарда правого шлуночка Myocardial hypertrophy of the right ventricle

890 / 1500
До оториноларинголога звернувся хворий 29-ти років зі скаргами на зниження слуху. При аудіологічному обстеженні вия-влено порушення звукопроведення, кісткова провідність не порушена. Про порушення роботи яких структур органу слуху йде мова? A 29-year-old patient came to the otorhinolaryngologist with complaints of hearing loss. During the audiological examination, a sound conduction disturbance was detected, bone conduction was not disturbed. What structures are disturbed the organ of hearing is being talked about?

Барабанної перетинки, ланцюга слухових кісточок Tympanic membrane, chain of auditory ossicles

Системи півколових каналів Semicircular canal systems

Спірального органу Spiral Organ

Отолітового апарату Otolith apparatus

Основи завитки та довгастого мозку Fundamentals of gyrus and medulla oblongata

891 / 1500
Пацієнт віком 46 років скаржиться на виникнення двоїння перед очима та опущення повік переважно в другій половині дня, які практично повністю зникають після відпочинку. Під час обстеження виявлено: легкий птоз з обох боків, обмеження рухів очних яблук вбік, диплопія, позитивна прозеринова проба. Встановіть діагноз. A 46-year-old patient complains of double vision and drooping of the eyelids mainly in the afternoon, which almost completely disappear after rest. During the examination, it was found: mild ptosis on both sides, restriction of eyeball movements to the side, diplopia, positive proserin test. Establish a diagnosis.

Синдром Кернса-Сейра Kearns-Sayre Syndrome

Над'ядерна офтальмоплегія, що прогресує Progressive supranuclear ophthalmoplegia

Міастенія Myasthenia

Окулофарингеальна м'язова дистрофія Oculopharyngeal muscular dystrophy

Оливопонтоцеребелярна дегенерація Olivopontocerebellar degeneration

892 / 1500
Хворий 23-х років скаржиться на інтен-сивний біль у лівому колінному суглобі. Об'єктивно: колінний суглоб збільшений у об'ємі, шкіра над ним гіперемована, при пальпації болючий. Загальний аналіз крові: еритроцити - 3,8·10^12/л, Hb- 122 г/л, лейкоцити - 7,4·10^9/л, тромбоцити - 183·10^12/л. Швидкість осідання еритроцитів - 10 мм/год. Час кровотечі за Дюке - 4 хв., час згортання крові за Лі-Уайтом - 24 хв. АПТЧ - 89 с. Ревматоїдний фактор - негативний. Який найбільш імовірний діаг-ноз? A 23-year-old patient complains of intense pain in the left knee joint. Objectively: the knee joint is enlarged, the skin over it is hyperemic, painful on palpation. General blood analysis: erythrocytes - 3.8·10^12/l, Hb - 7.4·10^9/l, platelets - 183·10^12/l erythrocyte sedimentation rate - 10 mm/h. Bleeding time according to Lee-White - 24 min. Rheumatoid factor - negative?

Ревматоїдний артрит Rheumatoid arthritis

Хвороба Верльгофа Werlhof's disease

Геморагічний васкуліт, суглобова форма Hemorrhagic vasculitis, articular form

Гемофілія, гемартроз Hemophilia, hemarthrosis

Тромбоцитопатія Thrombocytopathy

893 / 1500
Пацієнтка віком 35 років скаржиться на часте, болюче, утруднене сечовипускання, постійні позиви до нього, появу кількох крапель крові у сечі в кінці процесу. Захворіла раптово після переохолодження, коли з'явились вищеописані скарги. Температура тіла - 36,6^oС. Під час пальпації спостерігається: болючість в ділянці сечового міхура. Під час УЗД виявлено: в сечовому міхурі мало сечі, стінки сечового міхура набряклі, рівномірно потовщені. В аналізах сечі спостерігається: лейкоцитурія (30-40 в полі зору), протеїнурія (0,099 г/л), еритроцитурія (5-7 незмінених еритроцитів). Який найімовірніший діагноз? A 35-year-old patient complains of frequent, painful, difficult urination, constant urges to urinate, the appearance of several drops of blood in the urine at the end of the process. She fell ill suddenly after hypothermia, when the above complaints appeared. Body temperature - 36.6°C. During palpation, it was found that there was little urine in the bladder, and uniform thickening of the bladder : leukocyturia (30-40 in the field of vision), proteinuria (0.099 g/l), erythrocytosis (5-7 unchanged erythrocytes). What is the most likely diagnosis?

Уретероцеле Ureterocele

Гострий цистит Acute cystitis

Гострий сальпінгоофорит Acute salpingo-oophoritis

Пухлина сечового міхура Bladder tumor

Конкремент сечового міхура Bladder calculus

894 / 1500
Пацієнтці 37 років, після збору анамнезу та огляду виставлено діагноз: дифузна форма мастопатії обох молочних залоз. Який інструментальний метод дослідження найбільш інформативний у цьому разі? The patient is 37 years old, after taking an anamnesis and examination, the diagnosis was made: diffuse form of mastopathy of both mammary glands. Which instrumental research method is the most informative in this case?

Мамографія Mammography

УЗД молочних залоз Ultrasound of mammary glands

Дуктографія Ductography

Біопсія молочної залози Breast biopsy

Пневмокістографія Pneumocystography

895 / 1500
Пацієнт віком 19 років скаржиться на сухий кашель, біль в м'язах, підвищення температури тіла до 39^oC. Протягом тижня спостерігалися біль в горлі, субфебрилітет. Об'єктивно виявлено жорстке дихання. Під час загального аналізу крові виявлено: лейкоцити - 10,0·10^9/л, лейкоцитарний зсув вліво, ШОЕ - 26 мм/год. В ході рентгенографії ОГП спостерігається підсилення легеневого малюнку, малоінтенсивні вогнищеві тіні в нижніх відділах правої легені. Які лікарські засоби найдоцільніше призначити? A 19-year-old patient complains of a dry cough, muscle pain, an increase in body temperature to 39^oC. During the week, a sore throat, low-grade fever were observed. About during the general blood analysis it was found: leukocytes - 10.0·10^9/l, leukocyte shift to the left, ESR - 26 mm/h. In the course of X-ray examination, there was an increase in the pulmonary pattern, low-intensity focal shadows in the lower departments of the right lung. Which drugs are most appropriate to prescribe?

Сульфаніламіди Sulfonamides

Антибіотики пеніцилінового ряду Antibiotics of the penicillin series

Аміноглікозиди Aminoglycosides

Цефалоспорини II або III покоління II or III generation cephalosporins

Макроліди Macrolides

896 / 1500
Хворий 25-ти років до психіатричної лікарні надходить вперше. Збуджений, говорить швидко й голосно, активно жестикулює. Без упину жартує, сміється, з обличчя не сходить посмішка. З лікарем розмовляє на ''ти'', пропонує йому зіграти в карти, залицяється до медсестер, запрошує їх на побачення. Розповідає, що останні 2 доби не спав, проте зовсім не відчуває втоми. Який препарат доцільно використати? A 25-year-old patient is admitted to a psychiatric hospital for the first time. He is excited, speaks quickly and loudly, gestures actively. He jokes and laughs non-stop, a smile never leaves his face. He talks to the doctor, offers him to play cards, flirts with the nurses, tells them that he hasn't slept in the last 2 days, but he doesn't feel tired at all. What drug is appropriate to use?

Ноотропний Nootropic

Антипсихотичний Antipsychotic

Антидепресант Antidepressant

Седативний Sedative

Снодійний Sleepy

897 / 1500
У хворого 25-ти років на 10-й день захворювання, яке супроводжувалося підвищенням температури тіла, везикульозним висипом, переважно на шкірі тулуба та волосистої частини голови, з'явилися сильний головний біль, блювання, атаксія, загальмованість, дискоординація рухів, тремор кінцівок. Діагностовано енце-фаліт. Ускладненням якої хвороби є цей стан? A 25-year-old patient on the 10th day of the disease, which was accompanied by an increase in body temperature, a vesicular rash, mainly on the skin of the trunk and scalp, with' a severe headache, vomiting, ataxia, dyscoordination of movements appeared. Encephalitis was diagnosed. What is the complication of this condition?

Краснуха Krasnukha

Кір Measles

Скарлатина Scarlatina

Везикульозний рикетсіоз Vesicular rickettsiosis

Вітряна віспа Chicken Pox

898 / 1500
Пацієнтка 55-ти років звернулася до лікаря жіночої консультації зі скаргами на біль внизу живота, появу кров'яних виділень через 3 роки після припинення менструації. Які додаткові методи обстеження необхідно провести у даної пацієн-тки для уточнення діагнозу? A 55-year-old patient turned to the doctor of the women's consultation with complaints of pain in the lower abdomen, the appearance of bloody discharge 3 years after the cessation of menstruation. What are the additional methods of examination is it necessary to conduct this patient to clarify the diagnosis?

Кольпоскопія Colposcopy

Лапароскопія Laparoscopy

Зішкріб на онкоцитологію Scraping for oncocytology

Роздільне діагностичне вишкрібання матки Separate diagnostic curettage of the uterus

УЗД органів малого таза Pelvic ultrasound

899 / 1500
Жінка 23 років скаржиться на часте болюче сечовипускання, тупі болі внизу живота. Наведені скарги з'явилися 10 днів тому після переохолодження. Біль і часті позиви до сечовипускання зменшуються від застосування тепла на ділянку сечового міхура. Температура тіла - 37,2^oC. Результати загального аналізу сечі: лейкоцити, свіжі еритроцити, незначна протеїнурія. Симптом Пастернацького негативний з обох боків. Які основні препарати в лікуванні цієї пацієнтки? A 23-year-old woman complains of frequent painful urination, dull pains in the lower abdomen. These complaints appeared 10 days ago after hypothermia. Pain and frequent urges to urinate decrease from applying heat to the bladder area. The results of the general analysis of urine: white blood cells, slight proteinuria. What are the main drugs in the treatment of this patient?

Гепарин Heparin

Курантил Curantyl

Уросептики Uroseptics

Глюкокортикоїди Glucocorticoids

Імунодепресанти Immunosuppressants

900 / 1500
У курсанта морського коледжу в періа-нальній ділянці на 2-х годинах умовного циферблату виявлена неболюча щільна виразка розмірами 1,5х0,5 см, дно якої має вигляд ''старого сала''. Який імовірний діагноз? A non-painful dense ulcer measuring 1.5x0.5 cm was found in the perianal area of ​​the naval college cadet at 2 o'clock on the conventional dial, the bottom of which looks like ' 'old lard'. What is the probable diagnosis?

Тріщина прямої кишки Fissure of the rectum

Рак анального відділу прямої кишки Anal rectal cancer

Нориця прямої кишки Fistula of the rectum

Нагноєння крипти прямої кишки Suppuration of the crypt of the rectum

Твердий шанкр (сифіліс) прямої кишки Hard chancre (syphilis) of the rectum

901 / 1500
Пацієнт віком 65 років скаржиться на задишку, кашель з виділенням рожевої мокроти, що піниться, відчуття нестачі повітря, страх смерті. О'бєктивно спостерігається: ортопное, шкіра бліда, акроціаноз, холодний липкий піт. Під час аускультації вислуховується: дихання жорстке, в нижньо-задніх відділах з обох сторін - вологі дрібно- та середньопухирчаті хрипи. ЧД - 40/хв. Тони серця різко приглушені. На верхівці серця - ритм галопу. Який найімовірніший діагноз? A 65-year-old patient complains of shortness of breath, cough with frothy pink sputum, feeling of lack of air, fear of death. Objectively observed: orthopnea, pale skin , acrocyanosis. During auscultation, breathing is hard, in the lower and back parts - rales - 40/min. Heart sounds are muffled the most likely diagnosis?

Тромбоемболія легеневої артерії Thromboembolism of the pulmonary artery

Набряк легенів Pulmonary edema

Вогнищева пневмонія Focal pneumonia

Інфаркт-пневмонія Pneumonia heart attack

Бронхіальна астма Bronchial asthma

902 / 1500
Потерпілий впав з драбини висотою 2 м. Лікар швидкої медичної допомоги припускає компресійний перелом хребців на рівні L1-L2. Об'єктивно спостерігається: АТ - 100/60 мм рт. ст., пульс - 104/хв. Яку допомогу на місці події потрібно надати потерпілому? The victim fell from a ladder with a height of 2 m. The emergency medical doctor assumes a compression fracture of the vertebrae at the L1-L2 level. Objectively observed: BP - 100/60 mm rt., pulse - 104/min. What assistance should be given to the victim at the scene?

Анестезія хребців за Шнеком, транспортування в стаціонар Anesthesia of vertebrae according to Schneck, transportation to hospital

Транспортування в стаціонар в положенні на боці Transportation to the hospital in the lateral position

Анальгетики внутрішньом'язово, транспортування в положенні сидячи Analgesics intramuscularly, transportation in a sitting position

Знеболення, протишокова терапія, транспортування в стаціонар Anesthesia, anti-shock therapy, transportation to a hospital

Знеболення, транспортування в лікарню на жорстких ношах Anesthesia, transportation to the hospital on rigid stretchers

903 / 1500
У хворої 26-ти років у зв'язку з системними ураженнями шкіри, судин, суглобів, серозних оболонок і серця, що виникли після фотосенсибілізації, припущено системний червоний вовчак. Вияв-лені LE-клітини, антитіла до нативної ДНК, поодинокі антицентромерні антитіла, РФ 1:100, RW позитивна, ЦІК- 120 од. Які імунологічні показники вважаються специфічними для цього захворювання? A 26-year-old patient has suspected systemic lupus erythematosus due to systemic lesions of the skin, blood vessels, joints, serous membranes, and heart that occurred after photosensitization Detected LE-cells, antibodies to native DNA, single anticentromeric antibodies, RF 1:100, CK-120 units. What immunological indicators are considered specific for this disease?

Підвищення ЦІК Increase in CIC

Антитіла до ДНК Antibodies to DNA

Імуноглобулін А Immunoglobulin A

Ревматоїдний фактор Rheumatoid factor

Антицентромерні антитіла Anticentromeric antibodies

904 / 1500
У сім'ї вживались в їжу овочеві (зокрема грибні) та фруктові консерви домашнього приготування. За 8 днів після чергового прийманні їжі у двох членів сім'ї з'явилися скарги на слабкість, послаблення зору, двоїння в очах та косоокість. Трохи згодом виникло утруднення мови, порушення ковтання. Після звернення до лікарні пацієнтів госпіталізували. Яке харчове отруєння виникло у цьому разі? The family used to eat home-made canned vegetables (especially mushrooms) and fruit. 8 days after the next intake of food, two family members from complaints of weakness, diplopia, and strabismus appeared. A little later, difficulty in swallowing appeared. After going to the hospital, the patient was hospitalized. What kind of food poisoning occurred in this case?

Афлотоксикоз Aflotoxicosis

Бактеріальна харчова токсикоінфекція Bacterial food poisoning

Сальмонельоз Salmonellosis

Ботулізм Botulism

Стафілококовий токсикоз Staphylococcal toxicosis

905 / 1500
Дитині 3 місяців через відсутність у матері грудного молока з 2,5 місяців у харчування введена суміш. Мати зазначає, що, попри на ретельний догляд, у дитини з'явилося стійке почервоніння у складках шкіри. За 3-4 дні на щоках і підборідді з'явилися сверблячі ділянки гіперемованої шкіри, наповнені серозним ексудатом, який підсихає, утворюючи кірочки та струпи та формуючи жовті кірки. На волосистій частині голови себорейна луска. Який діагноз найімовірніший? Due to the mother's lack of breast milk since 2.5 months, a 3-month-old child has been fed formula. The mother notes that, despite careful care, the child with there was persistent redness in the skin folds. In 3-4 days, itchy areas of hyperemic skin appeared, which dried up, forming crusts and yellow crusts. What is the most likely diagnosis ?

Стафілодермія Staphyloderma

Лімфатико-гіпопластичний діатез Lymphatic-hypoplastic diathesis

Ексудативно-катаральний діатез Exudative-catarrhal diathesis

Алергічний (атопічний) діатез Allergic (atopic) diathesis

Нервово-артритичний діатез Nervous-arthritic diathesis

906 / 1500
Пацієнта госпіталізовано до лікарні з діагнозом «трихінельоз». Вживання якого продукту стало причиною розвитку цього гельмінтозу? The patient was admitted to the hospital with a diagnosis of trichinellosis. What product was the cause of the development of this helminthiasis?

Яловичини Beef

М'яса птиці Poultry

Кролячого м'яса Rabbit

Баранини Lambs

Свинини Pork

907 / 1500
До гінекологічного стаціонару шпиталізовано жінку зі скаргами на різкі болі внизу живота, запаморочення. З анамнезу відомо: остання менструація була 6 тижнів тому. Об'єктивно спостерігається: шкірні покриви бліді, АТ - 80/60 мм рт. ст., пульс - 94/хв. Симптом подразнення очеревини в нижніх відділах позитивний. Бімануально встановлено: матка дещо збільшена, придатки збільшені справа, болючі. Позитивний симптом Промтова. Встановіть ймовірний діагноз. A woman was hospitalized in a gynecological hospital with complaints of sharp pains in the lower abdomen, dizziness. It is known from the anamnesis: the last menstruation was 6 weeks ago. Objectively observed: skin pale, blood pressure - 94/min. Peritoneal irritation is positive. The uterus is slightly enlarged, painful. Probable diagnosis.

Апоплексія яєчника Ovarian apoplexy

Розрив кісти правого яєчника Rupture of right ovarian cyst

Порушена позаматкова вагітність Disrupted ectopic pregnancy

Перекрут ніжки кісти правого яєчника Torsion of the pedicle of the cyst of the right ovary

Гострий аднексит Acute adnexitis

908 / 1500
Пацієнт лікувався з приводу інфаркту міокарда. На 13 день відбулося посилення болю в грудній клітці, з'явилася задишка. Об'єктивно спостерігається: температура - 38,2^oС, пульс - 112/хв., ЧД - 26/хв., під правою лопаткою вислуховуються дрібнопухирчасті хрипи. На 15 день був діагностований правобічний ексудативний плеврит. У загальному аналізі крові спостерігається: лейкоцити - 8,9·10^9/л, еозинофіли - 8 %. ШОЕ - 24 мм/год. Яке ускладнення інфаркту міокарда виникло у пацієнта? The patient was treated for a myocardial infarction. On the 13th day, chest pain increased, shortness of breath appeared. Objectively observed: temperature - 38.2^ oC, pulse - 26/min, fine vesicular rales are heard under the right shoulder blade. Right-sided exudative pleurisy was diagnosed on the 15th day: leukocytes - 8.9·10^9/l, eosinophils - 8%. ESR - 24 mm/h. What complication of myocardial infarction did the patient have?

Повторний інфаркт міокарда Recurrent myocardial infarction

Синдром Дресслера Dressler syndrome

Пневмонія Pneumonia

Серцева астма Cardiac Asthma

Тромбоемболія легеневої артерії Thromboembolism of the pulmonary artery

909 / 1500
За годину після годування немовляти молочною сумішшю у дитини виникли такі симптоми захворювання: ціаноз губ, слизових оболонок, нігтів, обличчя, потім приєдналася нудота, посилене слиновиділення, біль у надчеревній ділянці, блювання, пронос. Лікар-педіатр виявив у дитини ознаки легенево-серцевої недостатності. В процесі розслідування встановлено, що молочна суміш приготовлена на воді, взятій із колодязя. Який найімовірніший діагноз? An hour after feeding the baby with milk formula, the child developed the following symptoms of the disease: cyanosis of the lips, mucous membranes, nails, face, then nausea, increased salivation, pain in epigastric area, vomiting, diarrhea. The pediatrician found signs of pulmonary heart failure. During the investigation, it was established that the milk mixture was prepared on water taken from the well?

Харчове отруєння хлорорганічними пестицидами Food poisoning by organochlorine pesticides

Харчове отруєння важкими металами Food poisoning by heavy metals

Нітратно-нітритна інтоксикація Nitrate-nitrite intoxication

Харчова токсикоінфекція Food poisoning

Стафілококовий токсикоз Staphylococcal toxicosis

910 / 1500
Пацієнтка віком 32 роки скаржиться на ниючий біль внизу живота, посилені болючі менструації. З анамнезу відомо: 2 аборти, пологів не було. Об'єктивно спостерігається: шкірні покриви бліді, живіт м'який, не болючий, над лоном пальпується пухлиноподібний утвір. Під час бімануального дослідження виявлено: тіло матки збільшене до 14 тижнів вагітності, з нерівною поверхнею, щільне та рухоме. Придатки не пальпуються. Який найімовірніший діагноз? A 32-year-old patient complains of aching pain in the lower abdomen, increased painful menstruation. It is known from the anamnesis: 2 abortions, there were no deliveries. Objectively observed: skin pale, the abdomen is not painful, a tumor-like formation is palpated during the bimanual examination: the body of the uterus is enlarged, with an uneven surface, and the appendages are not palpable. What is the most likely diagnosis?

Вагітність Pregnancy

Вузлувата лейоміома матки Nodular leiomyoma of the uterus

Ендометріоз Endometriosis

Пухлина нирки Kidney tumor

Кістома яєчника Ovarian cystoma

911 / 1500
Пацієнтка віком 82 років надійшла у відділення кардіологічної реанімації зі скаргами на гострий біль за грудиною, відчуття нестачі повітря, слабкість. Під час рентгенологічного дослідження органів грудної порожнини виявлено, що поперечний розмір тіні серця збільшений, форма тіні трикутна із закругленими кардіодіафрагмальними кутами. Скорочення серця малої амплітуди, аритмічні. Виявлені рентгенологічні ознаки, найімовірніше, відповідають: An 82-year-old patient was admitted to the cardiac intensive care unit with complaints of sharp pain behind the sternum, a feeling of lack of air, weakness. During an X-ray examination of the chest cavity, it was found that the transverse size of the shadow of the heart is increased, the shape of the shadow is triangular with rounded cardiophrenic angles. Heart contractions of small amplitude, arrhythmic. The detected radiological signs, most likely, correspond to:

Міокардиту Myocarditis

Ексудативному перикардиту Exudative pericarditis

Аортальному стенозу Aortic stenosis

Дилатаційній кардіоміопатії Dilated cardiomyopathy

Тріаді Фалло Triadi Fallo

912 / 1500
Пацієнта віком 43 років шпиталізовано до лікарні за 40 хвилин після виникнення гострого болю в епігастрії, що згодом перемістився у праву здухвинну ділянку. Об'єктивно спостерігається: різке напруження м’язів передньої черевної стінки, позитивний симптом Щоткіна-Блюмберга, Мюссі-Георгієвського, Чугаєва, Бернштейна. Печінкова тупість відсутня. Який найімовірніший діагноз? A 43-year-old patient was admitted to the hospital 40 minutes after the onset of acute pain in the epigastrium, which later moved to the right iliac region. Objectively observed: sharp tension in the muscles of the anterior abdominal wall, a positive symptom of Shtotkin-Blumberg, Chugaev, Bernstein. There is no hepatic dullness. What is the most likely diagnosis?

Гострий холецистит Acute cholecystitis

Ниркова колька Renal colic

Гострий панкреатит Acute pancreatitis

Гострий апендицит Acute appendicitis

Перфоративна виразка шлунка Perforative gastric ulcer

913 / 1500
У військового 34 років, страх, тривога, роздратованість, пригнічений настрій, жахливі сновидіння про минулі події з'явилися за 6 місяців після перенесеного бойового стресу. Об'єктивно встановлено: занурений у світ власних переживань та уявлень, тривожний, сумний, відсторонений від інших пацієнтів, спостерігаються <<флешбеки>>. Який попередній діагноз? A 34-year-old military man, fear, anxiety, irritability, depressed mood, terrible dreams about past events appeared 6 months after experiencing combat stress. Objectively established: immersed in the world of own experiences and ideas, anxious, sad, withdrawn from other patients, <> are observed. What is the previous diagnosis?

Гостра реакція на стрес Acute reaction to stress

Соматизований розлад Somatized disorder

Посттравматичний стресовий розлад Post-traumatic stress disorder

Депресивний розлад Depressive disorder

Іпохондричний розлад Hypochondriac disorder

914 / 1500
Пацієнт віком 38 років скаржиться на підвищення температури тіла до 39^oС, мерзлякуватість, профузний піт, тупий біль у попереку, що поширюється у надлобкову ділянку. Об'єктивно спостерігається: напруження м'язів у поперековому відділі, болісні відчуття під час постукування у поперековій ділянці з обох боків. У загальному аналізі крові спостерігається: лейкоцитоз - 12·10^9/л. У загальному аналізі сечі виявлено: протеїнурія - 0,7 г/л, лейкоцитурія 15-20 у полі зору, бактеріурія більше 100 000 на 1 мл сечі. Який найімовірніший діагноз? A 38-year-old patient complains of an increase in body temperature to 39^oС, chills, profuse sweat, dull pain in the lower back, spreading to the suprapubic area. Objectively observed: muscle tension in the lumbar region, painful sensations during tapping in the lumbar region on both sides: leukocytosis - 12·10^9/l. In the general analysis of urine: proteinuria - 0.7 g /l, leukocyturia 15-20 in the field of vision, bacteriuria more than 100,000 per 1 ml of urine. What is the most likely diagnosis?

Туберкульоз нирок Kidney tuberculosis

Гострий цистит Acute cystitis

Гострий пієлонефрит Acute pyelonephritis

Гострий гломерулонефрит Acute glomerulonephritis

Нирковокам'яна хвороба Kidney stone disease

915 / 1500
Хлопчика 2-х років госпіталізовано з приводу зменшення маси тіла, нестійких випорожнень, анорексії, які з'явилися після введення в раціон манної каші (з 5 місяців). Дитина адинамічна, млява, шкіра бліда, суха, підшкірно-жировий шар відсутній. Живіт здутий, напружений. Під час перкусії у верхній частині живота тимпаніт, шум плеску, випорожнення пінисті, світлого кольору, смердючі. В копроцитограмі: нейтральний жир - багато. Який наступний крок у веденні пацієнта буде найбільш доречним? A 2-year-old boy was hospitalized due to a decrease in body weight, unstable stools, anorexia, which appeared after the introduction of semolina into the diet (from 5 months). The child is apathetic, the skin is pale, the subcutaneous fat layer is absent. During percussion in the upper part of the abdomen, there is foamy, foul-smelling stool. In the coprocytogram, there is a lot of fat What would be the most appropriate next step in patient management?

Рентгенографія органів черевної порожнини X-ray of abdominal organs

Негайне оперативне втручання Immediate operative intervention

Розробка індивідуального плану харчування з великим вмістом клітковини Development of an individual meal plan with high fiber content

Визначення IgA до тканинної трансглютамінази Determination of IgA to tissue transglutaminase

Призначення антибіотиків широкого спектру дії Prescription of broad-spectrum antibiotics

916 / 1500
Чоловік 27-ми років, спортсмен, під час тренування раптово втратив свідомість. Пульс і артеріальний тиск не визначаються. Дихання відсутнє. Тони серця не вислуховую-ться. Зіниці широкі. На ЕКГ: поліморфна шлуночкова тахікардія. Розпочато компресію грудної клітки. Який наступний крок буде найбільш доречним? A 27-year-old man, an athlete, suddenly lost consciousness during training. The pulse and blood pressure are not determined. There is no breathing. Heart sounds are not heard. Pupils wide. On the ECG: ventricular tachycardia. Chest compressions are started. What would be the most appropriate next step?

Внутрішньовенне введення дигоксину Intravenous digoxin

Електрична дефібриляція Electrical defibrillation

Вагусні проби Vagus tests

Черезстравохідна кардіостимуляція Transesophageal cardiac stimulation

Внутрішньовенне введення лідокаїну Intravenous administration of lidocaine

917 / 1500
Сімейний лікар вирішив зайнятися підприємницькою діяльність в сфері надання первинної медичної допомоги населенню. Яку організаційно-правову форму підприєм-ництва доцільно обрати? A family doctor decided to start a business in the field of providing primary medical care to the population. What organizational and legal form of business should be chosen?

Підприємство зі створенням юридичної особи Enterprise with the creation of a legal entity

Товариство з повною відповідальністю General Liability Company

Акціонерне товариство Joint joint stock company

Товариство з обмеженою відповідальністю Limited Liability Company

Приватний підприємець без створення юридичної особи Private entrepreneur without creating a legal entity

918 / 1500
Жінка 62-х років доставлена у приймальне відділення зі скаргами на сильний пекучий біль за грудиною, задуху. В анамнезі: 10 років гіпертонічна хвороба. Об'єктивно: стан середньої важкості, шкіра бліда, ціаноз губ, над легенями везикулярне дихання. Тони серця приглушені, ритмічні. Акцент II тону над аортою. Артеріальний тиск - 210/120 мм рт.ст. Частота серцевих скорочень = пульс - 76/хв. На ЕКГ: підвищення сегменту ST в I, aVL, V5-V6 відведеннях. Найбільш імовірний діагноз: A 62-year-old woman was brought to the reception department with complaints of severe burning pain behind the sternum, suffocation. History: 10 years of hypertension. Objectively: condition of moderate severity, cyanosis of the lips, muffled heart sounds over the aorta, 210/120 mm Hg, heart rate: 76/min. ST segment elevation in I, aVL, V5-V6 leads. The most likely diagnosis:

ТЕЛА BODIES

Гіпертонічний криз, ускладнений нестабільною стенокардією Hypertensive crisis complicated by unstable angina

Гіпертонічний криз, ускладнений гострим інфарктом міокарда Hypertensive crisis complicated by acute myocardial infarction

Неускладнений гіпертонічний криз Uncomplicated hypertensive crisis

Гіпертонічний криз, ускладнений гострою лівошлуночковою недостатністю Hypertensive crisis complicated by acute left ventricular failure

919 / 1500
Восьмирічна дівчинка скаржиться на біль у навколопупковій ділянці, що з'являється вранці натщесерце. Діагностовано гастродуоденіт, асоційований з Н. pylori. Які препарати входять до складу ерадікаційної терапії H. pylori? An eight-year-old girl complains of pain in the periumbilical area, which appears in the morning on an empty stomach. Gastroduodenitis associated with H. pylori is diagnosed. What drugs are included in the eradication therapy of H pylori?

Алюмінію фосфат, фамотидин, метронідазол Aluminum phosphate, famotidine, metronidazole

Колоїдний субцитрат вісмуту, прифінія бромід, азитроміцин Colloidal bismuth subcitrate, prifinia bromide, azithromycin

Мебеверин, фуразолідон, омепразол Mebeverin, furazolidone, omeprazole

Тримебутин, рокситроміцин, ранітидин Trimebutin, roxithromycin, ranitidine

Пантопразол, кларитроміцин, метронідазол Pantoprazole, Clarithromycin, Metronidazole

920 / 1500
У пацієнта віком 30 років після вживання грибів виникли нудота і блювання, зникла сеча. На третій день симптомів звернувся за медичною допомогою. Під час лабораторних досліджень виявлено: рівень креатиніну підвищився до 700 мкмоль/л, сечовина - до 32 ммоль/л. Якою буде лікувальна тактика в цій ситуації? A 30-year-old patient developed nausea and vomiting after eating mushrooms, and urine disappeared. On the third day of symptoms, he sought medical help. Laboratory tests revealed: creatinine level increased to 700 μmol/l, urea - up to 32 mmol/l. What will be the treatment tactics in this situation?

Дезінтоксикаційна терапія Detoxification therapy

Антидотна терапія Antidote therapy

Гемодіаліз Hemodialysis

Перитонеальний діаліз Peritoneal dialysis

Сечогінні препарати Diuretics

921 / 1500
У п'ятиденної дитини спостерігається дифузна еритема, пухирі, ерозії, тріщини та лущення епідермісу. Хлопчик має вигляд ошпареного окропом. Під час огляду: загальний стан дитини дуже тяжкий. Виражене занепокоєння. Температура тіла 39,8^oС. Дитина відмовляється від їжі. Позитивний симптом Нікольського. Гіперестезія. Який діагноз імовірний? A five-day-old child has diffuse erythema, blisters, erosions, cracks and peeling of the epidermis. The boy has the appearance of being scalded with boiling water. On examination: the general condition of the child is very serious Pronounced anxiety. The child refuses to eat. What is the probable diagnosis?

Мiкотична еритема Mycotic erythema

Пухирчатка новонародженого Neonatal pemphigus

Псевдофурункульоз Фiнгера Pseudofurunculosis of Finger

Флегмона новонародженого Phlegmon of a newborn

Ексфолiативний дерматит Exfoliative dermatitis

922 / 1500
Після перенесеного геморагічного інсульту хворий з метою відновлення мови та рухів лівої нижньої та верхньої кінцівок проходить курс масажу, фізпроцедур та занять з логопедом. До якого виду профілактики належать вказані заходи реабілітації? After suffering a hemorrhagic stroke, the patient undergoes a course of massage, physical procedures and classes with a speech therapist in order to restore speech and movements of the left lower and upper limbs. To what type of prevention do these measures belong rehabilitation?

Індивідуальна профілактика Individual prevention

Третинна профілактика Tertiary prevention

Громадська профілактика Public prevention

Вторинна профілактика Secondary prevention

Первинна профілактика Primary prevention

923 / 1500
У юнака 18 років раптово з'явився сильний головний біль, блювання, підвищення температури тіла до 39^oС. Об'єктивно встановлено: положення в ліжку із закинутою головою та підведеними до живота ногами. Загальна гіперестезія. Позитивні симптоми Брудзинського та Керніга. Вогнищева неврологічна симптоматика відсутня. У лікворі - нейтрофільний плеоцитоз. Який найімовірніший діагноз? An 18-year-old boy suddenly developed a severe headache, vomiting, and an increase in body temperature up to 39°C. Objectively established: position in bed with head thrown back and legs drawn up. Brudzinsky's and Kernig's symptoms are absent. In the cerebrospinal fluid, what is the most likely diagnosis?

Субдуральна гематома Subdural hematoma

Абсцес головного мозку Brain abscess

Енцефаліт Encephalitis

Субарахноїдальний крововилив Subarachnoid hemorrhage

Менінгіт Meningitis

924 / 1500
Дитині 4 місяці. Захворіла гостро, з підвищення температури до 37,8^oC і покашлювання. На 3-й день кашель посилився, приєдналася задишка. Перкуторно над легенями прослуховується тимпанічний звук, аускультативно - з обох сторін велика кількість дрібноміхурчатих вологих та свистячих хрипів під час видиху. Який попередній діагноз? The child is 4 months old. She became acutely ill, with a temperature rise to 37.8^oC and a cough. On the 3rd day, the cough intensified, shortness of breath joined. Percussion over the lungs a tympanic sound is heard, on both sides a large number of small bubbles and whistling sounds during exhalation. What is the preliminary diagnosis?

Гострий бронхіт Acute bronchitis

Обструктивний бронхіт Obstructive bronchitis

Бронхопневмонія Bronchopneumonia

Вогнищева пневмонія Focal pneumonia

Гострий бронхіоліт Acute bronchiolitis

925 / 1500
Юнака віком 19 років шпиталізовано до гематологічного відділення зі скаргами на біль у правому плечовому суглобі, що виник після його забою. З анамнезу відомо, що така клінічна картина спостерігалась неодноразово з раннього дитинства. Об'єктивно спостерігається: суглоб збільшений в об'ємі, різко болючий при пальпації. В аналізі крові визначається: еритроцити - 3,7·10^12/л, Hb - 110 г/л, тромбоцити - 175·10^9/л, лейкоцити - 6,9·10^9/л, ШОЕ - 25 мм/год, протромбіновий індекс - 90%, час рекальцифікації - 280 хв, час згортання крові: початок - 10 хв, кінець - 38 хв, АЧТЧ - 90 хв, фібриноген - 3,5 г/л. Який найімовірніший діагноз? A 19-year-old young man was hospitalized in the hematology department with complaints of pain in the right shoulder joint that occurred after his slaughter. It is known from the anamnesis that such a clinical picture has been observed more than once Objectively observed: the joint is enlarged, sharply painful during palpation: erythrocytes - 3.7·10^12/l, Hb - 110 g/l, platelets - 175·10. ^9/l, leukocytes - 6.9·10^9/l, ESR - 25 mm/h, prothrombin index - 90%, recalcification time - 280 min, blood clotting time: start - 10 min, end - 38 min, APTC - 90 min, fibrinogen - 3.5 g/l. What is the most likely diagnosis?

Аутоімунна тромбоцитопенія Autoimmune thrombocytopenia

Гемофілія Hemophilia

Геморагічний васкуліт Hemorrhagic vasculitis

Тромбоцитопатія Thrombocytopathy

Аутоімунна коагулопатія Autoimmune coagulopathy

926 / 1500
Жінці віком 25 років у зв'язку з перенесеним бактеріальним тонзилітом призначено антибіотик, який вона не приймала. Через 2 тижні з'явилися набряки обличчя, олігурія, піднявся артеріальний тиск. Лабораторно виявлено: креатинін крові - 340 мкмоль/л, сечовина - 42 ммоль/л. Яке ускладнення розвинулося у пацієнтки? A 25-year-old woman was prescribed an antibiotic in connection with previous bacterial tonsillitis, which she did not take. After 2 weeks, facial swelling, oliguria, blood pressure rose blood pressure. Laboratory revealed: blood creatinine - 340 μmol/l, urea - 42 mmol/l. What complication developed in the patient?

Гострий гломерулонефрит, нефритичний синдром Acute glomerulonephritis, nephritic syndrome

Гострий гломерулонефрит, гостра ниркова недостатність Acute glomerulonephritis, acute renal failure

Гострий гломерулонефрит, нефротичний синдром Acute glomerulonephritis, nephrotic syndrome

Гострий гломерулонефрит, артеріальна гіпертензія Acute glomerulonephritis, arterial hypertension

Хронічний гломерулонефрит, хронічна ниркова недостатність Chronic glomerulonephritis, chronic renal failure

927 / 1500
Анестезіолог проводить наркоз під час ургентної операції з приводу розлитого перитоніту жінці 65-ти років вагою 135 кг. Спроби інтубації трахеї після введення міо-релаксантів тричі поспіль виявились невдалими. Штучна вентиляція легень через лицьову маску ефективна. Яким чином забезпечити прохідність дихальних шляхів і проведення штучної вентиляції легень? The anesthesiologist administers anesthesia during an emergency operation for spilled peritonitis to a 65-year-old woman weighing 135 kg. Attempts to intubate the trachea after the introduction of myo-relaxants were unsuccessful three times in a row. Artificial ventilation of the lungs through a face mask is effective. How to ensure the patency of the respiratory tract and the implementation of artificial ventilation of the lungs?

Проводити штучну вентиляцію легень через лицьову маску Perform artificial lung ventilation through a face mask

Проводити штучну вентиляцію легень, встановивши орофарингеальний повітровід Perform artificial lung ventilation by installing an oropharyngeal airway

Проводити штучну вентиляцію легень, встановивши назофарингеальний повітровід Carry out artificial lung ventilation by installing a nasopharyngeal airway

Проводити штучну вентиляцію легень, встановивши ларингеальну маску Carry out artificial lung ventilation by installing a laryngeal mask

Проводити штучну вентиляцію легень через назальні канюлі Perform artificial lung ventilation through nasal cannulas

928 / 1500
Жінка 37-ми років звернулася до лікаря зі скаргами на болі у ділянці попереку, підвищення температури тіла до 38^oC. При фізикальному обстеженні позитивний симптом Пастернацького, більше виражений справа. При лабораторному дослідженні у загальному аналізі крові: гемоглобін - 115 г/л, еритроцити - 3,9·10^12/л, лейкоцити - 15,2·10^9/л, ШОЕ- 28 мм/год. У загальному аналізі сечі: колір - світло жовтий, питома вага - 1018, білок - 0,42 г/л, лейкоцити - 20-30 у полі зору, багато слизу. Який діагноз є найбільш імовірним? A 37-year-old woman consulted a doctor with complaints of pain in the lower back, an increase in body temperature to 38°C. During the physical examination, Pasternacki's symptom was positive, more pronounced On the right, in the general blood analysis: hemoglobin - 115 g/l, erythrocytes - 3.9·10^12/l, ESR - 28 mm/h urine analysis: color - light yellow, specific gravity - 1018, protein - 0.42 g/l, leukocytes - 20-30 in the field of vision, a lot of mucus. What is the most likely diagnosis?

Гострий апендицит Acute appendicitis

Гострий пієлонефрит Acute pyelonephritis

Тубулоінтерстиційний нефрит Tubulointerstitial nephritis

Гострий гломерулонефрит Acute glomerulonephritis

Гострий аднексит Acute adnexitis

929 / 1500
До лікарні шпиталізовано п'ятирічну дівчинку після електротравми. Стан дитини вкрай тяжкий, дівчинка без свідомості, самостійне дихання відсутнє. Під час проведення серцево-легеневої реанімації на ЕКГ спостерігаються хвилі різної форми та амплітуди з частотою 320/хв, пульс на периферії та центральних артеріях - відсутній. Яка невідкладна допомога у цьому разі? A five-year-old girl was hospitalized after an electric shock. The child's condition is extremely serious, the girl is unconscious, she is not breathing on her own. During cardiopulmonary resuscitation, the ECG shows waves of different shapes and amplitudes with a frequency of 320/min, a pulse on the periphery and central arteries is absent. What is the emergency help in this case?

Інтубація трахеї Tracheal intubation

Електрична дефібриляція Electrical defibrillation

Трансфузія кристалоїдів 10 мг/кг в/в Transfusion of crystalloids 10 mg/kg IV

Уведення розчину лідокаїну 20 мг в/м Introduction of lidocaine solution 20 mg IV

Прямий масаж серця Direct heart massage

930 / 1500
Чоловік 19-ти років, звернувся до лікаря зі скаргами на безболісні ерозії на голівці статевого члена, які з'явилися 2-3 дні тому. У анамнезі незахищений статевий контакт приблизно місяць тому. При обстеженні на голівці статевого члена ерозія овальної форми діаметром до 5 мм, безболісна, яскраво-червоного кольору. Пахвинні лімфовузли не збільшені. Яке першочергове лабораторне дослідження буде найбільш доречним? A 19-year-old man turned to the doctor with complaints of painless erosions on the head of the penis that appeared 2-3 days ago. He has a history of unprotected sex contact about a month ago. On examination, an oval-shaped erosion of up to 5 mm in diameter. The inguinal lymph nodes are not enlarged. What would be the most appropriate laboratory test?

- -

Реакція імунофлюоресценції Immunofluorescence reaction

Реакція іммобілізації блідих трепонем Reaction of immobilization of pale treponems

Мікрореакція преципітації з кардіоліпіновим антигеном Microreaction of precipitation with cardiolipin antigen

Культуральне дослідження Cultural Research

931 / 1500
Дитина народилася на 8 місяці вагітності. У неї встановлено: мікроцефалію, катаракту, ваду серця. Мати дитини на 2-му місяці вагітності хворіла: було нетривале підвищення температури тіла до 37,5^oС, збільшення лімфатичних вузлів та дрібноплямистий висип на обличчі, тулубі й кінцівках, який пройшов без залишкових явищ. Який найбільш імовірний попередній діагноз у дитини і матері? The child was born in the 8th month of pregnancy. She was diagnosed with: microcephaly, cataracts, heart defect. The child's mother was ill in the 2nd month of pregnancy: there was a short-term increase in body temperature up to 37.5°C, an increase in lymph nodes and a small papular rash on the face, trunk and limbs, which passed without residual phenomena. What is the most likely preliminary diagnosis for the child and the mother?

Краснуха Krasnukha

Герпетична інфекція Herpes infection

Цитомегаловірусна інфекція Cytomegalovirus infection

Токсоплазмоз Toxoplasmosis

Хламідійна інфекція Chlamydial infection

932 / 1500
39-річна хвора жінка скаржиться на підвищення температури тіла до 37,8^oC протягом двох діб, часте сечовипускання, ниючий біль у поперековій ділянці. Зазначені симптоми виникли вперше після переохолодження. Під час фізикального обстеження спостерігається болючість пальпації в ділянці нирок. Результат загального аналізу сечі: pH - лужна, білок - 0,099 г/л, лейкоцити вкривають усе поле зору, еритроцити - 0 у полі зору, циліндри - 0 у полі зору. Результат загального аналізу крові: гемоглобін - 140 г/л, лейкоцити - 9,2·10^9/л, ШОЕ - 30 мм/год. Який препарат слід вибрати для проведення емпіричної антибактеріальної терапії? A 39-year-old sick woman complains of an increase in body temperature to 37.8^oC for two days, frequent urination, aching pain in the lumbar region. These symptoms appeared for the first time after hypothermia. During the physical examination, palpation is observed in the area of ​​the kidneys. The result of the general analysis of urine: alkaline, protein - 0.099 g/l, leukocytes cover the entire field of vision, erythrocytes - 0 in the field of vision. The result of a general blood test: hemoglobin - 140 g/l, leukocytes - 9.2·10^9/l, ESR - 30 mm/h. Which drug should be chosen for empirical antibacterial therapy?

Фосфоміцин Fosfomycin

Триметоприм-сульфаметоксазол Trimethoprim-sulfamethoxazole

Амікацин Amikacin

Ципрофлоксацин Ciprofloxacin

Амоксицилін Amoxicillin

933 / 1500
Під час зовнішнього акушерського обстеження спостерігається: живіт ової-дної форми, у лівій бічній частині матки пальпується спинка плода, у правій - дрібні частини плода, біля дна - щільна частина, що балотує. Серцебиття плода прослуховується ліворуч та вище пупка. Якими є положення, позиція і передлежання плода? During the external obstetric examination, the following is observed: an oval-bottomed abdomen, the back of the fetus is palpated in the left lateral part of the uterus, in the right - small parts of the fetus, near the bottom - dense The fetal heartbeat is heard on the left and above the navel. What is the position, position and presentation of the fetus?

Поперечне положення, I позиція, частина, що передлежить, відсутня Transverse position, I position, the preceding part is missing

Поздовжнє положення, II позиція, тазове передлежання Longitudinal position, II position, pelvic presentation

Поздовжнє положення, I позиція, головне передлежання Longitudinal position, I position, main presentation

Поздовжнє положення, I позиція, тазове передлежання Longitudinal position, I position, pelvic presentation

Поздовжнє положення, II позиція, головне передлежання Longitudinal position, II position, main presentation

934 / 1500
До сімейної лікарки звернулася жінка 54 років для профілактичного огляду. Зріст - 164 см, вага - 84 кг, АТ - 130/80 мм рт. ст. Менопауза 5 років. Мати пацієнтки померла від раку молочної залози, молодша сестра страждає на мастопатію. Під час огляду ущільнень у молочних залозах не виявлено, органи малого тазу в межах вікової норми. Цитологія шийки матки без особливостей. Що порекомендувати жінці? A 54-year-old woman came to the family doctor for a preventive examination. Height - 164 cm, weight - 84 kg, blood pressure - 130/80 mm Hg. Menopause 5 The patient's mother died of breast cancer, no lumps were found in the mammary glands. Cervical cytology was normal for the woman.

Регулярно проводити самообстеження молочних залоз Regular self-examination of the mammary glands

Прийти на наступний профогляд за 2 роки Come to the next professional examination in 2 years

Приходити на огляд кожних 3 місяці Come for an inspection every 3 months

Пройти МРТ молочних залоз Pass MRI of mammary glands

Проходити маммографію 1 раз на рік Get a mammogram once a year

935 / 1500
Пацієнтка віком 23 роки впродовж двох років відмічає похолодання пальців рук, вони набувають синюшно-білого кольору та стають нечутливими. Через 5-10 хвилин виникає почервоніння шкіри та пальці теплішають, що супроводжується різким болем. Який попередній діагноз? A 23-year-old patient has noticed coldness in her fingers for two years, they become bluish-white in color and become insensitive. After 5-10 minutes, the skin reddens and the fingers warm up , which is accompanied by sharp pain. What is the previous diagnosis?

Хвороба Бюргера Burger's disease

Хвороба Рейно Raynaud's disease

Поліневрит Polyneuritis

Облітеруючий атеросклероз Obliterating atherosclerosis

Облітеруючий ендартериїт Endarteritis obliterans

936 / 1500
Пацієнтка віком 63 роки оперована з приводу багатовузлового еутиреоїдного зобу великих розмірів. З технічними труднощами вимушено виконана субтотальна резекція обох часток щитоподібної залози. На 4-й день після операції з'явилися судоми м’язів обличчя і верхніх кінцівок, біль в животі. Позитивні симптоми Хвостека і Труссо. Чим найімовірніше зумовлений такий стан пацієнтки? A 63-year-old patient underwent surgery for a large multinodular euthyroid goiter. With technical difficulties, a subtotal resection of both lobes of the thyroid gland was forced to be performed. On the 4th day after the operation with 'convulsions of the muscles of the face and upper limbs appeared. Positive symptoms of Khvostek and Trousseau. What is the most likely cause of this condition of the patient?

Недостатність паращитоподібних залоз Parathyroid insufficiency

Післяопераційний гіпотиреоз Postoperative hypothyroidism

Трахеомаляція Tracheomalacia

Пошкодження зворотнього нерву Injury of the recurrent nerve

Тиреотоксичний криз Thyrotoxic crisis

937 / 1500
Хлопець 17 рокiв скаржится на кровоточивiсть ясен. В анамнезі вказані: носовi кровотечi, загальна слабкiсть. Об'єктивно встановлено: помірна блiдiсть шкiряних покривiв i слизової оболонки рота, на слизовiй щiк, язика та м'якого пiднебiння множиннi петехiї. У кровi виявлено: еритроцити - 3,8 Т/л, Hb - 104 г/л, КП - 0,82, лейкоцити - 4,9 Г/л, тромбоцити - 80 Г/л, час зсiдання кровi - 5 хв., ШОЕ - 18 мм/год. Який найбiльш iмовiрний дiагноз? A 17-year-old boy complains of bleeding gums. The anamnesis indicates: nosebleeds, general weakness. Objectively established: moderate pallor of the skin and mucous membrane of the mouth, on mucosa of the cheeks, tongue and soft palate, multiple petechiae were found in the blood: erythrocytes - 3.8 T/l, Hb - 104 g/l, KP - 0.82, leukocytes - 4.9 g/l, platelets - 80. g/l, blood clotting time - 5 min., ESR - 18 mm/h. What is the most likely diagnosis?

Істинна поліцитемія Polycythemia True

Хронiчний лiмфолейкоз Chronic lymphocytic leukemia

Залiзодефiцитна анемiя Iron deficiency anemia

Перніціозна анемія Pernicious anemia

Ідіопатична тромбоцитопенічна пурпу-ра Idiopathic thrombocytopenic purpura

938 / 1500
У повторнонароджуючої з початком термінових пологів з'явились геморагічні виділення з піхви. Під час внутрішнього акушерського дослідження виявлено: шийка матки згладжена, відкриття вічка на 6 см, внутрішнє вічко на 1/3 перекрито губчастою тканиною. На решті ділянки пальпується плідний міхур. Пологова діяльність активна. Яка тактика в цій ситуації? With the onset of term labor, a woman who gave birth again developed hemorrhagic discharge from the vagina. During the internal obstetric examination, it was found that the cervix was smoothed, the opening of the eye was 6 cm, the internal eye 1/3 is covered with spongy tissue. The amniotic sac is palpable. What is the tactic in this situation?

Переривання вагітності Termination of pregnancy

Стимуляція пологової діяльності Stimulation of labor activity

Кесарський розтин Caesarean section

Гемостатична терапія Hemostatic therapy

Амніотомія Amniotomy

939 / 1500
Чоловік 58-ми років звернувся до лікаря зі скаргами на кашель зі слизово-гнійним мокротинням, задишку при фізичному навантаженні. Зазначені симптоми турбують протягом 14-ти років. Щоденно викурює 1,5 пачки сигарет з 17-ти років. При фізикальному обстеженні пульс - 96/хв., артеріальний тиск - 130/80 мм рт.ст. Перкуторно над усією поверхнею легень коробковий звук. Аускультативно дихання ослаблене, на фоні подовженого видиху розсіяні сухі хрипи. За даними спірометрії співвідношення ОФВ_1/ФЖЄЛ - 30%. Який діагноз є найбільш імовірним? A 58-year-old man consulted a doctor with complaints of a cough with mucous-purulent sputum, shortness of breath during physical exertion. These symptoms have been bothering him for 14 years. Daily smokes 1.5 packs of cigarettes since the age of 17. On physical examination, pulse - 130/80 mmHg dry wheezing. According to spirometry, the FEV_1/FJEL ratio is 30%. What is the most likely diagnosis?

Хронічне обструктивне захворювання легень (ХОЗЛ) Chronic obstructive pulmonary disease (COPD)

Рак легень Lung cancer

Фіброзуючий альвеоліт Fibrosing alveolitis

Бронхіальна астма Bronchial asthma

Бронхоектатична хвороба Bronchoectatic disease

940 / 1500
До лікаря-гінеколога звернулася мати дванадцятирічної дівчинки для проведення дитині вакцинації проти високоонкогенних типів вірусу папіломи людини. З якого віку доцільно виконувати це щеплення? The mother of a twelve-year-old girl turned to a gynecologist to vaccinate her child against highly oncogenic types of the human papilloma virus. From what age is it advisable to carry out this vaccination?

14-15 років 14-15 years

18-19 років 18-19 years

16-17 років 16-17 years

20-21 рік 20-21 years

12-13 років 12-13 years

941 / 1500
У десятирічного хлопчика зріст становить 129 см, що знаходиться в групі нижче середніх величин шкали регресії, маса тіла - 30,8 кг і ОГК - 60,5 см відповідають зросту. Число постійних зубів - 14 за норми 18pm3. Щорічне збільшення зросту - 3 см за норми 4-6, ЖЄЛ - 1800 мл, м'язова сила правої руки - 16,5 і лівої - 15,5 кг відповідають віку. За минулий рік переніс пневмонію. Хворіє на хронічний бронхіт. Оцініть фізичний розвиток дитини. A ten-year-old boy's height is 129 cm, which is in the group below the average values ​​of the regression scale, body weight - 30.8 kg and OGK - 60.5 cm correspond to The number of permanent teeth is 14 per year. The annual increase in height is 4-6 per year suffered from pneumonia for a year. He has chronic bronchitis. Assess the child's physical development.

Біологічний вік відстає від календарного, фізичний розвиток низький, гармонійний, 4 група здоров'я Biological age lags behind the calendar age, physical development is low, harmonious, 4th health group

Біологічний вік відповідає календарному, фізичний розвиток низький, дисгармонійний, 4 група здоров'я Biological age corresponds to the calendar age, physical development is low, disharmonious, health group 4

Біологічний вік відстає від календарного, фізичний розвиток нижче середнього, гармонійний, 3 група здоров'я Biological age lags behind the calendar, physical development is below average, harmonious, health group 3

Біологічний вік випереджає календарний, фізичний розвиток середній, гармонійний, 2 група здоров'я Biological age is ahead of calendar age, physical development average, harmonious, 2nd health group

Біологічний вік згідно календарного, фізичний розвиток нижче середнього, дисгармонійний, 3 група здоров'я Biological age according to the calendar, physical development below average, disharmonious, health group 3

942 / 1500
До лікарні шпиталізовано чоловіка віком 45 років. В анамнезі пацієнта наявне варикозне розширення вен нижніх кінцівок. Він раптово відчув біль у грудній клітці та ядуху. Об'єктивно спостерігається: набухання шийних вен, ціаноз. На ЕКГ виявлено: ознака Мак-Джина-Уайта (QIII-SI), P- pulmonale, відхилення електричної вісі серця вправо, ознаки перевантаження правих відділів серця. Визначення рівня якої речовини в сироватці крові необхідно провести для підтверження діагнозу ? A 45-year-old man was hospitalized. The patient's anamnesis has varicose veins of the lower extremities. He suddenly felt pain in the chest and throat. Objectively observed: Swelling of the neck veins, cyanosis. The ECG revealed: Mc-Gin-White's sign (QIII-SI), deviation of the electrical axis of the heart to the right. Determination of the level of the substance in the blood serum is necessary to confirm the diagnosis ?

D-димеру D-dimer

Аланінамінотрансферази Alanine aminotransferases

Аспартатамінотрансферази Aspartate aminotransferases

Серцевого тропоніна Cardiac Troponin

Креатинфосфокінази Creatine phosphokinase

943 / 1500
Дівчинка 12 років поступила до гінекологічного відділення зі скаргами на рясні кров'янисті виділення зі статевих шляхів протягом 10 діб. В анамнезі вказано менархе з 11 років, менструації нерегулярні. Під час ректо-абдомінального обстеження патології не виявлено. Який попередній діагноз? A 12-year-old girl was admitted to the gynecological department with complaints of profuse bleeding from the genital tract for 10 days. The anamnesis indicates menarche since the age of 11, menstruation is irregular. During the recto-abdominal examination, no pathology was detected. What was the previous diagnosis?

Аномальна маткова кровотеча Abnormal uterine bleeding

Синдром Штейна-Левенталя Stein-Leventhal syndrome

Ідіопатична тромбоцитопенічна пурпура Idiopathic thrombocytopenic purpura

Генітальний інфантилізм Genital infantilism

Гіповітаміноз Hypovitaminosis

944 / 1500
Жінка віком 25 років шпиталізована до гінекологічного відділення зі скаргами на підвищення температури тіла до 38,7^oС, біль унизу живота, гнійні виділення з піхви. З анамнезу відомо, що 6 років тому їй було встановлено внутрішньоматкову спіраль. Під час вагінального обстеження виявлено: шийка матки циліндрична з явищами ендоцервіциту, виділення з цервікального каналу гнійні, пальпуються нитки спіралі. Тіло матки нормальних розмірів, болюче. Придатки з обох боків болючі. Параметрії вільні. Яка тактика лікаря найбільш доцільна у цьому разі? A 25-year-old woman was hospitalized in the gynecological department with complaints of an increase in body temperature up to 38.7°C, pain in the lower abdomen, purulent discharge from the vagina. It is known from the anamnesis , that she had an intrauterine spiral installed 6 years ago. During the vaginal examination, the cervix was cylindrical with symptoms of endocervicitis, the uterine thread was palpable, and the appendages were painful on both sides. What is the most appropriate doctor's tactic in this case?

Антибіотикотерапія Antibiotic therapy

Аналіз вагінальних виділень, антибіотикотерапія Analysis of vaginal secretions, antibiotic therapy

Роздільне вишкрібання матки Separate uterine scraping

Гістероскопія, антибіотикотерапія Hysteroscopy, antibiotic therapy

Видалення внутрішньоматкової спіралі, антибіотикотерапія Removal of intrauterine spiral, antibiotic therapy

945 / 1500
Чоловік 62 років. Протягом останніх 3 років без видимих причин стала зростати безпорадність, зниженя пам'яті. У психічному стані встановлено: правильно називає своє прізвище, ім'я, по батькові, дату народження; вік сказати не може. Дезорієнтований у місцевості, у часі. Пам'ять знижена на поточні та віддалені події. Інтелект знижений. Порушені вищі коркові функції з явищами афазії, апраксії, агнозії. Критика відсутня. Визначіть провідний патогенетичний механізм формування описаного стану: The man is 62 years old. Over the past 3 years, for no apparent reason, helplessness and memory loss began to increase. In his mental state, it was established: correctly pronounces his last name, first name , date of birth.Disoriented in time.Decreased intelligence with aphasia, agnosia the mechanism of formation of the described state:

Нейродегенеративний Neurodegenerative

Гіперліпідемія Hyperlipidemia

Дефіцит норадреналіну Noradrenaline deficiency

Надлишок ацетилхолінтрансферази Acetylcholine transferase excess

Надлишок серотоніна в синаптичній щілині Excess serotonin in the synaptic cleft

946 / 1500
Жінка 36-ти років звернулася до лікаря зі скаргами на пригнічений настрій, тривожність, дратівливість та нагрубання молочних залоз, що виникають кожного місяця. Найчастіше подібні симптоми виникають за 2 тижні до менструації та зникають після неї. Який з перерахованих діагнозів є найбільш імовірним? A 36-year-old woman consulted a doctor with complaints of depressed mood, anxiety, irritability, and engorgement of the mammary glands that occur every month. Most often, similar symptoms occur after 2 weeks before menstruation and disappear after it. Which of the listed diagnoses is most likely?

Альгодисменорея Algodysmenorrhea

Преклімактеричний синдром Pre-menopausal syndrome

Передменструальний синдром Premenstrual syndrome

Ранній патологічний клімакс Early pathological menopause

Вторинна психогенна аменорея Secondary psychogenic amenorrhea

947 / 1500
Жінка 46-ти років доставлена до відділення невідкладної допомоги зі скаргами на постійний, інтенсивний абдомінальний біль протягом 8 годин, нудоту та блювання. В анамнезі гіпертригліцеридемія, яку пацієнтка не лікувала. При фізикальному обстеженні температура тіла - 38^oC, артеріальний тиск - 100/60 мм рт.ст., пульс - 122/хв. Іктеричність кон'юнктив, при пальпації черевної стінки позитивний симптом флюктуації, позитивні симптоми Хвостека та Труссо. При лабораторному дослідженні в сироватці крові натрій - 142 ммоль/л, калій - 3,1 ммоль/л, бікарбонат - 32 ммоль/л, креа-тинін - 106 мкмоль/л, кальцій - 0,9 ммоль/л. Який перший крок у веденні пацієнтки буде найбільш доречним? A 46-year-old woman was brought to the emergency department with complaints of constant, intense abdominal pain for 8 hours, nausea and vomiting. She has a history of hypertriglyceridemia, which the patient does not During the physical examination, the body temperature was 38°C, the blood pressure was 100/60 mmHg, the conjunctiva was icteric, and the abdominal wall was palpated with positive symptoms of Khvostek laboratory research in blood serum sodium - 142 mmol/l, potassium - 3.1 mmol/l, bicarbonate - 32 mmol/l, creatinine - 106 μmol/l, calcium - 0.9 mmol/l. What is the first step in management of the patient will be the most appropriate?

Апротиніну 10 000 АТрОд (антитрипсинових одиниць) внутрішньовенно Aprotinin 10,000 ATrOd (antitrypsin units) intravenously

Негайна літотрипсія Immediate lithotripsy

Визначення концентрації алкоголю в крові Determination of alcohol concentration in blood

Натрію хлориду 0,9%, кальцію глюконат та фентаніл внутрішньовенно Sodium chloride 0.9%, calcium gluconate and fentanyl intravenously

Ендоскопічна ретроградна холангіопанкреатографія (ЕРХПГ) Endoscopic retrograde cholangiopancreatography (ERCP)

948 / 1500
Чоловік віком 31 рік декілька років періодично лікувався з приводу радикуліту попереко-крижового відділу хребта. Протягом останніх 2-х років з'явився біль у суглобах верхніх та нижніх кінцівок, що посилюється у другій половині ночі. Об'єктивно виявлено: припухлість колінних суглобів, болючість при пальпації попереково-крижового відділу хребта, обмеження рухливості грудної клітки під час глибокого дихання. У зв'язку з підозрою на анкілозуючий спондиліт призначено рентгенограму хребта та тазових кісток. Які зміни на рентгенограмі найімовірніші? A 31-year-old man has been periodically treated for sciatica of the lumbosacral spine for several years. Over the past 2 years, pain has appeared in the joints of the upper and lower extremities , which increases in the second half of the night. Objectively revealed: swelling of the knee joints, pain during palpation of the lumbar-sacral spine, limitation of chest mobility during deep breathing. In connection with the suspicion of ankylosing spondylitis, an X-ray of the spine and pelvic bones was prescribed What are the most likely changes on the X-ray?

Остеопороз, узурація суглобової поверхні Osteoporosis, wear of the articular surface

Звуження суглобової щілини, остеофітоз Narrowing of the joint space, osteophytosis

Розширення суглобових щілин, субхондральний склероз Expansion of joint spaces, subchondral sclerosis

Двосторонній сакроілеїт Bilateral sacroiliitis

Численні кистовидні просвітлення Numerous brush-like enlightenments

949 / 1500
Чоловік 78-ми років, правильно називає своє прізвище, ім'я, дату народження. Не може назвати поточну дату, свій вік, дезорієнтований у місці перебування. Не пам'ятає про смерть дружини, яка померла 5 років тому, а також нічого не може повідомити про останні роки свого життя. Мова уповільнена, словниковий запас обмежений, часто не може згадати назви предметів. У пробі на запам'ятовування десяти слів відтворює три слова. Прості арифметичні дії виконує з помилками. Не може пояснити сенсу поширених прислів'їв і приказок. Який синдромальний діагноз є найбільш імовірним? A 78-year-old man correctly names his surname, first name, date of birth. He cannot name the current date, his age, he is disoriented in his location. No remembers the death of his wife, who died 5 years ago, and cannot report anything about the last years of his life.Language is delayed, vocabulary is limited, he often cannot recall the names of objects.Reproduces three words in the ten-word recall test . Performs simple arithmetic operations with errors. Cannot explain the meaning of common proverbs and sayings. What is the most likely syndromic diagnosis?

Депресивний синдром Depressive syndrome

Галюцинаторний синдром Hallucinatory syndrome

Деменція Dementia

Маячний синдром Delirious Syndrome

Маніакальний синдром Manic syndrome

950 / 1500
Хвора 34-х років після швидкої зміни положення тіла з горизонтального на вертикальне різко зблідла, впала, шкіра стала вологою, кінцівки холодні, зіниці розширились, пульс пришвидшений, ниткоподібний, АТ- 50/25 мм рт.ст. Який стан у неї, імовірно, розвинувся? A 34-year-old patient, after a rapid change in body position from horizontal to vertical, turned pale, fell, the skin became moist, the extremities were cold, the pupils dilated, the pulse accelerated, threadlike , blood pressure - 50/25 mm Hg. What condition did she probably develop?

Шок Shock

Кома Comma

Синдром Морганьї-Адамса-Стокса Morganhi-Adams-Stokes syndrome

Колапс Collapse

Фібриляція шлуночків Ventricular fibrillation

951 / 1500
Чоловік 45 років госпіталізований зі скаргами на біль, що виник раптово у лівій половині грудної клітки та епігастральній ділянці, утруднене дихання, задишку, нудоту, одноразову блювоту. Захворів гостро, після підняття великої ваги. Об'єктивно встановлено: дихання поверхневе, ЧДР - 38/хв, ліва половина грудної клітки відстає під час дихання. Під час перкусії прослуховується тимпанічний звук, дихання не прослуховується. Пульс - 110/хв, слабкого наповнення. АТ - 100/60 мм рт. ст. Яка патологія розвинулась у хворого? A 45-year-old man was hospitalized with complaints of pain that appeared suddenly in the left half of the chest and epigastric area, difficulty breathing, shortness of breath, nausea, one-time vomiting. He became acutely ill , after lifting a heavy weight. It is established that breathing is shallow, the left half of the chest lags during breathing, the pulse is not heard, the pulse is weak - 100/60 mm Hg. What pathology has developed in the patient?

Відрив хорди мітрального клапана Tear of the mitral valve chord

Закритий пневмоторакс Closed pneumothorax

Госта тампонада серця Host cardiac tamponade

Відкритий пневмоторакс Open pneumothorax

Клапанний пневмоторакс Valvular pneumothorax

952 / 1500
Чоловік віком 27 років скаржиться на кашель із виділенням гнійного харкотиння, біль у грудній клітці під час дихання, підвищення температури тіла до 38,6^oС. Захворів гостро після переохолодження. Був госпіталізований до стаціонару. Після лабораторного та рентгенологічного дослідження йому діагностовано позагоспітальну пневмонію. З якого антибіотика найдоцільніше почати лікування? A 27-year-old man complains of a cough with purulent sputum, pain in the chest during breathing, an increase in body temperature to 38.6^oC. He became acutely ill after hypothermia. He was hospitalized. After a laboratory and X-ray examination, he was diagnosed with community-acquired pneumonia. Which antibiotic is the most appropriate to start treatment with?

Тетрацикліну гідрохлорид Tetracycline hydrochloride

Амоксицилін клавуланат Amoxicillin clavulanate

Ванкоміцин Vancomycin

Лінкоміцин Lincomycin

Гентаміцину сульфат Gentamicin sulfate

953 / 1500
Хворий 58-ми років звернувся до лікаря зі скаргою на невелике безболісне випинання в пахвинній ділянці протягом останнього місяця. Випинання не збільшується, з'являє-ться при стоянні та зникає в положенні лежачи на спині. Утрудненого сечовипускання та закрепів немає. Пальпація черевної стінки безболісна. Ректальне дослідження патологічних змін не виявило. Що із перерахованого є найбільш доцільним наступним кроком лікаря у даній ситуа-ції? A 58-year-old patient turned to the doctor with a complaint of a small painless bulge in the inguinal region during the last month. The bulge does not increase, appears when standing and disappears in the supine position. There is no difficulty urinating. Palpation of the abdominal wall does not reveal any pathological changes. Which of the following is the most appropriate step for the doctor in this situation?

Провести відкриту пластику із застосуванням сітки негайно Make an open plastic using a grid immediately

Провести тонкоголкову пункційну біопсію Perform fine-needle puncture biopsy

Проінформувати пацієнта про ознаки защемлення і відпустити додому Inform the patient about signs of entrapment and let him go home

Направити на КТ-дослідження черевної порожнини та малого тазу Send for CT scan of abdomen and pelvis

Накласти бандаж Apply bandage

954 / 1500
У дитини 10-ти років, що проживає у великому промисловому центрі, відзначаються енцефалопатія, поліневрит, зміна психічного розвитку, зниження здатності до навчання. У крові: ретикулоцитоз, базофільна зернистість еритроцитів. Отруєн-ня якою речовиною можна припустити? A 10-year-old child living in a large industrial center has encephalopathy, polyneuritis, changes in mental development, reduced learning ability. In the blood: reticulocytosis, basophilic granularity of erythrocytes. Poisoning by what substance can be assumed?

Міддю Copper

Ртуттю Mercury

Залізом Iron

Свинцем Lead

Цинком Zinc

955 / 1500
У потерпілого опікова травма 15% поверхні тіла II-III ступеня. На 20-ту добу після травми у хворого спостерігається різке підвищення температури тіла, загальна слабкість, прискорене везикулярне дихання, загострення рис обличчя, артеріальний тиск - 90/50 мм рт.ст., пульс - 112/хв. Яке ускладнення можна припустити? The victim has a burn injury of 15% of the body surface, II-III degree. On the 20th day after the injury, the patient has a sharp increase in body temperature, general weakness, accelerated vesicular breathing, aggravation of facial features, blood pressure - 90/50 mm Hg, pulse - 112/min. What complications can be assumed?

Гнійний бронхіт Purulent bronchitis

Гостра інтоксикація Acute intoxication

Пневмонія Pneumonia

Сепсис Sepsis

Анаеробна інфекція Anaerobic infection

956 / 1500
До лікаря звернулася жінка 45 років зі скаргами на загальну слабкість, підвищену втомлюваність, роздратованість, занепокоєння, зниження пам'яті, багаторазовий проніс, лущення та пігментацію відкритих ділянок тіла (шиї, кистей та підошв). Недостатність якого вітаміну може бути причиною такого стану паціє-нта? A 45-year-old woman came to the doctor with complaints of general weakness, increased fatigue, irritability, anxiety, memory loss, repeated runny nose, peeling and pigmentation of open areas of the body (neck, hands and soles). What vitamin deficiency can be the cause of this condition of the patient?

Фолієвої кислоти Folic acid

Рибофлавіну Riboflavin

Нікотинової кислоти Nicotinic acid

Тіаміну Thiamine

Ретинолу Retinol

957 / 1500
У хлопчика 14-ти років бронхіальна астма середньої тяжкості, період загострення. Який препарат необхідно призначити для зняття гострого нападу експі-раторної задишки? A 14-year-old boy has bronchial asthma of moderate severity, a period of exacerbation. What drug should be prescribed to relieve an acute attack of expiratory shortness of breath?

Сальбутамол Salbutamol

Дексаметазон Dexamethasone

Кромолін Na Cromolin Na

Строфантин (серцевий глікозид) Strophantin (cardiac glycoside)

Лазолван Lazolvan

958 / 1500
Пацієнта 28 років госпіталізовано на 9-й день хвороби зі скаргами на підвищення температури до 39^oC, головний біль, загальну слабкість, закреп, порушення сну. Об'єктивно спостерігається: на шкірі живота - поодинокі розеоли, Ps - 78/хв, печінка збільшена на 2 см. Поставте діагноз. A 28-year-old patient was hospitalized on the 9th day of the illness with complaints of an increase in temperature up to 39^oC, headache, general weakness, constipation, sleep disturbances. About' objectively observed: on the skin of the abdomen - isolated roseolae, Ps - 78/min, the liver is enlarged by 2 cm. Make a diagnosis.

Сепсис Sepsis

Висипний тиф

Черевний тиф Typhoid

Лептоспіроз Leptospirosis

Бруцельоз Brucellosis

959 / 1500
У п'ятирічної дитини діагностовано саркому Юїнга з ураженням лівої стегнової кістки та множинними метастазами в легені. Яке лікування доцільно застосувати у цьому разі? A five-year-old child was diagnosed with Ewing's sarcoma with damage to the left femur and multiple lung metastases. What treatment should be used in this case?

Дистанційна гаматерапія Remote gamma therapy

Комбіноване (променеве та хірургічне) лікування Combined (radiation and surgery) treatment

Ампутація ураженої кінцівки Amputation of the affected limb

Поліхіміотерапія Polychemotherapy

Антибіотикотерапія Antibiotic therapy

960 / 1500
Чоловік віком 35 років скаржиться на періодичні болі в епігастрії в нічний час. Діагностовано виразку цибулини дванадцятипалої кишки. Який неінвазивний метод діагностики інфекції Helicobacter pylori, що володіє найбільшою чутливістю і специфічністю, необхідно призначити пацієнту насамперед? A 35-year-old man complains of periodic pain in the epigastrium at night. A duodenal bulb ulcer is diagnosed. What non-invasive method of diagnosing Helicobacter pylori infection has the highest sensitivity and specificity, must be prescribed to the patient first?

Гастроскопія з біопсією Gastroscopy with biopsy

Визначення IgG, IgM у сироватці крові Determination of IgG, IgM in blood serum

Визначення фекального антигену Definition of fecal antigen

13С-сечовинний дихальний тест 13C-urea breath test

Визначення IgА, IgM у слині Determination of IgA, IgM in saliva

961 / 1500
Жінку віком 20 років шпиталізовано до приймального відділення зі скаргами на підвищення температури тіла до 38,5^oС, пітливість, біль в ділянці попереку та живота справа, головний біль і загальну слабкість. Об'єктивно спостерігається: позитивний симптом Пастернацького справа. У загальному аналізі крові визначається: гемоглобін - 115 г/л, еритроцити - 3,9·10^12/л, лейкоцити - 15,2·10^9/л, ШОЕ - 28 мм/год. У загальному аналізі сечі визначається: колір сечі - світло жовтий, питома вага - 1,018, білок - 0,42 г/л, лейкоцити - 15-20 в полі зору, циліндри - 3-5 в полі зору, бактерії - <<++>>. Який попередній діагноз є найімовірнішим? A 20-year-old woman was hospitalized to the reception department with complaints of an increase in body temperature up to 38.5°C, sweating, pain in the lower back and right abdomen, headache and general weakness. Objectively observed: Pasternak's symptom is determined in the general blood analysis: hemoglobin - 115 g/l, erythrocytes - 3.9·10^12/l, leukocytes - 15.2·10^9/l. , ESR - 28 mm/h. In the general analysis of urine, the color of urine is light yellow, specific gravity - 1.018, protein - 0.42 g/l, leukocytes - 15-20 in the field of vision, cylinders - 3-5 in the field vision, bacteria - <<++>>. What is the most likely preliminary diagnosis?

Гострий панкреатит Acute pancreatitis

Гострий аднексит Acute adnexitis

Гострий холецистит Acute cholecystitis

Гострий апендицит Acute appendicitis

Гострий пієлонефрит Acute pyelonephritis

962 / 1500
Хвора скаржиться на підвищення температури тіла до 38^oC протягом 2-х діб. Об'єктивно: позитивний симптом постукування в поперековій ділянці зліва. Загальний аналіз сечі: питома вага - 1015, білок - 0,99 г/л, еритроцити - 6-8 в п/з, лейкоцити - 30-32 в п/з. Який попередній діагноз? The patient complains of an increase in body temperature to 38^oC for 2 days. Objectively: a positive symptom of tapping in the lumbar region on the left. General analysis of urine: specific weight - 1015, protein - 0.99 g/l, erythrocytes - 6-8 in p/z, leukocytes - 30-32 in p/z. What is the previous diagnosis?

Гострий пієлонефрит Acute pyelonephritis

Гострий цистит Acute cystitis

Амілоїдоз нирок Kidney amyloidosis

Гострий гломерулонефрит Acute glomerulonephritis

Сечокам'яна хвороба Urolithiasis

963 / 1500
Пацієнтку віком 25 років шпиталізовано до гінекологічного відділення зі скаргами на підвищення температури до 38,5^oC, біль унизу живота, гнійні виділення з піхви. Захворіла гостро за тиждень після штучного аборту. Об'єктивно спостерігається: пульс - 100/хв, АТ - 110/70 мм рт. ст., живіт м'який, болючий у нижніх відділах. Під час гінекологічного обстеження виявлено: матка збільшена у розмірах, м'яка, болюча, придатки без змін, склепіння вагіни вільні. Виділення з піхви значні, гноєподібні. Який діагноз найімовірніший? A 25-year-old patient was hospitalized in the gynecology department with complaints of an increase in temperature to 38.5^oC, pain in the lower abdomen, purulent discharge from the vagina. She became acutely ill within a week after an artificial abortion, the pulse is 100/min, the abdomen is soft, painful in the lower parts. During the gynecological examination, the uterus is enlarged , painful, the appendages are unchanged, the vaginal vault is free. Vaginal discharge is large, pus-like. What is the most likely diagnosis?

Гострий метроендометрит Acute metroendometritis

Параметрит Parameter

Гострий аднексит Acute adnexitis

Лохіометра Lochiometer

Пельвіоперитоніт Pelvioperitonitis

964 / 1500
Хворого 28-ми років доставлено через 2 години після автодорожньої катастрофи в тяжкому стані зі скаргами на біль в животі. Отримав удар рулем у живіт. В ході об'єктивного обстеження вияв-лено: живіт не бере участі в акті дихання, напружений, під час пальпації різко болісний, спостерігається захисне напруження м'язів живота, позитивні симптоми подразнення очеревини, спостерігається зникнення печінкової тупості. Артеріальний тиск - 90/60 мм рт.ст., пульс - 120/хв. Якою буде подальша тактика лікування? A 28-year-old patient was brought 2 hours after a road accident in serious condition with complaints of stomach pain. He was hit by a steering wheel in the stomach. During the objective the examination revealed: the abdomen does not take part in the act of breathing, it is tense, during palpation it is sharply painful, there is a protective tension of the abdominal muscles, positive symptoms of peritoneal irritation, the disappearance of hepatic dullness is observed. Blood pressure - 90/60 mm Hg. , pulse - 120/min. What will be the further treatment tactics?

Ультразвукове дослідження Ultrasound examination

Проведення лапароцентезу Laparocentesis

Лапаротомія Laparotomy

Холод на живіт Stomach cold

Лапароскопія Laparoscopy

965 / 1500
Працівник 39 років звернувся до лікаря зі скаргами на ниючий біль та відчуття оніміння у кистях та передпліччях, зниження м'язової сили рук, порушення сну, роздратованість та зниження слуху. Дані професійного анамнезу свідчать про те, що робітник протягом 12 років працює бурильником, використовуючи свердло вагою 20 кг. Об'єктивно встановлено: шкіра кистей має синюшний відтінок, відмічається набряк кінчиків пальців, стертість шкірного малюнка, легка деформація міжфалангових суглобів, зниження тактильної, температурної та больової чутливості. Який найбільш імовірний діагноз? A 39-year-old employee went to the doctor with complaints of aching pain and numbness in the hands and forearms, decreased arm muscle strength, sleep disturbances, irritability, and hearing loss Data from the professional history indicate that the worker has been working as a driller for 12 years, using a drill weighing 20 kg. temperature and pain sensitivity. What is the most likely diagnosis?

Вібраційна хвороба, зумовлена впливом загальної вібрації Vibration disease caused by exposure to general vibration

Нервово-циркуляторна дистонія Nervous circulatory dystonia

Хвороба Рейно

Ревматичний поліартрит Rheumatoid polyarthritis

Вібраційна хвороба, зумовлена впливом локальної вібрації Vibration disease caused by exposure to local vibration

966 / 1500
У трирічної дитини з моменту народження спостерігається затримка випорожнень. Кожні 3-4 дні мати робить дитині клізми. Хлопчик відстає від своїх однолітків у фізичному розвитку. Об'єктивно спостерігається: блідість шкірних покривів, здуття живота. Яке захворювання можна припустити у дитини? A three-year-old child has delayed defecation since birth. Every 3-4 days, the mother gives the child enemas. The boy lags behind his peers in physical development. It is objectively observed : paleness of the skin, bloating. What disease can be assumed in the child?

Перитоніт Peritonitis

Хвороба Гіршпрунга Hirschsprung's disease

Пухлина кишечника Intestinal tumor

Глистна інвазія Worm infestation

Копростаз Coprostasis

967 / 1500
Дитині віком 12 років встановлено діагноз «двобічна пневмонія мікоплазмової етіології, легкий перебіг захворювання». Який з наведених препаратів потрібно призначити для лікування? A 12-year-old child was diagnosed with 'bilateral pneumonia of mycoplasma etiology, mild course of the disease'. Which of the following drugs should be prescribed for treatment?

Амінопеніциліни Aminopenicillins

Протигрибкові Antifungal

Аміноглікозиди Aminoglycosides

Макроліди ІІ покоління Second generation macrolides

Цефалоспорини І покоління I generation cephalosporins

968 / 1500
У хворої, що страждає на атопічну форму бронхіальної астми, виявлено єдиний алерген до шерсті домашнього собаки +++. Забрано килими з дому, зроблено косметичний ремонт квартири, використовується кондиціонер повітря. Проте у хворої щоночі повторюються напади задухи, незважаючи на патогенетичне лікування. Яка лікувальна тактика, розрахована на довгострокову перспективу, потенційно допоможе хворій зменшити чутливість організму до алергену? A patient suffering from an atopic form of bronchial asthma was found to have a single allergen to the hair of a domestic dog +++. Carpets were taken from the house, cosmetic repairs were made to the apartment, used air conditioner. However, the patient has repeated attacks of suffocation every night, despite the pathogenetic treatment. Which treatment tactic, designed for the long term, will potentially help the patient to reduce the sensitivity of the body to the allergen?

Антигістамінна терапія Antihistamine therapy

Продовження попереднього лікування Continuation of previous treatment

Направлення на спелеотерапію Referral to speleotherapy

Специфічна гіпосенсибілізація Specific hyposensitization

Заняття ЛФК за Бутейко Physical exercise classes for Buteyko

969 / 1500
У 29-річної жінки протягом останніх місяців з'явилися скарги на біль у правій клубовій ділянці, проноси з домішками слизу і гною, біль у стегнових суглобах, періодичне підвищення температури тіла. Живіт під час пальпації м'який, чутливий в правій клубовій ділянці. В ході іригографії виявлено: рельєф слизової оболонки нагадує «бруківку», ілеоцекальний перехід звужений. Яке захворювання можна припустити? In recent months, a 29-year-old woman complained of pain in the right iliac region, diarrhea with impurities of mucus and pus, pain in the hip joints, periodic increase body temperature. Abdomen is soft during palpation, sensitive in the right iliac region. During irrigography, the topography of the mucosa resembles a 'cobblestone'. What disease can be assumed?

Псевдомембранозний ентероколіт Pseudomembranous enterocolitis

Хвороба Уїппла Whipple's disease

Хвороба Крона Crohn's disease

Глютенова ентеропатія Gluten enteropathy

Туберкульозний ілеотифліт Tuberculous ileotyphlitis

970 / 1500
У жінки 32 років за 6 годин після фіброфагогастроскопії підвищилася температура тіла до 39^oC, болі за грудиною, сухість у роті, напростає задишка. Під час обстеження виявлено позитивний симптом Романова-Герке та підшкірну емфізему в надключичному просторі зліва. Який попередній діагноз? 6 hours after fibrophagogastroscopy, a 32-year-old woman's body temperature rose to 39^oC, pains behind the sternum, dry mouth, shortness of breath. During the examination, a positive Romanov-Herke symptom and subcutaneous emphysema in the left supraclavicular space. What is the previous diagnosis?

Піопневмоторакс Pyopneumothorax

Гострий абсцесс легені Acute lung abscess

Емпієма плеври Empyema of the pleura

Гострий медіастиніт Acute mediastinitis

Пневмонія Pneumonia

971 / 1500
Дитині 2 років поставлений діагноз: правостороння нижньодольова полісегментарна пневмонія. Виберіть типові аускультативні дані, характерні для цього захворювання в періоді розпалу: A 2-year-old child is diagnosed with right-sided lower lobe polysegmental pneumonia. Select the typical auscultatory data characteristic of this disease in the acute period:

Жорстке дихання Hard breathing

Послаблене дихання, крепітація Weakened breathing, crepitation

Значно послаблене дихання, хрипи не вислуховуються Significantly weakened breathing, no wheezing

Жорстке дихання, дрібнопухирчасті вологі хрипи Hard breathing, small bubbling wet rales

Жорстке дихання, сухі свистячі хрипи Hard breathing, dry whistling wheezing

972 / 1500
У жінки 45-ти років після переохолодження гостро з'явився біль в надлобковій та поперековій ділянці, різі в кінці сечовипускання, хибні потяги до сечовипускання. Сеча каламутна, з домішками крові. Лікар припустив наявність інфекції сечових шляхів. Які лабораторні дані найбільш характерні для цього захворювання? A 45-year-old woman, after hypothermia, developed acute pain in the suprapubic and lumbar region, cutting at the end of urination, false urges to urinate. Urine is cloudy, with blood impurities. The doctor assumed the presence of a urinary tract infection. What laboratory data are most characteristic of this disease?

Підвищення рівню креатиніну та сечовини крові Increase in creatinine and blood urea

Протеїнурія більш ніж 3,0 г на добу Proteinuria more than 3.0 g per day

Макрогематурія Macrohematuria

Протеїнурія менш ніж 3,0 г на добу Proteinuria less than 3.0 g per day

Лейкоцитурія, макрогематурія Leukocyturia, macrohematuria

973 / 1500
У пацієнтки віком 29 років, яка скаржиться на безпліддя та порушення менструального циклу по типу олігоменореї, під час обстеження виявлено: зріст - 160 см, маса тіла - 91 кг, відзначається ріст волосся на обличчі, стегнах. Під час бімануального дослідження з обох сторін виявлено збільшені щільної консистенції яєчники, розмірами 5x6 см. Ці ж дані підтверджені під час УЗД. Яка причина скарг жінки? In a 29-year-old female patient who complains of infertility and irregular menstrual cycle type oligomenorrhea, during the examination it was found: height - 160 cm, body weight - 91 kg , the growth of hair on the face and thighs is noted. During the bimanual examination, enlarged ovaries, measuring 5x6 cm, were found. The same data were confirmed during the ultrasound. What is the reason for the woman's complaints?

Адреногенітальний синдром Adrenogenital syndrome

Передменструальний синдром Premenstrual syndrome

Синдром склерокистозних яєчників (Штейна-Левенталя) Syndrome of sclerocystic ovaries (Stein-Leventhal)

Андробластома яєчників Androblastoma of the ovaries

Хронічний двохсторонній аднексит Chronic bilateral adnexitis

974 / 1500
Під час дослідження свинини у 24 зрізах на компресоріумі виявлено 2 трихінели. Що потрібно зробити з м'ясом? During examination of pork, 2 trichinella were found in 24 cuts on the compressorium. What should be done with the meat?

Проварити дрібними шматками Cook in small pieces

Реалізувати через мережу громадського харчування Implement via catering

Технічно утилізувати Technically Dispose

Глибоко заморозити Deep Freeze

Переробити на ковбасні вироби Process into sausage products

975 / 1500
Хвора 30-ти років скаржиться на субфебрилітет впродовж 3-х тижнів, зниження апетиту та працездатності, пітливість (особливо в нічний час), нездужання. Об'єктивно: при перкусії легень зміни відсутні; при аускультації хрипи в проекції верхньої долі правої легені. Рентгенологічно в S2 сегменті визначається щільне вогнище середньої інтенсивності 6 мм в діаметрі. Встановіть попередній діагноз: A 30-year-old patient complains of low-grade fever for 3 weeks, decreased appetite and work capacity, sweating (especially at night), malaise. Objectively: on percussion of the lungs there are no changes; on auscultation of the upper lobe of the right lung, a dense focus of 6 mm in diameter is determined.

Негоспітальна пневмонія Community-acquired pneumonia

Вогнищевий туберкульоз легень Focal pulmonary tuberculosis

Периферичний рак легень Peripheral lung cancer

Метастатичний рак легень Metastatic lung cancer

Туберкульома легень Pulmonary tuberculosis

976 / 1500
Вагітна жінка на 38 тижні вагітності була госпіталізована в пологовий будинок зі скаргами на різкий біль по всьому животі, який виник годину тому, незначні кров'янисті виділення зі статевих шляхів. Пологова діяльність відсутня. Об'єктивно спостерігається: пацієнтка бліда, пульс - 100/хв., АТ - 100/60 мм рт.ст., серцебиття плода відсутнє. Під час пальпації виявлено: матка напружена, болюча. Встановіть діагноз. A pregnant woman at 38 weeks of pregnancy was hospitalized in the maternity ward with complaints of sharp pain in the entire abdomen, which occurred an hour ago, slight bloody discharge from the genital tract There is no labor activity. The patient is pale, pulse - 100/60 mm Hg. During palpation it is found that the uterus is tense.

Передчасне відшарування плаценти Premature placental abruption

Гостра ниркова недостатність Acute renal failure

Розрив матки Rupture of uterus

Інтранатальна загибель плода Intranatal fetal death

Передлежання плаценти Placenta previa

977 / 1500
Чоловік 50 років скаржиться на загальну слабкість запаморочення, інтенсивний біль у попереку, правому підребер'ї, періодичну втрату свідомості, які з'явились після порушення технологічного процессу на виробництві фарб. Шкіра землистого відтінку, склери іктеричні, акроціаноз. АТ - 100/60 мм рт. ст. Тони серця приглушені, печінка +3 см, край щільний. У крові виявлено: еритроцити - 2,0·10^12/л, Hb - 90 г/л, тільця Гейнца-Ерліха, лейкоцити - 5,6·10^9/л, ШОЕ - 15 мм/год. Метгемоглобін - 62%, загальний білірубін - 84,0 мкмоль/л, непрямий білірубін - 71,7 мкмоль/л. Який діагноз найімовірніший? A 50-year-old man complains of general weakness, dizziness, intense pain in the lower back, right hypochondrium, periodic loss of consciousness, which appeared after a violation of the technological process at the factory The skin is icteric, blood pressure is 100/60 mmHg, the liver is dense, erythrocytes are 2.0·12/l - 90 g/l, Heinz-Ehrlich bodies, leukocytes - 5.6·10^9/l, ESR - 15 mm/h, methemoglobin - 62%, total bilirubin - 84.0 μmol/l, indirect bilirubin - 71, 7 μmol/L. What is the most likely diagnosis?

Набута гемолітична анемія Acquired hemolytic anemia

Гостре отруєння аніліном Acute aniline poisoning

Хронічне отруєння ціанідами Chronic cyanide poisoning

Гострий вірусний гепатит Acute viral hepatitis

Гостре отруєння толуолом Acute toluene poisoning

978 / 1500
Чоловік 35 років три дні тому був прооперований із приводу гострого апендициту. Післяопераційний період проходив без ускладнень, спостерігається деяка подразливість, неспокійний сон. Увечері на третю добу після операції став неспокійним, метушливим, чув, як на вулиці співають дівчата. Відганяв від себе <<білих мух>>. Уважає, що він у себе вдома, у часі дезорієнтований. Соматично встановлено: гіпергідроз, АТ - 160/100 мм рт. ст., температура тіла - 37,4^oС, тахікардія. Зі слів родичів багато років зловживає спиртним. Діагностуйте психічний розлад пацієнта: A 35-year-old man was operated on for acute appendicitis three days ago. The postoperative period passed without complications, there is some irritability, restless sleep. In the evening on the third day after the operation, restless, heard girls singing on the street. He thinks he is disoriented at home. Hyperhidrosis - 160/100 mmHg. body temperature - 37.4°C, tachycardia. According to relatives, he has been abusing alcohol for many years. Diagnose the patient's mental disorder:

Гіпертоксична шизофренія Hypertoxic schizophrenia

Токсико-інфекційний психоз Toxic-infectious psychosis

Гострий шизоафективний психоз Acute schizoaffective psychosis

Синдром Ганзера Hanzer syndrome

Алкогольний делірій Alcoholic delirium

979 / 1500
Пацієнтка віком 27 років скаржиться на різкий біль внизу живота, запаморочення. З анамнезу відомо: остання менструація 2 тижні тому. Тест на ХГЧ - негативний. Об'єктивно спостерігається: шкірні покриви бліді, АТ - 80/60 мм рт. ст. Пульс - 92/хв. Живіт напружений, болючий більше справа в нижніх відділах. Під час піхвового дослідження виявлено: матка нормальних розмірів, придатки болючі під час пальпації, нависання заднього склепіння. Який найімовірніший діагноз? A 27-year-old patient complains of sharp pain in the lower abdomen, dizziness. It is known from the anamnesis: the last menstruation was 2 weeks ago. The hCG test is negative. Objectively observed : skin is pale, blood pressure is 80/60 mm Hg, the abdomen is tense, more painful in the lower parts. . What is the most likely diagnosis?

Гострий апендицит Acute appendicitis

Загострення хронічного правостороннього аднекситу Exacerbation of chronic right-sided adnexitis

Апоплексія яєчника Ovarian apoplexy

Перекрут ніжки кісти яєчника Ovarian cyst pedicle torsion

Позаматкова вагітність Ectopic pregnancy

980 / 1500
У новонародженого терміном ґестації 31 тиждень наростає млявість, м'язова гіпотонія та пригнічення свідомості. Аналіз ліквору встановлено: підвищена кількість еритроцитів, білка та підвищений вміст глюкози. Який найбільш вірогідний діагноз? A newborn with a gestational age of 31 weeks develops lethargy, muscle hypotonia, and depression of consciousness. Analysis of the cerebrospinal fluid revealed: an increased number of erythrocytes, protein, and an increased glucose content. What is the most probable diagnosis?

Сепсис Sepsis

Внутрішньочерепний крововилив Intracranial hemorrhage

Анемія Anemia

Внутрішньоутробна інфекції Intrauterine infection

Менінгіт Meningitis

981 / 1500
Жінка 38 років скаржиться на слабкість, підвищення температури тіла до 38,5^oС, кашель зі скудним мокротинням, утруднення вдиху. Захворіла після переохолодження 2 доби тому. Рентгенологічне дослідження визначило локальне затемнення у нижній частці правої легені. У крові виявлено: лейкоцити - 14 Г/л, ШОЕ - 44 мм/год, СРБ - 140 мг/л. Який найімовірніший діагноз? A 38-year-old woman complains of weakness, an increase in body temperature to 38.5°C, a cough with scanty sputum, difficulty breathing. She fell ill after hypothermia 2 days ago. X-ray the study revealed local darkening in the lower lobe of the right lung. The blood showed: leukocytes - 14 G/l, ESR - 44 mm/h, CRP - 140 mg/l. What is the most likely diagnosis?

Гострий бронхіт Acute bronchitis

Бронхіальна астма Bronchial asthma

Негоспітальна пневмонія Community-acquired pneumonia

Плеврит Pleuri

Госпітальна пневмонія Nosocomial pneumonia

982 / 1500
У двохрічного хлопчика, що хворіє на ГРВІ, на тлі підвищення температури тіла до 39,5^oС розвинувся напад генералізованих тоніко-клонічних судом із втратою свідомості, що тривав 3 хвилини. Після нападу дитина у свідомості, дещо сонлива. Під час неврологічного огляду патології не виявлено, нервово-психічний розвиток відповідає віку. Раніше подібних нападів, як і будь-яких порушень з боку ЦНС не було. Який найімовірніший варіант судомного синдрому має місце у дитини? A two-year-old boy suffering from SARS developed an attack of generalized tonic-clonic seizures with loss of consciousness against the background of an increase in body temperature to 39.5^oC 3 minutes. After the attack, the child is somewhat sleepy. During the neurological examination, the neuropsychological development corresponds to the age. There were no such attacks, as well as any disturbances from the central nervous system. What is the most likely variant of the convulsive syndrome does the child have?

Фебрильні судоми Febrile convulsions

Симптоматична епілепсія Symptomatic epilepsy

Психогенний неепілептичний напад Psychogenic non-epileptic seizure

Спазмофілія Spasmophilia

Типові абсанси Typical absences

983 / 1500
Пацієнтка віком 50 років скаржиться на гострий біль в епігастральній ділянці, блювання, різку загальну слабкість. Хворіє протягом 2 діб. Об'єктивно спостерігається: блідість шкіри, язик сухий. Пульс - 100/хв, слабкого наповнення, АТ - 110/70 мм рт. ст. Живіт під час пальпації м'який, помірно болючий в епігастрії та мезогастрії, симптомів подразнення очеревини немає. Перистальтика значно посилена у мезогастрії зліва. Під час рентгенологічного обстеження виявлено: чаші Клойбера зліва. Який найімовірніший діагноз? A 50-year-old patient complains of sharp pain in the epigastric area, vomiting, severe general weakness. She has been ill for 2 days. Objectively observed: pale skin, dry tongue Pulse - 100/min, blood pressure - 110/70 mmHg. Abdomen is moderately painful during palpation. Peristalsis is significantly increased in the left mesogastric area examination revealed: Kleuber's cup on the left. What is the most likely diagnosis?

Кишкова непрохідність Intestinal obstruction

Гострий холецистит Acute cholecystitis

Перфоративна виразка дванадцятипалої кишки Perforating duodenal ulcer

Харчове отруєння Food poisoning

Гострий панкреатит Acute pancreatitis

984 / 1500
Пацієнтка віком 44 роки скаржиться на постійний пекучий біль, почервоніння шкіри та набряк в ділянці лівої гомілки, загальне нездужання, озноб, підвищення температури тіла до 39^oС. Початок захворювання пов'язує з подряпиною шкіри лівої гомілки 2 дні тому. Об'єктивно спостерігається: в середній третині лівої гомілки гіперемія шкіри, яскрава, з чіткими межами, які підвищені над незміненою шкірою. Виявлено помірний набряк м'яких тканин, їхня болючість під час пальпації, в центрі гіперемованої шкіри - струп 2х0,2 см, що покриває поверхневу ранку. Яке ускладнення мікротравми лівої гомілки спостерігається у пацієнтки? A 44-year-old patient complains of constant burning pain, reddening of the skin and swelling in the area of ​​the left lower leg, general malaise, chills, an increase in body temperature up to 39°C. Start the disease is associated with a scratch on the left leg 2 days ago. Objectively observed: hyperemia of the skin, bright, with clear boundaries, which are elevated above the unchanged skin on palpation, in the center of the hyperemic skin - a 2x0.2 cm scab covering the superficial wound. What complication of the microtrauma of the left leg is observed in the patient?

Газова гангрена Gas gangrene

Флегмона Phlegmon

Бешиха Beshikha

Гострий тромбофлебіт глибоких вен Acute thrombophlebitis of deep veins

Гострий гнійний остеомієліт Acute purulent osteomyelitis

985 / 1500
У жінки 34-х років з попередньо нормальною менструальною функцією цикли стали нерегулярними, за даними тестів функціональної діагностики - ановуляторними. Молочні залози пальпаторно болячі, виділяється молоко (галакторея). Яке дослідження показано хворій в першу чергу? A 34-year-old woman with previously normal menstrual function has irregular cycles, according to functional diagnostic tests - anovulatory. The mammary glands are painful on palpation, milk is secreted (galactorrhoea) What study is shown to the patient in the first place?

Прогестеронова проба Progesterone test

Визначення рівня гонадотропінів Determining the level of gonadotropins

Ультразвукове дослідження органів малого тазу Ultrasound examination of pelvic organs

КТ головного мозку CT brain

Визначення рівня пролактину Prolactin level determination

986 / 1500
У неврологічне відділення клінічної лікарні госпіталізовано хворого зі скаргами на порушення ходи, вимушений сміх. Об'єктивно: маскоподібне обличчя, гіпертонус м'язів. Хворий тривалий час працював на різних промислових об'єктах, де реєструвались перевищення гігієнічних нормативів хімічних речовин. Який шкідливий виробничий чинник міг зумовити виникнення вказаної патології? A patient was admitted to the neurological department of a clinical hospital with complaints of gait disturbance, forced laughter. Objectively: a mask-like face, hypertonic muscles. The patient worked for a long time at at various industrial facilities, where exceeding the hygienic standards of chemical substances was registered. What harmful production factor could cause the occurrence of the indicated pathology?

Бензол Benzene

Ртуть Mercury

Кадмій Cadmium

Свинець Lead

Марганець Manganese

987 / 1500
Пацієнт віком 55 років звернувся до сімейного лікаря зі скаргами на малопродуктивний кашель, переважно вранці. З анамнезу відомо, що він палить упродовж 20 років. Останні 2-3 роки відмічає задуху під час фізичного навантаження. Які спірометричні показники, виявлені у пацієнта, підтвердять діагноз хронічного обструктивного захворювання легень? A 55-year-old patient came to the family doctor complaining of a non-productive cough, mainly in the morning. From the anamnesis, it is known that he has been smoking for 20 years. The last 2-3 years notes shortness of breath during physical exertion. Which spirometric parameters found in the patient will confirm the diagnosis of chronic obstructive pulmonary disease?

Зниження ЖЕЛ < 80% та зниження ОФВ1/ФЖЕЛ < 0,9 Reduction of LVEF < 80% and reduction of FEV1/FLV < 0.9

Зниження ОФВ1 < 80% та зниження ОФВ1/ФЖЕЛ < 0,7 FeV1 reduction < 80% and FEV1/FJEL reduction < 0.7

Збільшення ОФВ1 > 10% та/або < 150 мл після прийому бронхолітика Increase in FEV1 > 10% and/or < 150 ml after taking a bronchodilator

Збільшення ЖЕЛ та збільшення ОФВ1/ФЖЕЛ > 0,8 Increasing LVEF and increasing FEV1/FLV > 0.8

Збільшення ОФВ1 > 12% та/або leqle200 мл після прийому бронхолітика Increase in FEV1 > 12% and/or leqle200 ml after taking a bronchodilator

988 / 1500
У породіллі 22-х років після відходження вод з'явились безперервні, дуже болючі перейми. Об'єктивно: розміри таза 25-28-31-21 см, маса плода - 4200 г. Матка постійно в тонусі, контракційне кільце на рівні пупка. Нижній сегмент матки болючий. При піхвовому дослідженні: шийка матки відкрита повністю, плідного міхура немає, голівка плода виповнює термінальну лінію, сагітальний шов в прямому розмірі. Серцебиття плода - 136/хв. Яка тактика лікаря буде найбільш доречною? A 22-year-old woman in labor had continuous, very painful contractions after her water broke. Objectively: the dimensions of the pelvis are 25-28-31-21 cm, the weight of the fetus is 4200 g. The uterus is constantly in tone, the lower segment of the uterus is painful. The cervix is ​​​​completely open, the fetal head fills the terminal line, the fetal heart beat is straight. 136/min. Which tactic of the doctor will be most appropriate?

Зробити плодоруйнівну операцію Perform a destructive operation

Зробити комбінований поворот плода на ніжку, з наступною його екстракцією Make a combined rotation of the fetus on the leg, followed by its extraction

Виключити пологову діяльність та зробити кесарів розтин Exclude labor and perform a caesarean section

Накласти акушерські щипці Apply obstetric forceps

Провести вакуум-екстракцію плода Carry out vacuum extraction of the fetus

989 / 1500
Чоловік 60 років під час надходження до лікарні скаржиться на відчуття розпирання за грудиною, яке з'являється під час швидкого ходіння та фізичного навантаження. Біль триває приблизно 5 хвилин і минає самостійно в стані спокою. Об'єктивно спостерігається: пульс - 75/хв, АТ - 140/80 мм рт.ст. Межі серця без змін. Тони серця дещо ослаблені, ритмічні, чисті. На ЕКГ змін не виявлено. Який попередній діагноз? A 60-year-old man is admitted to the hospital complaining of a feeling of distension behind the sternum, which appears during brisk walking and physical exertion. The pain lasts for about 5 minutes and passes independently at rest: pulse - 140/80 mm Hg. Heart sounds are slightly weakened, no changes are detected on the ECG ?

Стенокардія напруги Tension Angina

Варіантна стенокардія Variant Angina

Вегетосудинна дистонія Vegetovascular dystonia

Міозит Myositis

Міжреберна невралгія Intercostal neuralgia

990 / 1500
У хворої 30-ти років припинилась менструація, а потім почала знижуватися гострота зору. Була виявлена первинна атрофія зорових нервів, бітемпоральна геміа-нопсія. Впродовж 2-х років хвора лікувалась окулістами амбулаторно та стаціонарно, але зір прогресивно падав. Консультація офтальмолога: гострота зору OD=0,02, OS= 0,03, виражена первинна атрофія зорових нервів. Який допоміжний метод обстеження може допомогти у постановці діагнозу? A 30-year-old patient stopped menstruating, and then her visual acuity began to decrease. Primary atrophy of the optic nerves, bitemporal hemianopsia was detected. Over the course of 2 years the patient was treated by ophthalmologists on an outpatient basis, but her vision was progressively declining. Ophthalmologist's consultation: visual acuity OD=0.03, pronounced primary optic nerve atrophy. What auxiliary method of examination can help in establishing the diagnosis?

Електроенцефалографія Electroencephalography

Пневмоенцефалографія Pneumoencephalography

Дослідження ліквору Liquor Research

Ехоенцефалографія Echoencephalography

Рентгенографія черепу X-ray skull

991 / 1500
Хворому 65-ти років з приводу застійної серцевої недостатності призначена фармакотерапія (фуросемід, спіронолактон, бісопролол, раміприл). Через 2 місяці звернувся до лікаря з приводу розвитку гінекомастії. Запідозрено побічну дію спіронолактону. На який з наведених препаратів можна його замінити у схемі лікування цього хворого? A 65-year-old patient was prescribed pharmacotherapy (furosemide, spironolactone, bisoprolol, ramipril) for congestive heart failure. After 2 months, he consulted a doctor about the development of gynecomastia. A side effect of spironolactone is suspected. Which of the following drugs can replace it in the treatment regimen of this patient?

Івабрадин Ivabradin

Триметазидин Trimetazidine

Еплеренон Eplerenone

Торасемід Torasemide

Ніфедипін Nifedipine

992 / 1500
Вагітна в терміні 11-12 тижнів доставлена до гінекологічного стаціонару з матковою кровотечею та переймоподібним болем унизу живота. Піхвове дослідження виявлено: піхва заповнена згортками крові, шийка матки розкрита на 2 см. У каналі визначається напружений плідний міхур. Матка збільшена до розмірів 11-12 тижнів вагітності, напружена. Виділення кров'янисті, дуже рясні. Яких заходів повинен вжити лікар? An 11-12-week pregnant woman was brought to a gynecological hospital with uterine bleeding and cramp-like pain in the lower abdomen. Vaginal examination revealed: the vagina was filled with blood clots, the cervix was open at 2 cm. A strained amniotic sac is detected. The uterus is enlarged to the size of 11-12 weeks of pregnancy. The discharge is very profuse. What measures should the doctor take?

Призначення прогестерону Prescription of progesterone

Проведення гемотрасфузії Hemotransfusion

Вишкрібання порожнини матки Uterine scraping

Проведення токолітичної терапії Carrying out tocolytic therapy

Консервативне спостереження Conservative observation

993 / 1500
З анамнезу відомо що у пацієнтки віком 70 років захворювання розпочалось близько півроку тому. Зі слів родичів у пацієнтки відмічалося різке погіршення пам'яті на поточні події, зокрема забувала дорогу додому, повертаючись з магазину, стала відчувати труднощі у виконанні повсякденних справ. Пам'ять на події минулого зберігалась. Спостерігалися прояви семантичної та амнестичної афазії. Періодично відмічається ехолалія. Настрій знижений, незначна тривожність. В анамнезі черепномозкова травма. Близько 15 років хворіє на цукровий діабет. Який найімовірніший діагноз? It is known from the anamnesis that a 70-year-old patient's illness began about six months ago. According to relatives, the patient's memory for current events was markedly deteriorating, in particular, she forgot the way When returning home, she began to experience difficulties in performing daily tasks diabetes. What is the most likely diagnosis?

Енцефаліт Encephalitis

Судинна деменція Vascular dementia

Пухлина головного мозку Brain tumor

Інволюційна депресія Involutional depression

Хвороба Альцгеймера Alzheimer's disease

994 / 1500
У породіллі 29 років на 3 добу після операції кесарського розтину з'явилась блювота, сильний біль внизу живота, потім по всьому животу, затримка стула та газів. Загальний стан важкий. Шкіряні покрови бліді, губи та язик сухі. Температура тіла - 39,4^oС. Живіт вздутий, симптоми подразнення очеревини позитивні у всіх відділах. Перистальтика кишківника послаблена. Діурез знижений. Матка велика, болюча під час пальпації, з неприємним запахом виділень із піхви. Про яку патологію можна думати? On the 3rd day after cesarean section, a 29-year-old woman in labor developed vomiting, severe pain in the lower abdomen, then throughout the abdomen, stool retention and gas. General condition The skin is pale, the body temperature is 39.4 °C, the peristalsis is weak, the uterus is painful during palpation from the vagina. What pathology can you think about?

Метроендометрит Metroendometritis

Панметрит Panmetrite

Інфекція післяопераційної рани Postoperative wound infection

Пельвіоперитоніт Pelvioperitonitis

Розповсюджений перитоніт Disseminated peritonitis

995 / 1500
У триденної доношеної дитини у аналізі крові спостерігається: рівень непрямого білірубіну - 345 мкмоль/л, погодинний приріст - 6,8 мкмоль/л. Стан дитини важкий. Об'єктивно спостерігається: зниження рефлексів, гіпотонія м'язів, тремор кінцівок. Кров дитини та матері несумісна за Rh- фактором. Який метод лікування найефективніший у цьому випадку? A three-day-old full-term baby has the following blood test: indirect bilirubin level - 345 μmol/l, hourly increase - 6.8 μmol/l. The child's condition is serious. About objectively observed: decreased reflexes, hypotonia of the limbs. The blood of the child and mother is incompatible according to the Rh factor. What is the most effective method of treatment in this case?

Прийом фенобарбіталу Reception of Phenobarbital

Фототерапія Phototherapy

Гемосорбція Hemosorption

Кортикостероїдна терапія Corticosteroid therapy

Замінне переливання крові Replacement blood transfusion

996 / 1500
У жінки 32 років спостерігаються ожиріння (переважно на плечах і тулубі), гірсутизм і порушення менструального циклу. На плечах, грудях, животі, стегнах наявні багряно-ціанотичні стрії, розтягнення шкіри. Яка причина появи стрій у цієї пацієнтки? A 32-year-old woman has obesity (mainly on the shoulders and trunk), hirsutism and irregular menstrual cycle. There are purple-cyanotic striae on the shoulders, chest, abdomen, thighs , stretching of the skin. What is the reason for the appearance of striae in this patient?

Катаболічна дія надлишку кортико-стероїдів Catabolic effect of excess corticosteroids

Гіперпродукція андрогенів Hyperproduction of androgens

Гіпоестрогенемія Hypoestrogenemia

Інсулінорезистентність Insulin resistance

Гіпергонадотропінемія Hypergonadotropinemia

997 / 1500
Під час хімічного аналізу води встановлено: підвищення концентрації азотовмістних солей, заліза і сульфатів. Який показник якості води є найінформативнішим та свідчить про свіже забруднення води органічними речовинами тваринного походження? During the chemical analysis of the water, the following was found: an increase in the concentration of nitrogen-containing salts, iron and sulfates. Which indicator of water quality is the most informative and indicates fresh contamination of water with organic substances of animal origin?

Fe Fe

NH_3 NH_3

NO_2 NO_2

SO_4 SO_4

NO_3 NO_3

998 / 1500
Здорова дівчина 16-ти років прийшла на профілактичний медичний огляд. Під час збору анамнезу вона повідомила, що у 3 роки перехворіла на вітряну віспу. Інших серйозних захворювань в анамнезі немає. Батьки відмов від щеплень не писали, вакцинувалася у дитячому садочку та школі відповідно до Національного календаря профілактичних щеплень. Фізикальне обстеження без відхилень. Призначення яких вакцин буде найбільш доречним на даний момент? A healthy 16-year-old girl came for a preventive medical examination. During the anamnesis, she reported that she contracted chicken pox at the age of 3. Other serious diseases in the anamnesis no. The parents did not write vaccinations, they were vaccinated in kindergarten and school in accordance with the National vaccination calendar. Which vaccines would be most appropriate at the moment?

КПК, АДП, ВПЛ KPC, ADP, VPL

Гепатит В, АаКДП Hepatitis B, AaKDP

АДП-М, ВПЛ ADP-M, VPL

ІПВ, АаКДП IPV, AaKDP

Гепатит В, АДП-М Hepatitis B, ADP-M

999 / 1500
Хвора 45-ти років госпіталізована зі скаргами на періодичний інтенсивний головний біль на тлі підвищення артеріа-льного тиску до 180/90 мм рт.ст., м'язову слабкість, часте сечовипускання (в тому числі вночі). З анамнезу відомо, що, незважаючи на комбінацію та дозування антигіпертензивних препаратів, артеріа-льну гіпертензію коригувати не вдається. Рівень калію сироватки крові - 2,0 ммоль/л, рівень натрію сироватки крові - 160,0 ммоль/л. В ході УЗД виявлено об'ємні утворення близько 1,0 см в діаметрі в обох наднирниках. Виконано селективний ендоваскулярний забір крові з наднирникових вен - визначається значне підвищення рівню кортизолу та альдостерону. Який діагноз у хворої? A 45-year-old patient was hospitalized with complaints of periodic intense headache against the background of an increase in blood pressure up to 180/90 mm Hg, muscle weakness, frequent urination (including at night). It is known from the anamnesis that, despite the combination and dosage of antihypertensive drugs, arterial hypertension cannot be corrected. The level of potassium in the blood serum is 2.0 mmol/l, the level of sodium in the blood serum is - 160.0 mmol/l. Volumetric formations of about 1.0 cm in diameter were detected in both adrenal glands. A selective endovascular blood sampling was performed - a significant increase in the level of cortisol and aldosterone was determined in the patient.

Андростерома Androsteroma

Альдостерома Aldosteroma

Хвороба Іценка-Кушинга Itsenko-Cushing disease

Синдром Іценка-Кушинга Itsenko-Cushing syndrome

Феохромоцитома Pheochromocytoma

1000 / 1500
Чоловіка 45-ти років доставлено до лікаря після повідомлення про випадок активного туберкульозу у його колеги з яким він працює в одному офісному приміщенні. Скарг не має. Після проведення туберкулінової проби розмір папули становить 14 мм. Даних про результат попередньої проби немає. Який з наступних кроків лікаря є найбільш доречним? A 45-year-old man was taken to the doctor after reporting a case of active tuberculosis in his colleague with whom he works in the same office space. He has no complaints. After performing a tuberculin the size of the papule is 14 mm. There is no data on the result of the previous test. Which of the following steps is the most appropriate?

Карантин на 3 місяці Quarantine for 3 months

Рентгенографія органів грудної клітки X-ray of chest organs

Повторна туберкулінова проба через 3 місяці Repeat tuberculin test after 3 months

Забір мокротиння для виявлення мікобактерій туберкульозу Sputum sampling to detect tuberculosis mycobacteria

Призначення ізоніазиду, рифампіцину та етамбутолу Prescription of isoniazid, rifampicin and ethambutol

1001 / 1500
У пацієнта діагностовано правобічний пневмоторакс та терміново показане дренування плевральної порожнини. Вкажіть місце проведення плевральної пункції. The patient was diagnosed with a right-sided pneumothorax and urgent drainage of the pleural cavity was indicated. Specify the place of pleural puncture.

У 2-му міжребір'ї по середньоключичній лінії In the 2nd intercostal space along the midclavicular line

У місці найбільшої тупості визначеної під час перкусії At the place of greatest dullness determined during percussion

У 6-му міжребір'ї по задній пахвовій лінії In the 6th intercostal space along the posterior axillary line

У проекції плеврального синуса In the pleural sinus projection

У 7-му міжребір'ї по лопатковій лінії In the 7th intercostal space along the scapular line

1002 / 1500
У жінки віком 68 років тиждень тому раптово з'явилися біль у лівій половині грудної клітки та задишка. Об'єктивно спостерігається: ціаноз, набряк шийних вен, пульс - 100/хв., АТ - 110/70 мм рт. ст., ЧД - 28/хв., печінка +4 см, ліва гомілка набрякла та різко болюча під час пальпації. Аускультативно виявлено: у легенях зліва, нижче лопатки, спостерігається притуплення легеневого звуку, наявні звучні вологі дрібнопухирцеві хрипи, межі серця розширені справа, акцент ІІ тону над легеневою артерією. На ЕКГ виявлено глибокі зубці S у відведеннях I, aVL та Q у відведеннях III, aVF, негативний зубець T y відведеннях III, aVF. Який діагноз найімовірніший? A 68-year-old woman suddenly developed pain in the left half of the chest and shortness of breath a week ago. Objectively observed: cyanosis, swelling of the neck veins, pulse - 100/min., blood pressure - 110/70 mmHg, blood pressure - 28/min., the left leg is swollen and sharply painful during palpation. Dullness is observed in the lungs on the left, below the scapula pulmonary sound, there are sonorous fine vesicular rales, the borders of the heart are widened on the right, the accent of the II tone over the pulmonary artery. Deep S waves in leads I, aVL and Q in leads III, aVF, negative T wave in leads III, aVF the most likely diagnosis?

Крупозна пневмонія Croup pneumonia

Плеврит Pleuri

Перикардит Pericarditis

Тромбоемболія гілок легеневої артерії Thromboembolism of the branches of the pulmonary artery

Інфаркт міокарда Myocardial infarction

1003 / 1500
Пацієнтка віком 40 років приймає <<Мерказоліл>> у дозі 50 мг/добу з приводу хвороби Грейвса. За 2 тижні від початку лікування відмічає підвищення температури тіла до 38,3^oС, біль у горлі та появу болючих виразок в роті. Загальний аналіз крові: еритроцити - 3,2 ·10^12 /л, Нb - 94 г/л, лейкоцити - 1,1 ·10^9 /л; ШОЕ - 26 мм/год. Яка ймовірна причина погіршення стану пацієнтки? A 40-year-old patient takes <> at a dose of 50 mg/day for Graves' disease. 2 weeks after the start of treatment, she notes an increase in body temperature to 38 ,3^oС, sore throat and the appearance of painful ulcers in the mouth. General blood analysis: erythrocytes - 3.2 ·10^12 /l, Hb - 94 g/l, leukocytes - 1.1 ·10^9 /l; ESR - 26 mm/h. What is the likely cause of the patient's deterioration?

Алергічна реакція на <<Мерказоліл>> Allergic reaction to <>

Розвиток агранулоцитозу Development of agranulocytosis

Тиреотоксичний криз Thyrotoxic crisis

Гостра респіраторна вірусна інфекція Acute respiratory viral infection

Афтозний стоматит Aphthous stomatitis

1004 / 1500
Чоловік 34-х років звернувся до лікаря зі скаргами на ранкову скутість, болі в поперековій ділянці, плечових суглобах, суглобах шиї, хребта, які посилюються при різких рухах. Захворювання почалося з болів у поперековій ділянці після переохолодження. При фізикальному обстеженні болючість при пальпації остистих відростків та здухвинно-крижового сполучення, сегментарна ригідність поперекової ділянки хребта, атрофія прилеглих м'язів. На рентгенограмі поперекового відділу хребта осифікація зв'язок і сухожиль, синдесмофіти, повне зрощення хребців. При лабораторному дослідженні в крові лейкоцити - 4,3·10^9/л, ШЗЕ- 37 мм/год. Який з наведених діагнозів є найбільш імовірним? A 34-year-old man consulted a doctor with complaints of morning stiffness, pain in the lumbar region, shoulder joints, neck joints, and spine, which worsen with sudden movements. The disease began with pain in the lumbar region after hypothermia. During the physical examination, pain during palpation of the spinous processes and the sacroiliac junction, segmental stiffness of the lumbar region, atrophy of the adjacent muscles. On the radiograph of the lumbar region, ossification of ligaments and tendons, complete fusion of the vertebrae. In the laboratory examination, leukocytes - 4.3·10^9/l, ЖЕ - 37 mm/h. Which of the following diagnoses is the most probable?

Анкілозуючий спондилоартрит Ankylosing spondylitis

Псоріатичний артрит Psoriatic arthritis

Реактивний артрит Reactive arthritis

Остеоартроз Osteoarthrosis

Ревматоїдний артрит Rheumatoid arthritis

1005 / 1500
Мати з дівчинкою 11-ти років звернулись в приймальне відділення лікарні зі скаргами на виражений біль в правій здухвинній ділянці. При обстеженні виявлені зміни в сечі: протеїнурія, лейкоцитурія, бактеріурія. Ультразвукове дослідження (УЗД) показало відсутність правої нирки в типовому місці. Яке дослідження найдоцільніше провести для уточнення діагнозу виявленого під час УЗД? A mother and an 11-year-old girl came to the hospital's reception department with complaints of severe pain in the right sinus area. During the examination, changes in the urine were detected: proteinuria, leukocyturia, bacteriuria. Ultrasound examination (ultrasound) showed the absence of the right kidney in a typical place. What is the most appropriate study to clarify the diagnosis found during ultrasound?

Мікційна цистографія Victory cystography

Оглядова урографія Review urography

Цистографія Cystography

Магнітно-резонансна томографія Magnetic resonance imaging

Ниркова артеріографія Renal arteriography

1006 / 1500
У жінки 53 років наступного дня після езофагоскопії з'явилась емфізема обох підключичних ділянок, болі під час ковтання за грудиною. Температура тіла підвищилась до 39^oС. Результат загального аналізу крові: лейкоцити - 18,8·10^9/л, П - 16%, ШОЕ - 24 мм/год. На оглядовій рентгенограмі органів грудної клітини спостерігається розширення тіні середостіння. Яке дослідження найдоцільніше виконати для уточнення діагнозу? The day after esophagoscopy, a 53-year-old woman developed emphysema of both subclavian areas, pain when swallowing behind the sternum. The body temperature rose to 39°C. The result of general blood analysis: leukocytes - 18.8·10^9/l, ESR - 24 mm/h. On the X-ray examination of the chest organs, an expansion of the mediastinal shadow is most appropriate to perform to clarify the diagnosis?

Пункцію плевральної порожнини Puncture of the pleural cavity

Рентгенографію органів грудної порожнини у прямій проєкції X-ray of chest cavity organs in direct projection

Рентгенографію стравоходу з контрастом X-ray of esophagus with contrast

Рентгенографію органів грудної порожнини у боковій проєкції X-ray of chest cavity organs in lateral projection

Фіброзофагоскопію Fibroesophagoscopy

1007 / 1500
Хвора 38-ми років, бухгалтер. На прийомі багато говорить, мова прискорена, швидко переключається з теми на тему, що робить розуміння мови хворої скрутним. Не може всидіти на стільці, жестикулює, несподівано починає співати, сміятися, декламувати вірші. Вважає себе ''найздорові-шою'', ''найщасливішою'', заявляє, що ''всі заздрять її зовнішності та чудовому голосу''. Зі слів родичів, останні 5 днів не спить вночі, відзначається підвищений апетит, гнівливість. 3 роки тому лікувалася у психіатра з приводу депресії. Який синдромальний діаг-ноз є найбільш імовірним? The patient is 38 years old, an accountant. At the appointment, she talks a lot, her speech is accelerated, she quickly switches from topic to topic, which makes it difficult to understand the patient's language. She cannot sit still on a chair, gestures, suddenly begins to sing, laugh, recite poems. She considers herself the 'healthiest', 'the happiest', declares that 'everyone envies her appearance and wonderful voice'. 5 days awake at night, there is increased appetite, anger. She was treated by a psychiatrist 3 years ago. What is the most likely syndromic diagnosis?

Деліріозний синдром Delirious syndrome

Маніакальний синдром Manic syndrome

Депресивний синдром Depressive syndrome

Синдром гіперактивності з дефіцитом уваги Attention deficit hyperactivity disorder

Шизофренія, параноїдна форма Schizophrenia, paranoid form

1008 / 1500
У восьмимісячної дитини відмічається тонічне напруження мімічних м'язів, карпопедальний спазм, ларингоспазм. Після цього розвинулися генералізовані клонічні судоми з втратою свідомості на декілька хвилин. Позитивні симптоми Хвостека, Труссо, Люста. Який невідкладний стан спостерігається у дитини? An eight-month-old child has tonic tension of facial muscles, carpopedal spasm, laryngospasm. After that, generalized clonic convulsions with loss of consciousness for several minutes developed. Positive Khvostek symptoms, Trousseau, Lusta. What emergency condition is observed in the child?

Афективно-респіраторні судоми Affective respiratory convulsions

Гіпокальціємічні (тетанічні) судоми Hypocalcemic (tetanic) convulsions

Фебрильні судоми Febrile convulsions

Задухо-ціанотичний напад Suffocation-cyanotic attack

Епілептичний напад Epileptic seizure

1009 / 1500
Чоловік 33 років госпіталізований до інфекційної лікарні на 7-й день захворювання зі скаргами на різку слабкість, високу температуру, біль у м'язах ніг і попереку, жовтяницю, темний колір сечі, головний біль. Захворів гостро з ознобу, підвищення температури тіла до 40^oC, головний біль, біль у литкових м'язах та попереку. На 4-й день хвороби з'явилася жовтяниця, на 5-й - носова кровотеча, крововилив у склери. Тривалість гарячки 6 днів. Діурез - 200 мл. Який найімовірніший діагноз? A 33-year-old man was hospitalized in an infectious disease hospital on the 7th day of the disease with complaints of sharp weakness, high temperature, pain in the muscles of the legs and lower back, jaundice, dark color of urine, headache. He fell ill with chills, body temperature up to 40°C, headache, pain in the calf muscles and lower back. On the 4th day of the illness, jaundice appeared, on the 5th - nosebleeds , hemorrhage in the sclera. Duration of fever - 200 ml. What is the most likely diagnosis?

Лептоспіроз Leptospirosis

Черевний тиф Typhoid

Вірусний гепатит А Viral hepatitis A

Сепсис Sepsis

Ієрсиніоз Yersiniosis

1010 / 1500
Жінка 32-х років, вчителька молодших класів, проходить обов'язковий профілактичний медичний огляд. Відомо, що у регіоні проживання спалах кору. Жінка скарг не має. За даними карти профілактичних щеплень порушень календаря немає, у дитинстві отримала усі необхідні щеплення. Останнє щеплення від дифтерії та правця у 26 років. Останній ПАП-тест зроблено у 30 років, без відхилень. При фізикальному обстеженні температура тіла - 37,2^oC, пульс - 80/хв., артеріальний тиск - 130/80 мм рт.ст. Що з перерахованого є найбільш доречною тактикою лікаря відносно цієї пацієнтки? A 32-year-old woman, a primary school teacher, undergoes a mandatory preventive medical examination. It is known that there is an outbreak of measles in the region of residence. The woman has no complaints. according to the preventive vaccination card, she received all the necessary vaccinations against diphtheria at the age of 26. The last Pap test was done at the age of 30. During the physical examination, the body temperature was 37.2°C - 80/min., blood pressure - 130/80 mm Hg. Which of the following is the most appropriate tactic for the doctor regarding this patient?

Ввести вакцину АДС-М Enter ADS-M vaccine

Направити на мамографію Send for mammography

Зробити тест на вірус папіломи людини (ВПЛ-тест) Do a human papillomavirus test (HPV test)

Вакцинації та скринінгу не потребує Does not require vaccination and screening

Визначити рівень IgG до вірусу кору Determine the level of IgG to the measles virus

1011 / 1500
Хлопчика 7-ми років, привели до лікаря зі скаргами на виражений тотальний ціаноз з фіолетовим відтінком, задишку, яка полегшується в положенні навпочіпки з притискуванням до живота колін. При огляді пальці у вигляді ''барабанних паличок'' та нігті як ''годинникові скельця'', аускультативно дуючий шум над легеневою арте-рією. При інструментальному дослідженні на електрокардіограмі вертикальна вісь серця. На ехокардіографії стеноз отвору легеневої артерії, гіпертрофія стінок правого шлуночка, дефект міжшлуночкової перегородки, декстрапозиція аорти. Який діагноз є найбільш імовірним? A 7-year-old boy was brought to the doctor with complaints of pronounced total cyanosis with a purple tint, shortness of breath, which is relieved in a squatting position with knees pressed to the abdomen. examination of the fingers in the form of 'drumsticks' and nails as 'watch glasses', auscultatory blowing noise over the pulmonary artery. During the instrumental examination on the electrocardiogram, stenosis of the pulmonary artery opening, hypertrophy of the right ventricle, ventricular septal defect, aortic dextroposition. What is the most likely diagnosis?

Транспозиція магістральних судин Transposition of trunk vessels

Дефект міжшлуночкової перегородки Ventricular septal defect

Стеноз отвору легеневої артерії Stenosis of the opening of the pulmonary artery

Відкрита артеріальна протока Open ductus arteriosus

Тетрада Фалло Tetrad of Fallot

1012 / 1500
Працівник, не забезпечений засобами індивідуального захисту органів зору, перебуває в умовах інтенсивного інфрачервоного випромінювання Який негативний наслідок для здоров'я працівника найбільш ймовірно може розвинутись в таких умовах? The worker, who is not equipped with means of individual eye protection, is in conditions of intense infrared radiation. What negative consequence for the worker's health is most likely to develop in such conditions?

Гіперметропія Hyperopia

Катаракта Cataract

Астигматизм Astigmatism

Міопія Myopia

Глаукома Glaucoma

1013 / 1500
Жінка 46 років звернулась до сімейного лікаря зі скаргами на біль та здуття живота після прийому їжі, яка містить молочні продукти, проноси, які з'являються після вживання цільного молока. Об'єктивно встановлено: живіт здутий, чутливий під час пальпації. Печінка, селезінка не збільшені. Який метод діагностики допоможе виявити причину стану? A 46-year-old woman consulted a family doctor with complaints of abdominal pain and bloating after eating a meal containing dairy products, diarrhea that appears after consuming whole milk Objectively established: the abdomen is swollen, sensitive during palpation. The liver, spleen is not enlarged. What diagnostic method will help to identify the cause of the condition?

Дихальний тест Breath test

Антигліадинові антитіла Antigliadin antibodies

УЗД органів черевної порожнини Ultrasound of abdominal organs

Фіброколоноскопія Fibrocolonoscopy

Копрограма Coprogram

1014 / 1500
У хворої 33-х років після нападу епілептичних судом виникли задуха, сухий кашель та підвищення температури тіла. При аускультації легень справа знизу дихання відсутнє, перкуторно - притуплення звуку. Огляд ротової порожнини виявив відсутність одного різця нижньої щелепи. Який діагностичний метод найбільш імовірно дозволить лікарю визначити причину дихальних розладів пацієнтки? A 33-year-old patient developed shortness of breath, a dry cough, and an increase in body temperature after an attack of epileptic convulsions. When auscultating the lungs from the lower right, there is no breathing, percussive sound dulling. Examination of the oral cavity revealed the absence of one incisor of the lower jaw. Which diagnostic method is most likely to allow the doctor to determine the cause of the patient's respiratory disorders?

Пряма ларингоскопія Direct laryngoscopy

- -

Бронхоскопія Bronchoscopy

Фіброезофагогастроскопія Fibroesophagogastroscopy

Непряма ларингоскопія Indirect laryngoscopy

1015 / 1500
Жінка віком 67 років звернулася до лікаря зі скаргами на новоутворення у верхньому відділі лівої молочної залози, яке вона виявила випадково. Пацієнтка направлена на маммографію і УЗД молочних залоз. Під час маммографії у верхньо-зовнішньому квадранті залози виявлено утворення високої щільності з тяжистими контурами і поліморфними мікрокальцинатами. Під час УЗД виявлено: гіперехогенне утворення вертикальної орієнтації з нерівними нечіткими контурами і аксилярний лімфатичний вузол розміром 2,5 см із порушенням структури. Встановіть діагноз. A 67-year-old woman turned to the doctor with complaints about a neoplasm in the upper part of the left mammary gland, which she discovered by accident. The patient was sent for mammography and ultrasound of the mammary glands. Under During mammography, a high-density formation with heavy contours and polymorphic microcalcifications was detected. During ultrasound, a hyperechoic formation with uneven, indistinct contours and a 2.5 cm axillary lymph node with a structural violation were detected.

Злоякісна пухлина з метастатичним лімфовузлом Malignant tumor with metastatic lymph node

Фіброаденома із звапненням Fibroadenoma with calcification

Олеогранульома Oleogranuloma

Доброякісна пухлина з ураженням лімфовузла Benign tumor with lymph node involvement

Гамартома з ураженням лімфовузла Hamartoma with lymph node involvement

1016 / 1500
Дитячий дошкільний заклад розміщений поблизу автомагістралі, вміст монооксиду вуглецю в повітрі якої перевищує ГДК у 3-4 рази. Виявлення якої речовини у крові дітей підтвердить шкідливий влив забрудненого повітря? The children's preschool is located near the highway, the content of carbon monoxide in the air of which exceeds the permissible limit by 3-4 times. The detection of which substance in the children's blood will confirm the harmful influence of polluted air?

Гемоглобіну Hemoglobin

Метгемоглобіну Methemoglobin

Карбоксигемоглобіну Carboxyhemoglobin

Карбгемоглобіну Carbhemoglobin

Редукованого гемоглобіну Reduced hemoglobin

1017 / 1500
Жінка 62 рокiв захворiла гостро з пiдвищенням температури до 39,8^oC, ознобом, iнтенсивним головним болем, ломотою в тiлi. Скарги на нудоту, одноразове блювання. За 18 годин вiдзначила збiльшення i рiзку болючiсть пахвинних лiмфовузлiв справа. Ще за 6 годин з'явились набряк правої гомiлки i гiперемiя шкiри із чiткими нерiвними контурами. Який діагноз найімовірніший? A 62-year-old woman became acutely ill with an increase in temperature to 39.8^oC, chills, intense headache, body aches. Complaints of nausea, one-time vomiting. For 18 hours later, she noticed an increase in the inguinal lymph nodes on the right side. In another 6 hours, swelling of the right lower leg and skin hyperemia with clear uneven contours appeared. What is the most likely diagnosis?

Сибiрка, едематозна форма Anthrax, edematous form

Чума, бубонна форма Plague, bubonic form

Бешиха, еритематозна форма Rapid, erythematous form

Флегмона правої гомiлки Phlegmon of the right shin

Туляремія, бубонна форма Tularemia, bubonic form

1018 / 1500
У дитини 3 років під час обстеження виявлено: висока температура, інтоксикація, блідість шкіри, геморагічні та некротичні елементи висипу на шкірі та слизових оболонках. У загальному аналізі крові виявлено: панцитопенія, агранулоцитоз, ретикулоцити відсутні, ШОЕ значно збільшена. Який імовірний діагноз? A 3-year-old child during the examination revealed: high temperature, intoxication, pale skin, hemorrhagic and necrotic elements of a rash on the skin and mucous membranes. A general blood test revealed : pancytopenia, agranulocytosis, reticulocytes are absent, ESR is significantly increased. What is the probable diagnosis?

Білково-дефіцитна анемія Protein deficiency anemia

Залізодефіцитна анемія Iron deficiency anemia

В_12-дефіцитна анемія B_12-deficiency anemia

Апластична анемія Aplastic anemia

Гемоглобінопатія Hemoglobinopathy

1019 / 1500
Чоловік віком 40 років скаржиться на різку слабкість, біль у м'язах та суглобах, підвищення температури до 38,6^oС, схуднення. Захворів гостро. З анамнезу відомо, що 6 місяців тому хворів на вірусний гепатит. Об'єктивно спостерігається: стан пацієнта середньої тяжкості, пульс - 80/хв, АТ - 175/95 мм рт.ст. На внутрішній поверхні передпліччя пальпуються болючі підшкірні вузли розміром до 1 см. На верхніх кінцівках відзначаються порушення чутливості за типом <<рукавичок>>. За результатами клінічного аналізу крові виявлено помірну анемію, лейкоцитоз, еозинофілію, прискорену ШОЕ. У клінічному аналізі сечі спостерігається помірна протеїнурія та мікрогематурія. Для якого захворювання характерні такі клініко-лабораторні показники? A 40-year-old man complains of sudden weakness, pain in muscles and joints, temperature rise to 38.6^oС, weight loss. He became acutely ill. From anamnesis it is known that he had viral hepatitis 6 months ago. Objectively, the patient's condition is moderate, pulse - 175/95 mm Hg. Painful subcutaneous nodules up to 1 cm in size are palpated on the inner surface of the forearm. On the upper extremities, sensitivity is noted according to the 'glove' type. According to the results of the clinical blood analysis, moderate anemia, eosinophilia, and accelerated ESR are observed. In the clinical analysis, there is a moderate proteinuria and microhematuria. For which clinical and laboratory indicators are characteristic?

Ревматизму Rheumatism

Системного червоного вовчака Systemic lupus erythematosus

Дерматоміазиту Dermatomyasitis

Геморагічного васкуліту Hemorrhagic vasculitis

Вузликового поліартеріїту Polyarteritis nodosa

1020 / 1500
Чоловік 35 років після переохолодження скаржиться на часте, малими порціями, болюче сечовипускання, підвищення температури тіла протягом двох діб до 38,6^oC, озноб. Під час пальцевого ректального обстеженя виявляється збільшена болюча простата. У крові виявлено: лейкоцитоз - 14,2·10^9/л. У сечі лейкоцитурія - 20-25 в п/з. Який діагноз імовірний? A 35-year-old man after hypothermia complains of frequent, small portions, painful urination, an increase in body temperature for two days to 38.6^oC, chills. During digital a rectal examination reveals an enlarged prostate. Leukocytosis is found in the blood - 14.2·10^9 in the urine. What is the probable diagnosis?

Гострий простатит Acute prostatitis

Гіперплазія простати Prostate hyperplasia

Пухлина сечового міхура Bladder tumor

Гострий цистит Acute cystitis

Гонорея Gonorrhea

1021 / 1500
Жінка 35 років поступила зі скаргами на болі внизу живота, які посилюються під час менструації і статевих актів та іррадіюють в піхву, мажучі кров'янисті виділення до та після менструації протягом 5 днів. Безпліддя протягом 9 років. Протизапальне лікування ефекту не дало. Під час бімануального дослідження виявлено: матка збільшена, щільна, болюча, гладка. Під час гістероскопії в ділянці дна матки видно темно-червоні утвори, з яких виділяється темна кров. Який діагноз відповідає цій клінічній картині? A 35-year-old woman came in with complaints of pain in the lower abdomen, which worsens during menstruation and sexual acts and radiates into the vagina, smearing bloody discharge before and after menstruation Infertility for 9 years. During the bimanual examination, the uterus is enlarged, painful, and dark red formations are visible at the bottom of the uterus does the diagnosis correspond to this clinical picture?

Рак матки Uterine cancer

Міома матки Uterine myoma

Внутрішній ендометріоз Internal endometriosis

Ендометрит Endometritis

Гіперплазія ендометрію Endometrial hyperplasia

1022 / 1500
Хворий 45-ти років доставлений машиною швидкої допомоги в ургентну клініку зі скаргами на біль в поперековій ділянці, що виникає несподівано, часте болюче сечовипускання, блювання. Об'єктивно: болючість в поперековій ділянці, позитивний симптом Пастернацького, біль при пальпації нирок та за ходом сечовода справа. В сечі: білок, свіжі еритроцити, лейкоцити. Який попередній діаг-ноз? A 45-year-old patient was taken by ambulance to an emergency clinic with complaints of sudden pain in the lumbar region, frequent painful urination, vomiting. Objectively : pain in the lumbar region, positive Pasternaksky's symptom, pain on palpation of the kidneys and the right ureter. In the urine: protein, fresh erythrocytes, leukocytes. What is the previous diagnosis?

Полікістоз нирок Polycystic kidneys

Гострий пієлонефрит Acute pyelonephritis

Гостра ниркова недостатність Acute renal failure

Сечокам'яна хвороба, ниркова колька Urolithiasis, renal colic

Гострий гломерулонефрит Acute glomerulonephritis

1023 / 1500
Під час огляду потерпілого після дорожньо-транспортної пригоди спостерігаються ціаноз, утруднене дихання. Стан його тяжкий, права половина грудної клітки відстає в акті дихання, міжреберні проміжки розширені справа, під час перкусії спостерігається коробковий звук, дихання під час аускультації не вислуховується. Який діагноз найімовірніший? During the examination of the victim after the traffic accident, cyanosis and difficulty breathing are observed. His condition is serious, the right half of the chest lags behind in the act of breathing, the intercostal spaces are widened on the right, a box sound is observed during percussion, breathing is not heard during auscultation. What is the most likely diagnosis?

Клапанний пневмоторакс Valvular pneumothorax

Тотальний гемоторакс справа Total hemothorax on the right

Пневмоперитонеум Pneumoperitoneum

Відкритий пневмоторакс Open pneumothorax

Гострий гнійний плеврит Acute purulent pleurisy

1024 / 1500
Лікар швидкої допомоги під час надання медичної допомоги особі, витягнутій із зашморга родичами, відзначив: відсутність пульсу на сонних артеріях, свідомості, самостійного дихання, корнеа-льних рефлексів та наявність трупних плям на спині і задній поверхні кінцівок. За якими ознаками можна констатувати настання смерті? The emergency physician, while providing medical assistance to a person who was pulled out of a coma by relatives, noted: the absence of a pulse on the carotid arteries, consciousness, independent breathing, corneal reflexes and the presence of cadaveric spots on the back and back surface of the limbs. What signs can be used to determine the onset of death?

Відсутність самостійного дихання Absence of independent breathing

Відсутність пульсу No pulse

Наявність трупних плям Presence of corpse spots

Відсутність корнеальних рефлексів Absence of corneal reflexes

Відсутність свідомості Lack of consciousness

1025 / 1500
Пацієнт хворіє на виразкову хворобу шлунку, не лікується, періодично турбує біль у епігастрії, відрижка кислим. Раптово з'явилась загальна слабкість, серцебиття, головокружіння та блювання 'кавовою гущею', пізніше з'явилась 'мелена'. Об'єктивно спостерігається: болючість в епігастрії, позитивний симптом Менделя. У аналізі крові виявлено: гемоглобін - 82 г/л, лейкоцити - 7,5·10^9, ШОЕ - 22 мм/год. Яке ускладнення виникло у пацієнта? The patient suffers from peptic ulcer disease, is not treated, is periodically disturbed by pain in the epigastrium, sour belching. Suddenly, general weakness, palpitations, dizziness and vomiting appeared 'thick', later appeared 'grind'. Objectively observed: pain in the epigastrium, positive Mendelian symptom: hemoglobin - 82 g/l, leukocytes - 7.5·10^9, ESR - 22 mm. /h. What complication did the patient have?

Емпієма плеври Empyema of the pleura

Портальна гіпертензія Portal hypertension

Кишкова непрохідність Intestinal obstruction

Малігнізація виразки Ulcer malignancy

Шлунково-кишкова кровотеча Gastrointestinal bleeding

1026 / 1500
У триденного новонародженого спостерігаються виражені явища інтоксикації та підвищена температура тіла. У ділянці нижньої третини лівого стегна є багряно-синюшна пляма з ознаками локального запалення і тенденцією до поширення. Поставте попередній діагноз: A three-day-old newborn has marked signs of intoxication and elevated body temperature. In the area of ​​the lower third of the left thigh, there is a purplish-bluish spot with signs of local inflammation and a tendency to spread. Put previous diagnosis:

Перелом стегнової кістки Fracture of femur

Гематогенний остеомієліт стегнової кістки Hematogenous osteomyelitis of femur

Сепсис Sepsis

Артрит лівого колінного суглоба Arthritis of the left knee joint

Флегмона стегна Phlegmon of the hip

1027 / 1500
У дитини 10-ти років, що була на вулиці у вітряну і морозну погоду, з'явився помірний біль і поколювання у пальцях рук і ніг. Хлопчик повернувся додому, батьки виявили побілілі кінчики пальців рук і ніг, втрату чутливості. Розпочато відігрівання уражених ділянок, знову з'явилася болючість і поколювання у пальцях. Бліде забарвлення перейшло у багряне, поколювання щезло, з'явилося легке свербіння і невелика набряклість пальців. Визначте ступінь відмороження у дитини: A 10-year-old child who was outside in windy and frosty weather developed moderate pain and tingling in his fingers and toes. The boy returned home , the parents found the tips of the fingers and toes to be white, the affected areas started to warm up, and the tingling in the fingers appeared again. The pale color changed to purple, and the tingling disappeared in a child:

Озноблення Enlightenment

Відмороження IV ступеня IV degree frostbite

Відмороження ІІ ступеня Defrosting of the 2nd degree

Відмороження ІІІ ступеня III degree frostbite

Відмороження І ступеня I degree frostbite

1028 / 1500
Хлопчик 9 років у тяжкому стані: температура тіла - 38-39^oC, носові кровотечі, біль у кістках. Об'єктивно спостерігається: різка блідість, геморагічний висип, виразково-некротичний стоматит. Збільшені всі групи лімфовуз-лів, печінка +5 см, селезінка +4 см. Яке дослідження є вирішальним для встановлення діагнозу? A 9-year-old boy is in serious condition: body temperature - 38-39^oC, nosebleeds, bone pain. Objectively observed: sharp pallor, hemorrhagic rash , ulcerative-necrotic stomatitis. All groups of lymph nodes are enlarged, the liver is +5 cm, the spleen is +4 cm. What research is decisive for establishing the diagnosis?

УЗД черевної порожнини Ultrasound of the abdominal cavity

Рентгенограма середостіння X-ray of the mediastinum

Імунологічний комплекс Immunology complex

Загальний аналіз крові General blood test

Мієлограма Myelogram

1029 / 1500
Чоловік 38 років скаржиться на періодичне утруднення ковтання як твердої, так і рідкої їжі протягом багатьох місяців. Іноді виникає сильний біль за грудниною, особливо після гарячого пиття. Відзначаються напади задухи в нічний час. Вагу не втрачав. Об'єктивно спостерігається: загальний стан задовільний, шкірні покриви звичайного забарвлення. У шлунково-кишковому тракті змін під час огляду не виявлено. На рентгенограмі органів грудної клітки - розширення стравоходу з рівнем рідини в ньому. Поставте діагноз. A 38-year-old man complains of intermittent difficulty in swallowing both solid and liquid food for many months. Sometimes there is severe pain behind the sternum, especially after hot drinks. There are attacks Suffocation at night. Objectively observed: the skin is of normal color. On the chest x-ray, there is an expansion of the esophagus diagnosis.

Рак стравоходу Esophageal cancer

Гастро-езофагальна рефлюксна хвороба Gastroesophageal reflux disease

Кандидоз стравоходу Esophageal candidiasis

Ахалазія стравоходу Esophageal achalasia

Міастенія Myasthenia

1030 / 1500
Пацієнтка віком 15 рокiв скаржиться на вiдсутнiсть менструацiї, періодичний біль у пiхвi. Під час огляду зовнiшнiх статевих органiв виявлено: оволосіння за жiночим типом, великi статевi губи розвинені нормально, прикривають малi, вхiд до пiхви прикритий перегородкою синюшно-багряного кольору, що випинається. Який найімовірніший дiагноз? A 15-year-old patient complains of the absence of menstruation, periodic pain in the vagina. During the examination of the external genitalia, it was found: female-type hair, the labia majora is normally developed, they cover the vagina, the entrance to the vagina is covered by a protruding bluish-purple septum. What is the most likely diagnosis?

Атрезiя дiвочої плiви Atresia of hymen

Генiтальний iнфантилiзм Genital infantilism

Аменорея невiдомої етiологiї Amenorrhea of ​​unknown etiology

Дисфункцiя яєчникiв Ovarian dysfunction

Ендометрiоз вульви Endometriosis of the vulva

1031 / 1500
Під час огляду померлого на місці ДТП чоловіка, лікар виявив на всій поверхні його спини та частково на бокових поверхнях тулуба багряно-фіолетові плями, які у разі натискання блідніють та поступово відновлюють своє забарвлення через 5 хвилин. Яке явище спостерігає лікар? During the examination of a man who died at the scene of a road accident, the doctor found purple-purple spots on the entire surface of his back and partially on the side surfaces of the body, which pale when pressed and gradually regain their color after 5 minutes. What phenomenon does the doctor observe?

Трупні плями Dead spots

Трупне заклякання Corpse Charm

Трупне висихання Corpse desiccation

Аутоліз Autolysis

Трупне охолодження Corporal cooling

1032 / 1500
У пацієнта 42-х років виник ''кинджальний'' біль в епігастральній ділянці, що thickspace поширився thickspace через thickspace кілька thickspace годин thickspace над thickspace всією поверхнею живота. Симптоми подразнення очеревини позитивні. Протягом багатьох років страждає на виразкову хворобу дванадцятипалої кишки. При рентгенографії живота: вільний газ під куполами діафрагми. Ваш імовірний діагноз: A 42-year-old patient developed a 'dagger' pain in the epigastric region that thickspace spread thickspace through thickspace for several thickspace hours thickspace over thickspace over the entire surface of the abdomen. Symptoms of peritoneal irritation have been suffering from duodenal ulcer for many years. Abdominal X-ray: free gas under the domes of the diaphragm:

Перфорація порожнистого органу. Перитоніт Perforation of a hollow organ. Peritonitis

Хронічний гастрит Chronic gastritis

Виразкова хвороба шлунка в стадії загострення Gastric ulcer disease in the acute stage

Гострий панкреатит Acute pancreatitis

Виразкова хвороба дванадцятипалої кишки Duodenal ulcer disease

1033 / 1500
У 3-місячної дитини на фоні субфебрильної температури тіла і риніту спостерігається блідість, ціаноз носогубного трикутника, виражена задишка експіраторного характеру, здута грудна клітка, сухий кашель, участь допоміжної мускулатури в диханні. Перкуторно встановлено: над легенями коробковий звук, під час аускультації на фоні подовженого видиху прослуховуються розсіяні сухі та дрібнопухірцеві вологі хрипи з обох боків. У крові виявлено: Hb - 112 г/л, еритроцити - 3,2·10^12/л, лейкоцити - 15,4·10^9/л, лімфоцити - 72%. Який попередній діагноз? A 3-month-old child has pallor, cyanosis of the nasolabial triangle, severe expiratory dyspnea, a swollen chest, dry cough, involvement of the auxiliary musculature in breathing. Percussion is established: over the lungs, during auscultation, scattered dry and small-vesicular rales are heard on both sides. In the blood, Hb - 112 g/l, erythrocytes - 3.2·10^12 /l, leukocytes - 15.4·10^9/l, lymphocytes - 72%. What is the previous diagnosis?

Гострий обструктивний бронхіт Acute obstructive bronchitis

Гострий (простий) бронхіт Acute (simple) bronchitis

Бронхіальна астма, приступний період Bronchial asthma, attack period

Двобічна пневмонія Bilateral pneumonia

Гострий бронхіоліт Acute bronchiolitis

1034 / 1500
У чоловіка віком 57 років на роботі з'явилися сильний головний біль з переважною локалізацією у потилиці, нудота, блювання, короткочасна непритомність. Об'єктивно спостерігається: ригідність потиличних м'язів, симптом Керніга з двох боків, загальна гіперестезія, розбіжна косоокість за рахунок лівого очного яблука. Ліквор геморагічний, тиск 300 мм вод. ст. Який найвірогідніший механізм захворювання? A 57-year-old man developed a severe headache at work with a predominant localization in the back of the head, nausea, vomiting, short-term fainting. Objectively observed: stiffness of the occipital muscles, Kernig's symptom on both sides, divergent strabismus due to the left eyeball, hemorrhagic fluid pressure of 300 mm of water. What is the most likely mechanism of the disease?

Паренхіматозний крововилив Parenchymal hemorrhage

Тромбоз мозкових судин Thrombosis of cerebral vessels

Крововилив у шлуночки мозку Hemorrhage in the ventricles of the brain

Спазм мозкових судин Spasm of cerebral vessels

Субарахноїдальний крововилив Subarachnoid hemorrhage

1035 / 1500
Чоловік віком 28 років звернувся до лікаря з приводу утворення на шкірі 4-го пальця правої кисті. Об'єктивно спостерігається: на тильній поверхні 4-ого пальця бородавчасті розростання, розмірами 3-8 мм, не болять, поверхня суха, м'яко-еластичної консистенції, спаяні з підлягаючими тканинами, запалення шкіри по периферії розростань немає. Яка патологія спостерігається у пацієнта? A 28-year-old man consulted a doctor about a growth on the skin of the 4th finger of the right hand. Objectively observed: warty growths on the back surface of the 4th finger , 3-8 mm in size, do not hurt, the surface is dry, soft-elastic consistency, welded to the underlying tissues, there is no inflammation of the skin on the periphery of the growths. What pathology is observed in the patient?

Професійний дерматит Occupational dermatitis

Заразливий молюск Mollus contagion

Звичайні бородавки Normal warts

Бородавчастий туберкульоз Warty tuberculosis

Вегетуюча піодермія Pyoderma vegetative

1036 / 1500
Жінка 32 років протягом 2 років відзначає періодичний нападоподібний біль у правому підребер'ї, який знімався но-шпою. Біль не завжди пов'язаний із вживанням їжі, іноді з'являється під час хвилювання, супроводжується болем у серці, серцебиттям. Об'єктивно спостерігається: емоційно лабільна, під час пальпації живота виникає невелика болючість у ділянці жовчного міхура. Яка найімовірніша патологія зумовлює таку картину? A 32-year-old woman for 2 years notes periodic attack-like pain in the right hypochondrium, which was relieved by a no-spa. The pain is not always associated with eating, sometimes appears during excitement, is accompanied by pain in the heart, palpitations. It is observed objectively: emotionally labile, during palpation of the abdomen there is a slight tenderness in the area of ​​the gallbladder. What is the most likely pathology that causes this picture?

Хронічний холангіт Chronic cholangitis

Дискінезія жовчовивідних шляхів Biliary tract dyskinesia

Дуоденіт Duodenitis

Хронічний холецистит Chronic cholecystitis

Хронічний панкреатит Chronic pancreatitis

1037 / 1500
У чоловіка 32 років спостарігається закрита черепно-мозкова травма, закрита травма грудної клітки, закритий перелом правого стегна. АТ - 100/60 мм рт. ст., Рs - 124/хв. ЧД - 28/хв. За 2 години після проведення скелетного витягування під місцевою анестезією лідокаїном відбулося різке погіршення стану: з'явився ціаноз обличчя і шиї, АТ - 60/40 мм рт. ст., ЧСС - 160/хв.,ЧД - 44/хв. Яке ускладнення, найімовірніше, трапилось? A 32-year-old man has a closed brain injury, a closed chest injury, a closed fracture of the right hip. Blood pressure - 100/60 mm Hg, Rs - 124/min BH - 28/min After carrying out skeletal extraction with local anesthesia, the condition worsened: cyanosis of the face and neck appeared, BP - 60/40 mm Hg. min., CH - 44/min. What complication most likely happened?

Інфаркт міокарду, кардіогенний шок Myocardial infarction, cardiogenic shock

Больовий шок Pain shock

Гостра постгеморагічна анемія Acute posthemorrhagic anemia

Жирова емболія легеневої артерії Fat embolism of the pulmonary artery

Тромбоемболія легеневої артерії Thromboembolism of the pulmonary artery

1038 / 1500
Чоловік 56 років хворіє на бронхоектатичну хворобу впродовж 18 років. Протягом останніх півроку спостерігаються виражені набряки гомілок, обличчя. У протеїнограмі зазначено: загальний білок - 52 г/л, альбумін-глобуліновий коефіцієнт - 0,7, alpha_2-глобуліни - 14%, beta-глобуліни - 17%, gamma-глобуліни - 24 г/л. ШОЕ - 52 мм/год. Добова протеїнурія - 4,5 г. ЕКГ: синусова тахікардія. Яке захворювання є найбільш імовірне? A 56-year-old man has been suffering from bronchiectasis for 18 years. Over the past six months, severe swelling of the legs and face has been observed. The proteinogram shows: total protein - 52 g/l, albumin-globulin ratio - 0.7, alpha_2-globulins - 17%, gamma-globulins - 24 g/l, daily proteinuria - 4.5 g tachycardia. What disease is most likely?

Туберкульоз нирок Kidney tuberculosis

Амілоїдоз нирок Kidney amyloidosis

Хронічний пієлонефрит Chronic pyelonephritis

Хронічне легеневе серце Chronic pulmonary heart

Хронічний гломерулонефрит Chronic glomerulonephritis

1039 / 1500
Хвора 60-ти років скаржиться на біль в міжфалангових суглобах кистей, який посилюється при роботі. Об'єктивно: дистальні та проксимальні суглоби II-IV пальців деформовані, з вузлами Гебердена, Бушара, болючі, з обмеженою рухомістю. Рентгенограма суглобів: суглобові щілини звужені, краєві остеофіти, субхондральний склероз. Який діагноз найбільш ймовірний? A 60-year-old patient complains of pain in the interphalangeal joints of the hands, which intensifies during work. Objectively: the distal and proximal joints of II-IV fingers are deformed, with Heberden's, Bouchard's nodes, painful, with limited mobility. X-ray of the joints: narrowed joint spaces, marginal osteophytes. What is the most likely diagnosis?

Деформуючий остеоартроз Deforming osteoarthritis

Псоріатичний артрит Psoriatic arthritis

Хвороба Бехтерєва Bekhterev's disease

Ревматичний артрит Rheumatic arthritis

Хвороба Рейтера Reiter's disease

1040 / 1500
Хлопчик 14 років скаржиться на втомлюваність, наявність набряків на обличчі, головний біль, малі порції сечі. Сеча кольору м'ясних помиїв. Перебуває на диспансерному обліку з приводу хронічного гломерулонефриту з 8 років. Креатинін крові - 0,350 ммоль/л, сечовина крові - 10,4 ммоль/л. Яке ускладнення, найімовірніше, зумовлює таку клінічну картину? A 14-year-old boy complains of fatigue, swelling on the face, headache, small portions of urine. The urine is the color of meat slops. He is in the clinic for chronic glomerulonephritis from the age of 8. Blood creatinine - 0.350 mmol/l, blood urea - 10.4 mmol/l. What complication most likely causes this clinical picture?

Хронічний гломерулонефрит Chronic glomerulonephritis

Туберкульоз нирок Kidney tuberculosis

Гостра ниркова недостатність Acute renal failure

Хронічна ниркова недостатність Chronic renal failure

Хронічна недостатність кровообігу Chronic circulatory failure

1041 / 1500
У хворого 24-х років, який перебував на лікуванні з приводу менінгококцемії, погіршився загальний стан. Об'єктивно: акроціаноз, кінцівки холодні, ЧД- 30/хв., ЧСС- 140/хв., АТ- 40/0 мм рт.ст., анурія. Який невідкладний стан розвинувся? The general condition of a 24-year-old patient who was being treated for meningococcemia worsened. Objectively: acrocyanosis, cold extremities, heart rate - 30/min ., heart rate - 140/min, blood pressure - 40/0 mmHg, anuria. What urgent condition has developed?

Септичний шок Septic shock

Анафілактичний шок Anaphylactic shock

Гіповолемічний шок Hypovolemic shock

Гемолітико-уремічний синдром Hemolytic uremic syndrome

Тромбоз легеневої артерії Thrombosis of the pulmonary artery

1042 / 1500
Чоловік 65 років звернувся зі скаргами на напади кашлю під час вживання рідкої їжі. Три місяці тому у пацієнта діагностовано рак верхньої третини стравоходу. Проходив променеву терапію. Яке ускладнення виникло у пацієнта? A 65-year-old man complained of coughing attacks while eating liquid food. Three months ago, the patient was diagnosed with cancer of the upper third of the esophagus. He underwent radiation therapy. What complications occurred in the patient?

Прорив абсцесу легені в плевральну порожнину Breakthrough of a lung abscess into the pleural cavity

Спонтанний пневмоторакс Spontaneous pneumothorax

Стеноз трахеї Stenosis of the trachea

Стравохідно-трахеальна нориця Esophago-tracheal fistula

Перфорація кардіальної виразки шлунку Perforation of cardiac gastric ulcer

1043 / 1500
До опікового відділення доставлено потерпілого чоловіка 28 років зі циркулярними опіками тулуба та нижніх кінцівок. Визначте розмір опікової поверхні у потерпілого: A 28-year-old male victim with circular burns of the trunk and lower limbs was brought to the burn department. Determine the size of the victim's burn surface:

20% 20%

45% 45%

30% 30%

72% 72%

36% 36%

1044 / 1500
У чоловіка віком 35 років, який хворіє на епілепсію, розвинувся великий судомний напад тривалістю 4 хвилини. За 10 хвилин після нападу свідомість не відновилася, виникли повторні тоніко-клонічні судоми. Який лікувальний засіб є препаратом вибору в цьому клінічному випадку? A 35-year-old man with epilepsy developed a grand convulsive seizure lasting 4 minutes. 10 minutes after the seizure, he did not regain consciousness, repeated tonic-clonic convulsions. What is the drug of choice in this clinical case?

Галоперидол Haloperidol

Діазепам Diazepam

Карбамазепін Carbamazepine

Амітриптилін Amitriptyline

Вальпроєва кислота Valproic acid

1045 / 1500
Пацієнт віком 76 років скаржиться на епізоди втрати пам`яті та запаморочення. З анамнезу відомо, що два роки тому він переніс гострий інфаркт міокарда. Об'єктивно спостерігається: ЧСС = пульс = 32/хв, АТ - 160/90 мм рт. ст. На ЕКГ виявлено: АВ-блокада ІІІ ступеня. Якою повинна бути подальша тактика лікаря? A 76-year-old patient complains of episodes of memory loss and dizziness. It is known from the anamnesis that he suffered an acute myocardial infarction two years ago. Objectively observed: Heart rate = 32/min, blood pressure - 160/90 mmHg. On the ECG, it was found that there is a third-degree AV block.

Імплантація електрокардіостимулятора Implantation of a pacemaker

Призначити пацієнту постійне приймання атропіну Appoint the patient to receive permanent atropine

Призначити пацієнту приймання beta-адреноблокаторів постійно Appoint the patient to take beta-blockers permanently

Призначити пацієнту постійне приймання ізадрину Assign the patient to constant reception of isadrin

Спостереження за пацієнтом один раз на місяць постійно Patient observation once a month continuously

1046 / 1500
Жінка 58-ми років з цукровим діабетом 2 типу, що компенсується дієтою та метформіном, готується до холецистектомії. Об'єктивно: зріст - 164 см, вага - 90 кг. ЧСС- 72/хв., АТ- 130/80 мм рт.ст. Живіт м'який, болючий у правому підребер'ї. Печінка не збільшена. Глюкоза натще - 6,2 ммоль/л. Глікований гемоглобін - 6,5%. Яка подальша тактика цукрознижувальної терапії? A 58-year-old woman with type 2 diabetes, compensated by diet and metformin, is preparing for cholecystectomy. Objectively: height - 164 cm, weight - 90 kg. Heart rate - 130/80 mm Hg. Abdomen is painful. Fasting glucose - 6.2 mmol/l. 5%. What are the further tactics of hypoglycemic therapy?

Призначення суміші інсулінів Prescription of insulin mixture

Призначення інсуліну короткої дії Prescription of short-acting insulin

Призначення глюренорму Appointment of glurenorm

Збереження схеми лікування Saving treatment plan

Призначення інсуліну подовженої дії Prescription of long-acting insulin

1047 / 1500
Пацієнтку віком 27 років із терміном вагітності 17 тижнів шпиталізовано до стаціонару для лікування. З анамнезу відомо: 2 самовільні викидні. Під час бімануального дослідження виявлено: матка збільшена до 17 тижнів вагітності, шийка матки вкорочена, вічко пропускає кінчик пальця. Встановлено діагноз: істміко-цервікальна недостатність. Вкажіть подальшу тактику ведення вагітної. A 27-year-old patient with a pregnancy of 17 weeks was hospitalized for treatment. From the anamnesis it is known: 2 spontaneous miscarriages. During bimanual examination it was found: the uterus was enlarged to 17 weeks of pregnancy, the cervix is ​​shortened, the eye passes the tip of the finger. The diagnosis is established: isthmic-cervical insufficiency. Indicate the further tactics of the pregnant woman.

Накласти шов на шийку матки Put a suture on the cervix

Токолітична терапія Tocolytic therapy

Переривання вагітності Termination of pregnancy

Провести гормональне лікування Conduct hormonal treatment

Зробити амніоцентез Do amniocentesis

1048 / 1500
Пацієнтка віком 27 років звернулася до лікаря зі скаргами на збільшення лімфовузлів на шиї справа і в пахвовій ділянці,нічну пітливість, підвищення температури тіла вище 38^oС. Під час морфологічного дослідження біоптату лімфатичного вузла виявлено: клітини Березовського-Штенберга. Який найімовірніший діагноз? A 27-year-old patient turned to the doctor with complaints of an increase in lymph nodes on the neck on the right and in the armpit, night sweats, an increase in body temperature above 38°C. During morphological examination of the lymph node biopsy revealed: Berezovsky-Shtenberg cells. What is the most likely diagnosis?

Злоякісна лімфома Malignant lymphoma

Туберкульоз лімфатичних вузлів Tuberculosis of lymph nodes

Лімфогранулематоз Lymphogranulomatosis

Хронічний лімфолейкоз Chronic lymphocytic leukemia

Метастази пухлини в лімфатичні вузли Tumor metastases in lymph nodes

1049 / 1500
Під час строкових пологів двійнею, після народження першої дитини масою 2800 г відійшли навколоплідні води. У процесі вагінального дослідження в піхві знайдено пульсуючу пуповину другого плоду. Голівка плода притиснута до термінальної лінії, легко відштовхується. Спроби заправити пуповину безуспішні. Серцебиття плода - 160/хв. Яка має бути тактика лікаря? During the term delivery of twins, after the birth of the first child weighing 2800 g, the amniotic fluid was removed. During the vaginal examination, a pulsating umbilical cord of the second fetus was found in the vagina. The fetal head was pressed against of the terminal line, it is easily pushed away. Attempts to tuck the umbilical cord are unsuccessful. The heart rate of the fetus is 160/min. What should be the doctor's tactics?

Лікувати гіпоксію плода і роди вести консервативно Treat fetal hypoxia and give birth conservatively

Операція кесарського розтину Caesarean section operation

Вакуумекстракція плода Vacuum extraction of the fetus

Комбінований зовнішньовнутрішній поворот плода на ніжку з подальшим його вилученням Combined external-internal rotation of the fetus on the leg with subsequent extraction

Накласти акушерські щипці Apply obstetric forceps

1050 / 1500
Пацієнт віком 35 років скаржиться на слабкість, підвищену пітливість, стомлюваність, біль в правому боці під час дихання, підвищену температуру тіла до 38^oС. Об'єктивно спостерігається: частота дихання - 28/хв, пульс - 100/хв. Права половина грудної клітки відстає під час акту дихання. Голосове тремтіння справа не проводиться, перкуторно - тупий тон, дихання ослаблене. Межі серця зміщені вліво. У загальному аналізі крові спостерігається: лейкоцити - 12·10^9/л, паличкоядерні - 13%, лімфоцити - 13%, ШОЕ - 38 мм/год. Який найімовірніший діагноз? A 35-year-old patient complains of weakness, increased sweating, fatigue, pain in the right side during breathing, increased body temperature up to 38°C. Objectively observed : respiratory rate - 100/min. The right half of the chest lags during the act of breathing. The voice is dull on the right, the heart's borders are shifted to the left. In the general analysis, leukocytes are observed - 12·10^9/l, rod cells - 13%, lymphocytes - 13%, ESR - 38 mm/h. What is the most likely diagnosis?

Пневмоторакс Pneumothorax

Правобічна пневмонія Right-sided pneumonia

Інфільтративний туберкульоз Infiltrative tuberculosis

Ателектаз легені Atelectasis of the lung

Ексудативний плеврит Exudative pleurisy

1051 / 1500
У виробничих умовах реєструються високі рівні шуму та забруднення повітря робочої зони ангідридом сірки. Який вид шкідливої дії на організм можуть справляти ці чинники в таких умовах? In production conditions, high levels of noise and air pollution of the working area with sulfur anhydride are registered. What kind of harmful effects can these factors have on the body in such conditions?

Комплексну Comprehensive

Комбіновану Combined

Специфічну Specific

Поєднану Combined

Роздільну Separate

1052 / 1500
Чоловіка 40 років доставлено до лікарні після закритої травми грудної клітки. Під час огляду права половина грудної клітки відстає під час дихання. Пальпаторно спостерігається різка болючість у проєкції V, VI, VII ребер на передній пахвовій лінії, підшкірна емфізема правих відділів тулуба. Перкуторно над правими відділами грудної клітки визначається тимпаніт. Аускультативно справа дихання не визначається, зліва - везикулярне. Якої хірургічної допомоги потребує хворий? A 40-year-old man was brought to the hospital after a closed chest injury. During the examination, the right half of the chest lags behind during breathing. Palpation shows sharp pain in the V projection, VI, VII ribs on the front axilla, subcutaneous emphysema of the right parts of the trunk. Auscultation of the right part of the chest is not determined, on the left - vesicular. What kind of surgical help does the patient need?

Дренування правої плевральної порожнини Drainage of the right pleural cavity

Негайна торакотомія Immediate thoracotomy

Дренування підшкірної емфіземи Drainage of subcutaneous emphysema

Каркасна стабілізація переломів ребер Frame stabilization of rib fractures

Туге бинтування грудної клітки Tight bandaging of the chest

1053 / 1500
На 10-й день після пологів до лікаря звернулася породілля зі скаргами на біль, що з'явився раптово, гіперемію та ущільнення в лівій молочної залозі, підвищення температури до 38^oC. Об'єктивно спостерігається: шкіра лівої молочної залози локально гіперемована в ділянці верхньо-зовнішнього квадранту, відчувається посилення болю під час її пальпації. Який діагноз найімовірніший? On the 10th day after giving birth, a woman in labor turned to the doctor with complaints of pain that appeared suddenly, hyperemia and compaction in the left mammary gland, an increase in temperature to 38°C. Objectively observed: the skin of the left mammary gland is hyperemic in the area of ​​the upper-external quadrant, there is an increase in pain during its palpation. What is the most likely diagnosis?

Лактаційний мастит Lactation mastitis

Рак лівої молочної залози Cancer of the left breast

Кіста лівої молочної залози з нагноєнням Left breast cyst with suppuration

Гемангіома молочної залози Breast hemangioma

Фіброаденома лівої молочної залози Left breast fibroadenoma

1054 / 1500
Чоловікові 61 року зроблено холецистектомію 7 діб тому. Супутнє захворювання - цукровий діабет. Яку дієту слід призначити хворому? A 61-year-old man underwent a cholecystectomy 7 days ago. The accompanying disease is diabetes. What diet should be prescribed to the patient?

Дієта № 9 Diet #9

Дієта № 10 Diet #10

Дієта № 15 Diet #15

Дієта № 5а Diet No. 5a

Дієта № 7 Diet #7

1055 / 1500
У пацієнта віком 27 років спостерігається постійне блювання, пронос у вигляді <<рисового відвару>>. Напередодні вживав невідомі спиртні напої і гриби. 3 дні тому повернувся з Індії, де перебував як турист. Об'єктивно спостерігається: температура - 35,6^oC, АТ не визначається. Шкіра суха, бліда, складки на ній не розправляються, пульс ниткоподібний, тони серця ослаблені. Який діагноз є найімовірнішим? A 27-year-old patient has constant vomiting, diarrhea in the form of <>. The day before, he consumed unknown alcoholic beverages and mushrooms. He returned from India 3 days ago , where he was as a tourist. Objectively observed: the temperature is 35.6°C, the blood pressure is not determined. The skin is pale, the folds are not straightened, the pulse is weak. What is the most likely diagnosis?

Сальмонельоз Salmonellosis

Ротавірусний гастроентерит Rotavirus gastroenteritis

Отруєння сурогатами алкоголю Alcohol surrogate poisoning

Холера Cholera

Отруєння грибами Mushroom poisoning

1056 / 1500
До лікаря звернулася хвора 55-ти років зі скаргами на збільшення щитоподібної залози, що спостерігається протягом останніх двох років, дискомфорт під час ковтання. Об'єктивно: ознаки гіпотиреозу, пальпаторно щитоподібна залоза щільна, не спаяна з навколишніми тканинами та рухлива при ковтанні. Регіонарні лімфатичні вузли не збільшені. У сироватці антитиреоїдні антитіла. Найбільш імовірний діагноз: A 55-year-old patient came to the doctor with complaints of an increase in the thyroid gland that has been observed for the past two years, discomfort when swallowing. Objectively: signs of hypothyroidism , the thyroid gland is dense on palpation, not fused to the surrounding tissues and mobile when swallowing. Antithyroid antibodies in the serum are the most likely diagnosis:

Ендемічний зоб Endemic goiter

Тиреоїдит Хашимото Hashimoto's thyroiditis

Серединна кіста шиї Middle neck cyst

Рак щитоподібної залози Thyroid cancer

Гострий тиреоїдит Acute thyroiditis

1057 / 1500
Пацієнт віком 40 років був прооперований із приводу флегмони поперекової ділянки. Після операції у нього різко підвищилася температура тіла до 38^oC, з'явилися явища інтоксикації, зросла кількість лейкоцитів у крові. У післяопераційній рані, на місці некротичної тканини, з'явилися бліді грануляції, заповнені гнійним умістом. Яке ускладнення виникло у пацієнта? A 40-year-old patient was operated on for phlegmon of the lumbar region. After the operation, his body temperature rose sharply to 38^oC, signs of intoxication appeared, the number of leukocytes in the blood. Pale granulations filled with purulent contents appeared in the postoperative wound?

Повторна флегмона Recurrent phlegmon

Сепсис Sepsis

Алергічна реакція Allergic reaction

Бешихове запалення Beshikov inflammation

Гнилісна флегмона Potential phlegmon

1058 / 1500
Пацієнта віком 50 років у тяжкому стані шпиталізовано до лікарні. На ЕКГ виявлено: ознаки гострого інфаркту міокарда. Об'єктивно спостерігається: пацієнт тривожний, напружений, у місті і часі не орієнтується, у своїй особистості орієнтується коректно. Відчуває слухові, а також яскраві зорові галюцінації страхітливого характеру, під впливом яких збуджений, схильний до агресивних дій. Висловлює уривчасті маячні ідеї. Який провідний психопатологічний синдром спостерігається у пацієнта? A 50-year-old patient in serious condition was admitted to the hospital. The ECG revealed: signs of an acute myocardial infarction. Objectively observed: the patient is anxious, tense, in the city and is not oriented in time, orients himself correctly. Experiences auditory and visual hallucinations of a frightening nature, under the influence of which he is prone to aggressive actions. What is the leading psychopathological syndrome observed in the patient?

Галюциноз Hallucinosis

Онейроїдний Oneiroid

Аментивний Amentary

Параноїдний Paranoid

Деліріозний Delirious

1059 / 1500
Чоловік 57 років скаржиться на кашель із виділенням великої кількості слизово-гнійного харкотиння (до 150 мл/добу). Об'єктивно спостерігається: пальці у вигляді <<барабанних паличок>>, нігті у вигляді <<годинникових стекол>>. Під час перкусії над легенями визначається вкорочений перкуторний звук, аускультативно вислуховуються велико- та середньопухирчасті вологі хрипи. Результат ЗАК: лейкоцитоз і зсув лейкоцитарної формули вліво. Рентгенографія ОГК показала: посилений легеневий малюнок, кільцевидні тіні. Бронхографія виявила: множинні циліндричні потовщення бронхів з чіткими контурами. Який попередній діагноз? A 57-year-old man complains of a cough with a large amount of mucous-purulent sputum (up to 150 ml/day). Objectively observed: fingers in the form of <>, fingernails in the form of <>. During percussion over the lungs, a short percussion sound is heard, and auscultatory sounds show: leukocytosis and a shift of the leukocyte formula to the left , ring-shaped shadows. Bronchography revealed: multiple cylindrical thickening of the bronchi with clear contours?

Бронхоектатична хвороба Bronchoectatic disease

Гангрена легені Gangrene of the lungs

Кіста легені Lung cyst

Ехінокок легені Echinococcus lung

Хронічна емпієма плеври Chronic pleural empyema

1060 / 1500
Жінка 45-ти років скаржиться на нападоподібний нестерпний біль у лівій половині обличчя тривалістю 1-2 хвилини. Напади провокуються жуванням. Захворіла два місяці тому після переохолодження. Об'єктивно: біль у точках виходу трійчастого нерва зліва. Дотик біля крила носа зліва викликає черговий напад з тонічною судомою м'язів обличчя. Який з перерахованих діагнозів найбільш імовірний? A 45-year-old woman complains of an attack-like unbearable pain in the left side of the face lasting 1-2 minutes. The attacks are provoked by chewing. She became ill two months ago after hypothermia. Ob' objectively: pain at the exit points of the trigeminal nerve on the left. Touching the left side of the nose causes another attack with a tonic spasm of the facial muscles. Which of the listed diagnoses is the most likely?

Гайморит Sinusitis

Невралгія язикоглоткового нерва Neuralgia of glossopharyngeal nerve

Артрит нижньощелепного суглоба Arthritis of the mandibular joint

Лицева мігрень Facial Migraine

Невралгія трійчастого нерва Trigeminal neuralgia

1061 / 1500
У юнака віком 18 років вперше діагностовано виразку цибулини дванадцятипалої кишки. Тест на Helicobacter pylori позитивний, рН шлункового соку - 1,0. Яка схема лікування є найдоцільнішою в цьому разі? An 18-year-old young man was diagnosed with duodenal ulcer for the first time. The test for Helicobacter pylori is positive, the pH of the gastric juice is 1.0. What treatment regimen is most appropriate in this if so?

Де-нол + трихопол De-nol + Trichopol

Кларитроміцин + омепразол Clarithromycin + Omeprazole

Омепразол + оксацилін Omeprazole + oxacillin

Квамател + амоксицилін Quamatel + amoxicillin

Де-нол + циметидин Denol + cimetidine

1062 / 1500
У чоловіка 30 років, що отримав опіки ІІ ступеня загальною площею 40% тіла, на четвертий день різко погіршився загальний стан, розвинулися інспіраторна задишка, частий кашель із пінистим харкотинням та ціаноз шкірних покривів. Під час аускультації в легенях прослуховується велика кількість вологих хрипів. АТ - 110/60 мм рт. ст., ЧСС - 100/хв., ЧД - 32/хв., ЦВТ - 100 мм водн. ст. Загальний білок - 50 г/л, Ht - 30%, Hb - 90 г/л. ЕКГ показує синусову тахікардію. Який механізм у патогенезі набряку легень є головним в цьому разі? A 30-year-old man who received second-degree burns over 40% of his body, on the fourth day his general condition worsened, inspiratory shortness of breath, frequent cough with frothy sputum developed and cyanosis of the skin. During auscultation, a large number of rales are heard. BP - 100/min., CVT - 100 mm Hg protein - 50 g/l, Ht - 90 g/l. ECG shows sinus tachycardia. What is the main mechanism in the pathogenesis of pulmonary edema in this case?

Гіповентиляція легень Hypoventilation of the lungs

Зниження осмотичного тиску плазми Reduction of plasma osmotic pressure

Зниження скоротливості міокарда Decreased myocardial contractility

Гіперволемія малого кола кровообігу Hypervolemia of a small circulatory circle

Порушення сурфактанту Surfactant violation

1063 / 1500
У дитини віком 1 рік із тетрадою Фалло, що хворіє на ГРВІ, раптово посилився ціаноз, з'явилася задишка, виникли судоми. Який невідкладний стан розвинувся у дитини? A 1-year-old child with tetralogy of Fallot, who is suffering from ARVI, suddenly developed cyanosis, shortness of breath, and convulsions. What emergency condition did the child develop?'

Задухо-ціанотичний напад Suffocation-cyanotic attack

Еклампсія Eclampsia

Напад пароксизмальної тахікардії Attack of paroxysmal tachycardia

Гостра серцева недостатність Acute heart failure

Фебрильні судоми Febrile convulsions

1064 / 1500
Син пацієнтки подав позов до суду після смерті його матері, яка хворіла на рак молочної залози. У зв'язку з тим, що у минулому вона пережила інсульт і мала діабет, їй була протипоказана агресивна терапія раку. Вона страждала від сильного болю, але лікарі амбулаторії не мали змоги отримати навіть 1 мг морфіну на її лікування. Який вид допомоги повинні були надати лікарі первинної ланки для полегшення стану пацієнтки? The patient's son filed a lawsuit after the death of his mother, who was suffering from breast cancer. Due to the fact that she had suffered a stroke in the past and had diabetes, aggressive cancer therapy was contraindicated for her. She was in severe pain, but the doctors at the outpatient clinic were not able to get even 1 mg of morphine for her treatment. What kind of care should the primary care doctors provide to alleviate the patient's condition?

Первинна медична допомога Primary medical care

Вторинна медична допомога Secondary medical care

Третинна медична допомога Tertiary medical care

Екстренна медична допомога Emergency medical assistance

Паліативна медична допомога Palliative medical care

1065 / 1500
Пацієнт скаржиться на висип у ділянці тулуба, згинальних поверхонь верхніх кінцівок, свербіж, що підсилюється в вечірній та нічний час. Об'єктивно спостерігається: на вказаних ділянках шкіри дрібні (до 12 мм в діаметрі) рожеві папули і везикули, розміщені попарно, екскоріації. Який найімовірніший діагноз? The patient complains of a rash in the trunk, flexion surfaces of the upper limbs, itching that intensifies in the evening and at night. Objectively observed: on the specified areas of the skin, small (up to 12 mm in diameter) pink papules and vesicles, located in pairs, excoriations. What is the most likely diagnosis?

Дерматит Dermatitis

Екзема Eczema

Короста Scabies

Вторинний папульозний сифілід Secondary papular syphilis

Червоний плоский лишай Red lichen planus

1066 / 1500
Після екстракції другого моляра у пацієнта підвищилася температура тіла, з'явилися біль у глотці зліва, інфільтрація, гіперемія нижнього відділу передньої піднебінної дужки, зміщення мигдалика до середньої лінії та вгору. Регіонарні лімфовузли болючі під час пальпації. Отоларинголог встановив діагноз «паратонзилярний абсцес». Який шлях розповсюдження інфекції характерний у цьому разі? After the extraction of the second molar, the patient's body temperature increased, there was pain in the pharynx on the left side, infiltration, hyperemia of the lower part of the anterior palatine arch, displacement of the tonsil to the midline and up. Regional lymph nodes are painful during palpation. The otolaryngologist diagnosed 'paratonsillar abscess.

Потрапляння стороннього тіла Foreign body ingress

Лімфогенний Lymphogenic

Одонтогенний Odontogenic

Гематогенний Hematogenous

Тонзилогенний Tonsylogenic

1067 / 1500
У жінки 32 років за 6 годин після фіброфагогастроскопії підвищилася температура тіла до 39^oC, болі за грудиною, сухість у роті, напростає задишка. Під час обстеження виявлено позитивний симптом Романова-Герке та підшкірну емфізему в надключичному просторі зліва. Який попередній діагноз? 6 hours after fibrophagogastroscopy, a 32-year-old woman's body temperature rose to 39^oC, pains behind the sternum, dry mouth, shortness of breath. During the examination, a positive Romanov-Herke symptom and subcutaneous emphysema in the left supraclavicular space. What is the previous diagnosis?

Гострий абсцесс легені Acute lung abscess

Піопневмоторакс Pyopneumothorax

Гострий медіастиніт Acute mediastinitis

Емпієма плеври Empyema of the pleura

Пневмонія Pneumonia

1068 / 1500
Мати прийшла на прийом до педіатра зробити своїй 2-місячній дитині планові щеплення. Для зручності, вона бажає аби дитині було введено комбіновану 6-компонентну вакцину, що містить антигени усіх збудників, проти яких рекомендовано вакцинуватися у цьому віці. Під час збору анамнезу вона зазначила, що іноді після пробудження її дитина починає раптово напружуватися, згинати голівку, ручки та ніжки. Зазвичай такі напади тривають не більше декількох секунд з інтервалами ''спо-кою'' у 10 секунд. Останній напад був декілька днів тому. Температура тіла - 37,2^oC , частота дихання - 25/хв., пульс - 104/хв., артеріальний тиск - 100/70 мм рт.ст. Яке рішення має прийняти лікар щодо подальшої імунізації цієї дитини? A mother came to the pediatrician's appointment to give her 2-month-old child routine vaccinations. For convenience, she wants the child to be given a combined 6-component vaccine containing antigens of all the pathogens that are recommended to be vaccinated at this age. During the anamnesis, she noted that sometimes after waking up, her child begins to tense up, bend the head, arms and legs. Usually such attacks last no more than a few seconds with 'calm' intervals '' in 10 seconds. The last attack was several days ago. Body temperature - 25/min., pulse - 100/70 mm Hg should the doctor accept regarding further immunization of this child?

Введення будь-яких вакцин на даний момент протипоказано Introduction of any vaccines is currently contraindicated

Ввести комбіновану 6-компонентну вакцину Enter a combined 6-component vaccine

Провести вакцинацію за схемою АКДП+гепатит В+Hib-інфекція Vaccinate according to the ACDP+hepatitis B+Hib infection scheme

Провести вакцинацію за схемою АДП+поліо+гепатит B+Hib-інфекція Vaccinate according to the scheme ADP+polio+hepatitis B+Hib-infection

Провести вакцинацію за схемою гепатит В+Hib-інфекція Vaccinate according to the hepatitis B+Hib infection scheme

1069 / 1500
Чоловік 27 років скаржиться на сухий кашель, задишку під час найменшого навантаження, біль у грудній кліт-ці, підвищення температури до 37,3^oС упродовж 3 тижнів. Колись зловживав наркотиками. Дихання жорстке, хрипів немає, тахікардія - 120/хв. Рентгенологічно з обох сторін легенів спостеріга-ються інтерстиціальні зміни. Під час бронхоскопії в лаважній рідині виявлені Pneumocista carinii. Які препарати будуть найефективнішими для лікування цього пацієнта? A 27-year-old man complains of a dry cough, shortness of breath during the slightest exertion, pain in the chest, an increase in temperature up to 37.3°C for 3 weeks. Used to abuse drugs. Breathing is hard, tachycardia - 120/min. Interstitial changes are observed on both sides of the lungs. During bronchoscopy, Pneumocista carinii is found to be the most effective for the treatment of this patient.

Цефамізин, нітроксолін Cefamycin, nitroxoline

Бісептол, кліндаміцин Biseptol, clindamycin

Еритроміцин, рифампіцин Erythromycin, Rifampicin

Інтерферон, ремантадин Interferon, Rimantadine

Ампіцилін, нізорал Ampicillin, Nizoral

1070 / 1500
Для організації водопостачання жителів селища вибрано вододжерело з умістом сульфатів, хлоридів, нітратів, заліза, що відповідає гігієнічним вимогам, проте з підвищеним умістом фтору (3 мг/дм^3). До розвитку якого захворювання може призвести вживання води з таким хімічним складом? For the organization of water supply to the residents of the village, a water source with a content of sulfates, chlorides, nitrates, iron, which meets hygienic requirements, but with an increased content of fluorine (3 mg/dm^ 3). What disease can be caused by the use of water with such a chemical composition?

Уролітіазу Urolithiasis

Подагри Gout

Жовчно-кам'яної хвороби Cholelithiasis

Карієсу Caries

Флюорозу Fluorosis

1071 / 1500
У хлопчика 7 років раптово виник біль, набрякло праве коліно. Напередодні він брав участь у кросі пересічною місцевістю. У сімейному анамнезі даних про гемофілію та підвищену кровоточивість немає. Об'єктивно спостерігається: температура тіла - 37,5^oC. Коліно болісне під час пальпації, гаряче на дотик, набрякле з локальним напруженням тканин над ним. У крові виявлено: Нb - 123 г/л, лейкоцити - 5,6·10^9/л, тромбoцити - 354·10^9/л, протромбіновий час - 12 сек (норма - 10-15 сек), частково активований тромбопластиновий час - 72 с (норма - 35-45 с). Час кровотечі нормальний, фактор VIII: - 5% від норми. Поставте діагноз. A 7-year-old boy suddenly developed pain, his right knee was swollen. The day before, he participated in cross-country skiing. There is no family history of hemophilia or increased bleeding. About objectively observed: body temperature - 37.5°C. Knee is painful during palpation, swollen with local tension of tissues above it. Blood - 123 g/l, leukocytes - 5.6·10 9/l, platelets - 354·10^9/l, prothrombin time - 12 sec (normal - 10-15 sec), partially activated thromboplastin time - 72 s (normal - 35-45 s). Bleeding time is normal, factor VIII : - 5% of the norm. Make a diagnosis.

Гемофілія В Hemophilia B

Нестача вітаміну К Vitamin K deficiency

Тромбоцитопенія Thrombocytopenia

Хвороба Шенлейна-Геноха Schönlein-Henoch disease

Гемофілія А Hemophilia A

1072 / 1500
Жінка віком 40 років з 15 років хворіє на епілепсію з генералізованими епілептичними нападами, які виникають 2-3 рази на місяць у нічний час та супроводжуються мимовільним сечовиділенням та дефекацією. Після психотравми (смерть батька) напади почастішали, виникають з періодичністю 2-3 хвилини, між нападами до свідомості не повертається, реакція зіниць на світло відсутня. Виберіть найімовірніший діагноз. A 40-year-old woman has been suffering from epilepsy since the age of 15 with generalized epileptic seizures that occur 2-3 times a month at night and are accompanied by involuntary urination and defecation. After the psychotrauma (father's death), the attacks became more frequent, occur with a frequency of 2-3 minutes, there is no return to consciousness between the attacks, the reaction of the pupils to light is absent. Choose the most likely diagnosis.

Істеричний напад Hysterical attack

Епілептичний статус Status epilepticus

Абсанс Absence

Збільшення кількості великих епінападів Increasing the number of large epiattacks

Епілептичний психоз Epileptic psychosis

1073 / 1500
Дівчина 18 років скаржиться на тяжкість і розпирання в правому підребер'ї. Стан посилюється після їжі чи прийнятті но-шпи. Живіт мякий, безболісний, печінка не збільшена. АсТ - 35 ОД, АлТ - 40 ОД. Аналіз крові показав: лейкоцити - 5,2 Г/л, ШОЕ - 7 мм/год. УЗД встановлено: печінка не збільшена. Жовчний міхур натщесерце 68 мл, після жовчогінного сніданку на 45 хв. - 64 мл. Стінка не потовщена. Який найбільш імовірний діагноз? An 18-year-old girl complains of heaviness and distention in the right hypochondrium. The condition worsens after eating or taking no-shpa. The abdomen is soft, painless, the liver is not enlarged. AsT - 40 U. Blood analysis showed: leukocytes - 7 mm/h. The liver was not enlarged, 68 ml after a choleretic breakfast. - 64 ml. The wall is not thickened. What is the most likely diagnosis?

Функціональна диспепсія Functional dyspepsia

Дискінезія жовчного міхура за гіпертонічним типом Gall bladder dyskinesia of the hypertensive type

Хронічний холецистит у стадії нестійкої ремісії Chronic cholecystitis in the stage of unstable remission

Дискінезія жовчного міхура за гіпотонічним типом Gall bladder dyskinesia of the hypotonic type

Синдром подразненого кишечника Irritable bowel syndrome

1074 / 1500
Жінка хворіє на бронхіальну астму із супутнім цукровим діабетом І типу, отримує в комплексній терапії: сальметерол, беклометазон, еуфілин, кромолін та преднізолон. Після призначення якої з наведених груп препаратів потрібна корекція дози інсуліну? A woman suffers from bronchial asthma with concomitant type 1 diabetes mellitus, receives in complex therapy: salmeterol, beclomethasone, euphylline, cromolyn and prednisolone. After the appointment of which of the following groups do you need insulin dose correction?

Кромони Kromoni

Метилксантини Methylxanthines

Пролонговані beta_2-агоністи Prolonged beta_2-agonists

Інгаляційні глюкокортикоїди Inhaled glucocorticoids

Системні глюкокортикоїди Systemic glucocorticoids

1075 / 1500
Жінка 58-ми років з цукровим діабетом 2 типу, що компенсується дієтою та метформіном, готується до холецистектомії. Об'єктивно: зріст - 164 см, вага - 90 кг. ЧСС- 72/хв., АТ- 130/80 мм рт.ст. Живіт м'який, болючий у правому підребер'ї. Печінка не збільшена. Глюкоза натще - 6,2 ммоль/л. Глікований гемоглобін - 6,5%. Яка подальша тактика цукрознижувальної терапії? A 58-year-old woman with type 2 diabetes, compensated by diet and metformin, is preparing for cholecystectomy. Objectively: height - 164 cm, weight - 90 kg. Heart rate - 130/80 mm Hg. Abdomen is painful. Fasting glucose - 6.2 mmol/l. 5%. What are the further tactics of hypoglycemic therapy?

Призначення інсуліну подовженої дії Prescription of long-acting insulin

Призначення суміші інсулінів Prescription of insulin mixture

Збереження схеми лікування Saving treatment plan

Призначення інсуліну короткої дії Prescription of short-acting insulin

Призначення глюренорму Appointment of glurenorm

1076 / 1500
Для проведення інтенсивної інфузійної терапії пацієнту з гострою дихальною недостатністю виконано катетеризацію підключичної вени за Сельдінгером. Після введення 600 мл інфузійного розчину стан пацієнта різко погіршився, збільшилося тахіпное з 26 до 40/хв., ДО зменшився з 400 мл до 250 мл. Під час аускультації дихання справа різко ослаблене. Перкуторно спостерігається тупість звуку. Яке ускладнення виникло у пацієнта? For intensive infusion therapy, a patient with acute respiratory failure underwent Seldinger's subclavian vein catheterization. After administration of 600 ml of infusion solution, the patient's condition worsened sharply, tachypnea increased from 26 to 40/min., DO decreased from 400 ml to 250 ml. During auscultation, the breath is sharply weakened. Dullness of sound is observed in the patient?

Гостра серцева недостатність Acute heart failure

Набряк легенів Pulmonary edema

Гідроторакс Hydrothorax

Набряк мозку Brain edema

Тромбоемболія легеневої артерії Thromboembolism of the pulmonary artery

1077 / 1500
З анамнезу відомо що у пацієнтки віком 70 років захворювання розпочалось близько півроку тому. Зі слів родичів у пацієнтки відмічалося різке погіршення пам'яті на поточні події, зокрема забувала дорогу додому, повертаючись з магазину, стала відчувати труднощі у виконанні повсякденних справ. Пам'ять на події минулого зберігалась. Спостерігалися прояви семантичної та амнестичної афазії. Періодично відмічається ехолалія. Настрій знижений, незначна тривожність. В анамнезі черепномозкова травма. Близько 15 років хворіє на цукровий діабет. Який найімовірніший діагноз? It is known from the anamnesis that a 70-year-old patient's illness began about six months ago. According to relatives, the patient's memory for current events was markedly deteriorating, in particular, she forgot the way When returning home, she began to experience difficulties in performing daily tasks diabetes. What is the most likely diagnosis?

Хвороба Альцгеймера Alzheimer's disease

Судинна деменція Vascular dementia

Енцефаліт Encephalitis

Пухлина головного мозку Brain tumor

Інволюційна депресія Involutional depression

1078 / 1500
В організм людини з атмосферного повітря надходять декілька хімічних речовин. Як називається тип комбінованої дії, де сукупний ефект перевищує суму ефектів кожної окремої речовини, що входить у комбінацію, в разі її ізольованої дії на організм? Several chemical substances enter the human body from atmospheric air. What is the type of combined action called, where the cumulative effect exceeds the sum of the effects of each individual substance included in the combination, in in case of its isolated effect on the body?

Потенціювання Potentiation

Поєднана дія Combined Action

Антагонізм Antagonism

Комплексна дія Complex Action

Ізольована дія Isolated Action

1079 / 1500
Чоловік скаржиться на болючість, почервоніння шкіри на правій литці. Об'єктивно встановлено: температура тіла - 38,5^oС, збільшені і болючі лімфатичні пахвинні вузли з правого боку, шкіра червона, межі між почервонінням і здоровою шкірою чіткі, є набряк, пухирці з темною рідиною всередині, пальпація болюча. Який попередній діагноз? The man complains of soreness, redness of the skin on the right calf. Objectively established: body temperature - 38.5°C, enlarged and painful inguinal lymph nodes on the right side, the skin is red, the boundaries between redness and healthy skin are clear, there is swelling, blisters with dark liquid inside, palpation is painful. What is the previous diagnosis?

Флегмона ноги Phlegmon of the leg

Вітряна віспа Chicken Pox

Бешиха Beshikha

Герпетична інфекція Herpes infection

Сибірка, шкіряна форма Anthrax, skin form

1080 / 1500
Дівчина 18 років скаржиться на напади чхання, переважно вранці, 15-20 разів поспіль, свербіння в носі, глотці, численні водянисті виділення з носа, нежить, біль в очах. Який із тестів є найінформативнішим для постановки діагнозу? An 18-year-old girl complains of sneezing attacks, mostly in the morning, 15-20 times in a row, itching in the nose, throat, numerous watery discharge from the nose, runny nose, pain in eyes. Which of the tests is the most informative for making a diagnosis?

Прик-тест Prick test

Провокаційна проба Provocative test

Загальний IgE Total IgE

Внутрішньошкірна проба Intradermal test

Крапельна проба Drop sample

1081 / 1500
Сімейний лікар вирішив зайнятися підприємницькою діяльність в сфері надання первинної медичної допомоги населенню. Яку організаційно-правову форму підприєм-ництва доцільно обрати? A family doctor decided to start a business in the field of providing primary medical care to the population. What organizational and legal form of business should be chosen?

Товариство з обмеженою відповідальністю Limited Liability Company

Акціонерне товариство Joint joint stock company

Підприємство зі створенням юридичної особи Enterprise with the creation of a legal entity

Товариство з повною відповідальністю General Liability Company

Приватний підприємець без створення юридичної особи Private entrepreneur without creating a legal entity

1082 / 1500
Працівник, не забезпечений засобами індивідуального захисту органів зору, перебуває в умовах інтенсивного інфрачервоного випромінювання Який негативний наслідок для здоров'я працівника найбільш ймовірно може розвинутись в таких умовах? The worker, who is not equipped with means of individual eye protection, is in conditions of intense infrared radiation. What negative consequence for the worker's health is most likely to develop in such conditions?

Астигматизм Astigmatism

Катаракта Cataract

Гіперметропія Hyperopia

Міопія Myopia

Глаукома Glaucoma

1083 / 1500
Чоловік скаржиться на болючу припухлість у ділянці підборіддя, загальне нездужання, головний біль. У цій ділянці виявлено гостро запалений щільний вузол конусоподібної форми. Шкіра над ним напружена, червона. Усередині вузла є виразка з прямовисними краями і некротичним стрижнем брудно-зеленого кольору. Підщелепні лімфатичні вузли справа збільшені і болючі. Який діагноз найімовірніший? A man complains of painful swelling in the chin area, general malaise, headache. In this area, an acutely inflamed, dense, cone-shaped node was found. The skin above it is tense, red. Inside the node there is an ulcer with vertical edges and a necrotic rod of dirty green color. The submandibular lymph nodes on the right are enlarged and painful. What is the most likely diagnosis?

Фурункул Furnish

Глибока трихофітія Deep ringworm

Карбункул Carbuncle

Туберкульоз Tuberculosis

Третинний сифіліс (гумозний) Tertiary syphilis (humous)

1084 / 1500
54-річний чоловік звернувся до лікаря зі скаргами на гострий біль за грудниною та задишку. В положенні лежачи біль посилює-ться. В анамнезі гіперхолестеринемія протягом 4-х років (лікує симвастатином). Задишку пов'язує із нещодавнім захворюванням на грип. При фізикальному обстеженні пульс - 92/хв., частота дихання - 16/хв., артеріальний тиск - 140/80 мм рт.ст. Під час огляду помітно розширені шийні вени. Аускультативних змін в легенях та серці не виявлено. На ЕКГ у всіх відведеннях елевація сегменту ST. На рентгенограмі збільшення тіні серця. Який діагноз є найбільш імовірним? A 54-year-old man consulted a doctor with complaints of sharp chest pain and shortness of breath. The pain worsens when lying down. He has a history of hypercholesterolemia for 4 years (treats with simvastatin).During the physical examination, pulse - 16/min, blood pressure - 140/80 mmHg veins. No changes in the lungs and heart are detected. On the ECG, the ST segment is elevated. What is the most likely diagnosis?

- -

Міокардит Myocarditis

Тромбоемболія легеневої артерії (ТЕЛА) Thromboembolism of the pulmonary artery (PEA)

Гострий інфаркт міокарда Acute myocardial infarction

Перикардит Pericarditis

1085 / 1500
Протягом останніх трьох років хвора жінка страждає на цукровий діабет. Сімейний лікар, з метою профілактики ускладнень цукрового діабету, регулярно призначає їй превентивне лікування. До якого виду профілактики належать вказані заходи? For the past three years, a sick woman has been suffering from diabetes. The family doctor, in order to prevent complications of diabetes, regularly prescribes preventive treatment for her. What kind of prevention do the specified measures?

Вторинна профілактика Secondary prevention

Первинна профілактика Primary prevention

Третинна профілактика Tertiary prevention

Індивідуальна профілактика Individual prevention

Громадська профілактика Public prevention

1086 / 1500
Добовий раціон харчування дівчинки 10 років містить: вітамін В_1 - 1,2 мг, вітамін В_2 - 0,6 мг, вітамін В_6 - 1,4 мг, вітамін РР - 15 мг, вітамін С - 65 мг. під час оцінювання харчового статусу у дівчинки виявлено хейлоз, глосит, ангулярний стоматит, кон'юктивіт. У дівчинки, найімовірніше: The daily diet of a 10-year-old girl contains: vitamin B_1 - 1.2 mg, vitamin B_2 - 0.6 mg, vitamin B_6 - 1.4 mg, vitamin RR - 15 mg, vitamin C - 65 mg. Cheilosis, glossitis, angular stomatitis, conjunctivitis were detected in the girl, most likely:

В_2 - гіповітаміноз B_2 - hypovitaminosis

В_1 - гіповітаміноз B_1 - hypovitaminosis

РР - гіповітаміноз RR - hypovitaminosis

С - гіповітаміноз C - hypovitaminosis

В_6 - гіповітаміноз B_6 - hypovitaminosis

1087 / 1500
Дівчинка віком 14 років хворіє на хронічний гломерулонефрит, хронічну ниркову недостатність. У дівчинки продовжує прогресувати анемічний синдром. Який препарат для патогенетичної терапії цієї анемії необхідно призначити? A 14-year-old girl suffers from chronic glomerulonephritis, chronic kidney failure. The girl's anemic syndrome continues to progress. What drug should be prescribed for the pathogenetic therapy of this anemia?

Фолієва кислота Folic acid

Ціанокобаламін Cyanocobalamin

Еритроцитарна маса Erythrocyte mass

Препарат заліза Iron drug

Еритропоетин Erythropoietin

1088 / 1500
Породілля 27-ми років, пологи ІІ, термінові, нормальні. 3 доба післяпологового періоду. Температура тіла - 36,8^oC, пульс - 72/хв., артеріальний тиск - 120/80 мм рт.ст. Молочні залози помірно нагрублі, соски чисті. Живіт м'який, безболісний. Дно матки на 3 п/п нижче пупка. Лохії кров'янисті, помірні. Який можна встановити діагноз? 27-year-old woman in labor, labor II, urgent, normal. 3 days postpartum. Body temperature - 36.8^oC, heart rate - 72/min. , blood pressure - 120/80 mm Hg. The mammary glands are soft, painless. The bottom of the uterus is bloody, moderate. ;

Фізіологічний перебіг післяпологового періоду Physiological course of the postpartum period

Лактостаз Lactostasis

Післяпологовий метроендометрит Postpartum metroendometritis

Залишки плацентарної тканини після пологів Remains of placental tissue after childbirth

Субінволюція матки Subinvolution of uterus

1089 / 1500
Під час огляду пацієнтки 64-річного віку лікар діагностував ожиріння (індекс маси тіла - 36 кг/м^2, об'єм талії - 118 см), артеріальну гіпертензію (170/105 мм рт. ст), порушення толерантності до вуглеводів. Під час обстеження виявлено підвищення рівню холестерину крові. Який діагноз хворої? During the examination of a 64-year-old female patient, the doctor diagnosed obesity (body mass index - 36 kg/m^2, waist circumference - 118 cm), arterial hypertension (170/105 mmHg), violation of carbohydrate tolerance. During the examination, an increase in blood cholesterol was detected. What is the patient's diagnosis?

Цукровий діабет Diabetes

Артеріальна гіпертензія Hypertension

Гіпоталамічний синдром Hypothalamic syndrome

Метаболічний синдром Metabolic syndrome

Аліментарне ожиріння Alimentary obesity

1090 / 1500
Хлопчик 3-х років надійшов з вираженим набряковим синдромом. Об'єктивно: блідий. Артеріальний тиск - 90/60 мм рт.ст. Олігурія. Загальний аналіз сечі: колір - жовтий, відносна густина - 1020, білок - 3,5 г/л, еритроцити - вилужені, 4-5 в п/з, лейкоцити - 2-3 в п/з. Добова протеїнурія - 6,6 г. Холестерин - 9,8 ммоль/л. Який наступний крок у веденні пацієнта буде найбільш доречним? A 3-year-old boy was admitted with severe edematous syndrome. Objectively: pale. Blood pressure - 90/60 mm Hg. Oliguria. General analysis of urine : color - yellow, relative density - 1020, protein - 3.5 g/l, erythrocytes - leached, 4-5 in p/z, leukocytes - 2-3 in p/z. Daily proteinuria - 6.6 g. Cholesterol - 9.8 mmol/l. What would be the most appropriate next step in the management of the patient?

Комп'ютерна томографія без контрасту Computed tomography without contrast

Проба Зимницького Zymnytskyi test

Аналіз сечі за Нечипоренко Urine analysis according to Nechiporenko

Біопсія нирки Kidney biopsy

Спостереження протягом тижня Observations during the week

1091 / 1500
Пацієнтка віком 65 років з цукровим діабетом лікувалась метформіном. У зв'язку з підвищеним АТ приймала сечогінні препарати. Поступово ефективність діуретика знизилась, з’явилась нудота, блювання. Об'єктивно спостерігається: пацієнтка не реагує на подразники, шкіра суха, запах ацетону відсутній. АТ - 180/100 мм рт. ст. Тони серця глухі, пульс - 98/хв, дихання везикулярне. Печінка - +4 см. У аналізі крові виявлено: глюкоза - 48 ммоль/л, Nа - 156 ммоль/л, К - 5,2 ммоль/л, сечовина - 15 ммоль/л. Яке ускладнення розвинулось у пацієнтки? A 65-year-old patient with diabetes was treated with metformin. Due to high blood pressure, she took diuretics. Gradually, the effectiveness of the diuretic decreased, nausea and vomiting appeared. Objectively observed: the patient does not react to stimuli, the smell of acetone is absent. Blood pressure is 180/100 mm Hg, the heart rate is 98/min. The liver is +4 cm revealed: glucose - 48 mmol/l, Na - 156 mmol/l, K - 5.2 mmol/l, urea - 15 mmol/l. What complication developed in the patient?

Кетоацидотична кома Ketoacidotic coma

Дисциркуляторна кома Dyscirculatory coma

Гіперосмолярна кома Hyperosmolar coma

Гіпоглікемічна кома Hypoglycemic coma

Лактацидотична кома Lactacidotic coma

1092 / 1500
Жінка 32 років поскаржилась на раптовий різкий головний біль, втратила свідомість, упала. Лікар швидкої допомоги відзначив важкий стан пацієнтки, сопор, менінгеальний синдром. У ста-ціонарі під час люмбальної пункції отримано кров'янистий ліквор, лікворний тиск - 260 мм водн. ст. Поставте попередній діагноз: A 32-year-old woman complained of a sudden sharp headache, lost consciousness, and fell. The emergency doctor noted the patient's serious condition, sopor, meningeal syndrome. In the hospital under at the time of the lumbar puncture, the cerebrospinal fluid was obtained, the cerebrospinal fluid pressure was 260 mm Hg. Make a preliminary diagnosis:

Менінгеома Meningioma

Ішемічний інсульт Ischemic stroke

Черепно-мозкова травма Traumatic brain injury

Менінгоенцефаліт Meningoencephalitis

Розрив аневризми, субарахноідальний крововилив Aneurysm rupture, subarachnoid hemorrhage

1093 / 1500
Пацієнт віком 30 років звернувся із скаргами на сильний нападоподібний біль у попереку справа, що віддає у пахову ділянку і внутрішню поверхню правого стегна, нудоту, здуття живота, блювання, порушення сечовиділення, що супроводжується різями і домішками крові у сечі. Об'єктивно спостерігається: пацієнт неспокійний, змінює положення в ліжку. З боку органів дихання і серцево-судинної системи патології не виявлено. Під час пальпації живота відмічається здуття, напруження м'язів і болючість у правому боці відповідно проекції правої нирки і за ходом правого сечоводу. Печінка і селезінка не збільшені. Симптоми запалення жовчного міхура негативні. Позитивний симптом Пастернацького справа. Який найімовірніший діагноз? A 30-year-old patient complained of severe attack-like pain in the lower back on the right, radiating to the inguinal area and the inner surface of the right thigh, nausea, abdominal distension, vomiting, violation of urination, accompanied by blood in the urine. Objectively observed: the patient changes the position in bed. On the part of the respiratory system and the cardiovascular system, bloating and muscle tension are noted pain in the right side of the right kidney and behind the right ureter. Symptoms of gallbladder inflammation are negative. What is the most likely diagnosis?

Жовчокам'яна хвороба Cholelithiasis

Сечокам'яна хвороба Urolithiasis

Туберкульоз нирки Kidney tuberculosis

Гломерулонефрит Glomerulonephritis

Пухлина нирки Kidney tumor

1094 / 1500
За 2 дні після вживання холодної їжі у чоловіка виникли скарги на різкий біль у горлі та ядуху. Дихання шумне, ЧД - 26/хв, температура тіла 39^oС. За результатами ларингоскопії: голосова щілина не проглядається, прикрита набряклими тканинами. Які термінові лікувальні заходи потрібно вжити? 2 days after eating cold food, the man complained of a sharp pain in the throat and pharynx. Breathing is noisy, BH - 26/min, body temperature 39^oС According to the results of laryngoscopy: the glottis is not visible, it is covered by swollen tissues. What urgent medical measures should be taken?

Штучну кисневу вентиляцію Artificial oxygen ventilation

Уведення спазмолітиків Introduction of antispasmodics

Внутрішньовенне уведення гормонів Intravenous administration of hormones

Трахеостомію Tracheostomy

Дезінтоксикаційну терапію Detoxification therapy

1095 / 1500
Дівчинка віком 13 років скаржиться на підвищення температури тіла до 37,4^oС протягом останніх 2 місяців після перенесеної ГРВІ. Під час огляду: худа статура, дифузне збільшення щитоподібної залози ІІ ступеня (щільна під час пальпації), екзофтальм, тахікардія. Який патологічний синдром спостерігається у пацієнтки? A 13-year-old girl complains of an increase in body temperature up to 37.4^oC during the last 2 months after having ARVI. During the examination: thin build, diffuse enlargement of the thyroid glands of the II degree (dense during palpation), exophthalmos, tachycardia. What pathological syndrome is observed in the patient?

Гіпопаратиреоз Hypoparathyroidism

Тиреотоксикоз Thyrotoxicosis

Тимомегалія Thymomegaly

Гіпотиреоз Hypothyroidism

Гіперпаратиреоз Hyperparathyroidism

1096 / 1500
Дівчинка 10 років потрапила до відділення із проявами кардиту. В анамнезі вказано: два тижні тому було загострення хронічного тонзиліту. Який етіологічний фактор найімовірніший у цьому разі? A 10-year-old girl was admitted to the department with manifestations of carditis. The anamnesis indicated: there was an exacerbation of chronic tonsillitis two weeks ago. What is the most likely etiological factor in this case?

Клебсієла Klebsiella

Пневмокок Pneumococcus

Протей Proteus

Стафілокок Staphylococcus

Стрептокок Streptococcus

1097 / 1500
Хлопець 17 рокiв скаржится на кровоточивiсть ясен. В анамнезі вказані: носовi кровотечi, загальна слабкiсть. Об'єктивно встановлено: помірна блiдiсть шкiряних покривiв i слизової оболонки рота, на слизовiй щiк, язика та м'якого пiднебiння множиннi петехiї. У кровi виявлено: еритроцити - 3,8 Т/л, Hb - 104 г/л, КП - 0,82, лейкоцити - 4,9 Г/л, тромбоцити - 80 Г/л, час зсiдання кровi - 5 хв., ШОЕ - 18 мм/год. Який найбiльш iмовiрний дiагноз? A 17-year-old boy complains of bleeding gums. The anamnesis indicates: nosebleeds, general weakness. Objectively established: moderate pallor of the skin and mucous membrane of the mouth, on mucosa of the cheeks, tongue and soft palate, multiple petechiae were found in the blood: erythrocytes - 3.8 T/l, Hb - 104 g/l, KP - 0.82, leukocytes - 4.9 g/l, platelets - 80. g/l, blood clotting time - 5 min., ESR - 18 mm/h. What is the most likely diagnosis?

Істинна поліцитемія Polycythemia True

Перніціозна анемія Pernicious anemia

Ідіопатична тромбоцитопенічна пурпу-ра Idiopathic thrombocytopenic purpura

Залiзодефiцитна анемiя Iron deficiency anemia

Хронiчний лiмфолейкоз Chronic lymphocytic leukemia

1098 / 1500
Пацієнт віком 24 роки скаржиться на гострий біль та висип у вигляді пухирців на лівій половині тулуба. Захворювання пов'язує з переохолодженням. Об'єктивно спостерігається: на шкірі тулуба зліва за ходом нерва згруповані дрібні пухирці, по периферії пухирців - запальна еритема. Який найімовірніший діагноз? A 24-year-old patient complains of acute pain and a rash in the form of blisters on the left half of the body. The disease is associated with hypothermia. Objectively observed: on the skin of the body On the left side of the course of the nerve are grouped small bubbles, on the periphery of the bubbles - inflammatory erythema. What is the most likely diagnosis?

Простий пухирчатий лишай Liquor simplex

Екзема Eczema

Оперізуючий лишай Zingles

Герпетиформний дерматоз Дюрінга Dühring's herpetiform dermatosis

Дерматит Dermatitis

1099 / 1500
Чоловік 56-ти років, звернувся до лікаря з носовою кровотечею, що розпочалася з лівої ніздрі 30 хвилин тому. Будь-які травми носа пацієнт заперечує. В анамнезі: частих носових кровотеч немає, подібний епізод вперше. Хворіє на фібриляцію передсердь (приймає варфарин) та артеріальну гіпертензію (лікується гідрохлортіазидом, атенололом). Температура тіла - 37,2^oC, пульс - 86/хв., артеріальний тиск - 120/70 мм рт.ст. Перед тим, як визначити необхідність тампонади носа, який першочерговий крок лікаря буде найбільш доречним? A 56-year-old man consulted a doctor with a nosebleed that started from the left nostril 30 minutes ago. The patient denies any injuries to the nose. In the anamnesis: there are no frequent nosebleeds, this is the first time he has atrial fibrillation (he takes warfarin) and hypertension (he is treated with hydrochlorothiazide, atenolol). Body temperature - 37.2°C, pulse - 86/min. Before determining the need for nasal tamponade, what is the most appropriate first step of the doctor?

Прикласти холодний компрес на перенісся Apply a cold compress to the bridge of the nose

Призначити каптоприл перорально Prescribe oral captopril

Ввести вітамін K внутрішньом'язово Inject vitamin K intramuscularly

- -

Оксиметазоліну інтраназально, попросити хворого затиснути крило носа та нахилитися вперед Oxymetazoline intranasally, ask the patient to pinch the wing of the nose and lean forward

1100 / 1500
Лікар-неонатолог оглядає доношену дитину від ІІ вагітності, ІІ термінових пологів, з масою тіла - 3980 г. З анамнезу відомо, що в пологах виникла первинна слабкість пологової діяльності, використовувалась акушерська допомога. Об'єктивно спостерігається: права рука приведена до тулуба, ротована, відсутні рухи в плечовому та ліктьовому суглобах, спостерігається симптом 'лялькової ручки', кисть в положенні долонного згинання. Годується грудьми, смокче активно. Який найімовірніший діагноз? A neonatologist examines a full-term child from the 2nd pregnancy, the 2nd emergency delivery, with a body weight of 3980 g. From the anamnesis, it is known that during childbirth there was a primary weakness of labor activity , obstetric care was used. Objectively observed: the right arm is brought to the body, there is no movement in the shoulder and elbow joints, the hand is in a palmar flexion position. What is the most likely diagnosis?

Парез Дюшена-Ерба Duchen-Erb paresis

Тотальний парез плечового сплетення Total paresis of the brachial plexus

Парез Дежерін-Клюмпке Parez Dezherin-Klumpke

Вивих плеча справа Right shoulder dislocation

Перелом плечової кістки справа Fracture of right humerus

1101 / 1500
Пацієнтку віком 42 роки шпиталізовано зі скаргами на тупий ниючий біль в попереку більше справа, інколи підвищення температури тіла до субфебрильної. З анамнезу відомо: 10 років тому під час вагітності був напад болю в правій половині попереку, підвищення температури тіла до 39^oС. Лікувалась антибіотиками. В останні роки почувала себе задовільно. Остані 5 років спостерігалося підвищення артеріального тиску. В аналізі сечі виявлено: білок - 0,66 г/л, лейкоцити - 10-15 в п/з, еритроцити - 2-3 в п/з. Який найімовірніший діагноз? A 42-year-old patient was hospitalized with complaints of a dull, aching pain in the lower back, more on the right side, sometimes with a rise in body temperature to subfebrile. It is known from the anamnesis: 10 years ago during pregnancy There was an attack of pain in the lower back, an increase in body temperature up to 39°C. In the last 5 years, an increase in blood pressure was observed: protein - 0.66 g/l. 10-15 in p/z, erythrocytes - 2-3 in p/z. What is the most likely diagnosis?

Хронічний пієлонефрит Chronic pyelonephritis

Хронічний гломерулонефрит Chronic glomerulonephritis

Туберкульоз нирок Kidney tuberculosis

Гіпертонічна хвороба Hypertensive disease

Уролітіаз нирок Kidney urolithiasis

1102 / 1500
Дівчинку 7-ми років доставлено до відділення невідкладної допомоги у важкому стані. Зі слів матері дитина почала скаржитися на біль у спині та шиї, після чого згодом їй було дуже важко вимовляти слова та випити склянку води. Об'єктивно: спастичність жувальних м'язів. Під час збору анамнезу лікар дізнався, що тиждень тому дитина гуляла надворі з друзями та уколола палець дерев'яною палицею, після якої у шкірі залишилася скіпка. Матір про інцидент завчасно повідомлена не була і видалила стороннє тіло декілька днів тому, коли випадково його помітила. Дівчинка не отримала жодного щеплення через особисті переконання матері. Скільки доз входить у первинний вакцинальний комплекс від збудника, що найбільш імовірно викликав захворювання у дівчинки? A 7-year-old girl was brought to the emergency department in critical condition. According to the mother, the child began to complain of back and neck pain, after which she was very it is difficult to pronounce words and drink a glass of water the incident was not reported in advance and she removed the foreign body a few days ago when she accidentally noticed it. The girl did not receive any vaccinations due to her mother's personal beliefs. How many doses are included in the primary vaccine complex that most likely caused the disease in the girl?

Шість Six

Чотири Four

Дві Two

Три Three

П'ять Five

1103 / 1500
Чоловік віком 44 роки, перед зверненням за допомогою до психіатра 6 днів щоденно вживав 0,5-0,8 л горілки. На 3 день вночі не міг заснути, став тривожним, скаржився на те, що бачить <<фільм жахів, який показують на стіні>>, чує крики з вулиці, з погрозами вбити його, вважає, що він знаходиться <<на заводі>>. Який найімовірніший діагноз? A 44-year-old man used 0.5-0.8 liters of vodka daily for 6 days before seeking help from a psychiatrist. On the 3rd day, he could not fall asleep, became anxious, complained that he sees a <>, hears screams from the street, with threats to kill him, believes that he is in a <>. What is the most likely diagnosis?

Алкогольний галюциноз Alcoholic hallucinosis

Алкогольний делірій Alcoholic delirium

Професійний делірій Professional delirium

Обсесивно-фобічний невроз Obsessive-phobic neurosis

Соматогений психоз Somatogenic psychosis

1104 / 1500
Пацієнтка віком 50 років скаржиться на гострий біль в епігастральній ділянці, блювання, різку загальну слабкість. Хворіє протягом 2 діб. Об'єктивно спостерігається: блідість шкіри, язик сухий. Пульс - 100/хв, слабкого наповнення, АТ - 110/70 мм рт. ст. Живіт під час пальпації м'який, помірно болючий в епігастрії та мезогастрії, симптомів подразнення очеревини немає. Перистальтика значно посилена у мезогастрії зліва. Під час рентгенологічного обстеження виявлено: чаші Клойбера зліва. Який найімовірніший діагноз? A 50-year-old patient complains of sharp pain in the epigastric area, vomiting, severe general weakness. She has been ill for 2 days. Objectively observed: pale skin, dry tongue Pulse - 100/min, blood pressure - 110/70 mmHg. Abdomen is moderately painful during palpation. Peristalsis is significantly increased in the left mesogastric area examination revealed: Kleuber's cup on the left. What is the most likely diagnosis?

Перфоративна виразка дванадцятипалої кишки Perforating duodenal ulcer

Гострий панкреатит Acute pancreatitis

Кишкова непрохідність Intestinal obstruction

Гострий холецистит Acute cholecystitis

Харчове отруєння Food poisoning

1105 / 1500
До лікарні шпиталізовано пацієнта віком 51 рік зі скаргами на пекучі болі за грудиною тривалістю близько 2 годин. На ЕКГ спостерігається: елевація сегмента ST у II, III, aVF відведеннях. Артеріальний тиск - 150/90 мм рт. ст., пульс - 90/хв. Який із наведених нижче препаратів відноситься до патогенетичної терапії цього захворювання? A 51-year-old patient was admitted to the hospital with complaints of burning pains behind the sternum lasting about 2 hours. The ECG shows: ST segment elevation in leads II, III, aVF Arterial pressure - 150/90 mm Hg, pulse - 90/min.

Альтеплаза Alteplase

Омепразол Omeprazole

Метронідазол Metronidazole

Мілдронат Mildronate

Предуктал Preductal

1106 / 1500
Жінка 32 років звернулась з приводу збільшення щитоподібної залози. Під час аварії на ЧАЕС перебувала у зоні підвищених радіоактивних опадів. Об'єктивно встановлено: щитоподібна залоза збільшена до ІІ ст., у ній пальпується щільний вузол, малорухомий, неболючий. Підщелепні лімфатичні вузли збільшені, неболючі. Під час УЗД у щитоподібній залозі виявлено гіпоехогенний утвір без чітких меж з кальцинатами. АТ - 120/70 мм рт. ст. Пульс - 78/хв. На основі приведених даних у хворої можна запідозрити: A 32-year-old woman applied for an enlarged thyroid gland. During the accident at the Chernobyl nuclear power plant, she was in the zone of increased radioactive fallout. It was objectively established: the thyroid gland was enlarged to the II century ., a dense, non-moving node is palpable. During ultrasound, a hypoechoic mass with calcifications was detected. Pulse - 78 mmHg. Based on the given data, the patient may be suspected of:

Підгострий тиреоїдит Subacute thyroiditis

Аденому щитоподібної залози Thyroid adenoma

Дифузний токсичний зоб Diffuse toxic goiter

Дифузний нетоксичний зоб Diffuse non-toxic goiter

Рак щитоподібної залози Thyroid cancer

1107 / 1500
У недоношеного новонародженого хлопчика, який народився від V вагітності, I пологів, у терміні гестації 27 тижнів, після народження дихання нерегулярне, становить <30/хв., SpO_2 - 70 %. Спостерігаються втягнення податливих ділянок грудної клітки, експіраторний стогін. Без кисневої підтримки виникає генералізований ціаноз. Аускультативно виявлено крепітуючі хрипи в базальних відділах. Дані про профілактику РДС відсутні. Яку тактику лікування потрібно вибрати? In a premature newborn boy born from V pregnancy, I delivery, at 27 weeks of gestation, after birth breathing is irregular, is <30/min., SpO_2 - 70%. There are retractions of the chest, expiratory wheezing. There is no evidence of wheezing in the basal regions. What treatment tactics should be chosen?

Введення препарату сурфактанту інтратрахеально в перші 15 хвилин після народження Intratracheal surfactant administration in the first 15 minutes after birth

Введення препарату сурфактанту інтратрахеально через 2 години після народження Intratracheal surfactant administration 2 hours after birth

ШВЛ мішком Амбу та маскою Ventilation with Ambu bag and mask

Киснева терапія за допомогою кисневого намету Oxygen therapy using an oxygen tent

Призначення антибактеріальної терапії Prescription of antibacterial therapy

1108 / 1500
Чоловік 30 років, пресувальник кабелів, скаржиться на млявість, погіршення пам'яті, біль у кінцівках. Об'єктивно спостерігаються блідість шкірних покривів, анемія, ретикулоцитоз, базофільна зернистість еритроцитів, підвищений вміст порфіріну в сечі. Симптоми якого захворювання спостерігаються у пацієнта? A 30-year-old man, a cable crimper, complains of lethargy, memory impairment, pain in the limbs. Objectively, pallor of the skin, anemia, reticulocytosis, basophilic granularity of erythrocytes, increased content of porphyrin in the urine. Symptoms of which disease are observed in the patient?

Сидероз Siderosis

Азбестоз Asbestosis

Сатурнізм Saturnism

Меркуріалізм Mercurialism

Бериліоз Berylliosis

1109 / 1500
35-річний чоловік звернувся до лікаря зі скаргами на сильний біль у правому оці, світлобоязнь, сльозотечу, погіршення зору. Скарги з'явились після роботи зі зварювальним апаратом. Контактні лінзи не використовує. Під час огляду: очна щілина звужена, помірна перикорнеальна ін'єкція, гострота зору збережена. Після інстиляції розчину анестетика, стан полегшився, що дало можливість провести огляд у щілинній лампі та виявити дефект рогівки розміром 3 мм, що зафарбувався флюоресцеїном. Яке лікування буде найбільш доречним? A 35-year-old man consulted a doctor with complaints of severe pain in the right eye, photophobia, lacrimation, impaired vision. The complaints appeared after working with a welding machine. He does not use contact lenses. During the examination: the eye slit is narrowed, the pericorneal injection is moderate, the visual acuity is preserved. After instillation of the anesthetic solution, the condition was relieved, which made it possible to conduct a slit-lamp examination and reveal a 3 mm corneal defect stained with fluorescein. . What treatment would be most appropriate?

Сухе тепло Dry heat

Інстиляція дексаметазону Dexamethasone instillation

Інстиляція сульфацилу натрію Sulfacyl sodium instillation

Інстиляція таурину Taurine instillation

Інстиляція офлоксацину Ofloxacin instillation

1110 / 1500
Хлопчик 7-ми років звернувся по допомогу до педіатра зі скаргами на повторювані епізоди нудоти і блювання протягом останніх 3-х днів. Блювання без домішок крові і жовчі, виникає переважно вранці і повторюється до 10-ти разів на день. Вживає здорову їжу. Це третій епізод блювання за останні 6 місяців. Об'єктивно: запах ацетону з рота, легкий ступінь дегідратації. Інші показники в нормі. Який діагноз найімовірніший? A 7-year-old boy sought help from a pediatrician with complaints of repeated episodes of nausea and vomiting during the past 3 days. Vomiting without blood and bile impurities, occurs mainly in the morning and is repeated up to 10 times a day. This is the third episode of vomiting in the last 6 months.

Гострий панкреатит Acute pancreatitis

Гострий гастрит Acute gastritis

Синдром циклічного блювання Syndrome of cyclic vomiting

Хронічний гастрит Chronic gastritis

Виразкова хвороба шлунку Gastric ulcer disease

1111 / 1500
Хлопчик 12-ти років звернувся до хірурга зі скаргою на нестерпний біль кінчика II пальця правої кисті. Лікарем з'ясовано, що 4 дні тому він вколов це місце дротом. Протягом останніх трьох днів з'явився різкий пульсуючий біль, набряк тканин та гіперемія дистальної фаланги пальця. Об'єктивно: температура тіла - 38,1^oC. Який з перерахованих діагнозів є найбільш імовірним? A 12-year-old boy turned to the surgeon complaining of excruciating pain in the tip of the II finger of the right hand. The doctor found out that 4 days ago he pricked this place with a wire During the last three days, sharp throbbing pain and swelling of the distal phalanx of the finger appeared. Objectively, the body temperature is 38.1°C.

Панарицій Panaricius

Пароніхія Paronychia

Еризипелоїд Erysipeloid

- -

Целюліт Cellulite

1112 / 1500
У хворого 30-ти років через 3 доби після розкриття флегмони шиї з'явилися скарги на задишку, гарячку, біль за грудиною, що посилювався під час закидання голови назад. Стан хворого прогресивно погіршувався. В ході рентгенологічного обстеження виявлено розширення тіні середостіння й зниження прозорості його відділів. Яке ускладнення розвинулося в даного хворого? A 30-year-old patient complained of shortness of breath, fever, pain behind the sternum, which worsened when the head was thrown back, 3 days after opening the phlegmon of the neck The patient's condition progressively worsened. During the X-ray examination, an expansion of the mediastinum and a decrease in the transparency of its parts were found.

Інфекційний ендокардит Infective endocarditis

Гострий гнійний медіастиніт Acute purulent mediastinitis

Гнійний тиреоїдит Suppurative thyroiditis

Заглотковий абсцес Pharyngeal abscess

Ексудативний перикардит Exudative pericarditis

1113 / 1500
Пацієнт віком 48 років скаржиться на частий стискальний біль за грудиною з іррадіацією в ліве плече і ліву лопатку. Напади виникають вночі, у повному спокої, тривають 10-15 хв. Змін загального стану та порушень на ЕКГ під час звертання до поліклініки не виявлено. Яке обстеження є найнеобхіднішим для уточнення діагнозу? A 48-year-old patient complains of frequent squeezing pain behind the sternum with radiation to the left shoulder and left scapula. The attacks occur at night, at complete rest, and last 10-15 minutes No changes in the general condition and abnormalities were detected during the visit to the clinic. What examination is most necessary to clarify the diagnosis?

Ехокардіографія Echocardiography

Катетеризація порожнин серця Catheterization of heart cavities

Повторна ЕКГ через тиждень Repeat ECG in a week

Холтерівське моніторування ЕКГ Holter ECG monitoring

Велоергометрія Cycle ergometry

1114 / 1500
Пацiєнтка віком 45 рокiв скаржиться на дискомфорт пiд час читання, почервонiння краю повiк, бiлi пiнистi видiлення в кутах очних щiлин протягом останнiх двох мiсяцiв. Об'єктивно спостерiгається: гiперемiя та потовщення вiльного краю повiк, розширення вивiдних протокiв залоз хряща повiк. Який найімовірніший діагноз? A 45-year-old patient complains of discomfort while reading, reddening of the edge of the eyelids, white foamy discharge in the corners of the eye slits for the past two months. Objectively observed: hyperemia and thickening of the free edge of the eyelids, expansion of the excretory ducts of the eyelid cartilage. What is the most likely diagnosis?

Хронiчний каналiкулiт Chronic canaliculitis

Аденовiрусний кон'юнктивiт Adenovirus conjunctivitis

Мейбомiєвий блефарит Meibomian blepharitis

Гострий дакрiоаденiт Acute dacryoadenitis

Бленорейний кон'юнктивiт Blenorrhoeic conjunctivitis

1115 / 1500
Пацієнт віком 32 роки хворіє на епілепсію. Раптово, без будь-яких причин, став збудженим. Об'єктивно спостерігається: дезорієнтований у власній особі та навколишньому середовищі, зорові та слухові галюцинації загрозливого змісту, маячні ідеї стосунку і переслідування; на обличчі вираз страху, гніву, люті; поведінка агресивна, з руйнівними діями. Визначте психопатологічний синдром. A 32-year-old patient suffers from epilepsy. Suddenly, without any reason, he became excited. Objectively observed: disoriented in his own person and environment, vision and auditory hallucinations of a threatening content, delusional ideas of the relationship and persecution; the expression of fear, anger, rage; define the psychopathological syndrome.

Параноїдний синдром Paranoid Syndrome

Деліріозний синдром Delirious syndrome

Сутінковий синдром Twilight Syndrome

Маніакальний синдром Manic syndrome

Гебефренічний синдром Hebephrenic syndrome

1116 / 1500
Хвора 55-ти років скаржиться на біль і хруст в лівому колінному суглобі, які виникають при ходьбі по сходах, періодичне ''заклинювання'' суглобу при рухах. 5 років тому була травма лівого коліна. Клінічний та біохімічний аналізи крові без особливих змін. Рентгенологічно виражений остеосклероз, остеофіти. Звуження суглобової щілини. Який попередній діагноз? A 55-year-old patient complains of pain and creaking in the left knee joint, which occur when walking up the stairs, periodic 'jamming' of the joint during movements. 5 years ago there was an injury to the left knee. Clinical and biochemical blood tests without special changes. Radiologically expressed osteosclerosis, osteophytes. Narrowing of the joint space. What is the previous diagnosis?

Подагричний артрит Gouty arthritis

Ревматоїдний артрит Rheumatoid arthritis

Остеоартроз Osteoarthrosis

Псоріатичний артрит Psoriatic arthritis

Реактивний артрит Reactive arthritis

1117 / 1500
У чоловіка віком 35 років, який хворіє на епілепсію, розвинувся великий судомний напад тривалістю 4 хвилини. За 10 хвилин після нападу свідомість не відновилася, виникли повторні тоніко-клонічні судоми. Який лікувальний засіб є препаратом вибору в цьому клінічному випадку? A 35-year-old man with epilepsy developed a grand convulsive seizure lasting 4 minutes. 10 minutes after the seizure, he did not regain consciousness, repeated tonic-clonic convulsions. What is the drug of choice in this clinical case?

Галоперидол Haloperidol

Амітриптилін Amitriptyline

Карбамазепін Carbamazepine

Діазепам Diazepam

Вальпроєва кислота Valproic acid

1118 / 1500
Пацієнт віком 38 років скаржиться на підвищення температури тіла до 39^oС, мерзлякуватість, профузний піт, тупий біль у попереку, що поширюється у надлобкову ділянку. Об'єктивно спостерігається: напруження м'язів у поперековому відділі, болісні відчуття під час постукування у поперековій ділянці з обох боків. У загальному аналізі крові спостерігається: лейкоцитоз - 12·10^9/л. У загальному аналізі сечі виявлено: протеїнурія - 0,7 г/л, лейкоцитурія 15-20 у полі зору, бактеріурія більше 100 000 на 1 мл сечі. Який найімовірніший діагноз? A 38-year-old patient complains of an increase in body temperature to 39^oС, chills, profuse sweat, dull pain in the lower back, spreading to the suprapubic area. Objectively observed: muscle tension in the lumbar region, painful sensations during tapping in the lumbar region on both sides: leukocytosis - 12·10^9/l. In the general analysis of urine: proteinuria - 0.7 g /l, leukocyturia 15-20 in the field of vision, bacteriuria more than 100,000 per 1 ml of urine. What is the most likely diagnosis?

Гострий пієлонефрит Acute pyelonephritis

Туберкульоз нирок Kidney tuberculosis

Нирковокам'яна хвороба Kidney stone disease

Гострий гломерулонефрит Acute glomerulonephritis

Гострий цистит Acute cystitis

1119 / 1500
Пацієнтка віком 31 рік звернулася до лікаря-гінеколога зі скаргами на затримку менструації на 2 тижні, ранкову нудоту, мажучі кров'янисті виділення з піхви. Тест на вагітність виявився позитивним. Під час УЗД виявлено: плідного яйця в порожнині матки немає. Які обстеження потрібно призначити пацієнтці? A 31-year-old patient turned to a gynecologist with complaints of delayed menstruation for 2 weeks, morning sickness, smearing bloody discharge from the vagina. The pregnancy test showed positive. During the ultrasound, it was found that there is no fertile egg in the uterine cavity. What examinations should be prescribed to the patient?

ХГ крові Hg blood

Визначення рівня прогестерону в крові Determining the level of progesterone in the blood

МРТ MRI

Визначення ХГ в крові в динаміці Determination of hCG in blood in dynamics

Визначення РАРР+ХГ в крові Determination of PAPP+ChG in blood

1120 / 1500
До лікарні шпиталізовано чоловіка віком 45 років. В анамнезі пацієнта наявне варикозне розширення вен нижніх кінцівок. Він раптово відчув біль у грудній клітці та ядуху. Об'єктивно спостерігається: набухання шийних вен, ціаноз. На ЕКГ виявлено: ознака Мак-Джина-Уайта (QIII-SI), P- pulmonale, відхилення електричної вісі серця вправо, ознаки перевантаження правих відділів серця. Визначення рівня якої речовини в сироватці крові необхідно провести для підтверження діагнозу ? A 45-year-old man was hospitalized. The patient's anamnesis has varicose veins of the lower extremities. He suddenly felt pain in the chest and throat. Objectively observed: Swelling of the neck veins, cyanosis. The ECG revealed: Mc-Gin-White's sign (QIII-SI), deviation of the electrical axis of the heart to the right. Determination of the level of the substance in the blood serum is necessary to confirm the diagnosis ?

Серцевого тропоніна Cardiac Troponin

Аланінамінотрансферази Alanine aminotransferases

D-димеру D-dimer

Креатинфосфокінази Creatine phosphokinase

Аспартатамінотрансферази Aspartate aminotransferases

1121 / 1500
Жінка 30-ти років скаржиться на загальну слабкість, утруднене ковтання їжі, сухість шкіри та ламкість волосся. Об'єктивно: температура тіла - 36,6^oC, ЧД- 16/хв., Ps- 92/хв., АТ- 110/70 мм рт.ст. Шкірні покриви та видимі слизові бліді. У крові: Hb- 65 г/л, еритроцити - 3,2·10^12/л, КП- 0,6, рет - 3%, лейкоцити - 6,7·10^9/л, е - 2%, п - 3%, с - 64%, л - 26%, м - 5%, ШОЕ- 17 мм/год. Сироваткове залізо - 7,4 мкмоль/л, загальний білок - 78 г/л. Дефіцит якого фактору зумовив виникнення захворювання? A 30-year-old woman complains of general weakness, difficulty swallowing food, dry skin and brittle hair. Objectively: body temperature - 36.6^oC, BH- 16/min., Ps- 92/min., BP- 110/70 mm Hg. Skin and mucous membranes are pale. Blood: Hb- 65 g/l, erythrocytes - 3.2·10^12 /l, CP - 0.6, ret - 3%, leukocytes - 6.7·10^9/l, e - 2%, p - 3%, c - 64%, l - 26%, m - 5% , ESR - 17 mm/h. Serum iron - 7.4 μmol/l, total protein - 78 g/l. What factor caused the disease?

Білку Squirrel

Фолієвої кислоти Folic acid

Вітаміну B_6 Vitamin B_6

Заліза Iron

Глюкозо-6-фосфатдегідрогенази Glucose-6-phosphate dehydrogenase

1122 / 1500
Вагітна жінка віком 25 років скаржиться на підвищення температури до 38,5^oC впродовж двох днів, кашель, задишку. Ці скарги з'явилися після переохолодження. Під час аускультації виявлено: крепітація та локалізовані вологі хрипи в нижній частині правої легені, там же спостерігається притуплення перкуторного звуку. У загальному аналізі крові спостерігається: лейкоцити - 11,0·10^9/л, ШОЕ - 22 мм/год. Який антибактеріальний засіб потрібно призначити? A 25-year-old pregnant woman complains of an increase in temperature to 38.5^oC for two days, cough, shortness of breath. These complaints appeared after hypothermia. During auscultation revealed: crepitation and localized moist rales in the lower part of the right lung, there is a dullness of the percussion sound. In the general blood analysis, leukocytes are observed - 11.0·10^9/l, ESR - 22 mm/h appoint?

Карбеніцилін Carbenicillin

Амікацин Amikacin

Левофлоксацин Levofloxacin

Амоксицилін Amoxicillin

Доксициклін Doxycycline

1123 / 1500
Чоловік віком 35 років звернувся до лікаря зі скаргами на різке збільшення ваги на 27 кг протягом 2 років, слабкість, випадіння волосся на голові, зниження потенції. Об'єктивно спостерігається: зріст - 174 см, вага - 104 кг. Відкладення жиру переважно на шиї та тулубі. Шкіра суха, багряно-ціанотичного кольору. АТ- 160/110 мм рт. ст. Під час лабораторного дослідження виявлено підвищення рівня АКТГ. На комп’ютерній томографії - мікроаденома гіпофіза. Який найбільш імовірний діагноз? A 35-year-old man consulted a doctor with complaints of a sharp weight gain of 27 kg over the course of 2 years, weakness, hair loss on the head, decreased potency. Objectively observed: height - 104 kg. Skin of a purple color - 160/110 mm Hg. computed tomography - pituitary microadenoma. What is the most likely diagnosis?

Хвороба Іценка-Кушинга Itsenko-Cushing disease

Гіпертонічна хвороба Hypertensive disease

Синдром Іценка-Кушинга Itsenko-Cushing syndrome

Хвороба Аддісона Addison's disease

Аліментарне ожиріння Alimentary obesity

1124 / 1500
У новонародженого на 3 добу виявлено деформацію, набряк та гематому м'яких тканин у надключичній ділянці ліворуч. Верхня кінцівка приведена до тулуба, пасивні рухи супроводжуються неспокоєм дитини. Який імовірний діагноз? On the 3rd day, the newborn was found to have deformation, edema and hematoma of soft tissues in the supraclavicular area on the left. The upper limb is brought to the trunk, passive movements are accompanied by restlessness of the child. What probable diagnosis?

Перелом лівої ключиці зі зсувом відламків Fracture of the left clavicle with displacement of fragments

Підокістний перелом лівої ключиці без зсуву відламків Subosseous fracture of the left clavicle without displacement of fragments

Остеомієліт лівої ключиці Osteomyelitis of the left clavicle

Акушерський параліч Ерба Erb's Obstetric Palsy

Флегмона новонародженого Phlegmon of a newborn

1125 / 1500
Пацієнтка віком 62 роки лікується з приводу двобічного гонартрозу, регулярно приймає диклофенак натрію протягом 2-х років. Останні півроку відмічає підйоми АТ до 160/100 мм рт. ст. Під час рентгенологічного дослідження колінних суглобів виявлено: зниження висоти міжсуглобової щілини, крайові розростання суглобових поверхонь та епіфізарний остеопороз. У загальному аналізі сечі спостерігається: питома вага - 1010, еритроцити - 5-6 в п/з, лейкоцити - 10-12 в п/з, циліндри - 0-1 в п/з, глюкоза - не виявлено, білок - 0,22 г/л. Глюкоза плазми натще - 6,3 ммоль/л. Яка ймовірна причина виявлених змін у загальному аналізі сечі? A 62-year-old patient is being treated for bilateral gonarthrosis, has been regularly taking diclofenac sodium for 2 years. For the past six months, she has been experiencing blood pressure rises to 160/100 mm Hg During the X-ray examination of the knee joints, a decrease in the height of the joint space, marginal growths of the articular surfaces, and epiphyseal osteoporosis were observed. /z, cylinders - 0-1 in p/z, glucose - not detected, protein - 0.22 g/l. Fasting plasma glucose - 6.3 mmol/l. What is the probable cause of the detected changes in the general analysis of urine?

Гіпертензивна нефропатія Hypertensive nephropathy

Хронічний гломерулонефрит Chronic glomerulonephritis

Сечокам'яна хвороба Urolithiasis

Діабетична нефропатія Diabetic nephropathy

Тубулоінтерстиціальний нефрит Tubulointerstitial nephritis

1126 / 1500
Хворий 35-ти років звернувся до сімейного лікаря з раптовим кинджальним болем в епігастрії; в анамнезі виразка шлунку. Об'єктивно: дошкоподібне напруження м'язів передньої черевної стінки, позитивний симптом Щоткіна-Блюмберга. При рентгенологічному дослідженні виявлено серпоподібне просвітлення під склепінням діафрагми. Яке ускладнення розвинулося? A 35-year-old patient went to the family doctor with a sudden stabbing pain in the epigastrium; a history of stomach ulcer. Objectively: plate-like tension of the muscles of the anterior abdominal wall , a positive Shtokkin-Blumberg symptom. A sickle-shaped light was detected under the diaphragmatic vault.

Пенетрація виразки шлунку Stomach ulcer penetration

Перфорація виразки шлунку Perforation of gastric ulcer

Виразкова кровотеча Ulcer bleeding

Малігнізація виразки шлунку Malignancy of gastric ulcer

Стеноз воротаря Stenosis of the goalkeeper

1127 / 1500
Хлопчик 10-ти рокiв скаржиться на підвищення температури тіла до 38,5^oC, припухлість суглобів пальців рук та ніг, ранкову скутість, біль у шийному відділі хребта. Проведено обстеження. У кровi: еритроцити - 2,6·10^12/л, Hb- 85 г/л, лейкоцити - 16,5·10^9/л, е.- 2%, п.- 8%, с.- 68%, л.- 16%, м.- 6%, швидкiсть осiдання еритроцитів - 28 мм/год. Позитивний ревматоїдний фактор. Диспротеїнемія з гіперглобуліне-мією. Встановіть діагноз: A 10-year-old boy complains of an increase in body temperature up to 38.5°C, swelling of the joints of fingers and toes, morning stiffness, pain in the cervical spine. An examination was carried out in the blood: erythrocytes - 2.6·10^12/l, Hb - 85 g/l, leukocytes - 16.5·10^9/l, e. - 2%, p. - 8%, .- 68%, l.- 6%, erythrocyte sedimentation rate - 28 mm/h. Dysproteinemia with hyperglobulinemia.

Гостра ревматична лихоманка Acute rheumatic fever

Ювенiльний ревматоїдний артрит Juvenile rheumatoid arthritis

Токсичний синовiїт Toxic synovitis

Реактивний артрит Reactive arthritis

Остеоартрит Osteoarthritis

1128 / 1500
Хвора 38-ми років звернулась до лікаря зі скаргами на слабкість, сонливість, біль у суглобах, збільшення ваги при зниженому апетиті, закрепи. Шкіра суха, потовщена. Обличчя одутле, амімічне. Очні щілини звужені, язик потовщений. Низький, дещо хрипкий голос. Тони серця - послаблені, пульс - 56/хв. Т4 вільний - знижений. Хворій треба постійно приймати: A 38-year-old patient turned to the doctor with complaints of weakness, drowsiness, joint pain, weight gain with reduced appetite, constipation. The skin is dry, thickened. Face Swollen, amicable. The eye slits are thickened. The heart sounds are weakened, the pulse is 56/min. The patient must take:

Глюконат кальцію Calcium gluconate

Тироксин Tyroxin

Фуросемід Furosemide

Карбонат літію Lithium Carbonate

Мерказоліл Mercazolil

1129 / 1500
Жінка 53-х років, звернулася до лікаря зі скаргами на розпираючий біль в м'язах лівої гомілки та набряк, що з'явилися тиждень тому. Раніше подібного стану не виникало. Температура тіла - 37,6^oC. При огляді гомілка в середній третині збільшена в об'ємі на 5 см. Шкіра гомілки та ступні синюшна, блискуча, відмічається місцева гіпертермія. При пальпації гомілки - різка болючість. Позитивні симптоми Хоманса та Мозеса. Пульс на артеріях стоп задовільний. Який діагноз є найбільш імовірним? A 53-year-old woman turned to the doctor with complaints of excruciating pain in the muscles of the left leg and swelling that appeared a week ago. Previously, a similar condition did not occur. Body temperature - 37.6°C. When examining the lower leg, the volume is increased by 5 cm. The skin of the lower leg and foot is bluish. When palpating the lower leg, there is a sharp pain Moses. The pulse on the arteries of the feet is satisfactory. What is the most likely diagnosis?

- -

Післятромбофлебітичний синдром Postthrombophlebitic syndrome

Хвороба Рейно Raynaud's disease

Гострий тромбоз підколінної артерії Acute thrombosis of popliteal artery

Гострий тромбоз підколінної вени Acute thrombosis of popliteal vein

1130 / 1500
У курсанта морського коледжу в періа-нальній ділянці на 2-х годинах умовного циферблату виявлена неболюча щільна виразка розмірами 1,5х0,5 см, дно якої має вигляд ''старого сала''. Який імовірний діагноз? A non-painful dense ulcer measuring 1.5x0.5 cm was found in the perianal area of ​​the naval college cadet at 2 o'clock on the conventional dial, the bottom of which looks like ' 'old lard'. What is the probable diagnosis?

Нориця прямої кишки Fistula of the rectum

Рак анального відділу прямої кишки Anal rectal cancer

Твердий шанкр (сифіліс) прямої кишки Hard chancre (syphilis) of the rectum

Тріщина прямої кишки Fissure of the rectum

Нагноєння крипти прямої кишки Suppuration of the crypt of the rectum

1131 / 1500
Підліток госпіталізований зі скаргами на обмеженість рухів у нижніх кінцівках. Захворювання розпочалось 2 дні тому з підвищення температури тіла до 38,3^oC, рідких випорожнень 3-4 рази на добу. Об'єктивно: температура тіла - 36,8^oC, активні рухи в нижніх кінцівках відсутні, у зоні ураження - арефлексія, гіпотонія м'язів, чутливість збережена. Менінгеальні симптоми слабко позитивні. Яке захворювання можна припустити? A teenager was hospitalized with complaints of limited movement in the lower limbs. The disease began 2 days ago with an increase in body temperature to 38.3^oC, loose stools 3-4 times per day. Objectively: body temperature - 36.8°C, no active movements in the affected area, muscle hypotonia. Meningeal symptoms are weakly positive?

Ботулізм Botulism

Травма поперекового відділу хребта Injury of the lumbar spine

Поліомієліт Polio

Cубарахноїдальний крововилив Cubarachnoid hemorrhage

Герпетичний менінгоенцефаліт Herpetic meningoencephalitis

1132 / 1500
Породілля 25 років госпіталізована з переймами впродовж 12 годин. Перейми тривають 25 секунд з інтервалом mbox3-4-7 хвилин, нерегулярні, різко болючі, біль поширюється від нижнього сегмента матки вгору. Передлежить голівка плода, притиснута до входу в малий таз, матка в гіпертонусі. Внутрішнє акушерське дослідження виявило: шийка матки згладжена, розкриття маткового вічка 3 см. Плодовий міхур цілий. Поставте попередній діагноз: A 25-year-old woman in labor was hospitalized with seizures for 12 hours. Seizures last 25 seconds with an interval of mbox3-4-7 minutes, are irregular, sharply painful, pain spreads from the lower segment The uterus is up. The head of the fetus is pressed against the entrance to the pelvis. The internal obstetric examination revealed: the cervix is ​​smooth, the opening of the uterine cavity is intact. Make a preliminary diagnosis:

Первинна слабкість пологової діяльності Primary weakness of labor activity

Дискоординована пологова діяльність Discoordinated birth activity

Вторинна слабкість пологової діяльності Secondary weakness of labor activity

Фізіологічний перебіг пологів Physiological course of childbirth

Дистоція шийки матки Cervical dystocia

1133 / 1500
Чоловік 52 років страждає на виражену задишку під час фізичного навантаження, непродуктивний кашель. Хворіє 8 місяців. Палить 30 років. Під час аускультації в легенях з обох боків вислуховуються крепітуючі хрипи - <<тріск целофану>>. ЧДД - 26/хв., SpO_2 - 92%. Під час спірометрії виявили: помірні порушення функції зовнішнього дихання за рестриктивним типом. Запідозрили інтер-стиційний процес. Який метод дослідження найбільш ефективно допоможе верифікувати діагноз та прогноз? A 52-year-old man suffers from pronounced shortness of breath during physical exertion, a non-productive cough. He has been ill for 8 months. He has been smoking for 30 years. During auscultation, crackles are heard in the lungs on both sides Wheezing - 26/min. During spirometry, we suspected an interstitial process forecast?

Рентгенографія органів грудної клітки X-ray of chest organs

Торакоскопія з біопсією Thoracoscopy with biopsy

Фібробронхоскопія Fibrobronchoscopy

Комп'ютерна томографія Computed tomography

Бактеріологічне дослідження харкоти-ння Bacteriological study of sputum

1134 / 1500
У новонародженого терміном гестації 31 тиждень спостерігаються гіпотонія та пригнічення свідомості. Гематокрит - 35%, а в загальному аналізі ліквору вия-влено підвищену кількість еритроцитів, білка та знижений вміст глюкози. Ці дані відповідають клінічній картині: A newborn with a gestational age of 31 weeks has hypotonia and depression of consciousness. Hematocrit is 35%, and in the general analysis of the cerebrospinal fluid, an increased number of erythrocytes, protein and a decreased content of glucose. These data correspond to the clinical picture:

Внутрішньоутробної інфекції Intrauterine infection

Сепсису Sepsis

Анемії Anemias

Менінгіту Meningitis

Внутрішньочерепного крововиливу Intracranial hemorrhage

1135 / 1500
Дівчинка віком 15 років скаржиться на затримку росту, відсутність менструацій і вторинних статевих ознак. Об'єктивно спостерігається: зріст - 153 см, антимонголоїдний розріз очей, широка шия, крилоподібні складки шиї, низька лінія росту волосся на шиї, плечовий пояс переважає над тазовим, молочні залози нерозвинені, відсутнє оволосіння на лобці. Виявлено гіпоплазію матки. Який найімовірніший діагноз? A 15-year-old girl complains of stunted growth, lack of menstruation and secondary sexual characteristics. Objectively observed: height - 153 cm, anti-Mongoloid cut of the eyes, wide neck, wing-like folds of the neck, low growth line of the neck, shoulder girdle prevails over the pelvic, mammary glands are absent, pubic hair is found. What is the most likely diagnosis?

Нейрофіброматоз Neurofibromatosis

Синдром Шерешевського-Тернера Shereshevsky-Turner syndrome

Нанізм Nanism

Гіпогеніталізм Hypogenitalism

Синдром Клайнфельтера Klinefelter syndrome

1136 / 1500
У чоловіка віком 42 роки на тлі психотравмуючих обставин (втрата роботи, хвороба дружини) поступово з'явилося безсоння, пригнічений настрій, зниження апетиту. Втратив інтерес до повсякденної діяльності, перестав справлятися з роботою, з’явилися думки про небажання жити. Які лікарські засоби потрібно йому призначити? A 42-year-old man, against the background of psycho-traumatic circumstances (loss of job, illness of his wife), gradually developed insomnia, depressed mood, decreased appetite. He lost interest in everyday activities , stopped coping with work, thoughts about unwillingness to live appeared. What medicines should he be prescribed?

Антипсихотики Antipsychotics

Снодійні Sedatives

Нейролептики Neuroleptics

Анксіолітики Anxiolytics

Антидепресанти Antidepressants

1137 / 1500
Чоловік 37 років скаржиться на гострий біль у лівій половині грудної клітки, задишку, що посилюється під час будь-яких рухів. Захворів раптово після значного фізичного навантаження. Під час обстеження встановлено: помірний ціаноз обличчя, ліва половина грудної клітки відстає в акті дихання. Перкуторно виявлено: зліва - тимпаніт, дихання ослаблене. ЧД - 24/хв. Тони серця ослаблені. ЧСС - 90/хв. На рентгенограмі видно лінія вісцеральної плеври. Назовні від неї легеневий малюнок відсутній. Який найімовірніший діагноз? A 37-year-old man complains of sharp pain in the left half of the chest, shortness of breath that worsens during any movement. He fell ill suddenly after significant physical exertion. During examination revealed: moderate cyanosis of the chest. Percussion revealed: heart rate - 24/min. Visceral pleura line she has no lung pattern. What is the most likely diagnosis?

Лівосторонній ексудативний плеврит Left-sided exudative pleurisy

Спонтанний пневмоторакс Spontaneous pneumothorax

Лівостороння пневмонія Left-sided pneumonia

Тромбоемболія легеневої артерії Thromboembolism of the pulmonary artery

Інфаркт міокарда Myocardial infarction

1138 / 1500
Виявлення яких структур під час патоморфологічного дослідження є патогномонічним для сказу? The detection of which structures during pathological examination is pathognomonic for rabies?

Тілець Гварнері Taurus Guarneri

Тілець Негрі Taurus Negri

Вузликів Ашофа Ashof's knots

Тілець Талалаєва Talalayev Taurus

Кристалів Шарко-Лейдена Charcot-Leyden crystals

1139 / 1500
Чоловік 30 років скаржиться на висип та свербіння шкіри стоп. Хворіє 3 роки. Об'єктивно в ділянці шкіри підошов спостерігаються групи пухирців, схожих на розварені сагові зерна, а також ерозії з уривками мацерованого епідермісу на периферії вогнищ. У міжпальцевих складках обох стоп видно тріщини, ерозії. Яка патологія найімовірніша? A 30-year-old man complains of a rash and itching of the skin of his feet. He has been ill for 3 years. Objectively, in the area of ​​the skin of the soles, groups of blisters similar to boiled sago grains are observed, and also erosions with fragments of macerated epidermis on the periphery of the foci. Cracks, erosions are visible in the interdigital folds of both feet. What is the most likely pathology?

Руброфітія стоп Rubrophyta of the feet

Дерматит Dermatitis

Псоріаз Psoriasis

Епідермофітія стоп Epidermophyta of the feet

Вторинний сифіліс Secondary syphilis

1140 / 1500
Пацієнта госпіталізовано до лікарні з діагнозом «трихінельоз». Вживання якого продукту стало причиною розвитку цього гельмінтозу? The patient was admitted to the hospital with a diagnosis of trichinellosis. What product was the cause of the development of this helminthiasis?

Яловичини Beef

Баранини Lambs

Кролячого м'яса Rabbit

Свинини Pork

М'яса птиці Poultry

1141 / 1500
Під час профілактичного огляду населення міста у 25% людей виявлено ознаки флюорозу: плямиста емаль зубів, у частини обстежених - генералізований остеосклероз із кальцифікацією міжхребетних зв'язок. Яка найімовірніша причина виникнення флюорозу? During a preventive examination of the city's population, 25% of people were found to have signs of fluorosis: mottled tooth enamel, some of the examined had generalized osteosclerosis with calcification of the intervertebral ligaments. What is the most likely the cause of fluorosis?

Надмірний вміст фтору в рослинних продуктах Excessive content of fluorine in vegetable products

Недостатній вміст фтору у грунті та воді Insufficient fluoride content in soil and water

Недостатній вміст фтору в тваринних продуктах Insufficient fluoride content in animal products

Недостатнє надходження фтору в організм із чаєм Insufficient intake of fluoride in the body with tea

Надмірний вміст фтору у грунті та воді Excessive fluoride content in soil and water

1142 / 1500
Хворий 52-х років звернувся до лікаря зі скаргами на задишку, кашель із мокротинням з домішками крові, підвищену пітливість вночі. В сімейному анамнезі хворих на рак легень немає. За даними картки профілактичних щеплень вакцину БЦЖ отримав вчасно. При фізикальному обстеженні температура - 37,2^oC, артеріальний тиск - 110/70 мм рт.ст., пульс - 100/хв., частота дихання - 20/хв., SpO_2 - 94% при кімнатному повітрі. При аускультації грудної клітки справа дихальні шуми не прослуховуються. На рентгенограмі ОГК правосторонній гідроторакс. Який наступний крок лікаря буде найбільш доречним? A 52-year-old patient consulted a doctor with complaints of shortness of breath, cough with blood-tinged sputum, increased sweating at night. There is no family history of lung cancer patients. According to the preventive vaccination card, he received the BCG vaccine on time. During the physical examination, the temperature was 37.2°C, the blood pressure was 110/70 mm Hg, the pulse was 100/min., the respiratory rate was 20/min., SpO_2 - 94% with room air. When auscultating the chest, no breathing sounds are heard. Right-sided hydrothorax will be the most appropriate step for the doctor?

Тест вивільнення гамма-інтерферону Interferon gamma release test

Проба Манту Mantoux Test

Бронхоскопія Bronchoscopy

УЗД легень Lung ultrasound

Діагностичний торакоцентез Diagnostic thoracentesis

1143 / 1500
У хворого частота дихання - 28/хв., тахікардія - до 100/хв., ознаки інтоксикації. В зоні правої лопатки притуплення перкуторного звуку, бронхіальне дихання, поодинокі дрібноміхурчасті та крепітуючі хрипи. Рентгенологічно: масивна запальна інфільтрація в середньому легеневому полі. Через три доби на фоні лікування, що проводилось, виник напад кашлю з виділенням 200 мл гнійної мокроти, після чого температура тіла знизилась до субфебрильної, стан покращився. На рівні кута лопатки на фоні інфільтрації легені виявлено округле просвітлення з горизонтальним рівнем. Встановіть діагноз: The patient's breathing rate is 28/min., tachycardia - up to 100/min., signs of intoxication. In the area of ​​the right scapula, dulling of the percussion sound, bronchial breathing, single X-ray: massive inflammatory infiltration in the middle lung field. After three days of treatment, a cough with 200 ml of purulent sputum appeared. The condition improved at the level of the scapula against the background of infiltration of the lung, a rounded illumination with a horizontal level was detected. Establish a diagnosis:

Емпієма плеври Empyema of the pleura

Гангрена правої легені Gangrene of the right lung

Правобічний бронхіт Right-sided bronchitis

Бронхоектатична хвороба Bronchoectatic disease

Абсцес легені Lung abscess

1144 / 1500
Чоловік 45-ти років, доставлений у відділення невідкладної допомоги зі скаргами на раптовий інтенсивний біль у поперековій ділянці, часте болісне сечовиділення, блювоту. Температура тіла - 36,8^oC, пульс - 82/хв., артеріальний тиск - 130/80 мм рт.ст. При фізикальному обстеженні болісність при пальпації поперекової ділянки, позитивний симптом Пастернацького. При лабораторному дослідженні у загальному аналізі сечі білок - 0,06 г/л, реакція - слабко кисла, лейкоцити - 3-4 в полі зору, еритроцити - 30-40 в полі зору. Який діагноз є найбільш імовірним? A 45-year-old man was brought to the emergency department with complaints of sudden intense pain in the lumbar region, frequent painful urination, vomiting. Body temperature - 36.8 ^oC, pulse - 82/min., blood pressure - 130/80 mmHg. During physical examination, pain during palpation of the lumbar area, positive Pasternaksky's symptom. In the laboratory examination, protein - 0.06 g/l, reaction - weakly acidic, leukocytes - 3-4 in the field of vision, erythrocytes - 30-40 in the field of vision. What diagnosis is most likely?

Гостра ниркова недостатність Acute renal failure

Гострий гломерулонефрит Acute glomerulonephritis

Гострий пієлонефрит Acute pyelonephritis

Полікістоз нирок Polycystic kidneys

Сечокам'яна хвороба Urolithiasis

1145 / 1500
До оториноларинголога звернувся хворий 29-ти років зі скаргами на зниження слуху. При аудіологічному обстеженні вия-влено порушення звукопроведення, кісткова провідність не порушена. Про порушення роботи яких структур органу слуху йде мова? A 29-year-old patient came to the otorhinolaryngologist with complaints of hearing loss. During the audiological examination, a sound conduction disturbance was detected, bone conduction was not disturbed. What structures are disturbed the organ of hearing is being talked about?

Системи півколових каналів Semicircular canal systems

Отолітового апарату Otolith apparatus

Основи завитки та довгастого мозку Fundamentals of gyrus and medulla oblongata

Барабанної перетинки, ланцюга слухових кісточок Tympanic membrane, chain of auditory ossicles

Спірального органу Spiral Organ

1146 / 1500
Чоловік 68 років скаржиться на тупий біль у череві, схуднення, слабкість, закрепи межують із рідким стільцем, багато темної крові у калі. Об'єктивно встановлено: шкіра землиста, суха. Під час пальпації живота в правій здухвинній ділянці - інфільтрат 6х9 см, який майже не зміщується. Hb крові - 68 г/л. Яка найімовірніша патологія може зумовлювати таку картину? A 68-year-old man complains of dull pain in the abdomen, weight loss, weakness, constipation bordering on liquid stool, a lot of dark blood in the stool. Objectively established: the skin is earthy , dry. During palpation of the abdomen, there is an infiltrate of 6x9 cm. Blood Hb is 68 g/l. What is the most likely cause of such a picture?

Пухлина сліпої кишки, кишкова кровотеча Cecal tumor, intestinal bleeding

Поліпоз ободової кишки, ускладнений кровотечею Polyposis of the colon complicated by bleeding

Неспецифічний виразковий коліт, ускладнений кровотечею Nonspecific ulcerative colitis complicated by bleeding

Дивертикулярна хвороба, ускладнена кровотечею Diverticular disease complicated by bleeding

Хвороба Крона, ускладнена кровотечею Crohn's disease complicated by bleeding

1147 / 1500
Хлопчик 9 років у тяжкому стані: температура тіла - 38-39^oC, носові кровотечі, біль у кістках. Об'єктивно спостерігається: різка блідість, геморагічний висип, виразково-некротичний стоматит. Збільшені всі групи лімфовуз-лів, печінка +5 см, селезінка +4 см. Яке дослідження є вирішальним для встановлення діагнозу? A 9-year-old boy is in serious condition: body temperature - 38-39^oC, nosebleeds, bone pain. Objectively observed: sharp pallor, hemorrhagic rash , ulcerative-necrotic stomatitis. All groups of lymph nodes are enlarged, the liver is +5 cm, the spleen is +4 cm. What research is decisive for establishing the diagnosis?

УЗД черевної порожнини Ultrasound of the abdominal cavity

Загальний аналіз крові General blood test

Мієлограма Myelogram

Імунологічний комплекс Immunological complex

Рентгенограма середостіння X-ray of the mediastinum

1148 / 1500
Пацієнтка 48 років із матковою кровотечею. В анамнезі вказано порушення менструального циклу впродовж року. Під час огляду шийки матки патології не виявлено. Бімануально встановлено: матка нормальних розмірів, неболюча, рухома, придатки без особливостей. Виділення кров'янисті, рясні. Що треба виконати на цьому етапі насамперед? A 48-year-old female patient with uterine bleeding. The anamnesis indicated a violation of the menstrual cycle for a year. During the examination of the cervix, no pathology was found. It was established bimanually: the uterus is of normal size, painless , mobile, appendages without features. Bloody discharge, abundant. What should be done at this stage first?

Гістеректомію Hysterectomy

Уведення окситоцину Oxytocin administration

Вишкрібання стінок матки Scraping the walls of the uterus

Уведення вітаміну К Vitamin K administration

Гормональну терапію Hormone therapy

1149 / 1500
За годину після годування немовляти молочною сумішшю у дитини виникли такі симптоми захворювання: ціаноз губ, слизових оболонок, нігтів, обличчя, потім приєдналася нудота, посилене слиновиділення, біль у надчеревній ділянці, блювання, пронос. Лікар-педіатр виявив у дитини ознаки легенево-серцевої недостатності. В процесі розслідування встановлено, що молочна суміш приготовлена на воді, взятій із колодязя. Який найімовірніший діагноз? An hour after feeding the baby with milk formula, the child developed the following symptoms of the disease: cyanosis of the lips, mucous membranes, nails, face, then nausea, increased salivation, pain in epigastric area, vomiting, diarrhea. The pediatrician found signs of pulmonary heart failure. During the investigation, it was established that the milk mixture was prepared on water taken from the well?

Нітратно-нітритна інтоксикація Nitrate-nitrite intoxication

Харчова токсикоінфекція Food poisoning

Харчове отруєння хлорорганічними пестицидами Food poisoning by organochlorine pesticides

Харчове отруєння важкими металами Food poisoning by heavy metals

Стафілококовий токсикоз Staphylococcal toxicosis

1150 / 1500
Пацієнт віком 58 років скаржиться на профузну макрогематурію з відходженням безформних згустків крові, странгурію. Макрогематурія, що не супроводжувалась болями та дизурією, вперше з'явилась 5 місяців тому без видимих причин, через декілька днів кровотеча самостійно припинилась. Який найімовірніший діагноз? A 58-year-old patient complains of profuse macrohematuria with the discharge of shapeless blood clots, stranguria. Macrohematuria, which was not accompanied by pain and dysuria, first appeared 5 months ago without for obvious reasons, after a few days the bleeding stopped on its own. What is the most likely diagnosis?

Конкремент сечового міхура Bladder calculus

Пухлина нирки Kidney tumor

Пухлина сечового міхура Bladder tumor

Гострий цистит Acute cystitis

Дивертикул сечового міхура Bladder diverticulum

1151 / 1500
У дитини віком 2 роки відзначаються часті та тривалі респіраторні захворювання та панкреатогенна форма мальабсорбції. Було припущено муковісцидоз. Яке дослідження необхідно провести для підтвердження цього діагнозу? A 2-year-old child has frequent and long-lasting respiratory diseases and a pancreatogenic form of malabsorption. Cystic fibrosis was suspected. What research should be done to confirm this diagnosis?

Бронхоскопію Bronchoscopy

Вміст хлоридів в потовій рідині Chloride content in sweat fluid

Каріотипування Karyotyping

Рентгенограму органів грудної клітки Rentgenogram of chest organs

Імунограму Immunogram

1152 / 1500
Дитині віком 12 років встановлено діагноз «двобічна пневмонія мікоплазмової етіології, легкий перебіг захворювання». Який з наведених препаратів потрібно призначити для лікування? A 12-year-old child was diagnosed with 'bilateral pneumonia of mycoplasma etiology, mild course of the disease'. Which of the following drugs should be prescribed for treatment?

Макроліди ІІ покоління Second generation macrolides

Амінопеніциліни Aminopenicillins

Протигрибкові Antifungal

Цефалоспорини І покоління I generation cephalosporins

Аміноглікозиди Aminoglycosides

1153 / 1500
Пацієнт віком 38 років скаржиться на кашель з виділенням гнійного мокротиння (до 60-80 мл на добу), підвищення температури тіла до 39^oС. Захворювання пов'язує з переохолодженням. Об'єктивно спостерігається: пульс - 96/хв, ритмічний. Артеріальний тиск - 110/60 мм рт. ст. Під час огляду виявлено: відставання правої половини в диханні. ЧД - 30/хв. Перкуторно локальне притуплення біля кута лопатки. Під час аускультації вислуховується: вологі різнокаліберні хрипи, амфоричне дихання. Який найімовірніший діагноз? A 38-year-old patient complains of a cough with the release of purulent sputum (up to 60-80 ml per day), an increase in body temperature to 39°C. The disease is associated with Objectively observed: pulse - 110/60 mm Hg. During the examination, the right side was found to be dull - 30/min . During auscultation: moist rales of various calibers, amphoric breathing are heard. What is the most likely diagnosis?

Гострий абсцес легені Acute lung abscess

Бронхіальна астма Bronchial asthma

Гострий бронхіт Acute bronchitis

Вогнищева пневмонія Focal pneumonia

Емпієма плеври Empyema of the pleura

1154 / 1500
Десятирічний хлопчик скаржиться на біль та припухлість колінних суглобів, ранкову скутість > 30 хвилин, субфебрильну температуру тіла, загальну слабкість. Під час огляду виявлено: колінні суглоби дефігуровані, теплі на дотик, порушена рухова активність уражених суглобів. Попередній діагноз: ювенільний ревматоїдний артрит. Вкажіть специфічний показник цього захворювання. A ten-year-old boy complains of pain and swelling of the knee joints, morning stiffness > 30 minutes, subfebrile body temperature, general weakness. During the examination, it was found: the knee joints are deformed, warm to the touch, impaired motor activity of the affected joints. Preliminary diagnosis: juvenile rheumatoid arthritis. Specify the specific indicator of this disease.

Підвищений вміст циркулюючих імунних комплексів (ЦІК) Increased content of circulating immune complexes (CIC)

Підвищення рівня ревматоїдного фактору Elevated rheumatoid factor

Диспротеїнемія, зниження вмісту альбумінів, гіперглобулінемія Dysproteinemia, decreased albumin content, hyperglobulinemia

Підвищення рівня С-реактивного протеїну Increased C-reactive protein

Підвищення антитіл до модифікованого цитрулінованого віментину (anti-MCV до SA-антигену) Elevation of antibodies to modified citrullinated vimentin (anti-MCV to SA-antigen)

1155 / 1500
Хлопчик 3 років доставлений до стаціонару в важкому стані. Об'єктивно: сомнолентність, гіперрефлексія, судоми, гіперестезія, невпинне блювання, температура тіла - 39,9^oC, ЧСС - 160/хв., АТ - 80/40 мм рт. ст. Яке дослідження слід провести насамперед? A 3-year-old boy was brought to the hospital in serious condition. Objectively: somnolence, hyperreflexia, convulsions, hyperesthesia, incessant vomiting, body temperature - 39.9^oC , heart rate - 160/min., blood pressure - 80/40 mm Hg. What research should be done first?

Рентгенографія черепа X-ray skull

Реоенцефалографія Rheoencephalography

Комп'ютерна томографія мозку Computed tomography of the brain

Люмбальна пункція Lumbar puncture

Ехоенцефалографія Echoencephalography

1156 / 1500
У двохрічного хлопчика, що хворіє на ГРВІ, на тлі підвищення температури тіла до 39,5^oС розвинувся напад генералізованих тоніко-клонічних судом із втратою свідомості, що тривав 3 хвилини. Після нападу дитина у свідомості, дещо сонлива. Під час неврологічного огляду патології не виявлено, нервово-психічний розвиток відповідає віку. Раніше подібних нападів, як і будь-яких порушень з боку ЦНС не було. Який найімовірніший варіант судомного синдрому має місце у дитини? A two-year-old boy suffering from SARS developed an attack of generalized tonic-clonic seizures with loss of consciousness against the background of an increase in body temperature to 39.5^oC 3 minutes. After the attack, the child is somewhat sleepy. During the neurological examination, the neuropsychological development corresponds to the age. There were no such attacks, as well as any disturbances from the central nervous system. What is the most likely variant of the convulsive syndrome does the child have?

Фебрильні судоми Febrile convulsions

Типові абсанси Typical absences

Симптоматична епілепсія Symptomatic epilepsy

Психогенний неепілептичний напад Psychogenic non-epileptic seizure

Спазмофілія Spasmophilia

1157 / 1500
Вагітна в термiнi 35 тижнiв, стан тяжкий. Скаржиться на головний бiль, порушення зору, миготiння <<мушок>> перед очима. Під час огляду виявлено: загальний набряк, АТ - 180/120 мм рт. ст. Раптово з'явилися фібрилярнi посмикування м'язiв обличчя, тонiчнi судоми. Дихання припинилося. За 1 хвилину дихання відновилося. З рота виділилася значна кiлькiсть пiни. Амнезiя. Протеінурія - 7 г/л. Який імовірний діагноз? She is 35 weeks pregnant, her condition is serious. She complains of a headache, impaired vision, flickering of <> in front of her eyes. During the examination, it was found: general swelling , BP - 180/120 mm Hg. Sudden twitching of the facial muscles. Breathing stopped. A large amount of foam was released from the mouth - 7 g/l. What is the probable diagnosis?

Епiлепсiя Epilepsy

Черепно-мозкова травма Traumatic brain injury

Передеклампсiя тяжкого ступеню Severe preeclampsia

Гiпертонiчний криз Hypertensive crisis

Еклампсiя Eclampsia

1158 / 1500
Хвора 42-х років скаржиться на наяв-ність болючого утворення у лівій сідниці, підвищення температури тіла до 38,2^oC. Тиждень тому завершила курс лікування з приводу поперекового радикуліту. У верхньо-зовнішньому квадранті лівої сідниці гіперемія, набряк. При пальпації - різко болюче ущільнення до 6 см в діаметрі з розм'якшенням у центрі. Ваш попередній діагноз: A 42-year-old patient complains of the presence of a painful lump in the left buttock, an increase in body temperature to 38.2^oC. A week ago, she completed a course of treatment for of lumbar radiculitis. In the upper-external quadrant of the left buttock, there is a sharp, painful induration up to 6 cm in diameter. Your previous diagnosis:

Післяін'єкційний абсцес Post-injection abscess

Підшкірний парапроктит Subcutaneous paraproctitis

Абсцедивний фурункул Abscess boil

Нагноєна атерома Suppurative atheroma

Карбункул сідниці Carbuncle of the buttock

1159 / 1500
Жінка скаржиться на набряк нижнiх кiнцiвок, ламкiсть нігтiв, сухiсть шкiри, зниження пам'ятi, слабкiсть. Мiсячнi вiдсутнi з моменту пологiв. Під час зовнiшнього огляду встановлено: гiпотрофiя статевих органiв та молочних залоз. З анамнезу вiдомо, що 2 роки тому пiд час пологiв виникла масивна кровотеча понад 2000 мл. Який імовірний дiагноз? A woman complains of swelling of the lower extremities, brittle nails, dry skin, memory loss, weakness. Menstruation has been absent since childbirth. During the external examination, it was found: hypotrophy genitals and mammary glands. It is known from the anamnesis that 2 years ago during childbirth there was a massive bleeding of more than 2000 ml. What is the probable diagnosis?

Серцево-судинна недостатнiсть Cardiovascular failure

Гiпотiреоз Hypothyroidism

Iнфантилiзм Infantilism

Синдром Шихана Sheehan Syndrome

Пiсляпологове ожирiння Postpartum obesity

1160 / 1500
Жінка 32 років протягом 2 років відзначає періодичний нападоподібний біль у правому підребер'ї, який знімався но-шпою. Біль не завжди пов'язаний із вживанням їжі, іноді з'являється під час хвилювання, супроводжується болем у серці, серцебиттям. Об'єктивно спостерігається: емоційно лабільна, під час пальпації живота виникає невелика болючість у ділянці жовчного міхура. Яка найімовірніша патологія зумовлює таку картину? A 32-year-old woman for 2 years notes periodic attack-like pain in the right hypochondrium, which was relieved by a no-spa. The pain is not always associated with eating, sometimes appears during excitement, is accompanied by pain in the heart, palpitations. It is observed objectively: emotionally labile, during palpation of the abdomen there is a slight tenderness in the area of ​​the gallbladder. What is the most likely pathology that causes this picture?

Хронічний холецистит Chronic cholecystitis

Дискінезія жовчовивідних шляхів Biliary tract dyskinesia

Хронічний холангіт Chronic cholangitis

Хронічний панкреатит Chronic pancreatitis

Дуоденіт Duodenitis

1161 / 1500
Пацієнт лікувався з приводу інфаркту міокарда. На 13 день відбулося посилення болю в грудній клітці, з'явилася задишка. Об'єктивно спостерігається: температура - 38,2^oС, пульс - 112/хв., ЧД - 26/хв., під правою лопаткою вислуховуються дрібнопухирчасті хрипи. На 15 день був діагностований правобічний ексудативний плеврит. У загальному аналізі крові спостерігається: лейкоцити - 8,9·10^9/л, еозинофіли - 8 %. ШОЕ - 24 мм/год. Яке ускладнення інфаркту міокарда виникло у пацієнта? The patient was treated for a myocardial infarction. On the 13th day, chest pain increased, shortness of breath appeared. Objectively observed: temperature - 38.2^ oC, pulse - 26/min, fine vesicular rales are heard under the right shoulder blade. Right-sided exudative pleurisy was diagnosed on the 15th day: leukocytes - 8.9·10^9/l, eosinophils - 8%. ESR - 24 mm/h. What complication of myocardial infarction did the patient have?

Серцева астма Cardiac Asthma

Тромбоемболія легеневої артерії Thromboembolism of the pulmonary artery

Синдром Дресслера Dressler syndrome

Повторний інфаркт міокарда Recurrent myocardial infarction

Пневмонія Pneumonia

1162 / 1500
Чоловік 62 років. Протягом останніх 3 років без видимих причин стала зростати безпорадність, зниженя пам'яті. У психічному стані встановлено: правильно називає своє прізвище, ім'я, по батькові, дату народження; вік сказати не може. Дезорієнтований у місцевості, у часі. Пам'ять знижена на поточні та віддалені події. Інтелект знижений. Порушені вищі коркові функції з явищами афазії, апраксії, агнозії. Критика відсутня. Визначіть провідний патогенетичний механізм формування описаного стану: The man is 62 years old. Over the past 3 years, for no apparent reason, helplessness and memory loss began to increase. In his mental state, it was established: correctly pronounces his last name, first name , date of birth.Disoriented in time.Decreased intelligence with aphasia, agnosia the mechanism of formation of the described state:

Гіперліпідемія Hyperlipidemia

Дефіцит норадреналіну Noradrenaline deficiency

Нейродегенеративний Neurodegenerative

Надлишок ацетилхолінтрансферази Acetylcholine transferase excess

Надлишок серотоніна в синаптичній щілині Excess serotonin in the synaptic cleft

1163 / 1500
У восьмимісячної дитини відмічається тонічне напруження мімічних м'язів, карпопедальний спазм, ларингоспазм. Після цього розвинулися генералізовані клонічні судоми з втратою свідомості на декілька хвилин. Позитивні симптоми Хвостека, Труссо, Люста. Який невідкладний стан спостерігається у дитини? An eight-month-old child has tonic tension of facial muscles, carpopedal spasm, laryngospasm. After that, generalized clonic convulsions with loss of consciousness for several minutes developed. Positive Khvostek symptoms, Trousseau, Lusta. What emergency condition is observed in the child?

Гіпокальціємічні (тетанічні) судоми Hypocalcemic (tetanic) convulsions

Афективно-респіраторні судоми Affective-respiratory convulsions

Фебрильні судоми Febrile convulsions

Епілептичний напад Epileptic seizure

Задухо-ціанотичний напад Suffocation-cyanotic attack

1164 / 1500
У приймальне відділення звернулися батьки з двомісячним хлопчиком, який напередодні ввечері впав із дивана на підлогу. Об'єктивно: дитина не контактує, млява, мали місце тоніко-клонічні судоми, в скроневій ділянці зліва напружена гематома. Яке із досліджень слід провести в першу чергу? Parents of a two-month-old boy who fell from the sofa to the floor the night before came to the reception department. Objectively: the child is unresponsive, lethargic, tonic-clonic seizures occurred convulsions, tense hematoma in the left temporal area. Which of the tests should be performed first?

Електроенцефалографія Electroencephalography

Комп'ютерна томограма голови Computer tomography of the head

Оглядова рентгенографія голови Overview X-ray of the head

Іонограма сироватки крові Ionogram of blood serum

Дослідження ліквору Liquor Research

1165 / 1500
Хлопчик 10-ти років звернувся в поліклініку зі скаргами на закладеність носу. Відомо, що подібні симптоми виникають періодично (весна, осінь). З анамнезу життя: атопічний дерматит. Батько дитини хворіє на бронхіальну астму. Об'єктивно: обличчя бліде, трохи набрякле. ЧД- 22/хв. Аускультативно: над легенями везикулярне дихання. Риноскопічно: слизова носової порожнини набрякла, бліда. Яке захворювання можна припустити? A 10-year-old boy came to the polyclinic with complaints of nasal congestion. It is known that similar symptoms occur periodically (spring, autumn). Life history: atopic dermatitis The child's father is suffering from bronchial asthma. Objectively, the face is slightly swollen. Auscultation: vesicular breathing. Nasal mucosa is pale.

Алергічний риніт Allergic rhinitis

Гострий аденоїдит Acute adenoiditis

Гострий гайморит Acute sinusitis

Рекурентне респіраторне захворювання Recurrent respiratory disease

Гострий риніт Acute rhinitis

1166 / 1500
Жінка 60-ти років надійшла до відділення невідкладної допомоги зі скаргами на сильний головний біль у потиличній ділянці, нудоту, блювання, запаморочення. При огляді хвора трохи загальмована, обличчя гіперемоване, артеріальний тиск - 220/130 мм рт.ст., пульс - 78/хв. При неврологічному обстеженні парезів не виявлено, рефлекси рівномірно жваві, м'язовий тонус не змінений, чутливість збережена. Менінгеальні симптоми відсутні. Який стан найбільш імовірно розвинувся у хворої? A 60-year-old woman came to the emergency department with complaints of a severe headache in the back of the head, nausea, vomiting, dizziness. On examination, the patient is slightly depressed, her face hyperemic, blood pressure - 220/130 mmHg, pulse - 78/min. No paresis was detected, muscle tone was not changed. Meningeal symptoms are most likely to have developed in the patient?

Гіпертонічний криз Hypertensive crisis

Транзиторне порушення мозкового кровообігу Transient cerebral circulation disorder

Пухлина головного мозку Brain tumor

Субарахноїдальний крововилив Subarachnoid hemorrhage

Ішемічний інсульт Ischemic stroke

1167 / 1500
У сім'ї вживались в їжу овочеві (зокрема грибні) та фруктові консерви домашнього приготування. За 8 днів після чергового прийманні їжі у двох членів сім'ї з'явилися скарги на слабкість, послаблення зору, двоїння в очах та косоокість. Трохи згодом виникло утруднення мови, порушення ковтання. Після звернення до лікарні пацієнтів госпіталізували. Яке харчове отруєння виникло у цьому разі? The family used to eat home-made canned vegetables (especially mushrooms) and fruit. 8 days after the next intake of food, two family members from complaints of weakness, diplopia, and strabismus appeared. A little later, difficulty in swallowing appeared. After going to the hospital, the patient was hospitalized. What kind of food poisoning occurred in this case?

Афлотоксикоз Aflotoxicosis

Ботулізм Botulism

Стафілококовий токсикоз Staphylococcal toxicosis

Сальмонельоз Salmonellosis

Бактеріальна харчова токсикоінфекція Bacterial food poisoning

1168 / 1500
Чоловік 45 років був доставлений машиною швидкої допомоги з підприємства, яке виготовляє анілін, з діагнозом: транзиторна ішемична атака. Об'єктивно встановлено: шкірні покриви та слизові оболонки синюшні. Мова дизартрична. Наявна дезорієнтація в просторі. У крові виявлено: еритроцити - 4,6·10^12/л, Нb - 143 г/л, КП - 0,9, лейкоцити - 5,6·10^9/л, тільця Гейнца - 14%, ретикулоцити - 18%, метгемоглобін - 36%, ШОЕ - 5 мм/год. Пацієнту встановлено діагноз: гостра інтоксикація аніліном середньої тяжкості. Який засіб антидотної терапії є найефективнішим в цьому разі? A 45-year-old man was brought by ambulance from an aniline manufacturing company with a diagnosis of transient ischemic attack. Objectively established: the skin and mucous membranes are bluish Disorientation in space was found: erythrocytes - 4.6·10^12/l, KP - 0.9, leukocytes - 5.6·10^1/l, Heinz bodies - 14%, reticulocytes - 36%, ESR - 5 mm/h. The patient is diagnosed with acute aniline intoxication. What is the most effective antidote therapy in this case?

Сукцимер Succimer

Натрія тіосульфат Sodium thiosulfate

Пентацин Pentacin

Десферал Desferal

Метиленовий синій Methylene Blue

1169 / 1500
У жінки 32 років спостерігаються ожиріння (переважно на плечах і тулубі), гірсутизм і порушення менструального циклу. На плечах, грудях, животі, стегнах наявні багряно-ціанотичні стрії, розтягнення шкіри. Яка причина появи стрій у цієї пацієнтки? A 32-year-old woman has obesity (mainly on the shoulders and trunk), hirsutism and irregular menstrual cycle. There are purple-cyanotic striae on the shoulders, chest, abdomen, thighs , stretching of the skin. What is the reason for the appearance of striae in this patient?

Гіпоестрогенемія Hypoestrogenemia

Катаболічна дія надлишку кортико-стероїдів Catabolic effect of excess corticosteroids

Гіпергонадотропінемія Hypergonadotropinemia

Інсулінорезистентність Insulin resistance

Гіперпродукція андрогенів Hyperproduction of androgens

1170 / 1500
Жінка 35-ти років звернулася до лікаря зі скаргами на зростаючий біль під час менструації протягом 1-го року. Тривалість менструального циклу 28 днів. 2 роки тому була проведена лапароскопічна перев'язка маткових труб. При пальпації матка збільшена до 8 тижнів вагітності, м'якої консистенції, болюча. Придатки з обох сторін не збільшені, при пальпації безболісні. Який діагноз є найбільш імовірним? A 35-year-old woman consulted a doctor with complaints of increasing pain during menstruation during the 1st year. The duration of the menstrual cycle is 28 days. 2 years ago she was laparoscopic tubal ligation. On palpation, the uterus is soft, painful. On palpation, the appendages are painless. What is the most likely diagnosis?

Аденоміоз Adenomyosis

Позаматкова вагітність Ectopic pregnancy

Гідросальпінкс Hydrosalpinx

Полікістоз яєчників Polycystic ovary

Зовнішній ендометріоз External endometriosis

1171 / 1500
Чоловік 56 років протягом тривалого часу страждає на цироз печінки з ознаками портальної гіпертензії. Відмічалися явища кровотечі з варикозно розширенних вен стравоходу. Під впливом терапії кровотечу було зупинено. Який із препаратів найбільш доцільно призначити для профілактики цього ускладнення? A 56-year-old man has been suffering from cirrhosis of the liver with signs of portal hypertension for a long time. Bleeding from varicose veins of the esophagus was noted. Under the influence of therapy, the bleeding was stopped. Which of the following which drugs are the most appropriate to prescribe for the prevention of this complication?

Пропранолол Propranolol

Вазопресин Vasopressin

Препарати заліза Iron preparations

Гепарін Heparin

Контрикал Contrical

1172 / 1500
П'ятирічна дитина скаржиться на нападоподiбний спазматичний кашель. З анамнезу відомо: хворiє 2 тижнi з появи сухого кашлю, після лiкування, що було неефективним, появилися репризи. Пiд час нападу кашлю обличчя дитини червонiє, шийнi вени набухають. Об'єктивно спостерігається: пацієнтка блiда, набрякле обличчя , крововиливи в склери. Аускультативно вислуховується: над легенями коробковий перкуторний звук, сухi хрипи. Під час рентгенологiчного дослідження виявлено: пiдвищення прозоростi легеневих полiв, посилення бронхiального малюнку. У аналізі кровi спостерігається: лейкоцити - 16·10^9/л, лiмфоцити - 72%, ШОЕ - 4 мм/год. Який найімовірніший діагноз? A five-year-old child complains of an attack-like spasmodic cough. It is known from the anamnesis: he has been sick for 2 weeks since the onset of a dry cough, after treatment, which was ineffective, relapses appeared. Under during a coughing attack, the child's face turns red, the neck veins swell. Objectively, the patient is pale, hemorrhagic in the sclera. Auscultation is heard: a percussive sound over the lungs. During the X-ray examination, an increase in the transparency of the lung fields is detected in the blood analysis: leukocytes - 16·10^9/l, lymphocytes - 72%, ESR - 4 mm/h. What is the most likely diagnosis?

Стороннє тiло в дихальних шляхах Foreign body in respiratory tract

Аденовiрусна iнфекцiя Adenovirus infection

Ларинготрахеїт Laryngotrachitis

Кашлюк Whooping cough

Туберкульозний бронхоаденiт Tuberculous bronchoadenitis

1173 / 1500
Пацієнтка віком 13 років скаржиться на біль у ділянці серця, серцебиття, відчуття жару, запаморочення, слабкість. З анамнезу відомо: скарги з'явилися 2 місяці тому, схудла на 4 кг, погіршилася успішність в школі, сон. Об'єктивно спостерігається: плаксива, дратівлива, тремор пальців рук, шкіра підвищеної вологості, тургор тканин знижений. ЧСС - 104/хв. Під час пальпації виявлено: щитовидна залоза збільшена до зоба ІІІ ступеня, ущільнена, неболюча. Який найімовірніший діагноз? A 13-year-old patient complains of heart pain, palpitations, a feeling of heat, dizziness, weakness. It is known from the anamnesis: the complaints appeared 2 months ago, she lost weight by 4 kg, performance at school has deteriorated. Objectively observed: tearful, irritable fingers, increased tissue turgor. Heart rate - 104/min. During palpation, the thyroid gland is enlarged to III degree , compacted, painless. What is the most likely diagnosis?

Ревматична хорея Rheumatic chorea

Дифузний токсичний зоб Diffuse toxic goiter

Неревматичний кардит Non-rheumatic carditis

Ендемічний зоб Endemic goiter

Вегето-судинна дисфункція Vegeto-vascular dysfunction

1174 / 1500
До гінеколога звернулася жінка 36-ти років зі скаргами на нерегулярні мізерні менструації впродовж останніх 6 місяців, посилений ріст волосся на обличчі та ''ог-рубіння'' голосу. В анамнезі два викидні, одні передчасні пологи. За даними УЗД матка та яєчники без особливостей. Яке обстеження доцільно провести для встановлення причини гормональних порушень? A 36-year-old woman came to the gynecologist with complaints of irregular scanty menstruation for the past 6 months, increased growth of facial hair and hoarse voice 'There is a history of two miscarriages, one premature birth. According to the ultrasound of the uterus and ovaries, it is advisable to conduct an examination to determine the cause of hormonal disorders.

Проведення гістероскопії Hysteroscopy

Каріотипування Karyotyping

Взяття біопсії ендометрію Endometrial biopsy

Проведення рентген-дослідження турецького сідла Carrying out an X-ray examination of the Turkish saddle

Гормональне обстеження Hormonal examination

1175 / 1500
У малюка 9 місяців спостерігається затримання розвитку зубів та подовжується строк зарощення тім'ячка, слабкість та пітнівість. Який вид гіповітамінозу може бути у малюка? A 9-month-old baby has a delay in the development of teeth and a prolonged period of hair growth, weakness and sweating. What kind of hypovitaminosis can a baby have?

Гіповітаміноз В_6 Hypovitaminosis B_6

Гіповітаміноз С Hypovitaminosis C

Гіповітаміноз А Hypovitaminosis A

Гіповітаміноз D Hypovitaminosis D

Гіповітаміноз В_1 Hypovitaminosis B_1

1176 / 1500
Чоловік 72-х років звернувся до лікаря зі скаргами на задишку, кашель, набряки гомілок, збільшення живота. Хворіє більше ніж 20 років на хронічне обструктивне захворювання легень (ХОЗЛ). Останні 3 роки є інвалідом II групи у зв'язку з патологією серця. При фізикальному обстеженні пульс - 92/хв, артеріальний тиск - 120/70 мм рт.ст., частота дихання - 24/хв. Шкірні покриви ціанотичні, набряки гомілок, при пальпації органів черевної порожнини позитивний симптом флуктуації. При аускультації акцент II тону над легеневою артерією, сухі хрипи над всією поверхнею легень. Який механізм розвитку змін з боку серця у цього хворого є найбільш імовірним? A 72-year-old man went to the doctor with complaints of shortness of breath, cough, swelling of the legs, an enlarged abdomen. He has been suffering from chronic obstructive pulmonary disease (COPD) for more than 20 years For the last 3 years, he has been disabled due to heart disease. On physical examination, the pulse is 92/min, the respiratory rate is 24/min, and the skin is edematous shins, during palpation of the abdominal organs, a positive symptom of fluctuation. During auscultation, the accent of the II tone over the pulmonary artery, dry rales over the entire surface of the lungs. What is the most likely mechanism of the development of changes on the part of this patient?

Вторинна легенева гіпертензія Secondary pulmonary hypertension

Рефлекс Китаєва Reflex Kitaev

Рефлекс Бейнбріджа Bainbridge Reflex

Кардіоваскулярний рефлекс Cardiovascular reflex

Дихальний рефлекс Respiratory reflex

1177 / 1500
Жінці віком 25 років у зв'язку з перенесеним бактеріальним тонзилітом призначено антибіотик, який вона не приймала. Через 2 тижні з'явилися набряки обличчя, олігурія, піднявся артеріальний тиск. Лабораторно виявлено: креатинін крові - 340 мкмоль/л, сечовина - 42 ммоль/л. Яке ускладнення розвинулося у пацієнтки? A 25-year-old woman was prescribed an antibiotic in connection with previous bacterial tonsillitis, which she did not take. After 2 weeks, facial swelling, oliguria, blood pressure rose blood pressure. Laboratory revealed: blood creatinine - 340 μmol/l, urea - 42 mmol/l. What complication developed in the patient?

Гострий гломерулонефрит, артеріальна гіпертензія Acute glomerulonephritis, arterial hypertension

Гострий гломерулонефрит, нефритичний синдром Acute glomerulonephritis, nephritic syndrome

Гострий гломерулонефрит, гостра ниркова недостатність Acute glomerulonephritis, acute renal failure

Хронічний гломерулонефрит, хронічна ниркова недостатність Chronic glomerulonephritis, chronic renal failure

Гострий гломерулонефрит, нефротичний синдром Acute glomerulonephritis, nephrotic syndrome

1178 / 1500
Хлопчику 15 років. Непокоїть періо-дичний головний біль, стомлюваність. При огляді дитина достатньо активна, психо-емоційний розвиток відповідає вікові, шкіра бліда, волога на дотик, з боку внутрішніх органів відхилень не виявлено. Артеріальний тиск - 120/80 мм рт.ст. Звертає на себе увагу надмірний розвиток жирової підшкірної клітковини, розподіленої рівномірно. Лікарем висловлене припущення про наявність у дитини ожиріння. Який показник першочергово повинен бути врахований для підтвердження діагнозу? The boy is 15 years old. He is concerned about periodic headaches, fatigue. During examination, the child is quite active, psycho-emotional development corresponds to his age, the skin is pale, moist to the touch, on the part of internal organs, no abnormalities were detected. Blood pressure - 120/80 mmHg. Attention is drawn to the excessive development of subcutaneous tissue. The doctor suggested that the child is obese ?

Відношення маси тіла до зросту Ratio of body weight to height

Товщина підшкірної жирової складки Thickness of the subcutaneous fat fold

Маса тіла Body weight

Індекс маси тіла Body mass index

Наявність ожиріння у членів родини Obesity in family members

1179 / 1500
Пацієнт віком 76 років скаржиться на утруднене сечовипускання вночі, до 3-х разів, відчуття неповного випорожнення сечового міхура. Хворіє протягом 2-х років. Об'єктивно спостерігається: після акту сечовипускання перкуторно над лоном визначається притуплення. Симптом Пастернацького негативний. Зовнішні статеві органи без особливостей. Під час дослідження через пряму кишку: передміхурова залоза збільшена у 2-3 рази, гладка, еластична, симетрична з чіткими контурами. Слизова кишечника над нею рухома. Який найімовірніший діагноз? A 76-year-old patient complains of difficulty urinating at night, up to 3 times, a feeling of incomplete emptying of the bladder. He has been ill for 2 years. Objectively observed : after urination, dullness is determined. Pasternaksky's symptom is negative. During the examination through the rectum: the prostate gland is enlarged by 2-3 times. The mucous membrane above it is mobile . What is the most likely diagnosis?

Абсцес передміхурової залози Prostate abscess

Туберкульоз простати Prostate tuberculosis

Аденома простати Prostate adenoma

Рак простати Prostate cancer

Хронічний простатит Chronic prostatitis

1180 / 1500
У жінки віком 29 років захворювання розпочалося раптово з підвищення температури тіла до 39^oС, появи на шкірі обличчя, чола, навколо очей, шиї та передньої поверхні грудної клітки висипань темно-бузкового кольору. Скелетні м'язи в'ялі, пальпаторно болючі, сила у них знижена. Відмічає затруднення ковтання. Лабораторно виявлено: різкопозитивні гострофазові показники, ШОЕ - 39 мл/год, еритроцити - 2,9·10^12, Нb - 72 г/л. Визначення рівня якого ферменту буде мати найважливіше значення для верифікації діагнозу? In a 29-year-old woman, the disease began suddenly with an increase in body temperature to 39^oC, the appearance of rashes on the skin of the face, forehead, around the eyes, neck and front surface of the chest dark purple rash. Skeletal muscles are weak, palpable, difficulty in swallowing. Laboratory findings: sharp positive ESR - 2.9·10^12, Hb - 72 g/l. Determination of the level of which enzyme will be most important for the verification of the diagnosis?

Трансамінази Transaminases

Мідьоксидази Copper oxidases

Креатинфосфокінази Creatine phosphokinase

Трансферину Transferin

Лужної фосфатази Alkaline phosphatase

1181 / 1500
Під час проходження допризивної комісії у хлопця 17 років виявлена артеріальна гіпертензія - АТ на руках 190/110 мм рт. ст. Скарг немає. Звертає на себе увагу непропорційна будова тіла - добре розвинений плечовий пояс і недорозвинені нижні кінцівки. Який метод дослідження, найвірогідніше, найбільш інфор-мативний для постановки діагнозу? During the pre-conscription commission, a 17-year-old boy was diagnosed with hypertension - blood pressure in the arms of 190/110 mm Hg. There are no complaints. The disproportionate structure attracts attention bodies - a well-developed shoulder girdle and underdeveloped lower limbs. Which research method is most likely to be the most informative for making a diagnosis?

УЗД нирок і наднирників Ultrasound of kidneys and adrenal glands

Визначення катехоламінів у сечі Determination of catecholamines in urine

Реносцинтіграфія Renoscintigraphy

Вимірювання АТ на нижніх кінцівках BP measurement on lower limbs

Доплер-сонографія судин Doppler sonography of vessels

1182 / 1500
Чоловік 48 років звернувся до лікаря зі скаргами на біль у попереку. Хворіє 3 дні після переохолодження. Після огляду лікар поставив діагноз: гострий попереково-крижовий радикуліт. Які ліки слід призначити хворому? A 48-year-old man went to the doctor with complaints of lower back pain. He has been sick for 3 days after hypothermia. After the examination, the doctor diagnosed acute lumbosacral sciatica. What medicines should be prescribed to the patient?

Десенсибілізуючі Desensitizing

Нестероїдні протизапальні засоби Nonsteroidal anti-inflammatory drugs

Вітаміни Vitamins

Кортикостероїди Corticosteroids

Антибактеріальні Antibacterial

1183 / 1500
Хлопчика 2-х років госпіталізовано з приводу зменшення маси тіла, нестійких випорожнень, анорексії, які з'явилися після введення в раціон манної каші (з 5 місяців). Дитина адинамічна, млява, шкіра бліда, суха, підшкірно-жировий шар відсутній. Живіт здутий, напружений. Під час перкусії у верхній частині живота тимпаніт, шум плеску, випорожнення пінисті, світлого кольору, смердючі. В копроцитограмі: нейтральний жир - багато. Який наступний крок у веденні пацієнта буде найбільш доречним? A 2-year-old boy was hospitalized due to a decrease in body weight, unstable stools, anorexia, which appeared after the introduction of semolina into the diet (from 5 months). The child is apathetic, the skin is pale, the subcutaneous fat layer is absent. During percussion in the upper part of the abdomen, there is foamy, foul-smelling stool. In the coprocytogram, there is a lot of fat What would be the most appropriate next step in patient management?

Рентгенографія органів черевної порожнини X-ray of abdominal organs

Розробка індивідуального плану харчування з великим вмістом клітковини Development of an individual meal plan with high fiber content

Визначення IgA до тканинної трансглютамінази Determination of IgA to tissue transglutaminase

Призначення антибіотиків широкого спектру дії Prescription of broad-spectrum antibiotics

Негайне оперативне втручання Immediate operative intervention

1184 / 1500
Протягом останніх трьох місяців пацієнтку віком 68 років турбує біль у серці тривалістю 10 хвилин, що виникає при найменшому фізичному навантаженні. Нітрогліцерином не користується через сильний головний біль. Неодноразово лікувалася з приводу ІХС, перенесла інфаркт міокарда. АТ періодично підвищується до 160/80 мм рт. ст. На ЕКГ спостерігаються рубцеві зміни ділянки задньої стінки лівого шлуночка. Аускультативно виявлено систолічний шум над аортою. Який діагноз найімовірніший? For the past three months, a 68-year-old patient has been troubled by heart pain lasting 10 minutes, which occurs with the slightest physical exertion. She does not use nitroglycerin because of severe headaches. She has been treated several times about myocardial infarction. Blood pressure rises to 160/80 mm Hg. On the ECG, a systolic murmur is detected over the aorta.

Стабільна стенокардія напруги ФК IV Stable angina pectoris IV

Аневризма аорти з розшаруванням Aortic aneurysm with dissection

Рецидивний інфаркт міокарда Recurrent myocardial infarction

Стабільна стенокардія напруги ФК ІІ Stable angina pectoris II

Нестабільна стенокардія Unstable angina

1185 / 1500
У дитини 4,5 років з вираженими ознаками зневоднення свідомість на рівні сопору, загальний ціаноз, токсичне дихання, анурія, АТ- 60/20 мм рт.ст., ембріокардія, відсутність пульсу на променевій артерії. Який болюс розчину NaCl 0,9% необхідно ввести на першому етапі невідкладної допомоги? A 4.5-year-old child with pronounced signs of dehydration has consciousness at the level of sedation, general cyanosis, toxic breathing, anuria, blood pressure - 60/20 mm Hg. , embryocardia, absence of a pulse on the radial artery. What bolus of NaCl solution 0.9% should be administered at the first stage of emergency care?

30 мл/кг 30 ml/kg

60 мл/кг 60 ml/kg

80 мл/кг 80 ml/kg

50 мл/кг 50 ml/kg

100 мл/кг 100 ml/kg

1186 / 1500
Хворий скаржиться на підвищення температури тіла до 39,4^oC, головний біль, блювання. Ригідність м'язів потилиці, симптом Керніга позитивний, вогнищевої симптоматики немає. Ліквор: цитоз - 19600 мкл, лімфоцити - 27%, нейтрофіли - 73%, білок - 6,3 г/л. Імовірний діагноз: The patient complains of an increase in body temperature up to 39.4^oC, headache, vomiting. Neck muscle stiffness, Kernig's symptom is positive, there are no focal symptoms. Liquor : cytosis - 19600 μl, lymphocytes - 27%, neutrophils - 73%, protein - 6.3 g/l. Probable diagnosis:

Субарахноїдальний крововилив Subarachnoid hemorrhage

Ентеровірусний менінгіт Enterovirus meningitis

Герпетичний менінгіт Herpetic meningitis

Менінгококовий менінгіт Meningococcal meningitis

Туберкульозний менінгіт Tuberculous meningitis

1187 / 1500
Під час дослідження згортувальної системи крові пацієнта перед операцією виявлено дефіцит VIII фактора - антигемофільного глобуліну А. Яке захворювання у пацієнта? During the examination of the patient's blood coagulation system before the operation, a deficiency of factor VIII - antihemophilic globulin A was detected. What disease does the patient have?

Гемофілія А Hemophilia A

Геморагічний васкуліт Hemorrhagic vasculitis

Гемофілія С Hemophilia C

Геморагічний ангіоматоз Hemorrhagic angiomatosis

Гемофілія В Hemophilia B

1188 / 1500
Хворий 34-х років перебуває на лікуванні в психіатричній лікарні з приводу загострення шизофренії. Об'єктивно: лежить в ліжку, рухливо загальмований, контакт відсутній. На запитання не відповідає. Поза одноманітна, гіпомімічний, наявний симптом ''хоботка'', воскова гнучкість м'язів, симптом ''повітряної подушки''. В такому стані перебуває близько тижня. Харчування парентеральне. Визначте наявний синдром розладу рухово-вольової сфери: A 34-year-old patient is being treated in a psychiatric hospital due to an exacerbation of schizophrenia. Objectively: he is lying in bed, immobilized, there is no contact. When asked, no responds. Monotonous, hypomimic, the present symptom of 'air cushion'.

Депресивний ступор Depressive stupor

Кататонічний ступор Catatonic stupor

Психогенний ступор Psychogenic stupor

Екзогенний ступор Exogenous stupor

Апатичний ступор Apathetic stupor

1189 / 1500
Під часрозслідування випадку масового отруєння з'ясувалося, що у автомеханіків, які проводили випробування дизельних двигунів у боксі майстерні, де була зіпсована витяжна вентиляція, наприкінці робочого дня з'явились такі симптоми: головний біль, нудота, блювання, шум у вухах, лабільність пульсу. Об'єктивно виявлено, що шкіра та слизові оболонки мають вишнево-червоний колір. Який токсичний чинник став причиною масового отруєння автомеханіків? During the investigation of the case of mass poisoning, it was found that at the end of the working day, auto mechanics, who were testing diesel engines in the workshop box, where the exhaust ventilation was damaged, had the following symptoms appeared: headache, nausea, vomiting, ringing in the ears, lability of the pulse. Objectively, it was found that the skin and mucous membranes have a cherry-red color. What toxic factor caused the mass poisoning of auto mechanics?

Оксид вуглецю Carbon monoxide

Діоксид вуглецю Carbon dioxide

Сірковуглець Carbon disulfide

Діоксид сірки Sulfur dioxide

Оксид азоту Nitrogen oxide

1190 / 1500
Жінка 46-ти років доставлена до відділення невідкладної допомоги зі скаргами на постійний, інтенсивний абдомінальний біль протягом 8 годин, нудоту та блювання. В анамнезі гіпертригліцеридемія, яку пацієнтка не лікувала. При фізикальному обстеженні температура тіла - 38^oC, артеріальний тиск - 100/60 мм рт.ст., пульс - 122/хв. Іктеричність кон'юнктив, при пальпації черевної стінки позитивний симптом флюктуації, позитивні симптоми Хвостека та Труссо. При лабораторному дослідженні в сироватці крові натрій - 142 ммоль/л, калій - 3,1 ммоль/л, бікарбонат - 32 ммоль/л, креа-тинін - 106 мкмоль/л, кальцій - 0,9 ммоль/л. Який перший крок у веденні пацієнтки буде найбільш доречним? A 46-year-old woman was brought to the emergency department with complaints of constant, intense abdominal pain for 8 hours, nausea and vomiting. She has a history of hypertriglyceridemia, which the patient does not During the physical examination, the body temperature was 38°C, the blood pressure was 100/60 mmHg, the conjunctiva was icteric, and the abdominal wall was palpated with positive symptoms of Khvostek laboratory research in blood serum sodium - 142 mmol/l, potassium - 3.1 mmol/l, bicarbonate - 32 mmol/l, creatinine - 106 μmol/l, calcium - 0.9 mmol/l. What is the first step in management of the patient will be the most appropriate?

Ендоскопічна ретроградна холангіопанкреатографія (ЕРХПГ) Endoscopic retrograde cholangiopancreatography (ERCP)

Апротиніну 10 000 АТрОд (антитрипсинових одиниць) внутрішньовенно Aprotinin 10,000 ATrOd (antitrypsin units) intravenously

Натрію хлориду 0,9%, кальцію глюконат та фентаніл внутрішньовенно Sodium chloride 0.9%, calcium gluconate and fentanyl intravenously

Визначення концентрації алкоголю в крові Determination of alcohol concentration in blood

Негайна літотрипсія Immediate lithotripsy

1191 / 1500
При вивченні середнього рівня та характеру різноманітності деяких лабораторних показників отримані такі дані: для загального білку крові - середнє квадратичне відхилення pm4 г/л, коефіцієнт варіації - 6%; для швидкість осідання еритроцитів відповідно pm2 мм/год, 23%. Яка з ознак, що вивчаються, є найбільш різноманітною? When studying the average level and nature of the diversity of some laboratory indicators, the following data were obtained: for total blood protein - mean square deviation of pm4 g/l, coefficient of variation - 6%; for the sedimentation rate of erythrocytes, respectively, pm2 mm/h, 23%. Which of the studied features is the most diverse?

Відмінності в різноманітності ознак відсутні There are no differences in the diversity of features

Швидкість осідання еритроцитів (ШОЕ) Erythrocyte Sedimentation Rate (ESR)

Для вивчення різноманітності потрібні додаткові дослідження More research is needed to study diversity

Для вивчення різноманітності потрібні додаткові розрахунки The study of diversity requires additional calculations

Загальний білок сироватки крові Total serum protein

1192 / 1500
Дитина народилася на 8 місяці вагітності. У неї встановлено: мікроцефалію, катаракту, ваду серця. Мати дитини на 2-му місяці вагітності хворіла: було нетривале підвищення температури тіла до 37,5^oС, збільшення лімфатичних вузлів та дрібноплямистий висип на обличчі, тулубі й кінцівках, який пройшов без залишкових явищ. Який найбільш імовірний попередній діагноз у дитини і матері? The child was born in the 8th month of pregnancy. She was diagnosed with: microcephaly, cataracts, heart defect. The child's mother was ill in the 2nd month of pregnancy: there was a short-term increase in body temperature up to 37.5°C, an increase in lymph nodes and a small papular rash on the face, trunk and limbs, which passed without residual phenomena. What is the most likely preliminary diagnosis for the child and the mother?

Краснуха Krasnukha

Токсоплазмоз Toxoplasmosis

Цитомегаловірусна інфекція Cytomegalovirus infection

Хламідійна інфекція Chlamydial infection

Герпетична інфекція Herpes infection

1193 / 1500
Жінка віком 64 роки хворіє на цукровий діабет 2 типу, отримує цукрознижуючу терапію: метформін по 850 мг 3 рази на добу. Глікемія натщесерце - 7,8 ммоль/л, через дві години після їжі - 10,5 ммоль/л, глікозильований гемоглобін - 8,7%. Під час огляду лікарем-офтальмологом було діагностовано діабетичну проліферативну ретинопатію сітківки обох очей. Яка подальша тактика лікування? A 64-year-old woman has type 2 diabetes, receives hypoglycemic therapy: metformin 850 mg 3 times a day. Fasting blood glucose - 7.8 mmol/l , two hours after a meal - 10.5 mmol/l, glycosylated hemoglobin - 8.7%. During the examination by an ophthalmologist, diabetic retinopathy of both eyes was diagnosed. What are the further treatment tactics?

Збільшити дозу метформіну до 3 500 мг на добу Increase metformin dose to 3500 mg per day

До цукрознижуючої терапії додати гліклазид 160 мг в день Add gliclazide 160 mg per day to hypoglycemic therapy

Перевести на інсулінотерапію Transfer to insulin therapy

Не змінювати терапію Do not change therapy

До цукрознижуючої терапії додати дапагліфлозин 10 мг в день Add dapagliflozin 10 mg per day to hypoglycemic therapy

1194 / 1500
У пацієнта віком 17 років об'єктивно спостерігається: ріст волосся на обличчі відсутній, гінекомастія, відзначається відкладення жиру на стегнах, високий голос. Пацієнт високого зросту за рахунок подовжених нижніх кінцівок при відносно короткому тулубі. Відзначається розумова відсталість. У букальному епітелії виявлено статевий хроматин. Який найімовірніший діагноз? A 17-year-old patient is objectively observed: there is no facial hair growth, gynecomastia, there is fat deposition on the thighs, a high voice. The patient is tall due to elongated lower limbs with a relatively short trunk. Mental retardation is noted. Sexual chromatin is found in the buccal epithelium. What is the most likely diagnosis?

Синдром Патау Patau syndrome

Синдром Клайнфельтера Klinefelter syndrome

Синдром Дауна Down Syndrome

Синдром Шерешевського-Тернера Shereshevsky-Turner syndrome

Синдром Едвардса Edwards Syndrome

1195 / 1500
У дитини 3 років під час обстеження виявлено: висока температура, інтоксикація, блідість шкіри, геморагічні та некротичні елементи висипу на шкірі та слизових оболонках. У загальному аналізі крові виявлено: панцитопенія, агранулоцитоз, ретикулоцити відсутні, ШОЕ значно збільшена. Який імовірний діагноз? A 3-year-old child during the examination revealed: high temperature, intoxication, pale skin, hemorrhagic and necrotic elements of a rash on the skin and mucous membranes. A general blood test revealed : pancytopenia, agranulocytosis, reticulocytes are absent, ESR is significantly increased. What is the probable diagnosis?

В_12-дефіцитна анемія B_12-deficiency anemia

Апластична анемія Aplastic anemia

Білково-дефіцитна анемія Protein deficiency anemia

Гемоглобінопатія Hemoglobinopathy

Залізодефіцитна анемія Iron deficiency anemia

1196 / 1500
Пацієнтка віком 23 роки скаржиться на тупий біль у правій та лівій поперекових ділянках. Під час об’єктивного обстеження в лівому підребер'ї та мезогастрії пальпується безболісна, горбиста нирка еластичної консистенції, права нирка не пальпується. Під час УЗД виявлено множинні чіткі порожнисті утвори з ехонегативним умістом у паренхімі правої нирки, розмірами від 1,2х1,7 см до 2х2,5 см, та аналогічні утвори у паренхімі лівої нирки розмірами 2,5х3,7 см, одна з яких локалізується у ділянці нижнього полюса і має розмір 8,3х11,7 см. Встановіть імовірний діагноз. A 23-year-old female patient complains of dull pain in the right and left lumbar regions. During the objective examination, a painless, lumpy kidney is palpated in the left hypochondrium and mesogastric area elastic consistency, the right kidney is not palpable. During ultrasound, multiple clear hollow formations with echonegative contents in the parenchyma of the right kidney, measuring from 1.2x1.7 cm to 2x2.5 cm, and similar formations in the parenchyma of the left kidney, measuring 2.5x3, were detected. 7 cm, one of which is localized in the area of ​​the lower pole and has a size of 8.3x11.7 cm. Establish a probable diagnosis.

Мультикістоз Multicystosis

Двобічний гідронефроз Bilateral hydronephrosis

Полікістоз нирок Polycystic kidney disease

Мегакалікоз Megacalicosis

Солітарні кісти нирок Solitary kidney cysts

1197 / 1500
У породіллі через 4 тижні після термінових пологів підвищилась температура тіла до 39^oC, з'явилися слабкість та біль у правій молочній залозі, озноб. Молочна залоза нагрубла, збільшена, чутлива при пальпації. Флуктуації у ділянці інфільтрату немає. В аналізі крові помірний лейкоцитоз. Який діагноз є найбільш імовірним? 4 weeks after the emergency delivery, the mother's body temperature rose to 39^oC, weakness and pain in the right mammary gland appeared, chills. The mammary gland thickened, enlarged, sensitive on palpation. There is no fluctuation in the infiltrate. In the blood analysis, what is the most likely diagnosis?

Лактостаз Lactostasis

Серозний мастит Serous mastitis

Гангренозний мастит Gangrenous mastitis

Мастопатія Mastopathy

Абсцедуючий мастит Abscessing mastitis

1198 / 1500
39-річна хвора жінка скаржиться на підвищення температури тіла до 37,8^oC протягом двох діб, часте сечовипускання, ниючий біль у поперековій ділянці. Зазначені симптоми виникли вперше після переохолодження. Під час фізикального обстеження спостерігається болючість пальпації в ділянці нирок. Результат загального аналізу сечі: pH - лужна, білок - 0,099 г/л, лейкоцити вкривають усе поле зору, еритроцити - 0 у полі зору, циліндри - 0 у полі зору. Результат загального аналізу крові: гемоглобін - 140 г/л, лейкоцити - 9,2·10^9/л, ШОЕ - 30 мм/год. Який препарат слід вибрати для проведення емпіричної антибактеріальної терапії? A 39-year-old sick woman complains of an increase in body temperature to 37.8^oC for two days, frequent urination, aching pain in the lumbar region. These symptoms appeared for the first time after hypothermia. During the physical examination, palpation is observed in the area of ​​the kidneys. The result of the general analysis of urine: alkaline, protein - 0.099 g/l, leukocytes cover the entire field of vision, erythrocytes - 0 in the field of vision. The result of a general blood test: hemoglobin - 140 g/l, leukocytes - 9.2·10^9/l, ESR - 30 mm/h. Which drug should be chosen for empirical antibacterial therapy?

Фосфоміцин Fosfomycin

Амікацин Amikacin

Триметоприм-сульфаметоксазол Trimethoprim-sulfamethoxazole

Амоксицилін Amoxicillin

Ципрофлоксацин Ciprofloxacin

1199 / 1500
Чоловік 45-ти років, звернувся до лікаря зі скаргами на дратівливість, підвищену втомленість, схуднення, серцебиття, перебої в роботі серця. При пальпації лівої долі щитоподібної залози пальпується утворення, щільно-еластичної консистенції, безболісне, яке при ковтанні зміщується разом з щитоподібною залозою. Позитивні очні симптоми, екзофтальм, порушення конвергенції. Артеріальний тиск - 135/80 мм рт.ст., пульс - 110/хв. При ультразвуковому дослідженні щитоподібної залози у лівій долі візуалізується гіперехогенне округле утворення розмірами 2х3 см, з чіткими контурами, однорідної структури. Який висновок лікаря після проведеного обстеження буде найбільш коректним? A 45-year-old man consulted a doctor with complaints of irritability, increased fatigue, weight loss, palpitations, interruptions in the work of the heart. When palpating the left lobe of the thyroid gland, palpable formation, of a dense and elastic consistency, which is displaced with the thyroid gland. Positive eye symptoms, violation of convergence. Arterial pressure - 135/80 mm Hg. During ultrasound examination of the thyroid gland In the left lobe, a 2x3 cm hyperechoic mass with clear contours and uniform structure is visualized. What conclusion of the doctor will be the most correct after the examination?

Дифузно-токсичний зоб Diffuse toxic goiter

Кіста щитоподібної залози Thyroid cyst

Рак щитоподібної залози Thyroid cancer

Вузол щитоподібної залози Thyroid nodule

Гострий тиреоїдит Acute thyroiditis

1200 / 1500
Жінка 32-х років скаржиться на посилення спастичного болю внизу живота після психоемоційного напруження. Випорожнення кишечника інтермітуючі: 2-3 випорожнення після пробудження чергуються із закрепами протягом 1-2 днів впродовж 6 місяців. Об'єктивно: маса тіла збережена, помірний біль під час пальпації сигмоподібної кишки. Hb- 130 г/л, лейкоцити - 5,2 Г/л, швидкість осідання еритроцитів - 9 мм/год. Ректороманоскопічне дослідження болісне через спастичний стан кишечника, його слизова оболонка не змінена. В просвіті кишечника багато слизу. Яке захворювання найбільш імовірне у даної пацієнтки? A 32-year-old woman complains of increased spastic pain in the lower abdomen after psycho-emotional stress. Intermittent bowel movements: 2-3 bowel movements after waking up alternate with constipation for 1-2 days for 6 months. Body weight is preserved, moderate pain during palpation of the sigmoid intestine. Hb - 5.2 G/l, erythrocyte sedimentation rate - 9 mm/h spastic condition of the intestine, its mucous membrane is not changed. There is a lot of mucus in the intestinal lumen. What disease is most likely in this patient?

Хвороба Крона Crohn's disease

Неспецифічний виразковий коліт Nonspecific ulcerative colitis

Синдром подразненого кишечника Irritable bowel syndrome

Синдром мальабсорбції Malabsorption syndrome

Гостра ішемія кишківника Acute intestinal ischemia

1201 / 1500
Жінка 22 років із підозрою на системний червоний вовчак скаржиться на <<летючий>> біль у суглобах рук і ніг, підвищення температури до 38,5-39^oС упродовж 3 тижнів, задишку, серцебиття, слабкість. Об'єктивно встановлено: еритема на щоках та носі. У крові виявлено: Нb - 90 г/л, тромбоцити - 135·10^9/л, ШОЕ - 43 мм/год. У сечі виявлено: білок - 2,66 г/л, еритроцитів - 8-10 в п/з. Виявлення яких антитіл у пацієнтки найбільш значиме для постановки діагнозу? A 22-year-old woman with suspicion of systemic lupus erythematosus complains of <> pain in the joints of her hands and feet, an increase in temperature to 38.5-39^oС for 3 weeks, shortness of breath, weakness. Objectively established: erythema on the cheeks and nose. Blood - 90 g/l, platelets - 43 mm/h urine was found: protein - 2.66 g/l, erythrocytes - 8-10 in p/z. Which antibodies are most significant for the diagnosis in the patient?

Кріоглобуліни Cryoglobulins

До фосфоліпідів To phospholipids

Ревматоїдний фактор Rheumatoid factor

До тромбоцитів To platelets

До 2-х спіральної нативної ДНК Up to 2-helix native DNA

1202 / 1500
Чоловік 35 років надійшов до клініки з явищами хірургічного сепсису, джерелом якого, ймовірно, був великий карбункул лопаткової ділянки. Під час обстеження виявлено вторинні гнійні вогнища в печінці і правій легені. Укажіть фазу хірургічного сепсису: A 35-year-old man came to the clinic with symptoms of surgical sepsis, the source of which was probably a large carbuncle of the scapula. During the examination, secondary purulent foci were found in the liver and right lungs. Specify the phase of surgical sepsis:

Термінальна Terminal

Септицемія Septicemia

Токсемія Toxemia

Гнійно-резорбтивна лихоманка Suppurative resorptive fever

Септикопіемія Septicopemia

1203 / 1500
Хворий 55-ти років скаржиться на біль у грудині, поперековому відділі хребта, ребрах. Анамнестично-патологічний перелом кісток правої гомілки. У крові: загальний білок - 110 г/л, позитивний М-градієнт. У сечі: білок Бенс-Джонса. Ваш діагноз: A 55-year-old patient complains of pain in the sternum, lumbar spine, ribs. Anamnestic pathological fracture of the bones of the right tibia. Blood: total protein - 110 g /l, positive M-gradient. In urine: Bence-Jones protein. Your diagnosis:

Мієломна хвороба Myeloma

Остеохондроз Osteochondrosis

Невралгія Neuralgia

Стенокардія напруги 2 ФК Angina of voltage 2 FC

Гломерулонефрит Glomerulonephritis

1204 / 1500
До лікаря звернувся чоловік зі скаргами на біль у нижній щелепі. Об'єктивно спостерігається: маса тіла знижена, шкірні покриви бліді, неприємний запах з рота, ясна кровоточать. На рентгенограмі щелепи виявлено явища остеопорозу. Пацієнт протягом 30 років працював на хімічному підприємстві. Для якого патологічного стану характерні ці симптоми? A man went to the doctor with complaints of pain in the lower jaw. Objectively observed: body weight is reduced, skin is pale, bad breath, gums are bleeding. An x-ray of the jaw revealed osteoporosis. The patient worked at a chemical plant for 30 years. What pathological condition are these symptoms characteristic of?

Хронічного остеомієліту нижньої щелепи Chronic osteomyelitis of the lower jaw

Отруєння метилртуттю Methylmercury poisoning

Отруєння перхлоратом калію Poisoning with potassium perchlorate

Флюорозу Fluorosis

Хронічного отруєння фосфором Chronic phosphorus poisoning

1205 / 1500
У хворої 34-х років сильний біль глибоко в орбіті, головний біль, підвищення температури тіла. Захворювання виникло гостро. Об'єктивно: повіки набряклі, шкіра червоного кольору, екзофтальм, обмеження рухів очного яблука. Ваш діагноз: A 34-year-old patient has severe pain deep in the orbit, headache, increased body temperature. The disease occurred acutely. Objectively: the eyelids are swollen, the skin is red , exophthalmos, restriction of eyeball movements. Your diagnosis:

Флегмона орбіти Phlegmon of the orbit

Абсцес повіки Eyelid abscess

Виразка рогівки Corneal ulcer

Блефарит Blepharitis

Ретробульбарний неврит Retrobulbar neuritis

1206 / 1500
Робітник цвяхового цеху 35 років, стаж на цьому виробництві - 10 років, працює в умовах високочастотного інтенсивного шуму. На черговому періодичному огляді поставлено діагноз: <<професійна тугоухість>>. Що є підставою для такого діагнозу? A 35-year-old nail workshop worker, 10 years of experience in this industry, works in conditions of high-frequency intense noise. At the next periodic examination, the diagnosis was made: < >. What is the basis for such a diagnosis?

Характеристика шуму на цьому виробництві Noise characteristic of this production

Результати дослідження показників ЦНС Results of research of central nervous system indicators

Результати дослідження стану внутрішнього вуха Results of the study of the condition of the inner ear

Стаж роботи на цьому виробництві Work experience at this production

Дані аудіометрії та гігієнічна оцінка умов праці Audiometry data and hygienic assessment of working conditions

1207 / 1500
До сімейної лікарки звернулася жінка 54 років для профілактичного огляду. Зріст - 164 см, вага - 84 кг, АТ - 130/80 мм рт. ст. Менопауза 5 років. Мати пацієнтки померла від раку молочної залози, молодша сестра страждає на мастопатію. Під час огляду ущільнень у молочних залозах не виявлено, органи малого тазу в межах вікової норми. Цитологія шийки матки без особливостей. Що порекомендувати жінці? A 54-year-old woman came to the family doctor for a preventive examination. Height - 164 cm, weight - 84 kg, blood pressure - 130/80 mm Hg. Menopause 5 The patient's mother died of breast cancer, no lumps were found in the mammary glands. Cervical cytology was normal for the woman.

Пройти МРТ молочних залоз Pass MRI of mammary glands

Прийти на наступний профогляд за 2 роки Come to the next professional examination in 2 years

Регулярно проводити самообстеження молочних залоз Regular self-examination of the mammary glands

Приходити на огляд кожних 3 місяці Come for a review every 3 months

Проходити маммографію 1 раз на рік Get a mammogram once a year

1208 / 1500
Юнак 17 років вимагає зробити йому пластичну операцію. Вважає, що з таким носом, як у нього, неможливо жити, де б він не з'явився, усі з нього сміються, кепкують за його спиною. Ходить, низько насунувши кашкета, низько опускає голову, до очей замотується шарфом. Об'єктивних підстав для ринопластики немає, ніс у хлопця майже класичної форми. Визначте психопатологічний стан: A 17-year-old boy demands plastic surgery. He believes that it is impossible to live with a nose like his, no matter where it appears, everyone with they laugh at him behind his back. He walks with his cap down, wraps his head with a scarf. There are no objective reasons for rhinoplasty. Define a psychopathological condition:

Дисморфоманія Dysmorphomania

Нав'язливі думки Intrusive thoughts

Іпохондричний невроз Hypochondriac neurosis

Синдром Капгра Capgra syndrome

Порушення схеми тіла Body schema violation

1209 / 1500
Чоловік 61 року звернувся до дільничного лікаря зі скаргами на напади стиснення за грудиною, які виникають під час ходьби до 200 м та зникають, якщо зупинитися. Уважає себе хворим близько року. Межі серця в нормі, тони помірно приглушені, ЧСС=Ps=76/хв., АТ - 130/80 мм рт. ст. Який найбільш імовириний діагноз у хворого? A 61-year-old man turned to the district doctor with complaints of chest compression attacks that occur while walking up to 200 m and disappear when he stops. He considers himself ill year. Heart rates are normal, heart rate=76/min., BP - 130/80 mmHg. What is the most likely diagnosis for the patient?

Хронічне обструктивне захворювання легень Chronic obstructive pulmonary disease

ІХС: Інфаркт міокарда без елевації сегменту ST CHD: Myocardial infarction without ST segment elevation

ІХС: Нестабільна стенокардія CHD: Unstable angina

Тривожний розлад Anxiety disorder

ІХС: Стабільна стенокардія CHD: Stable angina

1210 / 1500
У породіллі на 12-ту добу післяпологового періоду раптово підвищилася температура тіла до 38,2^oС. Спостерігає-ться загальна слабкість, біль в ділянці правої молочної залози протягом 1 доби. Під час огляду встановлено: молочна залоза напружена, гаряча, у правому верхньому квадранті пальпується інфі-льтрат, болючий, щільної консистенції. Який діаг-ноз найбільш імовірний? On the 12th day of the postpartum period, the mother's body temperature suddenly rose to 38.2°C. There is general weakness, pain in the area of ​​the right mammary gland during 1 day. During the examination, the mammary gland is tense, hot, and the infiltrate is palpable in the right upper quadrant. What is the most likely diagnosis?

Аномалії розвитку молочних залоз Mammary gland development anomalies

Пухлина молочної залози Breast tumor

Лактостаз Lactostasis

Лактаційний мастит Lactation mastitis

Мастопатія Mastopathy

1211 / 1500
У дівчинки з тетрадою Фалло, після психоемоційного збудження виник задухо-ціанотичний напад. У цьому стані мати з дитиною звернулася до педіатра, який проводив прийом у дитячій поліклініці. Який препарат невідкладної допомоги необхідно ввести першим? A girl with tetrad of Fallot, after psycho-emotional excitement, had a suffocating-cyanotic attack. In this state, the mother and the child turned to the pediatrician, who held an appointment at the children's clinic. What should an emergency drug be administered first?

Калію хлорид Potassium chloride

Папаверін Papaverine

Провести інтубацію трахеї і дати кисень Intubate the trachea and give oxygen

Дигоксін Digoxin

Пропранолол Propranolol

1212 / 1500
У трирічної дитини спостерігаються напади, що супроводжуються ціанозом, різким занепокоєнням, присіданням навпочіпки. Об'єктивно виявлено: деформація фаланг пальців у вигляді 'барабанних паличок', нігті у формі 'скелець годинника'. Межі серцевої тупості зсунуті вліво та вправо, у ІІ міжребер'ї біля лівого краю грудини визначається систолічне дрижання, вислухується грубий систолічний шум з р.max. у ІІ міжребер'ї, ІІ тон над основою серця ослаблений. Під час рентгенологічного дослідження виявлено: серце у вигляді 'дерев'яного черевика', легеневий малюнок виражений слабо. Який найімовірніший діагноз? A three-year-old child has seizures accompanied by cyanosis, sharp anxiety, squatting. Objectively revealed: deformation of the phalanges of the fingers in the form of 'drumsticks', nails in The borders of the heart are shifted to the left and to the right, a systolic murmur is heard near the left edge of the sternum, and the second sound over the base of the heart is weakened X-ray examination revealed: the heart in the form of a 'wooden shoe', the pulmonary pattern is weak. What is the most likely diagnosis?

Тетрада Фалло Tetrad of Fallot

Первинний бактеріальний ендокардит Primary bacterial endocarditis

Дилатаційна кардіоміопатія Dilated cardiomyopathy

Дефект міжшлуночкової перетинки Defect of interventricular membrane

Дефект міжпередсердної перетинки Atrial membrane defect

1213 / 1500
У хірургічному відділенні лікується жінка віком 65 років із приводу флегмони сідничної ділянки. Під час мікробіологічного дослідження рани збудником хірургічної інфекції виявлені неклостридіальні анаероби - протей та бактероїди. Яку лікувальну тактику треба застосувати насамперед у лікуванні цієї інфекції? In the surgical department, a 65-year-old woman is being treated for phlegmon of the buttocks. During the microbiological examination of the wound, non-clostridial anaerobes - proteus and bacteroids - were found to be the causative agent of the surgical infection. What are the treatment tactics should be used primarily in the treatment of this infection?

Застосування протигангренозної сиворотки Application of anti-gangrenous serum

Антибіотикотерапію + профілактичне щеплення Antibiotic therapy + preventive vaccination

Промивання гнійної порожнини антисептиками Washing of purulent cavity with antiseptics

Антибактеріальну терапію Antibacterial therapy

Радикальне висічення уражених тканин Radical excision of affected tissues

1214 / 1500
Жінка 49-ти років звернулася до лікаря зі скаргами на головний біль, припливи жару до голови, шиї, підвищену пітливість, серцебиття, підвищення артеріального тиску до 170/100 мм рт.ст., дратівливість, безсоння, плаксивість, послаблення пам'яті, рідкі мізерні менструації, збільшення маси тіла на 5 кг протягом останніх півроку. Який діагноз є найбільш імовірним? A 49-year-old woman consulted a doctor with complaints of headache, hot flushes to the head, neck, increased sweating, palpitations, blood pressure increase to 170/100 mmHg, irritability, insomnia, tearfulness, memory loss, scanty menstruation, weight gain of 5 kg during the last six months. What is the most likely diagnosis?

Клімактеричний синдром Climacteric syndrome

Соматоформний розлад Somatoform disorder

Передменструальний синдром Premenstrual syndrome

Посткастраційний синдром Post-castration syndrome

Артеріальна гіпертензія Hypertension

1215 / 1500
Чоловіка 63 років госпіталізовано зі скаргами на біль у поперековій ділянці ліворуч, погіршення апетиту, слабкість, періодичну появу крові в сечі протягом місяця. Шкіра бліда. Анемія: еритроцити - 3,1·10^12/л, Нb - 101 г/л, ШОЕ - 37 мм/год., протеїнурія - 0,37 г/л, гематурія на все поле зору, креатінемія - 0,270 ммоль/л. Яке діагностичне припущення? A 63-year-old man was hospitalized with complaints of pain in the lumbar region on the left, loss of appetite, weakness, periodic appearance of blood in the urine for a month. The skin is pale. Anemia: erythrocytes - 3.1·10^12/l, Hb - 101 g/l, ESR - 37 mm/h, proteinuria - 0.37 g/l, hematuria in the entire field of vision, creatininemia - 0.270 mmol/l. What is the diagnostic assumption ?

Новоутворення нирки Kidney Neoplasm

Гострий пієлонефрит Acute pyelonephritis

Гострий гломерулонефрит Acute glomerulonephritis

Амілоїдоз нирок Kidney amyloidosis

Інфекційна токсична нефропатія Infectious toxic nephropathy

1216 / 1500
У хворого 40-ка років ранковий кашель з виділенням слизово-гнійного харкотиння, підвищення температури тіла до 37,6^oC. Тютюнопаління з 17-ти років. Об'єктивно: при аускультації в легенях жорстке дихання, розсіяні сухі хрипи. ЗАК: Л- 12 Г/л, ШОЕ- 18 мм/год. При бронхоскопії: катарально-гнійний ендобронхіт. Поставте діагноз: A 40-year-old patient has a morning cough with mucus-purulent sputum discharge, an increase in body temperature to 37.6^oC. He has been smoking since the age of 17. About Objectively: during auscultation in the lungs, scattered dry wheezes. АК - 12 G/l. At bronchoscopy: make a diagnosis of purulent endobronchitis.

Позагоспітальна пневмонія Community-acquired pneumonia

Рак легень Lung cancer

Бронхіальна астма Bronchial asthma

Туберкульоз легень Pulmonary tuberculosis

Хронічний бронхіт Chronic bronchitis

1217 / 1500
У пацієнта віком 48 років, який хворіє на артеріальну гіпертензію, раптово виникло запаморочення, бiль у потиличнiй дiлянцi, нудота та світлобоязнь. Об'єктивно спостерігається: сопор, гiперемiя обличчя, АТ - 190/100 мм рт. ст., пульс - 70/хв, температура тiла 36,8^oС. Горизонтальний нiстагм. Сухожильнi рефлекси - без чіткої різниці сторін, посилені. Ригiднiсть потиличних м'язiв - 4 поперечні пальці, двобiчний симптом Кернiга (+). Який попереднiй дiагноз? A 48-year-old patient suffering from arterial hypertension suddenly developed dizziness, pain in the occipital region, nausea and photophobia. Objectively observed: sopor, hyperemia face, blood pressure - 190/100 mmHg, body temperature 36.8°C. bilateral Kernig sign (+). What is the previous diagnosis?

Інсульт-гематома гіпертензивна Stroke-hematoma hypertensive

Субдуральна гематома Subdural hematoma

Субарахноїдальний крововилив Subarachnoid hemorrhage

Абсцес головного мозку Brain abscess

Гостра гiпертонiчна енцефалопатiя Acute hypertensive encephalopathy

1218 / 1500
Жінка 25 років госпіталізована до гінекологічного відділення зі скаргами на біль унизу живота, підвищення температури до 39,7^oC. Об'єктивно спостерігається: АТ - 120/80 мм рт. ст., пульс - 108/хв., задовільного наповнення і напруги. Живіт помірно здутий, різко болючий у нижніх відділах. Симптом Щоткіна-Блюмберга позитивний в гіпогастральній ділянці. Піхвове обстеження виявило: матка та придатки не пальпуються через напруження передньої черевної стінки. Заднє склепіння піхви нависає, різко болюче. Який діагноз найімовірніший? A 25-year-old woman was hospitalized in the gynecology department with complaints of pain in the lower abdomen, an increase in temperature to 39.7^oC. Objectively observed: blood pressure - 120/80 mmHg, pulse - 108/min. Abdomen is moderately bloated, sharply painful in the hypogastric area. Vaginal examination revealed that the uterus and appendages are not palpable walls. The posterior vault of the vagina hangs down, sharply painful. What is the most likely diagnosis?

Гострий ендометрит Acute endometritis

Пельвіоперитоніт Pelvioperitonitis

Позаматкова вагітність Ectopic pregnancy

Апоплексія яєчника Ovarian apoplexy

Гострий аднексит Acute adnexitis

1219 / 1500
Під час огляду потерпілого після дорожньо-транспортної пригоди спостерігаються ціаноз, утруднене дихання. Стан його тяжкий, права половина грудної клітки відстає в акті дихання, міжреберні проміжки розширені справа, під час перкусії спостерігається коробковий звук, дихання під час аускультації не вислуховується. Який діагноз найімовірніший? During the examination of the victim after the traffic accident, cyanosis and difficulty breathing are observed. His condition is serious, the right half of the chest lags behind in the act of breathing, the intercostal spaces are widened on the right, a box sound is observed during percussion, breathing is not heard during auscultation. What is the most likely diagnosis?

Тотальний гемоторакс справа Total hemothorax on the right

Гострий гнійний плеврит Acute purulent pleurisy

Клапанний пневмоторакс Valvular pneumothorax

Відкритий пневмоторакс Open pneumothorax

Пневмоперитонеум Pneumoperitoneum

1220 / 1500
Першовагітна з'явилась в жіночу консультацію в 37 тижнів вагітності. Скарг немає. Протягом останніх 2 тижнів набрала 2 кг ваги. Визначаються набряки ніг. АТ - 120/70 мм рт. ст. Білок у сечі - 0,8 г/л. Поставлено діагноз: пре-екламсія легкого ступеня. Яка лікувальна тактика? A first-time pregnant woman came to the antenatal clinic at 37 weeks of pregnancy. There are no complaints. She has gained 2 kg of weight over the past 2 weeks. Swelling of the legs is detected. BP - 120/70 mm Hg. Protein in urine - 0.8 g/l. Diagnosed: mild pre-eclampsia?

Термінове розродження Urgent delivery

Пролонгування вагітності Prolongation of pregnancy

Амбулаторне лікування Outpatient treatment

Стаціонарне лікування Inpatient treatment

Кесарський розтин Caesarean section

1221 / 1500
Жінка 35-ти років надійшла до відділення інтенсивної терапії з нападами судом з періодичністю 2-3 хвилини, між якими не відбувається покращення свідомості, реакція зіниць на світло відсутня. З дитинства хворіє на епілепсію. Зазвичай напади виникають 1-2 рази на місяць у нічний час та супроводжуються мимовільним сечовиділенням та дефекацією. Почастішання нападів та вищевказані ускладнення виникли на тлі перенесеного грипу. Який стан найбільш імовірно розвинувся у хворої? A 35-year-old woman came to the intensive care unit with convulsions every 2-3 minutes, between which there is no improvement in consciousness, and the reaction of the pupils to light is absent. She has been suffering from epilepsy since childhood. Usually, seizures occur 1-2 times a month and are accompanied by involuntary urination and defecation.

Епілептичний статус Status epilepticus

Істеричний невроз Hysterical neurosis

Епілептичний психоз Epileptic psychosis

Гіпокальціємічний криз Hypocalcemic crisis

- -

1222 / 1500
Сім'я звернулася за прогнозом потомства у зв'язку з народженням дитини з тяжким дефектом закриття невральної трубки. Дитина померла у неонатальному періоді. Ваша порада родині: The family applied for the prognosis of the offspring due to the birth of a child with a severe neural tube closure defect. The child died in the neonatal period. Your advice to the family:

Планувати вагітність, оскільки це більше не повториться Plan pregnancy because it won't happen again

Відмовитися від народження дітей Refuse to have children

У разі вагітності провести біопсію хоріону, кордоцентез In case of pregnancy, conduct a chorionic biopsy, cordocentesis

Провести цитогенетичне обстеження батьків Conduct cytogenetic examination of parents

Провести преконцепційну профілактику, запланувати вагітність, провести пренатальну діагностику Conduct preconception prevention, plan pregnancy, conduct prenatal diagnosis

1223 / 1500
Чоловік 65 років раптово знепритомнів, посинів. Прід час обстеження встановлено: пульс та АТ на периферичних судинах не визначаються, тони серця не вислуховуються. На ЕКГ спостерігаєть-ся: нерегулярні, різної величини і форми хвилі, які реєструються без інтервалів. Які заходи необхідно провести насамперед? A 65-year-old man suddenly fainted, turned blue. During the examination, it was found that the pulse and blood pressure in the peripheral vessels were not determined, heart sounds were not heard. The ECG showed: Irregular waves of different sizes and shapes, which are recorded without intervals. What measures should be taken first?

Електрична дефібриляція Electrical defibrillation

Тимчасова кардіостимуляція Temporary cardiac stimulation

В/в введення строфантину IV administration of strophanthin

Штучне дихання Artificial respiration

В/в введення адреналіну IV administration of epinephrine

1224 / 1500
На прийомі у сімейного лікаря дитина, яка народилася доношеною, здоровою і дотепер нічим не хворіла. Дитина може гратися іграшками годину і більше. Ходить вздовж меблів при підтримці за одну руку або самостійно. Має мовний запас із 8-12 слів. За проханням обнімає батьків та прагне схвалення, підтвердження свого успіху. Визначте імовірний вік дитини: At the family doctor's appointment, a child who was born full-term, healthy and has not been sick so far. The child can play with toys for an hour or more. He walks along the furniture with support for one hand or independently. Has a language reserve of 8-12 words. Upon request, hugs parents, seeks confirmation of his success. Determine the probable age of the child:

12 місяців 12 months

7 місяців 7 months

6 місяців 6 months

9 місяців 9 months

8 місяців 8 months

1225 / 1500
Чоловік 35 років після переохолодження скаржиться на часте, малими порціями, болюче сечовипускання, підвищення температури тіла протягом двох діб до 38,6^oC, озноб. Під час пальцевого ректального обстеженя виявляється збільшена болюча простата. У крові виявлено: лейкоцитоз - 14,2·10^9/л. У сечі лейкоцитурія - 20-25 в п/з. Який діагноз імовірний? A 35-year-old man after hypothermia complains of frequent, small portions, painful urination, an increase in body temperature for two days to 38.6^oC, chills. During digital a rectal examination reveals an enlarged prostate. Leukocytosis is found in the blood - 14.2·10^9 in the urine. What is the probable diagnosis?

Гонорея Gonorrhea

Гострий простатит Acute prostatitis

Пухлина сечового міхура Bladder tumor

Гіперплазія простати Prostate hyperplasia

Гострий цистит Acute cystitis

1226 / 1500
Жінка 35 років поступила зі скаргами на болі внизу живота, які посилюються під час менструації і статевих актів та іррадіюють в піхву, мажучі кров'янисті виділення до та після менструації протягом 5 днів. Безпліддя протягом 9 років. Протизапальне лікування ефекту не дало. Під час бімануального дослідження виявлено: матка збільшена, щільна, болюча, гладка. Під час гістероскопії в ділянці дна матки видно темно-червоні утвори, з яких виділяється темна кров. Який діагноз відповідає цій клінічній картині? A 35-year-old woman came in with complaints of pain in the lower abdomen, which worsens during menstruation and sexual acts and radiates into the vagina, smearing bloody discharge before and after menstruation Infertility for 9 years. During the bimanual examination, the uterus is enlarged, painful, and dark red formations are visible at the bottom of the uterus does the diagnosis correspond to this clinical picture?

Ендометрит Endometritis

Внутрішній ендометріоз Internal endometriosis

Міома матки Uterine myoma

Гіперплазія ендометрію Endometrial hyperplasia

Рак матки Uterine cancer

1227 / 1500
Юнак 18 років під час госпіталізації до стаціонару скаржиться на загальну слабкість, підвищення температури тіла до 37,5^oC, зниження апетиту, нудоту, важкість у правому підребер'ї, зміну кольору сечі та калу. Хворіє протягом 5 днів. Об'єктивно спостерігається незнач-на жовтяниця шкіри та склер. Печінка виступає на 3 см, чутлива під час пальпації. Сеча темно-коричневого кольору, кал світлий. Уживає водопровідну воду. Який діагноз найімовірніший?

Лептоспіроз Leptospirosis

Черевний тиф Typhoid

Вірусний гепатит В Viral hepatitis B

Малярія Malaria

Вірусний гепатит А Viral hepatitis A

1228 / 1500
На прохідників під час виконання робіт у гірничих виробітках діє мінеральний пил у вигляді аерозолів дезінтеграції. Для визначення рівня небезпеки виникнення професійних пилових захворювань вивчали хімічні та фізичні властивості пилу. Яка із властивостей пилу визначає загалом глибину його проникнення в дихальні шляхи? Pedestrians are exposed to mineral dust in the form of disintegration aerosols during mining operations. To determine the level of danger of occupational dust diseases, the chemical and physical properties of dust were studied. What from the properties of dust determines in general the depth of its penetration into the respiratory tract?

Розчинність Solubility

Електрозарядженість Electrical charge

Вміст діоксиду кремнію Content of silicon dioxide

Дисперсність Variance

Форма пилових часток Shape of dust particles

1229 / 1500
Дівчинка, 7 років, надійшла у відділення зі скаргами на біль у горлі, підвищення температури до 39^oC. Під час огляду стан дитини важкий, шийні лімфатичні вузли до 1,5 см. При пальпації печінка на 3 см, селезінка на 2 см виступає з-під краю реберної дуги. При лабораторному дослідженні крові: еритроцити - 4,0·10^12/л, Hb- 121 г/л, кольоровий показник - 0,9, тромбоцити - 190·10^9/л, лейкоцити - 19·10^9/л, е- 0, п/я- 1, с/я- 0, л- 87, м- 2, швидкість зсідання еритроцитів - 36 мм/год. Що з перерахованого буде найбільш доречним наступним кроком у веденні пацієнта? A 7-year-old girl came to the department with complaints of a sore throat, temperature rise to 39^oC. During the examination, the child's condition is serious, cervical lymph nodes to 1.5 cm. During palpation, the liver is 3 cm, the spleen protrudes 2 cm from the edge of the costal arch. During the laboratory blood test: erythrocytes - 4.0·10^12/l, Hb - 121 g/l, color index - 0.9, platelets - 190·10^9/l, leukocytes - 19·10^9/l, e- 0, p/ya- 1, s/ya- 0, l- 87, m- 2, speed erythrocyte sedimentation rate - 36 mm/h. Which of the following would be the most appropriate next step in the patient's management?

Аналіз крові на виявлення гетерофільних антитіл Blood analysis for detection of heterophilic antibodies

Спостереження протягом 2 тижнів Observation for 2 weeks

Дослідження кісткового мозку Bone marrow research

Повторний загальний аналіз крові через 1 тиждень Repeat general blood test in 1 week

- -

1230 / 1500
Для вивчення фізичного розвитку дітей та підлітків широко використовують антропометричні дослідження. Виберіть із наведеного фізіометричний метод досліджень: Anthropometric studies are widely used to study the physical development of children and adolescents. Choose a physiometric research method from the following:

Визначення форми грудної клітки Determining the shape of the chest

Визначення життєвої ємності легень Determining the vital capacity of the lungs

Вимірювання зросту Height measurement

Визначення форми хребта Definition of spine shape

Визначення маси тіла Determination of body weight

1231 / 1500
Мати з дівчинкою 11-ти років звернулись в приймальне відділення лікарні зі скаргами на виражений біль в правій здухвинній ділянці. При обстеженні виявлені зміни в сечі: протеїнурія, лейкоцитурія, бактеріурія. Ультразвукове дослідження (УЗД) показало відсутність правої нирки в типовому місці. Яке дослідження найдоцільніше провести для уточнення діагнозу виявленого під час УЗД? A mother and an 11-year-old girl came to the hospital's reception department with complaints of severe pain in the right sinus area. During the examination, changes in the urine were detected: proteinuria, leukocyturia, bacteriuria. Ultrasound examination (ultrasound) showed the absence of the right kidney in a typical place. What is the most appropriate study to clarify the diagnosis found during ultrasound?

Мікційна цистографія Mictation cystography

Магнітно-резонансна томографія Magnetic resonance imaging

Ниркова артеріографія Renal arteriography

Оглядова урографія Review urography

Цистографія Cystography

1232 / 1500
Жінка 48 років звернулась із приводу того, що протягом останніх 8-9 місяців менструації дуже рясні, призводять до анемізації, порушена працездатність. Протягом 2 років спостерігається гінекологом з приводу фіброміоми матки. Об'єктивно встановлено: шийка матки циліндрична, чиста, вічко закрите. Тіло матки збільшене до 9-10 тижнів вагітності, цупке, рухоме, безболісне. Придатки з обох сторін не визначаються. Параметрії вільні. Склепіння глибокі. Виділення слизові. Який найбільш вірогідний діагноз? A 48-year-old woman complained about the fact that during the last 8-9 months, menstruation is very abundant, leads to anemia, impaired working capacity. For 2 years, she has been observed by a gynecologist for Objectively, the uterus is cylindrical, the eye is closed, the uterus is tender, the appendages are not defined. The discharge is deep the most likely diagnosis?

Дисфункциональна маткова кровотеча Dysfunctional uterine bleeding

Фіброміома матки та вагітність Uterine fibroids and pregnancy

Рак ендометрію Endometrial cancer

Ендометріоз із переважним ураженням тіла матки Endometriosis with predominant damage to the body of the uterus

Фіброміома матки Uterine fibromyoma

1233 / 1500
Пацієнтка 49 років скаржиться на нерегулярність циклу впродовж 18 місяців, міжменструальні кровотечі та <<приливи>>, які її дуже турбують; наполягає на негайному проведенні лікування. Яку процедуру треба провести перед призначенням терапії? A 49-year-old patient complains of cycle irregularity for 18 months, intermenstrual bleeding and <>, which bother her a lot; she insists on immediate treatment. What procedure should be performed before prescribing therapy?

Призначення медроксипрогестерону ацетату Medroxyprogesterone acetate prescription

Біопсія ендометрію Endometrial biopsy

Гістеректомія Hysterectomy

Послідовна терапія естрогенами та прогестином Sequential estrogen and progestin therapy

Застосування естрогенового крему Estrogen cream application

1234 / 1500
Пацієнт 28-ми років скаржиться на періодичні болі в ногах та руках, мерзлякуватість стоп, зміну кольору 3 пальців кистей. Болі тривають вже протягом 1 року, але зміна кольору пальців виникла вперше. Він викурює по 20 сигарет в день протягом 12-ти років. Об'єктивно: шкіра ніг бліда, прохолодна, тургор знижений, гіпотрихоз. Який діагноз є найбільш імовірним? A 28-year-old patient complains of periodic pains in the legs and arms, frostbite of the feet, and discoloration of 3 fingers. The pain has lasted for 1 year, but the discoloration He has been smoking 20 cigarettes a day for 12 years. Objectively, the skin of the legs is pale, the turgor is reduced. What is the most likely diagnosis?

Діабетична нейропатія Diabetic neuropathy

Системна склеродермія Systemic scleroderma

Облітеруючий тромбангіїт Thrombangiitis obliterans

Облітеруючий атеросклероз Obliterating atherosclerosis

- -

1235 / 1500
При плановому обстеженні 22-річної вагітної (30 тижнів) двічі у сечі виявлена ізольована бактеріурія. Вагітність перебігає без патології. Визначте тактику ведення вагітної: During a routine examination of a 22-year-old pregnant woman (30 weeks), isolated bacteriuria was detected twice in the urine. The pregnancy proceeds without pathology. Determine the tactics of managing the pregnant woman:

Фітотерапія Phytotherapy

Динамічне спостереження Dynamic Observation

Призначення ципрофлоксацину Ciprofloxacin Prescription

Призначення ампіциліну Prescription of ampicillin

Фізіотерапевтичне лікування Physiotherapy treatment

1236 / 1500
Пацієнтка 29 років скаржиться на болючі, тривалі менструації. З анамнезу відомо: менархе з 13 років помірно болючі, але рясні. З 19 років болючість менструацій посилилась, подовжилась їх тривалість. У шлюбі і статеве життя з 25 років, протизаплідних засобів не застосовує, не вагітніє. Під час гістероскопії у ділянці дна матки спостерігаються темно-червоні утвори, з яких виділяється темна кров. Який діагноз відповідає клінічній картині? A 29-year-old patient complains of painful, prolonged menstruation. It is known from the anamnesis: menarche since the age of 13 has been moderately painful, but abundant. Since the age of 19, the pain of menstruation has increased, they have become longer duration. In marriage and sexual life since 25 years, does not use contraceptives. During hysteroscopy, dark red formations from which dark blood is secreted are observed?

Поліп ендометрію Endometrial polyp

Аденокарцинома Adenocarcinoma

Внутрішній ендометріоз Internal endometriosis

Субмукозна міома матки Submucosal myoma of the uterus

Залозисто-кістозна гіперплазія ендометрію Glandular-cystic endometrial hyperplasia

1237 / 1500
Під час огляду трупа людини, яка померла внаслідок повішення, виявлено: трупні плями під час натискування зникають, відновлюються за 50 сек, трупне заклякання помірно виражене тільки в жувальних м'язах, у м'язах шиї та пальців кисті. Температура тіла - 31,0^oC. Визначте час настання смерті? During examination of the corpse of a person who died as a result of hanging, it was found: corpse spots disappear when pressed, recover in 50 seconds, corpse incantation is moderately expressed only in chewing m in the muscles of the neck and fingers. Body temperature - 31.0°C. Determine the time of death?

1-2 години тому 1-2 hours ago

10-18 годин тому 10-18 hours ago

6-7 годин тому 6-7 hours ago

16-24 години тому 16-24 hours ago

8-10 годин тому 8-10 hours ago

1238 / 1500
Хвора 68-ми років із застійною серцевою недостатністю, ФВ ЛШ textless40%, отримує наступну схему фармакотерапії: раміприл, торасемід, бісопролол, клопідогрель, дигоксин. Під час чергового обсте-ження була виявлена часта поліморфна шлуночкова екстрасистолія. Який з призначених препаратів необхідно вилучити зі схеми лікування? A 68-year-old patient with congestive heart failure, LVEF textless40%, receives the following regimen of pharmacotherapy: ramipril, torasemide, bisoprolol, clopidogrel, digoxin. During the next the examination revealed frequent polymorphic ventricular extrasystole. Which of the prescribed drugs should be removed from the treatment regimen?

Клопідогрель Clopidogrel

Бісопролол Bisoprolol

Раміприл Ramipril

Торасемід Torasemide

Дигоксин Digoxin

1239 / 1500
У пацієнтки віком 30 років під час обстеження виявлено добову протеїнурію - 2,2 г. Під час біопсії виявлено: зміни стінок капілярів клубочків у вигляді дротяних петель. Про яку морфологічну форму ураження нирок свідчать результати біопсії? During the examination, a 30-year-old patient was found to have daily proteinuria - 2.2 g. During the biopsy, it was found: changes in the walls of the glomerular capillaries in the form of wire loops. About which the morphological form of kidney damage is evidenced by biopsy results?

Мезангіальний гломерулонефрит Mesangial glomerulonephritis

Дифузний вовчаковий гломерулонефрит Diffuse lupus glomerulonephritis

IgA нефропатія IgA nephropathy

Нефросклероз Nephrosclerosis

Мембранозний гломерулонефрит Membranous glomerulonephritis

1240 / 1500
Пацієнтка віком 19 років скаржиться на відсутність менструацій, млявість, зменшення ваги. З анамнезу відомо: рік тому - патологічні пологи зі значною крововтратою. Після пологів лактації не було. Під час піхвового дослідження виявлено: піхва вузька, матка зменшена, яєчники не пальпуються. Лабораторно виявлено гіпоестрогенемію. Який наймовірніший діагноз? A 19-year-old patient complains of the absence of menstruation, lethargy, and weight loss. It is known from the anamnesis: a year ago - pathological childbirth with significant blood loss. There was no lactation after childbirth. During the vaginal examination, it was found that the vagina is narrow, the uterus is not palpable. Hypoestrogenemia was detected in the laboratory. What is the most likely diagnosis?

Синдром Шихана Sheehan Syndrome

Синдром Штейна-Левенталя Stein-Leventhal syndrome

Астено-вегетативний синдром Astheno-vegetative syndrome

Туберкульоз геніталій Genital tuberculosis

Гіпотиреоз Hypothyroidism

1241 / 1500
На підприємстві з виробництва синтетичних миючих речовин збільшилися випадки захворювання ринітами, вазоринітами, фарингітами, дерматитами. У повітрі робочої зони присутній пил миючих речовин. Фахівці вважають, що клінічні прояви й зростання захворюваності зумовлене впливом пилу. Які особливості дії пилу полягають в основі патогенезу? At the enterprise for the production of synthetic detergents, cases of rhinitis, vasorinitis, pharyngitis, and dermatitis have increased. There is dust from detergents in the air of the working area. Experts believe that the clinical manifestations and the increase in morbidity is due to the influence of dust. What features of the action of dust are the basis of pathogenesis?

Фіброгенна дія Fibrogenic action

Алергічна дія Allergic action

Подразнююча дія Annoying action

Канцерогенна дія Carcinogenic action

Загальнотоксична дія General toxic effect

1242 / 1500
У жінки 32-х років скарги на двосторонній біль у руках і припухлість пальців, відчуття скутості впродовж кількох годин після пробудження, втомлюваність. Об'єктивно: набряк другого та третього п'ястково-фалангових і проксимальних міжфалангових суглобів, позитивні симптоми поперечного стискання кисті. Яке захворювання найбільш імовірне? A 32-year-old woman complains of bilateral pain in the hands and swelling of the fingers, a feeling of stiffness for several hours after waking up, fatigue. Objectively: swelling of the second and third metacarpal-phalangeal and proximal interphalangeal joints, positive symptoms of transverse compression of the hand. What disease is most likely?

Подагричний артрит Gouty arthritis

Остеоатрит Osteoarthritis

Реактивний артрит Reactive arthritis

Ревматичний артрит Rheumatic arthritis

Ревматоїдний артрит Rheumatoid arthritis

1243 / 1500
В районі відзначається зростання серцево-судинних захворювань серед населення. Сімейний лікар (лікар загальної практики) планує вжити заходів з профілактики серцево-судинних захворювань. Визначте, які з наведених заходів належать до первинних: There is an increase in cardiovascular diseases among the population in the district. The family doctor (general practitioner) plans to take measures to prevent cardiovascular diseases. Determine which of the following measures belong to the primary:

Комплексне обстеження Comprehensive survey

Модифікація факторів ризику Modification of risk factors

Направлення на санаторно-курортне лікування Direction for sanatorium-resort treatment

Направлення на стаціонарне лікування Referral to inpatient treatment

Направлення на лікування в профільний диспансер Referral for treatment to a specialized dispensary

1244 / 1500
Чоловік віком 52 роки звернувся зі скаргами на періодичні болі за грудиною, порушення проходження твердої їжі, схуднення на 5 кг за 3 тижні. Під час фіброезофагогастроскопії виявлено: вільна прохідність стравоходу до рівня 37,5-38 см від краю верхніх різців, де визначається верхній полюс екзофітного утворення, що звужує просвіт до 1/2 від норми. За результатами біопсії: аденокарцинома. Яке передракове захворювання найімовірніше у пацієнта? A 52-year-old man complained of periodic pains behind the sternum, impaired passage of solid food, weight loss of 5 kg in 3 weeks. Fibroesophagogastroscopy revealed: free passage esophagus to the level of 37.5-38 cm from the edge of the upper incisors, where the upper pole of the exophytic formation is determined, which narrows the lumen to 1/2 of the norm. According to the results of the biopsy: adenocarcinoma. What is the most likely precancerous disease in the patient?

Стравохід Баррета Barrett's Esophagus

Дивертикул стравоходу Esophageal diverticulum

Стан після опіку стравоходу Condition after esophageal burn

Поліп стравоходу Esophageal polyp

Ліпома стравоходу Esophageal lipoma

1245 / 1500
У вогнищах облисіння на голові волосся обламане на рівні 4-6 мм від поверхні шкіри, відзначається гіперемія, мукоподібне лущення. Імовірний попередній діагноз? In foci of baldness on the head, the hair is broken off at the level of 4-6 mm from the surface of the skin, there is hyperemia, muco-like desquamation. Possible preliminary diagnosis?

Себорейне облисіння Seborrheic alopecia

Дискоїдний червоний вовчак Discoid lupus erythematosus

Мікроспорія волосистої частини голови Microsporia of scalp

Гніздова плішивість Nest baldness

Вторинний сифіліс Secondary syphilis

1246 / 1500
У шестирічної дитини, яка хворіє на екзантемну інфекцію, на 8-й день захворювання повторно підвищилася температура тіла до 39,6^oС, з'явився головний біль, блювання, спостерігалася хитка хода, скандована мова. Під час огляду виявлено залишкові елементи висипу (кірочки), атаксію, дискоординацію рухів, тремор кінцівок, горизонтальний ністагм, нестійкість у позі Ромберга. Вкажіть препарат вибору для лікування ускладення, що виникло у дитини. A six-year-old child suffering from an exanthem infection, on the 8th day of the disease, the body temperature rose again to 39.6^oС, a headache appeared, vomiting, wobbly gait, slurred speech were observed. During the examination, residual elements of the rash (crusts), ataxia, limb tremor, instability in Romberg's position were found. Specify the drug of choice for the treatment of the child's condition.

Ацикловір Acyclovir

Занамівір Zanamivir

Озельтамівір Oseltamivir

Варіцело-зостерний імуноглобулін Vicella-zoster immunoglobulin

Рибавірин Ribavirin

1247 / 1500
Під час скринінг-дослідження у двотижневого новонародженого було виявлено фенілкетонурію. Яке лікування необхідно призначити дитині для запобігання тяжким ускладненням у майбутньому? During the screening examination, phenylketonuria was detected in a two-week-old newborn. What treatment should be given to the child to prevent serious complications in the future?

Вітамінотерапію Vitamin therapy

Сонячні ванни Sunbaths

Антибіотикотерапію Antibiotic therapy

Спеціальну дієту Special diet

Гормональну терапію Hormone therapy

1248 / 1500
У дитини 10 мiсяцiв на тлi легкого перебiгу ГРВІ з'явилися повторнi клонiчнi судоми. Під час огляду констатовано чiткi прояви рахiту середньої важкостi. Рiвень кальцiю кровi - 1,6 ммоль/л, iнтервал Q-T на ЕКГ подовжений. Даних про перинатальне ушкодження ЦНС немає. Спинно-мозкова рiдина iнтактна, витiкала пiд тиском. Вигодовування штучне, без овочевих страв. Яке захворювання проявилося на тлi ГРВI? A 10-month-old child had repeated clonic convulsions against the background of a mild acute respiratory viral infection. During the examination, clear manifestations of moderate rickets were noted. Blood calcium level - 1.6 mmol/l, the QT interval on the ECG is prolonged. There is no evidence of CNS damage. The cerebrospinal fluid was leaking under pressure. What disease was manifested against the background of GERD?

Енцефалiтична реакцiя Encephalitic reaction

Енцефалiт Encephalitis

Спазмофiлiя Spasmophilia

Нейротоксикоз Neurotoxicosis

Менiнгiт Meningitis

1249 / 1500
Хлопчик віком 10 років скаржиться на задишку під час ходіння, підвищення температури тіла до 38^oС , біль і припухлість обох колінних суглобів. Два тижні тому переніс ангіну. Об'єктивно спостерігається: припухлість, гіперемія, обмеження рухів в колінних суглобах, розширення меж серця вліво, тахікардія, приглушення тонів серця, систолічний шум на верхівці. Який найімовірніший діагноз? A 10-year-old boy complains of shortness of breath while walking, an increase in body temperature up to 38°C, pain and swelling of both knee joints. Two weeks ago he suffered from angina. About objectively observed: swelling, hyperemia, limitation of movements in the knee joints, expansion of the heart borders to the left, tachycardia, muffled heart sounds, systolic murmur at the apex. What is the most likely diagnosis?

Неревматичний кардит Non-rheumatic carditis

Синдром Стілла Still's Syndrome

Гостра ревматична лихоманка Acute rheumatic fever

Інфекційно-алергічний артрит Infectious-allergic arthritis

Системний червоний вовчак Systemic lupus erythematosus

1250 / 1500
Дитина 1 місяця зригує, які виникають щоразу, щойно її після годування грудьми вкладають у ліжечко. Огляд педіатра відхилень у стані дитини не виявив. Виникнення зригувань лікар пояснив особливостями травної системи у цьому віці та дав поради щодо догляду за дитиною. Які це поради? A 1-month-old child vomits, which occur every time as soon as she is put in a crib after breastfeeding. The pediatrician's examination did not reveal any abnormalities in the child's condition. The doctor explained the occurrence of vomiting by the peculiarities of the digestive system system at this age and gave advice on caring for the child. What are these tips?

Дати соску одразу після годування грудьми Give nipple immediately after breastfeeding

Теплові процедури на живіт Heat procedures on the abdomen

Вертикальне положення дитини одразу після годування Vertical position of the child immediately after feeding

Укладання дитини на живіт після годування Putting baby on stomach after feeding

Масаж живота Abdominal massage

1251 / 1500
Хвора 45-ти років госпіталізована зі скаргами на періодичний інтенсивний головний біль на тлі підвищення артеріа-льного тиску до 180/90 мм рт.ст., м'язову слабкість, часте сечовипускання (в тому числі вночі). З анамнезу відомо, що, незважаючи на комбінацію та дозування антигіпертензивних препаратів, артеріа-льну гіпертензію коригувати не вдається. Рівень калію сироватки крові - 2,0 ммоль/л, рівень натрію сироватки крові - 160,0 ммоль/л. В ході УЗД виявлено об'ємні утворення близько 1,0 см в діаметрі в обох наднирниках. Виконано селективний ендоваскулярний забір крові з наднирникових вен - визначається значне підвищення рівню кортизолу та альдостерону. Який діагноз у хворої? A 45-year-old patient was hospitalized with complaints of periodic intense headache against the background of an increase in blood pressure up to 180/90 mm Hg, muscle weakness, frequent urination (including at night). It is known from the anamnesis that, despite the combination and dosage of antihypertensive drugs, arterial hypertension cannot be corrected. The level of potassium in the blood serum is 2.0 mmol/l, the level of sodium in the blood serum is - 160.0 mmol/l. Volumetric formations of about 1.0 cm in diameter were detected in both adrenal glands. A selective endovascular blood sampling was performed - a significant increase in the level of cortisol and aldosterone was determined in the patient.

Феохромоцитома Pheochromocytoma

Синдром Іценка-Кушинга Itsenko-Cushing syndrome

Хвороба Іценка-Кушинга Itsenko-Cushing disease

Андростерома Androsteroma

Альдостерома Aldosteroma

1252 / 1500
У пацієнта віком 29 років 3 дні тому з'явився біль пульсуючого характеру в ділянці прямої кишки, підвищення температури тіла, загальна слабість. Пальпаторно виявлено: локальна болючість в анальній ділянці на 6 годині. Під час пальцевого дослідження прямої кишки виявлено: болючий інфільтрат, що досягає не вище гребінцевої лінії. Який найімовірніший діагноз? 3 days ago, a 29-year-old patient developed pain of a pulsating nature in the region of the rectum, increased body temperature, general weakness. Palpation revealed: local soreness in the anus area at 6 o'clock. During a digital examination of the rectum, a painful infiltrate was found, reaching no higher than the comb line. What is the most likely diagnosis?

Пухлина прямої кишки Rectal tumor

Гострий парапроктит Acute paraproctitis

Гострий геморой Acute hemorrhoids

Гострий простатит Acute prostatitis

Гостра анальна тріщина Acute anal fissure

1253 / 1500
Пацієнт віком 37 років неодноразово перебував на лікуванні в психіатричній лікарні. Поточна госпіталізація зумовлена порушеннями поведінки у вигляді замкненості, відмови від їжі, періодичного хаотичного збудження зі стереотипними рухами. Під час огляду виявлено: пацієнт перебуває в ліжку, загальмований, контакт відсутній, на запитання не відповідає, поза одноманітна, гіпомімічний, симптом «хоботка», воскова гнучкість м'язів, симптом «повітряної подушки». Такий стан триває близько тижня. Встановіть попередній діагноз. A 37-year-old patient was repeatedly treated in a psychiatric hospital. The current hospitalization is due to behavioral disorders in the form of withdrawal, refusal to eat, periodic chaotic excitement with stereotyped movements. Under the time of the examination revealed: the patient is in bed, inhibited, does not respond to questions, the posture is monotonous, the symptom is 'proboscis', the symptom is 'air cushion'. This condition lasts about a week. Establish a preliminary diagnosis .

Шизофренія, гебефренічна форма Schizophrenia, hebephrenic form

Реактивний ступор Reactive stupor

Шизофренія, кататонічна форма Schizophrenia, catatonic form

Шизофренія, параноїдна форма Schizophrenia, paranoid form

Шизофренія, проста форма Schizophrenia, simple form

1254 / 1500
Хворий 20-ти років надійшов у лікарню на 9-й день хвороби. Захворювання пов'язує з вживанням недостатньо просмаженої свинини. Захворювання почалось з появи періорбітальних набряків, лихоманки. Об'єктивно: температура тіла - 38,5^оC) Обличчя одутле. Різко виражений набряк повік. Пальпація литкових м'язів різко болюча. В крові: гіпереозинофілія. Яка етіологія захворювання? A 20-year-old patient was admitted to the hospital on the 9th day of illness. The disease is associated with the consumption of insufficiently fried pork. The disease began with the appearance of periorbital edema, fever Objectively: the body temperature is 38.5°C. The face is swollen. Palpation of the calf muscles is painful. What is the etiology of the disease?

Трихінели Trichinella

Ехінококи Echinococci

Волосоголовці Hairheads

Аскариди Ascarids

Лептоспіри Leptospira

1255 / 1500
Дитина 2 років була в контакті з хворим на оперізуючий герпес. На 15-й день контакту в неї підвищилась температура тіла до 39,3^oC, з'явився висип на тулубі, кінцівках, обличчі та волосистій частині голови, який проходив стадії: пляма, папула, визикула, кірочка. Який імовірний діагноз? A 2-year-old child was in contact with a patient with herpes zoster. On the 15th day of contact, her body temperature rose to 39.3^oC, appeared a rash on the trunk, limbs, face and scalp, which went through the stages: spot, papule, vesicle, crust. What is the probable diagnosis?

Натуральна віспа Smallpox

Ентеровірусна екзантема Enterovirus exanthema

Вітряна віспа Chicken Pox

Оперізуючий герпес Herpes zoster

Простий герпес Herpes simplex

1256 / 1500
Батьки восьмирічного хлопчика звернулися до лікаря зі скаргами на відставання дитини у зрості. З анамнезу відомо: народжений доношеним, із вагою 3100 г, довжиною тіла 50 см. Батьки нормального зросту. На момент огляду хлопчика співвідношення зросту до віку = -3, вага - теж знаходиться у діапазоні < -2. <<Кістковий вік>> дорівнює 5 років. Патології з боку інших органів і систем не виявлено. Яке дослідження необхідно призначити дитині для уточнення діагнозу? The parents of an eight-year-old boy turned to the doctor with complaints about the child's growth retardation. It is known from the anamnesis: he was born full-term, with a weight of 3100 g, a body length of 50 cm. The parents of a normal At the time of the examination, the boy's height-to-age ratio is in the range of <-2. No pathology from other organs and systems has been found for the child clarifying the diagnosis?

Визначення рівня лютеїнізуючого гормону Determining the level of luteinizing hormone

Визначення рівня соматотропного гормону Determining the level of somatotropic hormone

Визначення рівня гонадотропного гормону Determining the level of gonadotropic hormone

Визначення рівня адренокортикотропного гормону Determining the level of adrenocorticotropic hormone

Визначення рівня тиреотропного гормону Determining the level of thyroid-stimulating hormone

1257 / 1500
При проведенні профогляду робітниць промислового підприємства була виявлена група жінок з лабораторно підтвердженою залізодефіцитною анемією. Окрім медикаментозного лікування, які продукти, як основні носії засвоюваного заліза, повинен рекомендувати лікар для споживання? During a professional examination of female workers at an industrial enterprise, a group of women with laboratory-confirmed iron-deficiency anemia was identified. In addition to drug treatment, which products, as the main carriers of absorbed iron, should the doctor recommend for consumption?

Хліб та хлібобулочні вироби Bread and bakery products

Хліб та рибу Loaves and fishes

М'ясо та м'ясопродукти Meat and meat products

Овочі та фрукти Vegetables and fruits

Молоко та молокопродукти Milk and milk products

1258 / 1500
26-річна вагітна жінка доставлена до відділення невідкладної терапії на 36-му тижні вагітності зі скаргами на інтенсивний головний біль у лобній ділянці. При фізикальному обстеженні: артеріальний тиск - 170/90 мм рт.ст., пульс - 85/хв., частота дихання - 15/хв., температура - 36,9^oC, набряки кінцівок. Серцебиття плоду - 159/хв. Під час огляду у жінки розвивається напад генералізованих тоніко-клонічних судом. Який препарат лікар має ввести першочергово? A 26-year-old pregnant woman was brought to the emergency department at the 36th week of pregnancy with complaints of an intense headache in the frontal area. On physical examination: blood pressure - 170/90 mm Hg, pulse - 85/min, temperature - 36.9°C, fetal heart rate - 159/min During the examination, the woman developed a generalized seizure tonic-clonic convulsions. What drug should the doctor administer first?

Ламотриджин Lamotrigine

Діазепам Diazepam

Фенітоїн Phenytoin

Магнію сульфат Magnesium sulfate

Натрію вальпроат Sodium valproate

1259 / 1500
Пацієнт віком 76 років скаржиться на епізоди втрати пам`яті та запаморочення. З анамнезу відомо, що два роки тому він переніс гострий інфаркт міокарда. Об'єктивно спостерігається: ЧСС = пульс = 32/хв, АТ - 160/90 мм рт. ст. На ЕКГ виявлено: АВ-блокада ІІІ ступеня. Якою повинна бути подальша тактика лікаря? A 76-year-old patient complains of episodes of memory loss and dizziness. It is known from the anamnesis that he suffered an acute myocardial infarction two years ago. Objectively observed: Heart rate = 32/min, blood pressure - 160/90 mmHg. On the ECG, it was found that there is a third-degree AV block.

Призначити пацієнту постійне приймання атропіну Appoint the patient to receive permanent atropine

Призначити пацієнту приймання beta-адреноблокаторів постійно Appoint the patient to take beta-blockers permanently

Імплантація електрокардіостимулятора Implantation of a pacemaker

Призначити пацієнту постійне приймання ізадрину Assign the patient to constant reception of isadrin

Спостереження за пацієнтом один раз на місяць постійно Patient observation once a month continuously

1260 / 1500
Лікар швидкої допомоги під час надання допомоги пацієнту з кровотечею проколов палець. З'ясувалося, що пацієнт є ін'єкційним наркоманом, обстежувався на ВІЛ, результат не відомий. Яких заходів слід вжити лікарю? An emergency physician punctured a finger while treating a bleeding patient. It turned out that the patient is an injection drug user, he was tested for HIV, the result is unknown. What measures should the doctor take?

Обробити забруднену ділянку шкіри, обстежитися на ВІЛ, розпочати хіміопрофілактику Treat the contaminated skin area, get tested for HIV, start chemoprophylaxis

Нічого не робити Do nothing

Обробити забруднену ділянку шкіри, розпочати лікування противірусними препаратами та антибіотиками Treat the contaminated skin area, start treatment with antiviral drugs and antibiotics

Обробити забруднену ділянку шкіри, обстежитись на TORCH-інфекції, розпочати лікування антибіотиками Treat contaminated area of ​​skin, examine for TORCH infection, start antibiotic treatment

Обробити забруднену ділянку шкіри, увести донорський імуноглобулін Treat the contaminated skin area, inject donor immunoglobulin

1261 / 1500
Хворого 28-ми років доставлено через 2 години після автодорожньої катастрофи в тяжкому стані зі скаргами на біль в животі. Отримав удар рулем у живіт. В ході об'єктивного обстеження вияв-лено: живіт не бере участі в акті дихання, напружений, під час пальпації різко болісний, спостерігається захисне напруження м'язів живота, позитивні симптоми подразнення очеревини, спостерігається зникнення печінкової тупості. Артеріальний тиск - 90/60 мм рт.ст., пульс - 120/хв. Якою буде подальша тактика лікування? A 28-year-old patient was brought 2 hours after a road accident in serious condition with complaints of stomach pain. He was hit by a steering wheel in the stomach. During the objective the examination revealed: the abdomen does not take part in the act of breathing, it is tense, during palpation it is sharply painful, there is a protective tension of the abdominal muscles, positive symptoms of peritoneal irritation, the disappearance of hepatic dullness is observed. Blood pressure - 90/60 mm Hg. , pulse - 120/min. What will be the further treatment tactics?

Холод на живіт Stomach cold

Проведення лапароцентезу Laparocentesis

Лапароскопія Laparoscopy

Ультразвукове дослідження Ultrasound examination

Лапаротомія Laparotomy

1262 / 1500
У пацієнтки віком 32 роки, що народжує вперше, почалися інтенсивні потуги з інтервалом 1-2 хв, тривалістю 55-60 с. Об'єктивно спостерігається: настає прорізування голівки плода. Промежина, висота якої 4 см, надмірно випинається. Шкіра промежини бліда, напружена, після припинення потуги із статевої щілини з'явився тоненький струмок крові. Вкажіть подальшу тактику ведення пологів. In a 32-year-old female patient, who is giving birth for the first time, intense efforts began with an interval of 1-2 minutes, lasting 55-60 seconds. Objectively observed: teething occurs the head of the fetus, the height of which is 4 cm. The skin of the perineum is pale, tense, a thin stream of blood appeared from the genital opening.

Кесарський розтин Caesarean section

Накладання акушерських щипців Applying obstetric forceps

Вакуум-екстракція плода Vacuum fetal extraction

Очікувальна тактика Waiting tactics

Епізіотомія Episiotomy

1263 / 1500
Чоловік 50 років скаржиться на наявність висипу на шкірі тулуба без суб'єктивних відчуттів, хворіє 3 місяці, не лікувався, висип прогресує. Об'єктивно встановлено: на шкірі тулуба дисемінована висипка у вигляді червоних негострозапальних лентикулярних та нумулярних папул із чіткими границями, укритих сріблястими лусочками, які легко знімаються при пошкрябуванні предметним склом, інтенсивніше пошкрябування призводить до появи крапкової кровотечі. Який імовірний діагноз? A 50-year-old man complains about the presence of a rash on the skin of the trunk without subjective sensations, he has been sick for 3 months, was not treated, the rash is progressing. Objectively established: on the skin body, a disseminated rash in the form of red non-acute inflammatory lenticular and nummular papules with clear borders, covered with silvery scales, which are easily removed when scraped with a glass slide, more intensive scraping leads to the appearance of spot bleeding. What is the probable diagnosis?

Дерматит Dermatitis

Екзема Eczema

Червоний плоский лишай Lichen planus

Псоріаз звичайний Psoriasis usual

Рожевий пітиріаз Pythyriasis rosea

1264 / 1500
Чоловік віком 49 років скаржиться на напади стенокардії, що виникають під час ходьби на відстань до 500 м. З анамнезу відомо, що він багато років хворіє на хронічний бронхіт. Під час обстеження виявлено: в легенях невелика кількість сухих розсіяних хрипів, ЧД - 18/хв. Межі серця розширені вліво, тони приглушені, ЧСС=ПС=86/хв., АТ - 160/100 мм рт. ст. У клінічному аналізі крові виявлено: Hb - 160 г/л, лейкоцити - 6,4 тис., ШОЕ - 7 мм/год. На ЕКГ спостерігається гіпертрофія лівого шлуночка. Застосування якої групи ліків протипоказане з урахуванням супутньої патології у пацієнта? A 49-year-old man complains of angina attacks that occur while walking up to 500 m. From the anamnesis, it is known that he has been suffering from chronic bronchitis for many years. During the examination, a small number of scattered wheezes were detected, heart rate was 18/min, heart rate was muffled, HR=86/min revealed: Hb - 160 g/l, leukocytes - 6.4 thousand, ESR - 7 mm/h. On the ECG, the use of which group of drugs is contraindicated in the patient's accompanying pathology?

Пролонговані нітрати Prolonged nitrates

Антагоністи кальцію Calcium antagonists

Антиагреганти Antiplatelets

Ангіопротектори Angioprotectors

Бета-блокатори Beta blockers

1265 / 1500
Хворий 45-ти років має скарги на постійний головний біль, біль у серці, колінних суглобах. Три роки тому був укус кліща з кільцевою еритемою на шкірі. Об'єктивно: акродерматит, двобічна пірамідна недостатність, розлади координації. Який діагноз найбільш імовірний? A 45-year-old patient complains of constant headache, pain in the heart, and knee joints. Three years ago, there was a tick bite with annular erythema on the skin. About' objectively: acrodermatitis, bilateral pyramidal insufficiency, coordination disorders. What is the most likely diagnosis?

Нейробореліоз Neuroborreliosis

Вірусний енцефаліт Viral encephalitis

Нейросифіліс Neurosyphilis

Розсіяний склероз Multiple sclerosis

Ішемічна енцефалопатія Ischemic encephalopathy

1266 / 1500
Під час профілактичного огляду населення одного з міст України у 25 % мешканців виявлені наступні симптоми: плямиста емаль зубів, генералізований остеосклероз із кальцифікацією міжхребцевих зв'язок. Яка найімовірніша причина виникнення цих симптомів? During a preventive examination of the population of one of the cities of Ukraine, 25% of the residents were found to have the following symptoms: spotted tooth enamel, generalized osteosclerosis with calcification of the intervertebral ligaments. What is the most likely cause occurrence of these symptoms?

Надмірний вміст фтору у воді Excessive fluoride content in water

Недостатній вміст фтору в тваринних продуктах Insufficient fluoride content in animal products

Недостатній вміст фтору у ґрунті та воді Insufficient fluoride content in soil and water

Надмірний вміст фтору в рослинних продуктах Excessive content of fluoride in vegetable products

Недостатнє надходження фтору в організм із чаєм Insufficient intake of fluoride in the body with tea

1267 / 1500
Громадянин заключив декларацію з сімейним лікарем приватного закладу. Для проходження чергового медичного огляду, лікар дав направлення хворому на біохімічний аналіз крові. Хто є платником медичної послуги у цьому разі? The citizen made a declaration with the family doctor of a private institution. To undergo another medical examination, the doctor referred the patient for a biochemical blood test. Who is the payer of the medical service in this case?'

Благодійні внески Charity contributions

Спеціальний фонд лікарні Special hospital fund

Пацієнт Patient

За рахунок місцевого бюджету At the expense of the local budget

НСЗУ NSZU

1268 / 1500
Хворий 10-ти років надійшов до клініки зі скаргами на експіраторну задишку, частота дихання - 30/хв. Погіршення стану пов'язує з зміною метеорологічних умов. Хлопчик стоїть на диспансерному обліку впродовж 4-х років з діагнозом бронхіальна астма, 3 ступінь, персистуюча. З чого слід розпочати надання невідкладної допомоги? A 10-year-old patient came to the clinic with complaints of expiratory shortness of breath, respiratory rate - 30/min. The deterioration of the condition is associated with a change in meteorological conditions. The boy is standing at the dispensary for 4 years with a diagnosis of bronchial asthma, grade 3, persistent. Where should emergency care be started?

beta_2-агоністи короткої дії short-acting beta_2 agonists

Адреналін Adrenaline

Дексаметазон Dexamethasone

Кларитин Claritin

Еуфілін Euphilin

1269 / 1500
При проведенні медичного огляду учнів середнього та старшого шкільного віку лікарі визначали відповідність біологічного розвитку та календарного віку за наступними критеріями: щорічне збільшення довжини тіла, осифікація кісток кисті, кількість постійних зубів. Який додатковий показник розвитку у ці вікові періоди найбільш імовірно мають включити лікарі? During the medical examination of middle and high school students, doctors determined the correspondence of biological development and calendar age according to the following criteria: annual increase in body length, ossification of hand bones, number of permanent teeth. What additional developmental indicator should doctors most likely include in these age periods?

Маса тіла Body weight

Розвиток вторинних статевих ознак Development of secondary sexual characteristics

М'язова сила кисті Hand muscle strength

Життєва ємність легень Vital lung capacity

Обвід грудної клітки Chest Circumference

1270 / 1500
Хворий 38-ми років доставлений машиною швидкої допомоги до хірургічного відділення зі скаргами на загальну слабкість, нездужання, чорний кал. Під час огляду хворий блідий, спостерігаються крапкові крововиливи на шкірі тулуба та кінцівок. При пальцевому дослідженні на рукавичці чорний кал. У крові: Hb - 108 г/л, тромбоцитопенія. В анамнезі: подібний стан спостерігався 1 рік назад. Встановіть діагноз: A 38-year-old patient was taken by ambulance to the surgical department with complaints of general weakness, malaise, black stool. During the examination, the patient is pale, spotty hemorrhages are observed on on the skin of the trunk and limbs. Black stool in the blood: Hb - 108 g/l. In the anamnesis: a similar condition was observed 1 year ago;

Виразкова кровотеча Ulcer bleeding

Пухлина прямої кишки Rectal tumor

Гемофілія Hemophilia

Тромбоцитопенічна пурпура Thrombocytopenic purpura

Неспецифічний виразковий коліт Nonspecific ulcerative colitis

1271 / 1500
Жінку віком 24 років шпиталізовано зі скаргами на блювання за типом <<кавової гущі>>, запаморочення, загальну слабкість. Під час об'єктивного обстеження встановлено: пацієнтка у свідомості, шкіра бліда, суха, над легенями аускультативно - жорстке дихання. ЧД - 20/хв, АТ - 100/80 мм рт. ст, пульс - 100/хв. Живіт м'який, безболісний. Перитонеальні симптоми негативні. Стілець рідкий, чорний. Діурез збережений. У клінічному аналізі крові виявлено: Нb - 95 г/л, еритроцити - 3,1 г/л, лейкоцити - 9,8 г/л. Який показник шокового індекса Альговера у цієї пацієнтки? A 24-year-old woman was hospitalized with complaints of vomiting of the type <>, dizziness, general weakness. During the objective examination, it was found: the patient in consciousness, skin is dry, respiratory rate is 20/min, pulse is 100/min. Stool is liquid. Diuresis is preserved. Clinical blood analysis revealed: Hb - 3.1 g/l, leukocytes - 9.8 g/l. What is this patient's shock index?

2 2

1,25 1.25

1 1

1,5 1.5

0,85 0.85

1272 / 1500
Хвора скаржиться на підвищення температури тіла до 38^oC протягом 2-х діб. Об'єктивно: позитивний симптом постукування в поперековій ділянці зліва. Загальний аналіз сечі: питома вага - 1015, білок - 0,99 г/л, еритроцити - 6-8 в п/з, лейкоцити - 30-32 в п/з. Який попередній діагноз? The patient complains of an increase in body temperature to 38^oC for 2 days. Objectively: a positive symptom of tapping in the lumbar region on the left. General analysis of urine: specific weight - 1015, protein - 0.99 g/l, erythrocytes - 6-8 in p/z, leukocytes - 30-32 in p/z. What is the previous diagnosis?

Амілоїдоз нирок Kidney amyloidosis

Гострий пієлонефрит Acute pyelonephritis

Гострий цистит Acute cystitis

Гострий гломерулонефрит Acute glomerulonephritis

Сечокам'яна хвороба Urolithiasis

1273 / 1500
У хлопця 18 років скарги на переймоподібний біль у животі, рідкі випорожнення до 6 разів на добу з домішками слизу та свіжої крові. Хворіє впродовж року. Схуд на 10 кг. Об'єктивно встановлено: живіт м'який, болючий по ходу товстої кишки, особливо зліва. Сигмовидна кишка спазмована. У крові виявлено: еритроцити - 3,2·10^12/л, Нb - 92 г/л, лейкоцити - 10,6·10^9/л, ШОЕ - 34 мм/год. Іригоскопія показала: товста кишка звужена, гаустри відсутні, контури нечіткі, симптом <<водопровідної труби>>. Який найбільш ймовірний діагноз? An 18-year-old boy complains of spasm-like abdominal pain, loose stools up to 6 times a day with impurities of mucus and fresh blood. He has been ill for a year. He lost 10 kg Objectively, the abdomen is soft, painful in the course of the colon, especially the left sigmoid colon. , 6·10^9/l, ESR - 34 mm/h. Irigoscopy showed: the colon is narrowed, the contours are unclear, the symptom of a 'water pipe'. What is the most likely diagnosis?

Амебна дизентерія Amoebic dysentery

Неспецифічний виразковий коліт Nonspecific ulcerative colitis

Туберкульоз кишечника Intestinal tuberculosis

Хвороба Крона Crohn's disease

Хронічний ентероколіт Chronic enterocolitis

1274 / 1500
Дочка викликала сімейного лікаря до своєї матері 77 років. У хворої онко-логічне захворювання в термінальній стадії. Вона скаржиться на постійні болі та інші виснажливі симптоми. До якого закладу охорони здоров'я найдоцільніше госпіталізувати цю жінку? The daughter called the family doctor to her 77-year-old mother. The patient has an oncological disease in the terminal stage. She complains of constant pain and other debilitating symptoms. To which institution of health care, is it most expedient to hospitalize this woman?

Соціальний пансіонат Social boarding house

Онкологічний диспансер Oncological dispensary

Багатопрофільна територіальна лікарня Multidisciplinary territorial hospital

Хоспіс Hospice

Територіальний центр соціального обслуговування пенсіонерів Territorial center of social service for pensioners

1275 / 1500
У чоловіка, який добу тому повернувся з Африки, у пахвовій ділянці було виявлено різко болючий пакет лімфовузлів, шкіра над яким гіперемована. Припускається чума, бубонна форма. Чим проводити екстрену профілактику контактним особам? In a man who returned from Africa a day ago, a sharply painful package of lymph nodes was found in the armpit, the skin over which is hyperemic. Plague, bubonic form is assumed. What to do emergency prevention for contact persons?

Гомологічний імуноглобулін Homologous immunoglobulin

Флуконазол Fluconazole

Доксициклін Doxycycline

Гетерологічна сироватка Heterologous serum

Празіквантел Praziquantel

1276 / 1500
Хворий 21-го року госпіталізований на 2-й день хвороби. Загальний стан тяжкий, температура тіла - 39^oC. На шкірі рясні геморагічні елементи неправильної форми. Діагноз: менінгококцемія. Наступного дня температура раптово знизилась, АТ- 80/40 мм рт.ст., частота пульсу - 120/хв. Акроціаноз. Яке ускладнення розвинулося у хворого? A 21-year-old patient was hospitalized on the 2nd day of illness. The general condition is serious, the body temperature is 39^oC. There are abundant hemorrhagic elements of irregular shape on the skin. Diagnosis : meningococcemia. The next day the temperature dropped, blood pressure - 80/40 mmHg. Acrocyanosis developed in the patient?

Гостра печінкова недостатність Acute liver failure

Гостра надниркова недостатність Acute adrenal insufficiency

Гостра серцева недостатність Acute heart failure

Гостра кровотеча Acute bleeding

Церебральна кома Cerebral coma

1277 / 1500
Серед населення, що мешкає поблизу підприємства з виробництва пестицидів, динамічно підвищується рівень вроджених вад розвитку, що виражаються в центральному паралічі, ідіотії та сліпоті новонароджених. Сполуки якої хімічної речовини можуть зумовити розвиток цієї патології? Among the population living near a pesticide production plant, the level of congenital malformations, expressed in central paralysis, idiocy and blindness in newborns, is dynamically increasing. Compounds of which chemical substance can cause the development of this pathology?

Кадмій Cadmium

Хром Chrome

Залізо Iron

Ртуть Mercury

Стронцій Strontium

1278 / 1500
Дитині 4 місяці. Захворіла гостро, з підвищення температури до 37,8^oC і покашлювання. На 3-й день кашель посилився, приєдналася задишка. Перкуторно над легенями прослуховується тимпанічний звук, аускультативно - з обох сторін велика кількість дрібноміхурчатих вологих та свистячих хрипів під час видиху. Який попередній діагноз? The child is 4 months old. She became acutely ill, with a temperature rise to 37.8^oC and a cough. On the 3rd day, the cough intensified, shortness of breath joined. Percussion over the lungs a tympanic sound is heard, on both sides a large number of small bubbles and whistling sounds during exhalation. What is the preliminary diagnosis?

Гострий бронхіт Acute bronchitis

Обструктивний бронхіт Obstructive bronchitis

Вогнищева пневмонія Focal pneumonia

Гострий бронхіоліт Acute bronchiolitis

Бронхопневмонія Bronchopneumonia

1279 / 1500
Пацієнтка віком 26 років, яка хворіє на ревматоїдний артрит впродовж 8 місяців, має набряки ліктьових, променево-зап'ясткових, колінних та гомілковостопних суглобів, ревматоїдні вузлики в ділянці ліктьових суглобів. У загальному аналізі крові спостерігається: ШОЕ - 57 мм/год., С-реактивний білок (+++). На рентгенограмах суглобів визначається виражений остеопороз. Що є базисною терапією в такій ситуації? A 26-year-old female patient, who has been suffering from rheumatoid arthritis for 8 months, has swelling of the elbow, wrist, knee and ankle joints, rheumatoid nodules in the area elbow joints. In the general blood test: ESR - 57 mm/h, C-reactive protein is determined on X-rays of the joints. What is the basic therapy in such a situation?

Мелоксикам Meloxicam

Метотрексат Methotrexate

Інфліксимаб Infliximab

Метилпреднізолон Methylprednisolone

Диклофенак натрію Diclofenac sodium

1280 / 1500
Пацієнт віком 35 років звернувся до лікарні зі скаргами на напади болю в правій поперековій ділянці, часте сечовипускання. В загальному дослідженні сечі білок 0,066 г/л; свіжі еритроцити 6-8 в полі зору. За даними УЗД помірний уростаз справа. Яке дослідження необхідно виконати пацієнту для встановлення діагнозу? A 35-year-old patient came to the hospital with complaints of pain attacks in the right lumbar region, frequent urination. In the general examination of urine, protein 0.066 g/l; fresh red blood cells 6 -8 in the field of vision. According to the ultrasound, moderate urostasis is needed for the patient to establish the diagnosis?

Ретроградну уретеропієлографію Retrograde ureteropyelography

Пнеморетроперітонеографію Pneumoretroperitoneography

Хромоцистоскопію Chromocystoscopy

Екскреторну урографію Excretory urography

Комп’ютерну томографію Computed tomography

1281 / 1500
Пацієнтка віком 17 років скаржиться на біль та набряк ІІ пальця правої кисті. З анамнезу відомо що 3 доби по тому зробила манікюр. Біль з'явився на другий день. Об'єктивно спостерігається: навколонігтьовий валик набрякший, гіперемований, нависає над нігтьовою пластинкою, болючий під час пальпації. Який найімовірніший діагноз? A 17-year-old patient complains of pain and swelling of the II finger of the right hand. From the anamnesis, it is known that she had a manicure 3 days later. The pain appeared on the second day. Objectively observed: the nail plate is swollen, hyperemic, hanging over the nail plate, painful during palpation. What is the most likely diagnosis?

Підшкірний панарицій Subcutaneous panaritium

Пароніхія Paronychia

Піднігтьовий панарицій Subungual panaritium

Еризипелоїд Erysipeloid

Шкірний панарицій Skin panaritium

1282 / 1500
Пацієнта 28 років госпіталізовано на 9-й день хвороби зі скаргами на підвищення температури до 39^oC, головний біль, загальну слабкість, закреп, порушення сну. Об'єктивно спостерігається: на шкірі живота - поодинокі розеоли, Ps - 78/хв, печінка збільшена на 2 см. Поставте діагноз. A 28-year-old patient was hospitalized on the 9th day of the illness with complaints of an increase in temperature up to 39^oC, headache, general weakness, constipation, sleep disturbances. About' objectively observed: on the skin of the abdomen - isolated roseolae, Ps - 78/min, the liver is enlarged by 2 cm. Make a diagnosis.

Черевний тиф Typhoid

Сепсис Sepsis

Бруцельоз Brucellosis

Лептоспіроз Leptospirosis

Висипний тиф Typhoid

1283 / 1500
Завiдувач вiддiлення стацiонару хоче провести експертну оцiнку виконання лiкарями-ординаторами медико-технологiчних стандартiв обслуговування пацiєнтiв. Яку документацiю вiн повинен перевiрити з цiєю метою? The head of a hospital department wants to conduct an expert assessment of the fulfillment of medical and technological standards of patient care by resident doctors. What documentation should he check for this purpose?

Журнал облiку оперативних втручань Operational interventions accounting log

Статистичнi карти пацiєнтiв, що вибули iз стацiонару Statistical maps of patients discharged from the hospital

Рiчний звiт лiкувально-профiлактичного закладу Annual report of the treatment and prevention facility

Карти лiкарських призначень Maps of medical appointments

Медичнi карти стацiонарних хворих Medical charts of inpatients

1284 / 1500
Пацієнт госпіталізований у нейрохірургічне відділення з приводу закритої черепно-мозкової травми, перелому скроневої кістки справа. Через 5 годин його стан різко погіршився, виникли порушення дихання, періодичні тонічні судоми, анізокорія (розширення правої зіниці). Яке ускладнення можна припустити у пацієнта? The patient was hospitalized in the neurosurgery department due to a closed craniocerebral injury, a fracture of the temporal bone on the right. After 5 hours, his condition deteriorated sharply, breathing disorders, periodic tonic convulsions occurred , anisocoria (enlargement of the right pupil). What complications can be assumed in the patient?

Ішемічний інсульт в ділянці правої ніжки мозку Ischemic stroke in the area of ​​the right leg of the brain

Абсцес головного мозку Brain abscess

Особливості перебігу струсу головного мозку Features of concussion

Виникнення епідуральної гематоми Emergence of epidural hematoma

Субарахноїдальний крововилив Subarachnoid hemorrhage

1285 / 1500
Мати 22-місячного хлопчика звернулася до лікаря зі скаргами на погане набирання ваги та худобу у дитини. Годує грудним молоком на вимогу, не менше 5 разів на день. Вага та зріст хлопчика нижче 3-го стандартного відхилення для його віку. У нього дугоподібна деформація ніг та потовщені зап'ястя. При лабораторному дослідженні підвищений рівень лужної фосфатази. Дефіцит якого вітаміну найбільш імовірно спричинив подібний стан у дитини? The mother of a 22-month-old boy went to the doctor with complaints of poor weight gain and cattle in the child. Breastfeeds on demand, at least 5 times a day. Weight and the boy's height is below the 3rd standard deviation. He has bowed legs and thickened wrists. On laboratory examination, the level of alkaline phosphatase is most likely to cause this condition in the child?

Вітаміну PP Vitamin PP

Вітаміну A Vitamin A

Вітаміну E Vitamin E

Вітаміну B_6 Vitamin B_6

Вітаміну D Vitamin D

1286 / 1500
Дівчина 22-х років прийшла у жовтні до свого сімейного лікаря на профілактичний огляд. Самопочуття задовільне, скарг не має. Протягом 3-х років хворіє на добре контрольований цукровий діабет I типу. Її 70-річна бабуся хворіє на рак грудей. Останній ПАП-тест було зроблено рік тому, відхилень не зафіксовано. Її зріст - 172 см, вага - 61 кг, ІМТ - 20 кг/м^2. Показник глікованого гемоглобіну A1c складає 6,5%, останній показник глюкометра - 6,54 ммоль/л. Яка рекомендація лікаря цій пацієн-тці буде найбільш доречною? A 22-year-old girl came to her family doctor for a preventive check-up in October. She is feeling well, has no complaints. She has been suffering from well-controlled diabetes for 3 years Type I diabetes. Her 70-year-old grandmother has breast cancer. The last Pap test was done a year ago. Her height is 172 cm, and her BMI is 20 kg/m2 A1c is 6.5%, the last reading of the glucometer is 6.54 mmol/l. Which recommendation of the doctor would be most appropriate for this patient?

Пройти тест на вірус папіломи людини (ВПЛ-тест) Take a human papillomavirus test (HPV test)

Вакцинуватися від грипу Get vaccinated against the flu

Пройти мамографію Get a mammogram

Модифікація дієти з метою зниження ваги Diet modification for weight loss

Легкі аеробні вправи щодня Light aerobic exercise every day

1287 / 1500
Після перенесеного геморагічного інсульту хворий з метою відновлення мови та рухів лівої нижньої та верхньої кінцівок проходить курс масажу, фізпроцедур та занять з логопедом. До якого виду профілактики належать вказані заходи реабілітації? After suffering a hemorrhagic stroke, the patient undergoes a course of massage, physical procedures and classes with a speech therapist in order to restore speech and movements of the left lower and upper limbs. To what type of prevention do these measures belong rehabilitation?

Третинна профілактика Tertiary prevention

Громадська профілактика Public prevention

Вторинна профілактика Secondary prevention

Індивідуальна профілактика Individual prevention

Первинна профілактика Primary prevention

1288 / 1500
У жінки 75 років, яка веде помірно активний спосіб життя, під час профілактичного огляду виявлена сироваткова концентрація загального холестерину на рівні 5,1 ммоль/л (208 мг/дл) та ЛПВЩ - 70 мг/дл. ЕКГ без патології. Яка з наведених дієтичних рекомендацій показана жінці? In a 75-year-old woman who leads a moderately active lifestyle, during a preventive examination, a serum concentration of total cholesterol at the level of 5.1 mmol/l (208 mg/ dl) and HDL - 70 mg/dl. ECG without pathology. Which of the following dietary recommendations is shown to the woman?

Збільшити вживання клітковини Increase fiber intake

Зменшити прийом простих вуглеводів Reduce intake of simple carbohydrates

Зменшити прийом насичених жирів Reduce saturated fat intake

Зменшити прийом холестерину Reduce cholesterol intake

Ніяких змін у дієті No change in diet

1289 / 1500
Жінка 21 року звернулась до хірурга за 5 годин після опіку окропом лівого передпліччя. Об'єктивно спостерігається: на ділянці 17x10 см були виявлені в'ялі пухирі, місцями розірваний і зсунутий епідерміс, дно рани білувато-сіре, сухувате, больова чутливість знижена. Після лікування пов'язками з розчином фурациліну з левосином та з гіпозолем на 29-ту добу після опіку некротичний струп відділився, у ділянці опіку рожевий поверхневий рубець. Якого ступеня опік був у пацієнтки? A 21-year-old woman consulted a surgeon 5 hours after a scald burn of her left forearm. Objectively observed: flaccid blisters were found on an area of ​​17x10 cm, torn in places and the epidermis is shifted, the bottom of the wound is whitish-gray, pain sensitivity is reduced. After treatment with dressings with a solution of furacilin and with hyposol, on the 29th day after the burn, the necrotic scab separated, in the area of ​​the burn, what is the degree of the burn was at the patient's?

ІІІ А ступеня III A degree

ІV ступеня IV degree

І ступеня I degree

ІІІ Б ступеня III B degree

ІІ ступеня II degree

1290 / 1500
Дівчина 19-ти років скаржиться на помірний свербіж та випадіння волосся на голові. Об'єктивно: на шкірі потиличної ділянки голови одиничне еритематозне вогнище округлої форми з чіткими межами, діаметром 3 см, з азбестоподібним лущенням на поверхні. Волосся у вогнищі обламане на висоті 6-8 мм. Який діагноз є найбільш імовірним? A 19-year-old girl complains of moderate itching and hair loss on the head. Objectively: on the skin of the occipital part of the head, a single round erythematous focus with clear borders, 3 cm in diameter, with asbestos-like peeling on the surface. The hair in the focus is broken off at a height of 6-8 mm. What is the most likely diagnosis?

Трихофітія Trichophytia

Мікроспорія Microsporia

Себорея Seborrhea

Псоріаз Psoriasis

Короста Scabies

1291 / 1500
Жінка 43-х років скаржиться на біль в животі, що посилюється нападами, нудоту, багаторазове блювання застійним кишковим вмістом, здуття живота, затримку газів. Хворіє 7 годин. Пульс - 116/хв. Язик сухий, коричневий. Живіт симетрично здутий, м'який, болючий. Перкуторно: тимпаніт. Аускультативно: кишкові шуми з металевим відтінком, шум сплеску, шум падаючої краплі. Який найбільш імовірний діагноз? A 43-year-old woman complains of abdominal pain that worsens with attacks, nausea, repeated vomiting of stagnant intestinal contents, abdominal distension, gas retention. She has been ill for 7 hours. Pulse - 116/min. Tongue is symmetric, soft, painful. Auscultation: guttural sounds, noise of a falling drop?

Гострий ерозивний гастрит Acute erosive gastritis

Гострий некротичний панкреатит Acute necrotizing pancreatitis

Гострий неспецифічний коліт Acute nonspecific colitis

Гострий деструктивний холецистит Acute destructive cholecystitis

Гостра кишкова непрохідність Acute intestinal obstruction

1292 / 1500
Чоловік 45 років доставлений до клініки у тяжкому стані. До госпіталізації хворів протягом 3 тижнів пневмонією. Об'єктивно встановлено: шкіра та слизові темно-землистого кольору, температура тіла - 38^oС, задишка у спокої, дихання зліва різко ослаблено. Спостерігається кашель із рясним харкотанням. Рентгенологічно виявлено затемнення лівого геміторакса. Який діагноз найімовірніший? A 45-year-old man was brought to the clinic in serious condition. Before hospitalization, he had been ill for 3 weeks with pneumonia. Objectively established: dark earthy skin and mucous membranes, body temperature - 38°C, shortness of breath on the left side. There is a cough with profuse sputum. X-ray revealed the darkening of the left hemithorax. What is the most likely diagnosis?

Пневмонія Pneumonia

Емпієма плеври Empyema of the pleura

Пухлина легені Lung tumor

Бронхіт Bronchitis

Пневмоторакс Pneumothorax

1293 / 1500
Жінка 62 рокiв захворiла гостро з пiдвищенням температури до 39,8^oC, ознобом, iнтенсивним головним болем, ломотою в тiлi. Скарги на нудоту, одноразове блювання. За 18 годин вiдзначила збiльшення i рiзку болючiсть пахвинних лiмфовузлiв справа. Ще за 6 годин з'явились набряк правої гомiлки i гiперемiя шкiри із чiткими нерiвними контурами. Який діагноз найімовірніший? A 62-year-old woman became acutely ill with an increase in temperature to 39.8^oC, chills, intense headache, body aches. Complaints of nausea, one-time vomiting. For 18 hours later, she noticed an increase in the inguinal lymph nodes on the right side. In another 6 hours, swelling of the right lower leg and skin hyperemia with clear uneven contours appeared. What is the most likely diagnosis?

Туляремія, бубонна форма Tularemia, bubonic form

Чума, бубонна форма Plague, bubonic form

Бешиха, еритематозна форма Rapid, erythematous form

Сибiрка, едематозна форма Anthrax, edematous form

Флегмона правої гомiлки Phlegmon of the right shin

1294 / 1500
Вагітна віком 25 років шпиталізована у зв'язку з матковою кровотечею. Загальна крововтрата - 250 мл. Після проведеного обстеження встановлено: вагітність 10-11 тижнів, самовільний викидень в ходу. Укажіть лікувальну тактику у цьому разі. A 25-year-old pregnant woman was hospitalized due to uterine bleeding. The total blood loss was 250 ml. After the examination, it was established: pregnancy of 10-11 weeks, spontaneous abortion in move. Indicate the treatment tactics in this case.

Провести гемотрансфузію Perform blood transfusion

Суворий ліжковий режим Strict bed rest

Призначити спазмолітики Prescribe antispasmodics

Призначити утеротонічні препарати Prescribe uterotonic drugs

Провести видалення плодового яйця Remove the fetal egg

1295 / 1500
У пацієнта віком 30 років після вживання грибів виникли нудота і блювання, зникла сеча. На третій день симптомів звернувся за медичною допомогою. Під час лабораторних досліджень виявлено: рівень креатиніну підвищився до 700 мкмоль/л, сечовина - до 32 ммоль/л. Якою буде лікувальна тактика в цій ситуації? A 30-year-old patient developed nausea and vomiting after eating mushrooms, and urine disappeared. On the third day of symptoms, he sought medical help. Laboratory tests revealed: creatinine level increased to 700 μmol/l, urea - up to 32 mmol/l. What will be the treatment tactics in this situation?

Гемодіаліз Hemodialysis

Дезінтоксикаційна терапія Detoxification therapy

Сечогінні препарати Diuretics

Перитонеальний діаліз Peritoneal dialysis

Антидотна терапія Antidote therapy

1296 / 1500
У чоловіка 73 років на тлі гострого трансмурального переднього інфаркту міокарда спостерігається зниження систолічного тиску (70 мм рт. ст), діурезу (100 мл/добу), підвищення креатиніну крові (480 мкмоль/л). У сечі спостерігається протеїнурія (0,066 г/добу). Чим зумовлене зниження діурезу? A 73-year-old man with an acute transmural anterior myocardial infarction has decreased systolic pressure (70 mm Hg), diuresis (100 ml/day), increased creatinine blood (480 μmol/l). Proteinuria is observed in the urine (0.066 g/day). What is the reason for the decrease in diuresis?

Гостра ренальна ниркова недостатність Acute renal failure

<<Застійна>> нирка <> kidney

Гострий тубулоінтерстиціальний нефрит Acute tubulointerstitial nephritis

Гломерулонефрит, що швидко прогресує Rapidly progressive glomerulonephritis

Гостра преренальна ниркова недостатність Acute prerenal renal failure

1297 / 1500
У хлопчика 3-х років під час ходьби та бігу збільшується права половина мошонки, яка в положенні лежачи зменшує-ться до нормальних розмірів. Діаг-ноз: In a 3-year-old boy, the right half of the scrotum increases during walking and running, which decreases to normal size when lying down. Diagnosis:

Парафімоз Paraphimosis

Фімоз Phimosis

Орхіепідидиміт Orchiepididymitis

Сполучна водянка оболонок правого яєчка Conjunctive dropsy of the membranes of the right testicle

Цистоцеле Cystocele

1298 / 1500
Чоловік 50-ти років звернувся до лікаря зі скаргами на біль в стегні після падіння на сходах. Піднятись та ходити не може. Права нога коротша за ліву на 5 см, ступня ротована назовні. Пульс на правій тильній артерії стопи задовільний. Стегно збільшене в об'ємі, деформоване, в середній третині болюча пальпація. Виявляється патологічна рухомість, крепітація. Який діагноз є найбільш імовірним? A 50-year-old man went to the doctor complaining of hip pain after falling down the stairs. He can't get up and walk. The right leg is 5 cm shorter than the left , the foot is rotated outwards. The pulse on the right back artery is satisfactory. The thigh is enlarged, painful palpation. What is the most likely diagnosis?

Хибний суглоб False joint

Закритий перелом кісток миски Closed pelvic fracture

Забій сідничного нерва Contusion of sciatic nerve

Вивих правого стегна Dislocation of right hip

Перелом діафіза стегнової кістки Fracture of the diaphysis of the femur

1299 / 1500
Пацієнтка віком 35 років скаржиться на часте, болюче, утруднене сечовипускання, постійні позиви до нього, появу кількох крапель крові у сечі в кінці процесу. Захворіла раптово після переохолодження, коли з'явились вищеописані скарги. Температура тіла - 36,6^oС. Під час пальпації спостерігається: болючість в ділянці сечового міхура. Під час УЗД виявлено: в сечовому міхурі мало сечі, стінки сечового міхура набряклі, рівномірно потовщені. В аналізах сечі спостерігається: лейкоцитурія (30-40 в полі зору), протеїнурія (0,099 г/л), еритроцитурія (5-7 незмінених еритроцитів). Який найімовірніший діагноз? A 35-year-old patient complains of frequent, painful, difficult urination, constant urges to urinate, the appearance of several drops of blood in the urine at the end of the process. She fell ill suddenly after hypothermia, when the above complaints appeared. Body temperature - 36.6°C. During palpation, it was found that there was little urine in the bladder, and uniform thickening of the bladder : leukocyturia (30-40 in the field of vision), proteinuria (0.099 g/l), erythrocytosis (5-7 unchanged erythrocytes). What is the most likely diagnosis?

Пухлина сечового міхура Bladder tumor

Конкремент сечового міхура Bladder calculus

Гострий цистит Acute cystitis

Уретероцеле Ureterocele

Гострий сальпінгоофорит Acute salpingo-oophoritis

1300 / 1500
До лікаря-педіатра звернулася мати з дитиною 3,5 років. Дитина 4-у добу хворіє, лихоманка в межах 38,7-39,2^oС. Захворювання почалося гостро з сухого кашлю. На другу добу захворювання з'явилися: одутлість обличчя, світлобоязнь та гнійний двобічний кон'юнктивіт. На четверту добу на тлі температури тіла 39,4^oС виникли плямисто-папульозні висипання за вухами та на обличчі. Дихання через ніс утруднене, у нижніх носових ходах невеликі слизисті виділення. У сім'ї є молодший брат віком 7 місяців. Які заходи захисту для цієї дитини повинен рекомендувати сімейний лікар? A mother with a 3.5-year-old child consulted a pediatrician. The child has been ill for the 4th day, with a fever of 38.7-39.2°C. The disease began acutely with a dry cough. On the second day, swelling of the face, photophobia, and purulent conjunctivitis appeared on the background of a body temperature of 39.4°C it is difficult through the nose, there is a small mucous discharge in the lower nasal passages. The family has a younger brother aged 7 months. What protective measures should the family doctor recommend for this child?

Профілактики не потребує, оскільки дитина захищена трансплацентарно Prophylaxis is not required, as the child is transplacentally protected

Провести вакцинацію від кору, краснухи, паротиту (КПК) Vaccinate against measles, rubella, mumps (KPK)

Призначити вітамін А 100 000 МО Prescribe vitamin A 100,000 IU

Виконати дом'язеву ін'єкцію 1,5 мл протикоревого імуноглобуліну Perform an intramuscular injection of 1.5 ml of measles immunoglobulin

Провести хіміопрофілактику Conduct chemotherapy

1301 / 1500
Хворий 42-х років доставлений до травматологічного відділення 2 години тому. Рентгенологічно: перелом кісток тазу. Об'єктивно: виведений з шоку. Самостійно не мочиться. Уретрорагія. Пальпується збільшений сечовий міхур. Болісна припухлість в ділянці промежини. Який попередній діагноз? A 42-year-old patient was brought to the trauma department 2 hours ago. X-ray: pelvic bone fracture. Objectively: brought out of shock. He does not urinate on his own. Urethrorrhagia. An enlarged bladder is palpated. Painful swelling in the perineum. What is the previous diagnosis?

Травма сечового міхура Bladder injury

Гостра ниркова недостатність Acute renal failure

Забій промежини Perineal bruise

Рефлекторна затримка сечовипускання Reflex micturition delay

Травма уретри Urethra trauma

1302 / 1500
До ортопеда звернулись батьки 3-мі-сячної дитини зі скаргами на асиметрію шкірних складок правого стегна. Під час огляду виявлено: різна кількість складок шкіри на правому та лівому стегні, їх асиметрія, обмеження розведення правої нижньої кінцівки зігнутої під кутом 90^o в кульшовому та колінному суглобах. Який діагноз є найбільш імовірним? Parents of a 3-month-old child turned to the orthopedist with complaints about the asymmetry of the skin folds of the right thigh. During the examination, it was found: a different number of skin folds on the right and left thigh , their asymmetry, limitation of extension of the right lower limb bent at an angle of 90° in the hip and knee joints. What diagnosis is most likely?

Контрактура кульшового суглоба Hip joint contracture

Вроджений вивих стегна Congenital hip dislocation

- -

Дисплазія кульшового суглоба Hip dysplasia

Ушкодження кульшового суглоба під час пологів Injury of the hip joint during childbirth

1303 / 1500
Вагітна жінка віком 19 років перебуває у відділенні патології вагітності з приводу вагітності в терміні 36 тижнів, тазове передлежання, великий плід, важка форма цукрового діабету. На КТГ у плода зареєстрована брадикардія до 90/хв. Пологової діяльності немає. Яка тактика ведення вагітності в цій ситуації? A 19-year-old pregnant woman is in the department of pregnancy pathology due to a 36-week pregnancy, breech presentation, large fetus, severe form of diabetes mellitus. CTG of the fetus registered bradycardia up to 90/min. There is no labor activity. What are the tactics of pregnancy management in this situation?

Родостимуляція Rodostimulation

Продовження вагітності Continuation of pregnancy

Виведення плода за тазовий кінець Expulsion of the fetus by the pelvic end

Розродження через природні пологові шляхи Birth through natural birth canals

Терміновий кесарів розтин Urgent cesarean section

1304 / 1500
У новонародженої дитини на третій день життя з'явилася жовтяниця шкіри та склер, загальний стан не порушений, сон і апетит без відхилень. З анамнезу відомо: дитина від ІІ пологів, у матері група крові - А (ІІ) Rh(+), у дитини - А (ІІ) Rh(+). Який стан виник у новонародженої дитини? On the third day of life, a newborn child developed jaundice of the skin and sclera, the general condition is not impaired, sleep and appetite are normal. It is known from the anamnesis: the child from II childbirth, the mother's blood group is A (II) Rh(+), the child's blood group is A (II) Rh(+). What condition did the newborn have?

Гемолітична хвороба Hemolytic disease

Пологова травма Birth trauma

Інфекційний гепатит Infectious hepatitis

Фізіологічна жовтяниця Physiological jaundice

Сепсис Sepsis

1305 / 1500
У військового 34 років, страх, тривога, роздратованість, пригнічений настрій, жахливі сновидіння про минулі події з'явилися за 6 місяців після перенесеного бойового стресу. Об'єктивно встановлено: занурений у світ власних переживань та уявлень, тривожний, сумний, відсторонений від інших пацієнтів, спостерігаються <<флешбеки>>. Який попередній діагноз? A 34-year-old military man, fear, anxiety, irritability, depressed mood, terrible dreams about past events appeared 6 months after experiencing combat stress. Objectively established: immersed in the world of own experiences and ideas, anxious, sad, withdrawn from other patients, <> are observed. What is the previous diagnosis?

Соматизований розлад Somatized disorder

Депресивний розлад Depressive disorder

Гостра реакція на стрес Acute reaction to stress

Посттравматичний стресовий розлад Post-traumatic stress disorder

Іпохондричний розлад Hypochondriacal disorder

1306 / 1500
У пацієнта, що хворіє на епілепсію, після самостійного припинення вживання протиепілептичних медикаментів раптово виник епілептичний статус генералізованих судомних нападів. Вкажіть препарати першої лінії для лікування цього стану. A patient with epilepsy suddenly developed status epilepticus with generalized seizures after self-discontinuation of antiepileptic drugs. Specify the first-line drugs for the treatment of this condition.

Топірамат, окскарбазепін, карбамазепін Topiramate, oxcarbazepine, carbamazepine

Левопромазин, клозапін, кветіапін Levopromazine, Clozapine, Quetiapine

Діазепам, лоразепам, мідазолам Diazepam, Lorazepam, Midazolam

Доксепін, амітриптилін, міансерин Doxepin, amitriptyline, mianserin

Габапентин, прегабалін, етосуксимід Gabapentin, Pregabalin, Ethosuximide

1307 / 1500
Дівчина 22-х років звернулася до лікаря зі скаргами на задишку, загрудинний біль, відчуття страху та ''стороннього тіла'' у горлі. Зазначені симптоми з'явилися в момент перегляду фільму з друзями і вже протягом 40 хвилин зберігаються і не зникають. Пацієнтка стверджує, що подібний напад у неї не вперше, але раніше вона могла опанувати цей стан та заспокої-тися. При фізикальному обстеженні перед лікарем тремтяча та спітніла дівчина, температура тіла - 37,5^oC, артеріальний тиск - 130/90 мм рт.ст., пульс - 112/хв., частота дихання - 18/хв., SpO_2 - 99% при кімнатному повітрі. Лікар не може заспокоїти пацієнтку. Вона стверджує, що помирає та благає про допомогу. Який наступний крок лікаря буде найбільш доречним? A 22-year-old girl consulted a doctor with complaints of shortness of breath, chest pain, a feeling of fear and a 'foreign body' in her throat. The indicated symptoms appeared at the time of watching a movie with friends, they persist for 40 minutes and do not disappear. The patient claims that this is not the first time she has had such an attack, but she was able to control this condition and calm down. During the physical examination in front of the doctor, the girl shivered and sweated body temperature - 37.5°C, blood pressure - 130/90 mmHg, pulse rate - 18/min, SpO_2 - 99% on room air. The doctor cannot calm the patient down claims to be dying and begs for help. What would be the most appropriate next step for the doctor?

Ввести метопролол внутрішньовенно Enter metoprolol intravenously

Призначити нітрогліцерин перорально та спостерігати мінімум 5 хвилин Give nitroglycerin orally and observe for at least 5 minutes

Призначити аспірин перорально та доставити до найближчого реперфузійного центру Prescribe aspirin orally and deliver to the nearest reperfusion center

Ввести феназепам внутрішньовенно Inject phenazepam intravenously

Розпочати курс флуоксетину перорально Start oral fluoxetine course

1308 / 1500
На зупинці автобуса чоловік похилого віку схопився за серце, а потім втратив свідомість. Декілька разів глибоко вдихнув з інтервалом у 8-10 секунд, а потім перестав дихати. Шкіра бліда, холодна. Пульс на сонних артеріях не визначається. Зіниці розширені, реагують на світло. Діагностована раптова коронарна смерть. З чого варто почати серцево-легеневу реанімацію? At the bus stop, an elderly man clutched his heart and then lost consciousness. He took several deep breaths at intervals of 8-10 seconds and then stopped breathing. Skin pale, cold. The pulse on the carotid arteries is not determined. The pupils react to light. What should I start cardiopulmonary resuscitation with?

Непрямий масаж серця Indirect heart massage

Трахеостомія Tracheostomy

Прийом Вальсальви Valsalva Reception

Массаж каротидного синуса Carotid sinus massage

Штучна вентиляція легень Artificial lung ventilation

1309 / 1500
У пацієнтки віком 29 років, яка скаржиться на безпліддя та порушення менструального циклу по типу олігоменореї, під час обстеження виявлено: зріст - 160 см, маса тіла - 91 кг, відзначається ріст волосся на обличчі, стегнах. Під час бімануального дослідження з обох сторін виявлено збільшені щільної консистенції яєчники, розмірами 5x6 см. Ці ж дані підтверджені під час УЗД. Яка причина скарг жінки? In a 29-year-old female patient who complains of infertility and irregular menstrual cycle type oligomenorrhea, during the examination it was found: height - 160 cm, body weight - 91 kg , the growth of hair on the face and thighs is noted. During the bimanual examination, enlarged ovaries, measuring 5x6 cm, were found. The same data were confirmed during the ultrasound. What is the reason for the woman's complaints?

Синдром склерокистозних яєчників (Штейна-Левенталя) Syndrome of sclerocystic ovaries (Stein-Leventhal)

Хронічний двохсторонній аднексит Chronic bilateral adnexitis

Передменструальний синдром Premenstrual syndrome

Андробластома яєчників Androblastoma of the ovaries

Адреногенітальний синдром Adrenogenital syndrome

1310 / 1500
Пацієнт 18 років хворіє з раннього дитинства. Під час обстеження встановлено: Нb - 110 г/л, еритроцити - 3,9·10^12/л, КП - 0,8, лейкоцити - 6,0·10^9/л; ШOЕ - 30 мм/год. Коагулограма показала: протромбіновий індекс - 95%, ретракція кров'яного згустка - 50%, час згортання крові - за 40 хвилин не відбулось, тривалість кровотечі - 3 хв. Які механізми складають основу патогенезу цього захворювання? An 18-year-old patient has been sick since early childhood. During the examination, the following was found: Hb - 110 g/l, erythrocytes - 3.9·10^12/l, CP - 0.8, leukocytes - 6.0·10^9/l; ESR - 30 mm/h. The coagulogram showed: prothrombin index - 50%, blood clotting time - 40 minutes. occurred, duration of bleeding - 3 minutes. What mechanisms form the basis of the pathogenesis of this disease?

Дефіцит вітаміну С Vitamin C deficiency

Недостатність у крові антигемофільного глобуліну А Antihemophilic globulin A deficiency in the blood

Екзогенний дефіцит заліза Exogenous iron deficiency

Імунне пригнічення кісткового мозку Bone marrow immune suppression

Наявність специфічних антитіл до ендотеліальних стінок судин Presence of specific antibodies to the endothelial walls of vessels

1311 / 1500
У хворої 65-ти років протягом 5 тижнів прогресує безбольова жовтяниця, шкірне свербіння, схудла на 10 кг, ахолія. Пальпується позитивний симптом Курвуа-зьє. Який попередній діагноз? A 65-year-old patient developed painless jaundice, skin itching, weight loss of 10 kg, acholia for 5 weeks. A positive Courvoisier symptom is palpated. What is the preliminary diagnosis ?

Рак підшлункової залози Pancreatic cancer

Рак печінки Liver cancer

Вірусний гепатит Viral hepatitis

Рак жовчного міхура Gall Bladder Cancer

Малярія Malaria

1312 / 1500
Чоловік 36 років скаржиться на постійний тупий біль у лівому підребер'ї після прийому жирної та копченої їжі, блювоту, яка не приносить полегшення. Калові маси блискучі, з неприємним запахом. Хворіє протягом 8 років; зловживає алкоголем, багато курить. Об'єктивно встановлено: астенічної тілобудови, шкіра бліда і суха, язик з білим налітом, живіт помірно здутий. Під час пальпації - біль у зонах Шоффара, Губергриця-Скульського, точках Дежардена, Мейо-Робсона. Про яке захворювання можна подумати насамперед? A 36-year-old man complains of constant dull pain in the left hypochondrium after eating fatty and smoked food, vomiting that does not bring relief. The stools are shiny, with unpleasant He has been sick for 8 years, he is a heavy smoker, he has an asthenic body, a white plaque, and pain in the areas of Shoffar, Gubergritsa, and Skulsky. , Mayo-Robson. What disease can you think of first?

Хронічний панкреатит у стадії загострення Chronic pancreatitis in the acute stage

Хронічний гастродуоденіт у стадії загострення Chronic gastroduodenitis in the acute stage

Рак голівки підшлункової залози Cancer of the head of the pancreas

Виразкова хвороба у стадії загострення Ulcer disease in the acute stage

Хронічний холецистит у стадії загострення Chronic cholecystitis in the acute stage

1313 / 1500
Чоловікові 72 років у зв'язку з миготливою аритмією та проявами недостатності кровоообігу призначено фуросемід, дігоксин, інгібітори АПФ. На 5-й день із початку лікування з'явились ниючі болі в животі, нудота, одноразове блювання. На ЕКГ виявлено: шлуночкова бігемінія, коритоподібне зміщення сегменту ST нижче ізолінії. Яке ускладнення найімовірніше виникло у хворого? A 72-year-old man was prescribed furosemide, digoxin, and ACE inhibitors in connection with atrial fibrillation and manifestations of circulatory failure. On the 5th day from the start of treatment, Abdominal pain, nausea, one-time vomiting. The ECG revealed: ventricular bigeminia, trough-like shift of the ST segment below the isoline. What complication most likely occurred in the patient?

Інтоксикація фуросемідом Furosemide intoxication

Медикаментозний гастрит Medicated gastritis

Гіперкаліємія Hyperkalemia

Інфаркт міокарду (абдомінальний варіант) Myocardial infarction (abdominal variant)

Дигіталісна інтоксикація Digital intoxication

1314 / 1500
Дівчинка 13 років надійшла до гінекологічного відділення зі скаргами на значні кров'янисті виділення зі статевих шляхів протягом 10 днів. З анамнезу відомо: менархе з 11 років, менструальний цикл нерегулярний. Під час ректо-абдомінального обстеження патології не виявлено. Який попередній діагноз? A 13-year-old girl came to the gynecological department with complaints of significant bleeding from the genital tract for 10 days. It is known from the anamnesis: menarche since the age of 11, menstrual cycle irregular. During the recto-abdominal examination, no pathology was detected. What was the previous diagnosis?

Поліп ендометрія Endometrial polyp

Хвороба Верльгофа Werlhof's disease

Ювенільна маткова кровотеча Juvenile uterine bleeding

Аденоміоз Adenomyosis

Травма зовнішніх статевих шляхів Injury of the external genital tract

1315 / 1500
До сімейного лікаря звернулася жінка 35-ти років зі скаргами на рясні менструації, які тривають до 10 днів. При гінекологічному огляді виявлено: шийка матки не змінена, матка в anterflexio, звичайних розмірів, рухома, неболюча. Придатки з обох боків без особливостей. Сімейний лікар поставив попередній діагноз: аномальна маткова кровотеча. Який із вказаних інструментальних методів є першочерговим у діагностиці цієї патології? A 35-year-old woman came to the family doctor with complaints of heavy menstruation that lasts up to 10 days. During a gynecological examination, it was found that the cervix is ​​unchanged, the uterus in anterflexio, mobile, painless. The family doctor made a preliminary diagnosis: abnormal uterine bleeding. Which of the indicated instrumental methods is the first priority in the diagnosis of this pathology?

УЗД трансвагінальне Transvaginal ultrasound

УЗД трансабдомінальне Transabdominal ultrasound

Лапароскопія Laparoscopy

Кольпоскопія Colposcopy

Кульдоскопія Culdoscopy

1316 / 1500
Другі пологи у повторнороділлі в терміні 36-37 тижнів. Навколоплідні води вилились 8 годин тому, пологова діяльність триває 4 години, регулярна, перейми через 3-4 хвилини по 35 секунд. Передлегла голівка притиснута до входу в малий таз. Роділля поскаржилась на раптовий різкий біль в животі. Пульс - 100/хв., АТ- 110/70-100/70 мм рт.ст. Матка напружена, не розслабляється поза переймами. Серцебиття плода приглушене - 100/хв. Підтікають навколоплідні води, забарвлені кров'ю. Вкажіть найбільш імовірний діагноз: Second birth in repeated labor at 36-37 weeks. Amniotic fluid spilled 8 hours ago, labor lasts 4 hours, regular, takes after 3-4 minutes 35 seconds. The head is pressed against the entrance to the pelvis. The woman in labor complained of sudden pain in the abdomen. Pulse - 110/70/70 mm Hg. The uterus does not relax. The fetal heartbeat is muffled - 100/min. Amniotic fluid is flowing, stained with blood. Specify the most likely diagnosis:

Розрив судин пуповини Rupture of umbilical cord vessels

Передчасне відшарування нормально розташованої плаценти Premature detachment of a normally located placenta

Розрив матки Rupture of uterus

Розрив шийки матки Rupture of the cervix

Неповне передлежання плаценти Incomplete placenta previa

1317 / 1500
До лікарні шпиталізовано вагітну у терміні гестації 38 тижнів, вагітність друга. Перша вагітність завершилася природніми пологами, народився хлопчик вагою 3600 гр. У жінки було виявлено ВІЛ-позитивний статус під час другої вагітності на 12 тижні гестації. Жінка отримує ААРТ із 22 тижня вагітності. Положення плода поздовжнє, головне передлежання. Передбачувана вага 3200pm200 гр. Серцебиття плода звучне, ритмічне, 122-138/хв. Вірусне навантаження становить 40 вірусних копій/мл. Яка тактика ведення пологів у цьому разі? A pregnant woman was hospitalized at 38 weeks' gestation, second pregnancy. The first pregnancy ended in natural childbirth, a boy weighing 3600 g was born. The woman was found to be HIV-positive During the 12th week of pregnancy, the woman receives AART from the 22nd week of pregnancy. What are the delivery tactics in this case?

Вести пологи консервативно Give birth conservatively

Провести індукцію пологів на 38 тижні вагітності Induce labor at 38 weeks of pregnancy

Кесаревів розтин на 38 тижнів вагітності Caesarean section at 38 weeks of pregnancy

Кесарів розтин на 40 тижнів вагітності Caesarean section at 40 weeks of pregnancy

Виключити ІІ період пологів Exclude II period of childbirth

1318 / 1500
Жінка 45 років, яка рік тому перенесла резекцію щитоподібної залози з приводу багатовузлового зобу, скаржиться на загальну слабкість, сонливість, постійне відчуття втоми, зниження працездатності, закрепи, сонливість. Температура тіла - 36^oC. Шкірні покрови сухі, морщиністі, почало випадати волосся. Аменорея. Який стан виник у пацієнтки? A 45-year-old woman who underwent resection of the thyroid gland a year ago due to a multinodular goiter complains of general weakness, drowsiness, a constant feeling of fatigue, reduced work capacity, constipation, drowsiness The body temperature is 36°C. The skin is wrinkled, the hair has started to fall out. What condition did the patient have?

Хронічний тиреоідит Chronic thyroiditis

Клімакс Climax

Гіпопаратиреоз Hypoparathyroidism

Гіпотиреоз Hypothyroidism

Тиреотоксикоз Thyrotoxicosis

1319 / 1500
У пацієнта 42-х років виник ''кинджальний'' біль в епігастральній ділянці, що thickspace поширився thickspace через thickspace кілька thickspace годин thickspace над thickspace всією поверхнею живота. Симптоми подразнення очеревини позитивні. Протягом багатьох років страждає на виразкову хворобу дванадцятипалої кишки. При рентгенографії живота: вільний газ під куполами діафрагми. Ваш імовірний діагноз: A 42-year-old patient developed a 'dagger' pain in the epigastric region that thickspace spread thickspace through thickspace for several thickspace hours thickspace over thickspace over the entire surface of the abdomen. Symptoms of peritoneal irritation have been suffering from duodenal ulcer for many years. Abdominal X-ray: free gas under the domes of the diaphragm:

Виразкова хвороба дванадцятипалої кишки Duodenal ulcer

Перфорація порожнистого органу. Перитоніт Perforation of a hollow organ. Peritonitis

Хронічний гастрит Chronic gastritis

Виразкова хвороба шлунка в стадії загострення Gastric ulcer disease in the acute stage

Гострий панкреатит Acute pancreatitis

1320 / 1500
Хвора 42-х років після відпочинку, встала з ліжка і відчула загальну слабкість, запаморочення, потемніння в очах, втратила свідомість. Об'єктивно: хвора непритомна, шкіра бліда, холодна, зіничні та сухожильні рефлекси збережені. АТ- 75/50 мм рт.ст., пульс - 100/хв. Який діагноз найбільш імовірний? A 42-year-old patient after rest got out of bed and felt general weakness, dizziness, darkening of the eyes, lost consciousness. Objectively: the patient is unconscious, the skin pale, cold, pupillary and tendon reflexes preserved. Blood pressure - 75/50 mmHg, pulse - 100/min. What is the most likely diagnosis?

Істеричний напад Hysterical attack

Епілептичний синдром Epileptic syndrome

Ортостатичний колапс Orthostatic collapse

Гіпоглікемічна кома Hypoglycemic coma

Ішемічний інсульт Ischemic stroke

1321 / 1500
Жінка 42-х років звернулася до лікаря зі скаргами на м'язову слабкість у верхніх і нижніх кінцівках. Об'єктивно: у періорбітальних ділянках еритема з бузковим відтінком, макульозна еритема на розгинальних поверхнях пальців. В аналізі крові вия-влено суттєве підвищення рівня креатинфосфокінази. Яким є імовірний діа-гноз? A 42-year-old woman consulted a doctor with complaints of muscle weakness in the upper and lower limbs. Objectively: in the periorbital areas erythema with a lilac tint, macular erythema on the extensor surfaces of the fingers. A significant increase in the level of creatine phosphokinase was detected in the blood test. What is the probable diagnosis?

Гіпопаратиреоз Hypoparathyroidism

Дерматоміозит Dermatomyositis

Системна склеродермія Systemic scleroderma

Хвороба Кушинга Cushing's disease

Системний червоний вовчак Systemic lupus erythematosus

1322 / 1500
Чоловік 57 років скаржиться на кашель із виділенням великої кількості слизово-гнійного харкотиння (до 150 мл/добу). Об'єктивно спостерігається: пальці у вигляді <<барабанних паличок>>, нігті у вигляді <<годинникових стекол>>. Під час перкусії над легенями визначається вкорочений перкуторний звук, аускультативно вислуховуються велико- та середньопухирчасті вологі хрипи. Результат ЗАК: лейкоцитоз і зсув лейкоцитарної формули вліво. Рентгенографія ОГК показала: посилений легеневий малюнок, кільцевидні тіні. Бронхографія виявила: множинні циліндричні потовщення бронхів з чіткими контурами. Який попередній діагноз? A 57-year-old man complains of a cough with a large amount of mucous-purulent sputum (up to 150 ml/day). Objectively observed: fingers in the form of <>, fingernails in the form of <>. During percussion over the lungs, a short percussion sound is heard, and auscultatory sounds show: leukocytosis and a shift of the leukocyte formula to the left , ring-shaped shadows. Bronchography revealed: multiple cylindrical thickening of the bronchi with clear contours?

Ехінокок легені Echinococcus lung

Бронхоектатична хвороба Bronchoectatic disease

Кіста легені Lung cyst

Хронічна емпієма плеври Chronic pleural empyema

Гангрена легені Gangrene of the lung

1323 / 1500
Хлопчик 9-ти років упав з дерева, вдарився потиличною ділянкою, спостерігалась короткочасна втрата свідомості. Стан дитини задовільний, турбують головний біль, запаморочення. На рентгенограмах черепа виявлено втиснений уламчастий перелом потиличної кістки в ділянці потиличного горба. Яка лікувальна тактика показана хворому? A 9-year-old boy fell from a tree, hit the occipital region, a short-term loss of consciousness was observed. The child's condition is satisfactory, he is bothered by headache, dizziness. X-rays of the skull revealed an imprinted fragmentary fracture of the occipital bone in the area of ​​the occipital hump. What treatment tactics are indicated for the patient?

Оперативне втручання Operative intervention

Гемостатична терапія Hemostatic therapy

Протизапальна терапія Anti-inflammatory therapy

Комплексне консервативне лікування Complex conservative treatment

Розвантажувальні люмбальні пункції Unloading lumbar punctures

1324 / 1500
Жінка за 10 днів після травми ділянки стопи відчула утруднення під час відкриття рота. Наступного дня з важкістю могла приймати їжу, з'явилося напруження м'язів потилиці, спини, живота. Під час прийому в стаціонар на 3 день хвороби встановлено тонічне напруження всіх груп м'язів, генералізовані судоми кожні 10-15 хвилин. Який найбільш імовірний діагноз? 10 days after the injury to the foot, the woman felt difficulty opening her mouth. The next day, she could hardly take food, there was tension in the muscles of the back of the head and back , abdomen. During admission to the hospital on the 3rd day of the illness, tonic tension of all muscle groups was established, generalized convulsions every 10-15 minutes. What is the most likely diagnosis?

Тетанія Tetany

Менінгітоенцефаліт Meningoencephalitis

Правець Tetanus

Епілепсія Epilepsy

Геморагічний інсульт Hemorrhagic stroke

1325 / 1500
Жінка 34 років звернулася до лікаря з приводу м'язової слабкості, спраги, підвищення сечовиділення в нічний час, парестезій, нападів судом. Об'єктивно встановлено: загальний стан задовільний, пастозність обличчя, ніг. Ps - 80/хв. АТ - 200/110 мм рт. ст., акцент ІІ тону над аортою. У крові виявлено: К^+ - З,1 ммоль/л, Na^+ - 165 ммоль/л. На ЕКГ: інверсія зубця Т, депресія сегменту S-T. Під час УЗД спостерігалася: гіперплазія правої надниркової залози. Який попередній діагноз? A 34-year-old woman consulted a doctor about muscle weakness, thirst, increased urination at night, paresthesias, seizures. Objectively established: general condition Satisfactory, blood pressure of the legs - 80/min, pressure of II tone above the aorta: K^+ - 165 mmol/l. On the ECG: inversion of the ST segment. During the ultrasound: hyperplasia of the right adrenal gland was observed?

Гіпопаратиреоз Hypoparathyroidism

Феохромоцитома Pheochromocytoma

Первинний гіперальдостеронізм Primary hyperaldosteronism

Есенціальна гіпертонія Essential hypertension

Глюкостерома Glucosteroma

1326 / 1500
У жінки 20-ти років головний біль, запаморочення, плаксивість, блювання, біль в ділянці серця, тахікардія, яка наростає за 6-7 днів до менструації. У перші дні менструації симптоми зникають. Поставте діагноз: A 20-year-old woman has a headache, dizziness, tearfulness, vomiting, heart pain, tachycardia, which increases 6-7 days before menstruation. the first days of menstruation, the symptoms disappear. Make a diagnosis:

Діенцефальний синдром Diencephalic syndrome

Альгоменорея Algomenorrhea

Синдром Штейна-Левенталя Stein-Leventhal syndrome

Метаболічна краніопатія Metabolic craniopathy

Передменструальний синдром Premenstrual syndrome

1327 / 1500
Жiнка віком 25 рокiв була госпіталізована в терміні вагітності 11 тижнів зі скаргами на біль внизу живота. З анамнезу відомо, що 2 тижнi тому вона перехворiла на краснуху в тяжкiй формi. Під час бiмануального дослiдження виявлено: шийка матки сформована, вiчко шийки матки закрите, матка збільшена до 11 тижнів вагiтності, придатки матки без особливостей. Яка лікарська тактика є найдоцiльнішою в цій ситуації? A 25-year-old woman was hospitalized at 11 weeks of pregnancy with complaints of pain in the lower abdomen. It is known from the anamnesis that 2 weeks ago she was ill with rubella in a severe form During the bimanual examination, it was found that the cervix is ​​formed, the cervix is ​​closed, the uterus is enlarged to 11 weeks of pregnancy, and the appendages are unremarkable.

Пролонгування вагiтностi Prolongation of pregnancy

Малий кесарiв розтин Small caesarean section

Введення утеротонікiв Introduction of uterotonics

Введення спазмолiтикiв Introduction of antispasmodics

Переривання вагітності Termination of pregnancy

1328 / 1500
Пацієнта віком 43 років шпиталізовано до лікарні за 40 хвилин після виникнення гострого болю в епігастрії, що згодом перемістився у праву здухвинну ділянку. Об'єктивно спостерігається: різке напруження м’язів передньої черевної стінки, позитивний симптом Щоткіна-Блюмберга, Мюссі-Георгієвського, Чугаєва, Бернштейна. Печінкова тупість відсутня. Який найімовірніший діагноз? A 43-year-old patient was admitted to the hospital 40 minutes after the onset of acute pain in the epigastrium, which later moved to the right iliac region. Objectively observed: sharp tension in the muscles of the anterior abdominal wall, a positive symptom of Shtotkin-Blumberg, Chugaev, Bernstein. There is no hepatic dullness. What is the most likely diagnosis?

Гострий холецистит Acute cholecystitis

Перфоративна виразка шлунка Perforative gastric ulcer

Гострий панкреатит Acute pancreatitis

Гострий апендицит Acute appendicitis

Ниркова колька Renal colic

1329 / 1500
У хворого 2 років із рецидивуючим бронхо-легеневим захворюванням, гіпотрофією, установлений діагноз: муковісцидоз. Яке дослідження повинно підтвердити клінічний діагноз? A 2-year-old patient with recurrent broncho-pulmonary disease, hypotrophy, diagnosed with cystic fibrosis. What research should confirm the clinical diagnosis?

Рівень лужної фосфатози в сироватці Serum alkaline phosphatase level

Рівень кальцію в крові Calcium level in blood

Наявність білку в сечі Presence of protein in urine

Вміст 2-фракції гамма-глобулінів Content 2-fraction of gamma-globulins

Кількість хлоридів у поті Number of chlorides in sweat

1330 / 1500
Дитина 5-ти років хворіє 2 тижні. Спочатку з'явилися напади кашлю, потім - репризи. Під час кашлю обличчя хворого червоніє, вени шиї набухають. Напади кашлю закінчуються блюванням. На рентгенограмі: підсилення бронхіального малюнка. Аналіз крові: лейкоцити - 16·10^9/л, лімф.- 72%, швидкість осідання еритроцитів - 4 мм/год. Який найбільш імовірний діагноз? A 5-year-old child has been ill for 2 weeks. At first, coughing attacks appeared, then repeated attacks. During coughing, the patient's face turns red, neck veins swell. Coughing attacks end with vomiting. On X-ray: increase in bronchial pattern. Blood count - 16·10^9/l, erythrocyte sedimentation rate - 4 mm/h. What is the most likely diagnosis?

Стороннє тіло Foreign body

Пневмонія Pneumonia

Обструктивний бронхіт Obstructive bronchitis

Кашлюк Whooping cough

Аденовірусна інфекція Adenovirus infection

1331 / 1500
Пацієнт скаржиться на головний біль, утруднення носового дихання, гнійні виділення з носу протягом 3 місяців. 4 місяці тому йому лікували верхній премоляр справа. Рентгенологічно визначається затінення правої верхньощелеп-ної пазухи. Під час діагностичної пункції гайморової пазухи отримано велику кількість густого крихтоподібного гною з неприємним запахом. Який діагноз імовірний? The patient complains of a headache, difficulty in nasal breathing, and purulent discharge from the nose for 3 months. 4 months ago, he was treated with the right upper premolar. Radiographically, the shadowing of the right maxilla is determined During the diagnostic puncture of the maxillary sinus, a large amount of thick pus with an unpleasant odor was obtained. What is the probable diagnosis?

Пухлина верхньощелепної пазухи Tumor of maxillary sinus

Хронічний гнійний гайморит Chronic purulent sinusitis

Хронічний гнійний одонтогенний гайморит Chronic purulent odontogenic sinusitis

Гострий гнійний гайморит Acute purulent sinusitis

Хронічний атрофічний гайморит Chronic atrophic sinusitis

1332 / 1500
Чоловік 22 років лежить у ліжку, високо підвівши голову, не відчуває незручності в такій позі. У бесіду вступає неохоче, реагує на шепітну мову, відповіді однослівні. Обличчя байдуже, гіпомімічне, чоло наморщене, губи витягнуті трубочкою. Рухається дуже мало, часто надовго застигає в малозручній позі. Такий стан розвинувся поступово, протягом тижня без видимих причин. Діагностуйте психічний стан хворого: A 22-year-old man lies in bed with his head raised high, he does not feel discomfort in this position. He enters the conversation reluctantly, responds to whispered speech, answers are monotonous. The face is indifferent , hypomimic, wrinkled forehead. Moves very little, often stays in an uncomfortable position for a long time. Diagnose the patient's mental state:

Психогенний ступор Psychogenic stupor

Кататонічний субступор Catatonic substupor

Депресивний субступор Depressive substupor

Екзогенний ступор Exogenous stupor

Апатичний субступор Apathetic substupor

1333 / 1500
Роділля, І вагітність, 38 тижнів, скаржиться на нерегулярний переймоподібний біль внизу живота та в попереку протягом 2 діб, вночі спала погано. При зовнішньому акушерському дослідженні - тонус матки підвищений, поздовжнє положення плода І позиція, передній вид, голівка плода рухома над входом в малий таз. Піхвове дослідження - шийка матки до 2,5 см, відхилена дозаду, розм’якшена нерівномірно, зовнішнє вічко відкрите до 0,5 см, внутрішнє вічко закрите. Ваш діагноз: Mother in labor, 1st pregnancy, 38 weeks, complains of irregular spasm-like pain in the lower abdomen and lower back for 2 days, slept poorly at night. During external obstetric examination - uterine tone elevated, longitudinal position of the fetus I position, front view, the fetal head is mobile above the entrance to the pelvis. Vaginal examination - the cervix is ​​up to 2.5 cm, deviated backward, unevenly softened, the external eye is open to 0.5 cm, the internal eye close. Your diagnosis:

Первинна слабкість пологової діяльності Primary weakness of labor activity

Дискоординована пологова діяльність Discoordinated birth activity

Передвісники пологів Forerunners of childbirth

Патологічний прелімінарний період Pathological preliminary period

Вторинна слабкість пологової діяльності Secondary weakness of labor activity

1334 / 1500
Жінка 65-ти років, вранці після сну відмітила затерплість в правих кінцівках, запаморочення, гикавку, зміну голосу, похлинання при вживанні рідини, затерплість та біль у лівій половині обличчя, хиткість ходи. Останні показники артеріального тиску - 160/100 мм рт.ст. У анамнезі гіпертонічна хвороба (систематично не лікується). При фізикальному обстеженні горизонтальний середньоамплітудний ністагм, більше виражений вліво, синдром Горнера ліворуч, гіпестезія лівої половини обличчя за цибулинним типом, правобічна гемігіпестезія, атаксія в позі Ромберга. Мова змінена - нечітка артикуляція, проте зміст і розуміння мови не порушені. Пацієнтці важко висунути язик з рота. Парезів кінцівок немає. Який метод буде найбільш доречним у підтверджені діагнозу цієї пацієнтки? A 65-year-old woman, in the morning after sleep noted numbness in the right limbs, dizziness, hiccups, change in voice, swallowing when drinking liquid, numbness and pain in the left half face, unsteadiness of gait. Recent blood pressure readings - 160/100 mm Hg. History of hypertension (not systematically treated). On physical examination, horizontal nystagmus, more pronounced on the left side, hypoesthesia of the left half of the face. , right-sided hemiesthesia, ataxia in Romberg's position. The language is unclear, but the meaning and understanding of speech is not disturbed. There are no paresis of the limbs. Which method would be most appropriate in this patient's diagnosis?

Реоенцефалографія Rheoencephalography

Комп'ютерна томографія (КТ) головного мозку Computed tomography (CT) of the brain

Ангіографія Angiography

Електроенцефалографія Electroencephalography

Магнітно-резонансна томографія (МРТ) головного мозку Magnetic resonance imaging (MRI) of the brain

1335 / 1500
Хворий на гемофілію А 18-ти років звернувся із скаргами на гострий біль у лівому колінному суглобі та збільшення його в об'ємі. Суглоб різко болючий під час рухів та пальпації, шкіра над ним гаряча на дотик. Яке лікування слід застосувати? An 18-year-old hemophiliac A patient complained of sharp pain in the left knee joint and an increase in its volume. The joint is sharply painful during movements and palpation, the skin over it is hot to the touch. What treatment should be applied?

В/в введення концентрату чинника VIII IV administration of factor VIII concentrate

Провести лікувальну пункцію суглоба Perform medical puncture of the joint

В/в введення свіжозамороженої плазми IV administration of fresh frozen plasma

В/в введення концентрату чинника IX IV administration of factor IX concentrate

Накласти холод та іммобілізувати суглоб Apply cold and immobilize the joint

1336 / 1500
Чоловіка 42-х років, шахтаря, через 12 годин звільнено з-під завалу. Об'єктивно: обидві гомілки і ступні блідого кольору. Пульсації периферійних судин немає. Чутливість та пасивні рухи в суглобах відсутні. Доставлений із джгутами на обох кінцівках. Які невідкладні заходи першої лікарської допомоги необхідно провести для запобігання міоглобінурії та гострої ниркової недостатності? A 42-year-old man, a miner, was freed from the rubble after 12 hours. Objectively: both lower legs and feet are pale in color. There is no pulsation of peripheral vessels. Tenderness and passive movements in the joints are absent. Delivered with tourniquets on both limbs. What emergency medical measures should be taken to prevent myoglobinuria and acute renal failure?

Джгут не знімати, дезінтоксикаційна терапія Do not remove tourniquet, detoxification therapy

Джгут не знімати, гіпербарична оксигенація Do not remove tourniquet, hyperbaric oxygenation

Зняти джгут, гіпербарична оксигенація Remove tourniquet, hyperbaric oxygenation

Джгут не знімати, ампутація кінцівок вище джгута Do not remove the tourniquet, amputation of limbs above the tourniquet

Зняти джгут, дезінтоксикаційна терапія Remove tourniquet, detoxification therapy

1337 / 1500
У жінки віком 35 років за 30 хвилин після внутрішньо-м'язової ін'єкції ампіциліну виникли різка слабкість, свербіж обличчя та рук, нудота, кашель, задишка, біль у грудній клітці. Об'єктивно спостерігається: ціаноз, набряк повік, обличчя, шиї, шкіра волога з червоним висипом. Пульс - 120/хв, АТ - 70/20 мм рт. ст., тони серця глухі, дихання часте, поверхневе з великою кількістю різнокаліберних вологих хрипів. З введення якого препарату необхідно негайно розпочати терапію? A 35-year-old woman developed sudden weakness, itching of the face and hands, nausea, cough, shortness of breath, 30 minutes after an intramuscular injection of ampicillin pain in the chest. Objectively observed: cyanosis, swelling of the face, neck, skin with a red rash. Pulse - 70/20 mm Hg, heart sounds are shallow with a large number of wet rales. With the introduction of which drug should the therapy be started immediately?

Преднізолону Prednisolone

Допаміну Dopamine

Еуфіліну Euphilina

Адреналіну Adrenaline

Астмопенту Astmopentu

1338 / 1500
Чоловік 30 років скаржиться на біль у гомілках обох ніг, який посилюється під час ходьби. Хворий змушений зупинятися через 150-200 м. Переміжна кульгавість з'явилась рік тому, відстань між зупинками скоротилася. Обидві ступні бліді, прохолодні. Пульс на тильних артеріях ступнів відсутній. Шкіра суха, на підошвах гіперкератоз, нігті потовщені, легко ламаються, матового кольору. На гомілках - ділянки облисіння. Яка стадія облітеруючого ендартеріїту у цього пацієнта? A 30-year-old man complains of pain in the lower legs of both legs, which worsens when walking. The patient is forced to stop after 150-200 m. Intermittent lameness appeared a year ago , the distance between the stops has decreased. The pulse on the back of the feet is absent. The skin is thickened, the nails break easily. What is the stage of alopecia in this patient?

І стадія - стадія функціональної компенсації The first stage - the stage of functional compensation

ІV стадія - стадія деструктивних змін IV stage - the stage of destructive changes

- -

ІІ стадія - стадія субкомпенсації II stage - subcompensation stage

ІІІ стадія - стадія декомпенсації III stage - decompensation stage

1339 / 1500
Пацієнт віком 45 років скаржиться на висип в ділянці шкіри тулуба, верхніх та нижніх кінцівок. Появу висипу пов'язує з нещодавно перенесеною стресовою ситуацією. Об'єктивно спостерігається: на шкірі запальні папули, що мають тенденцію до розповсюдження та вкриті рихлими сріблясто-жовтими лусочками. Під час пошкрябування елеметів висипу спостерігається симптом 'стеаринової плями'. Генеалогічній анамнез не обтяжений. Який найімовірніший діагноз? A 45-year-old patient complains of a rash on the skin of the trunk, upper and lower limbs. He associates the appearance of the rash with a recently experienced stressful situation. Objectively observed: inflammatory papules with a tendency to spread and covered with loose silver-yellow scales. When scraping the rash, the symptom of a 'stearin spot' is observed. What is the most likely diagnosis?

Екзема Eczema

Червоний плоский лишай Lichen planus

Псоріаз Psoriasis

Алергічний дерматит Allergic dermatitis

Дерматофітія Dermatophytia

1340 / 1500
Чоловіка 40 років доставлено до лікарні після закритої травми грудної клітки. Під час огляду права половина грудної клітки відстає під час дихання. Пальпаторно спостерігається різка болючість у проєкції V, VI, VII ребер на передній пахвовій лінії, підшкірна емфізема правих відділів тулуба. Перкуторно над правими відділами грудної клітки визначається тимпаніт. Аускультативно справа дихання не визначається, зліва - везикулярне. Якої хірургічної допомоги потребує хворий? A 40-year-old man was brought to the hospital after a closed injury to the chest. During the examination, the right half of the chest lags behind during breathing. On palpation, there is a sharp tenderness in the projection V, VI , VII ribs on the front axillary line, subcutaneous emphysema of the right parts of the trunk. Percussion is determined over the right parts of the chest. Breathing is not determined on the right, on the left - vesicular. What surgical help does the patient need?

Негайна торакотомія Immediate thoracotomy

Дренування підшкірної емфіземи Drainage of subcutaneous emphysema

Туге бинтування грудної клітки Tight bandaging of the chest

Дренування правої плевральної порожнини Drainage of the right pleural cavity

Каркасна стабілізація переломів ребер Frame stabilization of rib fractures

1341 / 1500
Аналіз захворюваності жителів села, розташованого поблизу хімічного підприємства, показав, що протягом останніх років помітно зросла кількість захворювань на подагру і рак стравоходу. З яким забруднювачем навколишнього середовища можна пов'язати таку динаміку? An analysis of the morbidity of residents of a village located near a chemical plant showed that in recent years the number of diseases such as gout and esophageal cancer has increased significantly. What environmental pollutant can be associated with 'do such a dynamic?

Стронцієм Strontium

Ртуттю Mercury

Молібденом Molybdenum

Нікелем Nickel

Марганцем Manganese

1342 / 1500
У чоловіка 73 років, який упродовж 5 років хворіє на доброякісну гіперплазію простати, вранці настала гостра затримка сечі. Закатетеризувати сечовий міхур неможливо. Якою буде невідкладна допомога? A 73-year-old man who has been suffering from benign prostatic hyperplasia for 5 years has acute urinary retention in the morning. It is impossible to catheterize the bladder. What will be the emergency care?

Призначення теплових процедур Assignment of thermal procedures

Пункція сечового міхура Bladder puncture

Призначення сечогінних препаратів Prescription of diuretics

Призначення адреноблокаторів Prescription of adrenoblockers

Призначення спазмолітиків та аналгетиків Prescription of antispasmodics and analgesics

1343 / 1500
До гінекологічного відділення звернулася жінка 25 років із скаргами на підвищення температури тіла до 38,6^oС, біль внизу живота, дизуричні розлади. Захворіла 3 доби тому, коли після штучного аборту з'явилися вказані симптоми. Під час гінекологічного дослідження виявлено: шийка матки циліндрична, вічко закрите. Тіло матки дещо збільшене, болюче, м'яке. Додатки матки не пальпуються. Виділення гнійно-кров'яні. За результатами аналізу крові встановлено: лейкоцитоз зі зміщенням формули крові вліво, прискорена ШОЕ. Який діагноз найбільш імовірний? A 25-year-old woman came to the gynecological department with complaints of an increase in body temperature to 38.6°C, pain in the lower abdomen, dysuric disorders. She became ill 3 days ago, when after an artificial abortion, the following symptoms appeared: the cervix was closed, the uterus was slightly enlarged, and the uterine appendages were not palpable : leukocytosis with a shift of the blood formula to the left, accelerated ESR. What is the most likely diagnosis?

Гострий ендоцервіцит Acute endocervicitis

Гострий ендометрит Acute endometritis

Гострий цистит Acute cystitis

Гострий сальпiнгоофорит Acute salpingo-oophoritis

Піосальпінкс Piosalpinx

1344 / 1500
Хворий 51-го року був збитий автомобілем. Скаржиться на біль у лівій половині тазу. При огляді з'ясовано, що конфігурація тазу не порушена, визначається припухлість лівої пахвинної ділянки. При пальпації: різка болючість в цій ділянці. Позитивні симптоми Ларрея, Габая і ''прилиплої п'яти''. Поставте попередній діагноз: The 51-year-old patient was hit by a car. He complains of pain in the left half of the pelvis. During the examination, it was found that the configuration of the pelvis is not disturbed, swelling of the left inguinal areas. On palpation: sharp pain in this area. Positive symptoms of Larray and 'sticky heel

Перелом верхньої гілки лобкової кістки Fracture of the upper branch of the pubic bone

Перелом сідничної кістки Fracture of the hip bone

Перелом верхньої ості клубової кістки Fracture of the upper spine of the iliac bone

Ушкодження лобкового симфізу Damage of pubic symphysis

Перелом дна вертлюгової западини Fracture of the acetabulum floor

1345 / 1500
Чоловік 32-х років госпіталізований на 5-й день хвороби зi скаргами на пiдвищення температури до 38,4^oC, загальну слабкiсть, появу в лiвiй пахвовій дiлянцi щiльного утвору 5х6 см, малоболючого, рухомого. Шкiра над утвором не змiнена. Обличчя хворого гiперемоване, пастозне. Частота серцевих скорочень - 95/хв., артеріальний тиск - 100/80 мм рт.ст. В загальному аналiзi кровi - помiрна лейкопенiя. Працює ветсанітаром. Який дiагноз є найбільш імовірним? A 32-year-old man was hospitalized on the 5th day of illness with complaints of an increase in temperature to 38.4^oC, general weakness, the appearance of a dense mass in the left axillary region 5x6 cm. The skin over the lesion is not changed. The heart rate is 95/min. In the general blood analysis, he has moderate leukopenia What is the most likely diagnosis?

Чума Plague

Туляремiя Tularemia

Доброякiсний лiмфоретикульоз Benign lymphoreticulosis

Ієрсиніоз Yersiniosis

Гiдраденiт Hydradenite

1346 / 1500
В пологовий будинок доставлена вагітна 32-х років. Зі слів родичів дома мала судоми, втратила свідомість. Турбує головний біль, загальна слабкість, свідомість дещо затьмарена. Вагітність ІІ, 34 тижні. АТ- 170/120 мм рт.ст., пульс - 100/хв. Спостерігаються генералізовані набряки, що виникли 3 тижні тому. Серцебиття плода - ритмічне, приглушене, 124/хв. Яка тактика? A 32-year-old pregnant woman was taken to the maternity ward. According to relatives, she had convulsions at home and lost consciousness. She is troubled by a headache, general weakness, consciousness is somewhat clouded. Pregnancy II , 34 weeks. BP- 170/120 mm Hg. Generalized swellings are observed, which occurred 3 weeks ago. The heartbeat is muffled, 124/min.

Інтенсивна терапія, родорозрішення в терміні 36 тижнів Intensive therapy, delivery within 36 weeks

Інтенсивна терапія, родорозрішення в терміні 37-40 тижнів Intensive therapy, delivery within 37-40 weeks

Інтенсивна терапія, родорозрішення впродовж тижня Intensive therapy, delivery within a week

Інтенсивна терапія та негайне родорозрішення Intensive therapy and immediate delivery

Інтенсивна терапія, родорозрішення впродовж доби Intensive therapy, delivery during the day

1347 / 1500
Чоловік віком 64 роки впав на лівий бік на підлогу. Об'єктивно спостерігається: вкорочення лівої кінцівки, зовнішня ротація стегна, симптом прилиплої п'яти. Під час пальпації та постукування по п'яті пацієнт відчуває біль у кульшовому суглобі. Який імовірний діагноз? A 64-year-old man fell on his left side on the floor. Objectively observed: shortening of the left limb, external rotation of the hip, the symptom of stuck heel. During palpation and tapping the heel, the patient feels pain in the hip joint. What is the likely diagnosis?

Перелом верхньої третини стегнової кістки Fracture of the upper third of the femur

Перелом шийки стегна Fracture of femoral neck

Забій кульшового суглоба Hip contusion

Перелом великого вертлюга Fracture of the greater acetabulum

Вивих стегна Dislocation of the hip

1348 / 1500
15-річна дівчина доставлена до лікаря зі скаргами на біль у суглобах з обмеженням рухів, ранкову скутість. Зі слів пацієнтки все розпочалося з болю та припухання обох колінних суглобів. Протягом 2-х років подібні симптоми поступово розвивались у гомілкових, ліктьових, п'ястково-фалангових та проксимальних міжфалангових суглобах. При обстеженні лімфаденопатія, гепатоспленомегалія та висип відсутні. При лабораторному дослідженні ШЗЕ- 42 мм/год, антинуклеарні антитіла (ANA) не виявлено, ревматоїдний фактор не виявлено. Який діагноз є найбільш імовірним у цієї пацієнтки? A 15-year-old girl was taken to the doctor with complaints of pain in the joints with restriction of movement, morning stiffness. According to the patient, it all started with pain and swelling of both knee joints. Over the course of 2 years, similar symptoms gradually developed in the tibial, ulnar, and proximal interphalangeal joints. Lymphadenopathy, hepatosplenomegaly, and rash were absent during the laboratory examination, and antinuclear antibodies (ANA) were not detected , no rheumatoid factor was detected. What is the most likely diagnosis for this patient?

Системний червоний вовчак Systemic lupus erythematosus

Гостра ревматична лихоманка Acute rheumatic fever

Ювенільний ідіопатичний артрит Juvenile idiopathic arthritis

Анкілозивний спондилоартрит Ankylosing spondylitis

Реактивний артрит Reactive arthritis

1349 / 1500
Чоловік страждає на попереково-крижовий радикуліт близько 10 років. Останнє загострення сталося близько 5 місяців тому. Болі по зовнішній поверхні стегна, гомілки, через тил стопи до великого пальця. Під час огляду встановлено: парез гомілкового м'яза, слабкість тильного згинання великого пальця. Рефлекси збережені. Який імовірний діагноз? A man has been suffering from lumbosacral sciatica for about 10 years. The last exacerbation occurred about 5 months ago. Pain on the outer surface of the thigh, lower leg, through the back of the foot to the big toe During the examination, it was found: paresis of the tibial muscle, weakness of the dorsiflexion of the thumb. What is the probable diagnosis?

Пухлина спинного мозку Spinal cord tumor

Мієліт Myelitis

Грижа диска L4-L5 L4-L5 disc herniation

Дисциркуляторна мієлопатія Dyscirculatory myelopathy

Грижа диска L5-S1 L5-S1 disc herniation

1350 / 1500
Пацієнт віком 42 роки скаржиться на періодичний біль в епігастрії, частіше вночі і зранку, натще печію, відрижку кислим. З'явилася загальна слабкість, серцебиття, головокружіння, потім - блювання <<кавовою гущею>>. У загальному аналізі крові визначається: гемоглобін - 92 г/л, лейкоцити - 7,5·10^9/л, ШОЕ - 22 мм/год. Яке ускладнення виникло у пацієнта? A 42-year-old patient complains of periodic pain in the epigastrium, more often at night and in the morning, on an empty stomach, heartburn, sour belching. General weakness, palpitations, dizziness appeared, then - Vomiting with <>. In the general blood analysis, it is determined: hemoglobin - 92 g/l, leukocytes - 7.5·10^9/l, ESR - 22 mm/h. What complication did the patient have?

Перфорація виразки дванадцятипалої кишки Perforation of duodenal ulcer

Малігнізація виразки Ulcer malignancy

Стеноз пілоруса Stenosis of the pylorus

Пенетрація виразки Ulcer penetration

Шлунково-кишкова кровотеча Gastrointestinal bleeding

1351 / 1500
Для боротьби з тютюнопалінням Україна ратифікувала Рамкову конвенцію Всесвітньої організації охорони здоров'я з боротьби з тютюном, у зв'язку із чим було заборонено палити в громадських місцях. До якого виду профілактики належать зазначені заходи? In order to combat tobacco smoking, Ukraine ratified the World Health Organization's Framework Convention on Tobacco Control, in connection with which smoking in public places was prohibited. what type of prevention do these measures belong to?

Вторинна профілактика Secondary prevention

Індивідуальна профілактика Individual prevention

Первинна профілактика Primary prevention

Громадська профілактика Public prevention

Третинна профілактика Tertiary prevention

1352 / 1500
Пацієнтка віком 82 років надійшла у відділення кардіологічної реанімації зі скаргами на гострий біль за грудиною, відчуття нестачі повітря, слабкість. Під час рентгенологічного дослідження органів грудної порожнини виявлено, що поперечний розмір тіні серця збільшений, форма тіні трикутна із закругленими кардіодіафрагмальними кутами. Скорочення серця малої амплітуди, аритмічні. Виявлені рентгенологічні ознаки, найімовірніше, відповідають: An 82-year-old patient was admitted to the cardiac intensive care unit with complaints of sharp pain behind the sternum, a feeling of lack of air, weakness. During an X-ray examination of the chest cavity, it was found that the transverse size of the shadow of the heart is increased, the shape of the shadow is triangular with rounded cardiophrenic angles. Heart contractions of small amplitude, arrhythmic. The detected radiological signs, most likely, correspond to:

Дилатаційній кардіоміопатії Dilated cardiomyopathy

Тріаді Фалло Triadi Fallo

Аортальному стенозу Aortic stenosis

Міокардиту Myocarditis

Ексудативному перикардиту Exudative pericarditis

1353 / 1500
Жінка 38 років скаржиться на тягучий біль внизу живота і в попереку протягом останнього місяця, який посилюється напередодні менструації; передменструальні темні кров'яні виділення. В анамнезі зазначено: 4 штучних аборти, 1 пологи. Під час УЗД виявлено окремі осередки підвищеної ехогенності у міометрії, збільшення передньо-заднього розміру матки, округлі гіпоехогенні включення діаметром 2 мм. Який діагноз найімовірніший? A 38-year-old woman complains of pulling pain in the lower abdomen and lower back during the last month, which worsens on the eve of menstruation; premenstrual dark bloody discharge. The anamnesis indicates: 4 artificial abortions, 1 delivery. During the ultrasound scan, some hyperechoic cells were detected in the myometrium, an increase in the anterior-posterior size of the uterus, and rounded hypoechoic inclusions with a diameter of 2 mm. What is the most likely diagnosis?

Гормонопродукуюча пухлина яєчників Hormone-producing ovarian tumor

Ретроцервікальний ендометріоз Retrocervical endometriosis

Ендометріоз яєчників Ovarian endometriosis

Аденоміоз Adenomyosis

Хоріонепітеліома Chorioepithelioma

1354 / 1500
Чоловік 28 років, скарги на біль у лівій нижній кінцівці протягом 6 місяців, переміжну кульгавість через 50-100 м, періодично нічні болі. Під час огляду кінцівка бліда, холодна. Артеріальна пульсація відсутня на гомільці. Який найімовірніший діагноз? 28-year-old man, complaints of pain in the left lower limb for 6 months, intermittent lameness after 50-100 m, occasional night pains. During examination, the limb is pale, cold. Arterial pulsation is absent in the lower leg. What is the most likely diagnosis?

Облітеруючий атеросклероз Obliterating atherosclerosis

Емболія стегнової артерії Femoral artery embolism

Хвороба Токоясу Tokoyasu disease

Синдром Рейно Raynaud's syndrome

Облітеруючий ендартеріїт Endarteritis obliterans

1355 / 1500
У пацієнта 47-ми років мітральний стеноз після гострої ревматичної лихоманки. Скаржиться на прогресуючу задишку, кашель, виражену загальну слабкість. На даний момент не може виконувати легку побутову роботу. Найбільш доцільна тактика лікування? A 47-year-old patient has mitral stenosis after acute rheumatic fever. He complains of progressive shortness of breath, cough, severe general weakness. At the moment, he cannot perform light household work . The most appropriate treatment tactic?

Призначення діуретиків Prescription of diuretics

Хірургічне лікування Surgical treatment

Антибактеріальна терапія Antibacterial therapy

Призначення антикоагулянтів Prescription of anticoagulants

Препарати наперстянки Dingular drugs

1356 / 1500
Роділля віком 29 років, пологи треті, у терміні 40 тижнів народила хлопчика масою 1900 г, зростом 48 см, із оцінкою за шкалою Апгар 7-8 балів. У ІІІ періоді пологів народилася плацента розмірами 17х16х1,5см, масою 340 г, із множинними петрифікатами, оболонки зеленуватого кольору. Яка патологія найімовірніше спричинила затримку внутрішньоутробного росту плода? A 29-year-old woman in labor, her third delivery, gave birth to a boy weighing 1900 g, height 48 cm, with an Apgar score of 7-8 points at 40 weeks. In the third period of labor, a placenta was born measuring 17x16x1.5 cm, weighing 340 g, with multiple petrifications, a greenish membrane. What pathology most likely caused intrauterine growth retardation?

Передчасні пологи Premature birth

Внутрішньоутробне інфікування плода Intrauterine fetal infection

Передчасне відшарування нормально розташованої плаценти Premature detachment of a normally located placenta

Гіпотрофія плода І стуненя Fetal hypotrophy and moaning

Плацентарна недостатність Placental insufficiency

1357 / 1500
У 29-річної жінки протягом останніх місяців з'явилися скарги на біль у правій клубовій ділянці, проноси з домішками слизу і гною, біль у стегнових суглобах, періодичне підвищення температури тіла. Живіт під час пальпації м'який, чутливий в правій клубовій ділянці. В ході іригографії виявлено: рельєф слизової оболонки нагадує «бруківку», ілеоцекальний перехід звужений. Яке захворювання можна припустити? In recent months, a 29-year-old woman complained of pain in the right iliac region, diarrhea with impurities of mucus and pus, pain in the hip joints, periodic increase body temperature. Abdomen is soft during palpation, sensitive in the right iliac region. During irrigography, the topography of the mucosa resembles a 'cobblestone'. What disease can be assumed?

Глютенова ентеропатія Gluten enteropathy

Хвороба Уїппла Whipple's disease

Псевдомембранозний ентероколіт Pseudomembranous enterocolitis

Туберкульозний ілеотифліт Tuberculous ileotyphlitis

Хвороба Крона Crohn's disease

1358 / 1500
Матір шестирічного хлопчика скаржиться на підвищення температури тіла у дитини до 37,9^oС, вологий малопродуктивний кашель, інспіраторну задишку під час фізичного навантаження, загальну слабкість. З анамнезу відомо, що захворів гостро 3 дні тому, після контакту із хворою на ГРВІ сестрою. Під час обстеження лікар-педіатр діагностувала позагоспітальну двобічну вогнищеву пневмонію, ІІ ступеня тяжкості, гострий перебіг, неускладнену форму, ДН І ступеня. Який антибактеріальний засіб потрібно призначити дитині? The mother of a six-year-old boy complains of an increase in the child's body temperature up to 37.9^oС, a wet low-productive cough, inspiratory shortness of breath during physical exertion, general weakness. From the anamnesis It is known that he became acutely ill 3 days ago, after contact with a sister with SARS. During the examination, the pediatrician diagnosed an out-of-hospital bilateral focal pneumonia, acute, uncomplicated form, DN of the 1st degree.

Цефалоспорини ІІІ покоління Third generation cephalosporins

Аміноглікозиди Aminoglycosides

Макроліди Macrolides

Цефалоспорини ІІ покоління II generation cephalosporins

Амоксицилін Amoxicillin

1359 / 1500
Дівчина 18-ти років звернулася ввечері до лікаря зі скаргами на сильний біль ''знизу справа'', нудоту та одиничний епізод блювання. Зранку відвідувала фестиваль вуличної їжі. Біль розпочався 4 години тому, почався з епігастрію і поступово перейшов у праву здухвинну ділянку. При фізикальному обстеженні артеріальний тиск - 110/70 мм рт.ст., частота серцевих скорочень - 79/хв., частота дихання - 16/хв., температура - 37,4^oC. При пальпації лівого нижнього квадранту черевної стінки, біль виникає у правому. Гінекологічний та ректальний огляд без патологічних змін. Дані лабораторних досліджень: лейкоцити - 12·10^9/л, еритроцити - 4,4·10^12/л, гемоглобін - 130 г/л, гематокрит - 39%, тромбоцити - 225·10^9/л, beta-ХГЛ - 2 мОд/мл (норма - менше 5 мОд/л у невагітних жінок). Який діагноз є найбільш імовірним? An 18-year-old girl consulted a doctor in the evening with complaints of severe pain in the lower right, nausea and a single episode of vomiting. In the morning, she attended a street food festival. The pain started 4 hours ago, started from the epigastrium and gradually moved to the right thoracic region. During the physical examination, blood pressure - 110/70 mm Hg, heart rate - 79/min, respiratory rate - 16/min., temperature - 37.4°C. On palpation of the left lower quadrant, pain occurs in the right side. Laboratory data: leukocytes - 12·10^9/l, erythrocytes - 4.4·10^. 12/l, hemoglobin - 130 g/l, hematocrit - 39%, platelets - 225·10^9/l, beta-hCG - 2 mU/ml (norm - less than 5 mU/l in non-pregnant women). most likely?

Емболія мезентеріальних судин Mesenteric vessel embolism

Кишкова непрохідність Intestinal obstruction

Гострий апендицит Acute appendicitis

Трубна вагітність Tap pregnancy

Харчова токсикоінфекція Food poisoning

1360 / 1500
Під час дослідження проби молока виявлено: колір - білуватий, запах - без особливостей, смак - характерний для молока, густина - 1,038, кислотність - 35^o Тернера, жирність - 3,2%. Визначте ступінь якості молока. During the examination of the milk sample, the following was found: color - whitish, odor - without any features, taste - characteristic of milk, density - 1.038, acidity - 35^o Turner, fat content - 3.2%. Determine the degree of milk quality.

Молоко умовно придатне Milk is conditionally suitable

Молоко фальсифіковане Milk is falsified

Молоко недоброякісне Milk is of poor quality

Молоко зниженої якості Low quality milk

Молоко доброякісне Good quality milk

1361 / 1500
Жінка 62-х років доставлена у приймальне відділення зі скаргами на сильний пекучий біль за грудиною, задуху. В анамнезі: 10 років гіпертонічна хвороба. Об'єктивно: стан середньої важкості, шкіра бліда, ціаноз губ, над легенями везикулярне дихання. Тони серця приглушені, ритмічні. Акцент II тону над аортою. Артеріальний тиск - 210/120 мм рт.ст. Частота серцевих скорочень = пульс - 76/хв. На ЕКГ: підвищення сегменту ST в I, aVL, V5-V6 відведеннях. Найбільш імовірний діагноз: A 62-year-old woman was brought to the reception department with complaints of severe burning pain behind the sternum, suffocation. History: 10 years of hypertension. Objectively: condition of moderate severity, cyanosis of the lips, muffled heart sounds over the aorta, 210/120 mm Hg, heart rate: 76/min. ST segment elevation in I, aVL, V5-V6 leads. The most likely diagnosis:

Гіпертонічний криз, ускладнений нестабільною стенокардією Hypertensive crisis complicated by unstable angina

Гіпертонічний криз, ускладнений гострою лівошлуночковою недостатністю Hypertensive crisis complicated by acute left ventricular failure

Неускладнений гіпертонічний криз Uncomplicated hypertensive crisis

ТЕЛА BODIES

Гіпертонічний криз, ускладнений гострим інфарктом міокарда Hypertensive crisis complicated by acute myocardial infarction

1362 / 1500
Доношений новонароджений вагою 3900 г. За даними УЗД: вроджена вада серця, коарктація аорти без гіпоплазії дуги аорти, м'язовий дефект міжшлуночкової перегородки діаметром 2 мм. Яка тактика лікування? A full-term newborn weighing 3900 g. According to ultrasound: congenital heart disease, coarctation of the aorta without hypoplasia of the aortic arch, muscular defect of the interventricular septum with a diameter of 2 mm. What are the tactics treatment?

Резекція коарктації аорти Resection of coarctation of the aorta

Ушивання дефекту міжшлуночкової перегородки Suture of a ventricular septal defect

Пластика дефекту міжшлуночкової перегородки Ventricular septal defect plastic

Пластика дуги аорти Aortic arch plastic

Пластика дефекту міжшлуночкової перегородки та резекція коарктації аорти Plastic of the defect of the interventricular septum and resection of coarctation of the aorta

1363 / 1500
Жінка 60-ти років з нормальною масою тіла отримує з добовим раціоном 50 г білка, 70 г жиру, 300 г вуглеводів. У раціоні переважають зернобобові, достатньо овочів, але обмежена кількість молока і молочних продуктів. Весною щоденно вживає щавель і ревінь. Ризиком якого захворювання для жінки у першу чергу може бути такий ра-ціон? A 60-year-old woman with a normal body weight receives 50 g of protein, 70 g of fat, 300 g of carbohydrates in her daily diet. The diet is dominated by legumes, enough vegetables, but a limited amount of milk and dairy products. In the spring, she consumes sorrel and rhubarb. What kind of disease can a woman face in the first place?

Сечокам'яної хвороби Urolithiasis

Атеросклерозу Atherosclerosis

Ожиріння Obesity

Гіпертонічної хвороби Hypertensive disease

Остеопорозу Osteoporosis

1364 / 1500
Чоловік 33 років госпіталізований до інфекційної лікарні на 7-й день захворювання зі скаргами на різку слабкість, високу температуру, біль у м'язах ніг і попереку, жовтяницю, темний колір сечі, головний біль. Захворів гостро з ознобу, підвищення температури тіла до 40^oC, головний біль, біль у литкових м'язах та попереку. На 4-й день хвороби з'явилася жовтяниця, на 5-й - носова кровотеча, крововилив у склери. Тривалість гарячки 6 днів. Діурез - 200 мл. Який найімовірніший діагноз? A 33-year-old man was hospitalized in an infectious disease hospital on the 7th day of the disease with complaints of sharp weakness, high temperature, pain in the muscles of the legs and lower back, jaundice, dark color of urine, headache. He fell ill with chills, body temperature up to 40°C, headache, pain in the calf muscles and lower back. On the 4th day of the illness, jaundice appeared, on the 5th - nosebleeds , hemorrhage in the sclera. Duration of fever - 200 ml. What is the most likely diagnosis?

Вірусний гепатит А Viral hepatitis A

Лептоспіроз Leptospirosis

Ієрсиніоз Yersiniosis

Сепсис Sepsis

Черевний тиф Typhoid

1365 / 1500
До дерматолога звернулася жінка 38 років зі скаргами на сухість та лущення шкіри. Під час огляду на розгинальних поверхнях ліктьових і колінних суглобів виявлено папульозний висип і дрібне лущення, у ділянці волосяних фолікулів є вузлики воскоподібного кольору, що виступають над поверхнею шкіри. Ці клінічні прояви, найімовірніше, пов'язані з недостатнім надходженням з їжею в організм: A 38-year-old woman consulted a dermatologist with complaints of dryness and peeling of the skin. During the examination, a papular rash and small peeling were found on the extensor surfaces of the elbow and knee joints, in the area of hair follicles are nodules of a waxy color protruding above the surface of the skin. These clinical manifestations are most likely associated with insufficient intake of food into the body:

Рибофлавіну Riboflavin

Ретинолу Retinol

Тіаміну Thiamine

Піридоксину Pyridoxine

Аскорбінової кислоти Ascorbic acid

1366 / 1500
Під час огляду триденної дитини виявлено криваве блювання, петехіально-плямистий висип, часті рідкі випорожнення чорного кольору. У крові виявлено: еритроцити - 4,2·10^12/л, Нb - 128 г/л, тромбоцити - 200·10^9/л, протромбіновий індекс - 40%. Результат тесту Апта: після додавання лугу колір розчину не змінився. Назвіть провідний механізм геморагічного синдрому у дитини: During the examination of a three-day-old child, bloody vomiting, petechial-spotted rash, frequent liquid black stools were found. In the blood, erythrocytes - 4.2·10^12 /l, Hb - 128 g/l, platelets - 200·10^9/l, prothrombin index - 40%. The result of the Apta test: the color of the solution did not change. Name the leading mechanism of the hemorrhagic syndrome in the child:

Низький рівень вітамін К-залежних факторів Low level of vitamin K-dependent factors

Низький рівень VIII і IX факторів згортання крові Low level of clotting factors VIII and IX

Наявність материнських антитіл до тромбоцитів дитини Presence of maternal antibodies to the child's platelets

Коагулопатія споживання Consumption coagulopathy

Інфекційний вплив на кістковомозкове кровотворення Infectious effect on bone marrow hematopoiesis

1367 / 1500
У юнака 18 років раптово з'явився сильний головний біль, блювання, підвищення температури тіла до 39^oС. Об'єктивно встановлено: положення в ліжку із закинутою головою та підведеними до живота ногами. Загальна гіперестезія. Позитивні симптоми Брудзинського та Керніга. Вогнищева неврологічна симптоматика відсутня. У лікворі - нейтрофільний плеоцитоз. Який найімовірніший діагноз? An 18-year-old boy suddenly developed a severe headache, vomiting, and an increase in body temperature up to 39°C. Objectively established: position in bed with head thrown back and legs drawn up. Brudzinsky's and Kernig's symptoms are absent. In the cerebrospinal fluid, what is the most likely diagnosis?

Менінгіт Meningitis

Субдуральна гематома Subdural hematoma

Абсцес головного мозку Brain abscess

Субарахноїдальний крововилив Subarachnoid hemorrhage

Енцефаліт Encephalitis

1368 / 1500
Для пацієнта віком 64 роки з пухлиною сигмоподібної кишки та хронічним тромбофлебітом глибоких вен правої нижньої кінцівки планується оперативне втручання. Виберіть оптимальний препарат для профілактики тромбозу глибоких вен у цього пацієнта. A 64-year-old patient with a tumor of the sigmoid colon and chronic thrombophlebitis of the deep veins of the right lower limb is scheduled for surgery. Choose the optimal drug for the prevention of deep vein thrombosis in this patient.

Фенілін Feniline

Ацетилсаліцилова кислота Acetylsalicylic acid

Звичайний гепарин Plain heparin

Низькомолекулярний гепарин Low molecular weight heparin

Реополіглюкін Rheopoliglyukin

1369 / 1500
Хвора 30-ти років скаржиться на свербіж шкіри, переважно у вечірній та нічний час. Хворіє 2 тижні. На шкірі міжпальцевих складок, молочних залоз, живота, сідниць та стегон множинні дрібнопапульозні та папуловезикульозні висипки, розташовані попарно, екскоріації. Висипки відсутні на обличчі та шиї. Подібну висипку має чоловік хворої. Найбільш імовірний діагноз: A 30-year-old patient complains of skin itching, mainly in the evening and at night. She has been ill for 2 weeks. On the skin of the interdigital folds, mammary glands, abdomen, buttocks and multiple small papular and papulovesicular rashes, located in pairs. There are no rashes on the face and neck. The patient's husband has the most likely diagnosis:

Короста Scabies

Герпес Herpes

Екзема Eczema

Нейродерміт Neurodermatitis

Епідермофітія Epidermophyta

1370 / 1500
Чоловік 58-ми років звернувся до лікаря зі скаргами на значне підсилення загальної слабкості за останні кілька тижнів, ціаноз, болі у животі, нудоту, періодичне блювання, запах ацетону з роту, гіперпігментацію шкірних складок. З анамнезу відомо, що пацієнт хворіє на туберкульоз. При фізикальному обстеженні артеріальний тиск - 80/40 мм рт.ст., пульс - 124/хв. При лабораторному дослідженні крові: натрій - 125 ммоль/л, хлориди - 74 ммоль/л, калій - 5,7 ммоль/л, глюкоза - 3,5 ммоль/л, сечовина - 14 ммоль/л. У загальному аналізі сечі: білок - 0,66 г/л, лейкоцити - 10-12 в п/з, еритроцити - 5-6 в п/з, циліндри - поодинокі. Який наступний крок у веденні пацієнта буде найбільш доречним? A 58-year-old man turned to the doctor with complaints of a significant increase in general weakness over the past few weeks, cyanosis, abdominal pain, nausea, periodic vomiting, the smell of acetone from the mouth, hyperpigmentation of the skin folds. From the anamnesis, it is known that the patient has tuberculosis. During the physical examination, the blood pressure is 80/40 mm Hg. During the laboratory examination, sodium is 125 mmol/l. chlorides - 74 mmol/l, potassium - 5.7 mmol/l, glucose - 3.5 mmol/l, urea - 14 mmol/l. In the general analysis of urine: protein - 0.66 g/l, leukocytes - 10 12 in p/z, erythrocytes - 5-6 in p/z, cylinders - single. What next step in the management of the patient will be most appropriate?

Визначення концентрації діастази в сечі Determination of diastase concentration in urine

Визначення концентрації лужної фосфатази в крові Determining the concentration of alkaline phosphatase in the blood

Визначення концентрації кортизолу в крові Determination of cortisol concentration in blood

Визначення концентрації кетонових тіл в сечі Determining the concentration of ketone bodies in urine

Визначення концентрації креатиніну в крові Determination of creatinine concentration in blood

1371 / 1500
Лікар при обстеженні хворого встановив ураження очей (гемералопія, синдром Біто), шкіри та її придатків, слизових оболонок та травного тракту. Попередній діагноз: хвороба Прасада. Чим зумовлений розвиток вказаної патології? During the patient's examination, the doctor found damage to the eyes (hemeralopia, Bito's syndrome), skin and its appendages, mucous membranes, and the digestive tract. Preliminary diagnosis: Prasad's disease. What is the cause the development of the specified pathology?

Дефіцитом цинку Zinc deficiency

Дефіцитом заліза Iron deficiency

Дефіцитом міді Copper deficiency

Дефіцитом марганцю Manganese deficiency

Дефіцитом ванадію Vanadium deficiency

1372 / 1500
У хворого 25-ти років на 10-й день захворювання, яке супроводжувалося підвищенням температури тіла, везикульозним висипом, переважно на шкірі тулуба та волосистої частини голови, з'явилися сильний головний біль, блювання, атаксія, загальмованість, дискоординація рухів, тремор кінцівок. Діагностовано енце-фаліт. Ускладненням якої хвороби є цей стан? A 25-year-old patient on the 10th day of the disease, which was accompanied by an increase in body temperature, a vesicular rash, mainly on the skin of the trunk and scalp, with' a severe headache, vomiting, ataxia, dyscoordination of movements appeared. Encephalitis was diagnosed. What is the complication of this condition?

Везикульозний рикетсіоз Vesicular rickettsiosis

Скарлатина Scarlatina

Краснуха Krasnukha

Вітряна віспа Chicken Pox

Кір Measles

1373 / 1500
Пацієнтка віком 47 років скаржиться на вкрай неприємні відчуття в ділянці живота: свербіж, поколювання, печіння. Пояснює це тим, що в неї <<дірка в шлунку>>, просить її обстежити. Під час об'єктивного дослідження жодної соматичної патології не виявлено. Яке порушення сприйняття у пацієнтки? A 47-year-old patient complains of extremely unpleasant sensations in the abdomen: itching, tingling, burning. She explains this by the fact that she has a <> , asks her to be examined. During the objective examination, no somatic pathology was detected. What kind of perception disorder does the patient have?

Ілюзії Illusions

Синестезії Synesthesia

Сенестопатії Senestopathy

Гіперестезії Hyperesthesia

Парестезії Paraesthesia

1374 / 1500
Чоловік 27-ми років звернувся до лікаря зі скаргами на біль та набряк у II та III пальцях лівої стопи, почервоніння та гнійні виділення з очей, різь та біль при виділенні сечі. Зі слів пацієнта має декількох статевих партнерів, зрідка користується презервативами. При фізикальному обстеженні лікарем виявлено двосторонній кон'юнктивіт та дактиліт пальців стопи. Обстеження інших органів і систем патології не виявило. У загальному аналізі крові ШЗЕ- 40 мм/год, біохімічний аналіз без відхилень. На рентгенограмах пальців стопи збільшення об'єму м'яких тканин, звуження суглобової щілини. Який діагноз є найбільш імовірним? A 27-year-old man consulted a doctor with complaints of pain and swelling in the II and III toes of the left foot, redness and purulent discharge from the eyes, tearing and pain in according to the patient, he occasionally uses condoms. During the physical examination, the doctor revealed dactylitis of the feet. In the general analysis of the blood, it was 40 mm/h analysis without deviations. On X-rays of the toes, an increase in the volume of soft tissues, narrowing of the joint space. What is the most likely diagnosis?

Реактивний артрит Reactive arthritis

Ревматоїдний артрит Rheumatoid arthritis

Сифіліс Syphilis

Синдром Шегрена Sjogren's syndrome

Подагра Gout

1375 / 1500
Чоловік 23 років звернувся до поліклініки зі скаргами на головний біль, погіршення зору, задишку, одутлість обличчя, загальну слабкість, зменшення кількості виділеної за добу сечі та забарвлення її типу <<м'ясних зливів>>. Три тижні тому переніс ангіну. Об'єктивно встановлено: помірна блідість, обличчя одутле, повіки набряклі; серцева діяльність ритмічна, І тон на верхівці ослаблений, систолічний шум. Пульс - 66/хв. АТ - 175/105 мм рт. ст. Симптом Пастернацького слабко позитивний з обох боків. У сечі виявлено: білок - 1,48 г/л, еритроцити - 35-40 в п/з, змінені; лейкоцити - 8-10 в п/з. Який діагноз є найімовірнішим? A 23-year-old man came to the clinic with complaints of headache, visual impairment, shortness of breath, puffiness of the face, general weakness, decrease in the amount of urine released per day and the color of its type Three weeks ago, he suffered from angina: moderate pallor, swollen eyelids, rhythmic heart rate, systolic heart rate - 66/min. 175/105 mm Hg, weakly positive on both sides: protein - 1.48 g/l, erythrocytes - 8-10 g/l . What is the most likely diagnosis?

Есенціальна гіпертонія Essential hypertension

Гострий пієлонефрит Acute pyelonephritis

Гострий гломерулонефрит Acute glomerulonephritis

Амилоїдоз Amyloidosis

Інфекційно-алергічний міокардит Infectious-allergic myocarditis

1376 / 1500
Дайте оцінку фізичного розвитку десятирічної дівчинки за шкалою регресії, якщо показники ваги тіла та окружності грудної клітки знаходяться у межах pm 1 сигми. Give an estimate of the physical development of a ten-year-old girl on a regression scale, if the indicators of body weight and chest circumference are within pm 1 sigma.

Дисгармонійний Disharmonious

Високий High

Середній Average

Низький Low

Гармонійний Harmonic

1377 / 1500
Пацієнт віком 27 років скаржиться на болі в симетричних суглобах рук, лихоманку, ранкову скутість. Захворів гостро. Лабораторно встановлено: ревматоїдний фактор, високий титр С-реактивного білка, високий титр антитіл до цитрулінованого віментину. Рентгенологічно виявлено одиничні ерозії міжфалангових суглобів. Яке початкове базисне лікування необхідно призначити у цьому разі? A 27-year-old patient complains of pain in the symmetrical joints of the hands, fever, morning stiffness. He became acutely ill. Laboratory tests revealed: rheumatoid factor, high titer of C-reactive protein, high titer of citrullinated vimentin. Single erosions of the interphalangeal joints were detected. What initial treatment should be prescribed in this case?

Метотрексат Methotrexate

Диклофенак Diclofenac

НПЗП NSAIDs

Целекоксиб Celecoxib

Інгібітори ксантиноксидази Xanthine oxidase inhibitors

1378 / 1500
Молода жінка звернулася до лікаря зі cкаргами на задишку при незначному фізичному навантаженні, інколи набряки на ногах, слабкість. В анамнезі часті запальні захворювання легень. Задишка вперше з'явилась у дитинстві та поступово зростала. При фізикальному обстеженні пульс - 78/хв., артеріальний тиск - 130/80 мм рт.ст., шкіра ціанотична, набряки на ногах. Аус-культативно тони серця ритмічні, грубий систолічний шум уздовж лівого краю груднини, акцент та розщеплення II тону над легеневою артерією. Пальпаторно печінка збільшена. Вибір лікарем якого діагностичного методу буде найбільш доречним? A young woman consulted a doctor with complaints of shortness of breath during minor physical exertion, sometimes swelling of the legs, weakness. She has a history of frequent inflammatory lung diseases. Shortness of breath appeared for the first time in childhood and gradually increased. During physical examination, pulse - 78/min., blood pressure - 130/80 mm Hg, cyanotic skin, rhythmic heart sounds along the left sternal edge, accent and splitting of the II tone over the pulmonary artery. The liver is enlarged. Which diagnostic method will be the most appropriate?

Ехокардіографія Echocardiography

Коронароангіографія Coronary angiography

Біопсія міокарда Myocardial biopsy

Електрокардіограма Electrocardiogram

Катетеризація серця Heart Catheterization

1379 / 1500
Під час обстеження пацієнта виявлено такі симптоми: прогресуюча кволість, зниження працездатності, швидка фізична та психічна втомлюваність, відчуття мерзлякуватості та голоду, втрата маси тіла. Установіть вид аліментарного захворювання: During the examination of the patient, the following symptoms were revealed: progressive frailty, reduced work capacity, rapid physical and mental fatigue, feeling of coldness and hunger, loss of body weight. Establish the type of alimentary disease :

Мінеральна недостатність Mineral deficiency

Недостатність харчових волокон Insufficient dietary fiber

Недостатність поліненасичених жирних кислот Deficiency of polyunsaturated fatty acids

Вітамінна недостатність Vitamin deficiency

Білково-енергетична недостатність Protein-energy deficiency

1380 / 1500
Пацієнтка віком 22 роки збуджена, поведінка неадекватна, свідомість сплутана. В анамнезі - цукровий діабет 1-го типу впродовж 4 років, перебіг лабільний. Отримує інсулінотерапію 54 ОД/добу. Об'єктивно спостерігається: шкіра волога, холодна на дотик, гіперрефлексія, зіниці розширені. Дихання везикулярне. АТ - 140/90 мм рт. ст., пульс - 88/хв. У аналізі крові виявлено: глікемія - 2,3 ммоль/л, аглюкозурія. Який найімовірніший діагноз? A 22-year-old female patient is excited, her behavior is inadequate, her consciousness is confused. She has type 1 diabetes in her history for 4 years, the course is labile. She receives insulin therapy 54 units/ Objectively, the skin is cold to the touch, the pupils are dilated, blood pressure - 88/min /l, aglucosuria. What is the most likely diagnosis?

Гіперосмолярна кома Hyperosmolar coma

Гостре порушення мозкового кровообігу Acute cerebrovascular accident

Кетоацидотична кома Ketoacidotic coma

Лактацидотична кома Lactacidotic coma

Гіпоглікемічна кома Hypoglycemic coma

1381 / 1500
Робітниця прядильного цеху фабрики з виробництва льону зі стажем роботи 15 років скаржиться на слабкість, задишку, кашель, відчуття стислості у грудях, приступи утрудненого дихання. Зазвичай найбільш тяжкі приступи ядухи з'являються при поверненні на роботу у понеділок (<<симптом понеділка>>), тоді як у вихідні дні самопочуття поліпшується. Під час аускультації над легенями прослуховуються розсіяні сухі хрипи з обох сторін. У крові - незначний лейкоцитоз. Назвіть захворювання пацієнтки: A worker in the spinning shop of a flax factory with 15 years of experience complains of weakness, shortness of breath, cough, a feeling of tightness in the chest, attacks of difficulty breathing. Usually the most severe attacks wheezing appears when returning to work (<>), while on the weekend, the patient's health improves.

Антракоз Anthracosis

Бісиноз Bisinosis

Силікоз Silicosis

Азбестоз Asbestosis

Сидероз Siderosis

1382 / 1500
Чоловік 62 років скаржиться на помірний біль у лівій ступні в ділянці плюснефалангового суглоба, який посилюється під час рухів. Хворіє протягом 12 років, захворювання почалося з <<гострого нападу>> болю. 2 роки тому під шкірою суглоба з'явився жовтувато-білий вузлик. Під час огляду виявлено суглоб деформований, синюшний. На рентгенограмі суглоба спостерігаються звуження суглобової щілини та чітко обмежені дефекти кісткової тканини в епіфізі (<<симптом пробійника>>). Який діагноз найімовірніший? A 62-year-old man complains of moderate pain in the left foot in the area of ​​the metatarsophalangeal joint, which worsens during movements. He has been ill for 12 years, the disease began with an <> pain. A yellowish-white nodule appeared under the skin of the joint. During the examination, the joint was found to be bluish. Narrowing of the joint space and clearly limited bone tissue defects in the epiphysis were observed (<>). . What is the most likely diagnosis?

Реактивний артрит Reactive arthritis

Синдром Рейтера Reiter syndrome

Подагричний артрит Gouty arthritis

Ревматоїдний артрит Rheumatoid arthritis

Остеоартрит Osteoarthritis

1383 / 1500
У хворого 37-ми років, ВІЛ-позитивного, впродовж 2-х тижнів наростала задишка, температура підвищилась до 37,7^oC. Діагностована пневмоцистна пневмонія. Який з препаратів етіотропної терапії показаний у цьому випадку? A 37-year-old HIV-positive patient developed shortness of breath over the course of 2 weeks, and the temperature rose to 37.7°C. Pneumocystis pneumonia was diagnosed. What from drugs of etiotropic therapy indicated in this case?

Метронідазол Metronidazole

Цефтріаксон Ceftriaxone

Ацикловір Acyclovir

Ко-тримоксазол Co-trimoxazole

Флуконазол Fluconazole

1384 / 1500
До лікарні шпиталізовано п'ятирічну дівчинку після електротравми. Стан дитини вкрай тяжкий, дівчинка без свідомості, самостійне дихання відсутнє. Під час проведення серцево-легеневої реанімації на ЕКГ спостерігаються хвилі різної форми та амплітуди з частотою 320/хв, пульс на периферії та центральних артеріях - відсутній. Яка невідкладна допомога у цьому разі? A five-year-old girl was hospitalized after an electric shock. The child's condition is extremely serious, the girl is unconscious, she is not breathing on her own. During cardiopulmonary resuscitation, the ECG shows waves of different shapes and amplitudes with a frequency of 320/min, a pulse on the periphery and central arteries is absent. What is the emergency help in this case?

Уведення розчину лідокаїну 20 мг в/м Introduction of lidocaine solution 20 mg IV

Прямий масаж серця Direct heart massage

Електрична дефібриляція Electrical defibrillation

Інтубація трахеї Tracheal intubation

Трансфузія кристалоїдів 10 мг/кг в/в Transfusion of crystalloids 10 mg/kg IV

1385 / 1500
Чоловік 58-ми років скаржиться на появу пухлиноподібних утворень на передній поверхні шиї та в пахвинній ділянці, слабкість. Пальпуються м'які, безболісні, рухливі шийні та пахвинні лімфовузли до 2 см в діаметрі. Печінка виступає на 2 см з-під краю реберної дуги, нижній полюс селезінки на рівні пупка. У крові: еритроцити - 3,5·10^12/л, Hb - 88 г/л, лейкоцити - 86·10^9/л, п/я - 1%, с/я - 10%, л - 85%, е - 2%, б - 0%, м - 2%, швидкість осідання еритроцитів - 15 мм/год, тіні Боткіна-Гумпрехта. Який найбільш імовірний діагноз? A 58-year-old man complains of the appearance of tumor-like formations on the front surface of the neck and in the groin area, weakness. Soft, painless, mobile cervical and inguinal lymph nodes are palpated up to 2 cm in diameter. The liver protrudes 2 cm from the edge of the costal arch, the lower pole of the spleen is at the level of the navel. In the blood: erythrocytes - 3.5·10^12/l, Hb - 88 g/l. 10^9/l, p/y - 1%, s/y - 10%, l - 85%, e - 2%, b - 0%, m - 2%, erythrocyte sedimentation rate - 15 mm/h, Botkin-Gumprecht shadows. What is the most likely diagnosis?

Лейкемоїдна реакція лімфоцитарного типу Lymphocyte type leukemoid reaction

Хронічний мієлолейкоз Chronic myelogenous leukemia

Хронічний лімфолейкоз Chronic lymphocytic leukemia

Гострий лейкоз Acute leukemia

Лімфогранулематоз Lymphogranulomatosis

1386 / 1500
У двомісячної дитини спостерігаються судоми, рецидивуючі вірусно-бактеріальні інфекції верхніх дихальних шляхів. Виявлено деформований череп, гіпоплазію тимусу та паращитовидних залоз. Під час лабораторного імунологічного обстеження виявлено: лімфоцитопенія, зниження рівня та проліферативної відповіді Т-лімфоцитів, нормальний рівень імуноглобулінів. Який найімовірніший діагноз? A two-month-old child has convulsions, recurrent viral and bacterial infections of the upper respiratory tract. A deformed skull, hypoplasia of the thymus and parathyroid glands were detected. During the laboratory immunological examination, lymphocytopenia was detected , a decrease in the level and proliferative response of T-lymphocytes, a normal level of immunoglobulins. What is the most likely diagnosis?

Первинний імунодефіцит, синдром Чедіака-Хігасі Primary immunodeficiency, Chediak-Higashi syndrome

- -

Первинний імуннодефіцит, синдром Ді-Джорджі Primary immunodeficiency, Di-Georgi syndrome

Загальний варіабельний імунодефіцит General variable immunodeficiency

Первинний імунодефіцит, хвороба Брутона Primary immunodeficiency, Bruton's disease

1387 / 1500
Дівчина 17 років скаржиться на біль і набряк 2-го пальця правої руки. 3 доби тому зробила манікюр. Біль з'явився другого дня. Об'єктивно спостерігається: навколонігтьовий валик набряклий, гіперемований, нависає над нігтьовою пластинкою, болючий під час пальпації. Який діагноз найімовірніший? A 17-year-old girl complains of pain and swelling of the 2nd finger of her right hand. She had a manicure 3 days ago. The pain appeared on the second day. Objectively observed: the nail plate is swollen, hyperemic, overhanging the nail plate, painful during palpation. What is the most likely diagnosis?

Еризипелоїд Erysipeloid

Пароніхія Paronychia

Піднігтьовий панарицій Subungual panaritium

Підшкірний панарицій Subcutaneous panaritium

Шкірний панарицій Skin panaritium

1388 / 1500
У пацієнта віком 27 років спостерігається постійне блювання, пронос у вигляді <<рисового відвару>>. Напередодні вживав невідомі спиртні напої і гриби. 3 дні тому повернувся з Індії, де перебував як турист. Об'єктивно спостерігається: температура - 35,6^oC, АТ не визначається. Шкіра суха, бліда, складки на ній не розправляються, пульс ниткоподібний, тони серця ослаблені. Який діагноз є найімовірнішим? A 27-year-old patient has constant vomiting, diarrhea in the form of <>. The day before, he consumed unknown alcoholic beverages and mushrooms. He returned from India 3 days ago , where he was as a tourist. Objectively observed: the temperature is 35.6°C, the blood pressure is not determined. The skin is pale, the folds are not straightened, the pulse is weak. What is the most likely diagnosis?

Отруєння сурогатами алкоголю Alcohol surrogate poisoning

Сальмонельоз Salmonellosis

Ротавірусний гастроентерит Rotavirus gastroenteritis

Холера Cholera

Отруєння грибами Mushroom poisoning

1389 / 1500
У 9-місячної дитини ВІЛ-інфіко-ваної жінки визначені гіпотрофія 2-го ступеня, полілімфаденопатія, гепатоспленомегалія, рецидивуючий орофарингеальний кандідоз, гіпохромна анемія (Нb - 80 г/л), нейтропенія; виявлено ДНК ВІЛ у крові. Поставте діагноз: In a 9-month-old child of an HIV-infected woman, hypotrophy of the 2nd degree, polylymphadenopathy, hepatosplenomegaly, recurrent oropharyngeal candidiasis, hypochromic anemia (Hb - 80 g /l), neutropenia; HIV DNA was detected in the blood. Diagnose:

ВІЛ-серопозитивна дитина HIV-seropositive child

- -

СНІД. Опортуністичні інфекції AIDS. Opportunistic infections

Безсимптомне носійство ВІЛ Asymptomatic HIV carrier

Персистуюча генералізована лімфаденопатія Persistent generalized lymphadenopathy

1390 / 1500
У пацієнта віком 58 років протягом 2-3 годин з'явилися множинні плями перед очима, після чого потемнішало перед правим оком. Під час обстеження виявлено: гострота зору становить 0,02 ексцентрично, зіниця помірно розширена, пряма реакція на світло знижена. В ході офтальмоскопії спостерігається: на очному дні множинні крововиливи різної величини і форми (симптом «розчавленого помідора»), диск зорового нерва набряклий, гіперемований. З анамнезу відомо про наявність гіпертонічної хвороби ІІ В ступеня. Який діагноз найімовірніший? A 58-year-old patient developed multiple spots in front of the eyes for 2-3 hours, after which it darkened in front of the right eye. During the examination, it was found that the visual acuity is 0.02 eccentrically, the pupil is moderately dilated, the direct reaction to light is reduced. During ophthalmoscopy, multiple hemorrhages of various sizes and shapes are observed (symptom of 'crushed tomato'), the optic nerve is swollen, hyperemic II B degree of disease. What is the most likely diagnosis?

Емболія центральної артерії сітківки Central retinal artery embolism

Гіпертонічна ангіопатія Hypertensive angiopathy

Діабетична ретинопатія Diabetic retinopathy

Гіпертонічна ангіонейропатія Hypertensive angioneuropathy

Тромбоз центральної вени сітківки Central retinal vein thrombosis

1391 / 1500
Матір восьмимісячного хлопчика скаржиться на здуття живота у дитини, рясні, пінисті випорожнення 3-4 рази на добу з неприємним запахом, зниження маси тіла. Шкірні покриви дитини під час огляду бліді і сухі, визначається збільшення в об'ємі живота, гепатомегалія. У загальному аналізі крові анемія. У біохімічному дослідженні крові - гіпопротеїнемія, гіпоальбумінемія, зниження холестерину, загальних ліпідів і alpha-ліпопротеїдів. Встановіть діагноз за результатами дослідження. The mother of an eight-month-old boy complains of abdominal distension in the child, copious, foamy stools 3-4 times a day with an unpleasant odor, weight loss. The child's skin during the examination is pale and dry, an increase in the volume of the abdomen is determined. In the general blood analysis, hypoproteinemia, hypoalbuminemia, a decrease in total lipids and alpha-lipoproteins are established.

Лактозна недостатність Lactose deficiency

Целіакія Celiac

Виразковий ентерит Ulcerative enteritis

Хвороба Крона Crohn's disease

Кишкова форма муковісцидозу Intestinal form of cystic fibrosis

1392 / 1500
У потерпілого опікова травма 15% поверхні тіла II-III ступеня. На 20-ту добу після травми у хворого спостерігається різке підвищення температури тіла, загальна слабкість, прискорене везикулярне дихання, загострення рис обличчя, артеріальний тиск - 90/50 мм рт.ст., пульс - 112/хв. Яке ускладнення можна припустити? The victim has a burn injury of 15% of the body surface, II-III degree. On the 20th day after the injury, the patient has a sharp increase in body temperature, general weakness, accelerated vesicular breathing, aggravation of facial features, blood pressure - 90/50 mm Hg, pulse - 112/min. What complications can be assumed?

Гнійний бронхіт Purulent bronchitis

Сепсис Sepsis

Анаеробна інфекція Anaerobic infection

Пневмонія Pneumonia

Гостра інтоксикація Acute intoxication

1393 / 1500
Амортизаційні видатки на відновлення томографу закладено у вартість томографії у розмірі 10% річних від його первісної вартості. Через який термін стане можливим оновлення томографу? Amortization expenses for the restoration of the tomograph are included in the cost of the tomography in the amount of 10% per annum of its initial cost. After what period will it be possible to update the tomograph?

Через 5 років After 5 years

Через 20 років After 20 years

Через 10 років After 10 years

Через 15 років After 15 years

Через 7 років After 7 years

1394 / 1500
Пацієнтка віком 15 рокiв скаржиться на вiдсутнiсть менструацiї, періодичний біль у пiхвi. Під час огляду зовнiшнiх статевих органiв виявлено: оволосіння за жiночим типом, великi статевi губи розвинені нормально, прикривають малi, вхiд до пiхви прикритий перегородкою синюшно-багряного кольору, що випинається. Який найімовірніший дiагноз? A 15-year-old patient complains of the absence of menstruation, periodic pain in the vagina. During the examination of the external genitalia, it was found: female-type hair, the labia majora is normally developed, they cover the vagina, the entrance to the vagina is covered by a protruding bluish-purple septum. What is the most likely diagnosis?

Атрезiя дiвочої плiви Atresia of hymen

Ендометрiоз вульви Endometriosis of the vulva

Генiтальний iнфантилiзм Genital infantilism

Аменорея невiдомої етiологiї Amenorrhea of ​​unknown etiology

Дисфункцiя яєчникiв Ovarian dysfunction

1395 / 1500
Під час зовнішнього акушерського обстеження спостерігається: живіт ової-дної форми, у лівій бічній частині матки пальпується спинка плода, у правій - дрібні частини плода, біля дна - щільна частина, що балотує. Серцебиття плода прослуховується ліворуч та вище пупка. Якими є положення, позиція і передлежання плода? During the external obstetric examination, the following is observed: an oval-bottomed abdomen, the back of the fetus is palpated in the left lateral part of the uterus, in the right - small parts of the fetus, near the bottom - dense The fetal heartbeat is heard on the left and above the navel. What is the position, position and presentation of the fetus?

Поздовжнє положення, I позиція, тазове передлежання Longitudinal position, I position, pelvic presentation

Поперечне положення, I позиція, частина, що передлежить, відсутня Transverse position, I position, the preceding part is missing

Поздовжнє положення, II позиція, головне передлежання Longitudinal position, II position, main presentation

Поздовжнє положення, I позиція, головне передлежання Longitudinal position, I position, main presentation

Поздовжнє положення, II позиція, тазове передлежання Longitudinal position, II position, pelvic presentation

1396 / 1500
Через 1,5 години після народження на 32 тижні у дитини спостерігаються роздування крил носа, хрюкаючий видих, тахіпное та втяжіння міжреберних проміжків. Забруднення амніотичної рідини меконієм виявлено не було. Температура новонародженого - 37,4^oC, пульс - 180/хв., частота дихання - 80/хв. Шкіра ціанотична. Аускультативно в легенях послаблене дихання з обох боків. РаО_2 - 32 мм рт.ст., РаСО_2 - 48 мм рт.ст. На рентгенограмі органів грудної клітки дифузний сітчасто-зернистий рисунок, повітряна ''бронхограма''. Який діагноз є найбільш імовірним? In 1.5 hours after birth at 32 weeks, the baby has flaring of the wings of the nose, grunting exhalation, tachypnea, and retractions of the intercostal spaces. Meconium contamination of the amniotic fluid was not detected The temperature of the newborn is 37.4°C, the respiratory rate is 80/min Art. Diffuse mesh-granular pattern on chest X-ray, air 'bronchogram'. What is the most likely diagnosis?

Крововилив у легені Hemorrhage in the lungs

Аспіраційна пневмонія Aspiration pneumonia

Ідіопатичний легеневий фіброз Idiopathic pulmonary fibrosis

Респіраторний дистрес-синдром Respiratory distress syndrome

Транзиторне тахіпное новонароджених Transient tachypnea of ​​newborns

1397 / 1500
Чоловік 45 років скаржиться на появу на шкірі поперека та кінцівок безболісних вузликових елементів, схильних до периферичного росту та злиття. Хворіє 2 роки. Загострення переважно весною. В анамнезі вказано: у батька пацієнта було подібне ураження шкіри. Об'єктивно спостерігається: патологічні елементи представлені краплевидними та монетовидними вузликами і бляшками, укритими білими лусочками. Який попередній діагноз? A 45-year-old man complains of the appearance of painless nodular elements on the skin of the lower back and limbs, prone to peripheral growth and fusion. He has been ill for 2 years. Exacerbations are mainly in the spring. The anamnesis indicates : the patient's father had a similar skin lesion. Objectively, the pathological elements are represented by teardrop-shaped and coin-shaped nodules, covered with white scales. What is the previous diagnosis?

Червоний плескатий лишай Red lichen planus

Себорейна екзема Seborrheic eczema

Атопічний дерматит Atopic dermatitis

Рожевий лишай Pink lichen

Псоріаз Psoriasis

1398 / 1500
Працівник оформлюється на роботу, пройшов медичний профілактичний огляд. Визнаний придатним для роботи в умовах даного виробництва. Який вид медичного профілактичного огляду прой-шов працівник? The employee is registered for work, passed a preventive medical examination. Recognized as suitable for work in the conditions of this production. What type of preventive medical examination did the employee undergo?

Плановий Planned

Цільовий Target

Попередній Previous

Періодичний Periodic

Комплексний Comprehensive

1399 / 1500
Жінка 29 років скаржиться на загальну слабкість, підвищену втомлюваність, зниження ваги, рідкі менструації малого обсягу. Рік тому були пологи, які ускладнилися масивною кровотечею. Об'єктивно спостерігається: пацієнтка астенічної будови, шкіра бліда і суха, волосся на голові рідке, під пахвами відсутнє. Молочні залози і статеві органи в стані гіпотрофії. Який попередній діагноз? A 29-year-old woman complains of general weakness, increased fatigue, weight loss, infrequent menstruation of small volume. A year ago, she gave birth, which was complicated by massive bleeding. Objectively observed : the patient is asthenic, the skin is pale and dry, the hair on the head is thin, the armpits are absent. The mammary glands and genitals are in a state of hypotrophy. What is the previous diagnosis?

Синдром Шихана Sheehan Syndrome

Нервова анорексія Anorexia nervosa

Астеноневротичний синдром Asthenoneurotic syndrome

Гіпопластична анемія Hypoplastic anemia

Пухлина гіпофізу Pituitary tumor

1400 / 1500
Чоловік 51 року доставлений до лікарні зі скаргами на біль у лівому боці, загальну слабість, сухість у роті, спрагу. За годину до госпіталізації, переходячи залізничну колію, упав лівим боком на рельси. Шкірні покриви та видимі слизові облонки бліді, холодний піт. Пульс слабкого наповнення, 100/хв. АТ - 85/55 мм рт. ст. Болючість та незначне напруження м'язів у лівому підребер'ї, слабо позитивний симптом Щоткіна-Блюмберга. Що можна запідозрити у хворого? A 51-year-old man was taken to the hospital with complaints of pain in the left side, general weakness, dry mouth, thirst. An hour before hospitalization, he fell while crossing the railway tracks on the left side.The skin and mucous membranes are pale. The pulse is 100/55 mm Hg. Pain and slight tension in the left hypochondrium Shttkin-Blumberg. What can be suspected in the patient?

Пошкодження шлунка Stomach damage

Пошкодження селезінки Spleen damage

Пошкодження печінки Liver damage

Пошкодження підшлункової залози Pancreas damage

Пошкодження товстої кишки Colon damage

1401 / 1500
У хворого 10 років діагностовано геморагічний васкуліт, шкіряна форма. Одним з основних лікувальних заходів є тривала преднізолонотерапія. З якою метою застосовується ця терапія? A 10-year-old patient was diagnosed with hemorrhagic vasculitis, cutaneous form. One of the main treatment measures is long-term prednisolone therapy. What is the purpose of this therapy?

Підвищення синтезу антитіл Increasing synthesis of antibodies

Зменшення синтезу патологічних імунних комплексів Decreased synthesis of pathological immune complexes

Підвищення синтезу простагландинів Increased synthesis of prostaglandins

Зменшення синтезу простагландинів Decreased synthesis of prostaglandins

Як замістна терапія As replacement therapy

1402 / 1500
Чоловік 53-х років хворіє на цукровий діабет протягом 5-ти років. Звертається до лікаря зі скаргами на болі та судоми в нижніх кінцівках, відчуття оніміння в них, мерзлякуватість. При обстеженні нижніх кінцівок ступні звичайного кольору, теплі на дотик, усі види чутливості значно знижені, гіперкератози, пульсація на артеріях кінцівок збережена. Яке ускладнення найбільш імовірно виникло у пацієнта? A 53-year-old man has been suffering from diabetes for 5 years. He goes to the doctor with complaints of pain and cramps in the lower limbs, a feeling of numbness in them, frostbite. When examining the lower extremities, the feet are normal in color, all types of sensitivity are significantly reduced, pulsation in the arteries of the extremities is preserved. What complication most likely occurred in the patient?

Синдром діабетичної стопи, нейропатична форма Diabetic foot syndrome, neuropathic form

Синдром діабетичної стопи, ішемічна форма Diabetic foot syndrome, ischemic form

Хронічна венозна недостатність Chronic venous insufficiency

Синдром діабетичної стопи, змішана форма Diabetic foot syndrome, mixed form

Облітеруючий ендартеріїт Endarteritis obliterans

1403 / 1500
Чоловік 28 років після піднімання ваги відчув сильний біль у попереку, який іррадіював у праву ногу. Звернувся до лікаря. Після огляду лікар поставив діагноз: гострий дискогенний попереково-крижовий радикуліт. Яке обстеження треба пройти хворому для підтвердження діагнозу? A 28-year-old man, after lifting weights, felt severe pain in the lower back, which radiated to the right leg. He consulted a doctor. After an examination, the doctor made a diagnosis: acute discogenic lumbosacral sciatica. What examination should the patient undergo to confirm the diagnosis?

Рентгенографія нирок Kidney X-ray

Електроміографія м'язів ніг Electromyography of leg muscles

Люмбальна пункція Lumbar puncture

Загальний аналіз сечі General urinalysis

МРТ поперекового відділу хребта MRI of the lumbar spine

1404 / 1500
У жінки віком 70 роки в ході флюорографії ОГК над лівим куполом діафрагми виявлено тінь неоднорідної структури. Рентгеноскопічне дослідження з контрастуванням виявило наявність у грудній порожнині абдомінального сегмента стравоходу. Який діагноз найімовірніший? In a 70-year-old woman, a shadow of a heterogeneous structure was detected during fluoroscopy of the OGK above the left dome of the diaphragm. X-ray examination with contrast revealed the presence of an abdominal segment of the esophagus in the chest cavity. What is the diagnosis the most likely?

Езофагіт Esophagitis

Грижа стравохідного отвору діафрагми Hernia of the esophageal orifice of the diaphragm

Дивертикул стравоходу Esophageal diverticulum

Ахалазія кардії Achalasia cardia

Доброякісна пухлина стравоходу Benign tumor of esophagus

1405 / 1500
Жінка 37-ми років звернулася до лікаря зі скаргами на болі у ділянці попереку, підвищення температури тіла до 38^oC. При фізикальному обстеженні позитивний симптом Пастернацького, більше виражений справа. При лабораторному дослідженні у загальному аналізі крові: гемоглобін - 115 г/л, еритроцити - 3,9·10^12/л, лейкоцити - 15,2·10^9/л, ШОЕ- 28 мм/год. У загальному аналізі сечі: колір - світло жовтий, питома вага - 1018, білок - 0,42 г/л, лейкоцити - 20-30 у полі зору, багато слизу. Який діагноз є найбільш імовірним? A 37-year-old woman consulted a doctor with complaints of pain in the lower back, an increase in body temperature to 38°C. During the physical examination, Pasternacki's symptom was positive, more pronounced On the right, in the general blood analysis: hemoglobin - 115 g/l, erythrocytes - 3.9·10^12/l, ESR - 28 mm/h urine analysis: color - light yellow, specific gravity - 1018, protein - 0.42 g/l, leukocytes - 20-30 in the field of vision, a lot of mucus. What is the most likely diagnosis?

Тубулоінтерстиційний нефрит Tubulointerstitial nephritis

Гострий пієлонефрит Acute pyelonephritis

Гострий аднексит Acute adnexitis

Гострий гломерулонефрит Acute glomerulonephritis

Гострий апендицит Acute appendicitis

1406 / 1500
Дитині 2,5 років. Хворіє другу добу. Захворювання почалось з підвищення температури до 37,8^oC, одноразового блювання, водянистої діареї до 5 разів на добу. Сьогодні блювання було двічі, температура тіла - 38,0^oC, апетит знижений, продовжується водяниста діарея. З чого потрібно почати лікування дитини? The child is 2.5 years old. He has been ill for the second day. The illness began with an increase in temperature to 37.8^oC, one-time vomiting, watery diarrhea up to 5 times a day. Vomiting occurred twice today, the body temperature is 38.0°C, the appetite is reduced, watery diarrhea continues. What is the need to start treating the child?

Призначити ніфуроксазид Prescribe nifuroxazide

Призначити оральну регідратацію Prescribe oral rehydration

Призначити лоперамід Prescribe loperamide

Призначити цефтріаксон Prescribe ceftriaxone

Призначити поліміксин Prescribe polymyxin

1407 / 1500
У жінки віком 25 років з'явився висип на шкірі обличчя, шиї, біль у суглобах. Об'єктивно спостерігається: стан важкий, еритема обличчя у вигляді <<метелика>>, температура тіла - 38,7^oС, суглоби кистей та променево-зап'ясткові суглоби набряклі. Тони серця приглушені, систолічний шум на верхівці серця, ЧСС = 102/хв. У нижніх відділах легень вислуховується шум тертя плеври. У загальному аналізі сечі виявлено білок - 0,36 г/л. Визначення рівня якого показника найінформативніше для верифікації діагнозу? A 25-year-old woman developed a rash on the skin of the face, neck, and joint pain. Objectively observed: the condition is severe, facial erythema in the form of << butterfly>>, body temperature - 38.7°C, wrist and carpal joints are swollen, systolic murmur at the top of the heart, heart rate = 102/min the general analysis of urine revealed protein - 0.36 g/l. Determination of the level of which indicator is the most informative for the verification of the diagnosis?

Рівні циркулюючих імунних комплексів Levels of circulating immune complexes

Aнтитіл до двоспіральної ДНК Antibody to double-helical DNA

С-реактивного протеїна C-reactive protein

Рівні імуноглобулінів A, M, G, E Levels of immunoglobulins A, M, G, E

Загального білка та білкових фракцій крові Total protein and protein fractions of blood

1408 / 1500
Вагітна 26-ти років хворіє на ревматизм із дитинства. Вагітність перша, термін - 10 тижнів. Із настанням вагітності стан здоров'я погіршився: посилилась задишка, з'явились набряки на нижніх кінцівках. Пульс - 86/хв., ціаноз губ, систолічний і діастолічний шуми на верхівці серця. Печінка виступає з-під реберної дуги на 4 см. Діагностовано СН ІІІ ступеня, фракція викиду лівого шлуночка менше 40%. Яка подальша тактика ведення вагітної? A 26-year-old pregnant woman has been suffering from rheumatism since childhood. It is her first pregnancy, the term is 10 weeks. With the onset of pregnancy, her health condition worsened: shortness of breath increased, with' there were swellings on the lower extremities. Pulse - 86/min., systolic and diastolic murmurs at the top of the heart. The liver protrudes from under the costal arch by 4 cm. The left ventricular ejection fraction is less than 40% tactics of managing a pregnant woman?

Медикаментозне переривання вагітності Medical abortion

Пролонгування вагітності з періодичним лікуванням у кардіологічному відділенні Prolongation of pregnancy with periodic treatment in the cardiology department

Переривання вагітності методом ва-куум-аспірації Termination of pregnancy by vacuum aspiration

Пролонгування вагітності, періодична госпіталізація до акушерського стаціонару в критичні терміни Prolongation of pregnancy, periodic hospitalization at an obstetric hospital in critical periods

Переривання вагітності шляхом малого кесаревого розтину Termination of pregnancy by small caesarean section

1409 / 1500
Чоловік 58-ми років, скаржиться на постійний біль в епігастрії, що іррадіює в спину і посилюється після прийому жирної їжі. Пацієнт відмічає слабкість, за останні 2 місяці втратив у вазі до 9 кг. Місяць тому з'явилась жовтяниця, що поступово зростає. Живіт м'який, печінка не збільшена. При лабораторному дослідженні крові гемоглобін - 68 г/л, лейкоцити - 9,0·10^9/л, швидкість зсідання еритроцитів - 36 мм/год. На УЗД: жовчний міхур великих розмірів, конкрементів не містить, холедох розширений, в головці підшлункової залози - округле утворення до 4 см в діаметрі з нечіткими контурами. Який діагноз є найбільш імовірним? A 58-year-old man complains of constant pain in the epigastrium, which radiates to the back and worsens after eating fatty food. The patient notes weakness, over the past 2 months he has lost in weight up to 9 kg. Jaundice appeared a month ago, the abdomen is soft, the liver is not enlarged. In the laboratory examination of blood - 68 g/l, leukocytes - 9.0·10^9/l sedimentation of erythrocytes - 36 mm/h. On ultrasound: the gallbladder is large, does not contain calculi, the choledochus is enlarged, in the head of the pancreas - a rounded formation with a diameter of 4 cm. What is the most likely diagnosis?

Виразкова хвороба дванадцятипалої кишки з пенетрацією в головку підшлункової залози Duodenal ulcer with penetration into the head of the pancreas

Хронічний індуративний панкреатит Chronic indurative pancreatitis

Рак шлунка з метастазами в печінку Gastric cancer with liver metastases

Хронічний холецистопанкреатит Chronic cholecystopancreatitis

Рак головки підшлункової залози Cancer of the head of the pancreas

1410 / 1500
У пацієнта 64-х років протягом останніх 2 місяців з'явилося відчуття утрудненого ковтання твердої їжі з затримкою її посередині стравоходу. Втрата ваги - 4 кг. Батько хворого помер від раку шлунка. Запідозрено рак стравоходу. За допомогою якого дослідження найбільш імовірно можна верифікувати цю патологію? A 64-year-old patient had a feeling of difficulty swallowing solid food with its retention in the middle of the esophagus during the last 2 months. Weight loss - 4 kg. The patient's father died from stomach cancer. Esophageal cancer is suspected. Which test is most likely to verify this pathology?

ФЕГДС з біопсією FEGDS with biopsy

Рентгенконтрастне дослідження стравоходу і шлунка X-ray contrast examination of esophagus and stomach

Дихальний уреазний тест Breathing urease test

Внутрішньостравохідна рН-метрія Esophageal pH-metry

Комп'ютерна томограма грудної клітки Computer tomography of the chest

1411 / 1500
Хворому 65-ти років з приводу застійної серцевої недостатності призначена фармакотерапія (фуросемід, спіронолактон, бісопролол, раміприл). Через 2 місяці звернувся до лікаря з приводу розвитку гінекомастії. Запідозрено побічну дію спіронолактону. На який з наведених препаратів можна його замінити у схемі лікування цього хворого? A 65-year-old patient was prescribed pharmacotherapy (furosemide, spironolactone, bisoprolol, ramipril) for congestive heart failure. After 2 months, he consulted a doctor about the development of gynecomastia. A side effect of spironolactone is suspected. Which of the following drugs can replace it in the treatment regimen of this patient?

Ніфедипін Nifedipine

Триметазидин Trimetazidine

Івабрадин Ivabradin

Еплеренон Eplerenone

Торасемід Torasemide

1412 / 1500
Жінка 40-ка років звернулася до лікаря зі скаргами на тупий ниючий біль в лівій поперековій ділянці, пітливість, зниження працездатності, субфебрильну температуру. У дитинстві перенесла туберкульоз шийних хребців. Лікувалася, знята з диспансерного обліку. При фізикальному обстеженні нирки не пальпуються, болючість в проекції лівої нирки. При лабораторному дослідженні в загальному аналізі сечі спостерігаються протеїнурія, піурія. Реакція сечі кисла. Яке дослідження найбільш імовірно дозволить уточнити діагноз цієї хворої? A 40-year-old woman consulted a doctor with complaints of dull, aching pain in the left lumbar region, sweating, reduced work capacity, low-grade fever. She suffered from tuberculosis of the cervical vertebrae as a child She was treated, removed from the medical records. The kidneys are not palpable. In the laboratory examination, proteinuria, pyuria are most likely to clarify the diagnosis of this patient.

Полімеразна ланцюгова реакція (ПЛР) сечі Polymerase chain reaction (PCR) of urine

Проба Манту та повторний візит через 2 дні Mantoux test and return visit in 2 days

Екскреторна урографія Excretory urography

Цистографія Cystography

Ультразвукове дослідження нирок Ultrasound study of kidneys

1413 / 1500
До лікаря звернувся чоловік 36-ти років зі скаргами на пекучий загрудинний біль, що зазвичай виникає через 1-1,5 години після вживання їжі. Зазначає, що біль підсилюється в горизонтальному положенні. При верхній ендоскопії виявлені 2 вогнищевих ураження нижньої третини слизової оболонки стравоходу до 5 мм в межах однієї складки. Яка тактика лікаря буде найбільш доречною? A 36-year-old man went to the doctor with complaints of burning chest pain, which usually occurs 1-1.5 hours after eating. He notes that the pain increases in the horizontal position. During the upper endoscopy, 2 focal lesions of the lower third of the esophagus were detected within one fold. What would be the most appropriate tactics of the doctor?

Хірургічне лікування Surgical treatment

Призначення ненаркотичних анальгетиків Prescription of non-narcotic analgesics

- -

Призначення інгібіторів протонної помпи Prescription of proton pump inhibitors

Призначення кларитроміцину Clarithromycin Prescription

1414 / 1500
Робітник, чоловік 46 років, упродовж 19 років контактував із парами ртуті в концентрації 0,09-0,18 мг/м^3. Обстежений невропатологом. Виявлено: астеніч-ний синдром, мікроорганічна симптоматика з інтенційним тремтінням, позитивними субкортикальними симптомами, наявність депо ртуті (відповідно 0,01 і 0,04 до і після провокації унітіолом), гіпотонічний тип РЕГ. Розвиток якого захворювання можна припустити? The worker, a 46-year-old man, was in contact with mercury vapors at a concentration of 0.09-0.18 mg/m^3 for 19 years. He was examined by a neurologist. It was found: asthenic syndrome, microorganism symptoms with intentional tremors, positive subcortical symptoms, mercury depot (0.01 and 0.04 before and after unitiol provocation, respectively), hypotonic type of REG. What kind of disease can be assumed to develop?

Церебральний атеросклероз Cerebral atherosclerosis

Неврастенія Neurasthenia

Хронічне отруєння ртуттю Chronic mercury poisoning

Паркінсонічний синдром Parkinsonian syndrome

Залишкові явища нейроінфекції Residual effects of neuroinfection

1415 / 1500
Жінка 35 років скаржиться на задишку, відчуття стискання в правій половині грудної клітки, кашель із виділенням невеликої кількості слизово-гнійного харкотиння. Хворіє тиждень. Скарги пов'язує з переохолодженням. Об'єктивно виявлено: температура тіла - 38,7^oС, легкий акроціаноз губ, Рs - 90/хв., АТ - 140/85 мм рт. ст. Права половина грудної клітки відстає в акті дихання. Перкуторно встановлено: справа нижче кута лопатки тупість з межею до верху, дихання не вислуховується. Який найімовірніший діагноз? A 35-year-old woman complains of shortness of breath, a feeling of tightness in the right half of the chest, cough with the release of a small amount of mucous-purulent sputum. She has been ill for a week. She associates the complaints with Objectively revealed: body temperature - 38.7°C, Рs - 140/85 mmHg below the angle of the scapula, dullness with a border to the top, breathing is not heard. What is the most likely diagnosis?

Негоспітальна пневмонія Community-acquired pneumonia

Ателектаз легені Atelectasis of the lung

Ексудативний плеврит Exudative pleurisy

Абсцес легені Lung abscess

Госпітальна пневмонія Nosocomial pneumonia

1416 / 1500
63-річний чоловік звернувся до лікаря зі скаргами на пекучий та ниючий біль шкіри грудної клітки протягом 5-ти днів. Три дні тому на цій ділянці виникло почервоніння та макулопапульозна висипка, яка через день прогресувала у везикули. По буднях він доглядає двох онуків віком 1 та 3 роки, які щеплені згідно Національного календаря профілактичних щеплень. Фізикальне обстеження виявило везикулярний висип на грудній клітці ззаду в межах дерматомів T6-T8 . Неврологічний статус без змін. Який метод постконтактної профілактики дітям буде найбільш ефективним та доречним? A 63-year-old man consulted a doctor with complaints of burning and aching pain in the skin of the chest for 5 days. Three days ago, redness and maculopapular swelling appeared in this area a rash that progressed to vesicles every day.He takes care of two grandsons aged 1 and 3 who are vaccinated according to the National Immunization Schedule.Physical examination revealed a vesicular rash on the posterior chest within dermatomes T6-T8. What method of post-contact prevention for children will be the most effective and appropriate?

Призначити Varicella Zoster-вмісну вакцину дітям Prescribe Varicella Zoster-containing vaccine to children

Уникати контакту з дідусем до зникнення у нього висипу Avoid contact with grandfather until his rash disappears

Призначити терапію ацикловіром дідусеві Prescribe acyclovir therapy to grandfather

Призначити терапію ацикловіром дітям Prescribe acyclovir therapy for children

Призначити дітям специфічний імуноглобулін до збудника Varicella Zoster Prescribe specific immunoglobulin for Varicella Zoster to children

1417 / 1500
Пацієнтка віком 32 роки скаржиться на виражену задуху, сухий кашель, підвищення температури тіла до 39^oС, пітливість. Під час бактеріоскопічного аналізу мокротиння виявлено: КСБ [+]. Проба Манту з 2 ТО - папула 21 мм. Під час рентгенологічно обстеження виявлено: в обох легенях візуалізуються численні симетрично розташовані вогнищеві тіні розмірами 1-2 мм, з нечіткими контурами, малої інтенсивності. Який найімовірніший діагноз? A 32-year-old patient complains of severe shortness of breath, a dry cough, an increase in body temperature up to 39°C, sweating. During bacterioscopic analysis of sputum, it was found: KSB [+] . Mantoux test with 2 TO - papule 21 mm. During the X-ray examination, numerous symmetrically located shadows of 1-2 mm size, with indistinct contours, are visualized. What is the most likely diagnosis?

Саркоїдоз Sarcoid

Хронічний дисемінований туберкульоз легень Chronic disseminated pulmonary tuberculosis

Вогнищевий туберкульоз Focal tuberculosis

Казеозна пневмонія Case pneumonia

Міліарний туберкульоз легень Miliary pulmonary tuberculosis

1418 / 1500
Одинадцятирічна дівчинка скаржиться на біль у суглобах, підвищення температури тіла до 38^oС, слабкість. Захворіла 5 днів тому. Об'єктивно спостерігається: плямисто-папульозний висип на переніссі та обличчі, суглоби не змінені, тони серця послаблені. АТ - 100/80 мм рт. ст. У загальному аналізі крові: еритроцити - 2,6 т/л, лейкоцити - 3,7 г/л, тромбоцити - 12г/л, ШОЕ - 45 мм/год, СРБ (+++). У загальному аналізі сечі: білок - 0,063 г/л, еритроцити - 15-20 в п/з, лейкоцити - 10-14 в п/з. Вкажіть імовірний діагноз. An eleven-year-old girl complains of pain in the joints, an increase in body temperature up to 38°C, weakness. She fell ill 5 days ago. Objectively observed: a spotted-papular rash on nose and face, the joints are not changed, blood pressure - 100/80 mm Hg: erythrocytes - 3.7 g/l, platelets - 12 g/l. , ESR - 45 mm/h, CRP (+++). In the general analysis of urine: protein - 0.063 g/l, erythrocytes - 15-20 in p/z. Specify the probable diagnosis .

Системний червоний вовчак Systemic lupus erythematosus

Дерматоміозит Dermatomyositis

Гостра ревматична лихоманка Acute rheumatic fever

Геморагічний васкуліт Hemorrhagic vasculitis

Вузликовий періартеріїт Nodular periarteritis

1419 / 1500
Хворий 30-ти років надійшов в клініку зі скаргами на задишку, гарячку, біль за грудиною, який посилюється при закиданні голови назад. 3 дні тому хворому проведена екстракція сьомого зуба нижньої щелепи праворуч і розкрита флегмона дна порожнини рота. Незважаючи на це, стан хворого прогресивно погіршувався. При рентгенологічному обстеженні в клініці виявлено розширення тіні середостіння і зниження прозорості його відділів. Яке захворювання виникло у даного хворого? A 30-year-old patient came to the clinic with complaints of shortness of breath, fever, pain behind the sternum, which worsens when the head is thrown back. 3 days ago, the patient underwent extraction of the seventh tooth of the lower jaw on the right and an open phlegmon of the floor of the mouth. Despite this, the patient's condition progressively worsened. During the X-ray examination in the clinic, an expansion of the mediastinum and a decrease in the transparency of the patient's departments were found?

Інфекційний ендокардит Infective endocarditis

Ексудативний перикардит Exudative pericarditis

Гнійний тиреоїдит Suppurative thyroiditis

Гострий гнійний медіастиніт Acute purulent mediastinitis

Заглотковий абсцес Pharyngeal abscess

1420 / 1500
У породіллі 29 років на 3 добу після операції кесарського розтину з'явилась блювота, сильний біль внизу живота, потім по всьому животу, затримка стула та газів. Загальний стан важкий. Шкіряні покрови бліді, губи та язик сухі. Температура тіла - 39,4^oС. Живіт вздутий, симптоми подразнення очеревини позитивні у всіх відділах. Перистальтика кишківника послаблена. Діурез знижений. Матка велика, болюча під час пальпації, з неприємним запахом виділень із піхви. Про яку патологію можна думати? On the 3rd day after cesarean section, a 29-year-old woman in labor developed vomiting, severe pain in the lower abdomen, then throughout the abdomen, stool retention and gas. General condition The skin is pale, the body temperature is 39.4 °C, the peristalsis is weak, the uterus is painful during palpation from the vagina. What pathology can you think about?

Панметрит Panmetritis

Метроендометрит Metroendometritis

Розповсюджений перитоніт Disseminated peritonitis

Інфекція післяопераційної рани Postoperative wound infection

Пельвіоперитоніт Pelvioperitonitis

1421 / 1500
Жінка 25 років скаржиться на висипку на шкірі міжпальцевих ділянок кистей, бокових поверхонь живота, що супроводжується свербежем, особливо у нічний час. Хворіє 4-й день. На цих ділянках шкіри є маленькі папуло-везикули у вигляді спарених цяточок, між ними - тонкі смужки сіруватого кольору. Який найбільш імовірний діагноз? A 25-year-old woman complains of a rash on the skin of the interdigital areas of the hands, the lateral surfaces of the abdomen, which is accompanied by itching, especially at night. She has been ill for the 4th day. there are small papulo-vesicles in the form of paired spots in areas of the skin, between them - thin strips of gray color. What is the most likely diagnosis?

Червоний плаский лишай Lichen planus

Алергічний дерматит Allergic dermatitis

Псоріаз Psoriasis

Нейродерміт Neurodermatitis

Короста Scabies

1422 / 1500
Чоловік 70 років за 2 тижні до госпіталізації у стаціонар спостерігав підвищення температури до 38^oС. З діагнозом пневмонія був направлений до лікарні. Під час огляду встановлено: блідість шкіри, тахікардія. У нижніх відділах легенів - дрібнопухирчаті хрипи. У V точці - короткий протодіастолічний шум. АТ - 140/40 мм рт. ст., пальпується нижній край селезінки. Hb - 40 г/л., еритроцити - 2,5·10^12/л, лейкоцити - 12·10^9/л, ШОЕ - 35 мм/год. На ЕКГ спостерігається: депресія ST в V 5,6. У сечі - протеїнурія. Який найімовірніший діагноз? 2 weeks before hospitalization, a 70-year-old man observed an increase in temperature to 38°C. He was referred to the hospital with a diagnosis of pneumonia. During the examination, it was found: pale skin , tachycardia. In the lower parts of the lungs - a short protodiastolic murmur. Hb - 40 g/l ^12/l, leukocytes - 12·10^9/l, ESR - 35 mm/h. On the ECG, there is depression in V 5.6. What is the most likely diagnosis?

Гострий міокардит Acute myocarditis

Гострий перикардит Acute pericarditis

Бактеріальний ендокардит Bacterial endocarditis

Хронічний мієлолейкоз Chronic myelogenous leukemia

Ревматична вада серця Rheumatic heart disease

1423 / 1500
Пацієнт віком 56 років скаржиться на зниження апетиту, слабкість, серцебиття, біль та відчуття печіння язика, відчуття важкості в надчеревній ділянці та почуття оніміння кінцівок. Об'єктивно спостерігається: блідність шкіри з лимонним відтінком, глосит Гантера, збільшення печінки, селезінки. У загальному аналізі крові виявлено: еритроцити - 2,8·10^12/л, гемоглобін - 100 г/л, кольоровий показник - 1,2. Еритроцити великі, часто овальні з тільцями Жоллі та кільцями Кебота. Який найімовірніший діагноз? A 56-year-old patient complains of decreased appetite, weakness, palpitations, pain and a burning sensation in the tongue, a feeling of heaviness in the epigastric area, and a feeling of numbness in the limbs. Objectively observed : pallor of the skin with a lemon hue, Ganter's glossitis, enlargement of the liver, spleen. In the general blood analysis, erythrocytes - 2.8·10^12/l, hemoglobin - 100 g/l, erythrocytes are large. often oval with Jolly bodies and Cabot rings. What is the most likely diagnosis?

Гемолітична анемія Hemolytic anemia

Залізодефіцитна анемія Iron deficiency anemia

Хронічний гепатит Chronic hepatitis

Апластична анемія Aplastic anemia

В_12(фолієво)-дефіцитна анемія B_12 (folic) deficiency anemia

1424 / 1500
Пацієнтка 40 років доставлена зі скаргами на переймоподібні болі внизу живота та надмірні кров'яні виділення зі статевих шляхів. Останні два роки менструації тривають до 16 днів, надмірні, із згустками, болісні. Під час бімануального дослідження встановлена наявність фіброматозного вузла, що народжується. Оберіть правильну тактику: A 40-year-old female patient was brought in with complaints of spasm-like pains in the lower abdomen and excessive bleeding from the genital tract. For the last two years, menstruation has lasted up to 16 days, excessive, with clots, painful. During the bimanual examination, the presence of a nascent fibromatous node is established. Choose the correct tactics:

Видалення фіброматозного вузла вагінально Removal of a fibromatous node vaginally

Надпіхвова ампутація матки без додатків Supravaginal amputation of the uterus without attachments

Вітамінотерапія Vitamin therapy

Екстирпація матки без додатків Extirpation of the uterus without attachments

Гормональний гемостаз Hormonal hemostasis

1425 / 1500
У дитини віком 2 доби, народженої на 32-ому тижні вагітності з вагою 1700 г наростають зміни з боку дихальної системи, що з'явилися через 8 годин після народження. У матері дитини - ІІІ вагітність, 2 пологи, абортів не було. Попередня дитина загинула від синдрому дихальних розладів. Об'єктивно спостерігається: оцінка за шкалою Сільвермана 6 балів, дихання з порушеним ритмом, апное, звучний видих, зниження тонусу м'язів. Під час аускультацїі: дихання помірно послаблене, вологі хрипи з обох боків. Рентгенологічно виявлено: наявність нодозно-ретикулярної сітки. Чим зумовлений синдром дихальних розладів у дитини? A 2-day-old child born at the 32nd week of pregnancy with a weight of 1700 g has increasing changes in the respiratory system that appeared 8 hours after birth The child's mother - 3rd pregnancy, no abortions. The previous child died from respiratory disorders. Objectively observed: 6 points on the Silverman scale, breathing with a disturbed rhythm, apnea, decreased muscle tone. During auscultation: breathing is moderately weakened, wheezing on both sides. X-ray revealed: the presence of a reticular mesh. What is the cause of the child's respiratory disorders?

Синдромом гіалінових мембран Hyaline membrane syndrome

Діафрагмальною грижею Diaphragmatic hernia

Ателектазами легенів Lung atelectasis

Внутрішньоутробною пневмонією Intrauterine pneumonia

Набряково-геморагічним синдромом Oedema-hemorrhagic syndrome

1426 / 1500
У пацієнта протягом декількох днів з'явилося обмеження периферичного зору ззовні, схоже на заслінку. Причину не знає. Око спокійне, оптичні середовища прозорі. Під час офтальмоскопії з носової сторони при рухах ока коливається сірий «парус» із судинами. Диск зорового нерва та судини не змінені. Який діагноз найімовірніший? For several days, the patient had a limitation of peripheral vision from the outside, similar to a valve. The cause is unknown. The eye is calm, the optical media is transparent. During ophthalmoscopy from the nose sides with eye movements, a gray 'sail' with vessels. The optic nerve disc and vessels are not changed. What is the most likely diagnosis?

Судинна патологія сітківки Retinal vascular pathology

Початкові прояви глаукоми Initial manifestations of glaucoma

Геміанопсія при неврологічній патології Hemianopsia in neurological pathology

Відшарування сітківки Retinal detachment

Підвивих кришталика Subluxation of the lens

1427 / 1500
На прийом до лікаря акушера-гінеколога жіночої консультації звернулася жінка зі строком вагітності 10 тижнів (перше звертання). Який з медичних документів повинен бути заведений лікарем у цьому випадку для контролю за перебігом вагітності? A 10-week pregnant woman applied for an appointment with an obstetrician-gynecologist at the women's consultation (first visit). Which of the medical documents should be prepared by the doctor in this case for pregnancy control?

Історія пологів Birth History

Індивідуальна карта вагітної, породіллі Individual card of a pregnant woman, a woman giving birth

Медична карта амбулаторного хворого Medical card of an outpatient

Карта обліку диспансеризації Dispensary accounting card

Контрольна карта диспансерного спостереження Dispensary monitoring control card

1428 / 1500
Під час вивчення захворюваності на туберкульоз легень були отримані дані про соціально-економічні умови життя та шкідливі звички хворих. Який з наведених методів дозволяє оцінити міру впливу цих чинників на захворюваність туберкульозом? During the study of the incidence of pulmonary tuberculosis, data were obtained on the socio-economic living conditions and bad habits of the patients. Which of the following methods allows you to assess the degree of influence of these factors on the incidence tuberculosis?

Обчислення коефіцієнта достовірності Calculation of reliability coefficient

Обчислення коефіцієнта регресії Calculation of regression coefficient

Обчислення показника відповідності Calculation of compliance indicator

Обчислення коефіцієнта кореляції Calculation of correlation coefficient

Обчислення стандартизованих показників Calculation of standardized indicators

1429 / 1500
Жінці віком 35 років через пневмонію призначено ампіцилін. За 30 хвилин після внутрішньом'язової ін'єкції препарату пацієнтка відчула різку слабкість, з'явився кашель, задишка, біль у грудні клітці. Об'єктивно спостерігається: ціаноз, набряк повік, обличчя з червоним висипом. Пульс - 120/хв, АТ - 70/20 мм рт. ст. Тони серця глухі, дихання часте, поверхневе, з різнокаліберними вологими хрипами. Варикозне розширення вен на правій гомілці. Яка найімовірніша причина раптового погіршення стану жінки? A 35-year-old woman was prescribed ampicillin due to pneumonia. 30 minutes after the intramuscular injection of the drug, the patient felt sharp weakness, cough, shortness of breath, pain appeared Objectively observed: cyanosis, red rash, heart rate - 70/min varicose veins on the right leg. What is the most likely cause of the sudden deterioration of the woman's condition?

Тромбоемболія легеневої артерії Thromboembolism of the pulmonary artery

Кропив'янка Hives

Анафілактичний шок Anaphylactic shock

Астматичний напад Asthma attack

Набряк Квінке Quincke edema

1430 / 1500
У хворого 35-ти років, який у зв'язку із хронічним гломерулонефритом 3 останні роки перебуває на гемодіалізі, з'явилися перебої в діяльності серця, гіпотонія, наростаюча слабкість, задишка. На ЕКГ: брадикардія, атріовентрикулярна блокада І ступеня, високі загострені зубці Т. Напередодні було грубе порушення питного та дієтичного режимів. Які біо-хімічні зміни є найбільш імовірною причиною вищевказаної клінічної картини? A 35-year-old patient, who has been on hemodialysis for the last 3 years due to chronic glomerulonephritis, had heart failure, hypotension, increasing weakness, shortness of breath. On the ECG: bradycardia, atrioventricular blockade of the first degree, high sharp T waves. The day before, there was a gross violation of the drinking and dietary regimes. What bio-chemical changes are the most likely cause of the above-mentioned clinical picture?

Гіпокальціємія Hypocalcemia

Гіперкаліємія Hyperkalemia

Гіпергідратація Hyperhydration

Гіпернатріємія Hypernatremia

Гіпокаліємія Hypokalemia

1431 / 1500
Для Прикарпатського регіону притаманно постійно висока (більше 80%) вологість атмосферного повітря. У холодний період року за помірно низьких температур повітря населення цього регіону відчуває сильний холод. Який шлях тепловіддачі тоді збільшується? The Carpathian region is characterized by constantly high (more than 80%) atmospheric air humidity. In the cold period of the year, with moderately low air temperatures, the population of this region feels very cold. What a way heat transfer then increases?

Конвекція Convection

Випромінювання Radiation

Радіація Radiation

Кондукція Conduction

Випаровування Evaporation

1432 / 1500
Жінка 23-х років. Скарги на ріжучий біль внизу живота, почастішання сечовипускання, підвищення температури тіла до 37,7^oC, збільшення виділень зі статевих шляхів жовтуватого кольору. Остання менструація 18 днів назад. В анамнезі: погіршення стану через 5-6 днів після незахищеного статевого акту. Симптом Щоткіна негативний. Бімануально: двобічне збільшення додатків. Діагноз: 23-year-old woman. Complaints of cutting pain in the lower abdomen, increased frequency of urination, increase in body temperature to 37.7^oC, increased secretions from the genital tract of yellowish color Last menstruation 18 days ago: worsening of condition after unprotected intercourse. Bimanual: bilateral enlargement.

Бактеріальний вагіноз Bacterial vaginosis

Ендометрит Endometritis

Апендицит Appendicitis

Кандидозний вульвовагініт Candida vulvovaginitis

Двобічний сальпінгоофорит Bilateral salpingo-oophoritis

1433 / 1500
Дитина віком 6 років захворіла гостро: підвищилася температура тіла, з'явилися головний біль і біль під час ковтання. Через 3 години з'явилися яскраво-червоний дрібно-крапчастий висип на гіперемованій шкірі, рясніший на боковій поверхні тулуба та в природних складках, відмежована гіперемія слизової оболонки ротоглотки, гнійний наліт на мигдаликах. Яке захворювання можна припустити у дитини? A 6-year-old child became acutely ill: body temperature rose, headache and pain during swallowing appeared. After 3 hours, bright red small spotted rash on the hyperemic skin, more abundant on the lateral surface of the body and in the natural folds, limited hyperemia of the mucous membrane of the oropharynx, purulent plaque on the tonsils. What disease can be assumed in the child?

Кір Measles

Інфекційний мононуклеоз Infectious mononucleosis

Краснуха Krasnukha

Дифтерія Diphtheria

Скарлатина Scarlatina

1434 / 1500
Пацієнтка 74 років скаржиться на біль та здуття живота, нудоту. Страждає на ішемічну хворобу серця, постінфарктний та дифузний кардіосклероз. Об'єктивно спостерігається: стан важкий, живіт здутий, черевна стінка слабко бере участь в акті дихання. Під час лапароскопії виявлено: у черевній порожнині невелика кількість каламутного випоту, одна з петель тонкої кишки темно-синюшнього кольору. Поставте діагноз. A 74-year-old patient complains of pain and abdominal distension, nausea. She suffers from coronary heart disease, post-infarction and diffuse cardiosclerosis. Objectively observed: the condition is severe, the abdomen is distended , the abdominal wall is weakly involved in the act of breathing. During laparoscopy, a small amount of cloudy effusion was found in the abdominal cavity, one of the loops of the small intestine is dark blue. Make a diagnosis.

Бешихове запалення Beshikov inflammation

Тромбоз мезентеріальних судин Thrombosis of mesenteric vessels

Гостра кишкова непрохідність Acute intestinal obstruction

Ішемічний абдомінальний синдром Ischemic abdominal syndrome

Заворот кишечнику Intestinal twist

1435 / 1500
Пацієнтка віком 12 років скаржиться на слабкість, запаморочення, головний біль, підвищення температури тіла до 38^oС. Об'єктивно спостерігається: температура тіла 37,8^oС, слизові та шкіра бліді, зів без змін. Пальпуються збільшені до 2-х см підщелепні та шийні лімфовузли, щільні, неболючі. Патологічних змін внутрішніх органів не виявлено. У загальному аналізі крові виявлено: еритроцити - 2,8·10^12/л, гемоглобін - 85 г/л, кольоровий показник - 0,9, лейкоцити - 10·10^9/л, еозинофіли - 0%, паличкоядерні - 1%, сегментоядерні - 8%, лімфоцити - 47%, ретикулоцити - 0,5%, тромбоцити - 60·10^9/л, бластні клітини - 44%. Який найімовірніший діагноз? A 12-year-old patient complains of weakness, dizziness, headache, an increase in body temperature up to 38°C. Objectively observed: body temperature 37.8°C , mucous membranes and skin are unchanged. Submaxillary and cervical lymph nodes are palpable, painless. No pathological changes in internal organs are detected: erythrocytes - 2.8·10^12/l , hemoglobin - 85 g/l, color index - 0.9, leukocytes - 10·10^9/l, eosinophils - 0%, rod-nuclear - 1%, segmentonuclear - 8%, lymphocytes - 47%, reticulocytes - 0.5 %, platelets - 60·10^9/l, blast cells - 44%. What is the most likely diagnosis?

Інфекційний мононуклеоз Infectious mononucleosis

Гострий лейкоз Acute leukemia

Лімфогрануломатоз Lymphogranulomatosis

Гострий еритромієлоз Acute erythromyelosis

Хронічний лімфолейкоз Chronic lymphocytic leukemia

1436 / 1500
Хлопчик 14 років скаржиться на втомлюваність, наявність набряків на обличчі, головний біль, малі порції сечі. Сеча кольору м'ясних помиїв. Перебуває на диспансерному обліку з приводу хронічного гломерулонефриту з 8 років. Креатинін крові - 0,350 ммоль/л, сечовина крові - 10,4 ммоль/л. Яке ускладнення, найімовірніше, зумовлює таку клінічну картину? A 14-year-old boy complains of fatigue, swelling on the face, headache, small portions of urine. The urine is the color of meat slops. He is in the clinic for chronic glomerulonephritis from the age of 8. Blood creatinine - 0.350 mmol/l, blood urea - 10.4 mmol/l. What complication most likely causes this clinical picture?

Хронічна недостатність кровообігу Chronic circulatory failure

Туберкульоз нирок Kidney tuberculosis

Гостра ниркова недостатність Acute renal failure

Хронічна ниркова недостатність Chronic renal failure

Хронічний гломерулонефрит Chronic glomerulonephritis

1437 / 1500
Пацієнтка віком 26 років скаржиться на переймоподібний біль в животі, проноси зі значною кількістю слизу і крові, підвищення температури тіла до 37,5-38,0^oС. Об'єктивно спостерігається: блідість шкіри та слизових оболонок, астенічна будова тіла. Під час пальпації виявлена болючість за ходом товстої кишки. В ході колонофіброскопії виявлено: стінка прямої та сигмоподібної кишки набрякла, ерозії, дрібні виразки, в просвіті слиз із кров'ю. Який імовірний діагноз? A 26-year-old patient complains of spasm-like abdominal pain, diarrhea with a significant amount of mucus and blood, an increase in body temperature to 37.5-38.0°C. Objectively observed: pallor of the skin and mucous membranes, tenderness of the colon during palpation, swelling of the rectal and sigmoid wall, small ulcers, mucus in the lumen. What is the probable diagnosis?

Хронічний ентерит Chronic enteritis

Рак товстої кишки Colon cancer

Неспецифічний виразковий коліт Nonspecific ulcerative colitis

Хвороба Крона Crohn's disease

Дизентерія Dysentery

1438 / 1500
Жінка 35 років, шліфувальниця, скаржиться на мерзлякуватість, похолодання та посиніння кінчиків пальців, тугорухомість у суглобах кистей, відчуття стягнутості шкіри обличчя та кистей. Під час огляду: амімія обличчя, звуження ротової щілини за типом <<кисета>>, шкіра на щоках та кистях потовщена, кінчики пальців рук бліді, холодні. Над легенями прослуховується коробковий звук, поодинокі сухі хрипи. результат аналізу крові: еритроцити - 3,8·10^12/л, лейкоцити - 4,8·10^9/л, ШОЕ - 45 мм/год. СРБ++. Який діагноз найбільш імовірний? A 35-year-old woman, a grinder, complains of frostbite, coldness and blueness of the fingertips, stiffness in the joints of the hands, a feeling of tightness of the skin of the face and hands. During the examination: amymia face, narrowing of the oral cavity, the skin on the cheeks and hands is thickened, the tips of the fingers are cold, a box sound is heard above the lungs, the result of the blood test is 3.8·10^12 /l, leukocytes - 4.8·10^9/l, ESR - 45 mm/h. What is the most likely diagnosis?

Системна склеродермія Systemic scleroderma

Облітеруючий ендартеріїт кінцівок Endarteritis obliterans of the limbs

Мікседема Myxedema

Вібраційна хвороба Vibration disease

Хвороба Рейно Raynaud's disease

1439 / 1500
У пацієнта віком 36 років, після вживання напередодні алкоголю та жирної їжі, вночі виник напад гострого артриту 1-го плесне-фалангового суглоба правої стопи. Під час огляду спостерігається: перший палець набряклий, шкіра синюшна, гаряча на дотик. У загальному аналізі крові виявлено: L - 12,0·10^12/л, зсув вліво, ШОЕ - 42 мм/год. Виберіть варіант стартової терапії. A 36-year-old patient, after consuming alcohol and fatty food the night before, developed an attack of acute arthritis of the 1st metatarsophalangeal joint of the right foot at night. During the examination, it is observed : the first finger is swollen, the skin is bluish, hot to the touch. The general blood test revealed: L - 12.0·10^12/l, shift to the left, ESR - 42 mm/h. Choose the starting therapy option.

Дипроспан внутрішньосуглобово Diprospan intra-articularly

Мазь з НПЗП місцево Ointment with NSAIDs locally

НПЗП NSAIDs

Преднізолон Prednisone

Колхіцин Colchicine

1440 / 1500
Чоловік 19-ти років, звернувся до лікаря зі скаргами на безболісні ерозії на голівці статевого члена, які з'явилися 2-3 дні тому. У анамнезі незахищений статевий контакт приблизно місяць тому. При обстеженні на голівці статевого члена ерозія овальної форми діаметром до 5 мм, безболісна, яскраво-червоного кольору. Пахвинні лімфовузли не збільшені. Яке першочергове лабораторне дослідження буде найбільш доречним? A 19-year-old man turned to the doctor with complaints of painless erosions on the head of the penis that appeared 2-3 days ago. He has a history of unprotected sex contact about a month ago. On examination, an oval-shaped erosion of up to 5 mm in diameter. The inguinal lymph nodes are not enlarged. What would be the most appropriate laboratory test?

Реакція іммобілізації блідих трепонем Reaction of immobilization of pale treponems

Реакція імунофлюоресценції Immunofluorescence reaction

Мікрореакція преципітації з кардіоліпіновим антигеном Microreaction of precipitation with cardiolipin antigen

- -

Культуральне дослідження Cultural Research

1441 / 1500
Жінка 55 років скаржиться на багаторазовий пронос, лущення та пігментацію відкритих ділянок тіла (шиї, кистей та ступней), роздратованість та занепокоєння. Про яку вітамінну недостат-ність йдеться? A 55-year-old woman complains of repeated diarrhea, peeling and pigmentation of open areas of the body (neck, hands and feet), irritability and anxiety. What vitamin deficiency is this? ?

Рибофлавіну Riboflavin

Нікотинової кислоти Nicotinic acid

Ретинолу Retinol

Тіаміну Thiamine

Пантотенової кислоти Pantothenic acid

1442 / 1500
У жінки, яка хворіє на ІХС, щотижня виникають напади серцебиття, задишки, перебої у роботі серця. На ЕКГ під час нападу спостерігається: інтервали R-R мають різну тривалість, зубці P відсутні, хвилі f, зубці R різної амплітуди. На ЕхоКГ виявлено ФВ - 38%. Призначена схема лікування (аміодарон, варфарин). Який із наведених нижче показників у цьому разі потребує постійного моніторування? A woman suffering from coronary heart disease has attacks of palpitations, shortness of breath, heart failure every week. On the ECG during the attack, the following is observed: RR intervals have different durations, There are no P waves, R waves of varying amplitude. On echocardiography, PV is detected (amiodarone, warfarin). In this case, which of the following indicators needs constant monitoring?

Рівень фібриногену Fibrinogen level

Міжнародне нормалізоване співвідношення International Normalized Ratio

Гемоглобін Hemoglobin

Протромбіновий індекс Prothrombin index

Рівень тромбоцитів Platelet level

1443 / 1500
Хлопчик віком 10 років лікувався в кардіологічному відділенні з приводу ревматизму, І атака, активна фаза, активність ІІ ступеня. Виписаний у задовільному стані. Який препарат найдоцільніше призначити для профілактики вторинного ревматизму? A 10-year-old boy was treated in the cardiology department for rheumatism, 1st attack, active phase, 2nd degree activity. He was discharged in a satisfactory condition. What drug is most appropriate to prescribe for prevention secondary rheumatism?

Еритроміцин Erythromycin

Біцилін-1 Bicilin-1

Біцилін-5 Bicillin-5

Ампіцилін Ampicillin

Оксацилін Oxacillin

1444 / 1500
Пацієнт віком 65 років скаржиться на задишку, кашель з виділенням рожевої мокроти, що піниться, відчуття нестачі повітря, страх смерті. О'бєктивно спостерігається: ортопное, шкіра бліда, акроціаноз, холодний липкий піт. Під час аускультації вислуховується: дихання жорстке, в нижньо-задніх відділах з обох сторін - вологі дрібно- та середньопухирчаті хрипи. ЧД - 40/хв. Тони серця різко приглушені. На верхівці серця - ритм галопу. Який найімовірніший діагноз? A 65-year-old patient complains of shortness of breath, cough with frothy pink sputum, feeling of lack of air, fear of death. Objectively observed: orthopnea, pale skin , acrocyanosis. During auscultation, breathing is hard, in the lower and back parts - rales - 40/min. Heart sounds are muffled the most likely diagnosis?

Інфаркт-пневмонія Pneumonia heart attack

Набряк легенів Pulmonary edema

Тромбоемболія легеневої артерії Thromboembolism of the pulmonary artery

Бронхіальна астма Bronchial asthma

Вогнищева пневмонія Focal pneumonia

1445 / 1500
У хлопчика, який хворіє на гемофілію А, за 7 годин після травми з'явився біль у колінному суглобі. Об'єктивно спостерігається: суглоб збільшився у розмірі, шкіра над ним гіперемована, гаряча на дотик, рухи у коліні різко обмежені. Що потрібно призначити хлопчику насамперед? A boy suffering from hemophilia A developed pain in the knee joint 7 hours after the injury. Objectively observed: the joint increased in size, the skin over him, hyperemic, hot to the touch, movements in the knee are sharply limited. What should be prescribed to the boy first?

Уведення кріопреципітату Introduction of cryoprecipitate

Пункцію суглоба Joint puncture

Уведення Е-АКК Entering E-ACC

Уведення вікасолу Vikasol administration

Уведення діцинону Introduction of dizinon

1446 / 1500
До жіночої консультації звернулась породілля на 18-й день після пологів зі скаргами на біль та ущільнення у правій молочній залозі. Температура тіла - 38,5^oС. Під час огляду встановлено: у ділянці соска - тріщина, болючість під час пальпації. Яке ускладнення виникло в післяпологовому періоді? On the 18th day after giving birth, a woman in labor came to the women's consultation with complaints of pain and tightness in the right mammary gland. The body temperature is 38.5°C. Under at the time of the examination it was established: there was a crack in the area of ​​the nipple, tenderness during palpation. What complication occurred in the postpartum period?

Додаткова долька молочної залози Additional breast lobe

Рак молочної залози Breast cancer

Лактаційний мастит Lactation mastitis

Кіста правої молочної залози Right breast cyst

Лактостаз Lactostasis

1447 / 1500
Жінка звернулася до гінеколога з приводу планування вагітності і одержала рекомендацію збільшити споживання продуктів із високим вмістом фолієвої кислоти, зокрема сої, хліба з борошна грубого помелу, фруктів, зелених листяних овочів. Такі зміни в раціоні харчування вагітної сприятимуть профілактиці: A woman consulted a gynecologist about pregnancy planning and was advised to increase her intake of foods high in folic acid, including soy, wholemeal bread, fruits, green leafy vegetables vegetables. Such changes in the diet of a pregnant woman will contribute to prevention:

Незарощення невральної трубки у плода Neural tube failure in fetus

Залізодефіцитної анемії у вагітної Iron deficiency anemia in a pregnant woman

Незарощення тім'ячка No overgrowth of crown

Багатоводдя Bagotovdya

Рахіту Rickets

1448 / 1500
Пацієнтка віком 44 роки скаржиться на загальну слабкість, сонливість, серцебиття, сухість шкіри, зниження працездатності. Об'єктивно спостерігається: шкіра звичайного кольору, пульс - 72/хв, АТ - 125/80 мм рт. ст., щитоподібна залоза дифузно збільшена до ІІ ступеня, щільна, рухлива, болюча. Під час УЗД виявлено: зниження ехогенності, неоднорідність ехоструктури, потовщення капсули залози. У аналізі крові спостерігається: рівень Т4 - знижений, ТТГ - підвищений, антитіла до тиреопероксидази та антимікросомальні антитіла значно підвищені. Який найімовірніший діагноз? A 44-year-old female patient complains of general weakness, drowsiness, palpitations, dry skin, reduced work capacity. Objectively observed: normal-colored skin, pulse - 72/min , blood pressure - 125/80 mmHg, the thyroid gland is diffusely enlarged to the II degree, during ultrasound, it is found: a decrease in echogenicity, a thickening of the gland capsule. In the blood analysis, the level of T4 is reduced , TSH is increased, antibodies to thyroperoxidase and antimicrosomal antibodies are significantly increased. What is the most likely diagnosis?

Рак щитоподібної залози Thyroid cancer

Підгострий тиреоїдит Subacute thyroiditis

Дифузний токсичний зоб ІІ ступеня Diffuse toxic goiter II degree

Дифузний еутиреоїдний зоб ІІ ступеня Diffuse euthyroid goiter II degree

Аутоімунний тиреоїдит Autoimmune thyroiditis

1449 / 1500
Пацієнтка віком 27 років скаржиться на різкий біль внизу живота, запаморочення. З анамнезу відомо: остання менструація 2 тижні тому. Тест на ХГЧ - негативний. Об'єктивно спостерігається: шкірні покриви бліді, АТ - 80/60 мм рт. ст. Пульс - 92/хв. Живіт напружений, болючий більше справа в нижніх відділах. Під час піхвового дослідження виявлено: матка нормальних розмірів, придатки болючі під час пальпації, нависання заднього склепіння. Який найімовірніший діагноз? A 27-year-old patient complains of sharp pain in the lower abdomen, dizziness. It is known from the anamnesis: the last menstruation was 2 weeks ago. The hCG test is negative. Objectively observed : skin is pale, blood pressure is 80/60 mm Hg, the abdomen is tense, more painful in the lower parts. . What is the most likely diagnosis?

Апоплексія яєчника Ovarian apoplexy

Гострий апендицит Acute appendicitis

Перекрут ніжки кісти яєчника Ovarian cyst pedicle torsion

Загострення хронічного правостороннього аднекситу Exacerbation of chronic right-sided adnexitis

Позаматкова вагітність Ectopic pregnancy

1450 / 1500
Хвора 38-ми років, бухгалтер. На прийомі багато говорить, мова прискорена, швидко переключається з теми на тему, що робить розуміння мови хворої скрутним. Не може всидіти на стільці, жестикулює, несподівано починає співати, сміятися, декламувати вірші. Вважає себе ''найздорові-шою'', ''найщасливішою'', заявляє, що ''всі заздрять її зовнішності та чудовому голосу''. Зі слів родичів, останні 5 днів не спить вночі, відзначається підвищений апетит, гнівливість. 3 роки тому лікувалася у психіатра з приводу депресії. Який синдромальний діаг-ноз є найбільш імовірним? The patient is 38 years old, an accountant. At the appointment, she talks a lot, her speech is accelerated, she quickly switches from topic to topic, which makes it difficult to understand the patient's language. She cannot sit still on a chair, gestures, suddenly begins to sing, laugh, recite poems. She considers herself the 'healthiest', 'the happiest', declares that 'everyone envies her appearance and wonderful voice'. 5 days awake at night, there is increased appetite, anger. She was treated by a psychiatrist 3 years ago. What is the most likely syndromic diagnosis?

Шизофренія, параноїдна форма Schizophrenia, paranoid form

Маніакальний синдром Manic syndrome

Деліріозний синдром Delirious syndrome

Депресивний синдром Depressive syndrome

Синдром гіперактивності з дефіцитом уваги Attention deficit hyperactivity disorder

1451 / 1500
У дитини віком 18 місяців з'явилося облисіння потилиці, неспокійний сон, підвищена пітливість. Про яку хворобу можна думати? An 18-month-old child developed baldness on the back of the head, restless sleep, increased sweating. What disease can you think about?

Анемія Anemia

Спазмофілія Spasmophilia

Хондродістрофія Chondrodystrophy

Фосфат-діабет Phosphate-Diabetes

Рахіт Rickets

1452 / 1500
У 200 пацієнтів з гіпертонічною хворобою досліджувалась величина артеріального тиску та вік пацієнта. Якою статистичною величиною потрібно скористатися, щоб виміряти силу зв'язку між вказаними ознаками? In 200 patients with hypertension, the value of blood pressure and the age of the patient were studied. What statistical value should be used to measure the strength of the relationship between the specified characteristics?

Коефіцієнтом Ст'юдента Student coefficient

Помилкою репрезентативності Representativeness error

Коефіцієнтом варіації Coefficient of variation

Сигмальним відхиленням By sigmal deviation

Коефіцієнтом кореляції Correlation coefficient

1453 / 1500
Під час дослідження свинини у 24 зрізах на компресоріумі виявлено 2 трихінели. Що потрібно зробити з м'ясом? During examination of pork, 2 trichinella were found in 24 cuts on the compressorium. What should be done with the meat?

Проварити дрібними шматками Cook in small pieces

Реалізувати через мережу громадського харчування Implement through a catering network

Переробити на ковбасні вироби Process into sausage products

Технічно утилізувати Technically Dispose

Глибоко заморозити Deep Freeze

1454 / 1500
У дитини 3-х місяців з ознаками рахіту мають місце симптоми Хвостека, Труссо, Маслова. Добу тому батьки спостерігали напад ціанозу, витрішкуватість, дитина не дихала, вкрилась липким потом. Через одну хвилину настав гучний видих, після чого стан нормалізувався. Яка основа виникнення вищезазначених симптомів захворювання дитини? A 3-month-old child with signs of rickets has symptoms of Khvostek, Trousseau, Maslov. A day ago, the parents observed an attack of cyanosis, lethargy, the child was not breathing, covered with sticky After one minute, there was a loud exhalation, after which the condition normalized. What is the basis of the above-mentioned symptoms of the child?

Підвищення рівня фосфору крові Increased blood phosphorus level

Метаболічний ацидоз Metabolic acidosis

Зниження рівня фосфору крові Decrease in blood phosphorus

Зниження рівня кальцію крові Decreased blood calcium level

Підвищення рівня кальцію крові Increased blood calcium level

1455 / 1500
Дівчинка 12 років поступила до гінекологічного відділення зі скаргами на рясні кров'янисті виділення зі статевих шляхів протягом 10 діб. В анамнезі вказано менархе з 11 років, менструації нерегулярні. Під час ректо-абдомінального обстеження патології не виявлено. Який попередній діагноз? A 12-year-old girl was admitted to the gynecological department with complaints of profuse bleeding from the genital tract for 10 days. The anamnesis indicates menarche since the age of 11, menstruation is irregular. During the recto-abdominal examination, no pathology was detected. What was the previous diagnosis?

Синдром Штейна-Левенталя Stein-Leventhal syndrome

Генітальний інфантилізм Genital infantilism

Ідіопатична тромбоцитопенічна пурпура Idiopathic thrombocytopenic purpura

Аномальна маткова кровотеча Abnormal uterine bleeding

Гіповітаміноз Hypovitaminosis

1456 / 1500
Чоловік 18-ти років звернувся до лікаря зі скаргами на переймоподібний біль в животі, часті рідкі випорожнення з домішками слизу та свіжої крові, що тривають 4 місяці. За цей час схуд на 10 кг. При фізикальному обстеженні живіт м'який, болючий по ходу товстої кишки зліва, сигмоподібна кишка спазмована. При лабораторному дослідженні крові еритроцити - 3,2·10^12/л, гемоглобін - 92 г/л, лейкоцити - 10,6·10^9/л, ШЗЕ- 34 мм/год. Результат бак. посіву випорожнень негативний. На колоноскопії виявлено запалення із втратою судинного малюнку, ділянка підвищеної кровоточивості 25 см з чіткими межами починаючи від краю анусу. Яка лікувальна тактика буде найбільш доречною? An 18-year-old man consulted a doctor with complaints of spasmodic abdominal pain, frequent loose stools with mucus and fresh blood, lasting 4 months. lost weight by 10 kg. During physical examination, the abdomen is soft, painful in the course of the colon, the sigmoid colon is spasmodic. During the laboratory examination of blood, erythrocytes - 3.2·10^12/l, hemoglobin - 92 g/l 10.6·10^1/L, 34 mm/h. Colonoscopy revealed inflammation with a 25 cm bleeding area starting from the edge of the anus most appropriate?

Переливання еритроцитарної маси Transfusion of erythrocyte mass

Призначення преднізолону та месаламіну Prednisone and mesalamine prescription

Призначення метронідазолу Prescription of metronidazole

Хірургічне лікування Surgical treatment

Повторний забір випорожнень на бактеріологічне дослідження Re-sampling of feces for bacteriological examination

1457 / 1500
Пацієнтка віком 12 років захворіла гостро з підвищення температури тіла до 38,5^oС та появи висипу на шкірі. Об'єктивно спостерігається: стан середнього ступеня тяжкості, гепатоспленомегалія, дрібнокрапчастий та плямисто-папульозний висип на згинальних поверхнях кінцівок, бічних поверхнях тулуба, внизу живота, відмежоване рожево-синюшне забарвлення кистей та стоп. Який найімовірніший діагноз? A 12-year-old patient became acutely ill with an increase in body temperature to 38.5^oС and the appearance of a rash on the skin. Objectively observed: a condition of moderate severity, hepatosplenomegaly , a small-spotted and papular rash on the flexor surfaces of the limbs, the lateral surfaces of the trunk, the lower part of the abdomen, a demarcated pinkish-bluish coloration of the hands and feet. What is the most likely diagnosis?

Вітряна віспа Chicken Pox

Інфекційний мононуклеоз Infectious mononucleosis

Кір Measles

Скарлатина Scarlatina

Псевдотуберкульоз Pseudotuberculosis

1458 / 1500
Під час організації санаторно-курортного харчування осіб, які постраждали внаслідок Чорнобильської катастрофи, було запропоновано забезпечити в раціонах регламентовану кількість нутрієнтів, що вступають у конкурентні взаємовідносини з радіонуклідами та запобігають їхньому всмоктуванню у шлунково-кишковому тракті. Які нутрієнти мають пріоритетне значення для забезпечення такої біологічної дії їжі? During the organization of sanatorium-resort nutrition for persons affected by the Chernobyl disaster, it was proposed to provide in the diets a regulated amount of nutrients that enter into a competitive relationship with radionuclides and prevent to their absorption in the gastrointestinal tract. What nutrients are of priority for ensuring such a biological effect of food?

Рослинні жири, харчові волокна Vegetable fats, dietary fibers

Жиророзчинні вітаміни, тваринні білки Fat-soluble vitamins, animal proteins

Залізо, цинк, легкозасвоювані вуглеводи Iron, zinc, easily digestible carbohydrates

Калій, кальцій, харчові волокна Potassium, calcium, dietary fiber

Магній, фосфор, рослинні білки Magnesium, phosphorus, vegetable proteins

1459 / 1500
Пацієнт віком 32 роки, дивлячись на малюнок на шпалерах, бачить, як лінії починають рухатися, утворюючи силуети химерних тварин. Замість люстри, що висить на стелі, бачить гігантського восьминога. Встановіть психопатологічний симптом. A 32-year-old patient, looking at a picture on the wallpaper, sees the lines begin to move, forming silhouettes of whimsical animals. Instead of a chandelier hanging from the ceiling, he sees a giant Octopus. Establish a psychopathological symptom.

Псевдогалюцинації Pseudohallucinations

Парейдолічні ілюзії Pareidolic illusions

Функціональні галюцинації Functional hallucinations

Дереалізація Derealization

Зорові галюцинації Visual hallucinations

1460 / 1500
У кар'єрі видобувається гранітний матеріал і подрібнюється за допомогою вибухів і ручних пневмомолотків. Під час буріння шпурів для вибухівки і подрібнення брил граніту робітники зазнають дії середньої і високочастотної локальної вібрації. Вимірювання рівнів вібрації показало, що вони перевищують гранично допустимі. Які специфічні зміни можуть виникати у робітників унаслідок тривалої роботи в таких умовах? Granite material is mined in the quarry and crushed using explosions and manual pneumatic hammers. During the drilling of holes for explosives and crushing of granite blocks, workers are exposed to medium and high-frequency local vibration. Measurements of vibration levels have shown that they exceed the maximum permissible. What specific changes can occur in workers as a result of long-term work in such conditions?

Сенсорна полінейропатія з ангіоспастичним синдромом Sensory polyneuropathy with angiospastic syndrome

Вегетосудинна дистонія Vegetovascular dystonia

Дерматити кистей Dermatize hands

Нейросенсорна приглухуватість Sensorydeafness

Психастенія Psychasthenia

1461 / 1500
Під час огляду у дванадцятирічної дівчинки виявлено ущільнення і незначну припухлість у ділянці лівої грудної залози, гіперемія відсутня. Вкажіть подальші рекомендації. During the examination, a twelve-year-old girl was found to have thickening and slight swelling in the area of ​​the left mammary gland, there is no hyperemia. Specify further recommendations.

УВЧ на грудну залозу UHF on breast gland

Антибіотикотерапія Antibiotic therapy

Параареолярний розріз із дренуванням Paraareolar incision with drainage

Динамічне спостереження без лікування Dynamic observation without treatment

Радіарний розріз над припухлістю з наступним дренуванням Radial incision over swelling followed by drainage

1462 / 1500
У дитини 2-х років на тлі відносного соматичного здоров'я після гри з ''мозаї-кою'' раптово виникли кашель, стридорозне дихання, потяг до блювання, ціаноз. При огляді дитини першочергово лікар має припустити: A 2-year-old child, against the background of relative somatic health, suddenly developed a cough, stridorous breathing, urge to vomit after playing with a 'mosaic' , cyanosis. When examining a child, the doctor should first assume:

Аспірацію стороннього тіла Foreign body aspiration

Пневмонію Pneumonia

Гострий ларинготрахеїт Acute laryngotracheitis

Гострий обструктивний бронхіт Acute obstructive bronchitis

Кашлюк Whooping cough

1463 / 1500
Через порушення техніки безпеки під час роботи з джерелами іонізуючого випромінювання робітник упродовж години піддавався загальному опроміненню в дозі близько 8 Гр. Яка форма променевої хвороби розвинулася у нього? Due to a violation of safety techniques while working with sources of ionizing radiation, the worker was exposed to general radiation at a dose of about 8 Gy for an hour. What form of radiation sickness did he develop?

Гостра променева хвороба, кишкова форма Acute radiation sickness, intestinal form

Гостра променева хвороба, церебральна форма Acute radiation sickness, cerebral form

Гостра променева хвороба, токсемічна форма Acute radiation sickness, toxemic form

Хронічна променева хвороба, кістковомозкова форма Chronic radiation sickness, bone marrow form

Гостра променева хвороба, кістковомозкова форма Acute radiation sickness, bone marrow form

1464 / 1500
Хлопчик 12-ти років надійшов до клініки зі скаргами на відчуття серцебиття, перебої та біль у серці, задишку. Ці симптоми найчастіше виникали після фізичних навантажень і закінчувались синкопальними станами. Згодом лікарем було встановлено діагноз гіпертрофічної обструктивної кардіоміопатії. Який препарат textbfПРОТИПОКАЗАНИЙ при лікуванні цієї патології? A 12-year-old boy came to the clinic with complaints of palpitations, interruptions and pain in the heart, shortness of breath. These symptoms most often occurred after physical exertion and ended with syncopal states Subsequently, the doctor diagnosed hypertrophic obstructive cardiomyopathy. Which drug is CONTRAINDICATED in the treatment of this pathology?

Дигоксин Digoxin

- -

Бісопролол Bisoprolol

Верапаміл Verapamil

Еналаприл Enalapril

1465 / 1500
На території міста відзначено підвищений рівень марганцю в атмосферному повітрі, питній воді та продуктах харчування сільськогосподарського походження. Як можна охарактеризувати дію цих чинників на здоров'я населення? An elevated level of manganese in the atmospheric air, drinking water and food products of agricultural origin was noted on the territory of the city. How can the effect of these factors on the health of the population be characterized?

Синергічна Synergistic

Роздільна Separate

Поєднана Combined

Комплексна Comprehensive

Комбінована Combined

1466 / 1500
У чоловіка маса тіла вище середньої норми на 40% при <<індексі Кетле>> - 29. Вкажіть ступінь ожиріння у пацієнта. The man's body weight is 40% above the average norm with <> - 29. Specify the degree of obesity in the patient.

Другий Second

Четвертий Fourth

Третій Third

Перший First

Надлишкова маса тіла Excess body weight

1467 / 1500
До лікаря звернулися діти школи-інтернату за 4-5 годин після обіду із скаргами на нудоту, блювоту, холодний піт і невеликий біль в епігастрії. На обід діти їли картопляний суп на м'ясному бульйоні, млинці з м'ясом, компот із сухофрук-тів. Млинці начинялися вареним м'ясом, яке подрібнювалось на м'ясорубці працівником їдальні, хворим панарицієм, і термічній обробці не піддавались. Який найімовірніший діагноз у цьому разі? The children of the boarding school turned to the doctor 4-5 hours after lunch with complaints of nausea, vomiting, cold sweat and a slight pain in the epigastrium. For lunch, the children ate potato soup on meat broth, pancakes with dried fruits. The pancakes were filled with boiled meat, which was ground on a meat grinder by a canteen worker, and were not subjected to heat treatment. What is the most likely diagnosis in this if so?

Ешеріхіоз Escherichia

Стафілококова інтоксикація Staphylococcal intoxication

Афлатоксикоз Aflatoxicosis

Сальмонельоз Salmonellosis

Ботулізм Botulism

1468 / 1500
Доношена дитина від 1-ї вагітності у вікової першороділлі, 1-х стрімких на 37-38 тижні пологів, народилась із масою 3000 г, зростом - 49 см. Під час оцінювання неврологічного статусу визначається зниження м'язового тонусу в правій руці, праве плече повернуте всередину, лікоть розігнутий, спостерігається пронація передпліччя, кисть зігнута - симптом ляльки. Рефлекс Моро не викликається. Рухи в пальцях збережені, хапальний рефлекс позитивний. Який попередній діагноз? A full-term child from the 1st pregnancy in an age-matched primiparous child, 1st fast at 37-38 weeks of childbirth, was born with a weight of 3000 g, a height of 49 cm. During the evaluation of the neurological status, a decrease in muscle tone is determined in the right arm, the elbow is extended, the forearm is bent - the Moro reflex is not elicited. The movements in the fingers are positive. What is the preliminary diagnosis ?

Параліч Клюмпке Klumpke's palsy

Параліч діафрагмального нерва Phrenic nerve palsy

Перелом ключиці clavicle fracture

Пологова травма: парез Дюшена-Ерба справа Birth trauma: Duchenne-Erb paresis on the right

Внутрішньочерепна пологова травма Intracranial birth injury

1469 / 1500
Жінка 25-ти років, яка хворіє на цукровий діабет з 9-річного віку, надійшла у нефрологічне відділення із значними набряками на ногах, обличчі, руках. Артеріальний тиск - 200/110 мм рт.ст.; Нb - 90 г/л, креатинін крові - 850 мкмоль/л, білок сечі - 1,0 г/л, лейкоцити - 10-15 в полі зору. Клубочкова фільтрація - 10 мл/хв. Яка тактика лікаря? A 25-year-old woman, who has been suffering from diabetes since the age of 9, was admitted to the nephrology department with significant edema on her legs, face, and hands. Blood pressure - 200/110 mm Hg; Hb - 90 g/l, blood creatinine - 850 μmol/l, leukocytes - 10-15 in the field of vision. Glomerular filtration - 10 ml/ min. What are the doctor's tactics?

Трансплантація печінки Liver transplant

Переведення у ендокринологічний диспансер Transfer to an endocrinological dispensary

Дієтотерапія Dietotherapy

Переведення у відділення гемодіалізу Transfer to the hemodialysis department

Активна консервативна терапія діабетичної нефропатії Active conservative therapy of diabetic nephropathy

1470 / 1500
У квартирі знайдено труп чоловіка 56 років без ознак насильницької смерті. Під час дослідження трупа встановлено, що трупні плями розташовані на задній поверхні тіла, у разі натискання повністю блідніють та повністю відновлюють своє забарвлення за 55 секунд, трупне заклякання добре виражене у м'язах щелепи, шиї та верхніх кінцівок, ректальна температура складає 27^oC, ознак гниття на трупі не виявлено. Яка давність настання смерті? The corpse of a 56-year-old man was found in the apartment without signs of violent death. During the examination of the corpse, it was found that corpse spots are located on the back surface of the body, when pressed, they turn completely pale and completely recover their color in 55 seconds, the cadaveric convulsion is well expressed in the muscles of the jaw, neck and upper limbs, the rectal temperature is 27°C, no signs of decay are found on the corpse. How long ago did death occur?

Від 18 до 24 годин From 18 to 24 hours

Від 2 до 6 годин From 2 to 6 hours

Від 24 до 36 годин From 24 to 36 hours

Від 12 до 18 годин From 12 to 18 hours

Від 6 до 12 годин From 6 to 12 hours

1471 / 1500
В організм людини з атмосферного повітря надходять декілька хімічних речовин. Як називається тип комбінованої дії, при якому сумісний ефект менший від суми ефектів кожної з речовин, що входить у комбінацію, при їх ізольованій дії на організм? Several chemical substances enter the human body from atmospheric air. What is the type of combined action called, in which the combined effect is less than the sum of the effects of each of the substances included in the combination , with their isolated action on the body?

Ізольована дія Isolated Action

Потенціювання Potentiation

Антагонізм Antagonism

Поєднана дія Combined Action

Комплексна дія Complex Action

1472 / 1500
Пацієнтка на 3-тю добу після штучного аборту госпіталізована до гінекологічного відділення у тяжкому стані з симптомами інтоксикації, болем у животі, гнійними виділеннями з піхви. Об'єктивно: стан жінки тяжкий, температура тіла - 38,8^oC, пульс - 100/хв., АТ- 110/70 мм рт.ст., матка м'яка, дно її на рівні пупка, симптоми подразнення очеревини позитивні. Який найбільш імовірний діагноз? On the 3rd day after the artificial abortion, the patient was hospitalized in the gynecological department in a serious condition with symptoms of intoxication, abdominal pain, purulent discharge from the vagina. Objectively: the woman's condition is severe, body temperature - 38.8°C, pulse - 100/min, blood pressure - 110/70 mm Hg, the uterus is soft, the bottom is at the level of the navel, the symptoms of peritoneal irritation are positive diagnosis?

Перфорація матки Uterine perforation

Гострий метроендометрит Acute metroendometritis

Гострий гнійний сальпінгоофорит Acute purulent salpingo-oophoritis

Пельвіоперитоніт Pelvioperitonitis

Позаматкова вагітність Ectopic pregnancy

1473 / 1500
Місто забезпечується питною водою з підземних артезіанських джерел, яка характеризується високою якістю і чистотою. Який з перерахованих методів обробки води, отриманої з підземних міжпластових напірних джерел, передбачається санітарними вимогами? The city is provided with drinking water from underground artesian springs, which is characterized by high quality and purity. Which of the listed methods of treatment of water obtained from underground interlayer pressure springs is provided by sanitary requirements ?

Знебарвлення Fading

Дефторування Defluorination

Коагуляція Coagulation

Опріснення Desalination

Знезараження Disinfection

1474 / 1500
Пацієнт віком 32 роки під час фізичних вправ зненацька відчув нестачу дихання, слабкість, біль в правій половині грудей з іррадіацією в праве плече, задишку, серцебиття. Об'єктивно спостерігається: стан тяжкий, тахікардія до 100/хв, АТ - 90/60 мм рт. ст. ЧД - 28/хв, права половина грудної клітки відстає при диханні. Перкуторно справа тимпанічний звук, дихальних шумів немає. Температура тіла - нормальна. Який найімовірніший діагноз? A 32-year-old patient suddenly experienced shortness of breath, weakness, pain in the right half of the chest with radiation to the right shoulder, shortness of breath, palpitations during physical exercises. Objectively the condition is severe, BP - 90/60 mmHg, right sided chest sound, no respiratory sounds the most likely diagnosis?

Пневмонія Pneumonia

Інфаркт міокарда Myocardial infarction

Інфаркт легені Pulmonary infarction

Спонтаний пневмоторакс Spontaneous pneumothorax

Судинний колапс Vascular collapse

1475 / 1500
У чоловіка на різних ділянках шкірного покриву, переважно симетрично, розташовані депігментовані плями, що не лущаться, різного розміру й обрисів. Частина волосся на них знебарвлена. Захворювання почалося в дитинстві. Появі плям не передували будь-які елементи висипки. Який імовірний діагноз? The man has depigmented spots that do not peel, of different sizes and outlines, on different areas of the skin, mostly symmetrically. Some of the hair on them is discolored. The disease began in childhood. The appearance of spots was not preceded by any elements of the rash. What is the probable diagnosis?

Вітиліго Vitiligo

Псоріаз Psoriasis

Різнокольоровий лишай Multicolored lichen

Рожевий лишай Pink lichen

Альбінізм Albinism

1476 / 1500
Жінка 52-х років звернулася до лікаря зі скаргами на швидку втомлюваність та задишку протягом останнього року з періодичним сухим кашлем. Пацієнтка відзначає, що останнім часом їй стало важко піднімати обидві руки та розчісувати волосся. При фізикальному обстеженні температура тіла - 37,4^оC, пульс - 76/хв., артеріальний тиск - 130/85 мм рт.ст., SpO_2 - 95% при кімнатному повітрі. При неврологічному дослідженні незначне зниження сили дельтоподібного м'яза. При аускультації легень - розповсюджені сухі хрипи. На щоках, навколо орбіт та на ліктях еритематозний висип. Який метод найбільш імовірно підтвердить діагноз у цієї пацієнтки? A 52-year-old woman came to the doctor with complaints of rapid fatigue and shortness of breath for the past year with a periodic dry cough. The patient notes that recently it has become difficult for her to lift both hands and comb hair.Body temperature - 37.4°C, pulse - 76/min, blood pressure - 130/85 mmHg, SpO_2 - 95% in room air the strength of the deltoid muscle. On auscultation of the lungs - diffused rales. On the cheeks, around the orbits and on the elbows. Which method is most likely to confirm the diagnosis in this patient?

Біопсія шкіри Skin biopsy

Виявлення антинуклеарних антитіл Detection of antinuclear antibodies

Біопсія м'яза Muscle biopsy

Рентгенографія ОГП OHP X-ray

КТ легень CT lung

1477 / 1500
У постраждалого в стані травматичного шоку після значной крововтрати, артеріальний тиск - 70 мм рт. ст., ЧСС - 140/хв. Під час гіповолемічного шоку: The victim is in a state of traumatic shock after significant blood loss, blood pressure - 70 mm Hg, heart rate - 140/min. During hypovolemic shock:

Збільшується центральний венозний тиск Central venous pressure increases

Виникають набряки на нижніх кінцівках Swelling occurs on the lower limbs

Знижується системний судинний опір Systemic vascular resistance decreases

Знижується серцевий індекс Decreasing heart index

Підвищується рН плазми Raising pH of plasma

1478 / 1500
Чоловік 50 років скаржиться на загальну слабкість запаморочення, інтенсивний біль у попереку, правому підребер'ї, періодичну втрату свідомості, які з'явились після порушення технологічного процессу на виробництві фарб. Шкіра землистого відтінку, склери іктеричні, акроціаноз. АТ - 100/60 мм рт. ст. Тони серця приглушені, печінка +3 см, край щільний. У крові виявлено: еритроцити - 2,0·10^12/л, Hb - 90 г/л, тільця Гейнца-Ерліха, лейкоцити - 5,6·10^9/л, ШОЕ - 15 мм/год. Метгемоглобін - 62%, загальний білірубін - 84,0 мкмоль/л, непрямий білірубін - 71,7 мкмоль/л. Який діагноз найімовірніший? A 50-year-old man complains of general weakness, dizziness, intense pain in the lower back, right hypochondrium, periodic loss of consciousness, which appeared after a violation of the technological process at the factory The skin is icteric, blood pressure is 100/60 mmHg, the liver is dense, erythrocytes are 2.0·12/l - 90 g/l, Heinz-Ehrlich bodies, leukocytes - 5.6·10^9/l, ESR - 15 mm/h, methemoglobin - 62%, total bilirubin - 84.0 μmol/l, indirect bilirubin - 71, 7 μmol/L. What is the most likely diagnosis?

Гострий вірусний гепатит Acute viral hepatitis

Гостре отруєння толуолом Acute toluene poisoning

Набута гемолітична анемія Acquired hemolytic anemia

Гостре отруєння аніліном Acute aniline poisoning

Хронічне отруєння ціанідами Chronic cyanide poisoning

1479 / 1500
Чоловік 38 років, вживає алкоголь впродовж 3 років. За 3 дні після запою став відчувати тривогу, страх. Бачив навколо себе павуків, черв'яків, стали переслідувати <<голоси>> осудливого характеру, поводив себе агресивно. У власній особі орієнтований правильно, у місці та часі - дезорієнтований. Який діагноз найімовірніший? A 38-year-old man has been drinking alcohol for 3 years. 3 days after drinking, he began to feel anxiety and fear. He saw spiders and worms around him, began to chase him < > of a judgmental nature, behaved aggressively. In his own person, disoriented in place and time. What is the most likely diagnosis?

Алкогольний параноїд Alcoholic Paranoid

Алкогольний галюциноз Alcoholic hallucinosis

Алкогольний делірій Alcoholic delirium

Алкогольна енцефалопатія Alcoholic encephalopathy

Патологічне сп'яніння Pathological intoxication

1480 / 1500
У лікарні з середньорічною кількістю ліжок 330, лікувалося - 4250 пацієнтів, виписано - 4540 пацієнтів, померло в стаціонарі - 42 пацієнти. Який показник можна розрахувати? In a hospital with an average annual number of beds of 330, 4250 patients were treated, 4540 patients were discharged, 42 patients died in the hospital. What indicator can be calculated?

Загальну смертність Total mortality

Лікарняну летальність Hospital mortality

Середнє число днів роботи ліжка на рік Average number of bed days per year

Подобову летальність Daily mortality

Частоту розбіжності діагнозів Frequency of discrepancy of diagnoses

1481 / 1500
Пацієнт віком 20 років скаржиться на почервоніння очей, сльозотечу, помірні гнійні виділення з кон'юнктивальної порожнини, відчуття стороннього тіла в очах. Об'єктивно спостерігається: гіперемія кон'юнктиви повік. На очних яблуках кон'юнктивальна ін'єкція. Рогівка прозора. Зіниця 3 мм в діаметрі, на світло реагує жваво. Кришталик, склисте тіло прозорі. Очне дно в нормі. Який найімовірніший діагноз? A 20-year-old patient complains of eye redness, lacrimation, moderate purulent discharge from the conjunctival cavity, the sensation of a foreign body in the eyes. Objectively observed: hyperemia of the Conjunctival injection. The pupil is 3 mm in diameter. The lens is clear. What is the most likely diagnosis?

Аденовірусний кератокон'юнктивіт Adenoviral keratoconjunctivitis

Алергічний кон'юнктивіт Allergic conjunctivitis

Гонобленорея Honoblenorrhea

Гострий бактеріальний кон'юнктивіт Acute bacterial conjunctivitis

Гострий іридоцикліт Acute iridocyclitis

1482 / 1500
Жінка 35 років оперована з приводу гострого холециститу. Перебіг раннього післяопераційного періоду був нормальним, виписана в задовільному стані. За 3 тижні після операції звернулася повторно. Скарги на жовтушність склер, підвищення температури тіла до 38,8^oС, озноб, помірний біль у правому підребер'ї. Живіт м'який, перитонеальних симптомів немає. АЛТ - 2,3 ммоль/л. Білірубінемія - 66 ммоль/л через пряму фракцію. Визначте найраціональнішу тактику ведення хворої: A 35-year-old woman was operated on for acute cholecystitis. The course of the early postoperative period was normal, she was discharged in a satisfactory condition. She returned 3 weeks after the operation. Complaints of sclera yellowness , an increase in body temperature up to 38.8°C, moderate pain in the right hypochondrium. ALT - 2.3 mmol/l. Determine the most rational tactics of managing the patient:

Антибактеріальна, протизапальна терапія Antibacterial, anti-inflammatory therapy

3D-комп'ютерна томографія, дистанційна холедохолітотрипсія 3D computer tomography, remote choledocholithotripsy

Ендоскопічна ретроградна холангіографія, літоекстракція Endoscopic retrograde cholangiography, lithoextraction

Лапароскопічна ендоскопічна холедохоскопія Laparoscopic endoscopic choledochoscopy

Лапаротомія, ревізія гепатикохоледоха Laparotomy, revision of hepaticocholedochus

1483 / 1500
Восьмирічний пацієнт спостерігається через затримку зросту. Народився в асфіксії з масою тіла 2800 г. В школі навчається добре. Батьки середнього зросту. Об'єктивно спостерігається: зріст - 107 см, маса тіла - 23 кг, недорозвинення лицьового скелета на кшталт лялькового обличчя. Волосся тонке, шкіра суха з іктеричним відтінком. Підшкірно-жирова клітковина розвинута добре на шиї, грудях, животі. М'язи розвинуті недостатньо. Який найімовірніший діагноз? An eight-year-old patient is observed due to growth retardation. He was born asphyxiated with a body weight of 2800 g. He studies well at school. Parents are of average height. Objectively observed: height - 107 cm, body weight - 23 kg, underdevelopment of the facial skeleton like a doll's face. The skin is dry with a icteric shade. The subcutaneous fat is well developed on the neck, chest, abdomen. What is the most likely diagnosis?

Синдром Фанконі Fanconi Syndrome

Краніофарінгіома Craniopharyngioma

Синдром Дауна Down Syndrome

Гіпофізарний нанізм Pituitary dwarfism

Хондродистрофія Chondrodystrophy

1484 / 1500
Після перенесеного ішемічного інсу-льту, зумовленого кардіоемболією, хворому з фібриляцією передсердь у якості засобу вторинної профілактики призначають: After an ischemic stroke caused by cardioembolism, a patient with atrial fibrillation is prescribed as a means of secondary prevention:

Ноотропні препарати Nootropic drugs

Оральні антикоагулянти Oral anticoagulants

Антагоністи кальцію Calcium antagonists

beta-адреноблокатори beta blockers

Аспірин чи клопідогрель Aspirin or clopidogrel

1485 / 1500
Пацієнтка 42 років звернулася до гінеколога зі скаргами на періодичні пекучі болі в середній частині молочної залози зліва. Больові відчуття не залежать від фази менструального циклу. Під час огляду, пальпації патології не виявлено. З якого інструментального методу необхідно почати обстеження? A 42-year-old patient consulted a gynecologist with complaints of periodic burning pains in the middle part of the left mammary gland. The pain does not depend on the phase of the menstrual cycle. During the examination, palpation No pathology was detected. What instrumental method should be used to begin the examination?

КТ молочних залоз CT breast

УЗД молочних залоз Ultrasound of mammary glands

Біопсія молочної залози Breast biopsy

МРТ молочних залоз MRI breast

Дуктографія Ductography

1486 / 1500
Головний лікар ЦПМСД має надати звіт щодо забезпеченості населення лікарями. За допомогою якого коефіціє-нту він може розрахувати даний показник? The chief physician of the CPMSD must provide a report on the provision of doctors to the population. By which coefficient can he calculate this indicator?

Наочності Visibility

Спеціальної інтенсивності Special intensity

Екстенсивності Extensives

Інтенсивності Intensities

Співвідношення Ratio

1487 / 1500
Хворий 60-ти років скаржиться на інтенсивні болі стискаючого характеру, локалізовані за грудниною з іррадіацією у ліву лопатку та нижню щелепу. Біль тривалий, не знімається нітрогліцерином. Об'єктивно: холодний піт. Хворий збуджений. Аускультативно: тахікардія, приглушеність тонів. Артеріальний тиск - 100/70 мм рт.ст. ЕКГ-комплекси QS та підйом S-T вище ізолінії в І, аVL, V1, V2, V3, V4. Який діагноз є найбільш імовірним? A 60-year-old patient complains of intense squeezing pains, localized behind the sternum with radiation to the left scapula and lower jaw. The pain is long-lasting, not relieved by nitroglycerin. Ob' Objectively: the patient is agitated. Tachycardia, muffled tones. ECG-complexes above I, aVL, V3. What is the diagnosis is the most likely?

Інфаркт міокарда Myocardial infarction

Лівосторонній плеврит Left-sided pleurisy

Тромбоемболія легеневої артерії Thromboembolism of the pulmonary artery

Стенокардія Принцметала Prinzmetal Angina

Міжреберна невралгія Intercostal neuralgia

1488 / 1500
Жінка звернулась зі скаргами на мажучі кров'янисті виділення, біль в попереку. Остання менструація 2,5 місяці тому. При об'єктивному обстеженні: шийка матки ціа-нотична, бочкоподібна, розширена, зовнішнє вічко розташоване ексцентрично, матка збільшена як до 5 тижнів вагітності (пісочний годинник), придатки з обох боків не пальпуються, виділення з цервікального каналу кров'я-нисті, незначні. Яку патологію можна припустити? A woman complained of spotting bleeding, lower back pain. The last menstruation was 2.5 months ago. On objective examination: the cervix was notic, barrel-shaped, expanded, the external eye is located eccentrically, the uterus is enlarged as before 5 weeks of pregnancy (hourglass), the appendages on both sides are not palpable, the discharge from the cervical canal is bloody, insignificant. What pathology can be assumed?

Шийкова вагітність Cervical pregnancy

Поліп цервікального каналу Cervical canal polyp

Субмукозний міоматозний вузол, що народжується Submucosal myomatous nodule at birth

Рак шийки матки Cervical cancer

Міома шийки матки Cervical myoma

1489 / 1500
Восьмирічна дівчинка скаржиться на біль у навколопупковій ділянці, що з'являється вранці натщесерце. Діагностовано гастродуоденіт, асоційований з Н. pylori. Які препарати входять до складу ерадікаційної терапії H. pylori? An eight-year-old girl complains of pain in the periumbilical area, which appears in the morning on an empty stomach. Gastroduodenitis associated with H. pylori is diagnosed. What drugs are included in the eradication therapy of H pylori?

Тримебутин, рокситроміцин, ранітидин Trimebutin, roxithromycin, ranitidine

Алюмінію фосфат, фамотидин, метронідазол Aluminum phosphate, famotidine, metronidazole

Колоїдний субцитрат вісмуту, прифінія бромід, азитроміцин Colloidal bismuth subcitrate, prifinia bromide, azithromycin

Мебеверин, фуразолідон, омепразол Mebeverin, furazolidone, omeprazole

Пантопразол, кларитроміцин, метронідазол Pantoprazole, Clarithromycin, Metronidazole

1490 / 1500
Пацієнт віком 29 років хворіє гормонозалежною бронхіальною астмою. Під час флюорографії в С2 правої легені виявлено округлу тінь з чіткими рівними контурами, середньої інтенсивності. Навколо неї кілька поліморфних вогнищевих тіней, у корені - кальцинат. Під час обстеження спостерігається: перкуторний звук над легенями з коробковим відтінком, вислуховуються розсіяні сухі хрипи. Аналіз крові без змін. Реакція на пробу Манту з 2 ТО ППД-Л - папула 22 мм. Який найімовірніший діагноз? A 29-year-old patient suffers from hormone-dependent bronchial asthma. During fluorography, a rounded shadow with clear even contours of medium intensity was detected in C2 of the right lung. Several polymorphic focal shadows were found around it , in the root - calcification. During the examination, there is a percussion sound over the lungs, scattered dry rales are heard. The reaction to the Mantoux test is 22 mm. What is the most likely diagnosis?

Аспергільома Aspergilloma

Пневмонія Pneumonia

Еозинофільний інфільтрат Eosinophilic infiltrate

Туберкульома Tuberculoma

Периферичний рак Peripheral cancer

1491 / 1500
Чоловік 54 років поступив до стаціонару зі скаргами на загальну слабкість, підвищення температури тіла до 38,6^oС, задишку під час значного фізичного навантаження, часті носові кровотечі. Працює на виробництві пластмаси, де контактує з ароматичними з'єднаннями. Об'єктивно встановлено: шкірні покриви бліді, сухі. У крові виявлено: еритроцити - 2,1·10^12/л, Hb - 90 г/л, лейкоцити - 2,2·10^9/л, еозинофіли - 1%, паличкоядерні - 1%, сегментоядерні - 75%, лімфоцити - 20%, мієлоцити - 3%, тромбоцити - 30·10^9/л, ШОЕ - 32 мм/год. Який найімовірніший діагноз? A 54-year-old man was admitted to the hospital with complaints of general weakness, an increase in body temperature to 38.6^oC, shortness of breath during significant physical exertion, frequent nosebleeds. He works in the production of plastics, where he comes in contact with aromatic compounds. ,2·10^9/l, eosinophils - 1%, rod-nuclear cells - 1%, segmentonuclear cells - 75%, lymphocytes - 20%, myelocytes - 3%, platelets - 30·10^9/l, ESR - 32 mm/h . What is the most likely diagnosis?

Хронічна інтоксикація свинцем Chronic lead intoxication

Хронічна інтоксикація тетраетилсвинцем Chronic tetraethyl lead intoxication

Хронічна інтоксикація бензолом Chronic benzene intoxication

Хронічна інтоксикація аніліном Chronic aniline intoxication

Хронічна інтоксикація нітробензолом Chronic nitrobenzene intoxication

1492 / 1500
Хлопчик 7 років перебуває на лікуванні протягом місяця. Під час госпіталізації спостерігалися виражені набряки, білок у добовій сечі - 4,2 г. У біохімічному аналізі крові утримується гіпопротеїнемія (43,2 г/л), гіперхолестеринемія (9,2 ммоль/л). Установіть провідний синдром гломерулонефриту, який, найвірогідніше, має місце у пацієнта? A 7-year-old boy has been receiving treatment for a month. During hospitalization, pronounced edema was observed, protein in daily urine - 4.2 g. In the biochemical analysis of blood, hypoproteinemia is observed ( 43.2 g/l), hypercholesterolemia (9.2 mmol/l). Establish the leading syndrome of glomerulonephritis, which is most likely to occur in the patient?

Ізольований сечовий Isolated urinary

Гематуричний Hematuric

Змішаний Mixed

Нефротичний Nephrotic

Нефритичний Nephritic

1493 / 1500
Вагітна 22-х років звернулась до жіночої консультації зі скаргами на кровотечу зі статевих шляхів, що розпочалася 2 дні тому, сильну втому та запаморочення. Термін вагітності 13 тижнів. За день вона використала 6 гігієнічних прокладок. При фізикальному обстеженні: артеріальний тиск - 90/60 мм рт.ст., температура - 37,8^oC, пульс - 125/хв., частота дихання - 15/хв. Розпочата інфузійна терапія. При вагінальному дослідженні: кров у піхві, шийка матки згладжена та розкрита. На УЗД плідне яйце в порожнині матки, серцебиття і рухи плода не визначаються. Який з наступних кроків у веденні пацієнтки буде найбільш доречним? A 22-year-old pregnant woman came to the women's consultation with complaints of bleeding from the genital tract, which began 2 days ago, severe fatigue and dizziness. The pregnancy period is 13 weeks. During the day, she used 6 sanitary napkins.During physical examination: blood pressure - 90/60 mm Hg, temperature - 37.8°C, pulse - 15/min. Infusion therapy was started. During vaginal examination: blood in the vagina, the cervix is ​​smoothed and opened. On ultrasound, the fetal egg in the uterine cavity, the heartbeat and the movements of the fetus are not determined. Which of the following steps in the management of the patient will be most appropriate?

Магнію сульфат внутрішньовенно Magnesium sulfate intravenously

Антибіотикотерапія Antibiotic therapy

Постільний режим та анальгетики Bed rest and analgesics

Метотрексат Methotrexate

Дилатація та кюретаж матки Dilatation and curettage of the uterus

1494 / 1500
Пацієнт скаржиться на біль, світлобоязнь, сльозотечу, зниження зору в правому оці. Гострота зору правого ока 0,5 не корегується, гострота зору лівого ока 1,0. Об'єктивно спостерігається: в правому оці очна щілина звужена, перикорнеальна ін'єкція очного яблука. Преципітати на задній поверхні рогівки. Зіниця звужена, реакція на світло уповільнена. В передньому відділі склистого тіла плаваючі помутніння. Очне дно в нормі. Який найімовірніший діагноз? The patient complains of pain, photophobia, lacrimation, reduced vision in the right eye. The visual acuity of the right eye is 0.5 uncorrected, the visual acuity of the left eye is 1.0. Objectively, the eye slit is narrowed, the eyeball is injected, the pupil is narrowed, the reaction to light is slowed down. What is the most likely diagnosis?

Гострий кератит Acute keratitis

Гострий напад глаукоми Acute attack of glaucoma

Задній увеїт Posterior uveitis

Бактеріальний кон'юнктивіт Bacterial conjunctivitis

Гострий іридоцикліт Acute iridocyclitis

1495 / 1500
У виробничих умовах реєструються високі рівні шуму та забруднення повітря робочої зони ангідридом сірки. Який вид шкідливої дії на організм можуть справляти ці чинники в таких умовах? In production conditions, high levels of noise and air pollution of the working area with sulfur anhydride are registered. What kind of harmful effects can these factors have on the body in such conditions?

Комплексну Comprehensive

Специфічну Specific

Комбіновану

Роздільну Separate

Поєднану Combined

1496 / 1500
До лікаря звернулася жінка 74 років зі скаргами на біль у пахвинній ділянці справа. Ці симптоми виникли раптово близько 2 годин тому. З анамнезу відомо: пацієнтка каже, що ці симптоми вже були у неї близько 3 тижнів тому, але самі зникли, після того як хвора лягла. Об'єктивно спостерігається: нижче пупартової зв'язки пальпується щільне, напружене утворення до 3,5 см у діаметрі, різко болюче, симптом <<кашльового поштовху>> негативний. Який діагноз найімовірніший? A 74-year-old woman came to the doctor with complaints of pain in the inguinal region on the right. These symptoms appeared suddenly about 2 hours ago. It is known from the anamnesis: the patient says that these she already had symptoms about 3 weeks ago, but they disappeared after the patient lay down. Objectively observed: a dense, tense mass up to 3.5 cm in diameter, sharply painful, is palpable below the Pupart ligament. shock>> negative. What is the most likely diagnosis?

Набута невправима стегнова грижа Acquired inoperable femoral hernia

Набута невправима пахова грижа Acquired inoperable inguinal hernia

Набута защемлена стегнова грижа Acquired femoral hernia

Паховий лімфаденіт Inguinal lymphadenitis

Набута защемлена пахова грижа Acquired inguinal hernia

1497 / 1500
До опікового відділення доставлено потерпілого чоловіка 28 років зі циркулярними опіками тулуба та нижніх кінцівок. Визначте розмір опікової поверхні у потерпілого: A 28-year-old male victim with circular burns of the trunk and lower limbs was brought to the burn department. Determine the size of the victim's burn surface:

45% 45%

30% 30%

36% 36%

72% 72%

20% 20%

1498 / 1500
Пацієнтка скаржиться на частi, ряснi, без патологiчних домiшок, пiнистi випорожнення, переймоподiбний бiль у навколопупковiй дiлянцi, бурчання в животi, лихоманку. З анамнезу відомо про вживання в їжу качиних яєць, зварених некруто. Який збудник захворювання найiмовiрніший? The patient complains of frequent, abundant, without pathological impurities, foamy stools, cramp-like pain in the peri-umbilical region, stomach rumbling, fever. From the anamnesis, it is known about eating hard-boiled duck eggs. What is the most likely causative agent of the disease?

Escherichia coli Escherichia coli

Salmonella enteritidis Salmonella enteritidis

V. cholerae V. cholerae

Shigella sonnei Shigella sonnei

Salmonella typhi Salmonella typhi

1499 / 1500
Чоловік 62 років скаржиться на слабкість, жар у язиці, заніміння нижніх кінцівок. Об'єктивно встановлено: шкіра бліда з субіктеричним відтінком, язик яскраво-червоний, смакові сосочки згладжені. Печінка - +3 см. Під час гастроскопії виявлена атрофія слизової оболонки. У крові виявлено: еритроцити - 1,2·10^12/л, Нb - 56 г/л, КП - 1,4, макроцитоз, лейкоцити - 2,8·10^9/л, еозинофіли - 2%, паличкоядерні - 4%, сегментоядерні - 50%, лімфоцити - 42%, моноцити - 2%, ретикулоцити - 0,1%, тромбоцити - 120·10^9/л, ШОЕ - 36 мм/год, білірубін - 29 ммоль/л, непрямий - 26 ммоль/л. Який найбільш імовірний діагноз? A 62-year-old man complains of weakness, heat in the tongue, numbness of the lower extremities. Objectively established: the skin is pale with a subicteric shade, the tongue is bright red, taste buds Liver - +3 cm. Atrophy of the mucous membrane was detected in the blood: erythrocytes - 1.2·10^12/l, KP - 1.4, macrocytosis - 2. ,8·10^9/l, eosinophils - 2%, rod-nuclear cells - 4%, segmentonuclear cells - 50%, lymphocytes - 42%, monocytes - 2%, reticulocytes - 0.1%, platelets - 120·10^9/l , ESR - 36 mm/h, bilirubin - 29 mmol/l, indirect - 26 mmol/l. What is the most likely diagnosis?

Фолієво-дефіцитна анемія Folate deficiency anemia

Гемолітична анемія Hemolytic anemia

Залізодефіцитна анемія Iron deficiency anemia

В_12-дефіцитна анемія B_12-deficiency anemia

Гіпопластична анемія Hypoplastic anemia

1500 / 1500
У хворої 33-х років після нападу епілептичних судом виникли задуха, сухий кашель та підвищення температури тіла. При аускультації легень справа знизу дихання відсутнє, перкуторно - притуплення звуку. Огляд ротової порожнини виявив відсутність одного різця нижньої щелепи. Який діагностичний метод найбільш імовірно дозволить лікарю визначити причину дихальних розладів пацієнтки? A 33-year-old patient developed shortness of breath, a dry cough, and an increase in body temperature after an attack of epileptic convulsions. When auscultating the lungs from the lower right, there is no breathing, percussive sound dulling. Examination of the oral cavity revealed the absence of one incisor of the lower jaw. Which diagnostic method is most likely to allow the doctor to determine the cause of the patient's respiratory disorders?

- -

Пряма ларингоскопія Direct laryngoscopy

Фіброезофагогастроскопія Fibroesophagogastroscopy

Непряма ларингоскопія Indirect laryngoscopy

Бронхоскопія Bronchoscopy